Você está na página 1de 1109

Geometría

TEORÍA Y PRACTICA
9pLu*¿ ¿ a á F . c tá . ° t(

O C uW e
V/U-A_

q I x
SAN MARCOS
AÑO 2003

Hecho el depósito legal, Ley N° 26905.

REG. N° 15013298-1075

Prohibida la reproducción parcial o total de la obra,


sin la previa autorización escrita del Editor de la misma.

Aníbal Jesús Paredes Galván - Editor


Jr. Natalio Sánchez 220 - Ofic. 304 - Jesús María

Impreso en Perú____________________ Printed in Perú

Composición, diagramación y montaje:


Editorial San Marcos
RUC 10090984344
PROLOGO DEL AUTOR
El hecho de contar con un libro que llegue al lector que necesiíaaprender; al lector que quiere
profundizar sus conocimientos y al que se interesa p o r la investigación, fu e lo que m e impulsó a
escribir el presente. Y esta tarea, concluida hoy, me permite poner al alcance de alumnos y colegas
un trabajo que, en mucho, cumple el objetivo propuesto.

En prim er lugar, la teoría se desarrolla de manera sencilla, sin descuidar la rigurosidad


en la secuencia lógica de Postulados y Teoremas. En este sentido hago prim ar lo básico sobre lo
secundario, para que el alumno se dé cuenta p o r sí solo de lo mínimo que necesita saber para
enfrentar con éxi lo un problema, desterrando así la idea de que la Geometría consiste en memorizar
más y más propiedades.

Luego, los problemas resueltos se presentan distribuidos de m enor a m ayor grado de


dificultad; hecho que el lector mismo notará. Para ello, en los capítulos iniciales , generalmente,
se han propuesto hasta tres bloques bien diferenciados. Además, se demuestran muchas
propiedades, en form a de problemas.

A parte del curso completo de Geometría elemental (Plana y del Espacio), se desarrollan
otros temas de suma importancia, como los de M áximos y Mínimos, Introducción a la Geometría
Analítica, Vectores, Isometrías, construcciones con regla y compás. Asimismo se dedica uncapítulo
a temas selectos, en el cual se muestran las demostraciones de propiedades importantes.

De otro lado, al fin a l de algunos capítulos el lector encontrará algunos comentarios


relacionados con historias, anécdotas y aportes importantes de personajes ilustres.

Al concluir, hallará un conjunto de problemas propuestos y ordenados p o r capítulos, con


clave de respuestas. Seguro que el presente libro contribuirá bastante en el aprendizaje y
preparación de los alumnos, a sí como, instrumento de desarrollo en clase, de los colegas, me
despido a la espera de recibir sugerencias para mejorar futuras ediciones, con un gran
agradecimiento al Sr. Aníbal Paredes Galvcin, p o r la confianza depositada en mi persona;
asimismo, a todos aquellos cuyo aporte en el tipeo de textos y gráficos ha sido de suma importancia.

Fernando M. A Iva Gallegos


INTRODUCCION

El denominador común del origen de la Geometría, en cada una de las antiguas


civilizaciones fue su afán de medir las tierras, su inclinación por las edificaciones descomunales
y su gran debilidad por la Astrología; con la cual, al tratar de predecir acontecimientos y
situaciones a través de los astros, condujo inevitablemente hacia el desarrollo vertiginoso de la
Astronomía, con su repercusión fructífera para su progreso geométrico.

Los griegos erigieron, sobre todo con Euclides, el edificio geométrico racional, reemplazan­
do la observación y la experiencia p o r las deducciones racionales, partiendo de definiciones,
axiomas y postulados, por un proceso deductivo.

Para tener una referencia del orden en quefueron apareciendo los sabios que enriquecieron
el conocimiento geométrico, en la era de la cultura Griega, tenemos :

T halesdeM ileto(640A .C .): Fundósu uEscuelade


Matemática y Filosofía ” llamada escuela Jónica.
Su más importante contribución, es el teorema que
lleva su nom bre:
AB DE
BC “ EF

Pitágoras (569-500 A .C .): Fue el discípulo más ilustre de la Escuela Jónica, formando
luego la famosa Escuela Pitagórica, cuyo lema era : uLos números rigen el Mundo”. Su
contribución más importante, es el teorema call­
eado a los triángulos rectángulos :

a2 + b2 = c 2

Aristóteles (384-322 A.C.), tuvo una casi directa contribución en el progreso de la


Geometría, orientando a los investigadores de la ciencia matemática y facilitando el
descubrimiento de los errores científicos.

Euclides (330-275 A.C), de quien no se sabe mucho sobre su biografía. La mayor parte de
su vida la pasó en Alejandría y enseñó en el Museum que ahí se fundó. Es realmente con él,
que la Geometría alcanza la jerarquía de una verdadera ciencia, reuniendo todos los
conocimientos conocidos en su obra inm ortal: u Los Elementos "
En esta época de oro de la Escuela de Alejandría, aparecieron además Arquímedes y
Apolonio.

5
La obra que más repercusión ha tenido en el pensamiento científico de todas los tiempos, es
a no dudarlo, "Los Elementos", Porque lo que él hizo no fue precisamente reunir todo el
conocimiento geométrico de su época o resumirlo, sino más bien, seleccionar de todo ese mar
de cuestiones geométricas sólo aquéllas que, de acuerdo a un plan cuidadosamente pre -
estructurado, formaron un verdadero sistema, justamente con las nuevos aportes que él incorporó
a esta ciencia.

Gran parte de su contenido proviene sobre todo de los pitagóricos y de Eudoxio; pero la
abstracción, ese afán de hacer primar el conocimiento puro sobre lo utilitario, por un lado y el
riguroso método deductivo por otro, fueron influencia de Platón y de Aristóteles, respectivamen­
te, doble influencia que, en la mente diáfana de Euclides, hace de conocimientos antes dispersos
un perfecto sistema racional.

En "Los Elementos" cuya aparición pertenece al siglo III A. C., Euclides sistematizó todas
las propiedades geométricas hasta entonces conocidas en forma tan completa y lógica, que los
tratados de Geometría hasta muchos siglos posteriores, estuvieron todos basados en é l , con
pequeñas variantes de orden, enunciado y notación; sólo las investigaciones modernas las han
completado decisivamente.

La geometría que se estudia en la secundaria es ¿a geometría euclidiana, y el concienzudo


estudio de ella nos dará una visión clara de su desarrollo lógico , de su importancia como modelo
del desenvolvimiento deductivo, y como base fundamental para el estudio de otras geometrías;

TRASCENDENCIA DE LOS ELEM ENTOS: Escrito sólo con el fin de dar un conoci­
miento completo y sintético de la geometría, como base para los que quisieran estudiar la
Filosofía, ha tenido tanta importancia en el desarrollo cultural de la humanidad, que de él se han
hecho más de 1500 ediciones, siendo después de la Biblia la obra que mayor difusión ha
alcanzado .

Entre otros destacados sabios de la época de entonces, podemos mencionar a denón,


Hipócrates de Chíos, Arquitas, Hiplas de Elis, etc .

El Autor

6
índice

Prólogo del Autor 7


* Introducción................................................................................................................................8

* Capítulo 1
- Términos Matemáticos..........................................................................................................11

* Capítulo 2

- Intersección de Figuras Piernas............................................................................................. 19


- Ejercicios y Problemas Resueltos........................................................................................ 24

Capítulo 3
&
- Segmentos...............................................................................................................................43
- Problemas Resueltos - Nivel 1..................................................................................... 47
-N iv e l I I ..................................................................................... 55
- Nivel m .................................................................................... 69

* Capítulo 4 ^

- Angulos.................................................................................................................................... 85
- Problemas Resueltos............................................................................................................. 91

* Capítulo 5
*

- Triángulos.............................................................................................................................. 127
- Problemas Resueltos —Nivel 1.................................................................................... 130
-N iv e l I I ................................................................................... 142
- NivelI I I ................................................................................ 153
- 'W

Capítulo 6
Congruencia de Triángulos................................................................................................. 167
Problemas Resueltos — Nivel I ................................................................. .................172
- Nivel II...,............................................................................... 182
-N iv e l I I I ........................................................................... 195

Capítulo 7
Wmrn W & . m i.IK.

- Polígonos............................................... 221
- Problemas Resueltos...........................................................................................................224

♦ Capítulo 8

- Cuadriláteros........................................................... 245
- Problemas Resueltos........................................................................................................... 247
' ' • • . ✓ ‘v i f l W

* ^ Capítulo 9 - |
- Circunferencia........................................................................................................................271
— P roblemas Resueltos........................................................................................................... 280

♦ i Capítulo 10

- Puntos Notables del Triángulo............................................................................................325


- Problemas Resueltos........................................................................................................... 329

* Capítulo 11

- Líneas Proporcionales........................................................................................................ 347


- Problemas Resueltos...........................................................................................................350

* ^Capítulo 12

- Semejanza de Triángulos................................................................................................... 365


- Problemas Resueltos........................................................................................................... 371

* Capítulo 13

- Relaciones Métricas en Triángulos Rectángulos........................................................... 395


- Problemas Resueltos........................................................................................................... 398

* Capítulo 14

- Relaciones Métricas en Triángulos Oblicuángulos..........................................................413


- Problemas Resueltos........................................................................................................... 420

* Capítulo 15

- Relaciones Métricas en la Circunferencia y Potencia.................................................... 441


- Problemas Resueltos........................................................................................................... 450

* Capítulo 16^

- Polígonos Regulares y Longitud de la Circunferencia.......................... 471


- Problemas Resueltos........................................................................................................... 477

* Capítulo 17

- Areas de las Regiones Planas............................................................................................503

* Capítulo 18

- Rectas y Planos 659

*|i Capítulo 1 9 |
- Rectas y Planos, Perpendiculares..................................................................................... 665

8
I

^Capítulo 2 0 j
Rectas y Planos Paralelos..................................................................................................675
Problemas Resueltos ( Capítulo 18, 19 y 20 ) ............................................................ 684
Problemas Propuestos........................................................................................................ 699

Capítulo 21
Angulos Diedros....................................................................................................................701

Capítulo 22
Proyecciones en el Espacio............................................................................................... 705

Capitulo 23

Simetría.................................................................................................................................. 719

Capítulo 24ie
Angulos Poliedros.................................. 723
Problemas Resueltos ( Capítulos 21, 22, 23, 24 ) ........................................................ 732

Capítulo 25
Poliedros................................................................................................................................753
Problemas Resueltos...........................................................................................................755
Problemas Propuestos........................................................................................................ 767

Capítulo 26

Prisma y Tronco de Prisma.................................................................................................771


Problemas............................................................................................................................. 776

Capítulo 27

Pirámide y Tronco de Pirámide..........................................................................................795


Problemas Resueltos...........................................................................................................798

Capítulo 28

Cilindros y Troncos de Cilindros........................................................................................815


Problemas..............................................................................................................................819

Capítulo 29
Cono y Tronco de C ono...................................................................................................... 831
Problemas Resueltos...........................................................................................................834

Capítulo 30
Esfera y Teoremas de Pappus-Guldim 845

9
Problemas 854

Capítulo 31
Isometrías..............................................................................................................................873

Capítulo 32

Introducción a la Geometría Analítica.


Problemas..............................................................................................................................902

Capítulo 33
Vectores................................................................................................................................. 911
Problemas Resueltos...........................................................................................................916

Capítulo 34

Máximos y Mínimos en Geom etría................................................................................... 959


Problemas Resueltos...........................................................................................................982

Capítulo 35
Temas Selectos: Demostración de teoremas y Propiedades en Poliedros............... 997-

Capítulo 36

I. Trazado de Paralelas y/o Perpendiculares con Escuadras............................... 1009


II. Construcciones Geométricas, con reglas y compás............................................1010
III. Los Tres Problemas Famosos de Construcción.................................................. 1023

Capítulo 37

Transformación de coordenadas.................................................................................. 1024-A


Problemas Propuestos.................................................................................................... 1024-K

Miscelánea de Problemas Propuestos...................................... „............................... 1025


Bibliografía........................................................................................................................ 1091
CAPITULO 1

TERMINOS MATEMATICOS

a) Proposición.- Enuncia una verdad demostrada o por demostrar.

b) Axioma.- Es una proposición evidente que no necesita demostrarse. Por ejemplo: “ El todo,
es igual a la suma de sus partes

c) Postulado.- Es la proposición, que sin tener la evidencia del Axioma, se admite sin
demostración, Por ejemplo “ Por dos puntos distintos pasa una, y sólo una recta"

d) Teorema.- Es una proposición que, para su aceptación, necesita demostrarse . Consta de:
Hipótesis y Tesis. La primera, indica los datos y se supone como cierta; la segunda indica
lo que se va a demostrar. Luego, viene el proceso de la demostración.

Ejemplo: La suma de las medidas de los ángulos de un triángulo, es 180.

e) Lema.- Es un teorema previo, que sin/e de base para la demostración de otras proposiciones.

f) Corolario.- Es una consecuencia deducida de un teorema demostrado.

g) Escolio.- Llamada de atención hecha a un teorema con objeto de aclaración o restricción.

h) Problema -Enunciado en el cual se plantea hallar una cantidad o construir alguna figura
según las condiciones dadas.

Objetivo y División.- La Geometría tiene por objeto el estudio de las figuras geométricas,
atendiendo a su forma, tamaño y relación entre ellas.

Para un mejor tratamiento se divide en : Geometría Plana y Geometría del Espacio.

(a) Geometría Plana ( Planimetría). - Estudia las figuras planas, por ejemplo: el triángulo, círculo,
etc.

(b) Geometría del Espacio (Estereométria).- Estudia a las figuras cuyos puntos no están en un
mismo plano; están en el espacio. Por ejemplo : La Pirámide, el Prisma, la Esfera, etc.

11
Figuras Geométricas. Clasificación.- Se llama figura geométrica a la representación de
líneas, superficies y sólidos, adoptando cierta forma y teniendo una determinada extensión.
A excepción del punto, el cual representa al conjunto unitario, toda figura se distingue de otra
por su tamaño y forma. Un punto queda perfectamente determinado por su posición en el
espacio.

Las figuras geométricas se distinguen en: Líneas, superficies y sólidos.

L IN E A S :

L. Recta - Todos sus puntos siguen una


misma dirección.

L. Quebrada.- Formada por un conjunto de dos o


más líneas rectas consecutivas, en
diferente dirección.

L. Curva.- Si no tiene tres puntos que sigan la


misma dirección.
4

L. Mixta Es la combinación de alguna línea recta


y alguna curva, consecutivas.

SUPERFICIES :

S. Plana o plano S. Curva

SOLIDOS :

Mediciones

La medida de una línea limitada, es un número positivo único, llamada longitud. ( Son
unidades de longitud : m; c m ; . . . e tc .).

El área, es un número positivo único que indica la medida de una superficie. ( Son unidades
de área : m2 , cm2 , . . . e tc .).

La medida del espacio que encierra un sólido, se expresa por un número llamado volumen.
( Unidades de volumen : m3 , cm3 ; e tc .).
CLASIFICACION.* Dos figuras, de la misma naturaleza, pueden s e r :

(a) Congruentes.- Si tienen igual forma y tamaño. Por ejemplo, dos cuadrados con igual longi­
tud de lado.

(b) Semejantes.- Cuando tienen igual forma y tamaños diferentes. Por ejemplo, un cuadrado
cuyo lado mide 10 unidades y otro cuyo lado mide 7 unidades.

(c) Equivalentes.-AI tener igual área o volumen, sin importar su forma. Dos superficies
equivalentes tienen igual área y dos sólidos equivalentes, igual volumen.

Congruentes (Igual área)


(Igual volumen)
Equivalentes

= , se lee : “es congruente, con ..

, se lee : “es semejante, con...”

<> , se lee : “ es equivalente a . . . “

Conceptos Primarios y Nomenclatura.- El punto, la recta y el plano, son entes geo­


métricos no definidos, sobre los cuales se apoyan las definiciones de otras representaciones

A
punto A r : recta r

Plano P

El punto es la mínima representación en Geometría. Una recta está conformada por un


conjunto infinito de puntos que siguen una misma dirección e ilimitada en ambos sentidos.
Se puede concebir al plano, como una superficie llana, perfectamente lisa, sin espesor e
ilimitada en todo sentido.

Rayo y Semirrecta.- La siguiente figura, muestra un rayo El punto o , se llama origen o


extremo y forma parte de la figura. Se denota, como : O P .

O P

A diferencia d«l rayo una semirrecta no considera el origen. Así, para la siguiente figura; la
semirrecta OP se denotará como : C P . o------------------------------------- ►
O P

13
Ejemplo 7.- El perímetro de un triángulo equilátero, es 18 El lado del cuadrado,
equivalente a dicho triángulo, tiene longitud :

A) 6 ^ 3 cm B) 6 cm C)4 / 3 cm D) 3 cm E) 3 / 3 cm

Solución
J L
El perímetro del triángulo es la suma de
longitudes de los tres lados. Entonces : < >
L+L + L=18^3
"1 r
/. L = 6 f3
-------- x ---------1

Como deben ser equivalentes, según enunciado :

área del □ = área del A

4
fórmula de área de un fórmula de área de un
cuadrado. — i t A equilátero.

Entonces: x2 = ( 6 ^ 3 ) . 2 ^ .

Es decir: x2 = 27 -> x = 3yf~3 Rpta : (E)

Ejemplo 2,~ Hallar la longitud x, del radio de la esfera equivalente al cono de revolución
adjunto,cuyo radio mide r = 6 ^ 2 cm y altura h = 12 cm.

A) 4 cm B) 6 cm C) 8 cm
D) 9 cm E) 12 cm

Solución

Por ser equivalentes : Volumen de la Esfera Volumen del cono


ji
4 3 7i r2 h
Según fórmula: — Tí X
3

Es decir: 4 x 3 = ( 6 ^ 2 )2 12 x= 6 Rpta:(B)

14
Ejemplo 3,-lndicar verdadero ( V ) o falso ( F ):

I.- Un cuadrado, puede ser congruente a un triángulo.


II.- Dos figuras congruentes, son siempre equivalentes.
Dos figuras equivalentes, son siempre congruentes.
IV.- Un cubo y un cuadrado, pueden ser equivalentes.
V.- Si un cuadrado y un triángulo, tienen igual perímetro, se llaman equivalentes
VI.- Dos rectángulos, son siempre semejantes.

Solución
I. (F). Las figuras congruentes deben tener igual forma y tamaño.
(V). Por definición.
(F). No siempre. Dos figuras equivalentes sólo requieren tener tamaños iguales, mas
no la forma. Por ejemplo, un cuadrado y un triángulo, pueden ser equivalentes.

IV. (F). La comparación debe ser entre figuras de la misma natualeza: figuras planas entre
sí o sólidos entre sí.

V. (F) Serán equivalentes, si sus áreas son iguales.

VI. (F) Por ejem plo:

no tienen la misma form a; sólo sus ángulos sor/congruen­


tes. ( Angulos congruentes, tienen igual medida.)

Ejemplo 4.-Con una cuerda de longitud L cm, ¿Cuál de las dos figuras dadas a continuación debe
formarse, para tener mayor área ?:

a) Un cuadrado b) Una circunferencia

Solución

Si se forma un cuadrado, cada lado deberá tener longitud : —

V, el á re a :

Si se forma una circunferencia de radio r : 2rcr = L

r=
2n

Y, el área del círculo: nt2 = — ... ( I I ).


4rc

Como te = 3,1416 _» 4 tc = 12,5664. Entonces,la expresión ( I I ) es mayor que ia


( I ). Se debe elegir la circunferencia.

15
Conjuntos convexos y no convexos

Cualquier figura geométrica, es un conjunto de puntos. Por ejemplo, un triángulo, es el conjunto


de puntos correspondiente a la reunión de los tres segmentos que determinan tres puntos no
colineales.

B
Así: AABC = AB U BC U AC

Nótese, en el gráfico adjunto, que el punto P, pertenece al triángulo. Q, no es un punto del triángulo.
Q está en la región interior al triángulo. R es punto de la región exterior al triángulo.

Definición.* Una región triangular, es el conjunto de puntos que comprende a un triángulo y su


interior.

Definición.- Un polígono, es la reunión de tres o más segmentos consecutivos trazados en


diferentes direcciones, tales como las figuras adjuntas. /

Análogamente al caso del triángulo, una región poligonal se define como el conjunto de puntos
correspondiente a un polígono y su interior. Para los dos últimos gráficos :
ABCDE, es un PENTÁGONO ( Polígono de 5 lados) y define una región pentagonal.
PQRSTUV, es un HEPTÁGONO ( Polígono de 7 lados ) y determina una región heptagonal.

Definición.- Un conjunto de puntos, se llama convexo, si el segmento determinado por dos puntos
cualesquiera del conjunto, está contenido en él.

Son ejemplos de conjuntos convexos :

Una región tnanguiar Un cono


Un círculo Un cilindro
(sólido) (sólido)

En el caso de la región triángular: A

V A,B e A ,( A * B AB c A

16
Análogamente, para los otros ejemplos.

Definición.- Un conjunto de puntos se llama no convexo, si existen al menos dos puntos distintos
de dicho conjunto, qe determinan un segmento con algunos puntos no comunes al conjunto.

Son ejemplos de conjuntos no convexos :

superficie superficie
cilindrica esférica

Ejemplo.- Indicar verdadero ( V ) o falso ( F ):

a) Una recta es un conjunto convexo ( )


b) Un plano es un conjunto convexo. ( )
c) Un triángulo es un conjunto convexo ( )
d) El exterior de un ángulo es un conjunto de puntos no convexo (. )
e) Un segmento es un conjunto convexo ( )

Solución

a) (V )

r V A,B € r ,( A * B ) => AB c r
A B

b) (V )

V P,G e H, ( P * Q ) =* PQ c H

c) (F )
B

3 M.N e A ABC / MN <z A ABC = AB U BC U AC

A
C
17
d) (V)

Existen puntos de PQ, que no pertenece al


conjunto de puntos exterior al Z AOB

e) (V)

V P^Pa e AB^P, * P2 )=>P1P2 c AB ,


A P¡ B

18
r

INTERSECCION DE FIGURAS PLANAS


Una figura plana, tiene todos sus puntos sobre un mismo plano.
s

L
En la figura (a), las rectas m y n~se intercedan en un punto. En ^ b ), r intersecta a la figura f en
dos puntos y para (c), la intersección de V y la figura L, es de tres puntos. En todos los casos
anteriores diremos que la figuras son secantesise cortan en 1, 2 ó 3 puntos respectivamente.

LINEAS CONVEXAS.- Son aquellas que se intersecan con alguna recta, en un máximo de dos
puntos.
Ejemplos:

LINEAS NO CONVEXAS.- Si alguna recta secante determina sobre ellas, más de dos puntos de
corte.
La Geometría Clásica, menciona estas figuras como cóncavas.
Ejemplos:

19
O bservaciones:

1) Dos rectas contenidas en un mismo


plano y que no se intersecan, reciben el
nombre de paralelas. Por ejemplo, m y
q . En este caso, escribiremos : m // cf
(“ m es paralela a q “).

A veces, suele decirse que las rectas se intersecan, para este caso, en el infinito.

2) Una recta y una circunferencia, pueden s e r :

Recta y circunferencia, Recta y circunferencia, No se intersecan,


tangentes entre sí. secantes entre sí. (cero puntos de inter­
( 1 punto de intersec - ( 2 puntos de intersec - sección ).
ción ). ción ).

3) Veamos algunos gráficos de intersección entre un triángulo y una circunferencia:

1punto 4 puntos 5 puntos 6 puntos

Por supuesto que, podrían hacerse otros gráficos para encontrar un número determi
nado de puntos: 1,2,3,4,5 ó 6.

Notamos que, el mínimo número de puntos de intersección (diferente de cero), entre


estas figuras, es uno y el máximo : 6.

4) Las fórmulas que damos a continuación, permiten encontrar el máximo número de


puntos de intersección entre figuras del mismo tipo, así como entre dos grupos
diferentes.

20
MAXIMO NUMERO DE PUNTOS DE CORTE

1) Para “n" rectas secantes:

4 (3 )
Así, por ejemplo, 4 rectas se cortan como máximo, en: = 6 puntos

2) Para “n” circunferencias secantes:

3 circunferencias secantes, se cortan,


como máximo, en 3(2) = 6 puntos.

3) Para “n" triángulos:

Si se tienen 10 circunferencias, en
contraremos como m á xim o :
3 x 10(9) = 270 puntos de corte.

4) Para “n” cuadriláteros convexos:

5) “n” pentágonos convexos se cortan, como máximo, en:

6) En general, “n“ polígonos convexos de “L" lados cada uno, se cortan como máximo, en:

21
Por ejem plo, “n' polígonos de 11 lados cada uno (convexos) tienen como fórmula para el
máximo número de puntos de corte: 11 n ( n - 1 ) . De modo que, 5 de estas figuras se
cortarán en un máximo de: 11 x 5 ( 4 ) = 220 puntos.

Ejemplo:¿ En cuántos puntos se cortan, como máximo, 10 icoságonos convexos ?

Solución

Un icoságono es el polígono de 20 lados. Luego, en la fórmula del 6), debemos reemplazar:

L = 20 =s> número de lados,


n = 10 => número de polígonos.

número de puntos = Ln ( n - 1 ) = 2 0 x 1 0 ( 9 ) = 1800.

Ejemplo : ¿ En cuántos puntos se intersecan, como máximo, 5 octógonos convexos ?

Solución

El octógono es un polígono de 8 lados. Entonces:


L = 8 y n = 5. En la fórmula del 6):

Ln ( n - 1 ) = 8 x 5 ( 4 ) = 160 puntos.

7) Dos polígonos convexos, de diferente número de lados, se intersecan, como máximo,


en un número de puntos equivalente al doble del número de lados menor.

Así, por ejemplo:

* 1 triángulo y 1 cuadrilátero:

* 1 cuadrilátero y 1 pentágono:

* 1 Decágono (10 lados) y un octógono ( 8 lados), convexos, se cortan como máximo en :


2 x 8 = 16 puntos.

* Un cuadrilátero y una circunferencia:

( La circunferencia se considera como un polígono de infinitos lados ).

22
Completar:

Como máximo, el número de puntos de corte entre:

a) Un triángulo y un pentágono convexo, es:

b) Un dodecágono convexo (12 lados) y un ¡coságono convexo (20 lados), es

c) Un polígono convexo de 50 lados y una circunferencia, e s : ............. .

8) Para “n" figuras cualesquiera (convexas ó no convexas), del mismo tipo, el máximo número

de puntos de corte, es:

Siendo K, el número máximo de puntos en que se cortan 2 de dichas figuras.

Por ejemplo, encontremos la fórmula para calcular el máximo número de puntos de corte
entre “n” elipses.

Una elipse, es de la forma:

Hallamos el valor de K, graficando dos elipses de


modo que se tenga el número máximo de puntos
de intersección entre ellas.

K = 4 puntos
Como máximo

Entonces, para “n" elipses, la fórmula se obtiene al reemplazar este valor de K en la expresión
anterior:
4 n ( n - i)

Ejem plo.- Hallar una formula para calcular el máximo numero de puntos de corte entre “n
figuras de la forma:

Solución

Graficamos dos de dichas figuras a fin de obtener el valor de K

• •
Para un” de estas figuras Juego

de reemplazar el valor de K en la fórmula del 8).


23
Ejercicios y Problemas resueltos

* Nota.- Vamos a reemplazar el enunciado:" Máximo número de puntos de corte”, p o r: MN PC.

1) Hallar el MNPC entre 10 rectas y 5 circunferencias, al cortarse todas estas figuras entre sí.

A) 65 B) 120 C) 145 D) 165 E) N.A

Solución

10 rectas.
\\ 5 circunferencias.

El método de solución consiste en contar por separado los puntos de corte: rectas
solas,circunferencias solas y al final la combinación.
El resultado se obtiene sumando los parciales. Asi:

a) Las 10 rectas solas, se cortan como máximo, en:

10(9)
= 45 p u n to s ( 1)

b) Las 5 circunferencias: 5 ( 4 ) = 20 p u n to s ( 2 ).

c) Para el número de puntos entre rectas y circunferencias:


Como cada recta corta a una circunferencia en 2 puntos y son 5 circunferencias; entonces
una recta corta a las 5 circunferencias en: 2 x 5 = 10 puntos. Pero, son 10 rectas; entonces
tendremos a q u í: 10 x 10 = 100 puntos. Esto mismo, es:

2 x 10 x 5 = 100 p u n to s (3 )

Número
de rectas.
Numero de puntos
entre una recta y Número de
una circunferencia. circunferencias

Finalmente, sumando los resultados parciales (1), (2) y (3);


45 + 2 0 + 100 = 165 puntos.
Rpta: ( D ).

2) Hallar el MNPC entre 11 rectas secantes y 5 triángulos, al cortarse todas estas figuras entre
sí.

A) 225 B) 125 C) 115 D) 175 E) 205


24
Solución

11 <

5 A

Veamos:
11(10) cc
a) Las 11 rectas, por sí solas: — - -------- ^ puntos (1 )

b) Los 5 triángulos entre sí: 3 x 5 ( 4 ) = 60p u n to s ........ ( 2 ).

c) Las 1 1 $ a los 5 a :
2 x 11 x 5 = 110 p u n to s ............( 3 ).
A A A

Número de A

Número de $
Número de puntos
entre 1$ y 1 A.

Luego, sumando los resultados (1), (2) y (3 ): 55 + 60 + 110 = 225 puntos. Rpta: ( A )

3) Hallar el MNPC entre 11 circunferencias y 8 triángulos al intersecarse todas estas figuras


entre sí.

A) 726 B) 706 C) 806 D) 906 E) 278

Solución

11 0
8 A

a) Las 11 circunferencias entre sí, se cortan como máximo, en: 11(10) = 110 puntos ... (1 )

b) Los 8 triángulos: 3 x 8 x 7 = 168 puntos ... ( 2 ).

c) Las 1 1 0 a los 8 A:
6 x 11 x 8 = 528 p u n to s ( 3 ).
♦ « t

6 puntos
( m áxim o).
©

25
Finalmente, sumando (1), (2) y (3), se obtienen
806 puntos.
Rpta: ( C ).

4) Hallar el MNPC entre 21 rectas secantes, 15 circunferencias y 1%triángulos, al intersectarse


todas estas figuras entre sí.

A) 4110 B ) 4100 C) 4001 D) 4020 ,e jN .A

Solución

El método es sim ilar: Evaluamos el máximo número de puntos de corte entre las rectas solas,
las circunferencias entre sí, los triángulos por si solos y luego hacemos las combinacio­
nes en grupos de dos. Asi:

21(20)
a) Las 2 1 ^ : ■ = 2 1 0 p u n to s .............. ......(1 )■

b) Las 1 5 © : 1 5 ( 1 4 ) = 210 p u n to s ........... ..... ( 2 ) .

c) Los 12 a : 3 x 12 ( 11 ) = 396 puntos ..... ( 3 ) .

d) 21 X a 15 0 :
2 x 21 x 15 = 630 p u n to s ..................... ( 4 ).
▲ ▲ A

2 ptos.

vT 0
e) Las 21 / a los 12 A :
2 x 21 x 12 = 504 p u n to s ................... ( 5 ).
A A A
2 ptos.

A
f)
6 x 15 x 12 = 1080 (6 )
A A A

6 ptos

26
El MNPC total lo obtenemos sumando los resultados parciales del (1) al (6)
210 + 210 + 396 + 630 + 504 + 1080 = 3030 ptos.
Rpta: ( E ).

5) Hallar el MNPC entre 21 triángulos y 10 cuadriláeros convexos, todos secantes entre sí

A ) 2080 B ) 2888 C ) 1880 D ) 2780 E ) 2880

Solución

: \ 21 a
L. 10 □

Procedemos como antes:

a) Los 21 triángulos se cortan como máximo,


en : 3 x 21(20) = 1260 p u n to s .............. (U

b) Los 10 cuadriláteros convexos: 4 x 10(9) = 360 p u n to s ................... ( 2 ).

c) Los 21A a los 10 a :

6 x 21 x 10 = 1260 ( 3 ).
▲ ▲ *


2 x 3 = 6 ptos

Número de lados menor.


( Ver el N9 7 ) de ésta teoría.

Sumando lo obtenido en a), b) y c ):

MNPC = 1260 + 3 6 0 + 1260 = 2880 puntos

Rpta: ( E ).

6) Hallar el MNPC entre 6 cuadriláteros convexos; 11 pentágonos convexos y 21 octógonos


convexos, al intersecarse todas estas figuras entre si.

A ) 7414 B) 7604 C ) 6704 D) 4706 E) N.A

27
Solución

En este caso, para figuras de la misma naturaleza usaremos : Ln ( n - 1 )


fórmula vista en el número 6) de teoría.

\ 6 □.
> 11 pentágonos. ( 5 lados )
✓ 21 octógonos. ( 8 lados ).

Se tienen:

Los 6 cuadriláteros convexos:


4 x 6 ( 5 ) = 120 p u n to s ............... (1 )

Los 11 pentágonos convexos:


5 x 1 1 ( 1 0 ) = 550 p u n to s (2 )

Los 21 octógonos convexos:


8 x 2 1 ( 2 0 ) = 3360 puntos (3 )
Ahora, en grupos de dos:

Los 6 □ y 11 pentágonos:
8 x 6 x 11 = 528 puntos
A A A

□ pentágonos.
2 x 4 = 8 ptos

menor numero
de lados.

6 □ y 21 octógonos.
8 x 6 x 21 = 1008 puntos
A A A

a octógonos.
2 x 4 = 8 ptos.

í
menor numero
de lados.
f) 11 pentágonos y 21 octógonos:

8 x 11 x 21 = 1848p u n to s ............( 6 ) .
▲ A

í
octógonos
2 x 4 = 8 ptos.
A pentágonos

número de
lados menor.

Finalmente, sumamos los resultados parciales del (1) al (6):


MNPC = 120 + 550 + 3360 + 528 + 1008 + 1848
MNPC = 7414 puntos.

Rpta: ( A ).

7) Hallar el MNPC entre 10 rectas paralelas, 5 rectas secantes y 6 triángulos, al intersecarse


todas estas figuras entre sí

A) 360 B) 340 C) 350 D) 370 E) 330

Solución

10 / paralelas
4

5 ¿ secantes
6 A

Tenemos:

a) Las 10 paralelas entre s í : cero puntos de corte

5(4)
= 10 puntos 2 ).

c) Los 6 A : 3x6( 5) = 90 puntos 3 ).

Ahora, en grupos de 2 :

d) 10 paralelas y 5 secantes:
1 x 10 x 5 = 50 p u n to s ...( 4 ) .
▲ A A

|____ número de
número de puntos entre una secantes
paralela y una secante. J número de
paralelas.

29
Las 10 paralelas a los 6 triángulos ;
2 x 10 x 6 = 120 puntos ......... ( 5 )
▲ ▲ ▲

2 puntos
paralelas.

5 rectas secantes y 6 triángulos :


2 x 5 x 6 = 60 puntos .........( 6 )
▲ ▲

2 puntos.
7 secantes

Sumando ahora, todos los resultados parciales :

MNPC = 0 + 10 + 90 + 50 + 120 + 60 => MNPC = 330 puntos

Rpta : ( E ).

Si a un grupo de rectas de un plano, se le agrega una, el máximo número de puntos de corte


se duplicaría. Hallar el número de rectas original.

A) 3 B) 4 C) 5 D) 6 E) 7

Solución

Si, inicialmente, hubieran “n” rectas, el número máximo de puntos de corte sería

puntos.
2

Al agregar una al grupo anterior: (n + 1) rectas ; estas se cortan, en :


( n + 1) ( n + 1- 1 ) (n+1)n puntos

Según enunciado, el segundo resultado debe ser el doble del primero. Luego :

(n + 1)n 2 , , n( n ~ 1)

Resolviendo esta sencilla ecuación : n = 3 rectas

Rpta: ( A ).

Si a un grupo de “n” rectas secantes se agrega una recta, el máximo número de puntos de
corte aumentaría en 12. Hallar el valor de “n”.

A) 12 B) 11 C) 13 D) 6 E) 24
Solución

Como, al agregar una recta, al grupo existente de un” rectas, la nueva debe cortar a cada
una de las anteriores en un punto, entonces el MNPC se incrementará en “n”.
Por lo tanto: n = 12

Rpta: ( A ).

Si a un grupo de un” rectas secantes se agregan dos rectas, el máximo número de puntos
de corte aumentaría en 15. Hallar “n”

A) 5 B) 6 C) 7 D) 8 E) N.A.

Solución

Cada una de las rectas nuevas determina en el grupo existente de “n” rectas, un total de “n”
puntos más. Entonces:

- Número de puntos en que las 2 nuevas rectas cortan a lasya existentes : 2n

- Número de puntos entre las 2 nuevas rectas : 1

Luego: 2n +1 = 15

De donde: n = 7.

Rpta:C

Si a un grupo de “n” triángulos se le quita uno, el máximo número de puntos de corte dismi­
nuye en 18. Hallar “n"

A) 3 B) 4 C) 6 D) 5 E) 9

Solución

Un triángulo corta a otro en 6 puntos , como máximo.


Al extraer un triángulo al grupo de “n”, éste cortará a cada uno de los (n- 1) restantes, en
6 puntos.

Luego; 6(n-1) = 18 n=4

Rpta: ( B ).

Al duplicarse el número de rectas secantes, el máximo número de puntos de corte se


quintuplica. Hallar el número inicial de rectas

A) 3 B) 6 C) 15 D) 10 E) N.A.

31
Solución

n( n - 1 )
Sea “n” el número inicial de rectas. Ellas determinan — - — puntos.

Si se duplica el número de rectas, ahora tendremos 2n rectas que se cortan en


2n( 2 n - 1 )
------- puntos.

Según enunciado éste último resultado debe ser cinco veces el anterior.
. 2 n (2 n -1 ) n(n-1)

Resolviendo : n= 3

Rpta: ( A ).

Si a un grupo de “n” polígonos convexos, de “L” lados cada uno, se agrega otro de la misma
naturaleza y cantidad de lados, el máximo número de puntos de corte se duplica. Hallar “n”.

A) Falta B) 6 C) 2 D) 3 E) 4
conocer “L”

Solución

Es fácil deducir que dos polígonos convexos de “L” lados cada uno se cortan como máximo
en 2L puntos. Luego,el nuevo polígono corta al grupo de “n”, en 2Ln puntos.

Como los “n^polígonos de WL” lados se cortan en Ln (n -1 ) puntos según fórmula y al colocar
el nuevo polígono ésta cantidad se duplica; entonces:

2Ln = Ln (n-1)

/. 2 = (n-1)

De donde: n = 3

Rpta: ( D ).

Hallar el número máximo de puntos de corte,entre 5 octógonos y 10 icoságonos, todos


convexos.

A ) 2670 B ) 2770 C ) 2760 D ) 2870 E ) 7260

Solución
( octógono : 8 lados; Icoságono: 20 lados ).

Los 5 octógonos: 8 x 5 (4) = 160 puntos.


* Los 10 icoságonos: 20 x 10(9) = 1800 puntos

* 5 octógonos y 10 icoságonos:
( 2 x 8 ) x 5 x 10 = 800 puntos

N9 de puntos entre 1 ^e
octógono y 1 icoságono. octógonos. icoságonos.

Sumando los resultados parciales :


160 + 1800 + 800 = 2760 puntos

Rpta: ( C ).

15) Hallar el máximo número de puntos de corte entre 10 rectas secantes, 6 triángulos y 11
cuadriláteros convexos.

A ) 1311 B ) 1312 C ) 1213 D ) 1321 E) N.A

Solución

10(- —
— 9 ) = 45 puntos.
* Las 10 rectas:

* Los 6 triángulos: 3 x 6 ( 5 ) = 90 puntos.

* Los 11 cuadriláteros: 4 x 11 ( 1 0 ) = 440 puntos.

* 10 rectas y 6 triángulos:

2 x 6 x 10 = 120 puntos
■¿x
* 10 rectas y 11 cuadriláteros:

2 x 10 x 11 = 220 puntos
- a *

* 6 triángulos y 11 cuadriláteros:

6 x 6 x 11 = 396 puntos.

En total, la suma: 1311 puntos

Rpta: ( A ).

33
16) Calcular el máximo número de puntos de intersección de 10 rectas paralelas, 12 rectas
secantes y 16 circunferencias secantes.

A) 1130 B) 306 C) 316 D ) 746 E ) 1098

Solución
1 0 / paralelas
12 / secantes
16 O secantes

19) Las 10 paralelas:


* ---------------*
* ----- 5--------►
»

<----- ► Cero puntos.

2®) Las 12 secantes:

n(n-1) 12(11)
= 66 puntos.

33) Las 16 O secantes : n ( n - 1 ) = 1 6 ( 1 5 ) = 240 puntos.

4®) Las 10 paralelas a las 12 secantes:

1 x 10 x 12 = 120 puntos.
... (1 paralela y 1 secante : 1 p u n to ).

5®) Las 10 paralelas a las 16 circunferencias:

2 x 10 x 16 = 320 puntos.

6a) Las 1 2 / secantes y 16 circunferencias:

2 x 12 x 16 = 384 puntos.

Sumando los resultados parciales:

0 + 66 + 240 + 120 + 320 + 384 = 1130 puntos

Rpta: ( A ).

34
Encontrar la cantidad de decágonos que se intersecan, sabiendo que al hacerlo determinan
como máximo 6250 puntos, en los cuales están también considerados los vértices.

A) 20 B) 50 C) 25 D) 60 E) 30

Solución

Sea “n" el número de decágonos. Luego:

Número total de vértices : 10n

Número máximo de puntos de intersección : lOn ( n -1 )

pordato: 10n + i0 n (n -1 ) = 6250

n2 = 625 => n = 25

Rpta: ( C ).

Si a un conjunto de rectas secantes, se le agregase una cantidad igual de rectas, su número


máximo de puntos de corte aumentaría en 330 . Calcular cuántas rectas tiene el conjunto.

A) 10 B) 25 C) 15 D) 12 E) 18

Solución

Si, inicialmente, hubieran “n” rectas, éstas se cortarían, en:


n (n -1 )
— -— puntos.

Al agregar otras “n" rectas al grupo anterior, habrán “2n” rectas que se cortarían en:
2 n (2 n -1 )
puntos.

Usando el dato numérico:


2 n (2 n -1 ) = n (n -1 )

Efectuando:
3n2 - n - 660 = 0
( 3n + 4 4 ) ( n - 1 5 ) = 0

De donde: n = 15

Rpta: ( C ).

Se tienen n circunferencias secantes. Si se quitan dos circunferencias, el número máximo


de puntos de corte disminuye en 30. Hallar n.

A) 9 B) 8 C) 6 D) 10 El 12

35
Solución

* Las “n” circunferencias secantes: n ( n - 1 ) puntos.

* Al quitar 2, las (n-2) circunferencias restantes, se cortan en: ( n - 2 )[ ( n - 2 ) - 1 ] puntos

* Con el dato : n ( n - 1 ) - 30 = ( n - 2 ) [ ( n - 2 ) - 1 ] .

Resolviendo, hallamos: n = 9

Rpta: ( A ).

20) Encontrar el número máximo de puntos de corte que hay entre “P decágonos convexos y
“F” cuadriláteros convexos.

A) 7F (F-1) B) 7F (2F-3) C) 4F (3F-2) D) 2F (11F-7) E) 3F (15F-8)

Solución

J F decágonos convexos

F cuadriláteros convexos

18) Los F decágonos (10 lados cada u n o ): 10 F (F-1)

28) Los F cuadriláteros: 4F (F-1)

38) Los F decágonos con los F cuadriláteros:


í \

N8 de puntos
de corte entre N8 de decágonos N8 de cuadriláteros
1 decágono y 1 cuadrilátero. /
v /
Así:
( 2 x 4 ) x ( F ) ( F ) = 8F
doble número de
--------------- lados menor

Finalmente, sumando los resultados parciales:

10F (F-1) + 4F (F-1) + 8F: 2F (11 F-7) puntos

Rpta: ( D ).

21) “n” polígonos convexos de" fia d o s cada uno se intersecan en 6240 puntos, como máximo
Si quitamos un polígono, el número de puntos de intersección disminuye en 312.
Hallar ( i + n ).

A) 33 B) 40 C) 42 D) 46 E) 44
36
Solución

Los “n" polígonos convexos, de Y lados : ¿n (n-1) = 6240 ... ( 1 ).

Al quitar un polígono, el # de puntos disminuye en 312, siendo éste el número de puntos que
dicho polígono determina en los otros :

2£(n-1) = 312

¿ (n -1 ) = 156 (2 )

6240
( 2 ) en (1 ) : n = n = 40 ( luego : i = 4.
156

Es decir ( i + n ) = 44

Rpta: ( E ).

Hallar el máximo número de puntos de intersección de 10 cuadriláteros no convexos

A) 540 B) 720 C) 820 D) 400 E) 360

Solución

Debemos usar la fórmula vista en el número 8) de la teoría, para encontrar el MNPC entre
“n” cuadriláteros no convexos y aquí reemplazar el valor de n:

. « i 0 r , K n (n -1 )
MNPC = ----- puntos.

K es el número máximo de puntos en que se cortan 2 cuadriláteros no convexos. Para ello,


tenemos el siguiente gráfico:

K = 16 puntos.

_ 16 n (n -1 )
Entonces, para “n” de estas figuras, la fórmula es: MNPC
2

Y si n=10 : MNPC = 8 x 1 0 ( 9 ) = 7 2 0 puntos

Rpta: ( B ).

37
23) Hallar el MNPC entre 5 elipses y 11 cuadriláteros no convexos

A ) 1360 B ) 1260 C ) 1460 D ) 1560 E) 960

Solución
( 5 e lip se s).

MNPC = ?
( 11 cuadriláteros
no convexos)

a) Para las 5 elipses: Usamos la fórmula vista en el número 8) de teoría en que hallamos para
un” elipses la expresión: 2n (n-1) puntos.

como n = 5, tendremos: 2 x 5 (4) = 4 0 puntos ...(1 ).

b) Para los 11 cuadriláteros no convexos, según la fórmula vista en el problema anterior:


8n (n-1) puntos. Reemplazando n = 11

obtenemos: 8 x 11 (10) = 880 p u n to s .................. ( 2 ).

c) Las 5 elipses a los 11 cuadriláteros no convexos:

8 x 5 x 1 1 = 440 puntos ............( 3 )


▲ ▲
cuadriláteros
8 puntos. no convexos.

elipses
Finalmente, sumando (1), (2) y (3):

MNPC = 40 + 880 + 440 = 1360 puntos

Rpta: ( A ).

24) Encontrar el número máximo de puntos de corte que hay entre 3 polígonos convexos de 2K
lados y 6 polígonos convexos de 3K lados cada uno.

A ) 102 K B) 112 K C ) 122 K D ) 164 K E ) 174 K

Solución

3 polígonos de 2K lados c/u


MNPC = ?
6 polígonos de 3K lados c/u

Usaremos la fórmula : L . n (n-1) ptos,


vista en el número 6) de teoría.

38
a) Para los 3 polígonos de 2K lados c/u:
( n = 3 y L = 2K ).

Se tienen: 2K x 3 ( 2 ) = 12K p u n to s ( 1)

b) Para los 6 polígonos de 3 K lados c/u:


( n = 6 y L = 3K ).

habrán: 3 K x 6(5) = 90 K p u n to s ............... ( 2 )

c) Los 3 polígonos de 2 K lados y 6 polígonos de 3 k lados, se cortan como máximo en

( 2 x 2K ) x 3 x 6 = 72K p u n to s ............( 3 )
A A A

número de polígonos
MNPC entre 1 polígono de 2 K de 3K lados c/u.
lados y otro de 3 k lados.
( doble menor número de lados.) número de polígonos
de 2 K lados c/u.

Sumando ahora los resultados (1), (2) y (3 ), tenemos :

MNPC = 1 2 K + 9 0 K + 7 2 K MNPC = 174K puntos

Rpta: ( E ) .

25) Deducir una fórmula para encontrar el número total de puntos en que se cortan “ n " circun
ferencias dispuestas como se indica :

A) n(n-1) B) 2n (n-1) C )3 n (n -1 ) D)n(n+1) E) 2n (n+1)

Solución

El análisis lo hacemos incrementando cada vez en uno el número de circunferencias.


Debemos relacionar el número de puntos con el número de circunferencias.
Así:

número de circunferencias numero de puntos

2 puntos ---------2 (2 -1 )
CD
GQD 4 puntos ------- ► 2(3-1)

GOQO 6 puntos ------- ► 2(4-1)


39
8 puntos - 2(5-1)

10 puntos - 2 (6 - 1)

“n” circunferencias ( fó rm u la )

26) Luego de disponer 50 circunferencias y 20 rectas paralelas, como indica la figura siguiente,
hallar el máximo número de puntos de corte.

A ) 2098

Solución

a) Las 50 circunferencias determinan entre sí,


según la fórmula deducida en el problema anterior
2 (50 - 1) = 98 p u n to s ( 1 ).

b) Las rectas entre sí: 0 p u n to s ( 2 ).

c) Cada recta corta a una circunferencia en 2 puntos. Una recta corta a las 50 circunferencias
en 2 x 50 = 100 ptos y las 20 rectas, en : 20 x 100 = 2000 p u n to s (3).

Luego, el MNPC se obtiene sumando los resultados (1), (2) y (3 ):


MNPC = 98 + 0 + 2000 = 2098 puntos.

Rpta: ( A ).

27) En la figura, las rectas L ^ y L2 son paralelas ente sí.


Sobre L1 se toman “m" puntos y sobre L2 , “n” puntos. <__________ ►M

* l2

Hallar el máximo número de puntos de corte en que las rectas determinadas por los “m"

puntos de L1 y “n“ puntos de L2 , cortan a la circunferencia.

A)mn B) 2mn C )m n(m n-1) D)2m n(m n-1) E) Ninguna.


40
Solución

Cada recta intersecta a la circunferencia, como máximo, en 2 puntos. El número de rectas


determinadas, lo obtenemos a s í:
H H
- Un punto de L1( con los un” puntos de L2 determinan “n" rectas. Luego, los “m” puntos de
H
L-i con los “n" puntos de L2 , determinan: mn rectas.

Entonces, el número de puntos en que esta cantidad (mn) de rectas corta a la circunferencia,
es: 2mn, como máximo.
“m" puntos
Rpta: ( B ). <— M

5 -^ u2
n puntos

28) Hallar MNPC entre “n” circunferencias, “2n” rectas secantes y '“n” triángulos, al cortarse
todas estas figuras entre sí. v

A) 5n (4n-1) B )4 n (5 n -1 ) C )5 n (4 n + 1 ) D )4 n (5 n + 1 ) E) Ninguna anterior

Solución n O
MNPC = ? 2n X
n A
a) Las “n” circunferencias: n ( n - 1 ) puntos.

2 n ( 2n-1) . .
b) Las 2n rectas secantes • i--------- -} = n( 2 n - 1 )
2 V ;

c) Los Mn” triángulos: 3 n ( n - 1 ) puntos.

d) un" circunferencias a “2n” rectas: 2 x n x 2n = 4n puntos.

e) wn” circunferencias a “n” triángulos: 6 x n x n = 6n2 puntos

f) u2n” rectas a “n” triángulos: 2 x 2n x n = 4n puntos

Sumamos los resultados parciales:

efectuando: MNPC = 5 n (4 n -1 )p to s

Rpta: ( A ) .

29) Se muestran “n” circunferencias concéntricas


y otras un’ circunferencias menores formando una argolla.

El máximo número de puntos de corte, es:

A) 2n B) 4n' C) 4n D) 2n(n-1) E) 2n(n+t)

41
Solución

El número de puntos entre las circunferencias que forman la argolla se determina así:

"A La argolla se obtiene al intersecar las


) circunferencias extremas ( 2 puntos más ).

"n" circunferencias en esta


posición : 2 ( n -1 ) puntos.

Entonces, el número de puntos en la argolla será: 2 ( n -1 ) + 2 = 2n p u n to s (1 ).

Cada circunferencia de la argolla corta a una de las concéntricas, en 2 puntos. Así que, las
“n* circunferencias de la argolla cortan a las “n" concéntricas, en:
2 x n x n = 2 n 2 .......... ( 2 ).

El número total de puntos de corte se obtiene sumando los resultados (1) y (2):

MNPC =2n + 2n2 = 2 n ( n + 1)

Rpta: ( E ).

Al número máximo de puntos de corte entre “n” polígonos convexos, de “L” lados cada uno,
se le suma el máximo número de puntos de corte entre “n" polígonos de “2L" lados cada uno,
obteniéndose en total 630 puntos. Hallar: L + n.

A) Faltan datos B) 13 C) 12 D) 11 E) 14

Solución

MNPC entre “n” polígonos de “L” lados : Ln ( n - 1 ).

MNPC entre “n” polígonos de H2L” lados : 2L x n ( n -1 ).

Según enunciado:
Ln ( n - 1 ) + 2 Ln ( n - t ) = 630
3 L n ( n -1 ) = 630
Ln ( n - 1 ) = 2 1 0

En factores primos, 210 es : 2 x 3 x 5 x 7

Escrito este producto en forma que contenga dos factores consecutivos, para luego
comparar con el primer miembro, tenemos:

Ln(n-1) = 7 x 6 x 5

De donde: L = 7 y n = 6.-. L + n = 13

Rpta: ( B ).
r

CAPITULO 3
V

SEGMENTOS

SEGMENTO.- Es la porción de recta limitada por dos puntos llamados extremos

El segmento AB de la figura adjunta,


x s
se denota: AB ó B A . Los puntos A y B son los extremos.

Si la longitud o medida del segmento AB es 10 unidades, podemos e s c rib ir: AB = 10 ó


m AB = 10. En este último caso, la m se lee: medida.

SEGMENTOS CONGRUENTES.- Son aquellos que tienen igual longitud.

Así, si AB y CD son congruentes, escribi­


remos: AB = CD ,
4 o simplemente: AB = CD
D

PUNTO MEDIO DE UN SEGMENTO.- Es aquel que lo divide en dos congruentes.Se diceque


dicho punto biseca al segmento.

M es punto medio-de AB
M AB
AM = MB o AM = MB =

PUNTOS COLINEALES.- Son los que pertenecen a una misma recta. Por ejemplo, los
puntos A, B, C, D, contenidos en la recta r.
Además , si se marcan sobre la recta en
el orden en que se mencionan, diremos 4 -------■--------- ■---------- ►
que A,B,C,D, son consecutivos. A B C D

43
Ejemplo.- ¿Cuántos segmentos se pueden contar en la figura adjunté?

B D

Se observan: AB, AC, AD, AE ; BC, BD, BE ; CD, CE y DE

En to ta l: 1.0 segmentos.

Nota.- En general, “n” puntos colineales y consecutivos, determinan

n ( n 1) segmentos.
2

Así, para el ejemplo a n te rio r: n = 5 puntos.


5/4 )
El número de segmentos que se obtiene =* ' = 10

OPERACIONES CON LOS SEGMENTOS:

Basados en el Postulado: “ El total es igual a la suma de sus partes”, tenemos

AB + BC = AC
B

Q R PQ + QR + RS = PS

AB+ BC + CD + DE + EF = AF.
AB + BE = AE ; AC + CD + DE = AE
B D
BD + DF = BF ; etc

También, podemos efectuar diferencias entre las longitudes de dos segmentos para
representar un tercero; por ejemplo:

MN = M T - N T ; NT = MT - MN.
M N T

* DISTANCIA ENTRE DOS PUNTOS.- Es la longitud del segmento que los une. Así, la '
distancia entre los puntos A y B, es AB. .___________________ .
A B

OBSERVACIONES:

En algunos gráficos, vamos a representar las longitudes de los seg


mentos con letras, usualmente, minúsculas. Por ejemplo:
k
El segmento AB , mide “x unida
A B > des de longitud” AB = x .

Q R
► Para la longitud de PR como:
PR = PQ + QR PR = a + b
n En este caso:

> FG = E G -E F FG = n - x

Si se enunciara como dato:


AB = 2BC ( La longitud de AB,
K— 2a
B
► el doble de la longitud de BC );
entonces, haciendo: BC = a; ten­
dremos: AB = 2a.

En aquellos casos de segmentos congruentes:

M
* N R K R

MN = NR RS = ST
JK s KL
o MN = NR

POLIGONAL.- Se dá este nombre al conjunto de dos o más segmentos consecutivos


trazados en diferentes direcciones, sin intersecarse dos no consecutivos.
P. convexa
ABCDE
R no conve­
xa PQRST

Cada segmento es un lado y cada punto es un vértice de la poligonal.


Una poligonal se llama convexa, si alguna recta la interseca, como máximo, en dos puntos.
La poligonal es no convexa, si la recta determina sobre ella más de dos puntos. Esta última
poligonal se menciona en algunos textos como cóncava.

POSTULADO DE LA MINIMA DISTANCIA.- “La mínima distancia entre dos puntos, es la


longitud del segmento que los une”.
De modo que, en la figura adjunta,
el menor camino para ir de A hacia B,
es A B . Entonces:

AB < AC + CB
«£ -
A B
E j e m p l o En el gráfico anterior: AC = 12 y CB = 8. Hallar el máximo valor entero de AB

Solución .- Tenemos :
AB < AC + CB
AB < 12 + 8
AB < 20
Entonces, el máximo valor entero de AB: 19.
45
POLIGONALES ENVUELTA Y ENVOLVENTE.- Se determinan al trazar dos poligonales
cuyos extremos coinciden, hacia un mismo lado
D
y sin intersecarse en algún otro punto.
Para el gráfico adjunto:

ACDEB Envolvente
AMNB Envuelta

( A y B, son los extremos comunes y las poligonales están a un mismo lado de AB)

TEOREMA DE POLIGONALES.- Toda poligonal envolvente es mayor que su respectiva


envuelta, de la misma naturaleza. Así, para el anterior gráfico:
AC + CD + DE + EB > AM + MN + NB. C

Vamos a demostrar este teorema para poligonales


de dos lados, como en la siguiente figura:

Demostración

Prolongamos AM hasta su intersección en H,


con B C . Luego, por el postulado de la mínima
distancia:

A ACH AC + CH > AM + MH
A MHB MH + HB > MB B

Sumando miembro a miembro:

AC + MH + CH + HB > AM + MB + MH AC + CH + HB > AM + MB
AC + CB > AM + MB.

Ejemplo.- En la figura adjunta:

AB = 10 ; BC = 12 ; CD = 11 y
AE = EF = FD = x

Hallar el máximo valor entero de x

A) 10 B) 12 C) 11 D) 9
Solución
Observamos, que:
ABCD envolvente
AEFD envuelta.
Entonces, por el Teorema de poligonales:
AE + EF + FD < AB + BC + CD
x + x + x < 10 + 12 +11
3x < 33
De donde: x < 11

Es decir, el máximo valor entero de V , es: 10

Rpta: ( A ) .

PROBLEMAS RESUELTOS
NIVEL I r
1) Los puntos colineales y consecutivos A, B, C y D,
Son tales que: AD = 18, BD = 13 y AC = 12
UMIr,r
Hallar D r
BC.

A) 6 B) 7 C) 8 D) 9 E) 5
Solución

BC = ?
Del gráfico: 18
AB = 1 8 - 13 AB = 5 12

Luego: BC = AC - AB B D
BC= 12-5 BC =7 \* 13

Rpta : ( B ).

2) P,Q Y R son tres puntos consecutivos de una recta.


PQ = 2QR + 1 y PR = 31. Hallar QR.

A) 9 B) 10 C) 11 D) 12 E) 8
Solución
Consideremos el gráfico adjunto. Incógnita: QR -= x.
Entonces:
PQ = 2QR + 1
PQ = 2X + 1 31
2x + 1
Luego, en el gráfico: +
PQ + QR = PR Q R
2x + 1 + x = 31
3x = 30
• • x = 10

Rpta: QR = 10 ...( B ).

47
Se tienen los puntos colineales y consecutivos A,B, C,D tales que
AB AD
AD = 24 , AC = 16 y
BC CD
Hallar: BC

A) 3 B) 4 C) 6 D) 3,6 E) 5

Solución

Consideremos el gráfico adjunto.


Incógnita : BC = x
24
Se observa:
Ifi 8
AB = 16 - x •—
IT D
y CD = 2 4 - 16 = 8 % 16 - x

Reemplazando en la expresión dada:


AB AD 16-x 24
BC CD 8
Entonces:
16-x
= 3 => 1 6 - x = 3x

16 = 4x => — = x = > 4 = x

Rpta: BC = 4 ...( B ).

A, C, D y E, son puntos colineales y consecutivos tal que D sea punto medio de CE y


AC + AE = 50. Hallar AD

A) 25 B) 12,5 C) 50 D) 20 E) N.A

Solución

Incógnita: AD = x.
Sean: CD = DE = a (... D, es punto medio de CE ).

Con el dato:
AC + AE = 50 H
•-
•LU

( x-a ) + ( x+a ) = 50 A

2x = 50
k i
x = 25

Rpta: AD = 25 . . . ( A) .
5) A, B y C, son puntos colineales y consecutivos, tales que 7AB = 8BC y AC = 45.
Hallar BC.

A) 25 B) 19 C) 23 D) 21 E) N.A

Solución
á-------------------- J
Incógnita: BC = x , (45-x) t X
*
El dato : B
7AB = 8BC
7 (45-x) = 8x = 3 15- 7x = 8x
315 = 15x
x = 21

Rpta: { D ).

6) Los puntos consecutivos A, M, B y C pertenecen a la misma recta. M, es el punto medio de


AC .
Hallar MB, si: AB - BC = 32.

A) 8 B) 32 C) 18 D) 16 E) 24

Solución

Con el gráfico adjunto: ----f i---- * -------- 1— — i-------- ►


MB = x Incógnita.

Si BC = a, A M
fe ---- yx a — — v— ■
entonces: MC = x + a y AM = x + a,
ya que AM = MB, por ser M punto
medio de AC .

Reemplazando en el dato :
A B * BC = 32
*
( x + a + x ) - a = 32
2x = 3 2
x = 16

Rpt a: MB = 16 ...( D ).

7) En una recta se tienen los puntos consecutivos A, B, C, D, cumpliendo la relación


4AB- BD - 2CD = 4.
Hallar AD, si AB = 3 y AC = 5.

A) 5 B) 6 C) 8 D) 9 E )7

49
Solución
Como : AB = 3 y AC = 5 BC = 2

Sea CD = x. Luego, reemplazando


en el dato : *\
4AB - BD - 2CD = 4 :
B D
4 ( 3 ) - ( 2 + x)-2x = 4 x = 2

Entonces: AD = 3 + 2 + x = 7.

Rpta: (E).

Sean los puntos colineales y consecutivos E, F, G y H


Si: EF = 8, GH = 9 y EG.GH + EF.FH = FG.EH,
Hallar FG.

A) 10 B) 12 C) 14 D) 17 E) N.A

S olución
x >|< 9 »l
Incógnita : FG = x H
Con el dato: EG.GH + EF.FH = FG.EH
y el gráfico :
( 8 + x )9 + 8 (x + 9 ) x( 8 + x + 9 )
72 + 9x + 8x + 72 = 8x + x 2 + 9x
144 = x2
De d o n d e :
/Í4 4 = x
12 = x

Rpta: FG = 12 ... ( B ).

En una recta se ubican los puntos consecutivos A, B, C, D y E, siendo C punto medio de


AE, además AB = CD. Calcular la longitud de BD. si AE = 18.

A) 6 B) 7 C) 8 D) 9 E) 10

S olución
9 9
r1- ■ \
BD = ? a

AB = CD = a
B D
AE = 18
AC = CE = 9 (y a que C es punto medio de AE ).
Del g rá fic o :

BD = BC + CD
BD = ( 9-a ) + a BD = 9

Rpta: ( D ).

10) M, Nt R, son puntos colineales y consecutivos, tales que 2MN + 3NR = 81

Hallar NR, si MR = 36

A) 12 B) 11 C) 10 D) 8 E) 9

Solución

NR = x = ?

Reemplazando, según el gráfico,


en el d a to : 20
(36 - x)
2MN + 3NR =81
•4
2 (36 - x ) + 3x =81 M N R
72 - 2x + 3x = 81
• x=9

Rpta : ( E ).

11) Los puntos A, B, C y D son colineales y consecutivos

Demostrar, que:

Solución

Comenzando del lado izquierdo de la expresión propuesta : +


B D

1
i i
AC + BD = AC + BC + CD

AC + BD = AD + BC .......... L.q.q.d.

12) A, B, P, C y D, son puntos colineales y consecutivos. CD = 2AB, BP = PC y AP = 12.

Hallar BD.

B) 16 C ) 18 D) 20 E) 24

51
Solución

Incógnita: BD

Del gráfico

BD = 2b + 2a =* BD = 2 ( b + a ) ( 1)

El dato: AP = 12 =* a + b=12

Reemplazando, en ( 1 ): BD = 2 ( 12 )

• BD = 24

Rpta: ( E ).

LN 1
13) Sean los puntos colineales y consecutivos L,M, N, P, Q, siendo : 2LM = MN y
MQ
NQ
H a lla r:
LM

A) 12 B) 1/12 C) 13 D)1/13 E) Ninguna

Solución

Se tienen :

* MN = 2LM Si LM = a,

entonces: MN = 2a

LN 1
* También,de : MQ = 5 ( LN ) = 5 ( 3a )
MQ

••• M Q = 15a NQ = 13a

NQ 13a
Entonces: t t 7 = -------=
LM a

Rpta: ( C ).

14) Sobre una recta se toman los puntos consecutivos P, Q y R. Entre los puntos Q y R se
HR
toma un punto H, tal que : PH = y QR - 4PQ = 28

Hallar QH.

A) 7 B) 5,6 C) 4,8 D) 4,5 E) N.A

52
Solución

Incógnita: QH = x

HR
Del dato : PH =
a .1.* 4a- ' y
HR = 4PH fc» ■- É
Q H R
Si PH = a HR = 4a

Reemplazando en el otro dato :


QR - 4PQ = 28
( x + 4a ) - 4 ( a-x ) = 28
x + 4a - 4a + 4x = 28
5x = 28
x = 5,6
Rpta: ( B ).

15) Sean los puntos colineales y consecuivos A, E, B, P y C ; E, es punto medio de AB y P


lo es de EC.
Hallar PC, Si: AB + 2BC = 36.

A) 8 B) 16 C) 18 D) 9 E) 12

Solución

Considerando el gráfico,
donde la incógnita es PC = x
r*
Según dato : AB + 2BC = 36 X B

Reemplazando:
2a + 2 ( 2x - a ) = 36
2a + 4x - 2a = 36 4x = 36
x =9
Rpta: PC'= 9 ... ( D )

16) En una recta se tienen los puntos consecutivos A, B, C siendo

AC + AB = - BC.
ü
AB
H a lla r:
BC
1 1
A) 3 C) 2
B> 6 °> 3

53
Solución

AB
* ---- = 9 A B
BC

* Del dato:

AC + AB = - BC

Luego:

( AB + BC ) + AB = - BC
ó
AB
2 AB = - BC
ü BC

Rpta: ( D ).

17) Sean los puntos colineales y consecutivos P, Q, R y S, tales que


PQ _ QR RS
3 " 4 " 5
y : 2PQ + 5QR + 8 RS =132
Hallar PQ.

A) 3 B) 6 C) 9 D) 12 E) 4

Solución
3x 4x 5x
Sea:
(/)*

PQ QR RS Q R
= X

PQ QR
Luego : = x => PQ = 3x = x =» QR = 4x

RS
= x => RS = 5x

Reemplazando en el otro dato :


2PQ + 5 QR + 8 RS = 132
2 ( 3x ) + 5 ( 4x ) + 8 ( 5x ) = 132
6x + 20x + 40x = 132
6 6 x = 132
x=2
Entonces : PQ = 3x = 3 ( 2 ) = 6

Rpta: ( B ).
n4
1 1
18) Los puntos A, C, D y B, son colineales y consecutivos. CD = AD = —DB y
2 AC
AB = 24 cm.
Hallar: B D - C D

A) 12 cm B) 14 cm C) 16 cm D) 18 cm E) 20 cm

Solución

Como:
1
CD = - AC AC = 2 CD 24
2x 9x
1
AD = - DB DB = 3 AD D B
O

Luego, si CD = x , entonces :
AC = 2x , AD = 3x
DB = 3 ( 3x ) DB = 9x
En el gráfico
AB = 24 => 12x = 24
• x=2
Se p id e :
B D - C D => 9x - x = 8x = 8 ( 2 ) = 16

Rpta: ( C ).

NIVEL n
,1 /

19) Se tienen los puntos colineales y consecutivos A, B, C, D y E, de modo que : AE = 36,


BD = 9, AC = 23 y AB - DE = 5.
Hallar CD.

A) 1 B) 1,2 C) 1,5 D) 2,5 ,E>2

Solución
36
Considerando el gráfico,
donde la incógnita es CD = x: Ü
B D
CE = A E - A C v * r * - l

23 13
CE = 3 6 - 2 3 CE = 13

Luego, en el dato :
AB DE = 5
O
23- (9-x) ( 1 3- x ) =

23-9 + x-13 + x = 5
De donde:
x =2

Rpta: CD = 2 ... ( E ).

Dados los puntos colineales y consecutivos A, B, C, D.E y F : AB = EF, BC = DE, AD = 18


y AF = 28.

Hallar CD

A) 7 B) 5 C) 4 D) 6 E) 8

Solución
28
Incógnita: CD = x. 18

Sean las longitudes: B D


*t*
AB = a y BC = b. Luego,

por dato :
EF = AB => EF = a
y DE = BC => DE = b
Del gráfico
DF = 28 - 18 => DF = 10
a + b = 10

Entonces, como : AD = 18
+
a +13 + x = 18
O
10 + x = 1 8
■ x = 8.

Rpta : CD = 8 ... ( E ).

En una recta se toman los puntos consecutivos C, R y Z ; además se toma HU” entre UR"
y *‘ZMcumpliéndose: 4CU = UZ y RZ - 4CR = 20.
Hallar RU.

A) 2 B) 4 C) 5 D) 6 E) 10

Solución

Considerando el gráfico adjunto,


a 4a
donde RU = x, es la incógnita:
h
Si CU = a . UZ = 4CU = 4a
R U
Además: RZ - 4CR = 20
Luego:

( x + 4a ) - 4 (a - x ) = 20
x + 4a - 4a + 4x = 20
5x= 20
* x=4

Rpta: RU = 4 ... ( B ) .

2 AB -B C
22) Se tienen los puntos colineales y consecutivos A, B, C, tal que AB =
AC
AC = 12.

H a lla r: AB

A) 8 B) 9 C) 7 D) 6 E )4

Solución

Incógnita: AB = x 1Z
En el gráfico: BC = 12 - x i
B
Reemplazando en el dato

AB -B C
AB = /
AC

2(x2- ( 1 2 - x ) 2 )
x =
12

6x = x - ( 1 4 4 - 2 4 X + x¿ )

De donde:

Rpta : AB = 8 ... ( A ).

OR
23) Sobre una recta se toman los puntos consecutivos M, N, O y R, tales que NO =

Calcular MO en función de MN y MR.

MN + MR
A) MO =

MR + 3MN
B) MO =

57
MR + 3MN
C) MO =
2
MR + MN
D) MO =

E) Ninguna anterior

Solución

D a to : OR
NO =
M N O R

Del g rá fico :
MO = MN + NO

OR
Luego : MO = MN + , usando el dato.

o mejor • MO = MN + — — ^ 2 , ya que OR = M R - M O

Despejando ahora MO
M R+3M N
MO =

Rpta: ( B ).

24) A, B, C, D y E, son puntos colineales y consecutivos tales que


2AB = 3BC = 4CD = 5DE y AE + BD = 56.

H a lla r: AB

A) 15 B) 28/3 C) 14 D) 16 E) 17

Solución
x_
Sean : 2AB = 3BC = 4CD = 5DE = x « -¿ 4 - 3 4

Luego:
B D t
2AB = x -> AB = - ; 3BC = x -» BC =
¿L O

4CD = x -» CD = 4 y 5DE = x -4 DE = 4
4 1 5

Reemplazando en el dato :
AE + BD = 56
/ \
( X X X X' ( X X^
con el g rá fico : —+ —+ — h •+*
Í2 3 4 5,1 l 3 4 ¡

58
28
Efectuando la s u m a : x = 56 —> x = 30
15

x 30 AD
Luego ; AB = — = —------> AB = 15

Rpta: ( A ).

25) Sean los puntos M, N y R , consecutivos y colineales. Hallar MN, si: MN - NR = 6 y

MN NR MR
+ + = 18

A) 12 B) 10 C) 14 D) 15 E) 18

Solución

* Incógnita: MN 6+a

* Como: MN - NR = 6
+
M N R
Entonces : MN = 6 + NR
- S i: NR = a -> MN = 6 + a

Colocamos esto en el gráfico. Luego : MR = 6 + 2a.

Reemplazamos ahora en el otro dato :


MN NR MR 6+a a 6+2a
+ + = 18 -> 1— + — + ■■- - = 18
2 3 6

A fin de cancelar los denominadores en el primer miembro, multiplicamos toda la expresión


por 6. ( m.c.m. de 2,3 y 6 ); obteniéndose:

( 6+a > f 6 + 2a
6x + 6 x —+ 6 x = 6x18
i 2 j 3 . 6

18 + 3a + 2a + 6 + 2a — 108

7a = 84 a = 12

Rpt a: MN = 6 + 1 2 = 18 ... ( E )

:6) A, M, B, C, N y D, son puntos colineales y consecutivos. M y N, bisecan AB y BD.res


pectivamente. Hallar BC, sabiendo además que :

NC = 4 , CD = MB y AD = 36

A) 1,2 B) 2,5 C) 1,8 D) 3 E) 2,8

59
Solución
Consideremos el gráfico adjunto
t

Incógnita: BC = x

Como: BN = x + 4 ND = x + 4 x+8 M x+8 B


Luego : CD = x + 8 -» MB = AM = x + 8 36
Entonces, usando el dato AD = 36 :
4x + 24 = 36
De donde:
x=3

Rpta: BC = 3 ... ( D ).

27) Para el gráfico adjunto, hallar el máximo


valor entero de y \ cuando “x” toma su
máximo valor entero.

A) 29 B) 27 C) 28 D) 26 E) 25

Solución

Primero encontraremos el máximo valor entero de “x".


Por el teorema de Poligonales: AE + ED < AB + BC + CD
Es decir: x + x + 1 < 10 + 9 + 1 1
2x + 1 < 30
2x < 29 —» x < 14,5 El máximo valor entero de x, es : x = 14
* Para hallar y , usamos el postulado de la mínima distancia
AD < AE + ED
y < 14 + 15 , ( con x = 14).
y < 29 Máximo valor entero : y = 28

Rpta: ( C ).

28) A, B, C y D, son puntos colineales y consecutivos tales que : AC + BD = 24.

Hallar la distancia entre los puntos medios de AB y C D .

A) 24 B) 48 C) 6 D) 12 E) 18

Solución

Sea M y N, puntos medios de AB y C D ,


respectivamente. Incógnita: MN.

60
Según el gráfico : MN = x + y + z . (1)
Por dato : AC + BD = 24
Luego : ( 2x + y ) + ( y + 2z ) = 24
2x + 2y + 2z = 24 M B N D

x + y + z = 12

Reemplazando en (1) : MN = 12

Rpta: ( D ).

A, B, C y D, son puntos colineales y consecutivos. M es punto medio de AB y N es punto


medio de C D .

AC + BD
Demostrar, que: MN =

Solución

Considerando el gráfico adjunto, A M B C N D

observamos que :
MN = MB + BC + CN

n
MNú =—
AB + o/™* + -----
BC CD
Es decir:

Dando común denominador en el 2fi miembro :


AB + 2BC + CD
MN = ---------------------

o, m e jo r:
AB+BC+BC+CD
MN =

Pero: AB + BC = AC y BC + CD = BD

Por lo tanto:
„ K1 AC + BD
MN = ...... I.q.q.d

Sean los puntos colineales y consecutivos : A, B, C y D. S i : AB = AC


CD BD
entonces :

A) AB = BC B) BC = CD C) AB = 2CD D) AB = CD E) Ninguna anterior.

61
Solución

Del Dato:
AB _ AC
CD " BD B D
\
Según el gráfico : AC = AB + BC y BD = BC + CD.

Reemplazando en lo a n te rio r:
AB AB + BC
CD BC + CD

Efectuando el producto de medios y extremos :


AB ( BC + CD ) = CD ( AB + BC )
Luego:
AB.BC + £& C D = A & C ÍÍ+ BC.CD
Q ueda:
AB.BC = BC.CD
Simplificando : AB = CD

Rpta: ( D ).

31) En una recta se tienen los puntos consecutivos A, B, C, D de modo que BC = 1 , CD = 2AB

AC CD
1 1
D)1
A>2 B> 3 E>2

Solución

AB = x = ? 2x
i

Con el gráfico y el dato B D

+— = 1
1
AC CD x+1 2x
Luego :
2 x + (x + 1)
= 1
2 x.( x + 1 )
De donde :
3x + 1 = 2 x + 2x

2 x 2 - x -1 = 0
2 X + 1 :0 -» X = — -
( 2 x + 1)(x - 1) = 0 2
x - 1 0 -> x= 1

62
Luego:
x = 1 -* AB = 1

Rpta: ( D ).

32) U, N, I, son puntos colineales y consecutivos UN - NI = 32.


M, biseca UN ; R, biseca NI y Q biseca MR. H a lla r: QN.

A) 32 B) 16 C) 18 D) 4 E) 8

Solución

Incógnita: QN= x yt 2x+¡j M x+a


_ u sv»
Sea : NR = a
Rl = a , QR = x + a,
MQ = x + a y UM = MN = 2x + a

Usando el gráfico, para reemplazar en el dato : UN - NI = 32

( 4x + 2a) - 2a = 32 4x = 32
x=8

Rpta: QN = 8 ... ( E ).

33) Sobre una recta, se marcan los puntos consecutivos P, Q, R, S y T, siendo :


PS.ST = PQ.QT. Entonces, es cierto que:

A) PQ = ST B) QR = RS C) PR = RT D) PQ = RS E) QR = ST

Solución

Reemplazando en el dato: ^ a t |(b c d ^


PS.ST = PQ.QT, las longitudes •--------♦ ---
R S T
representadas en el gráfico:
(a + b + c )d = a ( b + c + d)
Efectuando:
ad + bd + cd = ab + ac + ad

De donde:
bd + cd = ab + ac

o, mejor aún : d ( b + c ) = a ( b + c )

cancelando (b + c): d = a -» ST = PQ

Rpta: ( A ).
63
Sobre una recta se marcan los puntos consecutivos O, A, B y M, de tal manera que :

MA + MB = - A B
2
Si además:OM = x . OA + y . OB

Hallar: x.y.

a» ^ b> - 4 c» ^ e>’

Solución
Se tienen :

MA + MB = - A B (|) •------- ------


2 K h O A B M
OM = x . OA + y . OB ... ( II ).

Vamos a partir de la expresión ( I ) para llegar a una expresión idéntica a la ( II ), y por


comparación deducimos los valores de “x” e “y”*
Así, de ( I ):

MA + MB = - A B
2
En términos de OM, OA y OB :

( O M - OA ) + ( O M - O B ) = | ( OB - OA )
Efectuando:

2 ( O M ) = | ( O B ) + O B - | ( O A ) + OA 2( OM ) = | ( OB ) - 1 ( OA)

De donde :

( OM ) = -. - i • OA + |-O B ( lli) .

Luego, comparando ( I I ) y ( I I I ), se observa, que :


1 5
x = — ; y = —
4 4
5
Se pide : x.y = - —

Rpta : ( B ).

Sobre una recta se toman los puntos consecutivos A, B y C de tal manera que BC es mayor
que AB. Se toman los puntos UP" medio de AB; HQ" medio de BC y "M" medio de AC.
Entonces la expresión QC - AP es igual a :

A)BM B )4 ? C)2 BM D )^ E) —
Sólución

Q C - AP = ?

Sean: BM = a
MQ = b

Luego : BQ = a + b = QC m — H
y: MC = a + 2b = AM B a M b a+b
i 3- ^3-
a + 2b
Entonces : AB = AM - BM
AB = a + 2b - b AB = 2b y AP = PB = b

Ahora, reemplazando en la pregunta del problema:


Q C - AP = ( a + b ) - b
QC -AP = a
Es d e c ir: Q C - AP = BM

Rpta: ( A ).

En una línea se tienen los puntos consecutivos A, B, C, D, E; siendo :


AC.AD = BE.CE ; BC.DE = 9 y AB.CD = 7.

H allar: AC 2 - CE2-

A) 16 B) 4 C) 2 D) 1 E) 8
Solución
Incógnita :
2 2
AC - CE
D a to s : A B C D E

AC.AD = B E .C E .... ( 1 )
BC.DE = 9 .............( 2 )
AB.CD = 7 .............( 3 )

DE ( 1 ) : AC.AD = BE.CE

Con el gráfico:
AC.(AC + CD) = (BC + CE). CE

Efectuando :

Á c 2 + AC.CD = BC-CE + CE 2
De donde :

ÁC 2 - C É 2 = BC.CE - AC.CD
Ahora, tratamos de acomodar el segundo miembro de ésta última expresión para usar los
datos ( 2 ) y ( 3 ):

AC - CE = BC ( CD + DE ) - ( AB + BC ) CD

AC - CE = BC.CD + BC.DE - AB.CD - BC.CD

Es d e c ir: AC - CE = B C .D E -A B .C D

Con ( 2 ) y ( 3 ) : AC - CE =9-7 =2

Rpta: ( C ).

Otra form a :
n I
Usando variable para las longitudes.
B D
Incógnita:

AC - CE = ( m + n )2 - ( r+q )2

Los datos
BC.DE = 9 nq = 9
AB.CD = 7 -> mr = 7

y de : AC.AD = BE.CE (m + n ) ( m + n + r ) = ( n + r + q ) ( r + q )
De ésto último, se obtiene:
(m + n )2 + (m + n ) r = n ( r + q ) + ( r + q )2
Es d e c ir:
( m + n )2 - ( r + q )2 = nq -mr

o también :
i
AC CE = 9 - 7

Rpta: AC -C E =2 ... ( C ) .

Los puntos A, B, C, D, E son colineales y consecutivos AC = 3BD, AB = DE y


AE - 5 BC = 28. Hallar CD.

A) 1 B) 2 C) 3 D) 4 E) 5
Solución

Incógnita : CD = x
3a
Sea BD = a

Según d a to : B C D
—►
AC = 3BD 4 2 a+x ^a-x
4
2 a+x *
AC = 3a a
Luego : BC = a -x y AB = 3a - ( a-x ) -» AB = 2a + x = DE

Por otro lado : AE - 5BC = 2 8 ............( dato ).

Con el gráfico : 5a + 2x - 5 ( a - x ) = 28

E fectuando:
7 x = 28
x=4

Rpta : CD = 4 ... ( D ).

Sobre una recta se toman los puntos consecutivos A, B, C, D, E y F, tal que


5
AC + BD + CE + DE = 26 m y BE = ( - ) AF.
O
Calcular AF.

A )6 m B)13 m C) 1 6 m D) 18m E)20

Solución

AF = ? ♦
A B C D

BE=§AF

* Del dato: AC + BD + CE + DF = 26

Agrupando en forma conveniente :


AE + BF = 26

Luego, desdoblamos B F :
AE + ( BE + EF) = 26

Ahora:
AF + BE = 26

con el d a to :

AF + f AF = 26
8

Esto es :

13
— AF = 26

AF = 16

Rpta: (C)
Sobre una recta se toman los puntos consecutivos A, B y C, cumpliéndose

AB . BC = a AC 2 y — +— = 0
BC AB

Luego:

A) a = — B) a = C) 0 = —! - D) 0 = — E) 0
1+0 2+0 1+ a 7 2 +a

Solución
D a to s:
A B
AB . BC = « A C 2 ..........( 1 )•

AB^ e ............... ( 2) .
BC AB ' '

D e (1 ) :

A B . BC = a ( AB + B C f
De donde :

AB . BC = a ( A B 2 + B C 2 ) + 2 a AB . BC
Es d e c ir:

AB . BC( 1 - 2 a ) = a ( AB 2 + B C 2 )
o, m e jo r:

1- 2a AB2+BC2
a A B . BC
desdoblando el 2 - miembro :

1- 2a AB2 BC2
+
a A B . BC AB . BC
Esto e s :

1- 2a AB BC
= -----+ ......... 3 .
a BC AB ' 7

Ahora, reemplazando ( 2J en ( 3 ) :
1- 2a
= 0
a

De donde, fácilmente obtenemos que :


1
a =
2+ 0

Rpta: (B)
40) Se tienen los puntos consecutivos A, B, C y D, si : CD = 12m. Calcular la longitud del
segmento que une los puntos medios de AC y A D .

A) 3 m B) 8 m C) 4 m D) 9 m E) 6 m

Solución
Consideremos el gráfico adjunto. Sean

M punto medio de AC
N punto medio de AD x +a 12

* Incógnita : MN = x A B M x n a C *
* Si NC = a MC = x + a = AM b*---- VVMM— — yvivwr

Luego, c o m o :
AN = ND
Con el g rá fico :
x + a + x = a +12
De donde :
2 x = 12 x = 6

Rpt a: (E).

( NIVEL in

41) Sean los puntos colineales y consecutivos M, N, R y T, tales que


ÑR = RT y MR.NR = 10.

H allar: M T 2 - M Ñ 2

A) 10 B) 15 C) 20 D) 30 E) 40

Solución
Sea el gráfico adjunto, donde se indica el
dato NR = R T , haciendo NR = RT = a i
y MN = b M R T

Se dá :
MR.NR = 10 ( b + a ) a = 10 (D -

Nos piden : MT - MN

MT 2 - MN 2 = ( b + 2a )2 - b2 = b2 + 4ab + 4a 2 - b2
= 4ab + 4a 2 = 4 ( b + a ) a

89
Es d e c ir: MT 2 - MN 2 = 4 ( b + a ) a
Usando la expresión ( I ) : MT 2 - MN 2 = 4 x 10 = 40

Rpta: ( E ).

A, B y C, son puntos consecutivos de una recta. M es punto medio de AC y N es


medio de B C .

Demostrar, q u e :
AB
MN =

Solución
b
Sea el g rá fico :
Tenemos: MN = MB + BN M B N C

S iendo:

BC
MB = AB - AM y BN =

Reemplazando en lo a n te rio r:
BC
MN = A B - A M +

AC
P e ro : AM = Luego:

MN = A
KAKI
B
A D
+ -----
2 2
o, m e jo r:

' A C -B C '
MN = A B -
y
y, como AC * BC = AB, entonces :

MN = A B -

AB
De donde, efectivamente : MN = — .......... I.q.q.d.

Sobre una recta se marcan los puntos colineales y consecutivos A, B, C y D, tal que
AB = CD.
1 1
Luego, la expresión : + ^ 5-
Es equivalente, a:

A) AB.BC b) AB2 + B C 2 C ) ( A B + B D )2 D) AB.BC E) AB.BC

Solución

Considerando el gráfico adjunto,


tenemos :
1 1
+ = ?
AB. AC BC.BD
B D
1 1
+
a(a+b) b(a+b)

b+a
ab( a + b )
Es d e c ir:
1 1 a+b
+
AB. AC BC.BD ab(a + b)

1 1 1
+
AB. AC BC.BD ab
Entonces
1 1 1
+
AB. AC BC.BD AB.BC

Rpta : ( D ).

44) Sean los puntos colineales y consecutivos A, B y C. Hallar la longitud de A B , s i :


AB + BC2 = 11 y AC = 9.

A) 10 B) 9 C) 8 D) 7 E) 6

Solución

Incógnita : AB = x

Del gráfico :
A
BC = 9 - x
A B
En el d a to :
A B + BC2 =11

x + ( 9 - x ) 2 = 11
De d o n d e :
x 2 - 17x + 70 = 0
( x-10 ) ( x-7 ) = 0

Resolviendo: x=10 ó x =7

Pero, como x < 9 , entonces : x = 7

Rpta : AB = 7 ... ( D ).

45) Los puntos A, B y C son cotíneales y consecutivos. AC = 56. M, N, R y S, son puntos


dios de AB , BC , AN y MC respectivamente.

Hallar: RS

A) 28 B) 14 C) 7 D) 18 E) N.A

Sotución

Sea el gráfico:

Incógnita : RS = x.
A ***** M d R x SB b Ñ *
Llamando SB = a, BN = b = NC y MR = d,
entonces:
MB = a ♦ x + d = AM
según, dato : AC = 56
Luego:
2x + 2a + 2b + 2d = 56 => x + a + b + d = 2 8 ............(1).
* [ Debemos hallar a + b + d, para reemplazar en ( 1 ) ].

Por otro lado:


SC = MS —> a + 2b = d + x ( 2 ).
AR = RN -> a + x + 2d = x + a + b ( 3 ).

Sumando, miembro a miembro, las expresiones ( 2 ) y ( 3 ) :

2a + 2b + 2d + x = a + b + d + 2x

Luego : a+b+d = x

Reemplazando ésto último, en ( 1 ) :


x + x =28
x = 14

Rpta: RS = 14 ...( B ).
46) Sobre una recta se ubican los puntos consecutivos A, B, C y D, siendo AB = 4 y CD ■

HaHar MN, si M biseca AC y N biseca B D .

A) 7 B) 8 C) 14 D) 9 E) 10

Solución

Considerando el gráfico apunto, sean :


MB = x, BC = y, CN = z. Luego, la incógnita, es : r
MN = x + y + z . (I). M * B yC 2 y+2
x+y 10 •p
Se observan
AM = MC AM = x + y
ND = BN ND = y + z

Entonces :
AB = 4 2x + y = 4 (II).
CD = 10 2 z + y = 10 (I I I ).

Efectuando la suma de las expresiones ( I I ) y ( I I I ), miembro a miembro :

2x +2y + 2z = 14
x+y+z = 7

Reemplazando esto en ( I ) : MN = 7

Rpta: ( A ).

47) En una recta se tienen los puntos consecutivos A, B, C, D tal que AB = 2BC = 3CD, luego
se consideran los puntos P y Q en AB y CD respectivamente
Hallar PQ s i : PB = QD y AP - CQ = 16.

A) 4 B) 44 C) 36 D) 20 E) 24

Solución
* PQ = ?

* Como AB = 2BC = 3CD Si CD = a,

entonces :
3a _2_ a
a-H ■*)
AB = 3a y BC = — a £ " " 8 -------- c ~ G ~ &
n n

* Además, por dato : PB = QD Sean PB = QD = n


Luego, la incógnita :
PQ = PB + BC + CQ

PQ = n + —a + ( a - n )

73
5
Es d e c ir: PQ = — a (1 ).

* Del otro dato : AP - CQ = 16

Con el gráfico:
( 3a - n) - (a-n) = 16
■va = 8
Finalmente, en ( 1 ) :
5
PQ = - x 8 = 20

Rpta: ( D ).

48) A, B, C, D, E, F, G y H, son puntos colineales y consecutivos, tales que :


2 ( AH ) = 3 ( BG ) = 5 ( CF ), y :
AD + BE + CF + DG + EH = 620

Hallar AH.

A) 60 B) 400 C) 160 D) 276 E) 300


Solución

* ah = ? •;— ■— +•— " — •— ---------------- -


A B C D E F G H
* D e: 2 (AH) = 3 ( BG ) = 5 ( CF )

Se obtiene :

* Por otro lado : AD + BE + CF + DG + EH = 620

Agrupando como se indica y haciendo uso del gráfico :

1__________ I

2 2
AH + - AH + - AH = 620
o 5
Efectuando :
31
AH = 620
15
AH = 300

Rpta: ( E ).

En una línea recta se ubican los puntos consecutivos A, B, C y D tal que: AB.AD = 3BC.CD.
Calcular a + b + c.

a b e
S i:
CD AB AC

A) 3 B) 4 C) 6 D) 7 E) 8

Solución

AB.AD = 3 B C .C D ( I ).
B C D

CD AB AC (II)

* A partir de ( I ), debemos llegar a una expresión idéntica a (II), para luego deducir los valores
de a, b y c. A s í :
AB.AD = 3BC.CD

En términos de AB, AC y CD, ya que éstos aparecen en ( I I ) :

AB. ( AC + CD ) = 3 ( AC - AB ). CD
AB. AC + AB.CD = 3 AC.CD - 3AB.CD
AB.AC = 3AC.CD - 4AB.CD

Dividiendo entre el producto AB.AC.CD :


AB.AC 3AC.CD 4 A B . CD
AB . AC . CD AB . AC . CD AB . AC . CD
Simplificando :
1 3 4
(III)
CD AB AC

Comparando ahora, las expresiones ( I I ) y ( I I I ), observamos que : a =1, b = 3 y c = 4

Luego:
a+b+c=8

Rpta: ( E ) .
50) En una línea recta "L" se conskJe*an los pontos consecutivos A, B, C y D, tal que :
AD - BC = 15.

Calcular la longitud del segmento que tiene por extremos fos puntos medios de AC y BD

A) 7 B) 6,5 C) 7,5 D) 0 E) 9

Solución

Supongamos que AB < BC < CD.

M, punto medio de A C .
6
N, punto medio de B D . A .BzM ^ ^
' z+x+y

Incógnita : MN.
Si MC = x ; CN = y _>

Sea, además : BM = z. Luego : B N = z + x + y = ND.


Aflora, usando el dato numérico :
AD - BC = 15
l
k
Con el gráfico :
( AM + MN + ND ) - ( BM + MC) = 15
Es d e c ir:
( x + x + y + z + x + y ) - ( z + x ) = 15

Sim plificando:
2x + 2y = 15

De d o n d e :
x + y = 7,5
MN = 7,5

Rpta: ( C ).

AB BE
51) Sean los puntos colineales y consecutivos A, B, E y C, siendo :
AC " BC

1 1 1
Demostrar que : — +
AB BC BE

Solución

A partir de : A B E C t
AB = BE
AC “ BC .............( 1

76
Debemos llegar, a :
1 1 1
+ (2 )
AB BC BE

Veamos, de ( 1 ) : AB.BC = AC.BE

Expresando en términos de AB, BC y BE :


AB.BC = ( AB + BC) . BE

Dividiendo cada miembro entre AB.BC :


AB . BC ( AB + BC )BE
AB . BC AB . BC
/ \
AB BC
1= + BE
\ AB . BC AB . BC
De donde :
1 1 1 1
+ l.q.q.d
BE BC AB AB BC BE

52) A, B y C, son tres puntos distintos del piano , tales que : AB = 5 y BC = 7. H a lla r:

a) El mínimo valor de AC.

b) El máximo valor de AC,

c) La diferencia entre los valores enteros máximo y mínimo de AC, sabiendo que A, B
y C no son colineales.

Solución

a) El mínimo valor de AC, corresponde al gráfico siguiente :


I
Para este caso : AC = 7 - 5 = 2 *l
Rpta: ( a ). B
i

b) El máximo valor de AC, se obtiene con el siguiente gráfico :


Luego : AC = 5 + 7 = 1 2
Rpta: ( b ). i i
B

c) Si A, B y C, no son colineales, determinan un triángulo.


Entonces :
AC < 5 + 7 -> AC < 1 2 ............( 1 ) . B

También :
AC > 7 - 5 -> AC > 2 .......... ( 2 ) .

77
De ( 1 ) : máximo valor entero de AC = 11

De ( 2 ) : mínimo valor entero de AC = 3.

Se pide : 11 - 3 = 8

Rpta: ( C ).

A, B, C y D.son puntos colineales y consecutivos.


AB J AD "
S i: = K
BC CD J

Demostrar,que

Solución

Usando el gráfico adjunto,


donde : BC = AC - AB.
B D
v CD = AD - AC.

Reemplazando en el dato :
AB
= K
BC

te n e m o s :

AB AD
= K
AC-AB \ AD-AC

efectuando : AB ( AD - AC ) = K.AD ( A C - A B )

AB.AD - AB.AC = K.AD.AC - K.AB.AD

trasponiendo términos :
( K + 1 ) AB.AD = K.AD.AC + AB.AC

dividiendo ahora cada término entre el producto de AB.AD.AC :

(K + 1 J A B . A D K.AD.AC + AB.AC
A B . A D . AC AB . AD . AC AB . AD . AC

Simplificando :
(K + 1 ) K 1
' ’ ~ + Iqqd
AC AB AD
54) A, E, C, B, N y D, son puntos colineales y consecutivos, siendo E punto medio de AC y N
punto medio de BD.
Hallar AB, s i : CD = 13 y EN = 10.

A) 7 B) 11 C) 12 D) 16 E) N.A

Solución

* Consideremos el gráfico adjunto :

* Incógnita : AB = 2x + y (1 ).

Según enunciado:

EN = 10 x + y + z = 1 0 ............( 2 ) .
CD = 13 y + 2z = 1 3 ............( 3 ) .

Para obtener la expresión de AB,dada por (1 ), efectuamos lo siguiente : multiplicamos por


2 miembro a miembro la ecuación ( 2 ) y al resultado restamos la ecuación ( 3 ). A s í :

2 ( x + y + z ) - ( y + 2 z ) = 2( 1 0 ) - 13

De donde :
2x + y =7
En ( 1 ) : AB = 7

Rpta: ( A )

55) Un tesoro está escondido en determinado punto de un camino recto que une las ciudades
A, B, C, D, ubicadas en ese orden. Un mapa indica la forma de hallarlo, del siguiente modo:

* Partir de “A” y detenerse en la mitad del camino a “C”

* Luego, seguir y caminar un tercio de la distancia a “D”

* Hacer luego un cuarto del camino hacia “B” y encontrará el tesoro.

Si la distancia entre A y B es 6 Km y la distancia de B a C es 8 Km, estando el tesoro a mitad


del camino entre A y D, ¿Que distancia separa a C y D?

A) 37 Km B) 38 Km C) 39 Km D) 40 Km E) 51 Km

Solución
8 tesoro
Según el enunciado," x " es l- H *
é D
“un cuarto del camino hacia B”;
3 _
esto e s :

X = 1 '< L i (D
4^3

79
Como el tesoro está a mitad del camino entre A y D :
^ d 7+d
7+ —+ X a= --------
3 2 (II)

De ( I ) y ( I I ) : d = 45

Luego:
AD = 7 + d AD = 52
CD = AD - AC

Rpta: CD = 38 Km. ... ( B ).

56) Para el gráfico adjunto :


x +y
i* >|« 2x’ V
PS = 18
Z ------------------------- R
Hallar el valor de “y”, sabiendo que “x” es un número entero.

A) 5 B) 6 C) 7 D) 8 E) N.A

Solución

x -> Entero. Para hallar x, usamos este dato y el hecho de que las longitudes deben ser
números positivos :

PQ > 0 —» y - x > 0 ............( I ).


QR > 0 -> 2x - y > 0 ............( II ).
\
Además, como : PS = 18 ( y - x ) + ( 2x - y) + ( x + y ) = 18

De donde : 2x + y = 18

Despejando y m: y = 18 - 2x ..........( I I I ).

Sustituyendo ahora, ( I I I ) en ( I ) :
18 - 2 x - x > 0
18 > 3x (a )
También, de ( I I I ) en ( I I ):
2x-(18-2x)>0
4x > 18

(P)
Luego, de ( a ) y ( p ) :
6 > x > 4,5
Siendo, por dato, x entero : x=5

Finalmente para el valor de My \ reemplazamos en ( I I I ) : y = 18 - 2 ( 5 )

Rpta: y = 8 ... ( D ).

80
57) En la figura :

P Q + RS = 18,3
PR + QS = 32,5

Hallar el mínimo valor entero de QR.

A) 8 B) 7 C) 6 D) 9 E) 5

Solución

Se tienen :

APQR -> PQ + QR > PR


AQRS —» RS + QR > QS

Sumando miembro a miembro :


PQ + RS + 2QR > PR + QS

Con los datos : 18,3 + 2QR > 32,5


2QR > 14,2
QR > 7,1

Luego, el mímino valor entero de QR, es 8

Rpta: ( A ).

58) El perímetro del A ABC, de la figura, es : AB + BC + AC = 2p


O, es un punto cualquiera, interior al triángulo.
Demostrar q u e :

Solución

Usando el postulado de la mínima distancia entre dos puntos y el teorema de las poligonales
envolvente y envuelta :
B

AC < OA + OC < AB + B C ( 1 ).

> BC < OB + OC < AB + A C ( 2 ).

AB < OA + OB < AC + B C ( 3 ).

81
Sumando ahora, miembro a miembro las expresiones ( 1 ) , ( 2 ) y ( 3 )
AB + BC + AC < 2( OA + OB + OC ) < 2 ( AB + BC + AC )

Usando el d a to :
2p < 2 ( OA + OB + OC) < 2 ( 2P )

De donde, efectivamente :
p < OA + OB+ OC < 2 p I.q.q.d.

Para el cuadrilátero ABCD, de la figura adjunta :


AB + BC + CD + AD = 2p
O, es un punto cualquiera, interior a ABCD.
Demostrar, q u e :
p < OA + OB + OC + OD< 3p

Solución
Por el Teorema de las poligonales envueltay envolvente y el postulado de la mínima distancia
entre dos p u n to s :
AD < OA + OD < AB + BC + CD
AB < OA + OB < BC + CD + AD
BC < OB + OC < AB + AD + CD
CD < OC + OD < BC + AB + AD

Sumando miembro a miembro :


AB + BC + CD + AD < 2( OA+OB+OC+OD ) < 3 ( AB + BC + CD + AD )

Con el d a to :
2p < 2 ( OA + OB + OC + OD ) < 3 ( 2 p )
De d o n d e :
p < OA + OB + OC + OD < 3 p I.q.q.d.

la figura adjunta, AC y BD se cortan en el punto O


AB + BC + CD + AD = 2p

Demostrar, q u e :
D
Solución
Del gráfico, se pueden e s c rib ir: AC < AB + BC
AC < AD + CD (+)

Luego: 2AC < AB + BC + CD + AD

AB+BC+CD+AD
AC <
Análogamente :
B D < AB + BC + CD + AD
2

Es d e c ir:
AC + BD < AB + BC + CD + AD

o con el dato : AC + BD < 2 p (I).

Por otro lado


AB < OA + OB
BC < OB + OC
(+)
CD < OC + OD
AD < OA + OD
Entonces :
AB + BC + CD + AD < 2 ( OA + OC + OB + OD )

2p < 2 ( AC + BD )

De donde : p < AC + B D ( I I ).

Finalmente, de ( I ) y ( I I ):
p < AC + BD < 2 p I.q.q.d

v*
. .. •'*Xv.».. •*.

* E N T E R A T E -^ p S .
Hace tal vez, cerca de seis mil años, que en Mesopotamia tuvo lugar uno de los
a co n te cim ie n to s m ás grandes que registra la h isto ria : La invención de la rueda.

Asimismo, descubrieron la relación constante entre la longitud de una circunferencia


y su diámetro^?t), asignándole el valor de 3. Este número lo hallaron trazando en una
misma circunferencia los cuadrados inscrito y circurfscrito, creyendo que
de la circunferencia era igual a ta sejjtisum a de báperfm etros de di

Extraído de : " Historia de las Matemáticas


Autor: E.T. Bell.
Fondo de Cultura Económica. México

83
CAPITULO 4

ANGULOS

Angulo, es la figura formada por dos rayos que tienen el mismo origen. Los dos rayos son
los lados del ángulo y el origen común es el vértice.

Para el gráfico adjunto,


AB y AC son los lados y
A es el vértice del ángulo.

N otación :

BÁC ó CÁB, Z B A C ó Z C A B , ( la letra del vértice al centro ).

o también : Z A , Á, etc.

Si los lados de un ángulo son dos rayos opuestos,


el ángulo se llama rectilíneo o llano :

MEDIDA DE UN ANGULO.- La medida de un ángulo se refiere a la abertura entre sus lados.


En este curso usaremos el Sistema Sexagesimal, cuya unidad es el grado sexagesimal,
expresado, a s í : 1®. Este ángulo unidad está contenido 180 veces en un ángulo llano.

Las unidades inferiores a 1o. son el minuto y el segundo sexagesimal cuyas equivalencias
y notaciones s o n :

1 o = 60' y 1 ‘ = 60“

Por ejemplo 40°10*32“, representa la medida de un ángulo de 40 grados 10 minutos 32


segundos. Por otro lado, son equivalentes las cantidades : 20,5° y 20° 30*.

La siguiente figura, muestra el uso de un transportador para medir ángulos.

85
z AOB = 30° ZAOC = 55°

o0
co
z AOD = Z AOE = 150°

o
00
O
AOF = ( ángulo lla n o )

o
ro
z

OI
BOC = 55° - 30°

ii

o
z BOD = 90° - 30°

O
O
II

O
C\l
O
z BOE = 150° - 30c

z BOF = f 80° - 30°1 = 150°1 F

Completar

Z COD = Z COF = Z DOF =

z COE = Z DOE = Z EOF =

A n gulos C ongruentes.- Dos ángulos son congruentes si tienen igual medida

Si el ángulo ABC es congruente


con el ángulo EOF, escribiremos :

ABC = EOF
O por comodidad : ABC = EOF ,si nos referimos a las medidas.

B isectriz de un A n g u l o L a bisectriz de un ángulo es el rayo que lo divide en dos ángulos


congruentes.

Para la figura adjunta, diremos que


OM biseca el ángulo AOB :

AOM = MOB

AOB
AOM = MOB =

CLASIFICACION DE LOS ANGULOS

Se clasifican los ángulos de acuerdo a su medida o a su posición con relación a otros

I) Por su m edida Pueden s e r : nulo, llano o rectilíneo, recto, agudo, obtuso, convexo,
cóncavo y de una vuelta.

1. Angulo nulo.- Mide 0o. Sus lados son dos rayos coincidentes.

B
BAC = 0o

88
Angulo llano o rectilíneo.- Mide 180°. Sus lados son dos rayos opuestos

A 180° B
<4- r\
o

Angulo recto.- Mide la mitad de un ángulo llano : 90

Decimos que BA y BC son


perpendiculares y escribimos : BA _L BC
B
Angulo agudo.- Todo aquel que mide más de 0o y mencs de 90°

0o < a < 90°

Angulo obtuso.- Mide más de 90° y menos de 180°.

90° < b < 180

Angulo convexo.- Cuya medida está comprendida entre 0o y

Angulo cóncavo.- Si mide más de 180° y menos de 360°

180° < c < 360

Angulo de una vuelta.- Se genera al girar


360e
un rayo, una vuelta completa alrededor de
su origen. Mide 360° o
Por su p o sició n .- Se clasifican en : consecutivos, adyacentes y opuestos por el vértice.

Angulos Consecutivos.- Dos ángulos son consecutivos si tienen el mismo vértice, un lado
común y los otros lados en regiones distintas del común.

Tres o más ángulos son consecutivos, si cada uno es consecutivo con su inmediato.
A
B

Dos ángulos Tres ángulos


consecutivos. consecutivos.
87
2. Angulos adyacentes.- Denominado también par lineal , son dos ángulos consecutivos,
cuyas medidas suman 180°.

AOB + BOC = 180°

3. Angulos opuestos por el vértice.- Son dos ángulos, cuyos lados forman dos pares de rayos
opuestos.

E je m .: á y b

Teorema.- Los ángulos opuestos por el vértice son congruentes.

D em ostración

á y 1 forman un par lineal, al igual que b y í

Luego :

á + 1 = 180' Restando miembro a miembro :


A A

b + 1 = 180 á - b = 0o
De donde :
á = b

Teorema.- Las bisectrices de dos ángulos adyacentes son perpendiculares


A A

Sean AOB y BOC los ángulos adyacentes.

OM^ biseca al AÓB y ON , biseca el BÓC.

Como:
AOB + BOC = 180 o

E ntonces:
2 0 + 2 a = 180
o

0 + a = 90°

Esto, es:
MON = 90° OM y ON son perpendiculares.

88
Angulos complementarios.- Dos ángulos se llaman complementarios si sus medidas suman

Si m y ñ son com plem entarios:

m + ñ = 90°

Se dice que uno es complemento del otro

Angulos suplementarios.- Dos ángulos se llaman suplementarios, si sus medidas suman

O bservaciones :
(1-) Todo ángulo agudo tiene complemento y suplemento.

(29) Los ángulos obtusos tienen sólo suplemento.

(39) Si a9 es la medida de un ángulo agudo, entonces las medidas de su complemento y


suplemento, son respectivamente : ( 9 0 ° - a ° ) y ( 1 8 0 ° - a ° ) .

(49) Los complementos de dos ángulos congruentes, son congruentes.

(59) Los suplementos de dos ángulos congruentes, son congruentes.

(6 9) Dos ángulos que tienen el mismo complemento ( o el mismo suplemento ), soncon-


gruentes.

POSICIONES RELATIVAS DE DOS RECTAS

En un mismo plano, dos rectas distintas, pueden ser secantes o paralelas.

(a) Secantes.- Si se intersecan. A su vez, se dividen, en:


O blicuas.- Si no determinan ángulo recto :

( _L se lee : “es oblicuo con").

P erpendiculares.- Si determinan ángulo recto :


*m

m in n
t

( l , se lee: “es perpendícualr c o n ").


89
(b) Paralelas.- Si no se intersecan.

*-1 11 I-2

( 11 ó / / , se lee : “es paralelo con”) \-+ L 2

ANGULOS DETERMINADOS SOBRE DOS PARALELAS Y UNA SECANTE

s i V . n r , , determinan los siguientes ángulos :

(I) A n g u lo s A lternos, que pueden s e r : * -----------------

(a) Alternos internos: 3= 5 y 4 = 6

* -------------------------------------

(b) Alternos externos: 1= 7 y 2 = 8 8

(II) A n g u lo s C orrespondientes : í = 5 ; 2 = 6 ; 3 = 7 y 4 = 8

(III) A n g u lo s C onjugados, que pueden ser :

(a) Conjugados internos : 3 + 6 = 180°.

y 4 + 5 = 180o -

(b) Conjugados externos : í + ¿ = 180o -

y 2 + 7 = 180o -

Nota.- En la figura siguiente, las rectas L t y L2 no son paralelas.

Se definen los pares de ángulos con los


mismos nombres, pero no se cumplen
las propiedades. Por ejemplo :

A *

4 y 6 son alternos internos, pero no son

ANGULOS DE LADOS PARALELOS

Estos pueden ser congruentes o suplementarios.

A) C ongruentes.- Si ambos son agudos o ambos obtusos.

B) S uplem entarios.- Si uno es agudo y el otro obtuso, ( sus medidas sumarán 180°).
Ejemplos

A) C ongruentes

— m=n

x=y=z

B) S uplem entarios

ANGULOS DE LADOS PERPENDICULARES

Estos ángulos resultan a) congruentes si ambos son agudos o los dos obtusos
b) suplementarios, si uno es agudo y el otro obtuso. Ejemplos:

r=s

PROBLEMAS RESUELTOS

Se tienen los ángulos consecutivos AÓB, BÓC y CÓD; siendo :

AÓC = 47°, BÓD = 51° y AÓD = 80°. Hallar la medida del BÓC.

A) 18o B) 28° C) 38° D) 16o E) 12o


Solución

Incógnita : BÓC

Del gráfico, observamos que

CÓD = 8 0 ° -4 7 ° CÓD = 33°

BÓC = 5 1 °-3 3 ° BÓC = 18°

Rpta: ( A ).

Se tienen los ángulos consecutivos AOB, BOC y COD, siendo 2( AÓB ) = 3( CÓD

AOC = 92° y BOD = 76°. Hallar la medida del BÓC.

A) 24° B) 16' C) 54' D) 44' E) 64'

Solución

x = ?

Por dato :

2 ( A 0 B ) = 3 ( c ÓD)

Con el gráfico : 2 ( 92° - x ) = 3 ( 76° - x )

Efectuando:
184° - 2x = 2 2 8 ° - 3 x
x = 44°
Rpta: ( D ).

Sean los ángulos consecutivos AÓB, BÓC, CÓD y DÓE; OB biseca AÓC; -
OC*. biseca AÓD y OD, biseca AÓE.

S i : 2( A Ó B ) + 3 ( B Ó C ) + 4 ( C Ó d ) + AÓE = 210°

Hallar la medida del AÓB.

A) 10° B) 21 C) 42 D) 5 E) 16

Solución

Incógnita : AÓB = x.
B tC
Dadas las bisectrices:
AÓB = BÓC = x

AÓC = CÓD = 2x y AÓD = DÓE = 4x


Reemplazando en el dato :

2( AOB ) + 3( BÓC ) + 4 ( CÓD ) + AÓE = 210


2 (x) + 3 (x) + 4 (2x) + 8 x = 210°
.*. 2 1 x = 2 1 0 °
x = 10'

Rpta: ( A ).

4) Se tienen los ángulos consecutivos AOB y BOC, donde AÓC = 102°. Se traza la bisectriz
OM del AÓB. Hallar la medida del BÓC, si B Ó C -M Ó B = 36°.

A) 51 B) 6 6 ' C) 6 8 ' D) 48' E) 58

Solución

Incógnita : BÓC = x

Consideremos el gráfico adjunto,


Podemos p la n te a r:
2 a + x = 1 0 2 ° .......................................... ( 1 )

Además, según enunciado :

B O C -M O B = 36
x - a = 36° a = x - 36 (2 )

Reemplazando ( 2 ), en ( 1 ), para hallar x :

2 ( x-36° ) + x = 102° x = 58'

Rpta: ( E ).

5) AÓB, BÓC, CÓD, DÓE y EOF , son consecutivos y AÓF llano. QB . biseca AÓC, OE

biseca DÓF y BÓE mide 112°.

Hallar la medida de CÓD .

A) 44' B) 54' Q64' D) 6 8 ' E) 34'

Solución
Sea el gráfico adjunto
A

Incógnita : COD = x.

T enem os:
a + 112° + ó 180°
a + <t> 68 °

93
Pe r o :
a +<|> + x = 112° 6 8 ° + x = 112°/ . x = 44'

Rpta: ( A ).

Se divide un ángulo convexo, por un rayo, en dos, cuyas medidas son entre sí como 2 a 3
Si el mayor de los ángulos parciales mide 32°, ¿Cuánto mide el ángulo total?

A) 52°30' B)53°30' C) 53°10' D) 53°01’ E) 53°20‘

Solución
A.

Considerando AOC el ángulo y OB el rayo


tal que las medidas de AÓB y BÓC sean
entre si, como 2 a 3, podemos e le g ir:

AÓB = 2 x
BOC = 3 x

Por dato : BOC = 32'


3x = 32°

32
x =

32° 160
Se pide: AÓC = 5x = 5. = 5 3 °2 0 '

Rpta: ( E ).

Se tienen los ángulos consecutivos AÓB y BÓC. Se traza OD bisectriz de A Ó B . Hallar


la medida del ángulo CÓD ; si AÓC + BÓC = 160.

A) 60° B) 40° C) 80° D) 100° E) 120°

Solución

Sea el gráfico indicado:


Incógnita : CÓD = a + ó
Por d a to : ,

A Ó C + BÓC = 160°

2 a + ó + 0 = 160° -» 2 a + 2 <J>= 160°


a + ó = 80°
o
Luego : CÓD = 80°

Rpta: ( C ).
8) En la figura adjunta :
x - y = 12°

Hallar el valor de a.

A) 6 o B) 24° C) 18° D) 12 E) 9'

Solución

Del gráfico, observamos que : x = 180° - a


y = 180° - 2 a

Reemplazando en el dato : x - y = 12°

Tenemos : (180° - a) - (180° - 2a) = 12°

Efectuando queda : a = 12°.

Rpt a: ( D ) .

A A

9) Se tienen los ángulos consecutivos AOB, BOC y COD. OM y O N , bisecan AOB y


*

respectivamente. Hallar la medida del MON, s i :


AÓC = 140° y BÓD = 80°

A) 90° B) 130° C) 120° D) 105° E) 110o

Solución
Consideremos el gráfico adjunto,
a

donde la incógnita, es : MON = a + [3 + ó-

Los d a to s :

AÓC = 140° —> 2 a + (3 = 140°


BÓD = 80° -> P + 2<D = 80°

Sumando miembro a miembro :


2 a + 2(3 + 2(0 = 2 2 0 °

a + (3 + <0 = 1 1 0 °

Rpta: MÓN = 110° ... ( E ).

10) Los ángulos consecutivos AOB y BOC, son tales que : AÓB = BÓC + 34°

Se traza ÓM . bisectriz del AÓC . Hallar la medida del ángulo MOB.

A) 17° B) 34° C) 68° D) 12° E) 22°

95
Solución
Sea el gráfico adjunto
A

y MON = x, la incógnita.

Haciendo BÓC = a, se obtienen :

AÓM = MÓC = x + a

Reemplazando, en el dato : o

AÓB = BÓC + 34°


x + a+ x = a + 34°
2x = 34° -» x = 17

Rpta: ( A ).

Al rededor del punto O, en forma consecutiva, se trazan los rayos Ó A , ÓB, ÓC, ÓD y ÓE,
de modo que : OC 1 OD ; OE y OB son rayos opuestos y OE biseca el AÓD .
r \

Hallar la medida del AÓB, si BÓC = — ( AÓB )


11 v '

A) 90° B) 110o C) 100° D) 120° E) 130°

Solución

Consideremos el g rá fic o :

Sea x, la medida del ángulo AOB. Entonces :

BÓC = — x
11
Como:

AÓE = 1 8 0 °-x ; EÓD = 1 8 0 ° - 9 0 ° - — x


11
y EÓD = AÓE, Entonces :

1 8 0 ° - 9 0 ° - — x = 1 8 0 °-x
11
Efectuando :

x - — x = 90°
11
De donde : x = 110°

Rpta: ( B ).

Hallar la medida de un ángulo, sabiendo que su complemento y suplemento suman 208°.

A) 62° B) 31° C) 29° D) 39° E) N.A


Solución

Sea x, la medida del ángulo pedido.


Entonces, según enunciado:
( 90° - x ) + (1 8 0 ° - x ) = 208°
2 7 0 ° - 2x = 208°

De donde : 2x = 62° -» x = 31°

Rpta: ( B ).

13) Las medidas de dos ángulos suplementarios son entre sí, como 3 a 7. Hallar el complemento
del menor.

A) 54° B) 32° C) 52° D) 36° E) N.A

Solución

* Sea x, la medida del menor.


El suplemento medirá : 180° - x

* Según enunciado :
x 3
180° - X _ 7 <;4
R esolviendo: x = 54°
y, el complemento : 90° - 54° = 36°

Rpta: ( D ).

14) El doble del complemento de un ángulo, más el triple del suplemento del mismo, es 500°.
Hallar la medida del ángulo.

A) 48° B) 22° C) 54° D) 24° E) 44°

Solución
- Sea x, medida del ángulo en mención. Según enunciado, planteamos :
2 ( 90° - x ) + 3 ( 180o- x ) = 500°

Efectuando :

180° - 2x + 540° - 3x = 500°


720° - 5 x = 500°
5x = 220° -> x = 44°

Rpta: (E).

15) El doble de la medida de un ángulo es igual al triple de la medida de su complemento.


Hallar la medida del ángulo.

A) 54° B) 36° C) 32° D) 27° E) 58°

97
Solución
Sea x la medida del ángulo. Entonces, del enunciado, planteamos
2x = 3 ( 90° - x )
Efectuando :
2x = 270° - 3x
5x = 270° -> x = 54°
Rpta: ( A ).

"6) H allar" B " s i : L , L2

A) 65°
B) 75°
C) 85°
D) 55°
E) N.A

Solución

En L, : P+ 5 0 ° + 5° + 2m = 180° b + 2m = 1 2 5 °........... (1 ).

Por ser correspondientes : 5o + 2m = m + 30°


m = 25°
Reemplazando m, en ( 1 ) :
P + 2 x 2 5 ° = 125° P = 75*
Rpta: ( B ).

17) En la región interior al ángulo AOB, se toma el punto P y se trazan PE, PF, PM,

perpendiculares a OA, OB y a la bisectriz, respectivamente. Hallar la medida del AÓB,

si 5(MPF) = 2(EPF)

A) 18° B) 72' C) 36 D) 54' E) 80'


Solución

Sea el gráfico :
- Incógnita: AÓB = 2x

- Por la teoría de ángulos de lados perpendiculares :


MPF = MÓF -» MPF = x
Y EPF = 180°-E O F EPF = 1 8 0 ° - 2 x

- Reemplazando en el dato : 5( M P F ) = 2 ( E P F ) -> 5(x) = 2 ( 1 8 0 ° - 2 x )


5x = 360° - 4x

98
De donde:
x = 40°

Luego ; AÓB = 80°

Rpta: ( E ) .

18) Si los 3/2 del complemento de un ángulo a es igual al suplemento del complemento del
'mismo ángulo.
Hallar ol

A) 15o B) 28° C) 18o D) 5o E) 8 o

Solución

Según enunciado planteamos la ecuación :


3
- ( 9 0 ° -a ) = 1 8 0 ° -(9 0 ° -a )

3
135° - - a = 90° + a

3
Luego : 135° - 90° = — a + a

45° = — a = 18°
2

Rpta : ( C ).

19) El suplemento del complemento de un ángulo es igual a 3/2 de la diferencia entre el


suplemento y el complemento de dicho ángulo.
Hallar el ángulo.

A) 38° B) 42° C) 45° D) 48° ' E) 50°

Solución

Sea x, medida del ángulo en mención. Según enunciado :

180° - ( 90° - x ) = - [ ( 180° - x ) - ( 90° - x ) ]

Efectuando:

90° + x = — [ 180o - x - 90° + x ]

3
90° + x = - [ 9 0 o] 90° + x = 135'

/. x = 45°

Rpta: ( C ).

99
20) Dada la recta p q y un punto O sobre ella, a un mismo lado se trazan los rayos O A y ÓB,

tal que OA sea interior al ángulo POB y AOP = 54°.

Hallar la medida de AÓB si QÓB es el suplemento del triple de B Ó A .

A) 27° B)36° C) 24° D) 30° E) 32°

Solución

Según enunciado :

POA + AOB + BOQ = 180°


54° + x + ( 180° - 3x ) = 180° x = 27°

Rpta: (A).

21) Hallar la medida de un ángulo, tal que el triple de su complemento sea igual al suplemento
de su mitad.

A) 36° B) 18o C) 54° D) 24° E) 48°

Solución
Sea x, la medida del ángulo pedido. Del enunciado, planteamos la ecuación :

3 ( 9 0 ° - x) = 180° - |

270o - 3x = 180° - “

5x
90° = — -» x = 36°

Rpta: ( A ).

22) La suma de las medidas de dos ángulos es 80° y el complemento de la medida del primero
es igual al doble de la medida del segundo. Calcular la diferencia de dichos ángulos.

A) 50° B) 60° C) 65° D) 70° E) 72°

Solución
* Sean x e y las medidas de los ángulos en mención.

Por dato: x + y = 8 0 ° ................(1)

También : 90° - x = 2y => x + 2y = 90° -» x + y + y = 90°

Con ( 1 ) : 80° + y = 90'

y =10°

100
Luego : x = 70°
Se pide :
x - y = 70o - 10° = 60'
Rpta: ( B ).

A A

AB y C E , son dos rectas secantes en el punto O, donde AOC < COB.

OF es un rayo interior al ángulo COB que forma un ángulo de medida 126° con la bisectriz

ÓM del BÓE ; siendo además OC, bisectriz del AÓ F.

Hallar la medida del ángulo FOB.

A) 108 B) 98 C) 106' D) 116° E) 118'

Solución
Según enunciado, hacemos el gráfico adjunto
A

Incógnita : FOB = x.

Sea : EÓM = a
MÓB = a
y AÓC = E Ó B ( opuestos por el vértice ).

AOC= 2a COF = 2a

Del gráfico : COF + FOM 4 MOE = 180°


2 a + 126° + a = 180°

a = 18°
Finalmente
x 126o - a
x 126o - 18°
x 108°
Rpta: (A)

Solución
A A

Primero hallaremos la medida de AOB y luego BOC

Sabemos, que : AÓD + AÓB + BÓD = 360°

Luego : a ÓD + AÓB + 80° = 360°

101
De donde : AÓD + AÓB = 280° .............. (1 ).

* Además, por dato : AÓD - AÓB = 12° .......( 2 ).

* Restando miembro a miembro las expresiones (1 ) y ( 2 )

AÓB - ( - AÓB ) = 280° - 12°

2 AÓB = 268°

AÓB = 134°

* Finalmente BOC = 180°-A O B

BÓC = 180° - 134°


BÓC = 46°
Rpta: ( D )

25) Dado el A ABC,


demostrar q u e :

Solución

Por B, se traza r 11 AC . Entonces, por ser,


respectivamente, alternos in te rn o s:

á = Á y
A A

♦ = c

Luego, c o m o : á + P + $ = 180'

Reemplazando : A + B + C = 180° l.q.q.d.

26) En la figura: L^IIl 4


Demostrar, q u e :


Solución
4
Trazando L L1 L2 ’ _____________________ X

a = a l2
Alternos internos
p = b
• • x = a +p

109
Reempla za n d o :

x = a + b .......... I.q.q.d

Nota.- En general, si L1 || L2:

Z a Z a
i i
la derecha la izquierda

a+b+c=x+y

27) Demostrar que, las bisectrices de dos ángulos conjugados internos, entre rectas paralelas,
son perpendiculares entre sí.

Solución
En efecto, sea AB una recta secante
a las paralelas y L2 .

Como : Á + B = 180° ( conjugados ).

2 ot + 2 p = 180°

Es d e c ir: a + p = 90°.

Pero, para la línea quebrada


x a+ p
x 90° AP 1 BP

28) H a lla r“0", si p ||tf:

A ) 130°
B ) 140°
C ) 120°
D) 100°
E ) 110°
Solución

Primero hallamos el valor de “a " ,


usando la propiedad :

100° = a + ( 180°- 3a)

De donde : a = 40°

Luego : m = 9 0 °- a m = 50° y
9 = 130° 0 = 130°

Rpta: ( A ).

103
29) Siendo L1 . Calcular “ 0

A) 100c
B) 80°
C) 120c
D) 60°
E) N.A.

Solución

En el a:
a + p + 0 = 180°. •(1)

Usando la propiedad en la línea


quebrada entre paralelas :

2 a = 2 0 ° + 180o - 2 p.

De donde : a + p = 100°

Reemplazando, en (1): 6 = 80°

Rpta: ( B ).

30) En la figura, se ha prolongado el lado

AC del A ABC, para graficar el BCE


( Z e xte rio r).

Demostrar, con la teoría de ángulos


de lados paralelos, que:

Solución

Se traza CF 11 AB. Entonces ;


A

a = A ............( Z Correspondientes )

<J> = B ( Z Alternos internos )

Luego :

BCE = a + <¡>

• • BCE = Á + B ........... I.q.q.d.

104
31) En la figura:
OE 1 BC y OP 1 AD

Marcar lo co rre cto :

A) a + p + <(> = 90°
B )a + P + $ = 180(
C) a + P = $
D) a + <|> = P
E) p + é = a

Solución

- Prolongamos BC hasta F.

- Por tener sus lados respectivamente


perpendiculares : x = a

- En el ACFD :
x + <(> = P
a +0 = p

Rpta: ( D ).

32) En la figura : x, y, z, son medidas de los


ángulos exteriores del AABC. Dem ostrar,
con la teoría de ángulos de lados paralelos,
que :

Solución
Por un punto cualquiera, “O” trazamos :

0 f||Á C ; OF||CB y Ó G ¡ | b A.

Luego :

EOF = x , FÓG = y , GÓE = z , por tener sus


lados respectivamente paralelos.

Siendo :
EOF + FÓG + GÓE = 360°
x + y + z = 3 6 0 °.......... I.q.q.d.

33) La figura adjunta, se llama cuadrilátero no convexo


Demostrar, usando la teoría de ángulos de
lados paralelos, que :

105
Solución
Tracem os: DE || BA y DF || BC

Luego:
ADE = a
Alternos internos
FDC = <¡>
A

y EOF = p ( z s de lados paralelos )

- S ie n d o :

x = ADE + EDF + FDC

Reemplazando:
x = a +p+ó I.q.q.d

34) En la figura adjunta : Á + C = 124'

OF 1 AB y OE | BC

Hallar la medida del EOF.

A) 124° B) 112° C ) 136°


D) 144° E) 146°

Solución

En el A ABC:

B = 1 8 0 °-( Á + C )

B = 180o - 124°

B = 56°

Con la prolongación hecha : OPF = B

( Alternos internos entre las paralelas EP y BC )


/. OPF = 56

Finalmente, en el AOFP
A A

EOF = P + F

EOF = 56° + 90'

EOF = 146°

Rpta: (E )

106
35) En la figura: ||

a + b = 224°.

Hallar el valor de x.

A) 13° B) 16° C) 26°


D) 23° E) 16°

Solución

- Con la prolongación hecha :

É = F .................. ( Z correspondientes )
É= 1 0 ° + x.

- En el cuadrilátero no convexo :

(1 8 0 ° - a ) + 10 ° + x + 8 o + x = b
198° + 2 x = a + b

Usando el dato : 198° + 2x = 224°

De donde : x = 13o

Rpta: ( A ).

36) En la figura :

t ? II l 2

El valor de x, es:

A) 29° B) 39° 0 )5 8 °
D) 41° E) 32°

Solución

- Por propiedad, en el cuadrilátero no convexo

a + ó + x = 9 0 ° .......... ( t ).

Además, para la línea quebrada y

2a+2<t>= 3 2 °+ 90°
cc+<t> = 61°

Reemplazando esto último, en (1 ) : 61° + x = 90°

De donde : x = 29°

Rpta: ( A ).

107
37) En la figura :
AÓC es llano. BÓD mide 112°.
OE biseca AÓB. OF biseca C Ó D .

Hallar la medida del EOF.

A) 126° B) 136° C) 146°


D) 106° E) 116°
D
Solución

Del gráfico :

EOF = ot +p + <|>.................. ( Incógnita ).

Se observan :

AÓC -> 2a + p = 1 8 0 °............( 1 ).

BÓD -> P + 2Ó = 1 1 2 °.......... ( 2 ) .

Sumando miembro a miembro ( 1 ) y (2 ) :

2 a + 2 p + 2 <t> = 292°

a+p + ó = 146° -» EOF =146°


Rpta: ( C ).

38) Se tienen los ángulos consecutivos AÓF, FÓB, BÓR y RÓC .donde ÓF y ÓR bisecan

los ángulos AÓB y FÓC respectivamente. Si: AÓB + 2( BÓC ) = 132°. Hallar la medida

del ángulo FOR.

A) 6 6 ° B) 25° C) 105° D)33° E) 70°

Solución

Incógnita : FOR = x

Por dato : RÓC = FÓR

y AÓF = FÓB.

Según enunciado : AÓB + 2( BÓC ) = 132°


*A o
Con el gráfico : 2 a + 2 ( 2x - a ) = 132°

Efectuando: 2a + 4x - 2a = 132°
4

x = 33°
Rpta: ( D ).

108
39) Se tienen los ángulos consecutivos AÓB, BÓC y CÓD donde CÓD = 2 ( b Ó C ) .

Se traza Ó M , bisectriz del ángulo COD. HaHar la medida del AÓD, si AÓM mide 70° y

BOD _ 2( A Ó M ) + COD
BÓC " AOC

A) 70° B) 96' C) 84* D) 98° E) 82

Solución
A

Incógnita : AOD

Se tiene :

BÓD 2 ( A Ó M ) + CÓD
BOC AOC

3 a _ 2 ( r + 2 a) + 2a
r = 3a.
a r + a

Entonces, como : r + 2a = 70° 3a + 2a = 70' a = 14'

Finalmente AÓD = r + 3a = 3a + 3a —» AÓD = 6 a


AÓD = 6(14°)
AÓD = 84°
Rpta: (C)

40) AÓC es un ángulo de medida 153° y OB un rayo interior, tal que AÓB > BÓC. OM es
bisectriz del BÓC y OR biseca el ángulo AÓM ( OR es interior al AÓB ).
A

En la región interior al ángulo BOM, se traza O P , bisectriz del ROC.

Hallar la medida del BÓP, si MÓP mide 15'

A) 15' B) 16' C) 12' D) 10' E) 14'

Solución

* Dato : AOC = 153°

* Según enunciado, tenemos


el gráfico a d ju n to :
A

Incógnita : BOP = x

* Luego:
BOM = x + 15° = MOC

109
Enseguida

PÓC * 15o ♦ x + 15o POC * 30° + x

Entonces; como : ROP * POC ROP = 30° + x


Pero: BOP = x RÓB = 30°

En consecuencia ROM = 30° + x + 15° ROM s 46° + x

Por lo tanto, también : AÓR = 45° + x

Finalmente, por dato : AÓC = 153°

Reemplazando según gráfico : 105° + 3x = 153 x = 16°

Rpta: (B ).

41) AÓB y BÓC son consecutivos, AÓC Hano y AÓB > BÓC
Se trazan : Q X , Bisectriz de AÓB.
OY, Bisectriz de BÓ C.
Ó Z , Bisectriz de XÓ C .
Hallar la medida de AÓB, si ZÓY mide 39° .
A) 126° B) 78° C) 156° D) 146' E) 166°
S o l u c i ó n

* Incógnita : AÓB

* Sean : ZÓB = a y BOY = p.

Luego se determinan las medidas


de los otros ángulos : ot + p = 39°.
Se p id e:
AÓB = 4 ( a + p )
( obtenido del gráfico )

Entonces : AÓB = 4( 39° ) => AÓB = 156°


Rpta: ( C ).

42) En la figura, hallar la medida del C Ó D , si:


* ÓB, biseca AÓC.
* Ó E, biseca CÓF
* DÓF = AÓB

* 1 7 ( AOB ) = 2 8 ( EOF )

A) 11 B) 10' C) 9' D) 12' E) 14'

110
S o iu c é ó n

- S eax, la medida del C O D .

Sea también: DOE = a

Entonces:

CÓE = x + a = E Ó F y DÓF = x + 2a = BÓC = AÓB

- Como AOF = 180° 4x + 6a = 180°


2x + 3a = 9 0 ° .......... (1 ).
Además, por dato: 17( AÓB ) = 28( EOF ).

ó: 17(x + 2a) = 28(x + a) a = — * .......... ( 2 ) .

- Reemplazando ( 2 ) , en (1 ): 2x + 3 . — x = 90°
6
Resolviendo: x = 12°

Rpta: ( D ).

En la figura: L, || L2

Hallar el valor de x.

A) 54* B) 64' C) 44'

D) 94‘ E) 84'

Solución

* En el A PRO : x = a + n (1 ).
A

* Por ser altemos extemos : EOG = 126°


2 a = 126°
a = 63°
También :
APB = MÓP ( ^ correspondientes ).
3n = a
3n = 63° n = 21°
* Finalmente, reemplazando los valores de a y
n, en ( 1 ) :
x = 63° + 21°
x = 84

Rpta: (E)

111
44) Hallar el valor de x,
. «-► i«->
si L-| | L2

A) 10° B) 20° C) 15'


D) 18° E) 12°

Solución

Con las prolongaciones hechas; usando los


triángulos y por ser conjugados internos :

3x + 7x = 180°
x = 18°

Rpta: ( D ).

45) En la figura: % I ; V

Hallar el valor de x.

A) 132' B) 162° C) 154'

D) 164* SÍ 144°

Solución

- Primero hallaremos a.

- Con los prolongaciones hechas:


a = 2a + 9 0 ° .............(1 )
También:
3a = ( 180o- a ) + 90°
a + 3a = 2 7 0 ° ........ ( 2 )

Reemplazando ( 1 ), en ( 2 ) :

2a + 90° + 3a = 270' a = 36

Luego, en (1 ) : a = 72° + 90' a = 162

Finalmente, para hallar x, planteamos según el gráfico

x + ( 180° - a ) = a ( Zexterno en el A ).
x = 2 a - 180°
x = 2 ( 1 6 2 ° ) - 180' x = 144°

Rpta: (E).

112
46) En el gráfico a + 0 = 120°
C alcular: a

A) 10°
B) 15°
C) 20°
D) 25°
E) 30°

Solución

Con las prolongaciones hechas: * A ABC :


0 - 90° + 8 + 90° - a = 180°
2 0 - a = 180° ............... ( I ).

Por dato: a + 0 = 120° (N ).


( I ) + ( II ) : 30 = 300° 0 = 100°
a = 20°

Rpta: ( A ).

47) Calcular x, s i : L 1 11 U2

A) 90°
B) 100°
C) 110°
D ) 120°
E) 130°

Solución

Como: L1 11 L 2 ; Con 0 : L 3 11 L4

Propiedad : 2a + 5a = 40° + 30°

a = 10'

1 Lo : x = y+p (I).

Pero: 2y = 0 + 2a

8 + 2 a _ 6 + 20° _ 8
2 2 2

113
2p + 0 = 180' En ( I ) :
0 0 0
P = 90° - - x = - + 10° + 90°
2

x = 100°

Rpta: ( B ).

48) En el gráfico:y+ p = 260° y L-i 11 L2 . Calcular x.

A) 30°
B) 50°
C) 60°
D) 80°
E) 70°

Solución

Propiedad: y = 180° - y + 180° - p


y = 360°-(y + P)
y = 100°

Propiedad: ( Z de lados perpendiculares )


z + y = 180° z = 80°

Pero: 30 + a + 3a + 0 = z
4 a + 40 = 80°
a + 0 = 20°

Propiedad : x = 3a + 30 x = 60'

Rpta: (C).

49) La diferencia entre la suma de suplementos y la suma de complementos de dos ángulos que
se diferencian en 2 0 °, es igual al doble de la suma de dichos ángulos.

Hallar la medida del mayor ángulo.

A) 40° B) 55( C) 60° D) 75( E) 80'

Solución

* Sea x, la medida del mayor ángulo.. Luego, el menor, mide: ( x - 20° ).

* Según enunciado :
[ ( 180°-x + 180° - ( x - 20° ) ] - [ 90° - x + 90° - ( x - 20° ) ] = 2 [ x + ( x - 20° ) ]

suma de suplementos suma de complementos suma de los Z s

114
Efectuando:
[ 380° - 2x ] - [ 200° - 2x ] = 2 [ 2x - 20° ]
180° = 4x - 40°

De donde: x = 55'

Rpta: ( B ).

Si a la medida de un ángulo se le resta dos grados más que la tercera parte de su


complemento, resulta un cuarto del suplemento del ángulo, disminuido en un grado. ¿Cuánto
mide dicho ángulo?

A) 45° B) 46 C) 44' D) 48' E) 38'

Solución

Sea “x, la medida del ángulo.

Según enunciado, planteamos la siguiente ecuación

90° - x
x - + 2 = —[ 180° - x 1 - 1
4 J

Efectuando:

x - 30° - ± + 2o
3

1^-32° = 4 4 *-*
3 4

Resolviendo : x = 48°, es la medida del ángulo.

Rpta: ( D ).

Alrededor de un punto O, en sentido horario, en forma consecutiva se trazan los rayos


O A , O B , O C y OD siendo OA 1 OB y OC _L OD . Hallar la medida del ángulo que

forman las bisectrices de AÓC y BÓD.

A) 135' B) 45' C) 120’ D) 150' E) 90'

Solución

Sea el gráfico adjunto, donde:


O P , bisectriz de AÓC

ÓR, bisectriz de BÓD

Incógnita: POR
a

Observamos, que : POR = a + p + 0

115
Como : a ÓB = 90° —> 2a + p = 90 sumando miembro a miembro
y CÓD = 90° -> 20 + p = 90' 2 a + 2 p + 2 0 = 180°

• • a + p + 0 = 90’

Entonces : POR = 90

Rpta: ( E ).

52) AÓB y BÓC son consecutivos ( AÓB > 2 B Ó C ). AÓC mide 150°. OM y OR, bisecan

AÓB y MÓC, respectivamente. Hallar la medida de RÓB, si MOR mide 43°


A) 32° B) 11 C) 33 ,0 ) 2 1 E) 17'
Solución
* Dato: AÓC = 150'
* Incógnita: RÓB = x

* Como AÓB > 2 BÓC

AOB
> BOC

Luego:
MÓB > BÓC ; lo cual indica que ÓR, bisectriz de MÓC, está entre OM y OB

* Usando el gráfico: AÓC = 150°


AÓM + MOR + RÓC = 150°
( 43° + x ) + 43° + 43° = 150°

De donde : x = 21

Rpta: ( D ).

53) Dados los ángulos consecutivos AÓB y BÓC, tal que: AÓB - BÓC = 100°.
OM, biseca AÓB, ON, biseca BÓC.
OR, biseca MÓN. Hallar la medida del RÓB
A) 20° B) 25 C) 30 D)35 E)40°
Solución
Sean: ROB = x
BÓN = a

Luego: NÓC = a¡

MOR = RÓN -> MÓR = x + a

AOM = MOB AÓM = 2x + a

116
En el dato: AÓB - BÓC = 100°.

Reemplazamos: (4x + 2 a ) - 2 a = 100°


4x = 100° -> x = 25°

Rpta: RÓB = 25° ... ( B ).

Se tienen los ángulos consecutivos AÓB, BÓC y CÓD, s ie n d o : AÓC - BÓD = 12°. Hallar

la medida del ángulo AÓC, sabiendo además que, el ángulo formado por las bisectrices
de AÓB y CÓD, mide 70°.

A) 82° B) 58° C) 41° D) 76° E) 38°

Solución
Consideremos el gráfico adjunto
y las variables indicadas. Incógnita:

AÓC = a + 70° - b
AÓC = 70° + a - b ( 1 ).

Por dato, además: AÓC - BÓD = 1 2 °

Con el gráfico : (a + 70° - b) - (70° - a + b) = 12°

Efectuando: 2a - 2b = 12°
a - b = 6o
A

Luego reemplazando esto, en (1 ) : AOC = 70° + 6 o


AÓC = 76°

Rpta: ( D ).

AÓB y BÓC, son ángulos consecutivos. AÓB mide 58° y BÓC > 58° . Se trazan

ÓM y ÓR , bisectrices de AÓC y BÓC, respectivamente.

Hallar la medida de! ángulo MOR.

A) 58° B) 32° C) 28° D) 29° E) N.A

Solución
Del g rá fico :
Incógnita MOR = x

Si BÓM = n, entonces :
BÓR = n + x = RÓC

117
* Además :
MOC = AOM
• ♦ 2x + n = 58° + n

De donde : x = 29

Rpta: ( D ).

56) Sean los ángulos consecutivos AÓB, BÓC y CÓD, de modo que: AÓB > BÓC > CÓD

Si : AÓB - CÓD = 28°, hallar la medida del ángulo formado por las bisectrices de
AOD y BÓC

A) 14° B) 28' C) 7' D) 56' E) N.A

Solución

* Dato: AOB - CÓD = 28

Sean:
OM, bisectriz de AOD
Ó N , bisectriz de BÓC

- Incógnita: MÓN = x

- S i : MÓB = a, entonces: NÓB = x + a • « NÓC = x + a

Supongamos, además, que : AÓM = r = C Ó D .

Luego, en el dato AOB - COD = 28°


Con el gráfico: ^ 15-
( r-a ) - ( r- ( x + x + a ) ) = 28

Efectuando: r -a - r + 2 x + a = 28'

2 x = 28

• • x = 14°

Rpta: ( A ).

57) Sean los ángulos consecutivos AOB, BOC, COD, DOE y EOF, donde : AÓF = 1 3 2 ° y
AÓD = BÓE = CÓF. Se traza ÓR, bisectriz del C Ó D .

Hallar la medida del BÓC, siendo además : EÓR = 38°.

A) 38° B) 48' C) 58' D) 28' E) 18‘

118
Solución

BOC = ?

Sean : AOD = BOE = COF = a.

y• COR = ROD = n.

Se o b serva:
BOC = BOE - COR - ROE
Es d e c ir:
BÓC = a - n - 3 8 ° ....................... (1 )

Además: AÓD + DÓF = 132°


ó: a + a - 2 n = 132° a - n = 6 6 °.... ( 2 ) .

Reemplazando ( 2 ), en ( 1 ) : BÓC = 6 6 ° - 38°


BÓC = 28°

Rpta: ( D ).

Las medidas de los ángulos consecutivos AOB, BOC y COD, suman 124° y el ángulo
BOC mide 32° ÓM y ÓN, bisecan AÓC y BÓD, respectivamente.
Hallar la medida del ángulo MON.

A) 92 B) 48' C) 46' D) 44' E) 36'

Solución

Datos: AOD = 124'


BÓC = 32°

Incógnita: MÓN

Sean:
AÓM = a = MOC y

BÓN = b = NÓD

Del gráfico, podemos o b se rva r:


MÓN = MÓC + CON MÓN = a + b - 32° (1) y
MÓN = AÓD - AÓM - NÓD -> MÓN = 124° - a - b .... ( 2 ).

Sumando las expresiones ( 1 ) y ( 2 ), miembro a miembro:


2( MÓN ) = 124°-32'
MON = 46°
Rpta: ( C ).
119
59) Se tienen los ángulos consecutivos AOB y BOC, siendo AÓC = 152°. Se trazan
Ó ?, OF, OM y OR, bisectrices de AÓB, BÓC, AÓF y EÓC, respectivamente.
Hallar la medida del ángulo MOR.

A) 76° B) 19o C) 34° D) 38° E) 52°

Solución

- Consideremos el gráfico adjunto


y las varibles indicadas.

- Incógnita: MOR = x

Del dato: AOC = 152°

2a + 2P + 2<(> + 2x = 152°
C*
De donde: a + p + 0 + x = 7 6 °...( 1 ).

- Para hallar a + p + 0 , planteamos,


a partir del gráfico, lo siguiente :

AOM = MOF -> 2a + p = x+0 ... (2).

y ROC = EOR p + x + 20 = a + p + x ... ( 3 ) .

- Sumando miembro a miembro, las expresiones ( 2 ) y ( 3 ) :

2 a + 2 p + 2 <t>+ x = <x+p + <(>+ 2 x

/. a + p + 0 = x

- Sustituyendo esto último, en (1): x + x = 76° -> x = 38°

Rpta: ( D ).

60) En la figura : HQB = 84°

QB 11 OD ,

Si EOF mide 72°. Hallar el valor de x.

A) 36° B) 16° C) 56°


D) 70° E) 66°

120
Solución
x = ?

EQD 2
Com o:
AQH 7

AQH = 7 ( EÓD )

Luego, s i : EÓD = a, entonces : AQH = 7a

Además, por tener sus lados respectivamente


paralelos : AQP = EÓD -> AQP = a

Siendo: DOF = EOF - EOD -> x = 7 2 ° -a ...(1 )


y : PQH + HQB = 180°
6 a + 84° = 180° -> a = 16°

Finalmente, reemplazando en (1 ) : x = 72° -16'

x = 66'

Rpta: (E).

En la fig u ra :

Hallar el valor de a :
1
A) 10 B) 8 ' C) 9'

D) 5o E) 6 (

Solución

Del g rá fic o : p = 180° - (7a + 8 a)


p = 180o - 15a

También: <f» = 4a + 2a
<(> = 6 a

Y,por último:
1 1 a = P + <|»

Reemplazando lo a n te rio r:
11a = ( 180o - 1 5 a ) + 6 a

De donde : a = 9o

Rpta: ( C ).
121
Hallar la medida del ángulo que forman :
<-> «->
i-i y l 2

A) 39' B) 32' C) 42'


D) 29 E) 38'

Con las prolongaciones hechas :


x + y = 1 8 0 °............( 1 ) .
( Z de lados perpendiculares )

Además, en el cuadrilátero no convexo


y = 42° + 40° + 59° A * ____________

y = 141°

Reemplazando, en (1 ) : x + 141° = 180° -» x = 39

Rpta: ( A ).

63) En la figura a d ju n ta ,^ 11 .
Hallar el valor de x.

A) 69' B) 59' C) 49'

D) 79' E) 89‘

S olución

Con la prolongación hecha, tenemos

24° + x = a + r .................... ( 1 )-
Por la propiedad :

También, en :

m + 2r = 72° -> m = 72° - 2 r ...( 2 ).

Y, en el cuadrilátero no co n ve xo : 2a = 90° + 24° + m


Con lo de ( 2 ): 2a = 114°+ 72o - 2 r
a + r = 9 3 ° ............
.......................( 3 )
Finalmente, reemplazando ( 3 ) en (1 ):
24° + x = 93° —» x = 69°

Rpta: ( A ).

122
64) En la figura, L1 L2

Hallar el valor de x.

A) 30° B) 36° C) 45°

D) 18° E) 15°

Solución

x = ?

Por propiedad de la línea quebrada entre rectas paralelas : a + b = 2x


( 1 )•
y m + r = 3x
- Además, por ser conjugados internos:

2a + 2r = 180° -> a + r = 90'


(2 )
y 2b + 2m = 180° -> b + m = 90'

Sumando miembro a miembro, las expresiones


encontradas en ( 2 ): a + b + m + r = 180°

Con las de ( 1 ): 2x + 3x = 180°

x = 36°

Rpta: ( B ).

65) En la figura: L-j 11 L 2

a + b = 224'

Hallar el valor de x.

A) 134' B) 112° C) 148'

D) 168' E) 158°

* x = ? ( Incógnita )
(180* - a)
* a + b = 224° ( dato ) (180* - 2 a )

- Por la propiedad de la línea quebrada


entre paralelas : x = a +p .......... ( 1 )

- También :
r ( 180° - a ) + ( 180° - b )
r 360° - ( a + b ). Con el dato
r 360° - 224° -> r = 136°

123
• Además, por tener sus lados respectivamente perpendiculares :
q + r = 180° -> q + 136° = 180° /. q = 44°

- De otro lado : q = (1 8 0 ° - 2a ) + (1 8 0 ° - 20 ).

44° = 360° - 2 ( a + 0 ).
C o n (1 ):
44° = 360o - 2x . De donde: x = 158°

Rpta: ( E ).

66) Si al número que expresa, en grados, la medida de un ángulo se le suma el cuadrado del
número que expresa su complemento, resulta el número que representa la medida de un
( ángulo llano. ¿Cuántos grados mide dicho ángulo?

A) 99° B) 90° C) 79° D) 80° E) 72°

Solución

Sea x° la medida del ángulo. Luego, según enunciado :

x + ( 90 - x )2 = 180°

Efectuando:
x2 - 179x + 7920 = 0

Factorizando:
(x - 80) (x - 99) = 0

De donde:
x = 80 ó x = 99

Pero:" x " no debe ser mayor que 90, de lo contrario no tendría complemento.

Por lo tanto : x = 80.

Rpta: ( D ).

67) Del gráfico, hallar el valor de “y” cuando “xMtoma su mínimo valor entero.

A) 88°

B) 68°

C) 78°

D) 98°

E) 58°

124
Solución

* Se debe cumplir: y > x , y < 2x

Luego: x < y < 2x (1 ).


Además:
( 2 x - y ) + ( x + y ) + ( y - x ) = 180°
2x + y = 180°
y = 180° - 2x

En (1 ): x < 180°- 2 x < 2x

180° - 2x < 2x

x > 45'

El mínimo valor entero de x : 46°.


El valor correspondiente para y, será : 180° - 2x = 88
Rpta: ( A ).

68) Dados los ángulos consecutivos AÓB, BÓC, CÓD y DÓE donde

OB Bisectriz del AÓD

OC Bisectriz del BOE

COD DOE A

y AOB es agudo

A __

¿Cuál es el mayor valor entero de la medida del COE ?

A) 59° B) 65' C) 64' D) 60* E) 73°

Solución

COD DOE
- Com o:

COD DOE
Haciendo: = x

Tenemos : CÓD = 2 x y DÓE = 3x


( colocamos esto en el gráfico ).
A

- Se indica que AOB es agudo. Entonces : 7x < 90°


90°
x < -z - (1 )•

125
- Nos piden ei mayor valor entero de la medida del CÓE;

Siendo : CÓE = 5x.

- Multiplicando la expresión (1), por 5 en ambos miembros :


90° 450°
5x < 5 ( —— ) -> 5x < —-

450°
COE < — — -» COE < 64° 17*

Entonces, el mayor valor entero de la medida del CÓE es: 64°

Rpta: ( C ).

126
CAPITULO 5
v

TRIANGULOS
Definición.- Se llama triángulo a la figura formada por la reunión de los segmentos
determinados al unir tres puntos no colineales.
B
* Notación : a ABC = AB U AC U BC
* Perímetro : 2p = AB + BC + AC
* Semiperímetro : p = ( AB + BC + AC ) / 2

Clasificación de los triángulos.- Se clasifican por sus lados y por sus ángulos:

I) Por sus lados.- Se clasifican en :

a) Escaleno.- No tiene lados congruentes.


b) Isósceles.- Tiene dos lados congruentes.El tercero es llamado base. Los ángulos en la base
son congruentes.
c) Equilátero.- Tiene sus tres lados congruentes. Cada ángulo interior mide 60°.

Por sus ángulos- Se clasifican en :

a) Triángulo rectángulo.- Tiene un ángulo recto. Los lados que determinan dicho ángulo se
llaman catetos y el tercero es la hipotenusa.
b) Triángulos oblicuángulos.- No tiene ángulo recto. Pueden s e r :

b,) Acutángulo.- Si sus ángulos interiores son agudos..


b2) Obtusángulo.- Tiene un ángulo interior obtuso.

127
Propiedades B ásicas:

En todo triángulo :

1) Las medidas de los ángulos interiores, suman 160°.

a + P + é = 180

2) Cualquier ángulo exterior mide igual que la suma de dos interiores no adyacentes.

3) Las medidas de los ángulos exteriores, uno por vértice, suman 360°.

x + y + z = 360

4) En un mismo triángulo, a mayor lado se opone mayor ángulo y viceversa

S i: B
AB > BC
entonces
C > A

5) En un mismo triángulo, a lados congruentes se oponen ángulos congruentes y viceversa

* S i: AB = AC,
A A

entonces : B = C

6) Cualquier lado es mayor que la diferencia de longitudes de ios otros dos y menor que su
suma.
* S i: c > b > a,
entonces : b > c - a
b<c+a

Líneas Notables del Triángulo.

1) Mediana.- Segmento que une un vértice con el punto medio del lado opuesto

Ejemplo : Mediana B M .

128
Mediatriz.- Recta perpendicular a un lado, levantada desde su punto medio

Ejemplo : mediatriz de AC B

Bisectriz interior.- Segmento que biseca un ángulo interior

Ejemplo: BD

Bisectriz exterior.- Segmento que biseca un ángulo exterior.

Ejemplo: BE

( AB > B C )

Altura.- Segmento perpendicular a un lado, trazado desde el vértice opuesto.

Ejemplos : BH
B B B

A"1 H
C — cH

Ceviana.- Se llama ceviana de un triángulo, al segmento que une un vértice con un punto
cualquier del lado opuesto o de su prolongación.

A ABC :

BQ, ejemplo de una ceviana interior.

BR, ejemplo de una ceviana exterior.

Caso del Triángulo rectángulo

Los ángulos agudos, son complementarios.


B

A + C = 90
A
La altura hacia lahipotenusa, divide al ángulo recto en dos ángulos congruentes a los agudos
del triángulo.

A ABC : BH es altura relativa a la hipotenusa


B

HBC = A

ABH = C
PROBLEMAS RESUELTOS
- NIVEL I

1) En un A ABC, BE es bisectriz interior. Hallar la medida de C, s i : AB = BE = EC

A) 30' B) 60' C) 32' D) 36' E) 15'

Solución

Incógnita: C = x

ABEC, isósceles : EBC = C = x

Luego : AÉB = C + EBC


( Z e xte rio r)

AEB = 2x

Además : ABE = EBC = x ( BE biseca el ABC )

A ABE, isósceles A = AEB = 2 x

E ntonces: Á + ABE + AÉB = 180°


2x + x + 2x = 180°.
x = 36°

C = 36°

Rpta: ( D ).

2) En la figura mostrada AB = BC
y el triángulo QSC es equilátero. Luego :

A) a = b B) 2a = b C )2 a = 3b
D) a = 2b E) a = b + 60'

Solución

Como : AB = BC
B
• • A = b + 60° = C

En el A A Q C :

a + 60° = b + 60° + b
( Z e xte rio r).

a = 2b

Rpta: ( D ).
130
3) La figura adjunta es un cuadrilátero no convexo ( cóncavo ). Usando alguna propiedad de
triángulos, demostrar que :

Solución
Prolongamos AD hasta su intersección con B C , en el
punto E. Usando el teorema del ángulo externo :

a DEC -+ x = é + C (1 ).

a ABE -+ é = Á + B .......... ( 2 ).

.*. Sustituyendo ( 2 ) en ( 1 ) :
x=A+B +C

4) En un a ABC, A B =12u y BC =18u. PorB, se traza paralela a A C , cortando a las bisectrices


de los ángulos extemos A y C, en los puntos P y Q, respectivamente. Hallar PQ.

A) 6u B) 24u C) 27u D) 30u E) N.A

S o lu c ió n

* Incógnita : PQ

* Sea el gráfico adjunto :

P = EÁP = ot
Q = FCQ = p
(ángulos alternos internos)

Luego :
* a APB, isósceles -+ PB = AB /. PB = 12
* a CBQ, isósceles -+ BQ =; BC BQ = 18

Entonces : PQ = PB + BQ = 1 2 + 1 8

PQ = 30

Rpta: ( D ).

5) En un a ABC, B = 90°, C = 22°, hallar la medida del ángulo formado por la bisectriz

del B y la mediatriz de AC.

A) 23' B) 22 C) 21 D) 20' E) 18
131
Solución
Sea P, el punto de corte de las líneas
indicadas . Incógnita : P = x

En el cuadrilátero no convexo sombreado

x + 45° + 22° = 90'

x = 23'

Rpta: (A ).

6) En un a ABC, AB = BC, se traza la ceviana interior BE. En el ABEC, se traza laceviana


EQ, tal que BE = BQ.

Si ABE mide 48°, hallar la medida del QÉC.

A) 48° B) 36' C) 24° D) 12 E) 28'

Solución

* Incógnita : QEC = x

* A ABC, isósceles :
A = C = a

* En el A EQC, por el Teorema del ángulo externo : EQB = QÉC + C

EQB = x + a

Luego, en el A EBQ, isósceles : BÉQ = EQB

* Finalmente, usando el Teorema del ángulo externo en el A ABE : BÉC = Á + ABE

2x + a = 48° + a 2x = 48°

x = 24'

Rpta: ( C ).

7) En un A ABC, recto en B, la altura BH corta a la bisectriz interior AD en el punto F


Demostrar que el A BFD es isósceles.

Solución

Consideremos la figura adjunta.


Sabemos, por propiedad para los
triángulos rectángulos, que:
A. A

ABH = C

132
Así, usando eí Teorema del ángulo extemo :

a ABF -» B?D = BÁF + ABF /. BFD = a + <(> Luego : BFD = BDA

a ADC BDA = DÁC + C /. BOA = a + <(> a BFD, es Isósceles y BF = BD

8) En la figura : Hallar V

A) 60°
B) 50°
C) 75°
D) 65°
E) 80°

Solución
* a ABO, Isósceles : AB = BD
* a DBC, Isósceles : BC = BD B

Luego : a ABC, Isósceles ya que AB = BC


a = 30° y x = 20° + a
x = 50°

Rpta: ( B) .

9) En la figura adjunta,hallar la medida


del ángulo ABC, sabiendo que ;
É = 60°, D = 70° y CD = DE = BE

A) 45°
B) 35°
C) 30°
D) 25°
E) N.A

Solución

Incógnita : ABC = x

* Trazamos DB

Como BE = DE y É = 60°

El a BED es equilátero. Entonces :

BD = DE y BDE = DBE = 60°

Luego : BDC = 70° - 60' BDC = 10°

133
* El a CDB, es isósceles. C = CBD = a

OC + CX + 1 0 °= 180° -> a = 85'

* Finalmente, en “B' x + a + 60° = 180°


x + 85° + 60° = 180°
x = 35°

Rpta: ( B ).

10) Del gráfico, hallar " x ",

S i: AB = BC a CD

A) 5o B) 10' C) 12°
D) 15° E) 18'

Solución
Trazamos BD. L u e g o :

a BCD, equilátero

CBD = 60° y BC = BD

A d e m á s: ABD = 160°

a ABD, isósceles:

AB = BD x + x + 160° = 180° x = 10'

Rpta: ( B ).

11) En un triángulo ABC se traza la ceviana exterior BF. Calcular la longitud de A F si BF = 8,


AC = 6 y el suplemento de BFC es el doble del ángulo C ( F en la prolongación de CA ).

A) 0,5 m B) 4 m C) 2 m D) 3 m E) 2,5 m

Solución

* AF = ?

Considerando el gráfico adjunto :


BFE es suplemento de BFC
BFE = 2C = 2a
Entonces, c o m o :
BFE FBC + C
U
2a FBC + a

134
FBC = a -> AFBC es isósceles -> FC = FB
FA + 6 = 8
FA = 2

Rpta: ( C ).
B
12) Del gráfico, demostrar que :

D em ostración

* Del g rá fico : ABCX -» x = a + B + <j>

AXCD —> D= a + B+ ó

Restando miembro a miembro : x - D = B-x

, , . B+D
De donde : x = — -—
2

13) Demostrar que, en todo triángulo, las bisectrices de un ángulo interior y otro ángulo exterior,
forman un ángulo que mide la mitad del tercer ángulo del triángulo.

Solución
B
- Sea el A ABC, donde AE y CE bisecan
un ángulo interior y otro exterior, respecti­ E
>
vamente; demostremos que :

C Q
2

- Con el Teorema del ángulo externo :

AAEC -> ECQ = EÁC + É $ = a + É ( 1 ).

A ABC -» BCQ = BÁC + B 2<|> = 2 a + B ........ ( 2 ).

Ahora, reemplazamos (1) en (2):

2 (a + E ) = 2a + B

2a + 2 E = 2a + B

De donde :

L.q.q.d.
14) Demostrar que, en todo triángulo.e I mayor ángulo que forman las bisectrices de dos ángulos
interiores, mide 90° más la mitad de la medida del tercer ángulo
Solución
Sea ABC, el triángulo.
Demostremos q u e :

* En efecto; tenemos , en el cuadrilátero no convexo A B C I:


A ÍC = a + B + <|> .......... ( 1 ).

y en el AAIC : a + A IC + 4> = 1 8 0 ° .....( 2 ).

Sumando miembro a miembro las expresiones ( 1 ) y ( 2 ) :


2 A ÍC + a + <¡) = a + B + $ + 1801

cancelando a y $ : 2 A IC = 180°+ B

De d o n d e : A ÍC = 90° + - . .. I.q .q .d.


2

15) El A ABC de la figura adjunta, es de


cualquier naturaleza. Demostrar que

Solución

En el AAEC : a + <f>+ É = 180°


a + (J> = 180° - É ( 1)

En el A ABC : BAC + B + ACB = 180°

180° - 2a + B + 180° - 2(¡> = 180

De donde: 2ct + 20 = 180°+ B

Con (1 ): 2 ( 180o - É ) = 180°+ B 3 6 0 °-2 É = 180°+ é

Luego : 2E = 180o - B

B
E = 90° - - l.q.q.d.
2

136
16) En la figura

P + Q = 230°.

Hallar la medida del ángulo ABO

A) 100° B) 00° C) 50°


D) 160° E) 130°

Scáuctím

* Incógnita: ABD

Por la propiedad demostrada anteriormente :


ABC
A ABC P = 90° +

CBD
ABCD Q = 90° +

Sumando miembro a miembro


ABC + CBD
P + Q = 180° +

Usando el dato y observando que ABC + CBD = A B D , tenemos :

ABD
230° = 180° +

De donde : ABD = 100

Rpta: ( A ).

17) Del gráfico, hallar V

A) 119°
B) 129°
C) 109°
D) 122°
E) N.A

Solución

Prolongamos EA y FC hasta P

Por propiedad :
A

A ABC P = 90° - —
2
64
P = 90° -

137
• • P = 58'

De otro lado, en el A EPF

x = 90° + -
2
58
x = 90° +

x = 119'
Rpta: ( A ).

18) Del gráfico, hallar el valor de x

A) 48°
B) 24°
C) 42°
D) 22°
E) 21°

Solución

Prolongamos AG y EC hasta su intersección en H.

Por la propiedad demostrada anteriormente :

A ABC h = b
2
H =42°

_ H 42°
AAEH
X “ 2 " 2

x = 21°

Rpta ( E ).

19) En la figura :
Á + B + C + D = 230°
Hallar el valor de x.

A) 50‘
B) 65(
C) 70'
D D) 60
E) 4 0

138
Solución
Con los trazos indicados

Teorema del Z externo

e = A + B
A A

p= C + D

Sumando miembro a miembro :


e+p=Á +B +C +D
con el d a to :
e + p = 230°

suma de las medidas de


También : y + e + P = 360' ángulos externos en el
triángulo sombreado.

Con lo a n te rio r: y + 230° = 360°.

y = 130°

Finalm ente: x = 180°- y


x = 50°

Rpta: ( A ).

20) En el triángulo ABC donde AB = 4,2 y BC = 8,2. Hallar la suma del máximo y mínimo va
lor entero de AC.

A) 18 B) 17 C) 16 D) 15 E) 14

Solución
Por el Teorema de desigualdad B
entre los lados de un triángulo :

8,2 - 4,2 < x < 8,2 + 4,2

4 < x < 12,4

Luego : Mínimo valor entero de x : 5

Máximo valor entero de x : 12

R p ta : 5 + 12 = 17 (B ).

21) En un A ABC, Á > C , se traza la bisectriz interior

Demostrar q u e :

139
S o lu c ió n

En ei gráfico, usando ©I Teorema del ángulo externo

A ABD BÓC = Á + 0

ABDC AOB = C + p

Restando miembro a miembro estas expresiones

BÓC - A Ó B - Á - C

En un A ABC, A > C, BH y BO son aftura y bisectriz interior, respectivamente

Demostrar q u e :

Solución

Haciendo uso del problema a n te rio r:


BDC - AÓB = Á - C ( 1 ).

En el A BHD :
HDB = 90° - HBD

AÓB = 90° - H B D ................ ( 2 ).

También, por ser ángulo externo en el A BHD :

BDC = 90° + H B D ............. ( 3 ).

Finalmente, remplazando ( 2 ) y ( 3 ), err ( 1 )

( 90° + HBD ) - ( 90® - HBD ) = Á - C

A- C
De d o n d e : HBD =

bisectrices
perpendiculares
Solución
Sea R el punto de intersección de las
bisectrices del Á y HBC . Bastará probar

que : R = 90°

En efecto. Sabemos que, para el ABC,


recto en B, HBC = Á
En el cuadrilátero no convexo : R = RBC + RAC + C

R= a + a + C R = 2 a + C ............( 1 )■

Pero, en el A ABC : Á + C = 90° 2a + C = 90°

Reemplazando en ( 1 ) : R = 90°

- Nota.- Este resultado constituye una propiedad para todo triángulo rectángulo. También se
demuestra que las bisectrices de los ángulos C y ABH son perpendiculares entre sí.

24) En la figura:

AB = BC = BD.

Demostrar, que :
a
x = —
2

S olución

Del gráfico
B D
- AABD, isósceles : A = BDA = ó

- A BCD, isósceles : C = BDC = x + <)>

- Por el teorema del ángulo externo :

l.q.q.d.

25) En la figura :

BC = CE = CD.
B C
Hallar el valor de a.

A) 36°
B) 60°
D C) 30°
D) 24°
E) 18°

141
Solución
Como CE = CB = C D ; entonces,
por la propiedad demostrada en el
problema 2 4 :

BCE a
EDB =

En el A A B D :
D
2a + x = 90'
a
2a + — = 90° -> a = 36
2
Rpta: (A).

26) En un A ABC, obtuso en B, BC = 2 y AC = 5. Hallar el valor o valores enteros que puede


tomar AB.

A) 5 B) 6 C) 4 D) 4; 5 y 6 E) 4 y 5

Solución

Como B es obtuso, éste será el mayor ángulo.


En consecuencia, el mayor lado es AC.
Luego : AB < AC

AB < 5 .......... ( t ).

Además, se debe c u m p lir: AB > AC - BC

AB > 5 - 2

AB > 3 ............( 2 ) .

Observando (1 ) y ( 2 ), el único valor entero de ABt es 4.

Rpta: ( C) .

27) El perímetro de un triángulo rectángulo es 18. El mínimo valor entero para la longitud de la
hipotenusa, es :

A) 7 B) 6 C) 5 D) 8 E) 9

Solución

Sea el triángulo rectángulo, con los lados mostrados.

D a to : a+b+c=18 -» a + b = 18 - c ... ( I )
Se debe c u m p lir: c > a
c > b (II)

142
Sumando miembro a miembro las expresiones en ( I I )

2c > a + b

Con lo obtenido en ( I ) :
2c > 18 - c

3c > 18 —> c > 6

Luego, el mínimo valor entero de c : 7

Rpta: ( A ).

28) Se tiene el triángulo isósceles ABC ( AB = BC ). Se toman los puntos G, M y F en


AB, BC y AC respectivamente tal que el triángulo FMG es equilátero.
Si GFA = a , BGM = p y FMC = 0 ; se cumple :

a - $ a + <(> p + <j> P-Ó p-2<t>


A) P = B) P = C) a = D) a = E) a =

Solución

- AABC, AB = BC /. A = C

- Con el teorema del ángulo externo :

AFAG -> Á + a = p + 6 0 ° ............ ( 1).


A FMC C + 4 = a + 6 0 ° ........... ( 2 ).

Efectuando (1) - (2 ):
( Á + c c ) - ( C + ó ) = ( p + 60° ) - ( a + 60° )
Siendo A = C :
p + <t>
ex —<¡> = p - a De donde : a =

Rpta: ( C ).

29) En un AABC, C - Á = 42e ; BE es bisectriz exterior. Hallar la medida del ángulo CEB

A) 42° B) 21 C) 12 D) 10,5 E) N.A

i Solución

Incógnita : É = x
Sean a y ó, medidas de los ángulos
C y A, respectivamente.
El dato : a - <i> = 42°.

Sea R un punto de AB .Usando el Teorema


A _ A A

del ángulo externo en el AABE : RBE = A + E

143
RÉE = ó + x. Entonce# ; CÓE * RÓE -> CÓE * ó + x
También, en el A C 8E : a = QBE + É
a = (4» + x ) + x

De donde : a - <t> = 2x

Con el dato : 42° = 2x -> x = 21°

Rpta: ( B ).

30) En un A ABC, AB = BC, CR es una ceviana interior, tal que RCB = 24°. La bisectriz del
ángulo ARC corta a AC en el punto O.
A

Hallar le medida del ángulo AQR.

A) 72° B) 56° C) 76° D) 78° E) 82°

Solución

Incógnita : AQR = x

Sean QRC = p y QCR = 4>


ARQ = p

A ABC, isósceles : Á = ACB = 24° + ó

* En el AQRC, por el Teorema del ángulo externo : P + 4> = x (1 ).

* Suma de las medidas de los ángulos del A ARQ : p + (2 4 ° + ó ) + x = 180°

p + 4>+ x = 1562 ............( 2 ) .

Reemplazando ( 2 ) en ( 1 ) : x + x = 1 56°.
x = 78°.

Rpta: ( D ).

31) En un triángulo ABC; AB + BC = 30 cm. Se ubica un punto M sobre AC; si AC = 2 0 cm.


Hallar el menor valor entero de BM.
%

A) 6 cm. B)4cm C) 2 cm. d )1 0 c m . e) 9 cm.

Solución
Por mínima distancia :

AB < x + AM

BC < x + MC

AB + BC < 2 x + AC

144
30 < 2 x + 20
5< x
x menor entero = 6

Rpta: ( A ).

En la fig u ra :

ti Lt II U
a + b = 134'

Hallar el valor de x

L2 A) 23' B) 33' C) 43'


D) 67 E) 53*
/
Solución
x = ?
9(T-a
90,--b
D ato: a + b = 1 3 4 °............(1 ). Q i R Lj
Con los trazos indicados,
y haciendo uso de la propiedad :

Se obtiene : E = 90° - b

Por el teorema del ángulo externo

APEQ —> x + m = 90° - b Sumando miembro a m iem bro:


APQR —> x = m + 90° - a 2 x + m = 180° - ( a + b ) + m

Con (1):
2 x = 180° - 134°
x = 23°
Rpta: ( A ).

ABC, es un triángulo isósceles, recto en B. E, es un punto exterior a ABC, relativo a BC,


tal que AB = BE. Hallar la medida del ángulo AÉC.

A) 30° B) 22,5° C) 60° D) 45' E) 37

Solución

Incógnita: AÉC = x
*
Como ABC es un triángulo isósceles y recto en B, entonces : AB = BC

145
Luego : BE = AB = BC.

El A ABE, es isósceles.

BÁE = ABE = a.
Y en el A BEC, también isósceles :
BCE = BÉC = a + x.

Por ser ángulo externo en los triángulos


sombreados : é = 90o + a
y: é = x + a + x

Igualando los segundos miembros :


2 x + a = 90° + a
De d o n d e : x = 45'

Rpta: ( D ).

En la figura :
AB = BC y EF = FH.
B Hallar el valor de “x*

A) 12°
B) 10°
C) 20°
D) 15°
E) 18°

X = ?
A A

A ABC, isósceles : A = C = a

Al trazar EH , resulta el AEFH, equilátero.


Luego : EHF = HÉF = 60°

y EHA = 50°
Por el Teorema del ángulo externo :
AAEH -> BÉH = Á + AHE 3 x + 60° = a + 50°
3x + 10° = a (

AHFC AHF = HFC + C 50° + 60° = 2x + a


110° = 2 x + a .........

De ( 1 ), con a = 3x + 10°, reemplazando en ( 2 ) :


110° = 2 x + 3x + 10c
x = 20°
Rpta: (C ).
35) En la figura,
demostrar que D

Demostración

* En el cuadrilátero no convexo DPEF :


DFE = x + (3 + r (1 ).

* por el Teorema del ángulo externo :

- AADF: D F E - A + 2p ( 2 )

- AFEC : DFE = C + 2r

Sumando miembro a miembro las expresiones de ( 2 ) :

2 ( DFE ) = Á + C + 2p + 2r

con (1): 2 (x + p + r)= A + C + 2p + 2r

.*. 2 x + 2 P + 2 r = Á + C + 2p + 2r

2x = A + C

A + C
De donde : x =

36) En un A ABC, se trazan las bisectrices interiores AE y C D . Las bisectrices de los ángulos
ADC y AEC se interceptan en el punto “P”.

Demostrar q u e :

B
Demostración
* En la figura sombreada,
por la propiedad demos­
trada en problema anterior
0 *

P=

147
Así:

A C
P= 2 + 2

A + C 180° - B B
P= P = 45° - -
4

37) En un A ABC, recto en B , AC = 22 ; sobre AC se toma el punto F y en la prolongación


de CB, el punto Q. FQ corta a AB en R. Hallar RF, si QRB = Á y QR = 6.

A) 10 B) 9 C) 6 D) 8 E )7

Solución /

Incógnita: RF = x.

Del dato : QRB = A = a

ARF = QRB = a

El ARFA, es isósceles.
Q'
Entonces : AF = RF

AF = x

a ^
En el AQBR : Q = 90° - a Q = C
y en A ABC : C = 90° - a AQF C, isósceles

L u e g o : FC = FQ ->

Finalmente.como : AF + FC = AC

i jj. i
Usando lo obtenido : x + x + 6 = 22

De donde :
x = 8

Rpta : RF = 8 ... ( D ).

38) Hallar el valor de x,

S i: AM = BM = MC = CQ y BE = EM

A) 10' B) 15 C) 20'
D) 18' E) 12'

148
Solución

Haciendo uso de los triángulos isósceles


y el Teorema del ángulo externo :

AMCQ CMQ = Q = x
y MCB = x + x = 2x ( Z externo ).

Además : AME = CMQ = x


(opuestos por el vértice).

AMBC MBC = MCB = 2 x (A isósceles).

AMBQ -> EMB = MBQ + Q , ( Z externo ).

EMB = 2x + x -> EMB = 3x

ABEM, isósceles : EBM = EMB= 3x


A A

Finalmente, en el AAMB isósceles : A = ABM = 3x

A + ABM + AMB = 180

3x+ 3x + 4x = 180°

Rpta: ( D ).

En el interior de un triángulo ABC, se toma el punto E, siendo AE = BE y AB = EC

S i: Z A B E = Z E C A = x, Z EAC = 2x, Z EBC = 5x. Hallar el valor de x.

A) 5o B) 10° C) 12° D) 15' E)18

Solución
x = ? B

En el cuadrilátero no convexo
( cóncavo ) ABEC :

Ó = 5x

Luego, el A EBC es isósceles: b = a v C


y el A ABC, BC = AB: ACB = BÁC = 3x

E ntonces:
BCA + BAC + ABC = 180°
12x =180°
• • x = 15*

Rpta: ( D ).
40) En la figura adjunta, trazar las alturas AH y CQ.

Hallar la medida del ángulo formado


por las bisectrices de los ángulos
HAB y BCQ.

A) 30‘ B) 45 C) 60'
D) 75' E) 90'

Solución
* Incógnita : x

* Del gráfico, luego de trazar las alturas,


como HBA = QBC; entonces:

HÁB = QCB, ya que :

HÁB + HBA = 90Q= QCB + QBC

* En el cuadrilátero no convexo ABCP :


ABC = a + x + a -h> ABC = 2 a + x a««
X
•rv
También, por ser ángulo exterior del AAHB : P Q

ABC = H + HAB
Igualando a lo a n te rio r:

2 a + x = H + HAB
2a + x = 90° + 2a

x = 90°
Rpta: ( E ).

41) En un A ABC, B = 36° y C = 84°. AE y CR , son cevianas internas , AR = AC y


BÁE = 12°. El ángulo AER, mide :

A) 48° B) 24' C)12 D) 30’ E) 25


Solución

* Incógnita: AÉR = x

* En el AABC : Á = 180° - ( 36° + 84° )


Á = 60°

AARC es equilátero ya que, por dato :


AR = AC.

RC = AR = AC y ACR = 60°

Entonces : RCE = 24°

150
* En el A AEC, CAE = 48° y ACE = 84°
Luego: AÉC = 180° - ( 48° + 8 4 °) = 48°

Resulta : EÁC = AÉC A ACE , isósceles : EC = AC

* Finalmente : ARCE , isósceles -> ERC = RÉC = x + 48

ERC + REC + RCE = 180'

( x + 4 8 °) + ( x + 4 8 °) + 24° = 180'

De d o n d e : x = 30

Rpta: ( D ).

42) En un A ABC, recto en B, se traza la altura B H , la cual es cortada en los puntos Q y M por
las bisectrices interiores AD y CE, respectivamente. Hallar MQ, si BE = 7 y BD = 10.

A) 3 B) 1,5 C) 2 D) 6 E) 4

Solución
Consideremos el gráfico adjunto.

Incógnita : MQ.

Sabemos, por el problema nQ7,


que el A QBD es isósceles

BQ = 10

Análogamente, e I AEBM también


es isósceles BM = 7

- Entonces : MQ = BQ - BM
MQ = 1 0 - 7 MQ = 3

Rpta: (A).

En un A ABC, C = 80°, se trazan las bisectrices interiores AE y BF. Hallar la medida


del menor ángulo formado por las bisectrices de los ángulos AEC y BFC.

A) 20° B) 24' C) 30 D) 40 E) 25'

Solución
Considerando el gráfico adjunto : ( hoja posterior).

En el cuadrilátero no convexo sombreado :


r + n + 80° = 180° - x
r + n + x = 1 0 0 °.......... ( I ) .
151
Por el Teorema del ángulo externo :
AABF 2r = 2a + p
A ABE -> 2n = a + 2 p (+ ) B

2 ( r + n ) = 3 ( a + P ), siendo :

a + p = 50°, del A ABC

Entonces:
2 ( r + n ) = 3(50°) -» r + n = 75o ... ( I I ) .

Sustituyendo (II), en ( I ) : 75° + x = 1 0 0 °

x = 25

Rpta: (E).

* Nota.- En general, se tendrá :

44) Hallar “a”, s i : AB = BE = BD = CD

A) 10°
B) 20°
C) 15°
D) 30°
E) 18°
Solución
* AABD, isósceles
ADB = 30° y DBE = 60

* Trazamos ED . Luego:

el A EBD es equilátero.
ED = BE = BD y BDE = 60°

* ACDE,resulta isósceles y recto en D : n = 45'

* Finalmente, en UE” : a + n = 60°


a + 45° = 60° -» a = 15°

Rpta: ( C ).

45) En el interior de un AABC, se toma el punto “O”, de modo que OA = OC = AB .


S i : ABC = 12x, OÁC = 3x y OCB = 2 x, hallar el valor de “x”.

A) 5' B) 6' C) 7' D) 8 E) 9'

152
Solución

Trazamos BO. En el A AOB, isósceles,


sea : AÓB = a ABO = a

Para el cuadrilátero no convexo ACBO :

a = 3x + 5x + (12x - a) a = 10x.

Volviendo al gráfico : OBC = 1 2 x - a OBC=:2x


ABOC es isósceles : OB = OC .

El AAOB, resulta equilátero

a = 60 10x = 60' x = 61

Rpta: ( B ).

NIVEL III

46) Del gráfico, hallar el valor de “x”.

A) 60°
B) 45°
C) 30°
D) 75°
E) 90°

Solución
* En el cuadrilátero no convexo BECF :
BFC = x + 2m + 2r

* Con el Teorema del ángulo externo :


A ABF : BFC = b + r + 2m
x + 2m + 2r = b + r + 2m
x+r = b (1 ).

AFCD : BFC = a + 2r + m
X + 2m + 2r = a + 2r + m
x+m = a ( 2 ).

* Finalmente, en el AAFD : FÁD + AFD + ADF = 180°

Luego : 180° - ( a + b ) + x + 2m + 2 r + 1 80° - ( a + b ) = 180'


153
De donde : x + 2m + 2r + 180° = 2a + 2b
«

Con (1) y (2): x + 2m + 2r + 180° = 2( x + m ) + 2 ( x + r )


Efectuando: 180° = 3x -> x = 60° /

Rpta: ( A ).

47) En la figura : AB > BC y CD > ED

Hallar “x” , si su valor es entero.

A) 65°
B) 110°
C) 115°
D ) 125°
E) 135°

Solución
Como x es entero, entonces la medida del ACB = ECD también lo es.

Recordando que “a mayor lado se opone mayor ángulo, en un mismo triángulo” :

A ABC AB > BC ACB > 6 4 ° ............( 1 ).


A CDE -> CD > ED 66° > ECD -> 66° > ACB ...... ( 2 ).

De ( 1 ) y ( 2 ) : 64° < ACB < 66’ ACB = 65°


Luego : x = 180° - ACB x = 115'

Rpta: ( C ).

48) Los lados de un triángulo están en progresión aritmética de razón 5 cm. Hallar elmínimo valor
entero que puede asumir el perímetro.
A) 29 cm. B) 30 cm. d )"3 i cm. D) 32 cm. E) 33 cm

Solución

Considerando las longitudes de los lados del triángulo como indica la figura :
El perímetro es :
P = AB + BC + AC B
P = 3x \< * + 5 )
(x-5)
* Con el Teorema de desigualdad triangular
/
BC - AB < AC < BC + AB \
* -c
(x+5)- (x-5) < x <(x+5)+(x-5)
10 < x < 2x

De a q u í: x > 10

154
Para el perímetro multiplicamos por 3 :
3x > 30
P > 30
t
E lm in im o va lo re n te ro .e s P = 31 cm

Rpta: ( C ).

En un A ABC, la bisectriz exterior del ángulo C. corta a la bisectriz del ángulo BAC en el punto
E. Por E se traza paralela a AC , interceptando a BC en F, AB en L y a la bisectriz del
ángulo BCA en N. ( L sobre NF ).

S i: NL = 3 cm y EF = 5 cm. Hallar A L .

A) 7 cm B) 9 cm. C) 5 cm. D) 8 cm. E) N.A

Solución
Del gráfico, por ángulos alternos
internos entre paralelas:
Z AEN = Z EAC,
Z FEC = Z ECP,
Z ENC = Z NCA,

De lo a n te rio r: A ALE, isósceles :


AL = LE (1 ).

A CFE, isósceles : CF = EF CF = 5
A NFC, isósceles : NF = CF -> NF = 5 LF = NF * NL
-> LF = 2. Luego : LE = LF + EF = 2 + 5
LE = 7. En (1 ) : AL = 7 cm.

Rpta: ( A ).

En la figura, AF, BE y CQ, son tres


cevianas cualesquiera, concurrentes,
del A ABC.
Si AB + BC + AC = 4, hallar los valores
enteros, mínimo y máximo, de la expresión
AF + BE + CQ.

A) 3 y 7 B ) 4 y 12 C ) 3 y 11 D) 4 y 9 E) 3 y 12

Solución
Por el Teorema de desigualdad entre los lados :
A ABE AB - AE < BE < AB + AE
A EBC BC - EC < BE < BC + EC

1S5
Sumando miembro a miembro y manteniendo los sentidos de desigualdad :

AB + BC - ( AE + EC )< 2 BE < AB +BC + ( AE + EC )

Esto es: AB + BC - AC < 2 BE < AB + BC + A C ..........( 1 ).


Análogamente : AB + AC - BC < 2 AF < AB + AC + B C .......... ( 2 ) .
y también : AC + BC - AB < 2 CQ < AC + BC + A B .......... ( 3 ) .

Sumando ahora, miembro a miembro, las expresiones ( 1 ), ( 2 ) y ( 3 ) :

AB + BC + AC < 2( AF + BE + CQ ) < 3( AB + BC + AC )

Usando el dato : 4 < 2( AF + BE + CQ ) < 3(4)


2 < A F + BE + CQ < 12

Valores Mínimo = 3
enteros Máximo = 11

Rpta: ( C ).

51) En un cuadrilátero convexo ABCD,Á= 60°,B = 130°, AB = BC y AD = AB + CD.


Hallar la medida del ángulo D.

A) 20° B) 50° C) 40’ D) 60' E) 55

Solución

* Datos: Á = 6 0 °,A § C = 130°.


AB = BC y AD = AB + CD

* Incógnita : D = x

* Sean : AB = a = BC y
CD = b AD = a+b

Sobre AD tomamos el punto E,


tal que AE = a = AB ED = b

L u ego:
A ABE Equilátero BE = a
AEBC Isósceles : (3 = 55°
AECD Isósceles : a = 180° - ( 60° + p )
a = 65°

Entonces : x = 50fi

Rpta: (B).

156
52) En un triángulo PQR, PQ = 6, se traza la ceviana interior QF , tal que :
A A A A

FQR = 3R y QF = 4 , Si P = 2R . Hallar FR.

A) 12 B) 10 C) 8 D) 11 E )9

Solución

* Sea R = a —> P = 2 a y FQR = 3 a


Q
* Incógnita: FR

* Trazamos QN , de modo que:


QNP = 2 a = P

Luego, APQN es isósceles : QN = 6 z«4._


P F N R
A QFN, Isósceles : FN = 4
y A QNR, Isósceles : NR = QN —> NR = 6

Entonces : FR = FN + NR = 4 + 6

FR = 10

Rpta: (10).

53) Los lados de un triángulo ABC, tienen longitudes: AB = 10, BC = x - 5 y AC = 2 x - 3 .

H a lla r:

a) ¿ Entre qué valores se encuentra x ?

b) ¿ Entre qué valores se encuentra el perímetro de ABC ?

c) Los valores enteros mínimo y máximo del perímetro y los respectivos valores para x.

d) ¿ Qué valor(es) entero(s) toma x; y el respectivo valor del perímetro?

Solución B

Hacemos uso del Teorema de la


desigualdad triangular:

* BC + AC > 10 ( x - 5 ) + ( 2 x - 3 ) > 10 x > 6 ............( I ).

* AC - BC < 10 -> ( 2 x - 3 ) - ( x - 5 ) < 10 x < 8 ............. ( I I ) .

( l ) y ( I I ) Pueden representarse como :

6 < x < 8 (a).

* Para desarrollar las preguntas usaremos ( a )

A sí:

157
a) Rpt a: x se encuentra entre 6 y 8.

b) Para hallar el perímetro: P = AB + BC + AC

P = 10 + ( x - 5 ) + ( 2 x - 3 ) P = 3x + 2 ... (y):
Debemos obtener la expresión para Pf a partir de ( a ).

6 < x < 8

Para ello, efectuamos las siguientes operaciones, miembro a miembro :


- Multiplicando por 3 :
18 < 3x < 2 4

- Sumando 2 : 20 < 3x + 2 < 26

Luego : 20 < P < 2 6 ... ( p )

Rpta : El Perímetro se encuentra entre 20 y 26 .

c) De ( p ) :

* Valor entero mínimo de P : 21


19
El respectivo valor de x se obtiene al igualar ( y ) con éste v a lo r: 3x + 2 = 21 x=

* Valor entero máximo de P; ( usando p ) : 25

Para x, con ( y ) : 3x + 2 = 25
23
Rpta: x =

d) Según ( a ) : 6 < x < 8. El único valor entero de x, es : 7.


y el correspondiente valor del perímetro P, se obtiene al reemplazar en ( y ) : P = 3x + 2
" P=3(7)+2

Rpta: P = 23

54) En un A ABC, Á = 2C y AB = 2 . Hallar BC, sabiendo que es un número entero.

A) 3 B) 4 C) 5 D) 6 E) N.A

Solución

Sea el gráfico adjunto. B


Trazamos la ceviana BR , •cx\

de modo que = Á
A ABR, isósceles : 2oc/\ nt
R
BR = AB = 2 .

158
En el A BRC : CBR + C = ARB

CBR + a = 2 a —» CBR = a

A RBC, isósceles RC = RB = 2

Se debe c u m p lir: BC < BR + RC


x<2 + 2 -> x < 4 o)
* A

Por otro lado, en el A ABC, como A > C

entonces : BC > AB x > 2 (2 )


Luego, de (1 ) y ( 2 ) :

Rpta: x = 3 (valor entero) ... ( A )

55) En un A ABC, AB = 4 y A = 2 C , se traza la bisectriz interior BD. Hallar AD, sabiendo que
BC toma su máximo valor entero.

A) 2 B) 4 C) 3 D) 2,5 E) N.A

Solución

-S i C = cc —> Á = 2 a B

Sean las longitudes : BC = a y AD = x

Prolongamos CA, hasta F, de modo


que F = C.

A FAB isósceles : FA = AB = 4
AF BC isósceles : FB = BC = a

En el A FAB F B < A F + AB a < 4 +4 —» a < 8

El máximo valor entero de “a” , es 7.


Por ser ángulo externo del ABDC : ADB = a + ó

Finalmente, en el AFBD, isósceles : FD = FB


x + 4 = a
x + 4 = 7 x = 3

Rpta : AD = 3 ... ( C ).

56) ¿ Cuántos triángulos escalenos, de perímetro menor que 10 y cuyos lados tienen valores
enteros para sus longitudes, existen ?

A) 1 B) 2 C) 3 D) 4 E) Ningún triángulo

159
Solución
Sean a, b y c, longitudes de los lados,
con a * b * c

Se debe c u m p lir:
a + b + c < 10 o
Además : a < b+c ( I I ):
Sumando miembro a miembro ( I ) y (II): 2 a < 10

« ♦

Esto nos conduce a concluir que cualquier lado (a, b ó c) es menor que 5
Supongamos que a > b > c. Como a < 5,tanteamos :

* Si a = 4 ; entonces en ( I ) : b + c < 6
y en: ( II): b + c > 4

De d o n d e : b + c = 5

Unicos posibles : b = 3 ; c = 2

Luego, cumplen las condiciones del problema : { a; b; c } = { 4; 3; 2 }

** Si a = 3. Al reemplazar en ( I ) :

b + c< 7. Y, en ( I I ) : b + c > 3

Además, por la elección hecha : b < a


c < a (+)

b + c < 2a ,
b + c < 6 , para a = 3
Entonces : 3 < b + c < 6
Posibilidades : ( con a = 3 )

b+c b c

3 1 —» No cumple b * a
A
H
2 2 —> No cumple b * c
4 1 —» No cumple b < a
5
3 2 —> No cumple b * a

No hay solución cuando a = 3

* ** Si a = 2, tampoco habría solución, ya que hemos puesto por condición que a > b > c.
Así el problema tiene 1 sola solución :
{ a; b; c } = { 4; 3; 2 }
Rpta: ( E ).
160
57) En un A ABC. Hallar el máximo valor entero de BC. Si AB = 4 y Á = 3C

A) 12 B) 11 C) 10 D) 9 E) 8

Solución

Sea C = a Á = 3a B

Tracemos A F , de modo que:


FÁC = C = a . Luego :

BÁF = 2 a -» A BAF, isósceles


BF = AB = 4.

* A FAC, isósceles : FC =. AF = x.

* EnelAABF: x < 4+4 x < 8

* BC tendrá valor máximo cuando x sea máximo

y máximo = 7' B C máximo = 4 + X = 11

Rpta: (B).

58) En la figura : AB = CE.

Hallar el valor de “x”

A) 20°
B) 30°
C) 45°
D) 60°
E) N.A

Solución

En el A ABC, trazamos la ceviana B D ,


de modo que BD = AB.

Luego, los triángulos ABD y BDC son


isósceles y el A DEC, equilátero ya que
•..76?
DC = CE y DCE = 60°. Así, el ABDE, 44‘ V B O » ,22' r ^ C

resulta isósceles :

2 ( x + 22°) + 76° = 180

De d o n d e : x = 30°

Rpta: ( B ).

161
59) En la figura : OB = AC

Hallar el valor de “x”

(60"*a) A) 30°
B) 20°
C) 15°
D) 24°
E) 36°

Solución

* Prolongamos AO hasta “E” tal que : CE = AC

Entonces :
A ACE, isósceles AEC = EAC = 48

En A OEC : OCE = 66°

Luego, A OEC : Isósceles


OE = CE

* Por otro lado :


BÓE = OÁB + A B O ( ¿ exterior, en el A AOB ).

BOE = 60°

En conclusión, el A BOE Equilátero BE = OB = OE

180° - BÉC
* Así, el A BEC, resulta isósceles : BCE = CBE =

/. BCE = CBE = 36

* Por último : x = OBE - CBE

x = 60°-36 x = 24

Rpta: ( D ).

60) En la figura, B = 20°, AB = B C , N A C = 50° y ECB = 20°.


Hallar el valor de “x”

A) 20°
B) 15°
C) 30°
D) 40°
E) 25°

162
Solución

* Con la figura dada, se deduce que el


A ACN es isósceles, donde AC = CN

* Trazamos luego, CL tal que CL = AC

E ntonces:
A LCN -» Equilátero
A ELC Isósceles

y en A ELN, isósceles :

LÑE + LÉN + ELN = 180°


t

2 ( x + 4 0 °) + 40° = 180°

x = 30°

Rpta: ( C ).

61) En la figura, BC = CE; Á = RCB = 36° ECR = 12° .

Hallar el valor de x, medida del ángulo .


ERC.
A) 24°
B) 36°
C) 20°
D) 15°
E) 30°

Solución

Incógnita: ERC= x
* A ARC, Z externo : BRC = Á + ACR BRC = 48°

* Sobre la prolongación de AB, tomemos


el punto Q, de modo que:
RCQ = QRC = 48°
RQ = QC y BCQ = 12°

Además, en el A ABC :
QBC = Á + ACB

QBC = 84°

Luego, en el A BCQ : BÓC + 84° + 12° 180° -> BÓC = 84°

163
Es d e c ir: A BCQ isósceles • « BC = QC y

Como según dato BC = CE CE = QC

Así, el A ECQ, resulta equilátero, ya que además : ECQ = 60° .

E ntonces: EQ = EC = CQ y por lo tanto EQ = RQ

E nseguida:
EQC = 60 RQE = 84° - 60° = 24'

RQE = 24(

Finalmente, en el A RQE isósceles


REQ = ERQ = x + 48°
RÉQ + ERQ + RQE = 180°
^
x + 48° + x + 48° + 24° = 180'

De donde : x = 30'

Rpta: ( E ).

En la figura, AE = BC.

Hallar el valor de “x”

A) 20°
B) 10°
C) 15°
D) 16°
E) N.A

Solución
- Debemos buscar un trazo conveniente

- Sobre la prolongación de AB, tomamos


el punto R, tal que al tra z a r: RC

ACR = 2 Á = 80°

BCR = 60°. Y como el RBC = 60°


resulta el A BRC equilátero.

Ahora, sea F un punto de A C , tal que:


RF = RC . RFC = 80°.

Entonces, para el A ARF : RFC = Á + ARF

80° = 40° + ARF -> ARF = 40'


Esto indica que el A AFR, es isósceles con : AF = RF AF = AE FyE,
son un mismo punto.

El gráfico será :

Luego, en el A BRE, isósceles :


EBR = BÉR = 60° + x

y: EBR + BÉR + BRE = 180°

60° + x + 60° + x + 40° = 180°


x = 10°

Rpta: ( B ).

165
r
CAPITULO 6

CONGRUENCIA DE TRIANGULOS
* Dos triángulos se llaman congruentes, si tienen sus lados y ángulos, respectivamente
congruentes.
r*% C

A ABC = ADEF
C D*— *—£*fr •
* “A lados congruentes, uno en cada triángulo, se oponen ángulos congruentes y viceversa.”

* Para que dos triángulos sean congruentes, deben cumplir con alguno de los casos de
congruencia. En ellos se menciona como requisito, que presenten tre s pares de elem entos
congruentes, siendo p o r lo m enos uno de e llo s un lado.

ler.C aso. (Postulado ALA).- Un par de lados y los ángulos adyacentes a ellos.

2do Caso. ( Postulado LAL ).- Dos pares de lados y el ángulo comprendido

3er. Caso.( Postulado LLL ).- Los tres pares de lados.

Existe un cuarto caso que se demuestra teniendo en cuenta los anteriores :

4to. Caso. ( Teorema ) : Dos triángulos serán congruentes si tienen dos pares de lados y
los ángulos opuestos a los mayores de dichos lados, respectivamente congruentes.

AB = DE
Sl * BC = EF

J BC > AB
D onde: \ : -
l y: A = D
Entonces : A ABC = ADEF

167
* observación.- Dos triángulos rectángulos, serán congruentes si tienen dos pares de
elementos congruentes ( aparte del ángulo recto ); los cuales pueden s e r :
a) Dos lados (caso LL.)
b) Un lado y un ángulo agudo (caso LA)

Por ejemplo :

/ «r
k

APLICACIONES DE CONGRUENCIA

1 Teorema.- “ Los segmentos paralelos, comprendidos entre rectas paralelas, son congruen
tes.”

AB DC y
S i: *

BC AD

Entonces :
AB = DC y
A
BC = AD

* D em ostración.-
- Al trazar BD :

ABD = BDC
alternos internos, entre paralelas B £L
CBD = ADB /

- Además, por tener BD común :


A ABD s A CDB . (ALA)
D
- L uego: AB = DC y BC = AD

* Nota.- El cuadrilátero ABCD, es un paralelogramo

Teorema de los p u n to s m edios (Base Media).- La paralela a un lado de un triángulo,


trazada por el punto medio de otro, corta al tercero en su punto medio. El segmento
determinado se llama base media o paralela media y mide la mitad de la longitud del lado al
cual es paralelo.
Si : AM = MB y
MN AC
Entonces :
BN = NC
AC
y MN =

168
MN, es base media relativa a AC

D em ostración

Sea MQ | BC. Luego:

AMQ = MBN
y Tienen sus lados paralelos

Á = BMN

A AMQ = A MBN ( ALA ) BN MQ


Pero : NC = MQ Entonces : BN NC
También : MN = AQ ; siendo : QC MN
AC
MN = AQ = QC =

C orolario.- En todo triángulo, el segmento que une los puntos medios de dos lados es
paralelo al tercero y mide la mitad de él.

B Si : AM = BM
v BN = NC

Entonces : MN AC
AC
MN =

Propiedad de la bisectriz.- Todo punto situado sobre la bisectriz de un ángulo, equidista de


sus lados.
A

Sea P, cualquier punto de la bisectriz del A.


PQ y PR, son perpendiculares a los lados
del ángulo.

C om o: A A QP = A A R P ( L A ) .

PQ = PR

Además : AQ = AR

Propiedad de la m ediatriz
Todo punto situado en la mediatriz de un segmento, equidista de sus extremos.

r : mediatriz de AB.
p P : cualquier punto de Y.
Como : A AMP = A BMP ( LL ).
A— * M*—- B
PA = PB

El AAPB,resulta isósceles: Á = B

169
Caso del tria n g u lo isósceles.- La altura relativa a la base es también mediana, bisectriz y
porción de mediatriz.

Si AB = BC

Altura
Mediana
BH, es: Bisectriz
Porción de mediatriz

Teorema.- En todo triángulo rectángulo, la mediana relativa a la hipotenusa mide la mitad


de ella.

AC
BM =

BM = AM = MC

D em ostración

Trazamos MN BC MN 1 AB

Por el teorema ns 2 : AN = NB

En el A AMB; MN es altura y mediana

según el ne 5 : BM = AM.

Teorema.- Tres o más rectas paralelas, equidistantes, determinan sobre cualquier recta
secante, segmentos congruentes.

S i: V J/ E / / 5 / / Í 4

y : d i — do — d

E ntonces: a = b = c
TRIANGULOS RECTANGULOS NOTABLES

1) Triángulo rectángulo isósceles :

b = aJ2 I longitud de la = ( longitud de 1 cateto ) x <J~2


I ^ hipotenusa.

o longitud de un longitud de hipotenusa


cateto.

2) Triángulo rectángulo de 30° y 60°:

longitud de hipotenusa
*Cateto Opuesto a 30° =

*Cateto Opuesto a 60° = f longitud de hipotenusa ' X /3


v

3) Triángulo Rectángulo de 15° y 75° :

* BH : altura hacia la hipotenusa AC


B

AC
BH =

* Nota.- Existen otros, aproximados, cuya relación de longitudes de lados indicamos a


continuación :
53°
4) De 37° y 53° : 5) De 16° y 7 4 °: 6) De — ó 26°30'

24n
37
7) De ó 18°30‘ 8) De 8o y 82°:

E je m p lo .-E n un A A B C . A b = 8 , Á = 45° y C = 30°, hallar BC

A) 4 B) 8 / 2 C) 6 / 2 D) 4 / 2 E) N.A

171
a

Solución
BC = ?

- Trazamos la altura B H . Luego : B

8
AAHB: BH = BH = 4 / 2
(45°) 30^c

- A BHC : BH se opone a 30°

BC = 8 / 2

Rpta: ( B ).

PROBLEMAS RESUELTOS

NIVEL I

1) UM" es punto interior al A ABC, equilátero, tal q u e : MÁC = 24° y MBC = 28°. Exteriormen-
te y relativo a AC, se toma un punto R, de modo que, el A ARM sea equilátero. Hallar la
medida del ángulo MRC.

A)52° B) 62° C) 48' D) 68' E) 58

Solución
X = ?
Completando los á n g u lo s:

ABM = 60° - 2 8 °= 32°


y MÁB = 60° - 24° = 36° ;

A dem ás: AMB = 112

También, como el A ARM es equilátero,


según dato : MAR = 60°

V RÁC = 60° - 24° = 36'

Entonces: A RAC = A MAB (postulado LAL : □ , 36°,


A A

Luego : ARC = AMB , (se oponen a lados congruentes de triángulos congruentes)

x + 60° = 112°

x = 52°

Rpta: ( A ).

172
En la figura : AB = BC ; AE = CD y
BED s BDE.

Hallar el valor de V

A) 20°
B) 22,5°
C) 25°
D) 30°
E) 18°

Solución
Colocando marcas iguales a los
segmentos congruentes, se observa

A ABC isósceles
ACB = BÁC = 3x

ABCD = A ABE -> ( L. L. L. ). D

BCD = BÁE -> y = 5x

En “C” : 3x + y = 180°

3x + 5x = 180' x = 22,5

Rpta: ( B ).

En un A ABC, AB = 12, Á = 78° y C = 39° ; la mediatriz de BC corta a AC en el punto


E. Hallar EC.

A) 6 B) 9 C) 12 D) 15 E) 8

Solución

Incógnita : EC = x

B - Por propiedad de la mediatriz :


EB = EC -> EB = x

Luego :
A BEC , isósceles : EBC = C = 39°

y AÉB = EBC + C ...( Z exte rio r)

AÉB = 78°

- Finalmente : A ABE, isósceles BE = BA

x = 12

Rpta: EC = 12... ( C ) .

173
4) En un A ABC, B = 78°; sobre AC se toma el punto E. Las mediatrices de AE y E C , cortan
a AB y BC en los puntos R y Q, respectivamente. Hallar la medida del ángulo REQ.

A) 102 B) 78 C) 39' D) 51 E) N.A

Solución
B
Del gráfico :
RÉQ = x .... (incógnita)

- Por propiedad de la mediatriz :

RA = RE y
QE = QC

A ARE , isósceles : AÉR = Á = a

y A EQC, ¡sóceles : QÉC = C = 0

- En ÜE” : a + ó + x = 1 8 0 °........ (1 ).

Pero, en el A ABC : A + C + B = 180

a + ó + 78° = 180'

a + ó = 102°

Reemplazando en ( 1 ) : 1 0 2 °+ x = 180'

De donde : x = 78°

Rpta: ( B ).

5) En un triángulo isósceles ABC, UM” es punto medio de AB y AC es la base. Se traza

MQ 1 AC (Q en A C ) . S i AQ = 2, hallar QC.

A) 4 B) 6 C) 8 D) 2 E) 3

Solución
QC = ?

Como ABC es un triángulo isósceles;


al trazar B H , altura relativa a la base :

HC = AH

En el A AHB, MQ es Base Media


QH = AQ
QH = 2

Luego : AH = 4 y HC = 4

174
E ntonces: QC = QH + HC
QC = 2 + 4 -> QC = 6

Rpta: ( B ).

6) El ángulo exterior B, de un triángulo ABC, mide 62°. Las mediatrices de AB y B C , cortan


a AC en los puntos E y F, respectivamente. Hallar la medida del ángulo EBF.

A) 56° B) 62 C) 52 D) 66' E) 64'

Solución
x = ?

- Por propiedad de la mediatriz :

EA = EB *-» A AEB, isósceles.


FB = FC -> A BFC, isósceles.

- En B : 62° + a + ó + x = 180° a + <J) + x = 1 1 8 °............. (1 ).

- Pero, por el Teorema del ángulo externo, en el A ABC : a + 0 = 62°


Reemplazando, en ( 1 ) :

62° + x = 118°
x = 56°

Rpta: ( A ).

7) En un A ABC, C = 15°, B = 90° y A C = 24, hallar la longitud de la bisectriz interior BD

A) 4 / 3 B) 2 / 3 C) 3 / 3 D) 5 / 3 , E ) 6 y[ 3

Solución
* BD = ?
B

Trazamos la altura BH. Luego, por propiedad


para esta línea, en el A ABC :

bh = ^ .= 21
4 4
BH = 6
/ \
BD
* En el A BHD (30°,60°) : BH = ( B H , opuesto a 60°).
V 2
\
BD
6 = 73
/
BD = 4 / 3

Rpta: ( A ).

175
8) ' En un A ABC, acutángulo, las alturas BH y AQ se interceptan en el punto E.
Si ÁE = BC.

Hallar la medida del Z BAC.

A) 30° B) 45 C) 60' D) 37 E) 53°

Solución

Por ser complementos del mismo


ángulo a, P = <(), entonces:
AAHE = ABHC

Luego AH = BH, lo que indica que el


A BHA es isósceles -» Z BAH = 45°

Z BAC = 45°

Rpta: ( B ).

9) En un triángulo ABC, isósceles, recto en B, hallar la distancia entre los pies de las
perpendicualres trazadas desde A y C, a una recta que pasa por B y corta a la hipotenusa,
sabiendo que A y C distan de dicha recta 5 y 12 unidades, respectivamente.

A) 13 B) 7 C) 8 D) 6 E) 9

Solución

- Sean : AQ = 5 y CH = 12
( AQ i r y CH _L r )

- Incógnita: HQ

- S i: BÁQ = a y

ABQ = p

a + p = 90°.

Luego: HBC = a ( complemento de p, en el ABC = 90° ).


y BCH => p ........... ( complemento de a, en A BHC ).

- Entonces : A AQB = A BHC ( Hipotenusa - Angulo agudo ).

BH = AQ -> BH = 5. HQ = BQ - BH
BQ = CH -> BQ = 12 I HQ = 12 - 5

HQ = 7

Rpta: ( B ).

176
10) En un A ABC, recto en B, el ángulo A mide 64°; M es punto medio de AC y E un punto de
BC, tal que BE = MC.

Hallar la medida del ángulo MEB.

A) 77° B) 66° C) 64' D) 76' E) 62

Solución
B
* Dato : BE = MC y AM = MC
BE = MC = AM
JK

Incógnita : BEM = x
Como, al trazar BM, es mediana
hacia la hipotenusa del A ABC :

BM = AM = MC .

A AMB Isósceles: ABM = A = 64° MBE = 26°


A MBE, isósceles : a = x y 26° = 180°

2x + 26° = 180°

x = 77°

Rpta: ( A ).

11) En la figura.
Hallar la distancia entre las rectas
paralelas m y ñ, si PE = 3.

A) 3
B) 6
C) 9
D) 8
E) 5

Solución
x = ?

- Para usar la propiedad de la bisectriz, trazamos


PQ 1 n y PR x m

PQ = PE ( AP, bisectriz del A ) -> PQ = 3


PR = PE ( BP , bisectriz de! B ) -» PR = 3

Entonces: x = PQ + PR = 3 + 3 x = 6

Rpta: ( B ).

177
12) Exteriormente a los lados AB y BC , de un triángulo ABC , se dibujan los equiláteros
AEB y BQC, respectivamente. Hallar la medida del mayor ángulo que forman AQ y EC.

A) 90° B) 135' C) 150' D) 120' E) N.A

Solución
Consideremos el gráfico adjunto.

Incógnita: x
Sean : BÁQ = a

ECB = <(> y ABC = p

Luego, en el cuadrilátero no
convexo ABCO : x = a + p + <¡>... (1).

Pero, observamos que :


A EBC = A ABQ ( Postulado LAL : -h=3~, 60° + P , )•

Entonces : CEB = BAQ a‘ = a

En el A EBC : a’ + 6 0 ° + p + <(> = 180'

a’ + p + <(> = 120°

a + p + ó = 120°
Reemplazando, en (1): x = 120

Rpta: ( D ).

13) En un A ABC, equilátero, sobre la prolongación de CA se toma el punto E y se traza

EG 1 BC ( G en BC ), cortando a AB en F.

Si EF=18/3 y F G =8/3 ¡ hallar AB.

A) 26 B) 24 C) 36 D) 40 E) 34

Solución
AB = ?

A ABC -> Equilátero (dato).


Trazamos AQ J_ EG

En los triángulos notables de 30° y 60°

AAQF -> = 9y¡~3 -> AF = 18 AB = A F + FB

AFGB -> ^ - x / 3 = 8 /3 -> FB = 16 AB = 34

Rpta: ( E ).

178
14) El ángulo C, de un triángulo ABC, ( obtuso en B ), mide 36°. Se trazan las alturas BF y A E .
Si “M” es punto medio de AB, hallar la medida del ángulo EMF.

A) 72' B) 36' C) 144' D) 108 E) 136

Solución

Incógnita: EMF = x

- En el A AEC :

EÁC = 90° - C -> EÁC = 54

- Por propiedad de la mediana


relativa a la hipotenusa :

A AEB EM = AM = MB A AME, isósceles


A AFB FM = AM = MB A AMF, isósceles

Luego : EÁF = a + P -» a + p = 54 ( 1)
- En el cuadrilátero no convexo EAFM :
x = Á + a + p

C o n (1 ) : x = 54° + 54° x = 108'

Rpta: ( D ).

15) En un A ABC, Á = 15°, C = 30° y AB = 8. Hallar AC.

A) 16 B) 8 / 2 C) 24 D) 15 E) 8 / 3

Solución
AC = ? H

- Como el ángulo externo B, mide 45°, trazamos


la altura AH. Así, en el A AHB (45°):

AB 8
AH = HB = AH = 4 / 2 = HB

- Pero, en A AHC (30°):


AC = 2 ( AH )
AC = 2 ( 4 / 2 )

AC = 8 / 2

Rpta: ( B ).

179
16) En un A ABC, AB =12, AB = 12 y AC =16, “M” es punto medio de BC. Se traza
BP, perpendicular a la bisectriz interior del ángulo A, ( P en dicha bisectriz ). Hallar PM.

A) 2 B) 3 C) 4 D) 1 E) 1,5

Solución
PM = ?

- Prolongamos BP hasta cortar a AC en R.


El A ABR, resulta isósceles por ser AP
altura y bisectriz a la vez.

AR = AB AR = 12. También : BP = PR

- En el A BRC : RC = 18 - 12 -> RC = 6
RC 6
y PM es Base Media : PM = PM = 3

Rpta: ( B ).

17) En un A ABC, AC = 10, hallar la longitud de la mediana A M , si: MÁB = 47° y MAC = 86 '

A) 4 B) 6 C) 8 D) 5 E) 10

Solución
- Incógnita: AM
- Tracemos :MN ÁCIJ

Por el Teorema de los puntos medios :


(Base Media)
AC
MN = MN = 5

- Se observan : NMA = MAC

NMA = 86°

Luego, en el A A M N : AÑM = 47'

A AMN es isósceles : AM = MN AM = 5

Rpta: ( D ).

18) En la figura adjunta :


BC = 6 y CD = 3 ^ 3
Hallar AB

A) 15 B) 12 C) 9
D
D) 18 E) 13

180
Solución
AB = x = ?
Prolongamos AB y DC, hasta cortarse en
el punto H. Deducimos fácilmente, que :
H = 90°

A BHC, notable ( 30° y 60° )

BH = — _> BH = 3
2

y HC = • í y - x </ 3 —> HC = 3 / 3

En el A AHD, también notable de 30° y 60°:


AH = ( HD ) J 3 , HD se opone al D = 60°

AH = ( 6 / 3 ) / 3 AH = 18

Entonces : x = AH - BH
x = 18 - 3 -» x = 15

Rpta: ( A ).

En la figura :

CM = MB y AB = 8.

Hallar CD.

D A) 4 B )4/3 C) 4

D) 8 E) 8 / 2

Solución
CD = ?
Trazamos BF y CE, perpendiculares
a AD. Luego :
Dr AB
A AFB( 30° ) -> BF = — C

/. BF = 4
ACEM = A B F M ^ CE = BF
CE = 4

A CED( 45° ) : CD = ( CE ) yp2

CD = 4 / 2

Rpta: ( C ).
20) En la figura: m || n

AB = BD ; CD = 4
y C = 45°
Hallar la distancia entre m y ñ

A) 2 / 2 B) 2 C) 3

D) 3 ^ 2 E) 4

Solución

* Trazamos DE 1 n
DE da la distancia entre m y n.
* El A ABD, es isósceles (dato : AB = BD)
BÁD = BDA = a

* Por ser alternos internos entre paralelas :


BDA = DÁE

DÁE = a

* AD es bisectriz del ángulo BAE. Trazamos DH X AC


Propiedad de la bisectriz : DE = DH

Siendo en el A CHD : DH = -?=- = 2 J~2 DE = 2 / 2


/2 V

Rpta: ( A ).

( n iv e l II)

21) En un triángulo ABC, la mediatriz de AC, corta a BC en el punto N. Luego, la altura BH corta
a AN en el punto E. Si: AE = 3 y BC = 13; hallar BN

A) 4 B) 6 C) 7 D) 5 E) 3

Solución
BN = x = ? (Incógnita)

Analicemos los á n g u lo s:

* Por propiedad de la mediatriz : NC = NA


A ANC, isósceles :

NÁC = C = a

* Si en el A AHR : ARH = ó ( a + § = 9 0 °);

182
entonces, para el A BHC : HBC = 909 - C = 90( a

HBC = <(>

- Luego, el A BNR es isósceles : RN = BN RN = x

- Por último, tenemos : BN + NC = BC


i
BN + AN = BC

x + ( 3 + x ) = 13

De donde : x = 5

Rpta : BN = 5 ... ( D ).

22) En un triángulo ABC, recto en B, la altura BH corta a la bisectriz interior AD en el punto E


Si BE = 8, hallar la distancia del punto medio de DC, a AC.

A) 8 B) 4 C) 6 D) 5 E )2

Solución

- Sea M, punto medio de DC y MQ 1 AC


B
/. Incógnita: MQ = x

- Sabemos que, para todo triángulo


rectángulo : ABH = C

- Se observa, que : A BED, isósceles


BD = BE BD = 8

Trazamos : DP _L AC
Por propiedad de la bisectriz : DP = DB -> DP = 8
DP
En e I A DPC, por ser Base media : MQ =

8
MQ = -

MQ = 4

Rpta: ( B ).

23) Dado el cuadrado ABCD, de lado “I”, se dibujan los triángulos equiláteros : A AED (interior)
y ACFD (exterior). Las prolongaciones de AE y FC se interceptan en el punto P.
Hallar la distancia de P a E F .

l/2 l/2 1 /2 1 /2 1/2


A) B) C) D) E)
8

1 O''
Solución
* Incógnita: PQ
* Del gráfico, el A EDF es rectángulo e
isósceles, por lo que EF = \^J~2.

* Además en el A EPF, donde Z EFP = 15'


y APEF = 75°, recto en P, PQ es altura
relativa a la hipotenusa :
EF
PQ =

PQ = 1 /2

Rpta: ( A ).

24) En un A ABC, obtuso en B e isósceles, sobre los lados AB y AC se toman los puntos E
y F, respectivamente, de modo que AE = FC y AF = BC.
S I : Z FBC = 27°. Hallar la medida del Z EFB.

A) 27° B) 42’ C) 30 D) 45 E) 60

Solución

* Incógnita : EFB = x AB<


Del gráfico, notamos que:
A AEF = A CFB ( LAL ).

Luego : Z AFE = CFB AFE = 27°


También, por ángulo externo en el ABFC
Z BFA = 27° + a
x + 27° = 27° + a x = a

Finalmente en el A ABF, isósceles : ó = 27° + x


y: oc + (j) + ( 27° + x ) = 180

De donde, con lo a n te rio r: x = 42°

Rpt a: EFB = 42o ... ( B ) .

A ADB, AAFC. y A BEC son triángulos equiláteros;


calcular DFE, si el ángulo ABC es recto.

A) 120' B) 135' C ) 145°


D) 150 E) 160'

184
Solución

Incógnita : DFE = 60° + x + y (1 ).

Como : DÁF = 60° - FÁB


y cc = 60° - FÁB

( ya que los triángulos ADB y AFC,


son equiláteros : DÁB = 60° = FÁC )

Entonces : DÁF = a => A DAF = ABAC ( L A L )

Análogamente : FCE = 60° - BCF y ó = 60° - BCF

FCE = $ A F E C = A ABC ( L A L )

Además, en el A ABC : ó + a = 90°

Finalmente, en ( 1 ) : DFE = 60° + x + y


DFE = 60° + Ó + a = 60° + 90° = 150

Rpta: (D).

En la figura ;
AE = E C ; AE 1 EC;
AB 1 BC ; ED 1 DC.
Si BC = 3 y ED = 5,
Hallar AB

A) 7 B) 8
D) 10 E) N.A
D
Solución
AB = ?

Trazamos AQ, perpendicular a la prolongación


de DE. Entonces, S i : ECD = <J> y CED = a
( a + ó = 90° ); se obtienen :
AÉQ = ó ( complemento de a ).

y QÁE = a ( complemento de ó ).

- A AQE s A EDC ( ALA ) AQ = ED -> AQ = 5

Enseguida: CD = BD - BC ; Pero BD = AQ = 5
CD = 5 - 3 -> CD = 2
lu e g o : QE = CD —> QE = 2

- Finalm ente: AB = QD AB = QE + ED

AB = 2 + 5 -> AB = 7

Rpta: ( A ).

27) Demostrar que en todo triángulo, el segmento que une los pies de las perpendiculares
trazadas desde un vértice a las bisectrices de los ángulos exteriores correspondientes a los
otros dos vértices, mide igual que el semiperímetro del triángulo.

Solución

En efecto, demostraremos que para el A ABC,


siendo BM y BN, las perpendiculares :

AB+BC+AC
MN =

Para ello prolongamos BM y BN hasta


interceptar a las prolongaciones de AC,
tal como en la figura.

Luego :
A EAB, isósceles, por ser AM altura y bisectriz
EA = AB y M punto medio de EB.

A BCF, isósceles, por ser CN altura y bisectriz


CF = BC y N punto medio de BF.

Finalmente en A EBF, MNes base media

EF EA+AC+CF
MN =

AB+AC+AB
De donde : MN =

l.q.q.d.

28) En un A ABC, AB = 5 cm, BC = 7 cm, AC = 9 cm, por el vértice B se trazan BL y B S ,


perpendiculares a las bisectrices, interior del Á y exterior del C, respectivamente ( L y S
en dichas bisectrices).

Hallar LS.

A) 6 B) 4 C) 5 D) 5,5 E) 6,5

186
Solución
MN = ?

A RAB, isósceles:
AM es altura y bisectriz
AR = AB = 5 y BM = MR

A BCE, isósceles:
CÑ es altura y bisectriz
CE = BC = 7 y BN = NE

A RBE, Base media :


RE
MN = MN = 5,5

Hallar DN:

Si AB = BC, AD = 2 cm,
CN = 5 cm y BE = 8 cm

A) 10 cm B) 15 cm C ) 13 cm

D) 9 cm E) N.A

Solución
Incógnita: DN

Por B, trazamos paralela a D N .


a = a ’ , <f> = <J)‘
( lados perpendiculares). a'
'v y
/Jk n f 3
AAR B = A B S C : RB = 3 , BS = 6 c
V
1/
L uego: 8
DN = D E + EN 5
1 n
DN = RB + BS N

• • DN = 9

Rpta; ( D ).

En la fig u ra :
AD = BC; BD = C D y el ACDR es equilátero

Hallar V

A) 20° B) 40‘ C) 30
D) 35° E) 45'
187
Solución

Trazamos BR. Luego :


A B RC = A ABD ( L A L ).
a = x

A BDR, isósceles : DBR = 20° + a = BRD

2 (20° + a ) + 80° = 180° a = 30'

X = 30°

Rpta: ( C ).

ABC, es un triángulo tal que Á = 24° y C = 29°.

Exteriormente y relativo a AC, se toma el punto E, siendo : EAC = 24° y ECA = 21


S i: BC = 5 . Hallar EC.

A) 4 B) 3 C) 3 / 2 D) 4^~2 E) 5

Solución
Incógnita : EC = ?

Observando los ángulos exteriores en B y E de


los triángulos ABC y AEC, concluimos que son
notables y para usarlos en triángulos rectángulos,
trazamos : CH 1 AB y CQ _L AE.

A s í: A BHC ( 53° ) HC = 4

Por Propiedad de la bisectriz, para el ángulo A : Q

CQ = CH CQ = 4

Finalmente, en el A EQC (45°) : EC = C Q .J 2

EC = 4 f 2

Rpta: ( D ).

En un triángulo acutángulo ABC, las alturas AF, BG y CH se cortan en O. Se toman M


L puntos medios de AO y BC respectivamente.
El triángulo MGL es :

A) Acutángulo B)Obtusángulo C)Rectángulo


D) Isósceles E) Equilátero
Solución
Del gráfico:
A AGO : GM AM = OM
v A BGC : GL BL = LC

( Por propiedad de la mediana hacia


la hipotenusa, en estos triángulos
rectángulos).

Luego, si FÁC = a y FCA = $ -» AGM = a y LGC = <)>

En el A AFC : a + ó = 90°
y en G : a + ó + MGL = 180°

Con lo a n te rio r: 90° + MGL = 180° MGL = 90

Rpta: El A MGL es rectángulo ... ( C ).

33) En la figura :

m | n y r
Hallar x, s i :
PQ = 12 y AB = 5

A) 13 B) 16 C) 18
D) 17 E) 19

Prolongamos A B , hasta cortar a r en R

Entonces : BR = PQ BR = 12.
4-»
Trazamos AT 1 n . Por propiedad de la bisectriz :

AT = AR
I
x = 5 + 12 x = 17

Rpta: ( D ).

34) En la figura :

AC = 13 y BC = 12

Hallar EF.

A) 4 B) 5 C) 7
D) 8 E) N.A

189
Solución
Incógnita : EF = x

En el A ABC, Teorema de Pitágoras :

Á B 2 + BC2 = ÁC2

Á B 2 + 122 = 132 - * AB = 5

Trazamos FH _L CB y F Q 1 A B .

Por propiedad de la bisectriz : FH = FE .*. FH = x (Para el ángulo HCE ).

A d e m á s: BQ = FH BQ = x.

También, para el BÁC, por ser ÁF* bisectriz :


AQ = AC 5 + x = 13 x = 8
Rpta: EF = 8 ... ( D ).

En un A ABC, B = 127a, “M” es punto medio de ÁC.Se traza MQ 1 BC, ( Q en B C ).


Hallar MQ, si BQ = 3 y QC = 6.

A) 2 B) 4 C) 3 D) 1 E) 2.5

Solución

MQ = ?

Como B = 127° el ángulo


exterior B, mide : 180° - 127° = 53°.
Por ello, es conveniente trazar la altura A H .

Luego;
A AHC -» MQ es Base Media. HQ = QC = 6
AM
MQ = - ............ ( 1) HB = HQ - BQ

HB = 3

En el A AHB ( 37° y 5 3 ° ) : AH = 4. Reemplazando esto, en (1 ) :

MQ = — -> MQ = 2
2
Rpta: (A).

En un A ABC. se traza la mediana A M . En el A ABM, se traza la mediana BQ . F, es punto


de ÁC. tal que MF II BQ. Si BQ = 12. hallar MF.

A) 8 B) 12 C) 6 D) 15 E) N.A
Solución
* Incógnita: MF = x

Prolongamos BQ hasta R. QR es base

media en el A AMF : QR = — -> QR = -

En el A RCB, MF es base media relativa a RB :


x
>.r- RB
MF = — —» x = -------- 2

Resolviendo : x = 8

Rpta MF = 8 ... ( A ) .

37) En la base de un triángulo isósceles ABC, ( AB = BC ) , se toma un punto cualquiera P,


y se trazan PE 1 AB ; PF 1 BC.
Si AH es altura, demostrar que :

Demostración
Sea el gráfico :

AB = BC

A = C
Vamos a demostrar lo propuesto, haciendo un trazo. Para ello, usamos el siguiente gráfico,
que es el mismo anterior indicando el trazo a re a liza r: AQ 1 FP.

E ntonces:
QÁC = C ... ( alternos internos,
ya que AQ 11 BC )

Luego: A A QP = AA EP
PQ = PE

A s í : AH = QF
AH = QP + PF
i
AH = PE + P F ; con lo cual queda
demostrado.

38) En la prolongación de la base AC de un triángulo isósceles ABC, se toma un punto cualquiera


P y se trazan PE 1 AB y PF 1 BC.
Si CH es altura, probar que:

191
Solución

Consideremos el gráfico adjunto.


AB = BC.

Para demostrar lo propuesto,


trazamos CR ± EP

CR I AB -> RCP = A ...( correspondientes )

A CRP = A CFP-> PR = P F ( 1 ).

L uego: CH = RE

Es decir : CH = PE - PR . Entonces, con ( 1 ) :

CH = PE - PF l.q.q.d.

39) En el interior de un A ABC , equilátero , se toma un punto cualquiera “P” y se trazan


PE 1 AB, PF _L BC, PR _L AC. Si “h" es la longitud de la altura del A ABC; demostrar,
que :

Solución
Sean :

Fig.1 : Gráfico del enunciado

Fig.2 : Indica el trazo a efectuar, para la demostración : MN II A C .


Entonces : BMN = Á = 60° y MÑB = C = 60° En el A MBN, equilátero, por la
propiedad vista en el problema anterior mediato :h 1 = PE + PF; ( h 1 estambiém la distancia
de M a BN ).

Luego : h = h1 h- PR h = PE + PF + PR
l.q.q.d.

40) En la figura, el A ABC es equilátero y P cualquier punto interior a la región común de los
ángulos exteriores B y C.

Demostrar q u e :

* h Longitud de la altura del A ABC

192
Solución
M
Por P, trazamos paralela a B C , cortando las
60
prolongaciones de AB y AC, en M y N,

respectivamente.
B
A AMN, equilátero. u A
> P

/6CT
En el A ABC, trazamos la altura desde A, (h). h

Para el A AMN, la altura trazada desde A, tiene /


b... J ''u 1
longitud h + QP ; esta debe ser igual a PE + PF, N

según propiedad anterior. Luego :


h + QP = PE + PF
h = PE + PF - PQ
I.q.q.d.

En un A ABC, recto en B, AB = 20; por A, se traza A Q , perpendicular a la bisectriz del ángulo


ACB. ( Q, en dicha bisectriz). Si “M” es punto medio de C Q , hallar la distancia de “M” a B C .

A) 10 B) 4 C) 5 D) 3 E) 9

Solución
Incógnita : MN ( MN X BC j

Prolongamos CB y AQ, hasta su intersección


en “R”, formándose el A ACR isósceles, por ser
CQ altura y bisectriz AC = CR y AQ = QR

Luego, trazamos QH X BR

Entonces, por base Media :

AB
A ARB : QH = QH = 10

A QHC : MN = — = — -> MN = 5
2 2

Rpta: ( C ).

En la figura
HÓR mide 120° y OQ es su bisectriz.
S i: PQ = 8 y
PH = 24

H allar: PR
A) 32 B) 28 C) 26
R
D) 30 E) N.A

193
Solución

Por P, trazamos paralela a OR, cortando


a OH y OQ, en A y B, respectivamente.
Entonces, en el A AOB, equilátero, la lon­
gitud de la altura es igual a PH + PQ ,
( propiedad del problema nc 3 7 ) e igual a
PR.
PR = PH + PQ
PR = 24 + 8 PR = 32

Rpta: (A).

43) En un A ABC, recto en B, BM es mediana y BH altura. P es un punto de B C , distante 3


y 4 unidades de BM y A C , respectivamente. Hallar BH

A) 5 B) 7 C) 9 D) 6 E) 8

Solución

BH = ?
B
Como el A MBC, es isósceles ;
( Propiedad : BM = AM = M C ) y
P un punto de su base BC :

BH = 3 + 4
( Propiedad del problema numero 37 )

BH = 7

Rpta: ( B ).

44) En la figura:

Hallar EH.si:

DG = DE, AF = 4m y AC = 9m

A) 5m
B) 2m
C) 3m
D) 4m
E) 1,5m

194
Solución
Incógnita: EH

Como, según dato : DG = DE, entonces


el AG D E es isósceles DGE = DÉ G—■(1)
Además, las alturas GQ y EH serán
congruentes entre sí.

Bastará hallar GQ
Tracemos, GR _L AC
Q E
Luego : RGA = DÉG (2 )

D e ( 1 ) y ( 2 ) : D G E = RGA .Pero,también,porseropuestos: AGF = DGE

Finalmente : A ARG = A AFG AR = AF AR = 4

Entonces: RC = AC - AR = 9 - 4 RC = 5
y GQ = RC GQ = 5 = EH

Rpta: ( A ).

NIVEL III

45) Exteriormente a un triángulo ABC, se dibujan los cuadrados ABDN y BCEF. Demostrar
que AF y CD son congruentes y perpendiculares.

Solución

* Según el gráfico :
DBC = ABF = 90° + p
•*

A DBC s A ABF ( Postulado LAL )

Luego : a = w y

AF = CD

* En cuadrilátero cóncavo ABCX : x =a + p + ó


x= w + p + ó

* Pero, en el A DBC :
w + p + ó = 90°

* Entonces : x = 90°.

Es decir, AF y CD son congruentes y perpendiculares.

195
46) Exteriormente a un triángulo ABC, se dibujan exteriormente los cuadrados ABDN y BCEF,
de centros O y P. Si MM" biseca AC, demostrar que OM y MP son congruentes y
perpendiculares.

* D em ostración

* Con los trazos indicados en la figura


y de acuerdo al problema a n te rio r:

AF = CD
( 1 ) AF _L CD

* Por el teorema de los puntos medios :

CD
A ADC OM CD y OM = .\ OM = MP
2
y a = 90° ,
AF
A AFC MP || AF y MP = Por (1)

47) Exteriormente a un triángulo ABC, se dibujan cuadrados de centros “O” ( sobre AB ), “P" (
sobre BC ) y “Q” ( sobre AC ).
Demostrar que OP y BQ son congruentes y perpendiculares.

Solución
A fin de aprovechar el problema anterior,
tomamos M, punto medio de AB
PM = MQ y PMQ = 90°

Además : OM = AM = MB
Luego, A O M P = A BMQ
( Postulado LAL )

También : e = x + a x + a = 90° + ó
e = 90° + ó

De donde, OP y BQ son perpendiculares y congruentes.

48) En la figura :

AH = 5 y HC = 13

H allar: PQ

A) 4 B) 6 C) 12
D) 7 E) 5

196
Solución

PQ = ?
Empezamos la solución indicando
variables para los ángulos A y C.
A A

Sean : A = a y C = <¡>

( a + ó = 90°. por ser B recto ).

Sabemos que, por ser BH altura


del A ABC :
ABH = C = 0

Además, como en todo rectángulo las diagonales son congruentes y se cortan en su punto
medio :
□ MBNH -> OM = OB = OH = ON.

A MOB : Isósceles. Entonces : OMB = OBM = <|>

En el AMBN, BE es altura hacia la hipotenusa MN.

EBN = NMB -» EBN = . En consecuencia, el A BQC es isósceles, por que

C = <t> = QBC ■Luego : BQ = QC . Además ABO es complemento de <j>; es decir

ABQ = a (y a que a + ó = 90° ).

Esto indica que el A AQB es isósceles :

A s í : AQ = BQ = QC = — -4 AQ = BQ = QC = — = 9.
2 2
y HQ = AQ - AH = 9 - 5 HQ = 4

Por último, como NHC = Á = a y BPN = 90° - ó = a

NHC = BPN -» NHC = HPQ El APQH es isósceles :

PQ = HQ -> PQ = 4

Rpta: ( A ).

En un triángulo ABC, recto en B, la hipotenusa AC mide 4 cm. y el ángulo C : 22,5°. Hallar


la longitud de la alura B H .

A) 1 B) 0,5 C) 2

D )/3 E )^ 2
197
Solución
BH = ?
B
Trazamos BM, mediana relativa a la
hipotenusa. Entonces, por propiedad de
esta línea en los triángulos rectángulos :
AC 4
BM = = -------> BM = 2.
~ 2
- Además; el A BMC es isósceles :
MBC = C = 22,5°. Luego:

AMB = MBC + C ... ( Z e xte rio r) AMB = 45°

- Finalmente, en el A B H M ( 4 5 ° ) : BH = ^ = J~2 BH = -J~2


J2 J2

Rpta: (E).

Nota.- En general, para un A ABC, donde


B = 90° y C = 22,5°:

50) En un A ABC, de lados, AB = c, BC = a y AC = b, hallar la distancia entre los pies de las


perpendiculares trazadas desde el vértice B, a las bisectrices de los ángulos interiores A
y c.
Solución
* Incógnita: MN

* Prolongamos las perpendiculares


BM y BN hasta E y F, luego : ABAF,
isósceles, por ser AN altura y bisectriz
AF = c y N es punto medio de BF.
A BCE, isósceles, por CM altura y bisectriz
CE = a y M el punto medio de BE.
EF
* En el A EBF, MN es Base media : MN =

Siendo : EF = a + c - b a + c - b
MN = ( i )

Mejor aún, recordando que a + c + b = 2p , ( perímetro del A ABC )


-> a + c = 2p - b

Reemplazando en la expresión ( 1 ) : MN = — —^ — —

Es decir: MN = p - b , Siendo p el semiperímetro del A ABC y MN || AC

198
En un A ABC, se traza la mediana BM . Luego en el A ABM se traza la mediana AN que
prolongada intercepta a BC en el punto E.

Demostrar q u e :
a) EC 2BE
t>) AN 3NE

Solución

Del gráfico, trazamos MF paralela a A E .


Por teorema de los puntos medios :
A BMF -> BE = EF ( 1 )-

y MF = 2 N E (2).

También, en el A AEC -» E F=FC ...(3).


y AE = 2 M F (4).

Luego: a) De ( 1 ) y ( 3 ) : BE = EF = FC

b) De ( 4 ) : AE = 2MF, con ( 2 ) : AE = 2 ( 2 NE )

AE = 4NE

En un triángulo ABC, se trazan la mediana BM y la latura AH. Calcularla medida del MBC,
s i : AH = BM ( H sobre BC ).

A) 60° B) 30' C) 37° D) 45 E) 53'

Solución
Dato : AH = B M (=a)
B
Incógnita : MBC = x

Tracemos MQ _L BC
MQ 11 AH . Luego, MQ es Base
Media en el A AHC, relativa a AH:
AH
MQ = MQ = —
2
BM
Entonces, en el A BQM, como MQ = x = 30'

Rpta: ( B ).

En un A ABC, B = 90°, BH es altura y BH = 6. Hallar la distancia entre los pies de las


A A

perpendiculares trazadas desde “H” a las bisectrices de ABH y HBC.

A) 3 B) 6 C) 2 D) 2 / 2 E)3/2

1 QQ
Solución
MN = ?

BH =t> B

Del g rá fico :

A EBH, isósceles, por ser BM


altura y bisectriz:
EB = BH = 6 y EM = MH

A HBF, isósceles, por ser BN


altura y b ise ctriz:
BF = BH = 6 y HN = NF

A EBF, recto e isósceles : EB = BF = 6 -> EF = 6 / 2

A EHF, por base media : MN = — -> MN = 3 / 2


2

Rpta: (E).

S i : BC = CA y AB = CD.

Calcular el valor de
B

A) 30°
B) 22°
C) 37°
D) 15°
E) 45°

Solución
Sean : AB = a = CD
Trazam os: CQ 1 AB y CH _LAD. B

Como el A ACB es isósceles


(dato : BC = CA)
g
Entonces : AQ = QB =
2

L u e g o : CH = A H

Finalmente, en el A CHD, el cateto CH mide


la mitad de la hipotenusa CD x = 30°

Rpta: ( A ).
55) En un triángulo ABC se traza la altura BH ( H en AC ), s i : AH = 4m, HC = 9m y
BÁC = 2 ACB. Calcular la medida del ángulo C. ( Aproximadamente ).

A) 16 B) 15' C) 20' D) 18° 30' E) 22° 30'

Solución

C = x = ? A = 2x.

Trazamos BE , de modo que:


BÉA = Á -» BÉA = 2x
El A ABE es isósceles
/. HE = AH -» HE = 4

Luego : EC = 5 y se deduce que:


EBC = x, resultando isósceles el
A BEC : BE = EC BE = 5

Finalmente, en el A BHE : Longitud de la hipotenusa = 5


y longitud de un cateto = 4 2x = 37°
x = 18° 30*

Rpta: ( D ).

56) En un triángulo ABC, recto en B, la altura BH mide 16u y Q es un punto de BC, tal que
AQ = 1 9 u y BAQ = ACB. Hallar la distancia de Q a AC.

A) 12u B) 13u C) 14u D) 15u E) N.A

Solución

* QF 1 AC ( distancia de Q a AC ) a

* Incógnita: QF
* Prolongamos AB y FQ, hasta R. B
3
* Sea : ACB = a -» BÁQ = a /1
y sea p, complemento de a en AQFC y 19
¿ p-ys
Luego : BQR = P y R = a 1 -1 «ft .

* A AQR, es isósceles.
Entonces : QR = AQ = 1 9 y AB = BR

Finalmente, observamos que BH es Base Media relativa a RF, en el AAFR


Así: RF = 2 ( BH ).
19 + QF = 2(16).
De donde : QF = 13

Rpta: ( B ).

201
57) En la figura

AB 1 BC ; AB = BC;
AE 1 EB y EAB = ECA

Hallar EC,
Si BE = 2.

A) 3 B) 4 C) 2 / 3

D) 3 / 2 E) 2 / 2

Solución

EC = ?

* Se observa : EBC = EAB = a

* A fin de obtener un triángulo congruente


al AEB, trazamos CH 1 BE.

Luego : A BHC = A AEB.


CH = BE CH = 2.

De otro lado : ACH = EÁB = p.

S ie n d o : BÁC = a + p = 45°;
entonces : ECH = a + p = 45°.

Así, en el A EHC, tendremos : EC = CH J~2 EC = 2 / 2

Rpta: ( E ).

58) En un A ABC, la mediatriz de AC corta a la bisectriz exterior del ángulo B, en el punto UP".
Hallar la medida del ángulo ACB, si : PÁB = 10° y PAC = 39°.

A) 49° B) 29' C) 39' D) 19° E) N.A

Solución
- Incógnita : x

- Trazamos P C . Por propiedad de


la mediatriz : PC = PA
en el A APC, isósceles : PCA = PAC
x + n = 3 9 ° ............( 1 ).

- Para usar la propiedad de la bisectriz,


trazamos ; PH _L BC y PQ ± AB
202
PH = PQ
- Luego : A PHC es congruente al A PQA n = 10'

- En ( 1 ) : x + 10° = 39° -> x = 29°

Rpta: ( B ).

59) En la figura adjunta:

AC = 12cm.

Hallar PQ.

A) 12cm B) 6cm C) 8 cm

D) 9cm E) 5cm

Solución

* Dato : AC = 12

* Incógnita: PQ
Con los trazos indicados :
A DBC, Isósceles ( BP es altura
y bisectriz).
DP = PC.

Propiedad de la bisectriz :

PM = PQ D'-

AC 12
Y en A DAC, PM es base media : PM = = 6

PQ = 6

Rpta: ( B ).

60) En la figura mostrada :

AB = BC ; AB 1 BC , AC 1 AE
AC = AE

Hallar el valor de “x”.

A) 30°
B) 15°
C) 37°
D) 18° 30'
E) 26° 30'

203
Solución

Sean AB = BC = a. Trazamos
EH 1 AB /. AAHE = A ABC

Luego : AH = HE = a.
En el triángulo rectángulo BHE, los
catetos son entre sí como 1 a 2.
53
Entonces : a = = 26°30’

y: 45° + x = 90° - a
45° + x = 90° - 26° 30

De donde : x = 18° 30'

Rpta: (D)

61) Del gráfico.

Hallar el valor de “x'

B S i : DC = 2BD.

A) 10°
B) 26,5°
C) 30°
D) 18,5°
E) 15°

Solución

Sea BD = a DC = 2a
prolongamos CB hasta el punto E,
de modo que EA _L AC. Entonces:
EÁB = C ( lados perpendiculares )
EÁB = x.

El A EAD, resulta isósceles


-> EB = BD y EA = AD

En el A EAC, AD es mediana relativa a la hipotenusa E C .


EC
Luego, por propiedad : AD = AD = 2 a y el A EAD resulta equilátero: 2x = 60'

x = 30’

Rpta: ( C ).

204
62) En la figura :
AQ = QN = NC y RQ = 2.
Hallar AP

A) 8
B) 10
C) 9
D) 12
E) 14
Solución

Se prolonga NP y AB , hasta cortarse


en “M”. El A APM resulta isósceles
porque PB es bisectriz y altura.
Entonces: BM = AB y MP = AP.
Luego, BQ es base media d e l :
A AMN BQ || MÑ.
También RQ es base media del
A ANP NP = 2 ( R Q ) -> NP = 4

En el A QBC : BQ = 2 ( NP ) , ( NP base media ) BQ = 8


(
A s í : BR = BQ - RQ = 8 - 2 BR = 6

Finalmente en el A APM : MP = 2 ( BR ) MP = 12 AP = 12

Rpta: ( D ).

63) En la figura : AM = MC.


EM | BF ; EG = 3 y GM = 4

Hallar AG.

A) 10
B) 12
C) 13
D) 11
E) 14
Solución
Del gráfico ; É = FBC = a
A EBG, isósceles
BG = EG = 3

Trazamos AR II E M . d e modo que EM sea


base media en el AARC, ya q u e AM = MC

205
AR = 2 ( EM ) AR = 14

Además : R = É = a ( Z correspondientes ).

Finalmente, en el A ARB, isósceles : AB = AR


AG + 3 = 14
« • AG = 11
M F
Rpta: ( D ).

64) En la figura:
A ABC y A CDE, son equiláteros
BD = 18 y BM = EM.
B Hallar CM.

A) 18
B) 12
C) 8
D) 9
E) 6
Solución
Incógnita : CM = x

* Por E, trazamos ER I CM.afin


de aprovechar el dato BM = EM.
Luego, por el Teorema de los pun­
tos medios , en el A BRE :
CR = BC y ER = 2 ( C M ) ER = 2x
CVeo* :
* Se observa que A ECR = A DCB
2x
( Postulado LAL )
ER = BD R
2x = 18 -» x =9

Rpta: CM = 9... ( D ) .

65) En la figura: q

Los triángulos ABC y CDE son equiláteros


M biseca AD y N biseca BE.

Demostrar que el triángulo MCN


es equilátero.

206
Solución

* Bastará probar que CM * CN y p + <J> = 60°

* Así tenemos : BCE = ACD = 120°


A BCE = A A C D (LAL).
Entonces BE = ADy
B
CN = CM, por ser CN y CM medianas relativas
a lados congruentes BE y A D .

También : AM = BN
A AMC = A B N C ( LLL ).
a = <(>

y como p + a = 60° p + $ = 60°


y esto es suficiente, porque el A MCN, isósceles,
A

tiene MCN = 60°, resultando equilátero.

66) En un A ABC, B = 105° y C = 30°, se traza la mediana AN. H allarla medida del NAC.

A) 10o B) 15o C) 20° D) 30° E) 5o

Solución

NÁC = a = ?

Sea BH 1 ÁC. B 105°

Trazamos HN. Con el A BHC :


HN = BN = NC
( propiedad )

A NHC, isósceles : NHC = 30°


A BHN, equilátero: HB = HN = BN.

El A A H B es isósceles : AH = HB AH = HN

Finalmente, en el A AHN, isósceles : a = 15°

Rpta: ( B ).

67) En un A ABC, N es punto medio de BC,Q es un punto de AC, próximo a Ay M un punto


de QC, tal que AQ =Q M y MC = AB. Hallar la medida del AÑQ, si B = 105° y C = 30°

A) 10° B) 7,5° C) 15° D )5° E) 12°

207
Solución

BÁC = 45°
Del gráfico, según el problema
B
anterior, se sabe que a = 15°.

Prolongamos CA hasta R, de
modo que AR = AB. Entonces

R = RBA = -5^5. = — = 22,5°

En el A RBC, se observa :
RQ = QC y BN = NC.
Luego, QN es base media : QN 11 RB NQC = R
NQC = 22,5'

Finalmente, en el A ANQ, por ángulo externo :


a + x = NQC
15° + x = 22,5° x = 7,5'

Rpta: NQC = 7,5o ... ( B ) .

68) En un triángulo ABC, recto en B, BC = 4, BM es bisectriz interior. Por “P , punto medio


de MC, se traza paralela a A B , cortando a la prolongación de BM en el punto “Q”.
Hallar FQ. \

A) 1 B) 2 C) 3 D) 4 E) V~2

Solución
* FQ = ?

* Por “C", trazamos paralela a F Q , cor­


tando a la prolongación de MQ en “H".

* Como HC 11 AB -► BCH = 90‘


A BCH , es isósceles.

Luego : CH = BC CH = 4

* En el A MCH, por ser Base Media :


CH 4
FQ =
2 “ 2
FQ = 2

Rpta: ( B )

208
69) En la figura :
AE = E C = BC
B
Hallar la medida del ABC, en función de V

A) 4r
B) 90° + 2r
C) 120o - r
D) 120°- 2 r
E) 120°+ r

Solución

Trazamos BE, CH 1 BE y EF 1 AB :
BH = HE = a
A AFE ~ A EHC -> EF = a
En el A BFE : BE = 2 EF

L u e g o : FBE = 30°

Además : HBC = 90° - r, en el A HBC


Entonces : ABC f FBE + HBC
ABC = 30° + 90° - r
ABC = 120° - r ( fórmula)
Rpta: ( C ).

70) En la figura : AB = BC,

AE biseca el Á , BM || AC y EM 1 BM

Hallar EB, si AB = 12 y BM = 3

A) 6
B) 4
C) 3
D) 5
A E )7

Solución 12

Incógnita: BE = x

* Prolongamos BM y AE hasta su intersección


A A

en P. Luego, P = PAC = a (alternos internos)


A ABP, isósceles : BP = AB = 12.

También : CBP = C = 2 a

209
* Enel A EBP, EBP = 2P = 2 a , trazamos la ceviana E R . d e m o d o q u e R = EBP = 2ot.

Luego, los triángulos REB y ERP son isósceles : ER = EB = x ;


RM = BM = 3 y RP = ER = x.

Finalmente: BM + MR + R P = BP -> 3+ 3 + x = 12 -> x = 6

Rpta: ( A ).

71) En la figura:
CD = 18.
B
Hallar BH.

A) 9
B) 6
C) 3
D) 2
A E) 8

Solución
BH = ?
- trazamos por C, CP perpendicular a B

la bisectriz del CÁD y la prolongamos


hasta su intersección con AD, en R.

Entonces :
A C P A = A APR s A ABC.
( Hipotenusa - ángulo agudo ) /\
D Q R
CP = PR = BC
p r > l o

- En el A CDR, trazamos PQ ± DR; PQ es Base media : PQ = ------ = — PQ = 9.


2 2

- Pero ; BH = PQ ^ ( alturas hacia lados congruentes en los triángulos con­


gruentes ABC y APR ).
BH = 9

Rpta: (A).

72) En la figura : BC = 3
Hallar CE.

A) 3
B) 3 / 2
C) 6
D )6j2
E) 4

210
Solución

Sea CH, perpendicular a la bisectriz


A

del CAD y que prolongada corta a


AD en F. Luego, por propiedad de la
b isectriz: CH = BC = 3.
En el A CAF, AH es altura y bisectriz
HF = CH = 3 y D

Por tener lados perpendicualres :


FCD = BÁH FCD = 2 a

Por el teorema del ángulo externo

A AEC -> CÉF = 2a + <f)


A CFD EFC = 2a + <)>

CÉF = EFC -» El A ECF es isósceles :

Rpta : -.(C ).

En un A ABC, se traza la mediana AM y luego BH 1 AM , ( H en AM ). S i: ABH


MAC = 2a y AB = 2 HM, hallar el valor de a.

A) 10° B) 30' C) 20 D) 18' E) 32

Solución

Sea HM = a AB = 2a

Para lograr que HM sea base media


en algún triángulo , aprovechando
que BM = MC, trazamos CQ, per­ ^ C

pendicular a BH. Luego, en el 2a

A BQC : BH = HQ y QC = 2 HM
—> QC = 2a.

Al trazar A Q . e l A BAQ resulta isósceles ya que AH es altura y mediana

AQB = ABH = a y AQ = AB = 2a

Finalmente, el A AQC es isósceles : QÁC = ACQ = 2 a


2a + (90° + a ) + 2 a = 180° a = 18°

Rpta: ( D ).
74) Del gráfico :

A Hallar el valor de “x”, si AB = CD

A) 31°
B) 31° 30'
C) 31° 45'
D) 32°
E) 32° 30'

Solución

* Trazamos DE 1 AC , cuya prolongación


corta en H a AB prolongada. Luego :
CDE = BÁC = x, resultando isósceles
el A ADH, y a q u e DB es altura y
bisectriz del ADH BH = AB.
Al trazar HC : AHC = HAC = x.

* Se traza BM 1 AC , resultando con­


gruentes los triángulos BMA y CED H
( hipotenusa-ángulo ) BM = CE = a

* BM es base media en el A AH E-> HE = 2 BM HE = 2a

53
* En el A HEC, catetos en relación de 1 á 2 r =

* Finalmente, en el A HEA : 2x + r =90'

53'
2x + = 90'

De donde : x = 31° 45'

Rpta: (C).

75) En la figura adjunta: AE = FC; y EBF = 90’

Hallar el valor de “x”

A) 7,5
B
B) 8 O
C) 9O
D) 10 O
E) 12 o

212
Solución
R
* Trazamos CR J_ AB .% CBR = BCR = 45°

Entonces : BR = RC.

* Sea “M” punto medio de AC.


Como AE = FC, por dato;
entonces AM = MC.

* Trazamos las medianas BM y RM


en los triángulos rectángulos EBF
y ARC, respectivamente.

Luego : EM = BM = MF y AM = RM = MC ( propiedad ).

* El A MRC resulta equilátero RC = RM = MC ( propiedad ).

* A BMF, isósceles : MBF = MFB = a

* Por ser ángulo externo en A FBC : a = x + 1 5 °............(1).

* Finalmente, en el A BRM, isósceles :


MBR = BMR = 75° a + x + 45° = 75° -» a + x = 30°

con (1) ; x + 15° + x = 30° x = 7,5°

Rpta: ( A ).

76) Hallar el valor de “x”.


Si AE = BC y BE = EC
B
A) 10°
B) 12°
C) 15°
D) 20°
E) 18°
A E
Solución

* Trazamos AM y EM, tal que el


A AME = ABEC, haciendo:
B
EM = EC y AEM = C = 2x
MÁB = x

* Según propiedad, en el cuadrilátero


no convexo ABEM : (3 = 120° - x

* En el A ABE : A + B + E = 180°
213
/. 3x + ( 120° - x ) + 4x = 180°

De donde : x = 10°

Rpta: ( A ).

77) En la figura : AE = BC

Hallar V
B
A) 10*
B) 12<
C) 15'
D) 18'
C E) 20‘

Solución
Tomamos el punto uO”; tal que OÁE = x
y OÉA = 2 x , para obtener el A A OE
congruente al A BEC ( postulado ALA ).

Entonces : OE = EC y OA = BE. B
Además, como BÁE = 3 x = BÉA,
en el A ABE : AB = BE.
Al trazar OC, resulta el A OEC isósceles
También el A AOC /. OC = OA.
ABOE = AOEC ( L A L ) .*. OBE = x.

En el cuadrilátero no convexo AEBO :


p = x + 3x+x-> p=5x
Finalmente, en el A AOB :
2p + 2 x = 180° -> x = 15'

Rpta: (C).

78 En la figura :

AB = BC + EC.

Hallar el valor de “x”.

A) 30°
B) 37°
C) 45°
D) 40°
E) N.A
214
Solución
Sean las longitudes : BC = a y EC = m.

Entonces, por dato : AB = a + m.


Tracemos CQ 1 BE. En consecuencia, el
A QBC es equilátero -> QB = BC = a
AQ = m.

Por otro lado, EB es mediatriz de QC.


Luego : EQ = EC = m

En el A AQE, isósceles : QÉA = Á = x..

En el A QEC, isósceles : EQC = QCE y EQC + QCE = QÉA ( ¿ externo )

EQC = QCE = -
2

Finalmente, por el Teorema del ángulo externo, en el A AQC :

Á + ACQ = BÓC

x + - = 60°
2

Rpta : ( D ).

79) En la figura : BE = EC.


Hallar AC, si AB = 12 y FC 4.

A) 15
B) 14
C) 13
D) 16
E) 18
Solución
AC = ?
T
* A BEC isósceles : EBC = C = ó

* Trazamos BQ _L AF y unimos
F con Q. Entonces :
A ABQ, isósceles AQ = A B = 1 2 . a
(M >)
También :
A AQF = A ABF ( LAL)

FQC = TBA = p
y AQF = ABF = p + $ ............( 1 ).
215
Además, por el Teorema del ángulo externo, en el A FQC : AQF = QFC + C

Con (1 ) : p + ó = QFC + <i> .\ QFC = p -+ AFCQ, isósceles.

Entonces: QC = FC = 4

Finalmente: AC = AQ + QC = 1 2 + 4 AC = 16

Rpta: ( D ).

80) Hallar el valor de “x", en la figura :


B C
A) 85'
B) 95'
C) 90
D) 75
E) 80'

Solución

* Se observa que CDB = 35°

* Trazamos E F . d e modo que


FD = CD, para o b te n e r:
AFED = ACED ( Caso LAL ).
Entonces : EF = EC y
EFD = ECD = 85°

* ABEC, isósceles : BE = EC
BE = EF

* En el ABEF, isósceles, EA biseca el BEF;


entonces EA 1 BF. Resultando el A BAF, isósceles : BAE = EAF = 25°.

* Finalmente : a = 30° ; p = 65' x= a + p x = 95'

Rpta: ( B ).

81) Del gráfico, hallar el valor de x.


B
A) 10c
B) 12c
C) 18°
D) 15°
E) 20°

216
Solución
* A AND, isósceles: AN = ND.

* Trazamos AE, perpendicualr a CD


Propiedad de la bisectriz, para
BCD : AE = AB.
Tam bién: BN = EN,

NÉC = NBC = 3 x y AÑE = 4 x

* Además : a = 90° - 3x = p
Luego, A AHE isósceles : AH = HE y como HE = HD AH = HD

* Entonces, en A AND, isósceles : A A H N : 2x + 4x = 90 ° .

Rpta: x = 15° ... ( B ).

82) Hallar el valor de “x”. Si AC = BE

B A) 10(
B) 12*
C) 15'
D) 18
e E) 20'
Solución

Prolongamos AC hasta UF", de modo


que el AABF sea isósceles, con AB = BF
F = 3 x y CBF = x
Resulta el A EBF isósceles, por ser:

EBF = F .\ EB = EF

Luego : AC = EF —» AE + EC = EC + CF
AE = CF -> A AEB = A FCB ( LAL ) p = x
AABC : P + 9x = 180° /. x = 18°

Rpta: (D).

83) Del gráfico, hallar el valor de x, si AB = BC

A) 10°
B) 5o
C) 20°
D) 15°
E) 18°

217
Solución
x =?
E
D a t o : AB = BC

En el AABE, al igualar la suma de las


medidas de los ángulos interiores , a
1809 : EAB + AÉB + ABE = 180°

2x + x + ( 90° + CBE ) = 180° A B


CBE = 90 ° - 3x

Con el trazo de BR _L AE ( R en la prolongación de EC ) :


A A _
___
__

CBH = AHB = 2 x ( Tienen lados perpendiculares entre s í ).


Se observa que el A REB es isósceles ( EH es altura y bisectriz a la v e z ) HR = HB.

Luego : AR = AB, porque "AE es mediatriz de R B . Entonces : RÁH = HÁB = 2 x

Por ser ángulo externo en el A CEB : P = CÉB + CBE

p = 2x + (90° - 3 x ) - > p = 90o - x (1 )•

Por otro lado, en el ARCB : a + P + 2x = 180°


Con (1 ) : a + ( 90° - x ) + 2x = 180°

a = 90° - x (2).

De ( 1) y ( 2 ) : a = p => El ACBR es isósceles. Por lo tanto: RB = BC. Luego:


resultando equilátero el A ARB.

Finalmente: R A B = 60°
4x = 60°
x =15°

Rpta: ( D ). .

En la figura: AB = AC y AE = EB. Hallar el valor de Mx”, siendo además : ECA = 30°.


E
A) 10°
B) 12°
C) 18°
D) 15°
E) 20°
Solución
Construimos el A AFC, equilátero
FCE = 30° y AAEC = AFEC.
( LAL ), nos conduce a : AE = EF

Luego : A AFE = A AEB ( LLL )


a = EAB —» a = 3x.

Finalmente :
FÁC = 60° -» a + 2 x = 60° 5x = 60

x = 12

Rpta: ( B ).

85) En la figura:

BM y AN son medianas del


A ABC , las cuales se intersecan en el
punto G.
( El punto G se llama Baricentro del trián
guio ABC).

Demostrar que : BG = 2 ( G M )
y AG = 2 ( G N )

Solución

Se pide demostrar la propiedad del Baricentro.


Para ello , tomamos P y Q t puntos medios de
AG y B G , respectivamente. Entonces, PQ es
Base Media del A ABG :
AB
PQ AB y PQ =

AB
También, para el AABC : MN 11 AB y MN =

De lo a n te rio r: PQ = MN y PQ MN.Entonces:
PQG = GMN y QPG = GÑM, por ser alternos
internos. Luego : A PGQ = A NGM, por el postu­
lado ALA. A s í : GM = GQ y GN = GP.
En consecuencia : BG = 2 (GM) y AG = 2 (GN);
quedando demostrada la propiedad.

219
* Nota.- En todo triángulo, las tres medianas se corlan en un punto llamado Baricentro.

- El Baricentro divide cada mediana en segmentos que son entre sí, como 2 a t .De modo que,

podemos e s c rib ir: BG = 2 ( GM ) ó GM = —BM y BG = —BM


3 3
Análogamente, para las otras medianas:

GN = — AN y AG = —AN
3 ' 3

GR = —CR y CG = - C R
3 3

* *

* ENTERATE: «rc-can * ^ 1
Los Postulados que Euclides cosideró en su obra "Los Elementos", fueron lá basé de
toda su estructura lógica. Ellos decían :
I) Desde cualquier punto a cualquier otro, se puede trazar una recta.
II) Toda recta limitada puede prolongarse indefinidamente en la misma dirección!®
III) Con cualquier centro y cualquier radio, se puede trazar una circunferencia! IIÉ ^:
IV) Todos los ángulos rectos son iguales entre sí. --v:- ^
V) Si dos rectas que están en un mismo plano áfe.cortan por una tercera y resulta que los
dos ángulos internos a un mismo lado de la secante suman menos que dos rectos, las
dos rectas deben cortarse y lo harán precisamente del lado de la secante en que los
ángulos internos suman menos de dos rectos.

220
POLIGONOS
Polígono, es una poligonal cerrada, de modo que dos lados no se corten.

La figura adjunta, es un polígono.

Elementos:
Vértices A, B, C, ...
Lados AB, BC, ...
Angulos interiores -» Á, B, ...
Angulo exterior: Ejm.: e.

Diagonal: Ejm.: BE. ÁBUBCUCDUDÉ.

Un polígono convexo se origina de una poligonal convexa ( como en el gráfico anterior). Su


contorno no puede ser cortado más que en dos puntos por una recta que no sea un lado.
Un polígono no convexo o cóncavo se obtiene de una poligonal no convexa. Su contorno
puede ser cortado en más de dos puntos por una recta que no sea un lado.

<\ Hexágono no convexo


\ \

Según el número de lados, un polígono se llama:

Triángulo 3 lados
Cuadrilátero 4 w
u
Pentágono 5
u
Exágono 6
Eptágono 7
U
Octógono 8
44
Nonágono 9
U
Decágono 10
u
Endecágono 11
u
Dodecágono 12
<4
Icoságono 20

Otros se mencionan según su número de lados. Por ejemplo, polígono de 19 lados

221
CLASIFICACION DE LOS POLIGONOS:

Se distinguen :

1.- Equilátero.-Tiene todos sus lados congruentes

W -7

2.- Equiángulo.- Tiene todos sus ángulos congruentes

3.- Regular.- Sus lados y ángulos son, respectivamente congruentes.

Todo polígono regular puede ser inscrito


y circunscrito a dos circunferencias que
tienen el mismo centro. ( Ver figura ).

ángulo central

Si “n" , es el número de lados del

360
polígono:
n

4.- Alabeado.- Sus lados están contenidos en diferentes planos

Ejm.
Exágono alabeado ABCDEF
( AD es una d ia g o n a l).

5.- Estrellado.- Se origina al prolongar los lados de un polígono convexo. El pentágono es el


polígono estrellado de menor número de lados.
C
Ejm.: Pentágono estrellado ABCDE.
D
Lados: AC, CE, ...
Vértices o “puntas”: A, B, ...

Ang.
Exterior

222
PROPIEDADES Y FORMULAS

1) En todo polígono, el número de lados es igual al número de vértices e igual al número de


ángulos interiores.

2) En todo polígono, d e u n u lados, desde cada vértice, se pueden trazar: ( n - 3 ) diagonales

El número total de diagonales, es:

3) Suma de las medidas de los ángulos interiores:

S ti = 1 8 0 ° ( n - 2 )
_

( Válida para todo polígono convexo y no convexo , a excepción de los estrellados y


alabeados).

Ejm.:

¡ = 180° ( 5 - 2 ) ¡ = 180°(6 - 2 )

• • S j = 540 • * S ¡ = 720°

4) En todo polígono convexo, las medidas de los ángulos exteriores, uno por vértice, suman
360°.
5) La medida de un ángulo central, en un polígono regular, es:

360
C =
n

6) En polígonos equiángulos, cada ángulo interior mide

, 180°( n - 2 )
I= i-------- L
n

y cada ángulo exterior:


360'
e=
n

7) En un polígono estrellado, los ángulos interiores, suman: 180° ( n - 4 ) y los exteriores: 720°.

223
PROBLEMAS RESUELTOS

1) ¿En qué polígono, el número de diagonales, es igual al número de lados?

A) En ninguno B) Exágono C) Pentágono D) Octógono E) Cuadrilátero

Solución
Sea wn" el número de lados. Luego:
n ( n - 3)
= n
2

n ( n - 3 ) = 2n —» n- 3 = 2 —> n = 5

Rpta: ( C ).

2) ¿Cuántos lados tiene aquel polígono convexo en el cual, la suma de las medidas de los
ángulos interiores es 5 veces la suma de las medidas de los ángulos exteriores?

A) 12 B) 10 C) 14 D) 8 E) N.A.

Solución
- n: Número de lados.

- Según enunciado:

S ?= 5(sé)

180° ( n - 2 ) = 5 ( 3 6 0 ° )

Luego:

( n - 2 ) = 5 ( 360° ) n - 2 = 10 -> n = 12
180°

Rpta: ( A) .

3) ¿Cuánto mide cada uno de los ángulos interiores de un polígono regular de 18 lados?

A) 138° B) 160° C) 120° D) 118° E) 145°

Solución

- 180° (n - 2 )
Sabemos que : i = -------------- , donde n es el número de lados. Luego, para n = 18,
n

W ángulo
cada ' I ■ * • mide
interior VI : *i = ---------------
180° ( 1 6 )
y 18

i = 160'

Rpta: (B)

224
La suma de las medidas de los ángulos internos de cierto polígono regular excede a la suma
de los ángulos externos en 900°. ¿Cuántos lados tiene el polígono?

A) 16 B) 18 C) 9 D) 12 E) 5

Solución
Sea n, el número de lados.

Se tiene: S ; = S¿ + 900®
I *

180° ( n - 2 ) = 3 6 0 °+ 900°

De donde: n = 9 lados

Rpta: ( C) .

El número de diagonales de un polígono regular, es igual a la suma del número de vértices,


número de lados y número de ángulos centrales. Hallarel número de lados de dicho polígono.

A) 6 B) 9 C) 12 D) 3 E) 5

Solución:
n número de lados.

Según enunciado, planteamos la ecuación:

# Diagonales = # Vértices + # Lados + # Angulos centrales.

n(n- 3)
—----------- = n + n + n
2

ü í ü z ll . 3„ _ a z i , 3 n= 9
2 2

Rpta: ( B ).

En un polígono regular se cumple que la suma de las medidas de un ángulo central, un án­
gulo exterior y un ángulo interior es 210°. Calcular el número total de diagonales.

A) 48 B) 50 C) 52 D) 54 E) 56

Solución
Sea n, el número de lados. Según enunciado, planteamos la ecuación:

360° 360° 180° ( n - 2 )


+ --------+ ----------i---------- L = 210°
n n n
De donde: n = 12

12( 9 ) — .
Entonces, el número de diagonales: — - — = 54

Rpta: ( D ).

225
7) Tres ángulos consecutivos de un octógono convexo, mide 90° cada uno. Hallar la medida
de cada uno de los restantes, sabiendo que son congruentes entre sí.

A) 171 B) 162 C) 152 D) 154 E) 160

Solución

Consideremos el gráfico adjunto


a -» Incógnita

La suma de medidas de los ángulos


interiores:

5 a + 3 x 90° = 180° ( 8 - 2 )

Según gráfico Por Fórmula

5 a + 270° = 1080°

De donde: a = 162

Rpta: ( B ).

8) Los ángulos internos de un pentágono convexo, tienen por medidas números consecutivos,
expresados en grados sexagesimales. Hallar la medida menor.

A) 108 B) 105 C) 107 D) 106 E) 109

Solución

- Sean las medidas de los ángulos, como indica la figura. Luego, la suma, según fórmula, es:

s¡ = 180° ( n - 2 ) S; = 180° ( 5 - 2 )

S- = 540

- Entonces:

x + ( x + 1 ° ) + ( x + 2°) + ( x - 1 ° ) + ( x - 2

De donde: x = 108

- Por lo tanto, el menor mide: x - 2 ° = 1 0 8 ° - 2 o = 106

Rpta: ( D ).

9) La suma de las m edidas de ángulos internos, m ás la sum a de las m edidas de ángulos


centrales de un polígono regular, es igual a ocho veces la sum a de las m edidas de los
ángulos exteriores. H allar el núm ero de diagonales de dicho polígono.

A) 65 B) 54 C) 119 D) 44 E) 104

226
Solución
n # de Lados
Por el enunciado, planteamos:

180° ( n - 2 ) + 360° = 8 x 360 n = 16

Luego, el número de diagonales: 16 ( = 104


2

Rpta: ( E ).

Calcular el número de diagonales de un polígono convexo equiángulo, en el cual la medida


de un ángulo interno es la novena parte de la suma de medidas de los ángulos internos de
un polígono estrellado cuyo polígono base es un dodecágono.

A) 9 B) 18 C) 35 D) 54 E) 135

Solución
Si i, es la medida de un ángulo interno,
del polígono convexo de “n” lados;
según enunciado:

i = -[1 8 0 ° ( 12- 4) ]
9

Suma de las medidas de los ángulos internos


del polígono estrellado de 12 lados.
i = 160

De otro lado, por fórmula para el polígono convexo equiángulo: i = 180 ^ n— ^


n

1809 ( n - 2 )
Entonces: -------------------- = 160® -> n = 18 • • # Diagonales = 18 P 5 ) -= 135
n

Rpta: (E)

Se tiene un decágono regular A B C D E Hallar la medida del menor ángulo que forman
las prolongaciones de AB y E D .

A) 72 B) 36 C) 54 D) 18 E) 9

Soiución
Sea, el gráfico adjunto

Incógnita: x

Medida de un Z interior. Jv
.D e c á g o n o R e g u la r - p
Medida de un Z exterior.

227
Se tiene
3609
e = e = ^ = 369
n 10Q

Luego: = 180° - e

= 180° -3 6

= 144

- Entonces, en el cuadrilátero no convexo BXDC:


/
x + 2e = i
x + 2 ( 3 6 ° ) = 144’
x = 72°

Rpta: ( A ).

12) En cierto polígono convexo, el número de triángulos obtenidos al unir un punto de uno de sus
lados con los vértices, es 9. Hallar el número de diagonales de dicho polígono.

A) 20 B) 27 C) 44 D) 54 E) 35

Solución
Si n, es el número de lados; entonces, el
número de triángulos obtenidos, según .
condición del problema, será: n -1
- Por dato: n -1 = 9 n = 10

-10(7)
Luego, el número de diagonales: — - — = 35

Rpta: ( E ).

13) El número de ángulos rectos que contiene la suma de las medidas de los ángulos interiores
de un polígono convexo, de “n” lados, es:

A) n - 2 B) n C) 2n D) 2 (n - 2)
e >i
Solución
- La suma de las medidas de los ángulos internos, es, por fórmula: 180° ( n - 2 )

- Luego, el número de ángulos rectos que contiene esta expresión, será:

180° (n - 2 ) _
90° “

Rpta: ( D ).

228
14) En un octógono equiángulo ABCDEFGH; AB = 3 ^ 2 y BC 1. Hallar la medida del ángulo
BAC.

A) 7°30' B) 22o30‘ C) 11°15 D) 13°15‘ E) 8

Solución

* Incógnita: BA C.

Cada ángulo exterior, del octógono

equiángulo, mide: = 45°


8

.-. Prolongando HA y CB, hasta su


punto de corte R:
RÁB = 45° = RBA
Luego : R = 90
3/2
- En el A ARB: AR = RB = AR = RB = 3
f*
- Resulta, el A ARC. con: AR = 3 y RC = 4

RAC = 53

45° + BAC = 53 BAC = 8

Rpta: ( E ).

15) Cada lado de un polígono regular mide 6cm. y el perímetro equivale al número que expresa
el total de diagonales, en cm. Hallar la medida de un ángulo central.

Solución

Según enunciado:

e c n(n- 3 ) \fc
6 + 6 + ... = —i L
2 o

“n” veces

6n = ü í-n.- 3 ) "n" Lados


2

De donde: n = 15

£ _ 360^ _ 240
* Medida de un ángulo central
15

Rpta: C = 24'

229
16) ¿Cuál es el polígono que tiene 119 diagonales?

A) 13 B) 15 C) 17 D) 14 E) 16

Solución

Siendo “n” el número de lados: — í l l = 119


2

n ( n - 3 ) = 238. Debemos descomponer 238 en dos factores que difieren en 3:

n ( n - 3 ) = 17(14) -

n = 17

Rpta: (C).

17) ¿Cuál es el polígono convexo en el que el número de diagonales es mayor en 133 que el
número de lados?

A) El de 19 lados. B) El de 23 lados C) El de 16 lados


D) El de 24 lados. E) El de 25 lados

Solución
Siendo “n” el número de lados, se tiene:

n(n- 3) _
= n + 133
2

De donde: n2 - 5n - 266 = 0

( n - 19 ) (n + 14 ) = 0

n = 19

Rpta: ( A ).

18) ABCDEF, es un exágono regular. Sobre BC, se toma un punto R, que al ser unido con F,
determina un segmento secante a AD en el punto Q. S i : ABQ = 4a y FRC = 9a.

Hallar el valor de a.

A) 10° B) 12 C) 9 D) 8 E) 15

Solución
-a = ?

- Considerando el gráfico adjunto; vemos que:

A AQF = A AQB ( Postulado L A L ) D

A F Q = ABQ
A

Es d e cir: AFQ = 4a
230
360s
- Además, el ángulo ZFA, exterior, mide: ZFA = ZFA = 60Q

Luego, por ser altemos internos:


QRC = QFZ
* • 9 a = QFA + AFZ

9 a = 4 a + 60°

De donde: a = 12°

Rpta: ( B ).

19) Se tiene un exágono equiángulo ABCDEF de tal manera que: AB = 2, BC = 6, EF = 1 y


AF = 9. Calcular las longitudes de CD y DE.

Solución
* Por ser equiángulo, cada ángulo exterior, AQ
/ N
X / \x
mide: 360°
= 60 o

* Al prolongar los lados como indica la figura,


se observa que los triángulos ABP, CQD ,
EFR y PQR, son equiláteros.

Luego: F 1 R
PR = 12 -» PQ = 12 x = 4

QR = 1 2 y = 7

Rpta:CD = 4 ; DE = 7

20) Si el número de lados de un polígono regular aumenta en 10, cada ángulo del nuevo polígono
es 3o mayor que cada ángulo del original. ¿Cuántos lados tiene el polígono original?

A) 25 B) 27 C) 20 D) 16 E) 30

Solución
Incógnita: n

Polígonos de Polígonos de
"n“ lados. (n+10) lados.

Polígono de
Polígono de "n" lados. ( n + 10 ) lados

180° ( n - 2 ) 180° (n + 1 0 - 2 )
a = P=
n n + 10

231
Por condición: p -a = 3
r 180» ( n + 8 ) 'í 180® (n - 2 ) '
= 3®
V n + 10 J n j
De donde, al simplificar: n ( n + 10 ) = 1200 n = 30

Rpta: ( E) .

21) En un polígono equiángulo la relación entre las medidas de un ángulo interior y otro exterior
es como 5 a 1.

¿Cuántas diagonales posee dicho polígono?


A) 27 B) 108 C) 54 D) 45 E) 35
Solución
i
♦ Según dato: —= 5
e
i = 5e

Pero: i + e = 180°
Luego: 5e + e = 180

De donde: e = 30°

Siendo “n" el número de lados, la suma de las medidas de los ángulos externos es 360
360°
Entonces: ne = 360 n =

Así: n = 12
12 i 12 - 31
Y, el número de diagonales: — ‘------------ }- = 54
2
Rpta: ( C ).

22) Dos números consecutivos, representan los números de vértices de dos polígonos con
vexos. Si la diferencia de los números de diagonales totales es 3.
¿Cómo se llama el polígono mayor?

Solución

* Sean los números de vértices de los polígonos: n y ( n + 1 ).

Los números de diagonales totales, se expresan:

n ( n - 3) ( n + 1 ) [ ( n + 1) —3 ]

Según enunciado:

( n + 1)[(n + 1 ) - 3 ] n ( n - 3)

Resolviendo: n = 4

El polígono de mayor número de lados: n + 1 = 5, es un pentágono

232
23) En un polígono regular ABCDEFGH..., de “n” lados, las prolongaciones de AB y GF se
cortan en el punto 2. Hallar el valor de “n", si el ángulo BZF mide 105°.

A) 12 B) 15 C) 20 D) 24 E) 18

Solución

Consideremos el gráfico adjunto,


'a *
donde se han prolongado, ade- X 105*\
más, BC y CD.

- x, es la medida de cada ángulo +105


externo del polígono.
- En el cuadrilátero no convexo DEFZ’

x + x + ( 2x + 105°) = 18 0° - x
polígonos regular de “ n* lados
Efectuando: x = 15

- Además, como todos los ángulos externos tienen por suma de sus medidas, 360°;
entonces: n ( 15o ) = 360°

De donde: n = 24 lados.

Rpta: ( D ).

24) Se llama “diagonal media” de un polígono, al segmento que une los puntos medios de dos
lados. ¿Cuántas diagonales medias tendrá un polígono de “n” lados?

Solución
Al unir los puntos medios de todos los
lados del polígono dado, se obtiene
otro de igual cantidad de lados y ade­
más todas las diagonales de éste últi­
mo ( ver fig u ra ).

Así:
n( n - 3 )
# diag. medias = n+ =>

25) Si de 4 vértices consecutivos de un polígono convexo se trazan 33 diagonales


¿Cuántas diagonales en total tiene dicho polígono?
A) 12 B) 33 C) 87 D) 54 E) 52

Solución

* Desde cada vértice se pueden trazar


( n - 3 ) diagonales.

* En el problema, con el gráfico adjunto

233
- Desde A 1 <n-3) diagonales
- Desde A 2 (n-3)
- Desde A3 ( n - 3 ) -1, diagonales ( ya se contó A 1 A 3 ).

- Desde A 4 ( n - 3 ) - 2, diagonales ( ya contamos A «, A 4 y A 2 A 4 )

Es decir:
Desde A, ( n - 3 ) diagonales
Desde A 2 ( n - 3 ) diagonales
Desde A 3 ( n - 4 ) diagonales
Desde A. ( n - 5 ) diagonales

Sumando: 4n -1 5 = 33 , ( según dato )

De donde; n = 12
n" Lados
12Í 9 )
* Se pide el total de diagonales: — ¿— i = 54

Rpta: ( D ).

26) ¿Cuál es el polígono convexo en el que la suma de su número de diagonales y su número


de lados es 435 ? Indicar el número de lados .
A) 29 B) 30 C) 31 D) 32 E) 28

Solución
Siendo “n” el número de lados; según enunciado, planteamos la ecuación

n ( n - 3)
---i--------- L + n := 435
2

Efectuando: n ( n -1 ) = 870 870 10


87 3
29 29
.
1

870 = 30 x 29
n ( n - 1 ) = 30 ( 29 )

De donde: n = 30

Rpta: (B)

27) Se tienen dos polígonos regulares en donde el número de diagonales difiere en 19 y las
medidas de sus ángulos externos están en la relación de 5 a 6 . Hallar el número de lados
mayor.

A) 6 B) 5 C) 10 D) 12 E) 8

234
Solución
Sean n y m, los números de lados, con n > m

Se tienen, por enunciado:

n( n - 3 ) m( m - 3) _
= 19 (a).
2

360'
n 5 m 5
——
»—
y• 360’ 6 n 6 (P )
m

Resolviendo ( a ) y ( P) : m = 10 y n = 12

Rpta: ( D )

La medida del ángulo interior de un polígono regular de “ N ” lados excede en 12° a la me


dida del ángulo interior de otro polígono de ( N -1 ) lados. Hallar “N”

A) 5 B) 6 0 )7 D) 8 E) 9

Solución
Se tiene:

180°(N - 2 ) 180°( N - 1 - 2 )
+ 12
N N- 1

Medida del Z interno Medida del Z interno


P. de N lados. R de ( N -1 ) lados.

Efectuando y simplificando, queda: N ( N -1 ) = 30

N(N-1) = 6(5)

Luego: N = 6

Rpta: (B)

Si al ángulo interno de un polígono regular se le disminuye en 10o, resulta otro polígono cuyo
2
número de lados es — del número de lados del polígono anterior.
w
Calcular el número de lados de ambos polígonos.

A ) 18 y 12 B) 12 y 8 C ) 24 y 16 D) 9 y 4 E )21 y 14

235
Solución
Medida del Z interno Medida del Z interno
del polígono de n del polígono de 2. n
lados. lados. 3

' 2
180' —n - 2
180°(n - 2 ) \ 3
- 10 '

n 2
—n
3

Efectuando, se obtiene; n = 18 lados.


O
el otro polígono tiene: ± x 1 8 = 12 lados.
3

Rpta: ( A ).

30) Si en un polígono convexo se trazan todas las diagonales de un vértice, dicho número de

diagonales más el número de triángulos formados es igual a — del número total de


18
diagonales. Hallar el número de lados del polígono.

A) 14 B) 17 C) 15 D) 18 E) 13

Solución
- Si n, es el número de lados del polígono en mención:

( n - 3 ) diagonales trazadas desde A.

( n - 2 ) triángulos formados.

- Por tanto:

5 n( n - 3 )
( n - 3 ) + ( n - 2 ) = — x - v— - .....

Efectuando: 5n2 - 87n + 1 8 0 = 0

( 5n - 12 ) ( n - 15 ) = 0 n = 15

Rpta: ( C ).

31) Calcular el número de lados de un polígono regular sabiendo que el número que representa
5
la medida de uno de sus ángulos externos es los — del número de diagonales.

A) 9 B) 5 C) 10 D) 12 E) 8

236
Solución
Sea n, el número de lados. Luego, según enunciado, planteamos la ecuación:

360 5 n( n - 3 )
= — x —i--------- -
n 9 2

Efectuando: n2 ( n - 3 ) = 360 x 9 x 2
5

ó: n2 ( n - 3 ) = 7 2 x 9 x 2

Debemos expresar el 2° miembro en forma conveniente, teniendo en cuenta que el producto


en el 1er. miembro es de un cuadrado perfecto y un factor que es 3 unidades menor respecto
a la base. Así:

n2 ( n - 3 ) = 144 x 9

n2 ( n - 3 ) = 122 x 9

De donde : n = 12

Rpta: ( D ).

32) Si el número de lados de un polígono convexo disminuye en 2, el número de diagonales del


nuevo polígono es menor en 15. Calcular la suma de las medidas de ángulos internos,
original.

A) 1000° B) 1987 C) 2000 D) 1440 E ) 1492


Solución
Se tiene:
n lados
* Polígono Original
n( n - 3 )
diagonales

( n - 2 ) lados
* Segundo polígono
(n -2 )(n -2 -3 )
diagonales.

Según enunciado, planteamos la ecuación:

(n -2 )(n -2 -3 )_ n (n -3 )

Efectuando: n = 10

- Se pide la suma de las medidas de ángulos internos: 180° (1 0 - 2 ) = 1440

Rpta: ( D ).

237
33) Si la diferencia entre el número de lados de dos polígonos es 3 y la diferencia entre el número
de diagonales es 15. Hallar el número de lados del polígono de menor número de lados.

A) 8 B) 6 C) 11 D) 4 E) 5

Solución

Sean n y ( n - 3 ), los números de lados de los polígonos en mención. Luego

# de diagonales del # de diagonales del


polígono de n lados polígono de (n - 3) lados.

n( n - 3 ) (n -3 )(n -3 -3 ) _ 1
2

Efectuando: n = 8

El polígono de menor número de lados: 8 -3 = 5

Rpta: ( E) .

34) Se tiene un polígono convexo de “n” lados. El cociente del número de triángulos que se
determinan al unir los vértices del polígono con un punto cualquiera de un lado, entre el
6
número de diagonales que se pueden trazar de un vértice, es igual a -? . Mn” será igual a:
5

A) 13 B) 9 C) 8 D) 11 E )7
Solución

vértice

V__ polígono de
n lados

* ( n - 1 ) triángulos * ( n - 3 ) diagonales desde


determinados. un vértice.

- Según enunciado:
n- 1 6
5 Resolviendo: n = 13
n~^3

Rpta: (A).

35) En un polígono regular de “n” lados, el número que expresa el total de diagonales, en cm.
representa la longitud de cada lado.
Hallar “n”, si el perímetro es 160 cm.

A) 16 B) 4 C) 9 D) 10 E) 8

238
Solución
Por dato:

L n( n - 3 )
L
2

Además: / \ y

Perímetro = 160

r»L = 160 -4 n .
2

ó: n2 ( n - 3 ) = 320

Debemos descomponer 320 en dos factores, siendo uno de ellos un cuadrado perfecto y
observando su relación con el otro. Así, obtenemos: n2 ( n - 3 ) = 82 x 5

De donde: n = 8

Rpta: ( E ).

36) Si un polígono de un” lados tuviera ( n - 3 ) lados, tendría ( n + 3 ) diagonales menos. ¿Qué
polígono es:

A) Triángulo B) Exágono C) Cuadrilátero D) Pentágono E) Pentadecágono

Solución

* Según enunciado tenemos la ecuación:

# de diagonales # de diagonales del


del polígono de polígono de ( n - 3 )
n lados. lados.

n( n - 3 )
2

Efectuando : n = 6

Rpta: (B)

37) Hallar los números de lados de dos polígonos regulares, sabiendo que sus ángulos internos
difieren en 20° y los externos suman 100°

A) 5 y 10 B) 6 y 9 C )4 y 11 D) 3 y 12 E )8 y 10

239
Solución
Sean n y m, los números de lados de dichos polígonos, respectivamente. Según enunciado,
podemos plantear:
180°( n - 2 ) 180°( m - 2 ) _
= 20 (I)
n m
360° 360°
y — + = 100° (II)
m n

Al efectuar el primer miembro de la ecuación ( I ), resulta:


360° 360°
= 20 '
m n

Sumando miembro a miembro, esta última ecuación y la ( I I )


720°
= 120'
m

3 6 0 ^ 360^1000
Reemplazando este valor, en ( I I )
6 n

Rpta: ( B) .

Si a un polígono regular se le aumenta dos lados, su ángulo externo disminuye en 9


¿Cuántos ángulos centrales tiene dicho polígono?
A) 32 B) 36 C) 10 D) 12 E) N.A.

Solución
Se tiene:
n lados
Polígono
Original 360°
—> medida del Z externo
n

( n + 2 ) lados
Segundo
360°
Polígono medida del Z externo
n+ 2

Según dato:
360° 360'
- 9
n+ 2 n
Efectuando: n ( n + 2 ) = 80

ó : n ( n + 2 ) = 8(10) n = 8

El número de ángulos centrales: 8

Rpta: (E ).
39) Hallar el número de lados de un polígono convexo, cuyo número de diagonales excede en
26 al deotro polígono convexo. Además el equivalente en ángulos rectos de la suma de
ángulos internos del primero, excede en 8, al número de ángulos rectos que contiene la suma
de las medidas de ángulos internos del otro.

A8 B) 9 C)11 D) 6 E) 10

Solución

- Sean: n -» # de lados del primer polígono.


m # de lados del segundo polígono.

Se plantean, según enunciado, las relaciones:


n( n - 3 ) m( m - 3 )
+ 2 6 ...... ( I )

2 ( n - 2 ) = 2( m - 2 ) + 8 (II)
De ( I I ), resulta: m = n -4

Reemplazando esto último, en ( I ):


n(n-3) _ (n-4)(n-7)
+ 8

Efectuando: n = 10

Rpta: ( E) .

40) Calcular la medida del ángulo interno del polígono regular, en el cual se pueden trazar 51
diagonales desde 8 vértices consecutivos.

A) 150° B) 162° C) 144° D) 160°

Solución

Considerando el gráfico del


polígono de “n" lados:

- Desde A 1 —> ( n - 3 ) diagonales. lados^>


- Desde A2 —> ( n - 3 ) diagonales.
- Desde A3 -» ( n - 4 ) diagonales, porque A, ya se contó.
- Desde A4 —> ( n - 5 ) diagonales; ya contamos A<¿ A4 y A¡A4 .
- Desde A5 ( n - 6 ) diagonales; porque ya se contaron A¡A5 , A¿AS y A3 A5
t .
- Desde A6 ( n - 7 ) diagonales.
- Desde A7 ( n - 8 ) diagonales.
- Desde A8 -» ( n - 9 ) diagonales.

Por dato, la suma de estas cantidades parciales, es 51. Luego

8n - 4 5 = 51 .*. n = 12

241
- Se pide la medida de un ángulo interno:

180° ( n - 2 )
n

180J<1 0 > = ,5 0
n

Rpta: ( A ).

) Demostrar que, en un polígono de “n” lados, desde “V” vértices consecutivos, se pueden
trazar:

( V + 1)( V + 2 )
V x n- Diagonales

Solución
En efecto; del anterior problema deducimos que

- Desde el vértice 1 n - 3 diagonales


- Desde el vértice 2 n - 3 diagonales
- Desde el vértice 3 —> n - 4 diagonales
- Del vértice 4 n - 5 diagonales

- Del vérice Vo -» n - ( V + 1 ) diagonales.

S um ando: V x n - ( 3 + 3 + 4 + 5 + ... + ( V + 1 ) )

o m ej or : V x n - ( 1 + 2 + 3 + 4 + 5 + ... + ( V + 1 ) )

= V x n- (v + 1)( v + 2 ) ■ como se quería demostrar.


2

* Nota: Por ejemplo, en un polígono de 18 lados, desde 7 vértices consecutivos, se pueden


trazar:

( 7 + 1)(7 + 2)
7x18- = 90 diagonales

Y, el total de diagonales de este polígono,es:

1 8 (1 5 )
= 135

242
42) Desde 7 vértices consecutivos de un polígono, se han trazado 55 diagonales. ¿Cuántas
diagonales en total, tiene dicho polígono?

A) 55 B) 60 C) 90 D) 77 E) 65

Solución

En este caso, para usar la fórmula demostrada en el anterior problema: V = 7; siendo


n ( # de lados ), desconocido.

Así:
(V + 1) ( V + 2) „ , (7 + 1 ) (7 + 2)
V x n - --------- —----------- =55 /. 7 n - --------- —----------- = 55
2 2

De donde: n = 13 lados.

Entonces, el número total de diagonales:


n ( n - 3 ) _ 13( 10)
2 2

Rpta: ( E) .

43) ¿ Cuál es el número de ángulos internos agudos que puede tener un polígono convexo de
n lados ?

A) 3 B) (n - 3) C) 2 D) 4 E )5

Solución
Adyacente a cada ángulo interno agudo, habrá un ángulo externo obtuso. Como los
exteriores deben sumar 360°, entonces no podrán haber cuatro ángulos externos obtusos
ya que la suma de sus medidas superarían esta cantidad. Por lo tanto, como máximo, un
polígono convexo podrá tener 3 ángulos interiores agudos.

Rpta: ( A ).

44) Las medidas de los ángulos interiores de un polígono convexo están en progresión aritmética
de razón 5o, siendo la medida del menor: 120°. Hallar el número de lados del polígono.

A) 16 ó 9 B ) 15 ó 10 C) 11 ó 14 D ) 12 ó 13 E) 18 ó 7
1 20°+ 3x5°
Solución
n # de lados.

Las medidas de los ángulos serán como


se indica en la figura. El mayor tendrá
medida: 1 2 0 ° + ( n - 1 ) 5 °

La suma:

120° + ( 120°+2 x 5 ) + ( 120°+3 x 5 ) + ... + ( 120°+( n-1 ) x 5o ) = 180° ( n - 2 )


v_______________________________________________________ j
Según gráfico. Por fórmula general.

243
Escribiendo el 1er. miembro de modo conveniente:

120° x n + ( 1 + 2 + 3 + ... + ( n - 1 ) ) x 5 °= 180°( n - 2 )


Es d e c i r : ______________ _______

120° n + ^ x 5° = 180° ( n - 2 )

120n + 1 n ( n - 1 ) = 180 ( n - 2 )
2

De donde:
n ( 2 5 - n ) = 144 n ( 2 5 - n ) = 16 x 9

Resolviendo, obtenemos dos valores posibles de n : 16 ó 9

Rpta: ( A ).

JUEGO DEL "SIM " :


George Simons, un matemático de nuestros tiempos, inventó un juego en el cual puedei
participar 2 ; 3 ; 4 ; n , jugadores. Consiste en : -

19 Marcaruna determ inadacantidadde pun­ X*

tos, en una circunferencia ( Por ejemplo,


9, en la figura ), distinguiéndolos por una
letra diferente.

29 Usando tinta de diferente color, dos juga­


dores ( ó más ), unen estos puntos , uno
a continuación de otro.

39 Gana el juego, el primero que forma


un triángulo, cuyos vérticesson 3 pun­
tos de la circunferencia, antes marca­
dos.

* Por ejemplo, un jugador ( a ) usa el trazo discontinuo ..... y otro jugador ((5), el trazo

1) Juega ( a ) : Traza CE 5) Juega ( ce) : CF ( j j r


w* «-• <n~

2) Juega ( ¡ 3 ) : Traza DG 6) Juega ( ¡ J ) : HF


3) Juega ( a ) : Traza CH 7) Juega { a ) : EF
4) Juega ( ¡ 5 ) : Traza HE * • Gana ( a ), con ei A ECF.

244
V

CUADRILATEROS
Estos polígonos pueden ser convexos, no convexos, equiláteros, equiángulos, etc.

Clasificación de los Cuadriláteros Convexos.

I.- PARALELOGRAMOS

Tienen sus lados opuestos respectivamente paralelos. Se dividen en:

a) Romboide.-Es el paralelogramo propiamente dicho.

b) Rombo.-Tiene sus 4 lados congruentes.


c) Rectángulo.- Tiene sus 4 ángulos congruentes.

d) Cuadrado.- Tiene sus 4 lados y 4 ángulos, respectivamente congruentes.

Propiedades:

1.- En todo paralelogramo, los ángulos y lados opuestos, son respectivamente congruentes.

2.- Las diagonales de todo paralelogramo se intersecan en su punto medio.

3.- Las diagonales del rectángulo, son congruentes.

4.- Las diagonales del rombo son perpendiculares y bisectrices.

5.- Las diagonales del cuadrado son congruentes, perpendiculares y bisectrices.

245
II.- TRAPECIOS

Tienen dos lados paralelos, llamados bases. La altura del trapecio es la distancia entre las
bases. Se dividen, en: *

a) Trapecio Escaleno.-Sus lados no paralelos tienen diferente longitud.

b) Trapecio Isósceles.-Sus lados no paralelos son congruentes.

c) Trapecio Rectángulo.-üno de sus lados es perpendicular a las bases.

a) Trapecio b) Trapecio c) Trapecio


Escaleno Isósceles Rectángulo

Propiedades:

f E l segmento que une los putos medios de los lados no paralelos se llama “base media",
“mediana” o “paralela media”; es paralelo a las bases y mide la semisuma de ellas.

2.- El segmento que une los puntos medios de las diagonales se ubica sobre la mediana y mide
la semidiferencia de las bases.

B n AD + BC
1) MN =
2

AD - BC
2) PQ =
2
A D
III.- TRAPEZOIDES

No tienen lados paralelos.

B
* Existe un tipo especial de trapezoide ,
llamado simétrico o contraparalelogramo.
En éste , una diagonal es porción de me-
diatriz de la otra.

246
PROBLEMAS RESUELTOS

1) En un AABC, se traza la altura B H . Si M, N y Q, son puntos medios de AB, BC y AC,


respectivamente: demostrar, que el cuadrilátero MNHQ, es un trapecio Isósceles.

Solución

( a ) AABC: Á y C , agudos ( b ) AABC: ACB, obtuso


BC
Sabemos que, por ser base media, en cada caso: MN 11 AC ; MQ = — ••• ( 1 )

BC
Y, por ser medianas relativas a la respectivas hipotenusas: HN = — ... ( 2 ).

Luego, de ( 1 ) y ( 2 ): MQ = HN. • • MNHQ, es un trapecio Isósceles.

2) ABCD, rectángulo
ÉO 1 ÁC y AO = OC = OE

Hallar el valor de x

A) 84° B) 90° C) 69°


D) 66° E) N.A
D
Solución
Al trazar BD, como:
AO = OC = BO = OD = OE;

en el AOBE; Isósceles:

x + x + 48° = 180°

x = 66°
D
Rpta: ( D ).

3) La figura ABCD, es un rectángulo


BH _L AC ; OBC mide 34°

BP biseca el HBO y
CP biseca el O C D .

Hallar la medida del ángulo BPC

A) 90° B) 68' C) 102° D) 136° E) 73°


0 4
Solución
A
- Incógnita: P

- Por ser rectángulo: OB = OC

A BOC, Isósceles: OCB = OBC = 34®


y BÓC = 180° - 2 ( 34° ) —> BOC = 112 °

- En el cuadrilátero no convexo BPCO:


P +CC + P = 1 1 2 °.......... ( I ).

- A BHO: 2a + 90° = 112® a = 11

- BCD: 34° + 2P = 90° -> P = 28'

- Reemplazando, en ( I ): P + 11° + 28° = 112' * • P = 73°

Rpta: (E)

4) En un rombo ABCD, A < 90°, se trazan BH y C R , perpendiculares a AD ( H en AD y R en


su prolongación ).
)•
Hallar HD, si: AR = 17 y HR = 11

A) 6 B) 7 C) 5 0 )4 E) 3

Solución

HD = ?

Del gráfico: A AHB = A DRC


/. AH = RD

Siendo: AH = 1 7 - 1 1
AH = 6 RD = 6

Luego: DH = 11 - RD -> DH = 1 1 - 6

DH = 5

Rpta: ( C ).

5) Hallar la longitud de la mediana de un trapecio ABCD, si:

BC = 3; Á = 53°, AB = 5 y D = 45°

A) 5,5 B) 7,5 C) 6,5


D) 4,5 E) N.A.

248
Solución
La longitud x, de la mediana, es:
BC + AD
x = (I)

Para hallar AD, trazamos BH y C Q ,

perpendiculares a A D .
BH— 3 — C
AAHB: BH = 4 y AH = 3

A CQD: CQ = 4 = QD

Además: HQ = BC = 3
AD = 10

3 + 10
En ( I ): x = — - — =6,5

Rpta: (C)

En un triángulo ABC, M es punto medio de A B . Se traza MH i AC; ( H en AC ). Hallar la


longitud de EF, si F está sobre B C , E es punto medio de HM y E F X H M , siendo
AH = 3 y HC = 7

A) 4 B) 5 C) 4,5 D) 5,5 E) 6
Solución
Incógnita: EF

Trazamos MN II AC; entonces,

en el A ABC, por ser base media

AC
MN =

MN = — = 5
2

-En el trapecio MNCH, EF

MN + HC
es mediana: EF =

5 + 7 EF = 6
EF =

Rpta: ( E ).

249
Demostrar que, en todocuadrilátero, si se unen consecutivamente los puntos medios de los
lados, se obtiene un paralelogramo.

Solución
Como el cuadrilátero en mención puede ser convexo, no convexo ó alabeado, la demostra­
ción es válida para todos los casos:

ABCD: Convexo ABCD: No convexo ABCD: Alabeado

Demostremos que MNPQ, es un paralelogramo. Veamos:

AC
A ABC: MN es Base Media MN 11 AC y MN =
1 x .11
AC
• v v .

A ACD: PQ es Base Media PQ I AC y PQ = MN = PQ

- En forma análoga, se demuestra que: y NP = MQ

Entonces: MNPQ, es un paralelogramo, ya que es suficiente con demostrar que los lados
opuestos son respectivamente paralelos.

En un trapezoide ABCD, Á + B = 136°; hallar la medida del mayor ángulo que forman las
bisectrices de los ángulos C y D.

Solución

Incógnita: x

Dato: Á + B = 136°

En el A : x + a + ó = 180° . . . (1)

En OABCD: 2a + 2<t» + Á + B = 360°

Con el dato:

2a + 2<(> + 136° = 360° a + p=112°


- Sustituyendo esto último, en ( 1 ): x + 112° = 180°

x = 68'

Rpta: (A )

* Nota.- En general tenemos la propiedad

9) En un cuadrilátero ABCD, Á - C = 22°; hallar la medida del menor ángulo que forman las
bisectrices de los ángulos B y D.

A) 22° B) 11 C) 33° D) 44' E) N.A.

Solución
B
- Consideremos el gráfico adjunto.
Incógnita: x

- En el cuadrilátero no convexo BCDE:


a + 13 + C = BÉD -> a + P+ C = 1 80° -x
a +p+C+x = 180°............( 1 ).

- En el aABCD: 2a + 2P + Á + C = 360°

a + p = 180o - A + C (2 )
A

- Reeplazando ( 2 ), en ( 1 ): 180a - A + C + C + x = 180°

De donde: ( P ropiedad).

- Sustituyendo el dato: x = 2 2 x = 11

Rpta: ( B ).

10) En la figura, MN es mediana dei trapecio ABCD

MR = RH y HT = TN.
Si BC = 36 y AD = 48

Hallar PQ

A) 28 B) 27 C) 30
D) 26,5 E) 28,5

251
Solución

- MN iI BC, AD, entonces: BP = PR y CQ = QT

BC + RT
- En el trapecio RBCT: PQ = ( 1 ).

.... BC + AD 36 + 48
- EnABCD: MN = ------------- = -------------- = 42

MN 42
y en el A MHN : RT = RT = 21

Ofi i O1
Reemplazando, en ( 1 ): PQ = --------------= 28,5

Rpta: (E )

11) Demostrar que, en todo triángulo, la suma de las distancias de los vértices a una recta
exterior, es igual a la suma de distancia de los puntos medios de los lados, a la recta.

S olución

- Sea el A ABC y M, N, Q,
puntos medios de los lados

- Por ser mediana en los trapecios:

- ABB’A’ :

AA’ + BB’ = 2 ( MM’ )

- BB’C’C’ :

BB’ + CC’ = 2 ( NN’ )

- ACC’A’ :

AA 1+ CC’ = 2 ( QQ’ )

Sumando miembro a miembro, estas tres relaciones:

2 ( AA’ + BB’ + CC’ ) = 2 ( MM’ + NN’ + QQ’ )

• • l.q.q.d.

12) En un rombo ABCD, M es punto medio de B C . La diagonal BD, corta a AM en el punto R.


Si RM = 10 y el ángulo BRM mide 53°; hallar BD.

A) 60 B) 70 C) 80 D) 36 E) 72

252
Solución
BD = ?

- Trazamos AC, la cual corta


a BD, en el punto O. Además,
sabemos que: AC 1 BD,
AO = OC y BO = OD.

- En el A ABC, R es Baricentro.
Por lo tanto: AR = 2 ( RM )
AR = 20

A AOR (3 7 ° y 53° ) : OR = - ( AR ) f (20) OR = 12


5 5

BR = 2 ( OR ), por propiedad del Baricentro. Entonces: BR = 24

A s í : BO = 36 = OD , en consecuencia : BD = 72

Rpta: (E).

13) En un paralelogramo ABCD, sobre la diagonal BD se toma el punto P. Por A, se traza paralela
a BD, cortando a la prolongación de CP en el punto R. Si CP = RP, BP = 12 y
PD = 5 ; hallar AR.

A) 8,5 B) 8 C) 9 D) 6 E )7

Solución

AR = ?

- Se prolongan AR y C D ,
hasta su corte en F.

- En el A RCF, PD es
Base media: RF = 2 ( PD )

RF = 10

- Luego, en el paralelogramo ABDF: AF = BD

AR + 10 = 12 + 5 AR = 7

Rpta: (E).

4) En un trapecio ABCD, BC || AD, AB = 6 , BC = 4 y AD = 14; las bisectrices de los


ángulos A y B, se cortan en el punto P. Hallar PQ, si Q está en CD y PQ 11 BC.

A) 5 B) 6 C) 7 D) 4 E) 8

253
Solución
PQ = ?

Se prolonga BP hasta cortar a AD en F:


A A

F = CBF . . . ( alternos internos ).


F = P

A BAF, es isósceles:

AF = AB -> AF = 6 FD = 8 A 6 F 8 D
14
también: BP = PF.

Finalmente, en el trapecio BCDF :


2

PQ = 4 + 8 PQ = 6
2

Rpta: ( B ).

En un romboide ABCD, AB = 5 y BC = 12; las bisectrices de los ángulos A y B se cortan


en el punto M y las de C y D en N. Hallar la longitud de MN.

A) 7 B) 8 C) 9 D) 10 E) 6

Solución

Incógnita: MN

Prolongamos AM y DN,

hasta E y F, respectivamente. Luego:

É = EAD = a y

F = FDA = p*( alternos internos ).

A ABE -> isósceles: BE = AB -> BE = 5 y AM = EM

AFCD -> isósceles: FC = CD FC = 5 y FN = ND

Por otro lado: EF = 1 2 - B E - F C EF = 2

En el trapecio AEFD; MN es mediana : mn = + EF


2

MN = 1 2 + 2 -> MN = 7
2

Rpta: (A)
ABCD, es un cuadrilátero no convexo, siendo D, el ángulo entrante. Si: C - Á = 32°, hallar
la medida del menor ángulo formado por las bisectrices de los ángulos B y D..

A) 32° B) 16° C) 64° D) 8 o E) N.A

Solución
Se tendrá el gráfico adjunto

Dato: C -A =32°

Incógnita: x

En el cuadrilátero no convexo
ABED: Á+ p + x = a (1 )

En ABCD:
A + 2p + C = 2 a (2)

Reemplazando (1 ) en ( 2 ):
A + 2p+ C = 2 ( A+ p + x )

efectuando: ( propiedad).

op°
entonces: x = — = 16°

Rpta: (B).

ABCD, es un romboide. Las distancias de lo vértices A, B, C y D, a una recta secante a los


lados BC y AD, son: A E , B F , C Q y DH.

Si : AE = 7, CQ = 28 y BF = 16; hallar DH

A) 20 B) 19 C) 21 D) 18 E) 37

Solución

Según el gráfico; por tener sus lados


respectivamente paralelos:
ABF = CDH

Trazamos : AP 1 BF y CR 1 DH.

Luego : PF = AE PF = 7 y
BP = 1 6 - 7 BP = 9

también : RH = CQ -> RH = 28.

255
- Además: A CRD = AAPB DR = BP DR = 9

- Finalmente: DH = DR + RH = 9 + 28 DH = 37

Rpta: (E).

Nota: Hemos demostrado, indirectamente, que:

o:

Cuando la recta corta a dos lados opuestos.

18) En un paralelogramo ABCD, BD = 1 8 ; M es punto medio de BC y N biseca CD


AM y AN, cortan a BD en los puntos R y S, respectivamente. Hallar RS.

A) 9 B) 6 C) 8 D) 12 E) 15

Solución

- Dato: BD = 18

- Incógnita: RS

Trazamos AC, la cual corta a BD


en el punto O. Sabemos que, por
ser un paralelogramo:

BD 18
BO s OD =
2
BO = OD = 9

Además: AO = OC

- R es Baricentro del A ABC y S es lo del A ACD.

Entonces: RO = ^ ( BO ) = — ( 9 ) -» RO = 3
o 3

1 • 1
y OS = - ( O D ) = - (9) OS = 3

- Por lo tanto : RS = RO + OS RS = 6

Rpta: ( B ).

Nota: En general, se tendrá la propiedad:

BR = SD = , para todo paralelogramo

256
19) En la figura, ABCD es un trapecio.

AM = MD, AE = EC, EQ = QM,

QR II AB y BC = 18. Hallar QR

A) 13 B) 13,5 C) 14

A M D D) 12 E) 15

Solución
- QR = ?
18
- Del gráfico: B F R C

BCA = CÁD = a ( alternos internos )

- A ABC, Isósceles: AB = BC AB = 18

- Trazamos EF II AB. Entonces:

EF=“ EF = — EF = 9

- CM es mediana hacia la hipotenusa del A ACD : CM = AM = MD. Resultando el


A AMC,isósceles: ACM = <x. Luego: CM 11 AB, ya que ACM y BAC son alternos internos
congruentes. Por lo tanto:
CM = AB -> CM = 18. También: EF I CM

Finalmente, en el trapeGio EFCM EF + CM


QR =

QR = 9 + 18
2
QR = 13,5

Rpta: (C )

20) AI intersecarse las bisectrices de los ángulos interiores de un rectángulo ABCD, se forma
un:

A) Rectángulo B) Trapecio Isósceles C) Rombo D) Romboide E) Cuadrado

Solución
E
Sea EFGH, el cuadrilátero formado. B Yéó ~\\ PAQ
Luego, es fácil deducir, que: Z45 45*>:
v y
\H / y

E = F = G = H = 90 o c<>
yy \%
/ \ 4 S

•'
4 \

' \
4 \ 4
Ahora, veamos: si HE = EF *v 45'

\
\
\
4
4
4
45-/:
'/V ^ +■ - —

D
Como el A ABP es congruente G'
con el AQCD: AB = BP = CD = QC

257
AH = HP = QF = FD

Además: P = 45° = Q —> PE = EQ

Entonces: HE = EF □ EFGH es un cuadrado

Rpta: (E )

21) En un cuadrado ABCD, cuyo lado mide 18cm. M y N , son puntos medios de AB y BC,
respectivamente. AN y CM, se cortan en el punto Q. Hallar QB.

A) 6 B) 6 / 2 C) 4 / 2 D) 6 / 3 E) 4 - /3

Solución
BQ = ?

Trazamos AC y prolongamos BQ
hasta el punto O. Se observa que Q
es el Baricentro del A ABC.

Entonces : BQ = — ( BO ).
O

= AC = 1 8 / 2 = g ^
S ie n d o : BO

reemplazando en lo anterior: BQ = — ( 9-^/ 2 ) BQ = 6 / 2

Rpta: (B)

22) En la figura, ABCD es un paralelogramo


y r una recta exterior. Demostrar,
que:

Solución

- Sea O, intersección de AC y BD
B
tracemos OM 1 r

- Por ser mediana en los trapecios:


a ' " . .

ACPT -> OM = a t -c. .. ( 1 ).


2
b + d
BRSD -» OM = — — .. (2 ).

258
ai igualar los segundos miembros : a + c = b + d
•••(••««■i I.C|*c|*d•

Nota: Además, es fácil demostrar, efectuando la suma de las expresiones (1 ) y ( 2 ) , que

a + b + c + d
OM =

En un paralelogramo ABCD, las distancias de los vértices A, B y C a una recta secante a


los lados AB y B C t miden 6 , 7 y 8 , respectivamente. Hallar la distancia del vértice D, a
la misma recta.

A) 21 B) 22 C) 20 D) 24 E) 18

Solución

Datos: AE = 6 , BF = 7
CG = 8

Incógnita: DH

Trazamos: BP 1 AE y CQ _L DH.
Entonces, por tener sus lados res­
pectivamente paralelos : PBA = QCD.
A APB = ADQC.

Luego HQ = CG -> HQ = 8 ; EP = BF = 7 y DQ = AP = AE + EP = 6 + 7
-+ DQ = 13.

Finalmente: DH = DQ + QH =13 + 8 DH = 21.

Rpta: (A).

Nota: Se ha demostrado de modo indirecto, que si la recta corta a dos lados consecutivos
del paralelogramo:

( P ropiedad)

Dado el trapecio ABCD, BC 11 AD ; AE = 2 ( EB ) y EF 11 BC

Demostrar que:

___ i
D

259
Solución

Sea M, punto medio de A E .


Entonces, como AE = 2 ( EB )
AM = ME = EB. Luego; al
trazar MR 11 EF:

Trapecio MBCR D

Trapecio AEFD —> MR = EF + A D - . ( 2 )

Sustituyendo ( 2 ) en ( 1 ):

EF + AD
BC +
EF =

2 EF = BC + EF + AD

2BC + AD
De donde: EF =

l.q.q.d

O bservación:

Si en el trapecio ABCD: BP = 2 ( PA ) y PQ 11 B C ; entonces:

Demostrar que, en todo triángulo, la distancia del Baricentro a una recta exterior, es igual al
promedio de distancias de los vértices, a la misma recta.

Solución

Sea ABC, el triángulo y G


su Baricentro.
Si r es la recta exterior
al A ABC, debemos demostrar,
que:
Veamos:
En el trapecio BB’ M’M, como BG = 2 ( GM ), por propiedad del Baricentro, entonces según
el problema anterior:
2MM* + BB'
GG' = ( 1)

Pero, en el trapecio ACC’A’: 2 MM’ = AA’ + CC’

Reemplazando esto, en (1):


AA' + CC' + BB’
GG' =

GG' = AA + + CC ^ como se pidió demostrar.

En un triángulo ABC, las distancias de los vértices A, B y C , a una recta secante a los lados
AB y BC, s o n AE = 17, BF = 10 y CQ = 11. Hallar la distancia GR, del Baricentro G
del A ABC, a la misma recta.

A) 6 B) 7 C) 8 D) 5 E) 4

Solución
Datos: AE = 17, B F = 10, CQ = 11

Incógnita: GR

Tracemos MN T a la recta. Entonces


como CG = 2 ( GM ), por propiedad
del Baricentro ; según propiedad
demostrada en un problerrta anterior,
para el trapecio MNQC:

2MN + CQ
GR = (U

AE - BF
Para hallar MN, trazamos EB y AF. Luego: MN =

en el trapecio AEBF. 2 MN = AE - BF. Reemplazando esto, en ( 1 )

AE - BF + CQ
GR =

o ( Propiedad, para todo triángulo )

17 + 1 1 - 1 0 18
Reemplazando valores: GR = GR = 6

Rpta: (A)
261
27) En la figura adjunta, G es Baricentro
del A ABC. Demostrar, que:

Solución

- Según el problema anterior, la distancia GR, del Baricentro a la recta, es

AE + CQ - BF
GR =

- En este caso: GR = 0
AE + CQ - BF
O =

De donde, efectivamente: BF = AE + CQ

28) En un trapecio ABCD, BC 11 AD , Á = 82°, BC = 4, CD = 14 y D = 16°

Hallar la longitud de la mediana.

A) 11 B) 10 C) 9 D) 14 E) 16

Solución

Sea la figura adjunta.


La longitud x , de la
mediana del trapecio ABCD, es

4 + AD
x = (I)

- Al trazar CR 11 AB : ABCR ,
paralelogramo: AR = 4 y
ACDR, Isósceles: RD = 14

AD = AR + RD -> AD = 4 + 14 -> AD = 18.

- Reemplazando, en ( I )

4 + 18 ,,
x = ----------- = 11
Rpta: (A )

29) En un trapezoide ABCD , Á = 53°, C = 98°, D = 45°, AB = 10 y CD = 1 1 -/2

Hallar AD
262
Solución
AD = ?

Se trazan BH y C E ,
perpediculares a AD.
Luego: BQ 1 CE

A CED ( 45° ) : CE = ED = 11
53* ¿ 4 5 “
A BHA ( 37° , 53° ): BH = 8 .

Luego: EQ = 8 y CQ = 3

En el A BQC ( 53° ): BQ = 4 HE = 4

Entonces: AD = AH + HE + ED

AD = 6 + 4 + 1 1

Rpta: AD = 21

En un trapecio ABCDt Á + D = 90°, BC 11 A D , BC = 4 y AD = 14. M , es punto medio

de BC y N, punto medio de AD. Hallar MN.

A) 5 B) 10 C) 8 D) 6 E) 3

Solución
- Considerando el gráfico

Á + D = 90° -> a + 4» = 90o... ( dato ).

- Incógnita: MN.

- Trazamos: ME 11 AB y MF 11 CD

AE = BM = 2

FD = MC = 2, E = A
A A

v F=D

Luego: EN = NF = 5

EMF = 90°.

- Observamos que MN es mediana relativa a la hipotenusa EF del triángulo rectángulo

EF 10
EMF: MN = MN = 5

Rpta: ( A )
263
31) En un trapezoide ABCD , Á = 6 0 °, AB = 8 - / 3 ,C D = 2 0 / 2 y D = 45°. Hallar la distancia
del punto medio M de BC, a AD.

A) 14 B) 13 C) 15 D) 18 E) 16

Solución

Sea ME 1 AD. Incógnita: ME.

- Trazamos BH y C Q , perpendiculares
a AD. Luego, en el trapecio HBCQ, ME
BH + CQ
es mediana: ME = (1 ) D
20^2

_ 8 /3
De otro lado: BH = — ^ — x / 3 BH = 12, (e n el A AHB )

_ 20/2
CQ = -> CQ = 20 , en el A CQD.
" V2

12 + 20
Reemplazando en ( 1 ): ME= ------------- -> ME = 16

Rpta: ( E ).

32) En un paralelogramo ABCD, “M” es un punto de C D , tal que: AMD = 74°. La mediatriz de
BM, corta a AM en “P” y a AD en “Q”.
Hallar la medida del ángulo APQ, si PM = CD.

A) 37° B) 32 C) 16° D) 53° E) 57°

Solución

- Incógnita: APQ = x

- AB = CD y PM = CD

AB = PM

- Por ser alternos internos entre


AB y CD :

BÁP = PMD -> BÁP = 74 9

- Opuestos por el vértice: SPM = APQ SPM = x

Trazamos PB y usamos la “Propiedad de la mediatriz”


PB = PM

264
AABP, Isósceles: BPA = BAP = 74°.
Luego, en el ABPM : a + a = 74° ( Z e xte rio r)

a = 37°

Finalmente: x = 9 0 °- a
x = 90 - 37 x = 53°

Rpta: (D)

En un trapezoide ABCD, AB = BC = CD, B = 144° y C = 96°.


Hallar la medida del ángulo D.

A) 36° B) 72° C) 54° D) 55° E) N.A

Solución
La figura 1, indica el problema propuesto
A

Incógnita: D = x

Fig.1. Fig.2

En la figura 2, se muestra la solución. Se trazan: por A, paralela a BC y por C, paralela a


AB, cortándose éstas en el punto R.
Entonces:

ABCR Rombo ARC = B = 1449 y

BCR = 180° - B BCR = 36°; luego: RCD = 60°

El A RCD, es equilátero RD = RC = C D y RDC = 60°.

En el A ARD, Isósceles : ARD = 156° y 2a = 180° -156° -» a = 12°

Finalm ente: x = a+RDC -> x = 12°+ 60° x = 72°

Rpta: ( B ).

Se tiene un cuadrilátero ABCD, en el cual B = D = 90° y C = 60°. La longitud del segmento


que une los puntos medios de las diagonales es de 3 cm. Hallar BD.

A) 4 ^3 cm. B J S ^ /lic m . C ) 6 j 3 cm. D) 6cm. E) N.A.

265
Solución
BD = ?
MH = 3, une los puntos
medios de las diagonales.

Por ser mediana relativa a


la hipotenusa:

A ABC BM = AM = MC
A ADC —* DM = AM = MC

- EnC: a + <¡) = 6 0

- En el cuadrilátero no convexo BCDM

BMD = a + ( a + <¡>) + <¡) BMD = 2 ( a + (¡>) = 2( 6 0 ° ) .% BMD = 120°

El A DMB, es Isósceles; MH 1 BD y BMH = HMD = = _ 59°


2 2

Luego, en el A MHB: BH = M H x / 3 . . . ( BH se opone a BMH = 60° )

BH = 3 / 3 .

Finalmente : BD = 2BH BD = 6 / 3

Rpta: (D)

35) En el gráfico:
2CD = 3AB, BC = 8 m. y AD = 6 m
B
Calcular AB

A) 3m.
B) 4m
C) 5m
D) 6 m
E) 7m
Solución
N
* Prolongamos AB tal que BN = 3k

* GBNCD (trapecio isósceles).

ND = BC = 8

* kADN : 5k = 10

k = 2 AB = 4.

Rpta: (B)
266
En un A ABC, AB = 5, B = 106° y BC = 15. Hallar la distancia del punto medio de AC,

a la bisectriz de! B.

A) 2 B) 3 C) 4 D) 5 E) N.A.

Solución
B
Sea UMMel punto medio de AC y MN
A

perpendicular a la bisectriz del B


Incógnita: MN ^C
Trazamos: AH 1 BN y CE 1 BN. Luego,
los triángulos AHB y BEC, son notables
( 37° y 53° )
É"
AH = 4 y EC = 12

El cuadrilátero AHCE es un trapecio y MN une los puntos medios de las diagonales


HE y AC.
MM EC - AH 12-4
MN = ------------- MN =
2
Rpta: ( C ).

En la figura, ABCD es un paralelogramo


B ____ C * distancia de P, a CD • 4
* distancia de B ,a AD: 18
Hallar la distancia de B, a CD
P
A) 11 B) 12 C) 13
D) 14 E) 10
jD
Solución
del gráfico:
E
a V - 7 .
PF = 4 . . . distancia de P, a C D .
q /
" \ ' " 7
BE = 18 .. . distancia de B, a AD

Incógnita: BR. / 1 8

/ 1i /A v\
Prolongamos BP y A D , hasta l' // \«
su intersección en el punto S: ¿T a " r n / P
A HD
S = SBC -> S = p

El ABAS, es isósceles. .\ BP = PS. Trazamos PH 1 AD. Luego, en el ABES, por ser


BE 18
base media : PH = PH = 9.

267
Enseguida, prolongamos FP hasta Q. PQ ± AB. Ahora, por propiedad de la bisectriz:
PQ = PH -> PQ = 9.

- Finalmente: BR = QF -> BR = QP + PF

BR = 9 + 4 • « BR = 13

Rpta: ( C ).

38) En un trapecio ABCD, BC 11 AD , AB = 15 , BC = 10 , CD = 17 y AD = 28.

Las bisectrices del Á y B , se cortan en P. Las del C y D, en Q. Hallar PQ

A) 3 B) 2 C)1 D) 1,5 E) 2,5

Solución
PQ = ?

- Se prolongan BP y CQ,
hasta cortar a AD en
F y E, respectivamente. Luego

BFA = FBC = a
y CÉD = ECB = r
( alternos internos).

* A BAF, isósceles: AF = A B = 15 y BP = PF

* ACDE, isósceles; ED = CD = 17 y CQ = QE

* Entonces: EF = 4

* Finalmente, al trazar BE y CF; en el trapecio BCFE:

BC -EF 10-4
PQ = PQ = 3

Rpta: (A )

39) En un trapecio ABCD, A = 2 D , se traza la altura B H . Si: BC = 2 ; AH = 1 y HD = 8 .


Hallar AB.
B- 2 -C
Solución

* AB = x = ?

* Trazamos BR 11 CD
D
RD = BC = 2

268
* En el A ABR, trazamos BS, tal que S = A = 2 a

* AABS, Isósceles -> BS = AB = x y HS = AH = 1

* A BSR, Isósceles —> SR = BS = x

* Para: HD = 8

1 +x +2 = 8 x = 5

Rpta: AB = 5

40) Dado el trapezoide ABCD y una recta


exterior r ; la suma de las distancias de
A, B, C, D a r . e s 8 8 . Hallar la distan­
cia de O a r .
D
A) 22 B) 33 C)44
D) 11 E) N.A.
x r
Solución
- Dato:
AL + BH + CV + DS = 8 8 B

- Incógnita: OE

- Como M, N, P, Q,
son puntos medios de los lados
del trapezoide ABCD, entonces ^D
MNPQ resulta paralelogramo.

Luego: MO = OP

Trazamos MT y P R . En el trapecio
. . __ MT + PR / .v
MPRT : OE = ................ ............(1 )

AL + BH AL + BH +C V + DS
- En ABHL: MT = Sumando : MT + PR =

CV + DS 88
- En CDSV : PR Con el dato : MT + PR =

MT + PR = 44. Reemplazando en (1 ): OE = 22

Rpta: (A)

269
r
CAPITULO 9

CIRCUNFERENCIA
Lugar G eom étrico.- Se dá este nombre, a toda figura, cuyos puntos gozan de una misma
propiedad.
Ejm. 1.- El Lugar Geométrico de todos los puntos de un plano, que equidistan de los
extremos de un segmento dado, es una recta ( mediatriz ).

L.G. de los puntos P,


que equidistan de A y B.
2
B
( PA = PB )

Ejm. 2.- El Lugar Geométrico, de todos los puntos del espacio, que equidistan de una
recta dada, es una superficie cilindrica.

L.G. de los puntos que equidistan de la recta L .


( Cualquier punto de la superficie cilindrica, está a una distancia r, de L ).

Ejm. 3.- El Lugar Geométrico de los puntos de un plano que equidistan de los lados de
un ángulo, es una recta. ( La bisectriz del ángulo y su rayo opuesto ).

L.G. de los puntos P,


— o tales que PQ = PR

D efinición de C ircunferencia.- Circunferencia es el Lugar Geométrico (L.G.), de todos los


puntos de un plano que equidistan de otro punto llamado centro. La distancia del centro a
cualquiera de los puntos del L.G. se llama radio.

L.G. de los puntos que equidistan del punto “O”.

( P p = P20 = P30 = radio).

271
Ejm. 4.- En el centro de un camino recto se ubica un pino. Se sabe que a 40m. del camino
y a 50m. del pino se encuentra un tesoro. ¿En cuántos lugares se debe buscar
el tesoro?
Solución
Considerando la figura adjunta, donde se indi­
ca el camino en cuestión y P es la posición del
pino; notamos que el L.G. de los puntos situa­
dos a 40m. del camino son dos rectas
paralelas a él ( L, y L2 ). Además, como el camino
\
tesoro se ubica a 50m. del pino, haciendo
centro en P y con radio igual a 50m. trazamos
una circunferencia (L.G. de los puntos situados
a esa distancia, del punto P). Luego, donde
corte esta circunferencia a las rectas Lt y L2 se
debe buscar el tesoro: puntos A, B, C y D. Es
decir, cuatro lugares.
Ejm. 5.- Dos árboles A y B están distanciados 20 metros. Un tercer árbol se quiere plantar
a 15m. de A y a 8 m. de B. ¿En cuántos lugares es posible hacer esto?

Solución

Con centros en A y B se trazan dos circunferencias,


de radios 15m. y 8 m .t respectivamente. Estas se
interceptan en los puntos P y Q. Luego, el tercer árbol
puede ser plantado en P o Q.

C írculo.- Es la porción del plano que comprende a una circunferencia y todos sus puntos
interiores.

Líneas notables en la circunferencia.-


Para el gráfico adyacente:
0 —> centro.
r radio.
AB —» cuerda.
QS -» diámetro
( cuerda m á xim a ).
AB —> arco.
ñ —» recta tangente
( T: punto de tangencia ).
m —> recta secante.
MF —» Flecha ó sagita de PQ.

Propiedad: Portres puntos colineales, pasa


una y sólo una circunferencia. Es decir, tres
puntos no colineales determinan una cir­
cunferencia.

Así, los puntos no colineales A, B y C,

272
determinan una circunferencia, la cual tendrá su centro en la intersección de las mediatrices
de AB y BC ( ó AC y A B ; AC y BC).
Ya que, por propiedad de la mediatriz:

Para AB : OA = OB OA = OB = OC = radio.

Para BC : OB = OC O centro de la circunferencia.

Angulos en la Circunferencia

t) Angulo Central: 5) Angulo Interior:

AB + CD
X =

O centro

2) Angulo Inscrito
6) Angulo Exterior: <j P

a.- De dos secantes

P=
ÁB - 65

3) Angulo Ex-inscrito: b.- De Secante y Tangente:


.P
ABC
e=

Observación:
c.- De dos Tangentes

4) Angulo Semi-inscrito:

AT
ATB = Nota:
Para este caso particular:

P + ÁB = 180°
T Punto de tangencia.

273
P ropiedades Básicas

Todo radio hacia el punto de tangencia


es perpendicular a la tangente.

En una misma circunferencia, o en


dos circunferencias congruentes, a
arcos congruentes corresponden
cuerdas congruentes y viceversa.

S i: AB' = CD, entonces: AB = CD

Cuerdas paralelas interceptan arcos


congruentes.

Si AB || CD-

Entonces: AC = BD.

Nota:
T
Si rn es tangente
y m II AB, entonces:

A I = ÍB

Las tangentes trazadas a la misma circunferen


cía, desde un punto común, son congruentes

PA = PB

Además PO biseca el P.

Todo diámetro perpendicular a una cuerda biseca a la cuerda y a los arcos respectivos.

M MN = diámetro

Si MN 1 AB, entonces

B AE = AB,

AN = NB y ÁM = MB

Las cuerdas equidistantes del centro, son congruentes.

Si “O” es centro y d, = d2,


A fd .'V - -
Entonces: AB = CD o d
De-dos cuerdas con diferente longitud, es mayor la más próxima ai centro.
■n C
Si “O" es centro y d, < d2,

Entonces: AB > CD.

Dos circunferencias congruentes secantes, determinan sobre ellas dos pares de arcos
congruentes.

Así, para las circunferencias congruentes de la


figura anterior:

D
AEB = AFB
y n
ACB = ADB

Teorem a de Poncelet.- En todo triángulo rectángulo, la suma de longitudes de los catetos,


es igual a la suma de longitudes de la hipotenusa y el diámetro de la circunferencia inscrita.
Así, para la figura adjunta:

AB + BC = AC + 2r

( r, se llama inradio del A ABC ) F

Demostración B

Del gráfico: AB = AE + EB AB = AE + r
(+>
BC = CF + FB BC = CF + r

Efectuando la suma indicada: AB + BC = AE + CF + 2r

Siendo: AE = AG y CF = CG:
AB + BC = AG + CG + 2r

AB + BC = AC + 2r Iqqd

Teorema de P itoth.- En todo cuadrilátero circunscrito a una circunferencia, la suma de


longitudes de lados opuestos, tiene un mismo valor.

Para el gráfico adjunto; luego colocar variables a


>UC
las tangentes:
AB = w + x
CD = 2 + y

Sumando miembro a miembro:

Z\D
AB + CD = w + z + x + y

AB + CD = AD + BC |

275
G eneralización del Teorem a de P itoth.- En todo polígono circunscrito a una circunferencia
y cuyo número de lados sea par, la suma de longitudes de los lados que ocupan lugar impar,
partiendo de un vértice cualquiera, es igual a la suma de los lados de lugar par.

Consideremos un polígono de 2n lados, como


en la figura adjunta.
Luego:

Ejem plo.- Para el Exágono circunscrito de la figura, si partimos del vértice A; en sentido
horario:

15 + 1 8 + 13 = x + 16 + 14 x = 16

- Si hubiéramos partido de B

x + 1 6 + 14 = 1 8 + 1 3 + 15 x = 16

Teorema de Steiner.- En todo cuadrilátero exinscrito a una circunferencia, la diferencia de


longitudes de lados opuestos tiene e! mismo valor.
* Un cuadrilátero se llama exinscrito, si las B
prolongaciones de sus cuatro lados son
tangentes a la misma circunferencia.

Así, para el gráfico adjunto:

AB - CD = BC - AD

Arco Capaz.- Es un arco de circunferencia que contiene los vértices de los ángulos inscritos
cuyos lados pasan por los extremos de una cuerda, llamada cuerda capaz.
Así, para la figura adjunta:

s %

APB, es el arco capaz de los ángulos de


medida ct°.
A B : cuerda capaz.

Se cumple:

APB = 360° 2a'

* Nota.- En el caso particular en que AB sea diámetro : u° = 90

(A o B

C uadrilátero In scrito .- También llamado cuadrilátero cíclico, es aquel que tiene sus cuatro
vértices sobre la misma circunferencia.

76
Propiedades:
1) Las medidas de dos ánguios opuestos suman 180

A + C = 180

B + D = 180

D em ostración:

BCD BAD
A + C = + ---------------

BCD + BAD 360'


A + C = = 180'

2) Las diagonales forman ángulos congruentes con lados opuestos.


B C

Ya que, cada uno mide la mitad del arco BC.


Además, es fácil demostrar, que:
_C
3) Un ángulo exterior es congruente con el ángulo
interior opuesto: B ^

C uadrilátero In scrip tib le .- Es aquel que puede ser inscrito en una circunferencia.
Si el cuadrilátero cumple con cualquiera de las propiedades del cuadrilátero inscrito, será
inscriptible.

Recta de S im pson:
Si “P", es un punto cualquiera de la circunferencia
circunscrita al A ABC, y:
PQ 1 AB ; PS 1 BC ; PH 1 AC

Entonces: Q, S y H son colineales.


m es la recta de Simpson, del punto P, para el
A ABC.

Se define así: “Es la recta que une los pies de las


perpendiculares trazadas a los lados de un trián­
gulo inscrito en una circunferencia, desde un
punto cualquiera de la circunferencia”.

D em ostración:
Bastará demostrar, que BSQ = HSC, ya que,
como B, S, C, son colineales, resultarán, opues-
A A

tos por el vértice BSQ y HSC con lo que


concluimos que Q, S y H están en línea recta.
vr**;
veamos:

- üABPC, inscrito -» QBP = PCA....(Propiedad n°3 del cuadrilátero inscrito)

Entonces: QPB = HPC .... ( Son complementos de QBP y P C A , en los triángulos


ü BQP y PHC, respectivamente ).
(1 )

- DQBSP inscriptible, ya que BQP + BSP = 1802


QPB = B S Q ....( 2 ).

- aP SH C -> inscriptible, por ser PSC = PHC = 902

HPC = HSC....( 3 ).

Finalmente, reemplazamos los equivalentes de QPB y HPC de ( 2 ) y ( 3 ), en ( 1 ):


BSQ = HSC Q , S , H están el línea recta.

POSICIONES RELATIVAS DE DOS CIRCUNFERENCIAS

1) EXTERIORES

OOi > R + r

* 2 tgs. Comunes exteriores: AB = CD

* 2 tgs. Comunes interiores: PQ = MN

2) TANGENTES EXTERIORES

T Punto de Tangencia

OOi > R + r

* 2 tgs. Comunes exteriores : AB = CD

1 tg. Común in te rio r: PQ 1 OOi

3) SECANTES

R - r < OOi < R + r

* 2 tgs. Comunes exteriores: AB = CD

* Propiedad: FG 1 OOi

278
4) TANGENTES INTERIORES

T Punto de Tangencia

OOi = R - r T

* 1 Tg. común: PQ 1 OOi Q

5) INTERIORES

OOi < R - r

6) CONCENTRICAS

O es el centro de ambas circunferencias

7) ORTOGONALES

Si: OTO! = 90°


T -» Punto de Tangencia

R - r < OOi < R + r...( Teorema de desigualdad


triangular).
OO 12 = R2 + r2 ( Teorema de Pitágoras ).
PROBLEMAS RESUELTOS

1) OA y O B, son radios de una circunferencia de centro O. Sobre ei menor arco AB se toma


el punto F. Si el ángulo AFB mide 130°, hallar la medida del ángulo AOB.

A) 130° B) 65 C) 50 D) 100 E) N.A.

Solución

- Consideremos el gráfico adjunto.


Incógnita: AÓB.

- Sabemos, que por ser ángulo central


AOB = A F B ( 1 ).

. ¿-n AMB * . . .
- También: AFB = --------- ....( Z inscrito)

i
130° = AMB .*. AMB = 260

>s
Luego: AFB = 360° - AMB AFB = 100°. Reemplazando, en (1 ): AÓB = 100

Rpta: ( D ).

2) En la figura: \
AE = 192° y BFD = 140

Hallar la medida del BMD.

A) 52' B) 42 C) 26°
D) 62 E) 72

Solución

BFD
- En la menor circunf. : c = ....(Z inscrito)

140°
C = C = 70°

- En la mayor c irc u n f.: ó = AE - BMD ....( Z exterior)

192o - BMD
70° =

'N
De donde: BMD = 52

Rpta: ( A ).

280
Se prolonga el diámetro BA de una circunferencia de centro O, hasta el punto P y se traza
la tangente PT. Hallar la medida del arco TB, si PT mide igual que el radio.

A) 45° B) 135 C) 60 D) 120 E) 150

Solución
La medida del arco TB es igual a
la del ángulo c e n tra l: x

O T 1 PT

Como PT = radio PT = OT. En el AOTP, isósceles: a = 45

x = 180°- a

x = 1 8 0 ° - 4 5 ° = 135 -> TB mide 135°.

Rpta: ( B ).

En la figura:

PB y PC son tangentes.

É mide 26° y F mide 25°. Hallar el valor de x

A) 51' B) 102 C) 94
D) 47 E) 6 8 °

Solución

En el cuadrilátero EAFP: x = É+ Á + F
x = 26° + Á + 25° x = 51° + A .... ( 1 ).

BO
Por ser ángulo inscrito: A = ; pero, según propiedad vista en la fórmula 6 c, de

ángulos en la circunferencia: BC + BPC = 180

Es decir: BC + x = 180° BC = 180o - x.

180° - x
Luego: A =

180° - x
Reemplazando esto último, en (1 ): x = 51° +

Efectuando: x = 94

Rpta: (C)

*V?1
5) La figura muestra dos circunferencias
congruentes. CD mide 164°. Hallar la
medida del ángulo A.

A) 82 B) 21 C) 48

D) 41 E) 42

Solución

Para la circunferencia EBCD: £ = CD BME ^ ^ ¿ exter¡or)

á = 1-6 4° - b m e d )

- En la circunferencia BAEN Á = ....(Z inscrito)

• • BNE = 2A

- Pero, por ser congruentes las circunferencias: BME = BNE

• • BME = 2A (2)
- Finalmente, reemplazando ( 2 ), en (1 ):

164°- 2 A
A = A = 4T

Rpta: ( D ).

6) En la figura: a + p = 136°.

Hallar la medida del arco AD.

A) 6 8 ° B) 64° C) 100
D ) 132° E) 136

Solución
Se tienen: AD + BC _ a ( Z interior).
2
AD - BC D
y • ó-= P ( Z e xte rio r).

Sumando miembro a miembro: AD + BC + AD - BC


= a + p

AD = a + p AD = 136

Rpta: ( E ) .

282
7) En te figura adjunta, Pt Q, R, S y T, son
A

puntos de tangencia. El B mide 44°.

Hallar el valor de x.

A) 44° B) 6 8 C) 22
D) 46° E) 23

Solución

- En el A EFX:
É + F + x = 180° (1)
- En las circunferencias:
pe;
E = — y PS = 180° - A , é = ( 2 ) .

2
QS
F= y QS = 180° - C F = 1 8 0 ° T .C .....( 3 ) .

- Sumando ( 2 ) y (3), miembro a miembro:


a

A + C
E + F = 180° -

Reemplazando esto último, en ( 1 ) :


A ^

180° - A + C + x = 180°

A + C
De donde x=

- Pero, en el A ABC : Á + C = 180° - B = 180° - 44° = 136'

Por lo tanto x = 68

Rpta: ( B ).

8) En la figura, AE es diámetro y N punto de tangencia.

B
Hallar el valor de x.

A) 15° B) 18' C) 12
D) 20° E) 10

283
Solución

Trazamos el radio ON ON 1 B C . B
Entonces: ON 11 A B . Luego : OÑA = NAB

( alternos in ternos) • ■ ONA = 2x.

A AON, isósceles: NÁO = OÑA

NAO = 2 x

Finalmente, en el A ABC:

BAC + C = 90°
li
4x + x = 90 x = 18
Rpta: (B)

En la figura:

AH = HC y A es punto de tangencia
Hallar el valor de x.

A) 18' B) 12 C) 10
D) 15 E) 20

Solución
x = ?
Como AH = HC y B H 1 AC,
el AABC es isósceles.
ABH = x = HBC

En la circunferencia:

AE
ABE = — = APE

• • x = a

A f

También BÁH = = A P B .............BAH = APB BAH = 4 a

T t
( Z semi-inscrito ) ( Z inscrito )

Entonces, en el AAHB : BAH + x = 90° -> 4a + a = 90° a = 18


x = 18°

Rpta: (A ).
10) En la figura.
AB es diámetro y PD tangente.
El ángulo P mide 32°.

Hallar la medida del ángulo ACD

A) 119' B) 109 C) 122


D) 148 E) 106

Solución
m.

- Incógnita: ACD

- Al trazar BC : A C B = 90°,
ya que AB es diámetro.

- Con OD : OD _L PD DOP = 90° - P = 90° - 32° DOP = 58°. Luego: BD = 58’

BD 58
Además: BCD = — , ( Z inscrito) BCD = BCD = 2 9 °

Entonces: ACD = ACB + BCD


ACD = 90° + 29° ACD = 119

Rpta: ( A ).

11) En la figura adjunta:


m
T y E son puntos de tangencia,
m 11 A C . Hallar el valor de x.
B
A) 30’ B) 45' C) 60
D) 36 E) 54'

- Como m II AC, entonces:


/' N ^ ~N "*\ ^ N B
AT = TC -» AT = TC = a.

- En C, por ser ángulo ex-inscrito:


r' \
TCE
TCE = 2 x

Luego: CE = TCE - a -» CE = 2x - a (1 )

285
y AE = 360° - ( AT + TCE )

AE = 360° - ( a + 2x ).....( 2 )
AE - CE
- En B, por ser ángulo exterior a la circunferencia: B=

Es decir: x = — — — , con (1 ) y ( 2 ):

360° - ( a + 2 x ) - ( 2 x - a )
Efectuando: x = 60
2

Rpta: ( C ).

12) En la figura, O es el centro de la


circunferencia.
Hallar el valor de x.

A) 40' B) 56 C) 34
D) 46 E) 44

Solución

Por ser Z exterior a la circunferencia: x = — — , donde AED = 180° por ser

180°- B C
AD diámetro x .... ( 1 ).

Como AÉB = 32? y AB = 2 ( AÉB) ( AÉB, es Z inscrito)


1^'
AB = 64'

De otro lado: *B --?P = P -* 64° _ CD = 24° -> CD = 16°

z"
- Entonces: BC = ABCD - AB - CD

BC = 1 8 0 ° - 6 4 o - 16 -* BC = 100

Reemplazando esto último, en (1 ):

180° - 10 0 (
x =

x = 40
Rpta: (A)

286
13) En el gráfico adjunto

Hallar el valor de x.
n,
iP

A) 90° B) 60° C) 75
D ) 135° E) 120

Solución
Q
Por propiedad, sabemos que
A A

x = A + BCD ^ ¿ formado por las bisectrices de P y Q en el cuadrilátero no convexo )


2
Pero, en el cuadrilátero inscrito: Á + BCD = 180°
180°
Entonces x= x = 90

Rpta: ( A ).

14) El cuadrilátero ABCD, de la figura adjunta,


se llama Bicéntrico por ser inscrito y cir­
cunscrito. Demostrar, que: x = 90°.

Solución
EF + MN
- En la circunferencia menor x = (1 )

- Por propiedad vista en teoría: EF = 1 8 0 ° - A

MN = 180o - C

Entonces: EF + MN = 360° - ( Á + C )

- Pero, en el □ ABCD: Á + C = 180

Entonces: EF + MN = 3 6 0 °-1 8 0 E F + MN = 180'

- Reemplazando en (1 ): x = 180° _> x = 90° iqqd

15) En la figura adjunta,


T es punto de tangencia.
Demostrar, que:
AB CD

287
Solución

- Bastará demostrar, que Á = D

- Se traza, por T, EF; tangente común


a las circunferencias. Luego;

Z
r ETB =
TB

semi-inscrito
= A -> ETB = A
L
Z inscrito

L FTC = TC r
=D FTC = D

Pero: ETB = FTC Á = D . Entonces: AB 11 CD, ya que Á y D son alternos internos


congruentes.

16) En el gráfico adjunto.

P es punto de tangencia
Demostrar, que:

PA = PB

Solución
- Por P, trazamos la tangente PQ, común
a las circunferencias.

- Entonces, por ser ángulo semi-inscrito:

PA A PB
P= y P=_
2
PA PB
PA = PB lo a d
• ■••■•••••la >J •u «

17) Para el gráfico de al lado

Demostrar, que:

AB I! CD

Solución
- Bastará demostrar, que: Á + C = 180'

- Trazamos PQ. Luego, en los cuadriláteros


inscritos: DABPQ a = Á
□ PQCD a + C = 180°

Con lo a nterior: Á + C = 180° • • Entonces: AB CD

288
18) AD y BC, son tangentes comunes
a las circunferencias.
Demostrar, que:
AB = CD

Solución
Según la fórmula vista en 6 c, de ángulos en la circunferencia ( teoría ).

Sabemos que: P + AB = 180' Entonces: P + AB = P + CD


r \

y : P + CD= 180' ÁB = CÓ
I.q.q.d

19) En la figura, A , B y C, son puntos de


tangencia.
Demostrar, que:

Solución

- Por B, trazamos la tangente m . ♦


A

Luego: ABC = x + y .... ( 1 ).

AE = EB ^ Á = ABE = x

EC = EB C = EBC = y

- En el A ABC:x + ( x + y ) + y = 180
x + y = 90°

Entonces, en ( 1 ) : ABC = 90o- -. ( Para cualquier par de circunferencias tangentes


exteriores).

20) En la figura:

ABC mide 112°.

Hallar el valor de x

A ) 112°
B) 56°
C) 6 8 °
D) 44°
E) N.A.

289
Solución

Se traza BE. Entonces:


ABC = a + <¡> a + <J>= 1129 .......(1 )

En la menor circunferencia:

EC D
EBC = EDC
2
Por ser Z exterior al A FPD

AFD = P + D AFD = x + <(> (2 )

En el cuadrilátero inscrito AFEB:

AFE + ABE = 180° ó AFD + ABE = 180°


Con ( 2 ) :
Usando (1 ): x + 1 1 2 ° = 180° .*. x = 68

Rpta: ( C ).

En la figura ,T y F son puntos de


tangencia.

E T - A C = 84°.
Hallar la medida del arco FT.

A) 108 B) 118 C) 128


D) 96° E) 138

Solución

Dato: ET - AC = 84
’N

Incógnita: FT

Si Á = 0 , entonces:

FT = 180° - ó (1)

También: BD = 2 Á = 2 ó

Además:
ET
ETD = — ( z semi-inscrito)

Si ETD = a ET = 2 a

Por otro lado: AC = 2B -» B = p y AC = 2p


En el A BTA : P + ó = a ......( Z exterior) <|> = a - p .......( 2 )

pero, por dato: ET - AC = 84' 2 a - 2 p = 84

a - p = 42°

En ( 2 ): $ = 42

Finalmente, reemplazando en (1 ) : FT = 180° - 42

FT = 138
Rpta: ( E ).

Desde un punto P, exterior a una circunferencia, se trazan la tangente PA y la secante PB C ,


siendo 32° la medida del ángulo APC. Hallar la medida del ángulo ABM, Si M es punto medio
del arco BC.

A) 122 B) 106 C) 102 D) 128 E) 118

Solución

Consideremos el gráfico adjunto: Incógnita:

ACM
ABM =

a + <)>
ABM = (1 )

Para la circunferencia: 2a + p + <)> = 3 6 0 °....(2 )

_ , ., AC — AB ~ . * • \
También: = P ..... ( Z exterior)

<¡> - p
= 32

• • Ó -P = 64°...... ( 3 ) .

Sumando miembro a miembro las expresiones ( 2 ) y ( 3 ):

2a + p + <|) + < | > - p = 360° + 64'

2a + 2<¡> = 424°

a + <t> = 2 1 2 °

Reemplazando esto, en ( 1 ):
212
ABM =

ABM = 106

Rpta: (B)
291
23) En la figura: A, B y C son colineales
Hallar la medida del ángulo EBF, si
los arcos AMF y EMC miden 254° y
262°, respectivamente:

A) 102° B) 83 C) 82
D) 78° E) 74'

Solución

- Datos: AMF = 254°, EMC = 262

- Incógnita: EBF

- Del gráfico: ABE + EBF + FBC = 180' (1 )

^ _ EBC 360° - EMC


- Por ser ángulo ex-inscrito: ABE = ABE *

A é E = 36CT— 262°_ ^ A b e = 4 9 ° .....( 2 )

T u.. ^ ABF rA r. 360°- A M F


También : FBC = ------- -» FBC = ---------- ■
---------

360° - 254'
FBC = FBC = 5 3 °...... ( 3 )

- Finalmente, reemplazando ( 2 ) y ( 3 ), en ( 1 ) :

49° + EBF + 53° = 180

De donde: EBF = 78

Rpta: (D).

24) En la figura adjunta, A, B, C, D, son puntos de tangencia.

AB + CPD = 298

Hallar el valor de x.

A) 18°
B) 72°
D
C) 77°
D) 52°
E) 62°

292
Solución
- Dato : ÁB' + CPD = 298

- Con las prolongaciones hechas:


AHEX : x + É = A H B ...... ( 1 )

- Pero, por propiedad: D

AHB = 180° - AB
(2 ) '’E
E = 180° - CD

- Reemplazando ( 2 ), en (1 ) :
z

x + 180° -C D = 1 8 0 ° - A B ó: x = CD - AB ; además: CD = 360° - CPD

Luego: x = 360° - CPD - AB -> x = 360° - ( CPD + AB )

Usando el dato:

x = 360° - 298° -+ x = 62

Rpta: ( E ).

25) Desde un punto P, exterior a una circunferencia, se trazan la tangente PA y la secante PBE
Si: PB = 7 y BE = 1 0 ; hallar la longitud del radio de la circunferencia, si PA 1 PE

a) 12 b) / T Í 9 c) 10 d) 11 e) 9

Solución
- O centro

- Se traza OM _L BE

BM = E M ( Propiedad )

Entonces: BM = — = 5
2

OAPM : rectángulo OA = MP
r = MB + BP
r = 5+7 r = 12

Rpta: (A).

26) En un cuarto de circunferencia de centro O y radios OA, OB, se toma el punto E y luego
AH 1 OE ; BP X OE ( H y P sobre OE ). Hallar EP, si AH = 15 y BP = 8

A) 7 B) 3,5 C) 2,5 D) 1 E) 2

293
Solución
- Incógnita: EP

- Con el gráfico:
O ÁH = PÓB = a ( Lados perpendiculares )

Análogamente: AOH = OBP = ó

A AHO = A OPB

Luego: OP = AH - OP = 15

OH = BP -> OH = 8

- Con el Teorema de Pitágoras, en el A OHA: OA = OH + AH

OA = 8 2 + 152 -» OA = 17 ( Longitud del radio )

Entonces: OE = 17 y EP = O E - O P

EP = 1 7 - 15 -> EP = 2
Rpta: (E).

27) Para el gráfico adjunto, marcar lo correcto:

B a) a + (3 + x = 180°

b) a + P = 2x

c) a +p+ x = 270°

d) a + p = 180° + x

e) a + P = 90° + x

Solución
Del gráfico:

AD + CF = 2 x ( z in te rio r: x )

DEF = 2( 1 8 0 ° - a ) Por la fórmula

ABC = 2( 1 8 0 ° - p ) de Z ex-inscrito.

Pero: AD + CF + DEF + ABC = 360

Reemplazando aquí, las tres expresiones anteriores:

2 x + 2 ( 180° - a ) + 2 ( 180° - P ) = 360° .

De donde

Rpta: (D)

294
28) En la figura, O es el centro y T punto de tangencia.
Si A = 28° y ET || AB.
Hallar la medida del ángulo ETC
1
A) 28°
B) 56°
C) 17°
D) 31°
E) 32°

Solución

- Incógnita: ETC = ?

- Si se traza el radio OT y se le prolonga


hasta el punto H : OT X ET y OH 1 AB

En el AAHO: AOH = 90° - 28°

AÓH = 62° = TÓC


A

- Siendo ETC, Z semi-inscrito:

TC '" 'N - - TOC 62°


ETC = — y TC = TOC ETC = ETC = 31°

Rpta: (D)

29) En la figura:
PQ I AB y AB = 12. AE
es diámetro ; CQ tangente

Hallar PQ.

A) 6 B) 4 C) 3
D) 5 E) N.A

Solución

Al trazar OQ:
OQ 1 CB y OQ II ÁC

.-. APQO es un paralelogramo y


como OA = OQ, resulta un rombo.
PQ mide igual que el radio V del
semicírculo;

PQ
PQ = 60° y PQC = PQC = 30® = B

295
PQ
Luego, en el A PCQ: PC = PC = — ( opuesto a 30° )

y en el A ACB: AC = AB { AC, se opone a 3 0 ° } . Luego :


2 2
r = 4

Rpta: PQ = 4... ( B ) .

30) La figura muestra un cuarto de circunferencia


y una semicircunferencia. T, es punto de
tangencia. Hallar la medida del arco PQ.

A) 30° B) 15° C) 7°30*


D) 22°30' E) 18°30’

Solución
- PQ = ?

- Trazamos MT y OQ .

- Llamando r , el radio de la semicircunferencia:


OB = 2r; □ OETM es un cuadrado y también:
OQ = OB -> OQ = 2r

- A OET: Isósceles EOT = 45

- A QEO: OQ = 2 ( OE ) EÓQ = 60

Luego: POQ = EOQ - EOT

PÓQ = 60° - 45°

PÓQ = 15°.

Por lo tanto: PQ = POQ PQ = 15

Rpta: ( B ).

31) En la figura, O y P son centros de las


circunferencias congruentes. Además O es
A
punto de tangencia. Hallar el valor de x.

A) 90°
B) 75°
C) 60°
D) 80°30’
E) 82°30’

296
Solución

Trazamos PO y OC PO _L AB.
OP es radio común a las circunferencias
OP = OA = OC = PC = PD.

En el AOPX, por ser Z exterior:


x = a + fj> ( 1 ).

A OPC equilátero Ó = 60°...... ( 2 )

A AOP isósceles OPA = 45°.

También: a + ct = 45°.....( Z exterior al A O P D ) • ♦ a = 22°30*...... ( 3 )

Reemplazando ( 2 ) y ( 3 ), en (1 ):
x = 22°30' + 60° x = 82°30‘

Rpta: ( E ).

En la figura, O y M son centros del cuarto de


circunferencia y la semicircunferencia, res­
pectivamente. T es punto de tangencia.
Hallar la medida del arco FB.

A) 16' B) 15° C) 30
D) 37 E) 18°30'
Solución
FB = ?

Con los trazos indicados:


90
53°
AAOM: p = (catetos con longitudes

en relación 1 a 2 ).
t
i
i
P = 90° - 2 p ( En el AAOP ) í
2r
P = 90° - 53° P = 37°

AC -FB
Pero: ( Z exterior en la

semicircunferencia mayor)

90° - FB
»
• • 37° = FB = 16'

Rpta: (A ).
297
33) En la figura, B, C y D son puntos de
tangencia.
El ángulo P mide 62°.
Hallar la medida del arco AB.

A) 124 B) 31° C) 60
D) 62° E) N.A

Solución
\

- Incógnita: AB
*m
- Trazamos BC. Entonces: AB = 2 x (1 )

- Al trazar, por C, la tangente común m :

BC
EBC = = BCE

Angulos semi-inscritos
CD
ECD = = CDP

-E nC : x + a + ó = 1 8 0 °...... ( 2 )

- En el cuadrilátero BCDP:

a + ( a + ó ) + Ó + 62° = 360° 2( a + <J>) = 298 —» a + ó = 149


- Luego, en ( 2 ): x + 149° = 180° x = 31

Reemplazando en (1 ): AB = 62

Rpta: ( D ).

En la figura, AD es diámetro. B, C, T y P

puntos de tangencia. CT mide 124°.


Hallar la medida del TP.

A) 44° B) 56 C) 22
D) 6 6 ® E) 36
Solución

- Incógnita: TP
* Como ET y EP son tangentes:
TP + TÉP = 180°.... (1 )

- Además: TBC + CT = 180°


TBC +124° = 180 TBC = 56°
298
Al trazar DB, como AD es diámetro: ABD = 90°
TBD = 90° - TBC
TBD = 90° - 56°
TBD = 34°

Luego: BD = 2 ( T B D )

( TBD es Z semi-inscrito en
la sem icircunferencia).

BD = 2 ( 34° ) = 6 8 °

Entonces: Á = -52. = — _> Á = 34°

Por ser Z exterior al A ABE: TEP = A + ABE


TÉP = 34° + ( 90° + 34° )
TÉP = 158°

Finalmente; reemplazando en (1 ) : TP + 158° = 180

TP = 22

Rpta: ( C ).

Desde un punto P exterior a una circunferencia, se trazan las tangentes PC y PD, formando

entre si un ángulo recto. Luego se traza la secante PAB, de modo que BC = 2 A C .

Hallar la medida del ángulo BPC.

A) 30° B) 45 C) 22°30l D) 7°30 E) 15

Solución
Incógnita: BPC = x

Por fórmula del ángulo exterior


a la circunferencia:

BC - AC 2a - a D
x =

a
• ■ x = 2 (1 )

En la circunferencia:

ABC = -^2. ( z inscrito) -> ABC = ^ ....(2).

299
Luego: ABC = x....( Por las relaciones ( 1 ) y ( 2 ) )

Así, el A BCP resulta isósceles: BC = CP

De otro lado, trazando los radios OB, OC y OD : CP = r BC = r

Entonces, el A OBC es equilátero BC = BOC = 60° 2 a = 60

a = 30°
30
En (1 ) : x =

Rpta: (E).

En la figura, T y B, son puntos de tangencia


El arco AB mide 6 8 ° y el arco BC 104°.
Hallar la medida del ángulo ATB.

A) 81' B) 71 C) 84
D) 76 E) 72

Solución
Datos: AB = 6 8 ° ; BC = 104

Incógnita: ATB = x

Por ser Z interior a la mayor circunferencia:


| _ AB + CD

i 689 + a „
x = ----------- -> a = 2 x - 6 8 9 ( 1)

Por B, trazamos la tangente m, común a ambas circunferencias

BT
Entonces: B = — = t ( Z in scrito ) B= t = x
i 2

( Z sem i-inscrito)

Z \

Luego: BAD = 2B BAD = 2x ( En la mayor circunferencia )

Finalmente, como: BAD + DC + CB = 360°. Entonces: 2x + a + 104° = 360

Con lo de (1 ): 2x + 2x - 6 8 ° + 104° = 360 x = 81


Rpta: ( A ).
En la figura, el arco AB mide 82°
T y F son puntos de tangencia.
Hallar el valor de x.

A) 82' B) 41' C) 49
D) 45 E) 60

Solución

Por ser DF y DT, tangentes a la menor


circunferencia: DF = DT y F = T ( 1)

En la mayor circunferencia: BC = 2x

También: BCT = 2T ( t es semi-inscrito )

CT = BCT - 2x (2 )
T
822 + CT
Siendo F in te rio r: F=

82s + BCT - 2x
Con lo de ( 2 ) : F= 2F = 82° + BCT - 2x
i
2F = 82° + 2 t - 2 x

Según (1 ) : F= T o = 82° - 2x -» x = 41

Rpta: (B)

En la figura, C y D son puntos de tangencia

AB mide 64°.
Hallar la medida del arco CE.

A) 128' B) 64° C) 72

D) 144 E) 148
Solución
CE = ?
Trazamos BD:

AB 64'
BDA = -> BDA = 32

- Al trazar CD *

CE = 2x (D
301
- En el A ACD : Á + C = x.....( Z exterior) <}>+ a = x (2)

- Por D, trazamos la tangente común m. Entonces

DC
RDC = C = — -> RDC = a
2

BD
También: BDR = Á BDR = Á = <b
A T

i
Z semi-inscrito Z inscrito

- En el punto D: 32° + <j) + a + x = 180

Con lo de ( 2 ) : 32° + x + x = 180° x = 74

Reemplazando en (1 ): CE = 148

Rpta: (E)

39)
En la figura adjunta, EB y FB son tangentes
Demostrar que:

Al trazar A B :

C O A
EBA APB y ABF
= ------- a ó r r = -----
AB
2 2
( Z s semi-inscritos )

Entonces:
APB + AB
EBF = I.q.q.d

40)
Las tres circunferencias de la figura, son
congruentes y T punto de tangencia entre

dos de ellas. ABT mide 52°.

Hallar la medida del TQC.

A) 104° B ) 142° C ) 128°

D ) 144° E ) 154°

302
Solución

Por T, trazamos la tangente MT común


interior a las dos circunferencias tangentes
exteriores y la tangente ER a la tercera.

Como son congruentes las circunferencias:

AT = ABT AT = 52

Por la propiedad del problema anterior:

E fF = AT + ABT = , E tF = 52

Entonces: F ÍR = 180° - ETF FTR = 128'

TC + TQC
Además: FTR = 128° = TC + TQ_?. , pero TC = TQC

r
TQC = 128

Rpta: (C)

O y Q son centros de las circunferencias


mostradas en la figura.
Hallar la medida del ángulo que forman las
tangentes por los puntos A y B.

A) 45' B) 60 C) 75
D) 90 E) 53

Trazamos las tangentes por A y B

Incógnita: x

Trazamos además OP y OB BE X OB

A POB, isósceles : P = (3 = OBP

AP A

En la menor circunferencia, por teoría del ángulo semi-inscrito: a = — = P A = P

Observamos que los ángulos correspondientes A y ABO , son congruentes :

( Á = p = ABO ) ÁÉ 11 OB , Entonces, como OB X BE, será x = 90°

Rpta: ( D ).

303
En la figura, AB y AC son diámetros
de las semicircunferencias. Si T es pun
to de tangencia y el arco AE mide 78°,
hallar la medida del arco FC.

A) 78 O B) 39° C) 90
D) 51 o E) N.A.

Solución

Sean a y x, las medidas de EF y FC,


respectivamente. Luego:
78° + a + x = 180°

a + x = 1 0 2 °......( I ).

Sabemos, por propiedad demostrada en un problema anterior;

AT = AEF -> AT = 78° + a (II).

AT
Angulo semi-inscrito en la menor sem icircunf: T =

AE + FC
Por ser ángulo interior a la mayor semicircunferencia: f =

AT AE + FC /
Entonces: AT = AE + FC /. con lo de ( I I ) y el gráfico

78° + a = 78° + x —> a = x

Reemplazando esto, en ( I ): x + x = 102

x = 51

Rpta: ( D ).

En la figura, A y B son puntos de tangencia.

BD mide 102° y BC mide 114

Hallar el valor de x.

A) 14'
B) 12'
C) 10
D) 15'
E) 16
Solución

Por B, trazamos la tangente común PQ.


Entonces, por ser ángulos semi-incritos: M
✓ \
QBD = — = -> QBD = 51°

BC 114
QBC = QBC = 57°

También: PBA = QBC (opuestos por el vértice)

PBA = 57°. Entonces: Á = PBA = 57

Además: PBE = QBD ( opuestos ) -> PBE = 51

Finalmente, en el A ABM: x + A + PBA + PBE = 180'

x + 57° + 57° + 51° = 180' -> x = 15

Rpta: (D).

En la figura: APD + BPC = 216°.


Hallar la medida del ángulo AZC, donde Z es
el punto de corte de las prolongaciones de
AB y CD.

A) 54° B) 72 C) 108
D) 36° E) 39

Considerando el gráfico adjunto


Z = ?

Trazamos A P . B P . E P . D P y CP:

AB
APB = — = AEB = a
' —N
DC
DEC = AEB -> DEC = = DPC = a

a BE a
BAE = — = BPE = (3

ED
EPD = — = ECD = <)>

305
- Reemplazando en el dato:
APD + BPC = 216'

(a + p + <D) + ( p + <|> + a ) = 2 1 6 ° -> 2 ( a + (J + ó ) = 2 1 6 9

a + P + 4> = 1 0 8 °...(1 )

- En el A EDC, por ser Z exterior: EDZ = a + <|>

En el A AZD: ZAD + Z + EDZ = 180'

Con lo a n te rio r: p + Z + a + <|) = 180'

Z + a + p + $ = 180'

Usando lo de (1 ): Z + 108° = 180

De donde: Z = 72

Rpta: ( B ).

45) Hallar el perímetro de un trapecio circunscrito a una circunferencia. La mediana del trapecio,
tiene longitud “m”.

A) 2m. B) 3m C) 4m. D) 8 m. E) 6 m

Solución

- Incógnita: a + b + c + d = ?

- Dato: m.

o + K
Por fórmula de la mediana: --------- = m

a + b = 2 m.

- Según Pitoth: c+d = a+b c + d = 2m

Entonces: a + b + c + d = 4m

Rpta: ( C ).

46) Se tiene un pentágono convexo ABCDE, circunscrito a una circunferencia, de modo que
AB + CD + AE = 11 y BC + ED = 7.

Hallar la longitud de la tangente trazada desde “A” a dicha circunferencia.

A) 2 B) 3 C) 4 D) 2,5 E) 3,5

306
Solución

Incógnita: x

Considerando el gráfico adjunto y los datos

AB + CD + AE = 11
x + m + n + q + x + e = 11
2 x + m + n + q + e = 11 (I)

También: BC + ED = 7
m+n+q+e = 7 (M )

Sustituyendo lo de ( I I ), en ( I ):
2x + 7 = 11 -> x = 2

Rpta: ( A ).

En la figura, hallar el valor de r

Si BE = FG y BH = 14.

A) 14 B) 3,5 C) 10
D) 8 E )7

Solución
r = ?
Tangentes: AF = AE y HC = GC. Por el Teorema de P oncelet: A B + B C = AC + 2r

luego: ( AE + EB ) + ( BH + HC ) = ( AF + FG + GC ) + 2r

t t íí
dato: iguales

Cancelando las longitudes iguales a ambos miembros:

BH = 2r -> 14 = 2r /. r = 7

Rpta: (E )

Se tienen tres circunferencias de radios 1,2 y 3 unidades, tangentes exteriores entre sí, dos
a dos. El radio de la circunferencia inscrita al triángulo formado al unir los centros de las
primeras circunferencias, es:

a) 1 b) 2 c) 1,5 d) / 3 e) /2

307
Solución
Como los lados del triángulo obtenido
tienen longitudes 3, 4 y 5, se trata de
un triángulo rectángulo.
Luego, por el teorema de Poncelet:

3 + 4 = 5 + 2x —> x = 1

Rpta: ( A ).

En un triángulo ABC, AB = 14, BC = 12 y AC = 10, la circunferencia inscrita es tangente


a AB en el punto E. Hallar BE.

A) 5 B) 6 C) 8 D) 7 E) 4,5

Solución
Consideremos el gráfico adjunto, donde

a = 12 , b = 10 y c = 14.

Tangentes:
BE = BG = X : AE = AF = y ;
CG = CF = z
^ A
Incógnita: BE = X

Se tienen:
AB —» x+y = c .... ( 1 ).

BC -+ x + z = a ..... .... ( 2 ).
AC —> y+z = b .... ( 3 ) .

Sumando miembro a miembro las relaciones (1 ), ( 2 ) y ( 3 ) :

a + b + c
2 (x + y + z ) = a + b + c .\ x + y + z =

a + b + c a + b +c
Con ( 3 ): x + b = x = ------------------ b

Haciendo: a + b + c = 2p ( p, es el semiperímetro del AABC)


( Fórmula)
o , ^ * 12 + 10 + 14
Con los datos: x= -------------------------10 -> x= 8

Rpta: ( C ).

Nota.- Además:
En un A ABC, la circunferencia ex-inscrita relativa al lado B C , es tangente a este lado en el
punto G y a las prolongaciones de AB y AC en E y F, respectivamente. Demostrar, que:

, siendo p el semiperímetro del AABC

Solución •

En el gráfico, por ser tangentes trazadas desde


el mismo punto se han hecho marcas iguales:

AE = AB + BE

AF = AC + CF

Sumando: AE + AF = AB + AC + BE + CF

1 1

2AE = AB + AC + BG + GC, ya que AF = AE ; BG = BE y GC = CF

2AE = AB + AB + AC + BC
AE

• i.^ c|.d
. i

) En la figura adjunta, T, Q, E, F, son puntos de tangencia. AB = 1 2 y A C = 18.

Hallar TQ.

A) 6 B) 3 C) 9 D) 4 E) 5

Solución
- Se tiene:
TQ = BQ - B T ( 1 ).

- Con propiedades demostradas en los


dos últimos problemas y el gráfico:
BQ = BE = AE - AB = p - AB
y BT. = p - AC

- Reemplazando en ( 1 ):

TQ = ( p - A B ) - ( p - AC )
C F
( Propiedad)

Con los datos:


TQ = 1 8 - 12 -> TQ = 6

Rpta: ( A ).

309
52) La suma de longitudes de los radios de las circunferencias ex-inscritas relativas a los catetos
de un triángulo rectángulo, es 18. La longitud de la hipotenusa, es:

A) 9 B) 6 C) 12 D) 36 E) 18

Solución
Sea el A ABC de la figura;
incógnita: AC.
Dato: ra + rc = 18

Si p, es el semiperímetro del triángulo,


sabemos, por propiedad demostrada
en un problema anterior, que:

AQ = p AB + ra = p

CH = p —> CB + r. = p

Sumando: AB + CB + ra + rc = 2p
AB + CB + ra
a + rC = AB + CB + AC

De donde : Propiedad.

Es decir: “En todo triángulo rectángulo, la suma de los exradios relativos a los catetos, es
igual a la longitud de la hipotenusa”.

* Para el problema, según dato: ra + rc = 1 8 • • AC = 18

Rpta: (E ).

53) En un triángulo ABC, recto en B, el inradio es r y el exradio relativo a AB : r.

Demostrar, que:

Solución
Sean I, E, los centros de las circunferencias C
en mención. Se observa : AB = AV + VB,
siendo: VB = r AB = AV + r ... (1 ).

B
Por propiedad: A IC = 90® + — -> A IC = 135s B

A ÍE = 45°. Además, se sabe que Al 1 AE

Entonces, el A IAE es rectángulo e isósceles.


Luego: Al = AE. También: AÉP = IÁV = a
( tienan sus lados perpendiculares ). Así:
A A P E = A IVA AV = EP .\A V = r_

- Reemplazando, en ( 1 ): AB = ra + r l.q.q.d

310
54) Demostrar, que: “En todo triángulo rectángulo, el exradio relativo a la hipotenusa es igual
al semiperímetro”.

Solución

Consideremos el A ABC, recto en B. rb,


es la longitud del exradio relativo a la hi­
potenusa AC y p el semiperímetro.
Por propiedad demostrada en un proble­
ma a n te rio r: BL = p B C

Pero: ET = BL « •
rh = P l.q.q.d

55) En un triángulo ABC, recto en B, sobre AC se toma el punto F y se trazan FP ± AB y


FQ 1 BC, respectivamente ( P y Q en AB y BC). Hallar el inradio del A ABC, si los
inradios de los triángulos APF y FQC, miden 3 y 4cm, respectivamente.

A) 5cm. B) 7cm. C) 3,5cm. D) 9cm. E) N.A.

Solución
B
Consideremos el gráfico siguiente

- Datos: r! = 3 y r2 = 4

- Incógnita: r

- Con el teorema de Poncelet:

A ABC -> AB + BC = AC + 2r ...( 1 ).


A APF -> AP + PF = AF + 2rt ...( 2 ) .
A FQC -> FQ + QC = FC + 2r2 ...( 3 ).

Sumando las expresiones ( 2 ) y ( 3 ), miembro a miembro:

AP + FQ + PF + QC = A F + FC + 2r, + 2r2

i i
Es decir: AP + PB + BQ + QC = AC + 2r1 + 2r 2

= AC + 2r 1 + 2r2 ...(4 )

Ahora, igualando los segundos miembros de ( 1 ) y ( 4 ) :

AC + 2r = AC + 2r 1 + 2r2

Reemplazando valores:

Rpta: ( B ).
311
56) En la figura:
Perímetro del A AEF 2P,
Perímetro del A MBN 2 P2
Perímetro del A TQ C 2 P3
Hallar el perímetro del A ABC,

si Pl + P2+ P3 = 18
A) 18 B) 27 C) 32
D) 36 E) 72
Solución
Haciendo uso de propiedad demostrada
en un problema anterior, tenemos el si­
guiente gráfico:

Entonces; el perímetro del A ABC, es:

AB + BC + AC = 2p 1 + 2p 2 + 2p 3
AB + BC + AC = 2(p, + p2 + p3)

AB + BC + A C = 2 ( 1 8 ) = 36

Rpta: ( D ).

Nota.- En general, se tendrá la propiedad

57)

En la figura, demostrar que el


cuadrilátero QEBF es inscríptible

Bastará demostrar que:


B + EQF = 180e. Veamos

- Trazamos QP. Luego:


A ABC -> Á + C + B = 180o... ( 1 )

- En los cuadriláteros inscritos:


□ AEQP EQP = 1809 - A
(* )
□ PQFC PQF = 180® - C

312
Pero, en el punto Q:
EQP + PQF + EQF = 360°

Con lo anterior ( * ): 180° - A + 180° - C + EQF = 360°

De donde : Á + C = EQF

Reemplazando esto último, en ( 1 ): EQF + B = 180°. Entonces podemos afirmar que


el d QEBF es inscriptible.

58) Sobre la hipotenusa AC de un triángulo rectángulo, se dibuja exteriormente el cuadrado


ACEF, de centro O. Hallar la medida del ángulo OBC.

A) 60° B) 30° C) 80° D) 45° E) 53°

Solución
- Como ACEF es un cuadrado:
AE 1 CF y EAC = EAF = 45°. B
El cuadrilátero ABCO es inscriptible,
ya que ABC + AOC = 180° A C

Entonces: OBC = OAC

OBC = 45°

Rpta: ( D ).

59)
En el gráfico adjunto: AB = 104° y EPB = 36°
( A y B son puntos de tangencia ).
P Hallar la medida del ángulo FBA.

A) 34° B) 17° C) 22°


D) 30° E) 40°
Solución
- Como A es punto de tangencia:

AE = AB

2 2
( Z semi-inscrito en la circunf. m a yo r).

AE 104°
- De otro lado: AFE = — = = 52°
2 2
( Z inscrito en la circunf. m e n o r).

313
- Entonces: AFP = ABP = 52°. Esto indica que el cuadrilátero AFBP es inscriptible.
Por lo tanto: FPA = FBA a = x.

Pero, en el A A P B : APB = 1 8 0 ° - 2 ( 5 2 ° ) APB = 76

a = APB - 36°
a = 76° - 36°
a = 40° —» x = 40

Rpta: (E ).

60)
En la figura, O es el centro del arco AB
PE X OA, PF 1 ÁB y P H lO B .

Hallar el valor de x.
B A) 102' B) 168 C) 129
D) 139 E) 142

Solución
- En el cuadrilátero no convexo EPHF:
x = a + 0 + E P H ...(1 ).
Siendo: EPH = EÓT
( Z de lados perpendiculares)
EPH = 7 8 ° .. ( 2 ).

- d EFPA, inscriptible ( AÉP = A FP ) ^ a = PÁF

- aPFHB, inscriptible ( PFB = PHB ) —> 0 = FBP

Entonces: a + <|> = PÁF + FBP


PB AP PB + AP 102
a + 0 =— + —> a + é = ------------- --- —> a + 0 = 5T (3 )
2 2 2

- Reemplazando ( 2 ) y ( 3 ), en ( 1 ): x = 51° + 78 x = 129

Rpta: ( C ).

61) En la figura , O es el centro de la circun


ferencia circunscrita al A ABC.
Hallar el valor de x.

A) 60° B) 75 C) 90
D) 120 E) 82

314
Solución
- En el A Q B X : x = a +ó (1 )

- El cuadrilátero AQHC es inscriptible, ya


que: AÓC = AHC.
A

Entonces : <t> = ACB

A AOB, isósceles: OA = OB = radio


OÁB = ABO = a
y AÓB = AB = 2p

Así: a + a + 2P = 180' a + p = 90'

ó : a + ó = 90°. Reemplazando, en (1 ):

Rpta: ( C ).

* Este problema dá la demostración del Teorema de Nagel.

62) Demostrar que: “En todo triángulo, las tres alturas


se cortan en un mismo punto” .

Solución
p\
Haremos la demostración considerando un trián­
PÁ *
gulo acutángulo.
ii
"A Q
Sea ABC el triángulo. Tracemos las alturas
/ >>
AQ y CP, las cuales se cortan en el punto L .
\ a\
Prolongamos BL hasta H. Tenemos que demos­
P V
trar, que: BH _L A C ; es d e c ir: x = 90°. t H
Veamos: A BHC —» x = a + P + ó ... (1 ).
( Z e xte rio r).

- d PBQL -» inscriptible, ya que los ángulos opuestos LPB y LQB suman 180°.

Entonces: LPQ = LBQ = p

- aAPQC -> inscriptible, porque : APC = AÓC = 90Q. Por lo tanto : PÁQ = PCQ = a ,

QÁC = QPC = p y PCA = PÓA = <t>-

Luego, en el A AQC : a + p + ó = 90°.

Reemplazando en (1 ): x = 90°. Es decir, BH es la tercera altura del A ABC.

315
Observaciones B

El punto L, se llama ORTOCENTRO del


A ABC.

Los vértices A, B, C; el ortocentro L y los


pies de las alturas: P, Q, H; determinan
cuatro cuadriláteros inscriptibles: PBQL,
LQCH, APLH y APQC.

* En el caso de un A rectángulo * En un triángulo obtusángulo:


C

/ \
* s
\ \
\ N
\

63) Hallar el valor de x, s i : AB = RC.

A) 10 B) 12 C) 9 D) 15' E) 8

Solución

Una forma de resolver estos problemas consiste


en formar un cuadrilátero inscriptible.A sí:

- Prolongamos BR hasta F, de modo que: B

ACF = ABF
Es decir: ACF = 6 x

Entonces, el üA B C F es inscriptible.
Por lo tanto: CÁF = FBC -> CÁF = 7 x

- Además , se observa que:


BCF = x + 6 x ^ BCF = 7 x /. El ABFC
es isósceles: BF = FC. Esto indica que
A RCF = A ABF ( LAL ) RF = AF.
316
Resultando el A AFR, Isósceles : a = 7x.

Finalmente, en el A BRC : a + 7x + x = 180°

i
7x + 7x + x = 180°.

De donde: x = 12

Rpta: ( B ).

64)
En la figura, M es punto medio de A B .
Demostrar que EM = MF. ( DS y CL
son cuerdas cualesquiera , que pasan
por M ).

D em ostración:
* Z DCL = Z DSL

* Se traza un diámetro que pase por M


OM _L ÁB

* ST X OM, Luego:

* PT = PS y MT = MS

* Z M T S = Z M S T y como A B ||S T ,
fentonces: Z EMT = Z MTS y
Z SMF = Z MST
( alternos internos entre paralelas ).

* En dCDST, inscrito, el Z TCD es suplemento del Z MST.


Z T C D y Z EMT son suplementarios.
Luego, el aECTM es inscriptible .*. Z E T M = Z E C M = DCL

* Finalmente: ZETM = ZM SF
MT = MS A EMT s A FMS
Z EMT = ZSM F ( A.L.A.)

EM = MF . . . . . . . . . . I.q.cj.d .

65) En la figura mostrada, AQ = BC. B


Hallar el valor de x.

A) 24‘ B) 12 C) 30
D) 48 E) 15

317
Solución
x = ?

Se dibuja la circunferencia, de centro O, B


circunscrita al A ABC; Entonces:

OA = OB = OC = radio

AB = 2( AÓB ) -> AB = 48°

y BC = 2( BÁC ) BC = 60° V

, /
s »»

A BOC, equilátero. BC = radio.

AÓB = -> AÓB = 48° ( Z c e n tra l).

A AOC, isósceles: AOC = ABC = 108° y OÁC = ACO = 36°

A AOQ, isósceles:
AÓQ = AQO = 72° BÓQ = 24° QÓC BÓC - BÓQ = 60° - 24° = 36°

A OQC, isósceles: OQ = QC

Finalmente: A BOQ = BCQ ( LLL ) .*. a = x ; siendo: a + x = 60° a = x = 30°

Rpta: (C)

Se muestra un triángulo y dos circun­


ferencias ex-inscritas. Demostrar, que:

AT = CF

Solución
Sabemos que, si p es el semiperímetro del A ABC:

AF = p y CT = p

Entonces: AF = CT AC + CF = CA + AT

De donde : CF = AT I.q.q.d.

Mostrar que: “ En todo triángulo, la mediatriz de un lado y la bisectriz del ángulo opuesto,
se cortan en la circunferencia circunscrita

Solución

Sea el A ABC; FM es mediatriz de AC. AM = MC. Entonces: ABM = MBC

Además: AF = FBC
Entonces, como:

~ FBC
htíh = —— ... ( z ex-inscr¡to ).

AP
FBA = — ... ( ¿ inscrito ).

Por lo tanto: EBF = F B A . Es decir, BF


biseca el ángulo exterior EBA, del AABC

68) En un A ABC, de incentro I ; excentro relativo a B C : E. IE corta a la circunferencia


circunscrita al A ABC, en el punto M. Demostrar, que:

Solución
- Con el gráfico; observando los ángulos:

- A IBM —» Isósceles IM = BM ...( 1 )

- Para el A ABE, el ángulo exterior en B:


ot + <J) = ex -h E -» E = ó

A BME es isósceles:

EM = B M ( 2 ).

De ( 1 ) y ( 2 ) : IM = EM .......... l. q . q . d

69) En un A ABC, de inradio r; rh es exradio relativo a A C . Hallar la longitud de la flecha MF del


lado AC.
A^ rb + r B) rb - r C) rh + r D) rh - r E) 2 ( r h - r )
2

Solución
Sean: I incentro
E excentro relativo a AC

Al trazar IT 1 AC y EH _L AC:
IT = r ; EH = rb

Sabemos, por el anterior problema,


que IM = EM.
El □ HITE, es un trapecio y MF une
los puntos medios de las diagonales

HT e IE . Entonces:

Rpta: ( B ). (P ropiedad)
319
70) Demostrar, que: “ En todo triángulo, la suma de las longitudes de los tres exradios, es igual
a la suma del inradio y cuatro veces el circunradio”. ( Relación de S te in e r).

Solución
Sea el A ABC:
Del gráfico, MN es mediatriz de BC AN
biseca el ángulo externo A dei triángulo ABC.
“E ” y ME ” son dos excentros del triángulo.

Se sabe que:

BQ = CT Problemas

MH = —— - Anteriores

Así: QH = HT. Esto indica que HN es


mediana del trapecio QTEbEc :

Además, MN es diámetro de la circunferencia circunscrita

Es decir: MN = 2R MH + HN = 2R

ra - r

De donde:
I.q.q.d.

71) En un triángulo acutángulo ABC, de circuncentro O, OM, ON y OH, son las distancias a los
lados AB , BC y AC , respectivamente. Demostrar, que:

( Teorema de Carnoth )

donde: R circunradio.
r inradio.
B
Solución
Del gráfico, apoyado en un problema

anterior: EM = — - ¡se puede escribir

rc - r
OM = OE - EM OM = R -
2 ,
320
Análogamente:

ON = R - fe - r
\

OH = R - ...(3 )
v /

Luego, sumando (1 ) + ( 2 ) + ( 3 ):

OM + ON + OH = 3R - ra + rb + rc ” 3 r
\

Pero, según la relación de Steiner L,a + brK +e r„ = 4R + r

Reemplazando
/
4R + r - 3 r N
OM + ON + OH = 3R -
V
De donde:
OM + ON + OH = R + r .......... l.q.q.d.

Nota: Si el A ABC, es obtuso en B:

OM + O N - O H = R + r

72)
En la figura r , , r2 , r3 y r4 , son
inradios de los triángulos ABC,
BCD , ACD y ABD , respectiva
mente. Demostrar, que:

(Teorema Japonés)

Solución
Sean “O” y “R", centro y radio de la circun­
ferencia circunscrita al cuadrilátero ABCD,
respectivamente.

Usando el Teorema de Carnoth.


( Prob. ant.)

En el A ABC: OM + ON - OQ = R + r.

321
En el A ACD : OH + OP + OQ = R + r3

Sumando: m.a.m.

OM + ON + OH + OP = 2R + r, + r3 ( 1 ).

También:

En el A ABD : OM + OH + OT = R + r4

En el A BCD: ON + OP - OT = R + r2

Sumando: m.a.m.

OM + ON + OH + OP = 2R + r2 + r4 ( 2 ).

De ( 1 ) y ( 2 ) :

2R + r, + r3 = 2R + r2 + r4

" r! + f 3 = r2 + r4 ........... '‘W *

* El descubrimiento de este teorema, se debe al matemático japonés MIKAMI KAYASHI.

73) En un triángulo rectángulo ABC recto en “B”, se traza la altura BH. Si r, rt y r2, son inradios
de los triángulos ABC, AHB y BHC, respectivamente ; demostrar, que:

BH = r + ^ + r2
Solución
Con el Teorema de Poncelet:
B
En el A AHB : BH + AH = AB + 2r,

En el A BHC : BH + HC = BC + 2r2

En el A ABC : AB + BC = AC + 2r

Sumando m .a .m. estas tres expre­


siones y cancelando términos a am­
bos lados de la igualdad;
( AH + HC = AC ), queda:

2BH = 2r, + 2r2 + 2r BH = r1 + r2 + r I.q.q.d.

74)
B C El cuadrilátero ABCD de la figura, es cir­
cunscrito. AC 1 BD , a, b, c y d, son
inradios de los triángulos APB, BPC, CPD
y APD, respectivamente.

Demostrar, que:

a+c = b+d
Solución
Con el Teorema de Poncelet

En el A APB : AP + PB = AB + 2a

En el A C P D : PD + PC = CD + 2c

Sumando m. a. m. estas dos expresiones:


( AP + PD ) + ( PB + PC ) = AB + CD + 2a + 2c

( AD + 2d ) + ( BC + 2b ) = AB + CD + 2a + 2c

o : AD + BC + 2d + 2b = AB + CD + 2a + 2 c ( ^ ).

Pero: AD + BC = AB + CD, por el Teorema de Pitoth

Cancelando las sumas iguales de ( 1 ): queda:

2d + 2b = 2a + 2c

b+d = a+c I.q.q.d

Dado un triángulo ABC, inscrito en una circunferencia; sobre AB se toma el punto P 1 y sobre

BC el punto P2. Los puntos tomados sobre estos arcos, determinan las rectas de Simpson
\

Li y 1- 2 , que se cortan formando un ángulo de medida x.

Demostrar, que:

Solución
En el gráfico, tenemos

Q 1 A C , PjS X A B , P{T 1 B C , para y

P2N 1 AC, P2V X BC, P2H X AB, para L2 .

Se prolonga RjQ y P2N , hasta E y F, res­

pectivamente. Luego se trazan P¡A y P2C


AE
Entonces: ABE = — = AF\E = a
2 1
En el ü A P ^ Q , inscriptible: ASQ = AP|Q

ASQ = a ASQ = ABE

; decir: SQ 11 BE, ya que los ángulos correspondientes ASQ y ABE, son congruentes,

[álogamente: FBC = = FP2C = (3 y en el dN V P 2C, inscriptible: NVC = NP2C

323
-> NVC = (3 /. NVC = FBC -> BF 11 VN

A s í: EBF = x ( Por la teoría de Z s de lados paralelos ).


\

EF
Siendo: EBF = — y EF =PTBP2 , ya que P^E I P2 F ;

i ■ r ñ c Pl^Fp P1 BP2 , j
Concluimos, que: EBF= —1 -> x = —1— - I.q.q.d
2 2

i.v ^* * ¡s . • ,♦

. , V i ' - - - v

ENTERATE: ■vr* w j¡^ . n,^-- • • • ' . • .

La estrella pentagonal, era el emblema de los : ^ ^ M i -


miembros de la famosa Escuela Pitagórica,
que Pitágoras (569 r5 0 0 A ;d e C ^ fu n d ó en
•\ •*'♦* 4 ^

Crotona. En las puntas estaban las


la palabra griega * S a lu d ", figura que debía |
-fr r f- . • ' " $:&?•

ser dibujada sin levantar la punta del Lápiz;


■m* J' NvX' ,

&0 ?

Se afirma que fueron ellos quienes construyeron el pentágono regular;
pentagonal y los cinco poliedros regulares. :.u;.Wbf-y--i ■: :::o *■
• » #w?
tv:: ..
.■í jaSv-

Es más conocido Pitágoras por su famoso teorema referente al triángulos n


x >|jfc*£rs^
V :5

3*24
r
CAPITULO 1 0

PUNTOS NOTABLES DEL TRIANGULO


1. CIRCUNCENTRO ( O ).- Es el punto de Intersección de las mediatrices del triángulo;
equidista de sus vértices y es el centro de la circu n fe re n cia
c irc u n s c rita ai triángulo.

OA = OB = OC

TRIANGULO ACUTANGULO TRIANGULO RECTANGULO

O Circuncentro

R Radio de la Circunferencia
circunscrita ( circunradio ).

Es un punto interior -» A acutángulo.


O Está sobre un lado -» A rectángulo.
Es un punto exterior Aobtusángulo

TRIANGULO OBTUSANGULO El ángulo AOC = 2B

2. ORTOCENTRO ( L ).- Es el punto de intersección de las alturas de un triángulo.


B

-3-

TRIANGULO ACUTANGULO
TRIANG. RECTANGULO TRIANGULO OBTUSANGULO
El ángulo ÁLC = 180o- B

325
BARICENTRO ( G ).- Es el punto de intersección de las medianas de un triángulo. Divide
a cada mediana en dos segmentos tales que uno es el doble del otro.

Es el centro de
1 gravedad de la
AG = 2GH ó : GH = — AM, AG = - A M región triangular
3 3
ABC.
t
BG = 2GP ó : GP = - B P , BG = - B P 2p \M
3 3 q I G 'rn

1
CG = 2GQ ó : GQ = - C Q , CG = - C Q
3 3 A

INCENTRO ( I ).- Es el punto de intersección de las bisectrices interiores de un trián­


gulo, equidista de los lados y es el centro de la circunferencia inscrita
al triángulo.

I Incentro
B
r radio de la circunferen
cia inscrita ( inradio ).

El ángulo A lé = 90° + ^

EXCENTRO ( E ).- Es el punto de intersección de dos bisectrices exteriores y una


bisectriz interior. Cada triángulo tiene tres excentros los cuales son
puntos exteriores al triángulo , equidistan de los lados y son los
centros de las circunferencias exinscritas al triángulo.

Excentro relativo a BC

r —> Radio de la circunferencia exins-


crita relativa al lado “a” (exinradio)
A A

E lánguloC ÉA = ^ ,e lá n g u lo CEB = 902--^ -

TRIANGULO MEDIANO.- Es el triángulo que se forma al unir los puntos medios de los
lados de un triángulo. MNP es el triángulo mediano del A ABC.

El circuncentro del triangulo


total coincide con el ortocentro
del triángulo mediano.
TRIANGULO ORTIGO.- Llamado también triángulo PEDAL, es el triángulo que tiene por
vértices los pies de las alturas de un triángulo dado.

a) Si el A ABC , es acutángulo
A DEF triángulo órtico o pedal del A ABC

L Ortocentro del A ABC


L Incentro del A DEF

A, B, C, excentros del triángulo DEF

F órm ula: DEF = 180° - 2BAC

* Sedemuestra, por ejemplo, que FL es bisectriz; oADEC, üAD LF y nFLEC, son inscripti-
bles. Entonces: DÁL = p = DFL = DCE -» LFE = LCE = p. En forma análoga , para
DL y EL.

b) Si ABC, es un triángulo obtusángulo,


obtuso en A:

APQH -> triángulo ortico o pedal


del A ABC

Ortocentro del A ABC.

Incentro del APQ H.

L, B ,C Excentros del A PQH

PQH = 2BAC - 180

* Nota.-Un triángulo rectángulo no tiene A órtico.

RECTA DE EULER.- En todo triángulo, el Ortocentro, Baricentro y Circuncentro pertenecen


a una recta llamada recta de Euler para el triángulo.

L —> Ortocentro.
RECTA pE G —» Baricentro.
EULER
0 —> Circuncentro

Propiedad

Además se cumple que "la distancia del


Ortocentro a un vértice es el doble de la
distancia del Circuncentro al lado opues­
t o " : BL = 2 OM.

Q97
CIRCUNFERENCIA DE EULER.- Llamada también circunferencia de los nueve puntos; es
la circunferencia que pasa por los puntos medios de los
lados de un triángulo, por los pies de las alturas y por los
puntos medios de los segmentos que unen cada vértice
con el Ortocentro.

M,NjP. Puntos medios de los lados.

U ^W . Puntos medios de AL, BL, CL


( L es el Ortocentro ).

D,E,F. Pies de alturas

R’ -» Radio de la circunferencia
de EULER.
R
R’ = (R C ircunradio).

Ortocentro
LQ' = O 'O
O —> Circuncentro

Es fácil demostrar que los puntos M, N, P, U, V, W, D, E y F, pertenecen a la misma


circunferencia. En efecto:

* MN II AC y MF = — = NP El trapecio MNPF, isósceles, será inscriptible

* Además: A ALB MV AL
VMP = AÉB = 90°, p o r te n e r sus la ­
A ABC MP || BC dos respectivamente perpendiculares

Entonces, el d MVPF será inscriptible, porque VMP = VFP = 90°. Es decir, V estará en la
circunferencia que contiene a M, N, P y F.

En forma análoga, se prueba que D, U, W y E , están en la misma circunferencia.

O bservaciones:

1) En todo triángulo isósceles, la recta de Euler es perpendicular a la base y además en ella


están contenidas el incentro y un excentro. "

* Por ejemplo, Si AB = BC;

0 -> Circuncentro
G Baricentro / \
/ +

I —» Incentro /
/
/
t

L Ortocentro r
/

E
/
/

E —> Excentro

2) En todo triángulo equilátero, el Ortocentro, Baricentro, Circuncentro e Incentro, coinciden.


Cualquier recta que pase por este punto, representa una recta de Euler.

328
PROBLEMAS RESUELTOS

1) ¿Qué fracción de la longitud de la hipotenusa, es la distancia del Circuncentro al Baricentro,


de un triángulo rectángulo?

1 1 1 1
E) N.A
A>2 C> 4 D>6

Solución
- Sea ABC, el triángulo, recto en B

O Circuncentro
G Baricentro

- Incógnita : OG, conociendo AC.


1
- Sabemos, que : OG = —( O B ) ( Propiedad del Baricentro ).

AC
Además, por ser mediana hacia la hipotenusa : OB =

OG = l í ^ OG = —( AC )
3 , 2 / 6 '

Rpta: ( D ).

2) A ABC: —> Excentro.


Hallar “x " :

A) 100'
B) 105'
C) 110
D) 115
E) 120'

Solución

Como “ E " es un excentro , BE y CE


son bisectrices de los ángulos externos
B y C; además AE biseca el A.

Por propiedad:

/. ACB = 80

ABC ABC
AEC = 252 = ABC = 50

329
Luego BÁC = 50° y a = 25
Finalmente: x = 80° + a x = 105

Rpta: ( B ).

3) A ABC: O -» Circuncentro

Hallar V .

A) 60-
B) 70'
C) 80
D) 90
E) 75'

Solución

* Como “O” es el circuncentro:


OA = OB = OC ( equidista de los vértices ).

A BOC, isósceles: OCB = 30° y BÓC = 120°

* Por propiedad de este punto notable :


(O —» centro ).
\

BC = BOC = 2 A

120° = 2 A A = 60

* Finalmente, en el A ABD : x = Á + 20°, (Z externo) x = 60° + 20° -> x = 80°


Rpta: ( C ).

4) A ABC: Excentro

Hallar “0”

A) 70° B) 80 C) 90°
D) 75° E) 65

Solución
4_j\ B
* Como W
E” es excentro, BE y AE son
bisectrices de los ángulos externos y
CE del C. Luego: a = 10° y
0 - 60° + a , en el A ACR -> 0 = 70

Rpta: ( A ).

330
5) A A B C , UT Incentro.

Hallar “x”

A) 40' B) 35 C) 36
D) 45 E) 60

B
* I Incentro BD y AE son
bisectrices. Sabemos, por propiedad , que

A ÍB = 90° +

Luego : a = 90° + —

* En el d IECD:

a + 110° + x + 100° = 360 90° + - + no° + x + 1 0 0 ° = 360°

De donde: x = 40°

Rpta: ( A ).

6) A A B C , “O1 Circuncentro

Hallar V :

A) 40° B) 50 C) 30
D) 60° E) 45

Solución

* Como “O” es circuncentro, sabemos


por propiedad , que: AÓB = 2C
( OA = OB = ra d io ).
y = 2x

* EnelDODOE: y + 110° + x + 100° = 360

• • 2 x + 1 1 0 °+ x + 100° = 360

De donde: x = 502

Rpta: ( B ).

7) ABCD , es un cuadrilátero convexo. Hallar la medida del menor ángulo formado por
ÁC y BD, si: BÁD = 60°, ABD = 50°, DBC = 65°, ADB = 70° y BDC = 55°

A) 70' B) 80 C) 75 D) 90 E) 85
331
Solución
x = ?

Del gráfico, se observa que “C” es un Excentro


de! A ABD AC es bisectriz de B Á D : a = 3 0 °
y x = 50° + a
x = 80°

Rpta: ( B ).

Halar el valor de “x”, en la figura:

A) 30°
B) 24°
C) 32°
D) 26°
E) N A .

Prolongamos BR y AR, notando que:


A AHB : H = 90°
AAQB : Q = 90°

Luego, BH y AQ son alturas del A ABC

R -» Ortocentro

Entonces: CP _L AB

Por lo que, en el A APC: x + PÁC = 90°

x + 64° = 90° x = 26°

Rpta: ( D ).

En la figura AB = B C , Bl = CE y BCXCE.

Hallar “a ” si I es incentro del A ABC.

A) 15°
B) 10°
C) 18°
D) 20°
E) 25°
Solución

A ABC, isósceles: ACB = BAC = 29


A AIC, isósceles: Al = IC
A ICE = A A IB
(L A L )
a a

E = ABI
*

E= a

En el A ABC:
a
2 a + 40 = 180 0 = 45° - — (1 )
2

En el A ICE: 2 a + a + ( 9 O ° + 0 ) = 180

Con ( 1 ) : 3a + 45° - j = 90°

De donde : a = 18°

Rpta: ( C ).

En la figura H es Ortocentro, I es Incentro


Hallar “ a

A) 18°
B) 30°
C) 15°
D) 20°
E) 22°30‘

Solución

Como H es Ortocentro: CQ X AB
QCA = ABM ( de lados perpendiculares )

QCA = a
A A

I, es incentro. Entonces: BCI = ICM

/. BCI = 2 a

Además: MBC = MBA -> MBC = a


M
En el A BMC : MBC + BCM = 90°
a + 4a = 90° a = 18

Rpta: ( A ).
11) Sean Ev E2, E3, excentros del A ABC, cuyo incentro es I. Entonces, I, es e l d e l A E 1E2E3.

A) Incentro B) Circuncentro C) Baricentro


D) Ortocentro E) No es punto notable del A E1E2E3

Solución
\ V' __ -

Como: IB IB E 2 ,
IA 1 AEi,
IC 1 CE2 ,

E-|C, E2A y E3B , son alturas del


A E, E2 E3 , siendo i , su Ortocentro.

Rpta: (D)

12) Dado el triángulo ABC, recto en B, se dibujan exteriormente los triángulos equiláteros ABD
y BCE. Las prolongaciones de DA y EC se cortan en “P". PB corta a DE en “J”.

H aiar la medida del ángulo BJD.

A) 90 B) 60 C) 45 D) 75 E) 30

Solución

* Según gráfico; al prolongar EB y DB

EBH 1 AD y DBQ _L EC

“B” es el Ortocentro del A DEP

Luego, PBJ es también altura: x = 90

Rpta: ( A ).

13) Demostrar que en todo triángulo acutángulo ABC, de ortocentro L y circuncentro O :


ABL = OBC.

Solución

- En efecto. Prolongamos el radio BO


hasta F.

- En el A AHB: Si BÁH = a

ABL = 9 0 ° - a (D -

- Pero: BC = 2 BAH

BC = 2 a

334
F _ 2 ex ^ ^

Entonces, en el A BCF: OBC = 90° - a ( 2 ).

de ( 1 ) y ( 2 ) :

.......... l.q.q.d

Siendo O, el Circuncentro de un triángulo ABC ; O, ,0 2 ,0 3,Circuncentros de AOC, BOC y


AOB , respectivamente. Entonces, O es e l , del A Oy0 2Or

A) Ortocentro B) Incentro C) Circuncentro D) Baricentro E) No es notable

Solución mediatriz
mediatriz de OC—..
Como OA = OB = OC, se deduce que de OB
O equidista de los lados del A 0 1 0 20 3

O, es el Incentro del dicho triángulo. c


mediatriz
Rpta: (B) de OA

En un triángulo ABC, acutángulo, de Ortocentro L ; las prolongaciones de las alturas cortan


a la circunferencia circunscrita, en los puntos P, Q y F, respectivamente. ¿Qué punto
notable, es "L", del A PQF?

A) Ortocentro B) Circuncentro C) Incentro D) Baricentro E) No es punto notable

Solución

Si: BAL = a -» LCB = a


( Z de los lados perpendiculares)

Entonces: BP = 2BÁL -» BP = 2 a

FB = 2LCB

FB = 2a

FB 2a da_. BP 2a
Luego: FQB = — = a y BQP = — = a
2 y 2
Concluimos que QL es bisectriz del FQ P.
En forma análoga, se demuestra que PL y FL bisecan FPQ y QFP

En consecuencia: L es Incentro del A FPQ.

Rpta: (C).

335
16) En un A ABC, de ortocentro L y circunradio R; hallar la longitud del circunradio del A ALC

R
A) R B) C )2 R D) - R E) N.A.
' 2

Solución

- Sea el A ABC de la figura, con circuncentro


O; sean O’ y R \ circuncentro y circunradio
del A ALC, respectivamente.

- B “» ortocentro del A ALC.

- Sabemos que, “en todo triángulo, la distancia


del ortocentro a un vértice, es el doble de la
distancia del circuncentro al lado opuesto”. Así

AABC -> LB = 2( OM ) .x Entonces: OM = O’M.


A ALC BL = 2( O ’M ) >> , e, A OCO’ es isósceles.

( P ropiedad)
Rpta: ( A ).

* Nota.- Análogamente, los circunradios de los triángulos ALB y BLC, tienen longitud R, cada
uno.

17) En la figura: O -> centro

BC _L AD, A B = 12 y OH 1 CD

Hallar OH.

A) 12 B) 6 C) 8
D) 9 E )4

Solución

- Incógnita: OH

- Se traza BD y luego BQ _L CD

- Se observa ; que si ABC = a

—> AC = 2 a y ADC = a

Entonces: FBL = LDQ


( Z de lados perpendiculares)

FBL = a

336
A ABL —> Isósceles
BL = AB BL = 12

Para el A BCD: L Ortocentro.


O Circuncentro

Luego, por propiedad OH = — —> OH = — OH = 6


2 2

Rpta: ( B ).

18) En un triángulo acutángulo ABC, BH es altura. Se trazan HR AB y HQ 1 BC


Si RQ = 10. Hallar el perímetro del triángulo pedal de ABC.

A) 10 B) 15 C) 25 D) 30 E) 20

Solución
B
Sea MNH el triángulo pedal. Se pide :
MH + MN + HN.

Recordemos que A, B y C son excentros del


A MNH. Esto implica que MA y NC son
;F
bisectrices de los ángulos exteriores M y N,
respectivamente.

Prolongando HR y HQ hasta cortar a MN


en E y F:

A EMH, isósceles ( MR es altura y bisectriz )


EM = MH y ER = RH ( 1 )•

A HNF, isósceles ( NQ, altura y bisectriz ).


NF = HN y HQ = QF (2)

Ahora, usando el Teorema de los puntos medios ( Base Media ), en el A EHF: EF = 2RQ

Esto es EM + MN + NF = 2RQ

■•i' >!• ■l'


MH + MN + HN = 2RQ, ( Por lo hallado en ( 1 ) y ( 2 ) )

Es d e c ir: .... ( Propiedad, para todo triángulo acutángulo


y obtusángulo).

En el problema , como : RQ = 10

Perímetro del A MHN = 20

Rpta: ( E ).

337
Nota.-S¡ ABC, fuera un triángulo obtusángulo;
obtuso en B. BH es altura. M
HR _L AB y HQ 1B C . A MHN es el
triángulo pedal del A ABC.

Entonces:
Perímetro A MHN = 2RQ.

19) En la figura, AB es diámetro y H cualquier punto de AB . EM = HM y HN = NB

Hallar el valor de V .

A) 90° B) 120c C) 100


D) 150 E) N.A.

* Trazamos EB y luego prolongamos


NM hasta el punto Q.

* Como AB es diámetro: AEB = 90°.

* En el A EHB, MN es base media


MN II EB, luego: NQ 1 AE.

* En el A AEN, EH y NQ son alturas


“M” es el Ortocentro.

Entonces : AF _L EN : x = 90°

Rpta: (A).

20) Del gráfico.


Hallar el valor de “x”. si AB = DC.

A) 20° B) 22,5 C) 30
D) 18,5 E) N.A.

B
A fin de tener un triangulo congruente
al ABD, aprovechando que AB = DC,
ubicamos el punto E, tal que:
ECD = x y EC = BD

A DEC s A A D B
(LA L).
338
Luego: ED = DA y EDC = BÁD. Concluimos que ED I AB

Tam bién: A ADE, isósceles -> EAD = AED = a

Se observa que AE y CE son bisectrices interiores del A ABC E, es su incentro


Entonces BE, bisecará al ABC : DBE = x y EBC = 2 x

Como ED 11 AB : BDE = DBA = x A DEB, isósceles : BE = ED.

Finalmente,el trapecio ABED es isósceles. Luego BAD = ABE -> 2a = 2x


-» a = x y , en el A A B C : 2a + 6 x = 180°

De donde : x = 22,5°

Rpta: ( B ).

En la figura adjunta

AB = AR.
Hallar la medida del ángulo ARC

A) 120' B) 150 C) 135


D) 115 E) 105

Solución

ARC = x = ? B

Trazamos BR y luego AP 1 BR en H

como AB = AR , entonces :
BH = HR y BÁH = HÁR = a

A BPR es isósceles ya que PH _L BR


en su punto medio HPR = HPB = ó

En el A APC, AR y CR son bisectrices.


R -> Incentro del A APC. Luego, PR biseca el APC : RPC = RPA = ó

En P ^ 30 = 180° ó = 60° y APC = 120°.

APC 120°
Por propiedad : x = 90° + -------- = 90° + -------- -» x = 150°

Rpta: ( B ).

339
22) En un A ABC, se traza la ceviana interior BE. S i: AE = BC; A = 4x; C = 2x y EBC =
Hallar el valor de “x”.

A) 10° B) 15 C) 20 D) 30 E) 18

Solución B

* Trazamos AQ y EQ, de modo que :


QÁE = 2 x y QÉA = x. A s í:
AA Q E s A CEB ( ALA ).
QE = BE

* A QEB, isósceles; trazamos EM _L B Q .

* A AME : Q es incentro —> a = 60°, y a q u e : P + 2 a = 180’

* A AME : x = 10°

Rpta: ( A ).

23) En la figura.

AB = BC.

Hallar el valor de " x "

A) 20' B) 15 C) 45
D) 30 E) 60

Solución

* Sea BQ J_ AC BQ biseca AC y

ABQ = QBC = 2 r ya que AB = BC.

Además: ABE = r = EBQ.

* El A AQC, resulta isósceles.

* En el A ABQ, E es un excentro ( BE biseca


un ángulo interior y QE biseca un ángulo
e xte rio r).

AE biseca el ángulo externo A, de dicho


triángulo ( FÁE = EÁQ ). Por propiedad :

AQB 60
x= x = 30

Rpta: (D)

340
En un triángulo ABC, AB = BC y B = 120°; se ubican “O” , “l” , “E” , Circuncentro, Incentro
A

y Excentro relativo a BC, respectivamente. Hallar la medida del ángulo IEO.

A) 25 B) 30 C) 45 D) 20 E) 35

Solución

A ABE, Isósceles
BE = AB

Como “O” es circuncentro :


OA = OB; luego, el A AOB
es equilátero.
OB = OA = a

Finalmente, A OBE: OB = BE
-> OÉB = 45°
x + 15° = 45°
x = 30°

Rpta: IÉO = 30° ... ( B )

En la figura :
hallar ux”, si AB = BC.

A ) 130°
B ) 120°
C ) 150°
C D ) 135°
E) 165°

Solución

Se trazan AC; BH 1 AC; cortando a OC en M y


luego MA.

Como el A ABC es isósceles: AH = HC, MA = MC

ABH = HBC y MÁB = MCB.

O , es Incentro del A ABM , ya que AO y BO

bisecan los ángulos MÁB y ABM, respectiva­

mente. Entonces: OMB = OMA = r

r = 60
341
AMB
Finalmente, por propiedad : x = 90° +

x = 90° + r x = 150

Rpta: ( C ).

26) El A ABC, de la figura, es recto en B. I , I , , l2 , son


B
incentros de los triángulos ABC , AHB y BHC,
respectivamente.

Demostrar, que:

Solución

* Primero demostremos que Bl 1 h l2 . Para

ello, trazamos Ah y C l2, cuyas prolonga­


ciones se cortarán en el incentro I del A ABC. B
Sabemos, por un problema anterior, que las
bisectrices de los ángulos Á y HBC, son
p e rp e n d ic u la re s e n tre sí; ta m b ié n las
bisectrices de los ángulos Ó y ABH. Luego:
AP _L BP y CQ J_ BQ. ( Además, recorde­
mos que, para todo triángulo rectángulo tam ­
bién: ABH = ACB y

HBC = B Á C - )
De lo anterior, observamos que en el A I, Bl2 , h P e l2 Q son alturas. Por lo tanto, I es el

Ortocentro del A l, Bl2. Entonces: Bl 1 h l2 , tal como queríamos demostrar. ( BR es la tercera


altura del A L B L ). ^
1 ¿ Recordar que, para todo
triángulo rectángulo:
B _ —

fig . 2 .
A ABC
* Para demostrar que Bl = I, l2 , extraemos la fig. 3 de la fig. 1, donde \-\B\2 = = 45'

( Esto es fácil de probar en la fig. 1 ):

342
Se observa que : A I, PB Isósceles
fig .3.
B I, P = PB . . . ( * )

RBI2 = P l|l 2 = a . . . ( * * ) por tener el mismo


complemento p.

Luego, de ( * ) y ( * * ) : A IPB = A I, Pl2,


por tener un cateto y ángulos agudos, res­
pectivamente congruentes.
2
R Bl = I I

27) Demostrar q u e :MEn todo triángulo, la distancia del Ortocentro a un vértice, es el doble de
la distancia del Circuncentro al lado opuesto”.

Solución

Consideremos el A ABC, donde L y O son el


Ortocentro y Circuncentro, respectivamente.
Demostraremos que:
LB = 2 ( ON )

Para ello, trazamos MN y tomamos E y F, puntos


medios de BL y A L , respectivamente. Luego,
usando el Teorema de los puntos medios ( Base
M e d ia ): A H N C

— AR 'i
En el AABC MN 11 AB y MN = — .*. MN = EF
y 2

— — AR __
En el AALB EF 11 AB y EF = — MN 11 EF
2 J

Además, por tener lados respectivamente paralelos : FEL = OÑM y EFL = OMN.

De todo lo anterior, concluimos que los triángulos MON y FLE, son congruentes. ( Postulado:
Angulo-Lado-Angulo).

Por lo ta n to : EL = ON y FL = OM. Es d e c ir: LB = 2( ON ) y LA = 2( OM ); con lo cual


queda demostrada la propiedad.

Nota.-Esta propiedad es también válida en triángulos obtusángulos. Demuestre el lector.


¿Qué se observaría de tratarse con un triángulo rectángulo?

28) Demostrar que: “ En todo triángulo, el Ortocentro, Baricentro y Circuncentro, son colineales”
Además:
La distancia del Ortocentro al Baricentro es doble de la distancia del Baricentro
al Circuncentro ”.
343
Solución
Sea el A ABC, donde L y O, son el Ortocentro y
Circuncentro respectivamente. Como AM es
mediana, bastará probar que AP = 2PM, para
concluir que P es el Baricentro del triángulo. B
Para ello, usaremos lo demostrado en el proble­
ma a n te rio r: AL = 2( OM ). O

Si T y V, bisecan AP y LP, respectivamente; M


XV
. al /
/ 1
entonces, en el A ALP : TV 11 AL y TV = — , i / 4 (o \ *
I / t* A
X \ \
por el Teorema de los puntos medios. Además □ n
\
\ \
A H
por otro lado; OM 11 TV ya que OM AL.
— AL
También, recordemos que ; UM = — .

AL
Entonces, concluimos que el A MOP es congruente al A TVP, ya que OM = TV =

PMO = P t v y TVP = MOP . ( Postulado Angulo-Lado-Angulo ). Luego : TP = PM y


AP = 2PM, con lo cual queda demostrado que P es el Baricentro del A ABC.

* El ortocentro, Baricentro y Circuncentro de todo triángulo, son colineales. La recta que los
contiene es llamada RECTA DE EULER.

** Asimismo, como VP = PO y LP = 2( VP ), entonces : LP = 2( PO ), con lo cual queda


demostrado que “La distancia del Ortocentro al Baricentro es el doble de la distancia del
Baricentro al Circuncentro”.

Nota.- A pesar que, para la demostración de estas dos propiedades se ha usado un triángulo
acutángulo, los resultados son válidos también para los triángulos obtusángulos y los
rectángulos. El lector puede probar esto.

29) Dado un triángulo ABC, inscrito, sobre el arco BC, se toma el punto P y se gráfica la recta
de Simpson correspondiente. Si L es el Ortocentro de ABC, demostrar que la recta de
Simpson corta a LP en su punto medio.

Solución

- Sea t ,1a recta y M el punto de intersec­


ción entre ' y LP. Debemos demostrar,
que : LM = MP.

- Prolongamos BL hasta el punto Q.


A LAQ y A LEQ, isósceles.
A A

LQE = QLE

- Además: QPF = LQE = BCP

344
- aRFCP, inscriptible : RFP = RCP
FT es mediana hacia la hipotenusa del A EFP, ya que, por todo lo a n te rio r:
TFP = EPF. Entonces : ET = TP.

- Pero, como L Q Ü P F y QLE = RFP, entonces: FR 11 EL. Por lo tanto, M será punto medio
de LP, ya q u e e n el A E L P : TM IEL y ET = TP, con lo cual queda demostrado lo pedido.

30) El Teorem a de M orley

En 1899, F. Morley descubrió uno de los teoremas


más sorprendentes de la geometría ele m e n ta l:
“Los tres puntos de intersección de las trisectrices
adyacentes de los ángulos de un triángulo cual­
quiera, forman un triángulo equilátero”.

Así, para el A ABC a d ju n to , AQ, AR, BR,

' C BP, CP y CQ, trisecan los ángulos A, B y C,


obteniéndose el A PQR equilátero.

Es dificultoso intentar demostrar el teorema


en forma directa. Veremos que es más sen­
cillo empezar por el triángulo equilátero y
construir un triángulo general que se puede
identificar luego con el triángulo dado ABC.

Sobre los lados Q R , R P , P Q , de un A


equilátero, dado, se construyen exterior-
mente triángulos isósceles P’QR , Q’RP y
R’PQ en los que los ángulos de la base a,
p, r cumplan:
a + p + r = 1 2 0 °, siendo: a < 60°, P < 60°, r < 60

En seguida se prolongan los lados por debajo de sus bases correspondientes, hasta
cortarse en los puntos A, B y C.
Como a + p + r + 60° = 180°, podemos deducir:

* A AQR Q A R = 60° - a, puesto que Z RQA = a + p y Z ARQ = a + r

* Para el A P’BC, concluimos que P es su Incentro, dado que P'P biseca el Z P ' , siendo
1802 —2 ot
la mitad del ángulo en P’ : --------------- = 90° - a

Z P'
y Z B P C = 180° - a = 90° + ( 90° - a ) = 90° +

* Análogamente, Q es el Incentro del A Q’CA y R lo es de R’AB. Por lo tanto, los tres ángulos
parciales en C son congruentes; sucediendo lo mismo con los que están en A y B.

345
Es decir, se han trisecado los ángulos del A ABC.

Los tres ángulos pequeños en A, mide cada uno: A ( Z A ) = 60° - a ;


3

De modo que : a = 60° - A (Z A ).


3

También: (3 = 60° - A ( Z B ).
3

r = 60° - ■» ( Z C ).

Escogiendo los valores para los ángulos de la base de los triángulos isósceles, aseguramos
que el procedimiento anterior concluye en un A ABC, semejante al triángulo dado. Queda
así completa la demostración.

:. x * .

OBSERVACIONES

concurren en él.
interiores. Pero
♦ y.'
efectivamente» V ‘

H ip ó te s is : Sea el A ABC. Al y Cl,


:
bisecan los ángulos BAC y BCA, ///- . w ü H v '

w -

T esis : Bl, biseca el ángulo ABC.

- D em ostración , IP i AB , IQ JLBC Z-

\ ; -t
v.
.fwí<U.'V>
Para el A : IH * IP. kA%
^ AA/Z! v€«..i--:
'i: ; ■
A
* * *

Para el C : IH = IQ. r
- s> • ■ •^ .N

■ ' >'*> •" 1 vi-» -'v*- y

; £><: A . '

el B, ' •: 'í¿-'.• AN^SÍ í-"' . t.'V>


.»✓ y S*>
•• • .... ,.
■ v *ív> **••• •
< 1 'H a ', i .V' •

- De otro m odo: A IP B = A IQ B IB P s IB Q ,
i& llk
Baricentro, Circuncentro -
■*:*«*' «
- .
.v *. •:*
> > .v :,v>
& ':•:-•;•.y
v:*:--' *: :•
•v v
.iy:-y.
*« <*> >:v** •:. • '■.■ ■v.yxA\- • *
* sXw

346
CAPITULO 11

LINEAS PROPORCIONALES
Teorema de Thales.- Tres o más rectas parale­
las, determinan sobre dos o más secantes a
ellas, segmentos proporcionales.
SÍ. L -111 Í.2 11 L.3

Entonces:

AB DE BC EF
De aquí:
AC ~ DF ’ Á C “ DF

D em ostración

Al dividir AB y B C , en m y n partes iguales


de longitud a, cada una y trazar paralelas a
las dadas, DE y EF quedarán divididos en m
y n partes iguales de longitud b, cada una.

Luego:

AB _ ma m DE mb m
BC na n V EF nb n
AB DE
I.q.q.d
BC " EF

C orolario.- Toda paralela a uno de los lados de un triángulo, que intersecta a los otros o a
sus prolongaciones, determina sobre ellos segmentos proporcionales.
F- Si EF i I AC, entonces:
BE = BF
EA FC

347
Prim er Teorem a de la B isectriz.- En todo triángulo, una bisectriz cualquiera, determina
sobre el lado opuesto, segmentos que son entre sí como los lados que concurren con dicha
bisectriz.
B

a) Bisectriz b) Bisectriz
interior: exterior:

D em ostración de a ) :

Trazando CR 11 BD: R = ABD = a


( C orrespondientes)
y BCR = DBC = a ( Alternos ). /R
r *

A CBR, isósceles: « .

BR = B C (1 ).

En el A ARC, por el Teorema de Thales:

AD AB
DC “ BR

Con lo de (1 ): = ^2 l.q.q.d
DC BC

Nota.- Para demostrar b ), trace CQ EB. ( Q en AB ).

Teorem a del Incentro.- En todo triángulo, el incentro determina en cada bisectriz,


segmentos que son entre sí, como la suma de las longitudes de los dos lados que concurren
con dicha bisectriz, es a la longitud del tercer lado.

A ABC, I incentro

D em ostración

Por el primer teorema de la bisectriz :

Bl AB
A ABD —> ( 1)
ID ~ AD

AD AB AD AB
En el A ABC :
DC BC AC - AD BC
AB AB + BC
Efectuando:
AD AC

Reemplazando esto último, en (1 ) :


Bl _ AB + BC
l.q.q.d.
ID " AC

Teorema del E xcentro.- En la figura adjunta,


E es el excentro relativo al lado AC del AABC

Entonces:

D ivisión A rm ónica.- Se dice que los puntos colineales y consecutivos A, B, C y D,


constituyen una “cuaterna armónica”, si se cumple que:

A B D

Cuando esto se cumple, se dice que B y D son los conjugados armónicos de A y C


También suele decirse que B y D dividen armónicamente al segmento AC.

Relación de Descartes.- De la expresión anterior, se demuestra que:

Propiedad.- En todo triángulo, dos vértices y los pies de las bisectrices, interior y exterior,
que parten del tercero constituyen una cuaterna armónica.

AABC:

BD: Bisectriz interior


BE: Bisectriz exterior

D em ostración
Por el primer teorema de la bisectriz, aplicado al A ABC, para las bisectrices, interior BD
y exterior BE:
AD AB EA AB
DC " BC V CE _ BC

349
Igualando los primeros miembros de ambas expresiones

l.q.q.d

Nota.- Para recordar la forma de la expresión que relaciona a los segmentos de una cuaterna
armónica, tenemos:

r- 29------ 32
AB AD 4S
J1 B D
BC " CD 2e 3- 49

Haz A rm ó n ic o .-S e dá este nombre al conjunto de rayos OA, OB, OC y OD, tal que A,
B ,C y D constituyan una cuaterna armónica.

En la figura:
Li 11 l 2 11 l 3 L
AB = 2 , CD = 5 , GH = 6 ,
QR = 8 y PQ = FG + 2.

Hallar FG.
3
B) 1 C)
A>2

6
°> 7 E)

Solución
FG = ?

PQ = FG + 2.

Por el Teorema de Thales :


FG PQ FG FG + 2
(i)-
EF QR EF 8
EF _ AB EF = 2 12
Cálculo de EF :
GH CD 6 5

FG FG + 2
Reemplazando en (1 ) :
12 8
5
6
Efectuando : FG =
7

Rpta: ( D ).

En un triángulo ABC, se trazan la ceviana interior AR y luego: RE 11 AC y EF 11 AR.


( E sobre AB y F en BR ).

S i : BF = 5 y FR = 3 ; hallar RC.

A) 2,4 B) 8 C) 2 D) 4 E) 4,8

Solución
B
Incógnita: RC = x

Con el Teorema de Thales:


X AE
A ABC
8 ” EB (1 )

AE _ 3
A ABR (2 )
EB " 5

Reemplazando ( 2 ), en ( 1 ) :

x 3
x = 4,8
8 5

Rpta: (E).

En un trapezoide ABCD, las bisectrices de los ángulos B y D, se cortan en un punto E,de


la diagonal AC. Si AB = 15 , BC = 10 y CD = 12 , hallar AD.

A) 15 B) 17 C) 18 D) 19 E) 20

Solución
Por el Primer teorema de la bisectriz :

AE 15
A ABC
EC 10 (1 )

AE AD
A ACD (2)
EC " 12

351
Igualando los segundos miembros de ( 1 ) y ( 2 )

AD 12 AD = 18
12 10

Rpta: ( C ).

4) En un A A B C , A B = 20, BC = 10 y AC = 21, se trazan las bisectrices, interior BD y exterior


BE.
Hallar DE.

A) 21 B) 25 C) 28 D) 30 E) 29

Solución
- Incógnita : DE = x + y (1

- Por el primer teor. de la bisectriz


AD AB
Interior
DC BC

21 - x 20 21
• •

x “ 10

EC _ BC y ^10
- Exterior -> y = 21
EA AB y + 21 ~ 2 0

- Reemplazando los valores de x e y, en (1 ) DE = 7 + 21 DE = 28

Rpta: ( C ).

5) En un A ABC , AB = 16 , se traza la mediana B M . Hallar BM , s i : MBC = Á + C

A) 8 / 2 B) 12 C) 8 D) 8 / 3 E) 4 / 3

Solución
BM = ? sE

- Por dato : MBC = A + C

Si Á = a , C = ó

Entonces: MBC = a + ó

- En el A ABC, por ser ángulo exterior:

EBC = Á + C

EBC = a + 0

352
Se observa que BC es bisectriz exterior en el A ABM. Entonces, por el Primer teorema de
la bisectriz, aplicado en este triángulo:

BM CM BM a DkJI 0
= -- —> --------- = — BM = 8
AB CA 16 2a

Rpta: ( C ).

En un AABC, BD es bisectriz interior. En los triángulos ADB y BDC, DE y DF son también,


respectivamente, bisectrices interiores.
S i : AE = 5 ; EB = 15 y B F = 1 2 .

Hallar FC.

A) 3 B) 4 C) 5 D) 2 E) 3,6

Solución

Sea el gráfico mostrado, donde


FC = x es la incógnita.
Con el 1er. Teorema de la bisectriz :

AD AB
A ABC
DC BC
AD 20
( 1 )-
DC 12+ x

AD _5_
A ADB Dividiendo miembro a miembro
DB 15
AD
DC x DB _ 15 AD 4
A BDC DC DC x
DB 12
DB 12

4 20
Reemplazando ésto último en (1 ) :
x 12 + x

De donde x = 3

Rpta: ( A ).

Si en la figura adjunta, EF es paralelo a


AC ; entonces en el A ABC , BD es:

A) Bisectriz B) Altura C) Mediana


D) Cualquier ceviana E) N.A.

353
Solución
BE BF
Como EF 11 AC, entonces por el Teorema de Thales: (1).
EA ~ FC

Por otro lado, según el 1er. teorema de la bisectriz :

BE BD BF BD
A ADB y A BDC
EA “ DA FC " DC

BD BD
Reemplazando en (1 ) :
DA " DC

DA = DC BD es mediana en el A ABC

Rpta: ( C ).

8 ). En la figura, O e I, son centros

S i: R = 5 r.

Bl
H a lla r:
ID

A) 1,5 B) 1,2 C) 1,4


D) 1,6 E) N.A

Solución

Bl AB 4- BC
- I , es incentro del A ABC : (1)
ID AC

( Teorema del Incentro )

Siendo AC = 2R.

- Por el Teorema de P oncelet: AB + BC = AC + 2r.

Bl AC + 2 r
- Colocando esto último, en ( 1 ) :
ID AC

Bl 2R + 2 r Bl R + r
ID 2R ID R

y, como según dato : R = 5r

Bl 5r + r B 6 „ „
Entonces : ID -> — = — = 1,2
5r ID 5

Rpta: (B)

354
9)
En la figura , AB, BC y AC son diámetros
AE = 4 , EF = 10 y CG = 12;

Hallar GH.

A) 5 B) 6 C) 4,8
D) 3,8 E) 5,2

Solucton
- Incógnita: GH = x

- Como AB, BC y AC son diámetros

AÉB = AFC = AHC = BGC = 902


BE II CF y BG II AH

- Por el Teorema de Thales:


AB x AB
BC ’ 12 BC 10
x 4_
x = 4,8
12 " 10
Rpta: ( C ).
• ^
10)
En la figura : "I
r

i.

ÉP II BC, QE II ÁB
CF = 16 y AC = 9.
i
n:
-V
Hallar EF.

A/* - V E X c n - - A) 15 B) 25 C ) 30
h—'— 9 — ------[•---------- 16
jy fío E) Ning. anterior.
f *
/, O
Solución
*
- Incógnita: EF = x + 16

Por el Teorema de Thales : T ^ V -0

tu ku xx ^ k
PQu " ^
En el A PEF : ( 1 )-
CF QF 16 QF

AE = PQ 9 - x PQ
En el A APF: (2)
EF QF x + 16 QF

355
x 9 - x
De ( 1 ) y ( 2 ) : de donde : x = 4
?6 x + 16 :

Luego : EF = 4 + 16 = 20

Rpta: ( D ).

11) En la figura:

AE = 5 , EF = 8 y CD = 6

Hallar DF.

A) 10 B) 9 C) 8,5
D 9,6 E) 10,2
D

Solución
- DF = x = ?

Trazamos EQ AC el trapecio AEQC B


es isósceles: QC = AE -> QC = 5

- En el A EQD, con el Teor. de Thales :

DF DC x _ 6
• m
x = 9,6
FE “ CQ 8 ~ 5

Rpta: (D)

12) En la figura:
AC
G es Baricentro del A ABC y CE =

Hallar QC, si BQ = 12.

A) 6 B) 3 C )4
D) 2 E) 1,5

Solución

- Incógnita: QC = x

Trazamos la mediana B M . Por propie


dad del Baricentro : BG = 2GM
GM = n , BG = 2n.
AC
- Como CE = .\ CE = AM = MC

356
Trazando ML 11 QE : LMC = QÉC ( alternos ) y, como QCE = LCM ; entonces
A MCL = A ECQ ( Postulado ALA ). CL = CQ -+ CL = x.

BQ BG 12 2n
En el A MBL, por el Teor. de Thales : ~ qM 2x ~ n

De donde : x = 3 —> QC = 3

Rpta: ( B ).

I, es incentro del A ABC y AM = MC.


Bl 3
S¡ 1 ID = 2 ’ BP = 6 Y QM = 4 ■

Hallar PQ.

A) 1 B) 2 C)3
D) 2,5 E) N.A.

Solución

I , incentro del A ABC Al, Bl, Cl son bisectrices.

En el A ABC, por el Teorema del Incentro:


AB + BC Bl AB + BC 3 / i \
AC " ID ^ AC 2

Con el Primer teorema de la bisectriz :

. AB BP AB 6 2AB 6 (u \
A ABM -> ------ -» -rr=r= --------- => —— ........ . ....... \ 11 )•
AM PM AC x +4 AC x+ 4
2

BC BQ BC 6 + x 2BC 6 + x
(III)
AMBC —>
MC QM AC AC ” 4

Sumando ( I I ) y ( I I I ) , para poder usar ( I ) :

' AB + BC 6 + x
+
\ AC x + 4

6 6 + x
Con lo de ( I ) : 2 + --------
/ x + 4 x + 4

» • Resolviendo : x = 2

Rpta: ( B ).
14) En un A ABC, CF es bisectriz interior. En el A AFC, FR es bisectriz. ( R en AC ). Luego
se traza BR, cortando a CF en el punto Q.

S i: BC = 12 , CR = 10 y BF = 5,5 ; hallar FQ.

A) 2 B) 2,5 C) 3 D) 3,5 E) 2,8

Solución
FQ = ? B
Vamos a usar el 1 er. teorema
de la bisectriz :

* A BFQ, FR es bisectriz exterior.


FQ RQ
Luego:
FB RB
FQ = RQ
5,5 " RB ( i)

BQ BC BQ 12
* Pero, en el A RBC, tenemos :
QR " RC QR 10

Esto indica q u e : 59. = 5 ó : BQ = 6 k y QR = 5k


QR 5
(k una con sta n te ).

Entonces : RB = BQ + QR = 6 k + 5k RB = 11 k.

FQ 5K
Reemplazando en (1 ) FQ = 2,5
5,5 11K

Rpta: ( B ).

15) En un triángulo A B C , BD y BE, son bisectrices interior y exterior respectivamente.


S i : AD = 5 y DC = 3 , hallar CE.

A) 12 B) 8 C) 16 D) 9 E) 10

Solución
- Incógnita: CE = x

- Sabemos, por propiedad, vista en teoría,


que A, D, C y E conforman una cuater­
na armónica:
AD _ EA 5 x + 8
—> — = --------
DC CE 3 x

Rpta: ( A ).

358
ABCD, es un cuadrilátero inscrito en una circunferencia ; AB = AD y AC es diámetro.
P, es un punto del arco BC. PA y PD cortan a BC en los puntos E y F, respectivamente

S i: BE = 3 y EF = 2 , hallar FC.

A) 10 B) 5 C) 7,5 D) 6 E) 9

Solución
Q
FC = x = ?

Como AB = AD AB = A D .

Trazando BP y PC:

BPA = — y APD = —
2 2
BPA = APD = a

Además, por dato AC es diámetro


Entonces : APC = 90°.

Luego:

FPC = 90° - EPF = 90° - oc


y QPC = 90° - BPE = 90° - a

Es decir: FPC = QPC.

Por lo tanto, PE y PC son bisectrices interior y exterior, del A BPF. Por propiedad , B, E,
BE CB • 3 x + 5
_____
F y C conforman una cuaterna armónica : • • x = 10
EF FC 2 x
Rpta: ( A ).

En la fig u ra :

AB es diámetro. DF 1 AB. CD = 10 ,
CF = 22 y DF = 18.

Hallar ED.

A) 15
B
B) 12
C) 9
D) 18
E) 6

359
Solución
- Incógnita: ED = x
M\ y
- Como AB es diámetro perpendicular
C -S ^í*
a ía cuerda DF; D
,Z / \ 10
AF = 2a = A C D ( 1 ). / / a \\

AF i /
k -—
22\\\ \ r
ACF = — ( z inscrito ). B

ACF = a MCE = a 18
X 2a \\
ACD
También : ECD = ( Z exinscrito).
2
2a
Con ( 1 ) : ECD = ECD = a

Entonces CE es bisectriz exterior, en el ACDF. Luego, por el primer teorema de la bisectriz-


10 X
—> x = 15
22 x + 18

Rpta: ( A ).

18) En la figura:

BD es diámetro perpendicular a AC ; AE = 12 ,
EF =10 y FG = 1 1 .

Hallar CG.

A) 13 B) 13,2 C) 13,6
D) 13,8 E) N.A.

Solución

* Incógnita : CG = x B

* Trazamos EC y BA

* Por propiedad de la mediatriz :


EC = EA = 12 y BA = BC
Luego: BÁE = ECB = a

Entonces: BG = 2BÁG = 2a

y BCG = — = a
2 D
360
• • CF es bisectriz en el A ECG.
CG GF x
* Primer teorema de la bisectriz :
CE FE 12 10

x = 13,2

Rpta: ( B ).

19) En un triángulo ABC; I : Incentro ; G : baricentro. S i : IG 11 A C . Demostrar que

Solución

- Trazando el gráfico conforme se señala B


Propiedad del Baricentro: BG = 2 GM

o sea : GB = 2n y GM = n

En el A ABC, por el Teor. del Incentro:

Bl AB + BC
ID AC (1)

En el A MBD por el Teor. de Thales:

Bl BG Bl H -

= 2 (2)
ID GM ID

Sustituyendo ( 2 ) y (1 ) :

AB + BC
AC

• • Efectivamente. AB + BC = 2 AC l.q.q.d

20) En un triángulo ABC, las cevianas interiores AF y BG se cortan en el punto R. Si: BC = 18,
QC = 3 AQ y BR = 2 RQ , hallar BF.

A) 7,2 B) 8,4 C) 9,6 E) 8

Solución

- Incógnita : BF = x

- Por dato : QC = 3 AQ ; Si AQ = a

QC = 3a.

Además BR = 2 RQ Si RQ = n,
entonces BR = 2n.

361
- Para hacer uso del Teorema de Thales, trazamos QP 11 AF

ABQP = FP = — ... FP = —
BF 2n 2 2

FP _ AQ 2 _ a
A AFC ->
FC AC 18 - x 4a

Efectuando : x = 6

Rpta: ( D ).

21) En un A ABC, la ceviana AR corta a la bisectriz interior BD en ei punto “M”. Si BR


RC = 12 y BM = M D , hallar AB.

A) 2,8 B) 2,4 C) 2,6 D) 2,5 E) N.A.

Solución

En el A ABC, por el Primer teorema de la


b ise ctriz:
AB AD
BC " DC
x AD
( 1)
14 DC

Tracemos DE II MR. Luego, MR es base


media del A DBE : ER = RB -> ER = 2

Además : EC = 10.

En el A ARC, por el Teorema de Thales:


AD RE AD 2
DC EC DC ” 10

Reemplazando esto último, en (1 ) :


x 2
-> x = 2,8
14 10
Rpta : AB = 2,8 ... ( A ).

22) Del gráfico, el valor de x, es

A) 30°
B) 45°
C) 22°30‘
D) 53°
E) 26°30‘

362
Solución
,.R
Por el Primer teorema de la bisectriz, en el
A ABM : 2a
AB = AD AB 2n
AB = 2 ( B M )
BM DM BM n

Luego, si BM = a AB = 2a

- Tracemos CR || BM. Entonces:


BCR = MBC (alternos internos)

BCR = x

En el A ARC, BM es base media, ya que AM = MC. • • RC = 2 ( BM )


RC = 2a. Tam bién: BR = AB BR = 2a.

Luego, el A BRC es isósceles:


CBR = BCR -> CBR = x

- Finalmente, en B : 4x = 180 x = 22°30'

Rpta: ( C ).

363
SEMEJANZA DE TRIANGULOS
En general, dos figuras semejantes, tienen igual forma y tamaños diferentes. Por ejemplo,
dos cuadrados, uno de 3cm. de longitud por cada lado y el otro de 10cm. de longitud por lado,
son semejantes:
J u

T 10
3

1 *1 r
l--------------- 1 0 -----------------1

( El símbolo - , se lee “ es semejante a...” )

Dos triángulos semejantes tienen sus ángulos congruentes, dos a dos, y sus lados
homólogos, proporcionales. Se llaman lados homólogos, uno en cada triángulo, aquellos
opuestos a ángulos congruentes.
Así:
B

A ABC - A D E F , y se cumplirá:
DE EF DF
=k (C onstante)
AB BC AC
365
Vu e y f t b / I

“k”, se llama “ razón de semejanza


1
Por ejemplo, si k = —, significa que los lados del A DEF tienen la mitad de longitud de los

lados del A ABC.

Además, alturas homólogas, medianas homologas, etc, son aquellas referidas a los lados
homólogos y la relación de sus longitudes puede igualarse a la relación de longitudes de los
lados.

Casos de Semejanza

Dos triángulos serán semejantes si cumplen con cualquiera de los siguientes casos:

1er. Caso.- Si tienen dos pares de ángulos congruentes.

B Si:
A A A A

A = D y C = F

Entonces:
A ABC ~ A DEF

* En consecuencia : B = É ( Los terceros ángulos resultan congruentes entre s í ).

2do. Caso.- Si tienen un par de ángulos congruentes y los lados que los forman, respecti
vamente proporcionales.
B-
A A
AB BC
S i: B = E y
DE “ EF

Entonces:

A ABC - A DEF

3er. Caso.- Los tres pares de lados, respectivamente proporcionales

AB BC AC
S i:
DE ~ EF “ DF ’

Entonces : A ABC ~ A DEF

366
Notas

En ia resolución de los problemas, el caso que más se usa es el primero.

Toda paralela a uno de los lados de un triángulo, determina con los otros o sus prolongacio
nes, un triángulo semejante al original.

A C
C A

S i; EF 11 AC -> A EBF - A ABC , en ambos casos.

EJEMPLOS

En un A ABC, UE” es un punto de AB y " P un punto de B C , 4ales aue :


EFB = Á , AC = 36 , EB = 24 y BC = 40.

Hallar EF.

A) 21 ,6 B) 20 C) 18 D) 9,6

Solución

P ortenercom únelB yeldato EFB = A = a ,


los triángulos EBF y CBA, son semejantes.

Luego:

opuestos opuestos
al B a a

EF EB AEBF
AC BC ACBA A C
36

Rpta: ( A ).

367
2) En un cuadrilátero ABCD, el ángulo externo D mide la mitad del ángulo interior B y la
diagonal BD biseca el ángulo ABC.

Hallar BD, si AB = 16 y BC = 9.

A) 11 B) 12 C) 14 D) 16 E) N.A.

Solución

ABC
D a to : CDE =

/. Si CDE = a

Entonces: ABC = 2 a

ABD = CBD = a

Como, en el A ABD :
BDE = Á + ABD ............( ¿ externo )

BDC + CDE = Á + ABD

Es decir :
BDC + a = Á + a —> BDC = Á

Luego : A ABD - A DBC.

En consecuencia BD AB
BDA = C y :
BC BD
BD 16
BD = 12
9 ~ BD

Rpta: ( B ).

3) En un A ABC, A = 2 C , se traza la bisectriz interior AE


Hallar A B , si BE = 4 y EC = 5.

A) 7,5 B) 6 C) 4,5 D) 7 E) N.A.

Solución
Del dato:
A = 2C
A

S i: C = a
Á = 2a
Luego: BÁE = EÁC = a , y en el

AAEC : AÉB = EÁC + C -> AÉB = 2 a

368
A ABE es semejante al A CBA :
AB BE AB
AB = 6
BC AB AB

Rpta: ( B ).

En un A ABC , B = 90°, de catetos AB = 12 y BC = 8 , se inscribe un cuadrado con uno


de sus vértices en B y el opuesto sobre la hipotenusa. Hallar la longitud del lado de dicho
cuadrado.

A) 4 B) 4,8 C) 4,5 D) 6 E) 4,2

Solución A
Sea BFEG, es cuadrado y x la longitud de su
lado:
Esfácildeducirquelostriángulos AFE y ABC
son semejantes.
Luego:
EF AF x _ 12 - x
CB AB 8 ” 12

De donde : x = 4,8
h— 8 ----- 1
Rpta: ( B ).

m un A ABC, AC = 27, por el Baricentro G, se traza EF paralelo a AC. ( E sobre AB y F


en BC ). Hallar EF.

A) 18 B) 13,5 C) 16 D) 24 E) N.A.

Solución
B
EF = ??
Es evidente, que : A EBF - A ABC.
Sea M el punto medio de AC. Luego, como
G es Baricentro, estará contenido en B M .
Además, por ser EF y AC lados homólogos
paralelos y tener común el vértice B, BG
será mediana homologa de BM. Entonces,
puede escribirse:
EF BG EF 2 ( Con la propiedad del Baricentro )
AC BM ^ 27 ” 3
Relación Relación
de lados. de medianas

De donde: EF = 18

Rpta: ( A ).

369
Teorema de Menelao

Sea el A ABC, de la figura. Una recta intercepta


a los lados AB, BC y a la prolongación de AC,
en los puntos E, F y G, respectivamente.
Se cumple, que:

AE BF CG
= 1
EB ' FC ' GA

* Sea CH 11 AE. ( La demostración


del Teorema se hará con semejan
za de triángulos).

Luego:

* A AEG - ACHG

AE GA
CH “ CG (1 )
* A EBF ~ AHCF:

CH FC
( 2)
EB ~ BF

Multiplicando miembro a miembro ( 1 ) y ( 2 ) :


AE CH GA FC
CH ‘ EB CG ' BF
AE GA FC
Cancelando CH :
EB CG BF
AE BF CG
De d o n d e : = 1 ... l.q.q.d
EB ' FC * GA

Teorema de C eva.-Sean A E , B F y CR, tres cevianas B


cualesquiera, del A ABC, concurrentes en el punto Q.
Se cumple, que:

AR BE CF ,
x — x = 1

D em ostración
Con el Teorema de Menelao:
AR BQ FC
A ABF ( transversal RQC ) -» = 1
RB ' QF * CA
370
CE BQ FA
A FBC ( transversal EQA ) = 1
EB * QF * AC

Dividiendo miembro a miembro estas dos expresiones

AR FC
RB Q CA = 1
CE '§<3 FA
EB ' Q F \ AC

Simplificando y efectuando con lo que queda:


AR EB FC
RB ‘ CE * FA

AR BE CF
o, m e jo r : 1 I. q *cj i d i
RB ‘ EC ' FA

PROBLEMAS RESUELTOS

1)
En la figura, T es punto de tangencia.

Hallar ET , s i : 1 + - = 0,2.
R r

A) 2 B) 10 C) 0,2
D) 5 E) 4

Solución
- Dato :
J_ 2 _ _2 _ r +R _ 1 . Rr
= 5
R + r “ 10 Rr 5 R +r

- Uniendo los centros y trazando MP

A O E T - AOPM

ET = OT ET R Rr
-» ET =
PM OM R+ r R + r

Con el dato: ET = 5

Rpta: ( D ).

2) En un triángulo ABC, la prolongación de la bisectriz interior BD, corta a la circunferencia


circunscrita, en el punto E. Hallar la longitud de AE, si BD = 16 y DE = 9.

A) 12,5 B) 7 C) 10 D) 12 E) 15

371
Solución
AE = ?

Del gráfico:
EC
EAC = — = EBC

Por tener a y É: AAED es semejante


al A BEA. Entonces: ADE = BÁE.
AE DE
Luego :
BE ” AE
AE
-> AE = 25 x 9
25 AE

De donde: AE = 15

Rpta: ( E ).

En un trapecio ABCD , BC 11 A D ; BC = 4, AD = 6 y la altura mide 8. Hallar la distancia del


punto de corte de las diagonales, a la mediana del trapecio.

3
A) B) 1 D)
C> 4 E> 3

Solución

Consideremos el gráfico:

Incógnita: PQ

Se sabe : EF = — — — = - — - = 1

AEPF - AAPD:

PQ EF
PH AD

relación relación
de alturas de
homologas lados.

PQ + 4
= 16 - p q ,±
5

Rpta: (A)
4) En un triángulo acutángulo ABC, se trazan las alturas AQ y CH. Hallar H Q ,s i:A C - 2 0 ,
BC = 25 y BH = 18.

A) 10 B) 14,4 C) 15 D) 16 E) 12

Solución
B
HQ = ?

* Del gráfico: □ AHQC es inscriptible, ya que


AHC = AQC. Entonces: BHQ = ACB y

HQB = BÁC.

Luego : A QBH - A ABC

HQ HB HQ 18
AC " CB 20 ” 25

De donde : HQ = 14,4

Rpta: ( B ).

5) En un A ABC, por los vértices A y C pasa una circunferencia que corta a AB en M y BC


en N. La tangente trazada por C, es paralela a A B .
Si AC = 12 y B C = 16, hallar NC.

A) 8 B) 7 C) 9 D) 10 E) 12

Solución B
- Incógnita: NC = x

- Del gráfico, al trazar AN:

NC
NAC = — = NCE

T T
Z inscrito Z semi
inscrito
NAC = NCE = a

- También, como CE II A B :

B = NCE B= a

Luego : A ANC - A BAC, por tener a y C ANC = BAC.


NC AC x 12
AC BC ^ 12 * 16
De d o n d e : x = 9

Rpta: ( C ).
373
6) En un triángulo ABC, isósceles, AB = BC ; la mediatriz de BC, corta a AC en el punto R
Luego, se traza RF 11 BC (F en A B ). S i : RF = 1 y RC = ^ fé , hallar AB.

A) 3 B) 2 C) 4 D) 2 / i E) 3 / 6

Solución

Datos : RF 11 BC ; AB = BC
RF = 1 ; R C = / 6

- Incógnita: AB = x

- Por propiedad de la mediatriz :


RB = RC —> RB = J e

RBC = C = a

- FRB = RBC = a
( alternos internos ).
A A

- ARF = C = a ( Z correspondientes ).

- A FBR ~ A RBA : BR = FB ) VZ = x ~ 1 x (x- 1) = 6


AB BR x /6

De donde : x = 3

Rpta: ( A ).

En la figura : AB A BC ; r y R, son radios


de las semicircunferencias tangentes a AC y
a la altura B H . ( R > r ). Hallar BH.

A) 4 / R 7 B) 4 / rV C )2 ^ R2 + r2

D) R + r E) N.A.

Solución
B
* BH = ?

- Con los trazos indicados:


BE = BH - R y BF = BH - r

- ABFO - AM EB:
BF OF
EM ' BE

374
BH - r r
. De donde, a! efectuar: BH = R + r
R BH - R

Rpta: ( D ).

En un A ABC, AB = 6 , BC = 8 y AC = 7. Por “B", se traza tangente a !a circunferencia


circunscrita, cortando en P a la prolongación de CA. Hallar PA.

A) 7 B) 8 C) 14 D) 9 E) 12

Solución
Incógnita: PA = x

Del gráfico:

AB
= PBA

Z inscrito. Z semi-
inscrito

Por tener P y a : A PBA - A PCB PÁB = PBC

Entonces :

A PBA 6 PB
A PCB PB 8 x +7

4x
De las dos primeras: PB = ( i )

6 PB
En las dos últimas : —= 6( x + 7 ) = 8( PB )
8 x + 7

Con lo de (1 ) : 6 ( x+ 7 ) = 8 ( 4x_ ).
3
De donde: x = 9

Rpta: ( D ).

Si O es centro , AB = a y BH = b.
Calcular BT ( T es punto de tangencia ).

ab (a + b) ,---------------
^ (a + b) B) J C) ^ b ( a + b )

D) ^ a ( a + b ) E) ^ a b
\

375
Solución
- x =

- Se observa :

TB
TAB = — = HTB
2

Z Z semi-
inscrito inscrito
o B
TAB = HTB = (¡)
_ *

Entonces: TBA = TBH = a ; por ser complementos de <¡>, en los triángulos ATB y THB,
respectivamente.

x a <r~ A ATB c
- Así: A ATB - A THB : - = - = /a b
b x ATHB
T T
opuesto a opuesto
0 a 90°
Rpta: (E)

10) En un A ABC , AB = 9 , BC = 8 y AC = 3. Sobre AB y BC, se toman los puntos E y F,


respectivamente ; de modo que EF sea tangente a la circunferencia inscrita al A ABC y
además EF 11 AC. Hallar la longitud EF.

A) 1 B) 2 C) 2,5 D) 2,1 E) 2,8

Solución

Sea el gráfico adjunto. Como EF 11 AC, B


entonces : A EBF - A ABC.

- Se puede escribir:

EF _ Semiperímetro del AEBF


AC Semiperímetro del A ABC ( 1 )•

- El semiperímetro del A ABC, es ;


p = 9 + 8_1 3 = io
( 2)

y sabemos además, por propiedad, que:


BT = p - A C
• I BT = 10 - 3 = 7

376
- Pero, para el A EBF, BT mide igual que el semiperímetro

Semiperímetro del A EBF = 7 ........... ( 3 ).

EF
- Reemplazando ( 2 ) y ( 3 ) en ( 1 ) : EF = 2,1
10

Rpta: ( D ).

11) Un trapecio ABCD, está inscrito en una circunferencia. Por C, se traza tangente a la
circunferencia, cortando a la prolongación de AD en el punto F.
Si BC = 10 y AC = 16, hallar AF.

A) 24 B) 23 C) 26,5 D) 27,5 E) 25,6

Solución
AF = x = ?

- Del gráfico:
BCR = F = a
( Z correspondientes ).

También:
BC
BAC = — = BCR

T t
Z Z semi-
inscrito inscrito

BAC = BCR = a

Además : BCA = p = CÁF ( alternos internos ).

10 = 16
- Entonces : A ABC - A FCA : x = 25,6
16 ” x

Rpta: ( E ).
4

B
12) Dado el A ABC, de altura BH = b y lado
AC = b ; demostrar que la longitud x, del cua
drado inscrito PQRS, es:

377
Solución
Se observa, que : A PBQ - A ABC

PQ BF
AC BH

Relación Relación
de de
lados alturas homologas

entonces : — = í - l.q.q.d
b h

13) B
En la figura. O es centro de la circunferencia.
AM = MB; MF 1 OA. Hallar la longitud de AB

A) 2 / a b B) ^ 2a( a + b ) C) ^ a ( a + b )

D) ^ b ( a + b ) E) ^ 2b( a + b )

Solución
- Incógnita: AB
B
Trazamos OM y OB . OM será
perpendicular a AB , ya que:
AM = MB. Además:

AOM = MOB = a AB = 2 a
■N

AB
c = C = a

- En el AAMO : AMH = AOM = a

- Por lo tanto, los triángulos ACB y AMF,


son semejantes; Entonces : AFM = B

AB AC
A s í: AB x AM = AF x AC AB x — = a( a + b )
AF ~ AM

Es d e cir: AB = ^ 2 a ( a + b )

Rpta: ( B ).

378
14) En un A ABC ,A B = c ,B C = a , B = 120°. La longitud de la bisectriz interior BD, es

(__ ,------------ (a + c ) 2ac ac


A) / a c B) ^ a2 + c2 C) - D) a + c ^a + c

Solución
■«R
- Datos: a ; c

- Incógnita: BD = x

- Al trazar CR 11 BD:
R = ABD (correspondientes)

R = 60

Además: CBR = 180e - ABC CBR = 60Q

- A BRC equilátero BR = RC = BC = a

BD _ AB ac
- A ABD - A ARC : • • x =
RC “ AR

Rpta: ( E ).

15) En la figura, B es punto de tangencia y rr II AC.


Hallar AB; si BE = 4 y EC = 5.
B
A) 5
B) 6
C) 8
D) 7
E) 9

Solución
Incógnita: AB = x

- Por ser alternos internos entre las paralelas:


C = PBC = ct
- También, por la misma razón, al trazar BF :

- De otro lado :

BEF
( Z inscrito) -> Á = = FBP ( Z semi-inscrito ).

379
« •

- En consecuencia : AFB = Á E! A ABF es isósceles : BF = AB BF = x

- Además, al trazar EF:

BE
( Z in scrito ) BFE = — = EBP <- ( Z semi-inscrito )
2
BFE = EBP = a

Entonces : A BEF - A BFC ; donde : BÉF = BFC

BF BE
Luego : x = 6
BC ~ BF

Rpta : AB = 6 ... ( B ).

16) Del gráfico, hallar “ x “

A) 2 / R r

B) J f í r

C)

D)

E) N.A.

Se unen los centros. Los centros con los puntos


de tangencia y las perpendiculares de los cen­
tros a estas rectas. Se calculan sus valores r
literales:

Del gráfico:
PA PQ
APAQ ~ AOBP
OB ~ OP
x - r x + r
x = J fír
R- x R+ x

Rpta: ( B ).

380
17) En un A ABC, de incentro “I”, por este punto se traza EF 11 A C , estando Esobre AB y F
en BC-
S i : AB = c , BC = a y AC = b , hallar EF.

ac a +b b( a + c ) 2ac
A) a + b + c B) D) a + b + c E) N.A
a +b + c

Solución
- EF = ?

- Como “ I ” es incentro, BD bisectriz

- A EBF ~ A ABC:
EF Bl
AC BD
T T
Relación Relación de
de lados bisectrices homologas

EF Bl
BD (1 )

Bl
- Por el Teor. del incentro :
ID

Bl
Con propiedad de proporciones, a fin de obtener
BD
Bl a +c Bl a + c
Bl + ID a + b + c BD a + b +c ( 2 )

EF a + c
- Reemplazando ( 2 ) en ( 1 ) : * m+ 'b V c

Rpta: (C)

18)

Dado el gráfico adjunto, hallar la


longitud del inradio del A ABC , si
AC = b y se tienen “n” circunferen­
cias de igual radio r.

381
Solución
En el gráfico, si O y M, son los centros de
la primera y última circunferencia, enton­
ces : OM Ii AC.

I, es incentro del A ABC, por lo tanto: IH


es radio de la circunferencia inscrita al
A ABC. ( |H _L AC )•

Se observa, que : A OIM ~ A AIC


IT OM
Entonces :
IH AC

IH - r 2 r ( n - 1)
Es d e a , : — - = — —

br
Rpta: IH =
b - 2 r ( n - 1)

19) En un triángulo ABC, la mediana AM corta a la ceviana BR en el punto F . Si AR = 2 RC


y AM = 10, hallar FM.

A) 3 B) 2 C) 4

Solución
- Dato: AM = 10.

Incógnita: FM.

- A fin de aprovechar la relación dada


AR = 2RC, trazamos RH I AM.

AR H C - A A M C :
RH a 2 U 1 _ RF = i ° (I).
AM 3a 10 3 3

HC a HC 1 uo
Además : — —^ --------- —— .. Si HC —n MH = 2n
MH 2a MH 2
y , entonces : BM = MC = 3n.
FM _ BM
- Finalmente, como : A FBM - A RBH
RH “ BH
FM 3n
Con ( I ) : 5n FM = 2

Rpta: ( B ).

382
En el trapezoide ABCD , M y N son puntos medios
B
de las diagonales. AE = a ,E B = b ,C F = c y
DF = d. Entonces:

A) a + c = b + d B) a . c = b . d
C )a + d = b + c D) a2 + c2 = b2 + d 2
E) a . d = b . c

Solución

Trazamos las perpendiculares AL, BR, CH


y DQ, a la recta MN.

Entonces:
AL a
A ALE - ABR E -> (1 )
BR ~ b
CH c
A CHF - A DQF —> ( 2 ).
DQ “ d

Adem ás: AALM = A CHM AL = CH AL = CH


(3 )
y A BRN = A DQN -4 BR = DQ BR DQ

Reemplazando lo de (1 ) y ( 2 ), en ( 3 ) :

Rpta: (E ).

En un trapecio ABCD , BC I i A D , las diagonales se cortan en el punto O y por él se traza


EF 11 BC ( E, sobre AB y F en CD ). Demostrar, que:

2BC . AD
(b ) EF =
BC + AD

Solución
Considerando el gráfico adjunto

EO EB
A EBO - A ABD : ( 1)
AD " AB
EO _ AE D
A EAO - A BAC : (2)
BC " AB

383
O ^ / < V / o . EO EO EB + AE EO EO AB
Sumando (1 ) y ( 2 ) : + -=-r-= -------------- -> — +
AD BC AB AD BC AB

“ +— = 1 EO- ADBC (3 )
AD BC AD + BC

En forma análoga, al hacer la relación de lados entre los triángulos semejantes OFD y BCD;
OCF y ACD, se demuestra que :
AD . BC
OF = (4 )
AD + BC

Luego, de ( 3 ) y ( 4 ) : EO = O F I.q.q.d.

O D p A H
y al sum arlas: EO + O F = E F = Lq„ d
BC + AD M

En la figura adjunta : AE = a , BF = b
y QH = x.

Demostrar, que: B

Solución
Por semejanza:

x EH
AEHQ - A E FB ( 1)
b EF

x HF
AQ H F - AAEF (2)
a “ EF

Sumando ahora, las expresiones ( 1 ) y ( 2 )

x x EH HF
a + b ” EF + EF

x x EH + HF x x EF
— + — ——— —> — -f — = —
a b EF a b EF

De d o n d e : 1 + 1 = 1 Lq.q.d
a b x
23) Para un trapecio isósceles de bases a y b, circunscrito a una circunferencia, hallar la longitud
del segmento que une a los puntos de tangencia de la circunferencia inscrita, con los lados
no paralelos.

2ab
A) ( a + b ) C) 2 / a b D) 4 ,/a b E )N . A

Solución a
H
Sea el trapecio ABCD.

Incógnita : EF = x

Como AQ = AE = QD = DF y
EB = BH = HC = CF

BE CF Q
EF II A D ; BC b
EA FD

- Trazamos AP 11 CD : PB = b - a y EN = b - x
a
A d em ás: BE = BH BE = -
2
b
AE = AQ -> AE = t
2

b
EN = AE b - x 2
- AAEN - AABP PB “ AB b - a a + b

Efectuando:

( Propiedad)

Rpta: ( A ).

24) Sobre el menor arco AB de una circunferencia se toma el p u n to " P ", distante 4 y 9 unida­
des, de las tangentes trazadas por " A " y " B " , respectivamente. Hallar la distancia de
" P " a ÁB.

A) 2 B) 4 C) 6
D) 8 E) 10
D
Solución
- Se traza la figura como dice
el enunciado: B

Dato: PC = 4 , PD = 9

385
Incógnita: PE
A

Según el gráfico: Se tiene los ángulos PA B inscrito y PBD semi-inscrito que tienen por
medida la mitad del arco PB, así:
» PB *
PAB = — = PBD
2

Análogamente:

PA
PBA = — = PAC

PC PA
APCA - APEB (1 )
PE " PB
PE PA
APEA APDB (2 )
PD PB

Igualando los primeros miembros, de ( 2 ) y ( 1 ) :

PE PC
—» ( Teorema de Pappus )
PD “ PE

Con los datos ; P E =4x9 -» PE = 6

Rpta: ( C ).

Dado el cuadrilátero inscrito ABCD ; “ P ” , es un punto cualquiera del arco CD.


Si PQ = 12 , PS = 3 , PR = 9 . Hallar PT.

A) 2 B) 3 C) 4 D) 5 E) 6

Solución

Se traza PC y PA
PÁT = PCD y PCR = PÁB
PR PC
APRC - APQA ....(1 ).
PQ PA
PC PS
A PSC ~ A PTA ...(2 ).
PA PT
PR PS
en (1 ) :
PQ PT

Propiedad,para todo
PT . PR = PQ . PS
cuadrilátero inscrito.
Con los datos:
PT x 9 = 12 x 3 PT = 4

Rpta: ( C ).
26)
En la figura, Bes punto de tangencia y BD 11 AF
D
Hallar la longitud de A F , si: AB = 18 y BC = 10

A) 12 B) 14 C) 14,8
D) 15 E) 14,4

- Incógnita: AF = x D

- Como BD 11 AF, entonces : D = DAF


A,

D = a ; luego, el AABD es isósceles


BD = AB BD = 18 y CD = 8.

- De otro lado:
BE ~
CBE = — = BAE

CBE = a. Enseguida : AÉB = EBD + D = a + a -> AÉB = 2 a

- También, en el cuadrilátero inscrito ABEF : BÉC = BÁF BÉC = 2 a

- Se observa : A BED, isósceles : ED = BE = a

x b 18
- A s í: A AEF ~ A DEC E ntonces: —
8 a O 10
b 18
A ABE ~ ABCE x = 14,4
a 10

Rpta: ( E ).

27)
En la figura , PA y PB son tangentes
AC = 8 , BC = 6 y AD = 10.

Hallar BD.

A) 7 B) 9 C) 7,5
D) 8,5 E) 7,2

Solución
- Se observa:

» AC * BC
ADC = — = CAP CDB = — = CBP

387
Entonces:
BD PB
ADBP - ABCP: ...(1 ).
BC “ PC

PA AD
APAD - APCA: — = 2 ).

Reemplazando (2),en (1 ); ya que PB = P A :


BD AD
BC ~ AC
D

De donde :

Con los datos : BD x 8 = 10 x 6

BD = 7,5

Rpta: (C)

28) E nlafigura: OA, OM, OB y OP constituyen los rayos de un haz armónico. Por “B” se traza
una paralela a OA. Se pide calcular EB.
Dato : LB = 4

A) BE = 4

4 /3
B) BE = —!
3
C) BE = 4 J 3

D) BE = 8

E) ISI.A.

Solución

- Incógnita: EB.

EBL OA LB = 4

Se tiene:

EB PB
APEB - A POA (1 )
OA PA

OA MA
A OAM ~ A LBM (2 )
LB BM

A fin de obtener EB en términos de LB, se multiplican las expresiones (1 ) y ( 2 ) , miembro


a miembro, cancelándose OA:

388
EB P B .M A * •

LB = p a BM : Per0, en el 9ráfic0 P . B . M . A , constituyen una cuaterna armónica.

PB PA EB
Luego : = 1 -» EB = LB
BM “ MÁ LB

Rpta: (A ).

29)
En la figura, L es ortocentro del A ABC.
Entonces.la relación entre los radios de las circun
ferencias indicadas, es:

A )a + b + c = x + y + z
B) a . b . c = x . y . z

C) ^~a + / b + /c = yfx + y[y + / z

D) a2 + b2 + c2 = x2 + y2 + z2
E) Ninguna.

Solución
Se observan:
ACQ = FBA ; QCB = HÁB ; CÁH = FBC

Entonces:

a AL
A AFL - ABHL
z LB
c CL
A AQL - ACHL AL
y
b LB
A LQB - A CFL
x CL

Multiplicando miembro a miembro estas tres relaciones y simplificando:


1 k = AL CL LB
z ‘ y ' x LB ‘ AL ' CL

Rpta: ( B ).

30) En la figura mostrada, G es Baricentro del


A ABC. AE = 6 ; EB = 10 y B F = 1 1 .
Hallar FC.

A) 4 B) 3,6 C) 4,8
D) 3,8 E) 4,4

389
Solución
Incógnita: FC = x

Trazamos la mediana CGM.


AB 16
• • AM = MB = — = y = 8

Luego : EM = 2

Por propiedad del Baricentro: CG = 2GM

GM = m CG = 2m.

En el A MBC, podemos usar el Teorema de Menelao, ya que EF es una transversal


BF CG ME 11 2m 2
= 1
FC ' GM ' EB m 10

x = 4,4
Rpta: ( E ).

En el A ABC, recto en B, se sabe que AE = 7


y r = 3. ( H, E y F son puntos de tangencia).

Hallar BG.

A) 7,5 B) 6 C) 4
D) 4,5 E) 6,5

Por el Teorema de Menelao, en el A ABC:


AE CF BG ,
x x = 1
EC FB GA
AE BG
Pero : EC = CF; de modo que lo anterior queda : = 1 (a).
FB GA
De otro lado, es fácil probar que :
FB = r = HB y : GA = GB + BA ó : GA = GB + BH + AH
i i
Siendo AH = AE. Entonces : GA = GB + r + AE
AE BG
Luego, en ( a ) : x = 1
r (G B + r + A E )

7 BG
Con los datos: = 1
3 X ( BG + 3 + 7 )

De donde: BG = 7,5

Rpta: ( A ).
32) En la figura, BM es mediana del A ABC y P
cualquier punto sobre ella. Se han prolongado
luego CP y AP.
B
Demostrar, que : EF II AC.

Solución
AE CF
Bastará demostrar, q u e :
EB “ FB

En efecto, con el teorema de Ceva:


AE BF CM
= 1
EB ‘ FC ' MA
AE BF
Siendo CM = MA, queda : = 1
EB ' FC
AE FC , AE CF
Es d e c ir: o
EB BF EB FB

EF 11 AC .......... I.q.q.d.

33)
En la figura mostrada

1 + — = 0,25
AP AQ

Hallar AC.

A) 25 B) 2,5 C) 5
D) 4 E) 8

Solución
- Para el A ABC:
Teor. de Menelao : AE . BF . CQ = EB . FC . Q A (1 )

Teor. de Ceva: AE . BF . CP = EB . FC . P A (2)

- Dividiendo miembro a miembro (1 ) y ( 2 ) :

AE . BF . CQ EB . FC . QA
AE . BF . CP " EB . FC . PA

CQ = QA
Simplificando :
CP ” PA

PA QA
o, en forma equivalente :
CP " CQ

391
AP QA Expresión que indica a los puntos A, P, C, Q,
Mejor todavía : c
PC ” CQ conformando una cuaterna armónica.

1 1
Entonces, por la relación de Descartes : +
AC AP AQ

Usando el dato : = 0,25 AC = 8


AC

Rpta: ( E ).

34)
La figura muestra un trapezoide ABCD y una
recta que intercepta a dos lados y las prolon­
gaciones de los otros.
D em ostrar, que :

AF BE CG
DH- i
H FB EC GD HA

Solución

* Trazamos BD, cortando en R a la recta

* Usando el Teorema de Menelao:

Aon -» ----
a ABD
A AF . ------
BR . -------
DH = i1 (a )
FB RD HA
CG DR BE H
ABCD = 1 (P )
GD ' RB ' EC

Multiplicando ahora las expresiones ( a ) y ( p ), miembro a miembro y cancelando las


distancias iguales en numerador y denominador:
AF BE CG DH
— 1 .......... I.q.q.d.
FB ' EC * GD ’ HA

35) Dado el trapezoide A B C D ; M y N, bisecan


BC y AD, respectivamente.

S i- ^ = -°°=k.
PB QC
Demostrar, que:

.y :

**

392
D em ostración

En efecto: * Sean : PB = a y QC = b
AP = ka y DQ = kb.

* Tracemos : CC' II PQ y DD' | PQ

* Teor. Thales:
PD' QD
PC’ " QC
ka + AD1 _ kb
a + BC* " b
De donde:
AD’ = k ( BC’ )

* Además : RM 11 UN ; por ser paralelos a la misma recta C'D'

Luego: RML = UÑL y UN = k( RM )

* Sea L’ , punto de intersección de MN y RU. Entonces:


A RML’ - A U N L ’ L’U = k ( L’R ). Pero, según Thales, PQ ¡ntersecta a RU
determinando sobre él, segmentos que son entre sí como k a 1. Por lo tanto, L’ pertenece
a PQ. Así, concluimos que L y L’ son un mismo punto. Es d e c ir: R , L y U, son colineales.
NL
Además, de la semejanza anterior = k , tal como se pidió probar.
LM

Finalmente, es fácil demostrar que : LP = LQ, al usar los trapecios D’C’RU y URCD.

393
/

CAPITULO 13
v. r .V !
■ - *-**-. . . .

¡i A . v.

RELACIONES METRICAS EN
TRIANGULOS RECTANGULOS
PROYECCIONES.- La proyección ortogonal de un punto Pf sobre una recta L, es el pie de
la perpendicular trazada desde P, a L. En la figura adjunta, PP* se llama proyectante.
Asimismo, la proyección de un segmento (cualquier figura, en general), se obtiene de
proyectar todos los puntos de dicha figura, sobre la recta.
A'B' Proyección ortogonal o simplemente proyección, de AB, sobre L.

P
B E
O

A i R

A M
/: N
ii
_ ii i N'
II D D__ Q n n ü n
P’ A1 D* C’ Q R* M’ ^r
B’ C*
N

Ejem plo. En la figura, se han trazado las alturas del A ABC:

AH -» Proyección de AB, sobre AC. B


HC Proyección de BC , sobre AC
u
“ AC, sobre AB.
it
PC u AC, sobre BC
QB

PB

395
RELACIONES METRICAS EN TRIANGULOS RECTANGULOS

En todo triángulo rectángulo, se cumplen las siguientes propiedades:

I. El cuadrado de la longitud de un cateto, es igual al producto de longitudes de la hipotenusa


y su proyección sobre dicha hipotenusa.

El cuadrado de la longitud de la altura relativa a la hipotenusa, es igual al producto de


longitudes de los segmentos parciales que determina en dicho lado.

Teorema de Pitágoras: La suma de los cuadrados de longitudes de los catetos, es igual al


cuadrado de longitud de la hipotenusa.

IV El producto de longitudes de los catetos es igual al producto de longitudes de la hipotenusa


y la altura respectiva.

V. La suma de las inversas de los cuadrados de longitudes de los catetos, es igual a la inversa
del cuadrado de la longitud de la altura.
Sea el A ACB, recto en C. Entonces:

( a2 + b2 = c2

(IV)

D em ostraciones

Se sabe, que : BCH = Á y HCA = B ( Propiedad )


a m
I. A BHC - A BCA: - = a2 = c . m
c a
b n
A CHA - A BCA: - =- -> b2 = c . n
c b
h m
A BHC - A CHA: - = — h2 = m . n
n h

Sumando los resultados de ( I ) : a2 + b2 = cm + en


a2 + b2 = c ( m + n )
a2 + b2 = c ( c ) .*. a2 + b2 = c2

* Pitágoras demostró de otro modo su teorema. Paraello, ver el capítulo 17: AREAS

IV Multiplicando los resultados de I : a2b2 = c^mn


Con ( I I ) : a2b2 = c2h2
De donde : ab = ch
1
V. D e(l): -4
3 cm en

396
S um ando:
1
+
1 1 1
+ ---- S
2
a2 b2 cm en c Vm n )

1 1 _ 1f m + n
+ Pero :m + n = c y mn = h2

CM
c { mn

n
a2
1 1 1
+
a2 b2 “ h2

Ejem plo

En el gráfico adjunto:

a2 = 25 x 9 -> a = /2 5 x / 9

a = 15

b2 = 25 x 16 b = /2 5 x / Í 6

b = 20

h2 = 9 x 16 -> h = 12

Ejem plo

En la figura, E, F y T son puntos de tangencia


r = 3 y AE = 5.

Hallar EC.

Solución
Incógnita : EC = x

Tangentes: EC = CF y AE = AT
□ OFBT -*• cuadrado

Teor. de Pitágoras:

AC = AB + BC

( 5 + x )2 = 82 + ( 3 + x )2
F
De donde : 3
x = 12 EC = 12
Rpta

397
PROBLEMAS RESUELTOS

1) A ambas orillas de un río crecen dos palmeras, una frente a la otra. La altura de una es de
30 codos, y la de la otra, es de 20. La distancia entre sus troncos, 50 codos. En la copa de
cada palmera hay un pájaro. De súbito los dos pájaros descubren un pez que aparece en
la superficie del agua, entre las dos palmeras. Los pájaros se lanzaron con la misma
velocidad y alcanzaron al pez al mismo tiempo. ¿A qué distancia del tronco de la palmera
mayor apareció el pez?

A) 10 codos. B) 15 codos. C) 20 codos. D) 25 codos. E) 30 codos.

Solución
* Como llegan al mismo tiempo y van a la
misma velocidad, entonces las distancias
hacia el pez son iguales ( d ).

Luego:
d2 = x2 + 3 O2
y d2 = 202 + (50 - x)2

Igualando los segundos miembros:


(50-x)
x2 + 900 = 400 + (50 - x)2
x = 20 50
Rpta: ( C ).

2) Hallar la longitud del radio de la circunferencia inscrita en un rombo cuyas diagonales miden
12 y 16cm., respectivamente.

A) 4 B) 4,8 C) 5 D) 4,2 E) 5,6

Solución

- Sea ABCD el rombo, con :


AC = 12 y BD = 16.

AO = OC = 6 y BO = OD = 8

- En el A BOC, por fórmula ( V ) :

.2
BO OC A
( OH es altura y radio )
v
1
_____
1 1
r 2 " 82 + 62

Efectuando: r = 4,8

Rpta: ( B ).

398
3) Una rueda descansa en el piso y está apoya­
da además sobre un ladrillo de 4 pulgadas de
alto, quedando su punto de apoyo en el piso
a 10 pulgadas del lado más cercano del ladri­
llo. Calcular el radio de la rueda.

A) 6,0 pulgadas. B) 8,0 pulgadas.


C) 12,5 pulgadas. D) 13,8 pulgadas
E) 14,5 pulgadas.

Solución

AAEO : AO = AE + OE

r2 = 102 + ( r - 4 )2

8r = 116

r = 14,5

Rpta: (E).

4)
En la figura, O es centro. AB 11 CD ; AB = 12 y CD = 10
Hallar r , sabiendo además que AB y CD , distan 1
unidad, entre sí.

B A) / 6 1 B) / 6 2 C) / 6 3
D) 8 E) / 6 9

- Trazamos OM 1 CD, cortando a AB en el


punto H . Entonces : CM = MD = 5 y
AH = HB = 6.

HM = 1 ( dato ).

- Sea OH = a

- Con el Teor. de Pitágoras:

AOMC -> OM + MC =O C ( a + 1)2 + 25 = r2 ( 1 ).

AOHB OH2 + H B 2 = O B 2 -> a2 + 36 = r2 (2).

Primero hallamos “a”, restando (1 ) - ( 2 ) :


( a + 1 )2 - a2 + 25 - 36 = 0
a = 5

399
Reemplazando en ( 2 ) : 2 2
r = a + 36
r2 = 25 + 36

• » r =
Rpta: ( A ).

En un triángulo ABC, recto en B, se traza la ceviana interior B R , tal que AB = BR


Hallar AB, s i : AC . AR = 72.

A) 6 / 2 B) 9 C) 6 d) 8 e) 3 / 2

Solución
Dato : AC . AR = 7 2 .......... ( 1 ).

AB = ? B

Se traza la altura B H . Luego, en


el A ABR, isósceles :
AR
AH = HR = — ......... ( 2 ) .

Por relación métrica ( I ), en el A ABC:

AB = AC . AH
Con ( 2 ) :
AR AC . AR
AB = AC. AB =

72
Con el dato ( 1 ) : AB = AB = 6

Rpta: ( C ).

En el gráfico, se tienen dos circunferencias ortogo­


nales de radios r y R . M y N son puntos de
tangencia.

Hallar MN.

A) 2 / R r B) / R2 + r2 C) / 2Rr

M D) t¡ 2 ( r 2 + r2 ) E) N.A.

Solución
Por ser ortogonales, las circunferencias : OA _L PA.

Trazamos OM, PÑ y PH 1 OM:


HMNP rectángulo : HP = MN.
Teor. de Pitagoras :

AOAP OP = R2 + r2 d )

AOHP HP = OP2 - OH2

Con (1 ) : HP = R2 + r2 - ( R - r f

HP = 2Rr

MN = >/ 2 R r

Rpta: (C).

Hallar la distancia OP entre los centros de las


circunferencias.

A) / R
B) R + r

C) R

D) 7 2 ( R 2 + r2 )

E) 2 / R r

Solución

Con los trazos indicados:


PH = PB - HB -> PH = R - r
y OH = OE + EH -» OH = R + r

A OHP, teor. de Pitágoras:

OP = OH + PH

OP = ( R + r )2 + ( R - r )

OP
)
Rpta: ( D ).

En la fig u ra , O es centro del cuarto de circi


ferencia y T punto de tangencia .
AH = 2 ; BE = 9.
Hallar la longitud del radio.

A) 11 B) 14 C) 15
D) 18 E) 17
401
Solución
r = ?

- Trazamos OT y por O , !a paralela


a HE: PA = r - 2 y QB = r - 9

También : OAP = BOQ = a

y AÓP = OBQ = 0

- A OPA = A BQO OP = BQ

OP = r - 9 ............( 1 ).

En el A OPA, teorma de Pitágoras OP + PA = OA


Con (1 ) : ( r - 9 )2 + ( r - 2 )2 = r2

Resolviendo : r = 17

Rpta: ( E ).

9) En un A ABC, recto en B, se trazan la altura BH; HE 1 AB y HF 1 BC ( E en AB y F en


BC ). S i : AE = 1 y FC = 8, hallar EB y BF.

Solución
- EB = x , FB = y ( Incógnitas ).

- EHFB, es rectángulo.

EH = BF EH = y

H F = EB HF = x

- Por fórmula ( I I ) :

A AHB HE" = AE . EB y2 = 1 . x y" = x (a).

ABHC HF = FC . BF -» x2 = 8y (p).
De ( a ) : x = y2 , reemplazando en ( ( 3 ) : ( y2 )2 = 8y y3 = 8 • « y = 2

Luego : x = y2 = 22 -> x = 4

Rpta : EB = 4 ; BF = 2

10) Demostrar que el segmento tangente


exterior común a dos circunferencias
tangentes exteriores, de radios V y
“R", tiene longitud:

402
Solución

Considerando el gráfico, donde AB


es el segmento en mención; “O” y
“ PM, centros, se traza PH 1 OA,
entonces:

Por teorema de Pitagoras:

HP = ( R + r f - ( R - r )

x2 = 4 Rr

= 2/R r

AB = 2 / r7 .......... I.q.q.d

En la figura, A, B y C, son puntos de tangencia. Demostrar, que:

Por propiedad a n te rio r:


AB = 2 / R x , AC = 2 / R r y BC = 2 / 7 x

Siendo : AB + BC = AC

• • 2 /R ~ x + 2 / r x ’ = 2 / R ?

Efectuando: /7 ( /R + / r ) = /R r

/R + /r _ 1
6 /R ? "7 7

/R /r 1
De donde :
/R r /R r /" x

1_ _J_ _ _t
I.q.q.d
/ r /R / x
12) Dada una semicircunferencia de centro 0 y
10 cm. de radio, se le inscribe un triángulo
rectángulo cuyo cateto CB mide 8 cm. En B se
traza la tangente BP que encuentra en P a la
perpendicular OH a la cuerda BC. La longi­
tud de la tangente BP será :
B
B) 20 /ü
V21
C) 10
21 21

D) 2 / 2 1 E) 30 J li
21
Solución
* BC = 8.
BC
* BH = — BH = 4

Por relaciones métricas en el A OBP:

1 1 1
2 +
42 x¿ 102
1 1 1
x2 " 42 102

1 102 - 42 84 2 1600 40
X = ------- x = • «

x2 ( 42 ) ( 102 ) 1600 84 /8 4

13) En un triángulo ABC, recto en B, se traza la altura B H . SL r , ^ y r2son inradios de los


triángulos ABC, AHB y BHC, respectivamente, demostrar que : r2 = ( r, )2 + ( r2 )2.

Solución
Los triángulos ABC, AHB y BHC, son
semejantes. Escribimos la relación de
inradios e hipotenusas : B
JL = JL = JL
AC AB BC

Elevamos al cuadrado :

r2 (»i)2 ( r2 f A H c
AC2 AB2 BC2

r2 ( ri )2 + ( r2 )2
Con propiedad de proporciones: — 2 ” — 2 — 2
AC Ad + d L»

Siendo los denominadores iguales; luego: r2 = ( r, )2 + ( r2 )2.

404

i
En la figura, AB y AC son diámetros

CT es tangente a AB.

Hallar ET, si AB = BC = 2r,

Je 2^6
A) r B) C)

D) r E) N.A.

Solución

Incógnita : ET = x

Como B es centro de la mayor


semicircunferencia y BT ± AT,
entonces AT = ET.
A ADC - A OTC :

AD AC Arx 4 r
= —> AD = —
OT OC 3

A OTC, con el Teor. de Pitágoras TC = 2 r / 2

DT AO DT _ r _ 2r/2
Teor. de T h a le s : . DT =
TC “ OC 2r/2 3r 3

Finalmente, por el teor. de Pitágoras en el A ADT:

2 ( 2rJ~2
——2 ——2 ——2 O 4r '
AT = AD + DT x = +
co

_ 2/6r
x= ET
3

Rpta: ( C ).

El rectángulo ABCD tiene la base AB = 16cm. El círculo tangente a los lados AB, BC y CD
y el círculo tangente a los lados CD, DA y AB se intersecan en los puntos M y N. Si la
distancia MN mide 8 cm., la altura del rectángulo, es:

A) 12cm. B) 11cm. C) 10cm. D) 9cm. 8


E) cm.

405
Solución
x = 2R

Pitágoras en el A OLM
R2 = 42 + ( 8 - R )2

R = 5

x = 10

Rpta: ( C ).

16) Se tiene un triángulo ABC recto en B, la circunferencia inscrita es tangente a la hipotenusa


en “T ”, se trazan luego TH 1 AB y TF T BC. Calcular AT . TC s i :
AH . HB + BF . FC = 100m2.

A) 50m2 B) 75m2 C) 100m2 D) 125m2 E) 150m2

ef + rs = mn

Dato : ef + rs = 100 mn = 100

Rpta: (C )

17) Del gráfico, hallar ux” en función de “a” y uc'

A)

a3
B) —
c

C)

D)S
0 3

406
Solución
Por relación métrica.

A CHB : RB = ax (1 )
A AC B : a2 = c ( H B )

HB =

Reemplazando esto en la

expresión (1 ): = ax

De donde : x =

Rpta: ( B ).

18) Sea el gráfico:


C

Demostrar, que

Solución
- Del prob. anterior:

_(A C )\
; Luego:

1 £
AC = b 3 . c 3

1 £
También : BC = a 3 . c 3

Por Pitágoras: ( BC )2 + ( AC )2 = ( AB )2 ;

£ i £ i
Sust. a3 . c3 + b3 . c3 = c

£ £ £
a3 + b3 = c3 .... I.q.q.d

407
19) En cada caso, AB es diámetro de las semicircunferencias. Demostrar, lo que se indica.

(a ) (b )

Solución

a) - Se traza AC y CB : ACB = 909.

- En el A ACB. por relación métrica:

CH = AH . HB l.q.q.d.

b) - Se traza CB.

- En el A ACB, recto en C, por relación métrica

AC = AB . AH l.q.q.d.

20) En la figura, AB, AC y BC son diámetros.


PQ tangente y HB _L AC.

Demostrar, que:

Solución
- Para la tangente, sabemos que :
PQ = 2 / R r ........... ( 1 ).

- Por propiedad ( a ) del problema a n te rio r;

HB = AB . BC

HB2 = 2R x 2 r -> HB = 2 / R r (2).

De (1 ) y ( 2 ) :
HB = PQ .......... l.q.q.d

408
21) En la figura, AO = OC ; AO y AC
son diámetros. Si AD = 2.

Hallar AB.

A) 2 / 3 B) 4 C) 3

0)4/2 E) 2 /2
Solución
- Por propiedad ( b ), demostrada en un problema a n te rio r:

AB =AC.AH AB = 2R . AH ( 1)

También : AD2 = AO . AH -> 22 = R . AH

R . AH = 4 ........... ( 2 ).

- Reemplazando ( 2 ), en ( 1 ) :

AB2 = 2 x 4 AB = 2 / 2

Rpta: ( E ).

22) En la figura, AC y AD son diámetros


BM = MH.
Hallar BH, s i: A H . CD = 27.

A) 6 B)3/3 C )2/3
D) 9 E) N.A.

Solución
- BH = ?

Dato : AH . CD = 27
- Por propiedad ( a ) demostrada en un problema a n te rio r: ( n® 19 )

BH = A H . HD

Es d e c ir: BH = AH . ( HC + CD )

BH = AH . HC + AH . CD

BH = AH . HC + 27 ............( 1 ).

- Además : MH = AH . HC

BH
P ero: MH =

409
BH
Entonces : = AH. HC ( 2 ).

- Reemplazando ( 2 ), en (1 ) : 2
BH = — + 27

De donde : BH = 6

Rpta:
w % w ( A ).
% % M m

23) Las diagonales de un trapecio son perpendiculares entre sí y tienen longitudes 6 y 8u


Hallar la longitud de la altura.

A) 9,6 u. B) 4,8 u. C) 2,4 u D) 5 u E) 6 u.

Solución
Sea el trapecio, ABCD, donde
AC = 6 y BD = 8.
B
Se traza CR BD CR = BD = 8

La altura relativa a AR, en el A ACR,


es la misma que la del trapecio.
En dicho triángulo se escribe:
1 1 1 1 1 1
+ +
( AC f ' ( CR f ' h2 8

h = 4,8

Rpta: ( B ).

24) En un trapecio isósceles se puede inscribir dos circunferencias de radios 3 y 4cm. El valor
de la mediana, será:

25 r r 13 r r 14 rr-
B) — <4 3 cm. C) — j 3 c m . D) — y 3 cm- E) N.A.

Solución
* Recordemos que para dos circunferencias
tangentes exteriormente, las dos tangentes
comunes exteriores y la interior, miden igual,
y se calculan en función de los radios.

PQ = TV = EF =

PF = FM = FQ = ET = EM = EV

* Para nuestro problema, sea ABCD el trape­


cio en mención.

410
Por lo anterior: PQ = 2 ^ 4 ( 3 )

PQ = 4 / 3

PF = FQ = 2 / 3

Sabemos además que las bisectrices de


los ángulos conjugados internos entre
dos paralelas, son perpendiculares, por
lo que:
CÓF = 90° = FÓ'D

* Por relaciones métricas :

A COF QP = CP x PF

32 = 1 x 2 / 3

A FO'D -> O 'Q = F Q x Q D -> 42 = 2 / 3 x 5 .


y 2

a + b 25
* Finalmente, la mediana : /3

Rpta: ( B ).

En la figura : “A" , “B" y «c«


25)
B son puntos de tangencia.

Probar que:

Se traza BC y se prolonga hasta F.


Se traza FA.
Se sabe, por propiedad, que :
AÓB = 90°. Luego, AF es diámetro.

En el triángulo FAB :
1 1
+ ------1
(A C )2 (A F )2 (A B )2
411
1 1 1
+
( AC) ¿ 4FT 4R r

AC = 2 r ' r
R + r

p
Nota: Análogamente : BC = 2 r '

Ei Mtensa cuerdas
. rr ••

Hace siglos, un matemático egipcio recibió el nombre d e Mtensa* cuerdas", a causa W í.V. .

- oeste^ lin a Vez


determinada la línea norte - sur, encontraba la posición de la otra formando un triángulo
del modo siguiente:
•:v-' >•

v ¿y.

distancias PS » 3, SQ 4 y QT .«■
Colocaba los nudos S y Q sobre la línea norte - sur* y con la cuerda tensa los fijaba con
estacas. Al hacer coincidir ios nudos P y T, obtenía un ángulo recto en S, y por lo tanto '
la dirección este - oeste» respecto a S,
s ; v,/ .■ . •:•>**'. , ..

• ♦t' **1* -;&H.' •


."V .
•••+

í5
Q
&
!4
i
I

_

13
0 0 : ■[ .

.'/ -t-V ■V * v , ' , . • 'v ;.

412
r \

RELACIONES METRICAS EN
TRIANGULOS OBLICUANGULOS
I) TEOREMA DE EUCLIDES:

1er. CASO.- En todo triángulo, el cuadrado de la longitud del lado que se opone a un ángulo
agudo, es igual a la suma de cuadrados de los otros dos, menos el doble producto de uno
de ellos , por la longitud de la proyección del otro sobre él.
Sea ABC , el triángulo ; donde a < 90. AH : proyección de AB sobre AC-
Entonces: «
B

D em ostración
A m H C
- Con el Teorema de Pitágoras : b

A BHC - a2 = BH2 + HC2 a2 = BH2 + ( b - m )2 .......... (1 ).

A AHB - BH2 = c2 - m2 ............( 2 ).

- Reemplazando ( 2 ), en ( 1 ) : a2 = c2 - m2 + ( b - m )2

De donde, efectuando y ordenando : a2 = b2 + c2 - 2bm ........... I.q.q.d.

2do. CASO.- En todo triángulo obtusángulo, el cuadrado de la longitud del lado opuesto al
ángulo obtuso, es igual a la suma de cuadrados de los otros dos, más el doble producto de
uno de ellos por la proyección del otro sobre él.
Consideremos el A ABC, obtuso en A. a > 90
B
Luego:

AH : proyección de AB, sobre AC.

413
Demostración
Con el Teorema de Pitágoras :

ABHC a2 = BH + HC 2
a = BH + ( m + b )2 ..........(1 )

ABHA BH = c2 - m2 (2).

( 2 ), en ( 1 ) : a2 = c2 - m2 + ( m + b )2

a2 = b2 + c2 + 2bm .......... I.q.q.d.

TEOREMA DE HERON.- En todo triángulo, la longitud de una altura, es igual al doble de la


inversa de la longitud del lado sobre el cual cae, por la raíz cuadrada del producto del
semiperímetro y su diferencia con la longitud de cada lado.
Consideremos los gráficos adjuntos; en cada caso, el triángulo en mención es ABC.
El semiperímetro p :
fig- 1 fi g.2
a + b + c
P= z------
B
La fórmula, para el Teorema de
Herón, con relación a la altura BH:

h = ÍU p ( p - a)(p-b>)(p-c )
P

D em ostración

Para ambas figuras, por el Teorema de Pitágoras : h2 + m2 = c2


h2 = c2 - m2

ó, mejor : h 2 = ( c + m ) ( c - m ) ........... (1 ).

Según el Teorema de Euclides :

b + c - a
fig-1 a2 = b2 + c2 - 2bm m =
2b
(2 )
a 2 - b2 - c
fig.2 a2 = b2 + c2 + 2bm m =
2b

Sustituyendo en ( 1 ), la primera de las expresiones indicadas en ( 2 ) :

l 2 2 2 x
/
b + c - a b2 + c 2 - a2 ^
h2 = c + C -
2b y 2b /

2
( bc + b + c - a2 2 2 2bc - b 2 - c 2 + a 2 )
h =2
4b
ó, mejor; al notar los desarrollos de binomios suma y diferencia:

( b + c )2 - a2 a2 - ( b - c )2
h2 =
4b

Cada expresión entre corchetes es una diferencia de cuadrados. Usando los equivalentes:
u2 _ (b + c + a ) ( b + c - a ) ( a + b - c ) ( a - b + c )
= ib 1

Teniendo en cuenta que : a + b +c = 2p

Se obtienen :b + c - a = 2p - 2a
a+b-c = 2p - 2c
a+c-b = 2p - 2b

En la expresión de h2 :
2 _ (2 p )(2 p - 2 a )(2 p - 2 c )(2 p - 2b)
4b
Es decir :
2 _ 16p ( p - a ) ( p - b ) ( p - c )
h* =
4 b2

De donde : h = — ^ /p ( p - a )( p - b ) ( p - c ) .......... I.q.q.d.

Nota.- Los pasos a seguir para la demostración, son análogos si se reemplaza la segunda
expresión de m, dada por ( 2 ), en (1 ).

TEOREMA DE LA MEDIANA.- En todo triángulo, la suma de cuadrados de las longitudes


de dos lados, es igual a dos veces el cuadrado de la longitud de la mediana hacia el tercer
lado, más la mitad del cuadrado de la longitud de dicho lado.
Sea BM una mediana del triángulo ABC.
Entonces:

D em ostración

Por el Teorema de Euclides

A ABM ( ler.caso, para el AMB ) : c2 = m2 + AM - 2AM . HM


(1)
A BMC ( 2do.caso, para el BMC ) : a2 = m2 + MC + 2MC . HM

415
Efectuando la suma miembro a miembro:

a2 + c2 = 2m2 + ÁM 2 + MC2 - 2 AM . HM + 2 MC . HM

Siendo AM = MC = ~ :

/
( r b > r b >
a 2 + c 2 = 2m 2 + + - 2 HM + 2 - IHM
,2j <2, y2)

De donde, efectivam ente:

a 2¿ + c 2¿ = 2m
« 2 k
¿+ — l.q.q.d

IV) TEOREMA DE LA PROYECCION DE LA MEDIANA En todo triángulo, la diferencia de


cuadrados de las longitudes de dos lados es igual al doble producto del tercero y la proyeción
de la mediana relativa a él.

D em ostración

Para el anterior gráfico, efectuando la diferencia de las expresiones obtenidas en ( 1 );


b
teniendo en cuanta que AM = MC = — :

a2 - c2 = 2MC x HM + 2AM x HM
b b
a2 - c2 = 2 x — * HM + 2 x — x HM

( HM es la proyección de BM sobre AC )

V) TEOREMA DE EULER.- En todo cuadrilátero, la suma de los cuadrados de las longitudes


de los lados, es igual a la suma de cuadrados de las longitudes de las diagonales, más el
cuadrado de la distancia entre los
B
puntos medios de las diagonales.
Sea ABCD, et cuadrilátero. M, pun­
to medio de AC y N, punto medio
de BD.

Entonces:

416
D em ostración
Con el Teorema de la mediana, anteriormente demostrado
p o n
— 2
CJ r ) B
A B M D i BM + M D = 2MN + ( 1 ).

p o o Af
A ABC : AB + BC = 2BM + ... ( 2 ).

—o -o 2 AP
A ACD : CD + AD = 2MD + - ^ - = ^ . . . ( 3 ) .

Sumando las expresiones ( 2 ) y ( 3 ), miembro a miembro

/
AB + BC + CD + AD =2 BM + MD + AC

Con lo de (1 ) :
/

BD
AB + BC + CD + AD =2 2MN2 + + AC
\ /

Efectuamdo y ordenando:

ÁB2 + BC2 + CD2 + ÁD2 = AC2 + BD2 + 4M Ñ 2 ........... I.q.q.d.

Nota : La propiedad es también válida para los cuadriláteros no cenvexo y alabeado

TEOREMA DE STEWART.- En todo triángulo, la longitud de una ceviana interior, puede


evaluarse con la siguiente expresión :
B
A ABC BE, ceviana interior.

a2m + c2n = x2b + mnb

417
D em ostración
B
Se traza la altura B H . Por el Teorema
de Euclides:
A ABE -» Para a < 90°:

c2 = m2 + x2 - 2m . H E ( 1 ).

A BEC -» Para § > 90°:

a2 = n2 + x2 + 2n . H E (2).

Ahora, multiplicando la expresión ( 1 )


por n y la ( 2 ) por m y efectuando la
suma miembro a miembro:

c2n + a2m = m2n + mn2 + x2n + x2m - 2mn . HE + 2mn . HE

Es d e c ir:

a2m + c2n = mn ( m + n ) + x2( n + m ) , pero : m + n = b

Entonces:

a2m + c2n = mnb + x2b .......... I.q.q.d.

VII) NATURALEZA DE UN TRIANGULO


A
Sean a, b y c, longitudes de los lados de un
triángulo ABC, con el lado mayor de longitud
a. Entonces, para saber si ABC es un trián­
gulo rectángulo, acutángulo u obtusángulo,
se comparan a2 y b2 + c2

(I) S i : a2 = b2 + c2 —>A ABC, es recto en A.


B
(II) S i : a 2 < b 2 + c2 -» A ABC, es acutángulo.

( I I I ) S i : a2 > b2 + c2 A ABC, es obtusángulo, con el Z A obtuso.

Ejem plo. ¿ De qué naturaleza es el triángulo cuyos lados tienen longitudes 5 ; 7 y 9 ?

Solución
Como : & = 81 y 52 + 72 = 25 + 49 = 74

Se observa: 92 > 52 + 72, entonces se trata de un triángulo obtusángulo. El ángulo obtuso


es el opuesto al mayor lado ( 9 ).

En ambos gráficos: a > 90°.


418
RELACIONES METRICAS EN TRIANGULOS OBLICUANGULOS
( RESUMEN )

1) Teorema de Euclides
a < 90 a > 90

a2 = b2 + c2 - 2bm a2 = b2 + c2 + 2bm

2) Teorem a de Herón:
B

h = ^ V p ( p ~ a ) ( p “ b )(p - c )

. . a + b +c . .
donde p = ----------------- semiperimetro

3) Teorema de la Mediana:

4) Teorem a de la p ro ye cció n de la m ediana B

BC - AB = 2 AC . HM

419
5) Teorema de Euler

( ABCD: cuadrilátero cualquiera ).

x : distancia entre los puntos me­


dios de las diagonales.

a2 + b2 + c2 + d2 = AC + BD + 4x2

6) Teorema de Stew art

BR : una ceviana cualquiera.

a2q + c^n = x2b + qnb

PROBLEMAS RESUELTOS

1) En un triángulo ABC, AB = BC, se traza la altura AH ( H en BC ) . Si: BC . CH = 18


Hallar AC.

A) 9 B) 6 C) 3 D) 5 E)3/3

Solución

- Datos: AB = BC
B C . CH = 18

- Incógnita: AC

- Por el Teorema de Euclides ( a < 90° )

AB = AC + BC - 2BC . CH

Cancelando AB y BC :

0 = AC - 2BC . CH

AC = 2BC . CH

Con el dato:

AC =2x18 -> AC = 6
Rpta: ( B ).

420
Los lados de un triángulo miden / 2 ; /6 y /8 • Hallar la longitud de la menor altura

A) y[6 B) J 2 C )/8 D) E) N.A.

Solución
La menor altura es referida hacia el mayor
lado:

No conviene el Teorema de Herón.

Primero, hallamos “m”, con el Teorema de


Euclides: ( a < 90°).
2 /—2 r~2
y[ 6 = j 2 + j 8 - 2 y[ 8 m

m=

Luego, del Teorema de Pitágoras;

h2 = 2 - m2 h=

Rpta: ( D ).

A dos circunferencias concéntricas de 7m y 9m de radio, se traza una secante tal que la


cuerda interceptada por la circunferencia mayor resulta dividida en tres partes iguales por
la otra circunferencia.
Dicha cuerda tiene por longitud:

A) 3 ^ 2 m. B )6 y]~2 m. C) 15m D) 12m E) N.A

Solución

AD = ?

Con el gráfico:

AB = BC = CD = 2x

En el A AOB, obtuso en B,
por el Teor. de Euclides:

AO = AB + BO + 2( AB )( BM )

92 = ( 2x )2 + 72 + 2( 2x )( x )

32 = 8x2 x = 2

Luego: AD = 6x AD = 12

Rpta: ( D ).
421
4) En la figura, O y B son centros. Los radios miden 6y 10. Hallar PE

A) 8
B) 7,5
0 )7
D) 7,2
E) 8,2

Solución
PE = ?

Se proyecta PE sobre OB
OH = PE. Luego, se trazan
los radios.
OP = 10 y BP = 6

En el A OPB, con el Teorema


de Euclides ( a < 9 0 ° ) :

PB = OP + OB - 2 x OB x OH
H B
62 = 102 + 102 - 2 x 10 x OH 10 —

De donde :
OH = 8,2 PE = 8,2

Rpta: ( E ).

5) En un A ABC, AB = 3 y BC = 7. Hallar el máximo valor entero de AC, sabiendo que el


B es agudo.

A) 10 B) 9 C) 8 D) 7 E) 6

Solución

Como el B es agudo, se debe cumplir:


B

AC < 32 + 72

AC < 5 8 AC < 7,62

El máximo valor entero de AC, es 7

Rpta: ( D ).

6) En un A ABC, AB = 13 ; BC = 14 y el B es obtuso.
Hallar el mínimo valor entero de AC.

A) 27 B) 26 C) 25 D) 20 E) 21

422
Solución

* Si el B es obtuso, sabemos que


se debe cumplir:

AC > 132 + 142

AC > 3 6 5 AC > 19,10

El mínimo valor entero de AC, es 20

Rpta: ( D ).

7) En un triángulo ABC, hallar la medida del Á , sabiendo que entre las longitudes de sus lados
se cumple:

(D -

A) 60 B) 30 C) 45 D) 75 E) N.A.

Solución
Dato : a2 = b2 + c2 - be (1 )
B

Teorema de Euclides:

a2 = b2 + c2 - 2b . AH (2)

De ( 1 ) y ( 2 ) :
b2 + c2 - be = b2 + c2 - 2bc . AH

AH = -

Luego, en el A AHB a = 30° y

( Propiedad, siempre que ( I ) se cumpla )

Rpta: ( A ).

8) En el interior de un rectángulo ABCD,


se toma un punto UP”.
Demostrar, que:

423
Solución
Por “P" se traza una perpendicular a las bases.
Aplicando el Teorema de Euclides:

En el A APD PD = PA +AD -2AD .AQ

En el A BPC PC = PB + BC -2BC.BH

Al restar miembro a miembro estas expresiones , y como : BC = AD ; AQ = BH ;

Queda : PD - PC = PA - PB PB -f PD = PA + PC l.q.q.d
p-
A
\
1 \
f
1
\

.1

Nota.- Demuestre el lector, lo mismo,


v L
V
<
\

para el caso en que el punto P sea 1


1
1
\

i s

exterior al rectángulo. 1
1
1
s
\
\
>
s
1
»
1
t
/ s

9) En un triángulo acutángulo ABC, se trazan las alturas AH y CQ.


S i: A B . A Q = 24u2 y CB . CH = 25u2 . Hallar AC.

A) 12 B) 5 C) 7 D) 15 E) 14

Solución
Teorema de Euclides

A -> BC = AB + AC -2AB.AQ...(1).

C AB = BC + AC - 2CB . C H ...( 2 ).

Sumando ( 1 ) y ( 2) , miembro a miembro :

BC + AB = AB + BC + 2 AC - 2 AB . AQ - 2C B . CH

Luego de cancelar términos iguales y sim plificar:

( Propiedad ).

Con los datos : AC = 2 4 + 25 AC = 7

Rpta: ( C ).

10) En un trapecio ABCD , BC I AD ; AB = 15, BC = 10, CD = 13 y AD = 24


Hallar la longitud de la altura.

A) 9 B) 14 C) 12 D) 13 E) N.A.

424
Solución
Se traza el trapecio, con los datos:
Se traza BE II CD. 10

/. BE = C D = 1 3, ED = B C = 10
\ 13
y AE = 14

rr i a ao c 1 5 + 1 3 + 14 _. E
En el A ABE : p = -------------------- = 21
14 +- 10

Por el Teorema de Herón:

h= -> /p (p -a)(p -b)(p -c )

Esto es : h= J 21( 6 )( 8 )( 7 ) h = 12
14

Rpta: ( C ).

ma , mb , mc , son longitudes de las medianas de un triángulo, cuyos lados tienen,


respectivamente, longitudes a, b y c . Demostrar:

Solución

Según el Teorema de la Mediana

b2
a2 + c2 = 2mg +

a2 + b2 = 2m? + -

b2 + c2 = 2m? +

Sumando miembro a miembro :

a2 + b2 + c 2
2 ( a2 + b2 + c 2 ) = 2 ( m\ + m^ + m2 ) +

De donde, efectivamente, ai d e sp e ja r:

2 2 2 2 2
3( a + b + c ) = 4 ( m + rrib + m ) .......... I.q.q.d

425
L , es longitud del lado del cuadrado ABCD, de
centro O .
D,es centro del arco AEC y EM = MD.
Hallar OM.

A ) ^ o
E) N.A.

Solución
OM = ?

El radio de la circunferencia mide:—

y el del cuarto de circunferencia : L

En el A EOD, por el Teorema de la


mediana:

ED
OE + OD = 2 ( OM ) +

2 i

L ^ f L
+ —

l 2 j V2

De donde : OM =

Rpta: ( A ).

En un trapecio ABCD, BC 11 A D ; AB = 13, BC = 10, CD = 19 y AD = 32, hallar la distancia


entre los puntos medios de las bases.

A) 12 B) 11 C) 10 D) 13 E) 9

Solución

Sean M y N , puntos medios


™ 13 /
de BC y A D , respectivamente

Incógnita : MN \ '
N ' T 5 D
22 rr
- 32
•i
----------
iJ
T razam os: MR | AB y
MT || CD.

Luego : ABMR y MCDT Paralelogramos

MR = 13 , MT = 19 , AR = TD = 5 y RT = 22
En el A RMT, con el Teorema de la Mediana :

132 + 192 = 2( MN )2 + ( 22 f MN = 12
2
Rpta: ( A ).

En la figura: BC - AB = 1 4 4
AM = MC y m L AQM = 90°.

Hallar QM.

A) 12 B) 6 C) 9
D) 8 E) 10

Solución
QM = ?

En el A AQM, por relación métrica : QM = AM . HM.

2 A C . HM
Pero : AM = -^5. : Entonces : QM2 = . HM QM~ = (I)

De otro lado, en el A ABC, HM es proyección de la mediana B M , sobre AC. Luego, por el

Teorema de dicha proyección : BC - AB =2AC.HM

Con el dato :
144 = 2AC . HM AC . HM = 72

Reemplazando esto último, en ( I ) :

QM2 = — QM = 6

Rpta: ( B ).

En un trapezoide ABCD , m ¿ B = m L D = 90°, AC = 1 7 y BD = 15. M, es punto medio


de AC y F de BD. Hallar MF.

A) 3 B) 2 C) 5 D) 1 E) 4

Solución

AC = 17 ; BD = 15 ; MF = ?

Con el Teorema de E u le r:

AB + BC + CD -f DA = AC + BD + 4MF

AC + AC = AC + B D +4MF

427
2
De donde : MF =
2
AC - BD 2

17 - 15‘
- Con los datos : MF =

Rpta: (E).

En la figura, el radio de la circunferencia es V y “A”, es centro del BD

Hallar CE.

A) 2r

B )r/2

C)r / 3

D)r / 6
E) N.A.

Solución

Se traza la diagonal AC. Se une AE y EO.

2r

O : Centro de la Circunferencia.

En el triángulo AEC ; Teorema de la Mediana :

(AC)
AE + EC = 2( O E ) +

(2 r/2 )
( 2 r f + EC = 2(r) +

EC = r / 2

Rpta: ( B ).
Dado un trapecio ABCD ; BC A D , AB = 5 , BC = 4 , CD = 3 6
y AD = . Hallar la longitud
de la diagonal AC.

A) 7 B) 8 C) 9 D) 6 E) 10

Solución
Se traza la figura con los datos

Se traza CE 11 AB.

En el triángulo ACD, por Teorema


de S tew art:

AC . ED + CD . AE = CE . AD + AE . ED . AD

2 AC + 9x4 = 25x6 + 4 x 2 x 6

AC = 9

Rpta: ( C ).

En un triángulo ABC, las medianas BM y CN son perpendiculares entre sí

Demostrar que :

Solución
Se traza la figura correspondiente.

En el triángulo ABC: “G”, Baricentro.


Se traza la mediana A Q .
1
En el triángulo BGC, GQ = — BC

3
Por propiedad : AQ = 3GQ AQ = —BC

Por Teorema de la Mediana :

2 BC
AB + AC = 2 AQ +

' 3 — Y2 BC2
AB + AC = 2 —BC i + o
V2 i 2

De donde :

AB + AC = 5BC
l.q.q.d

429
19) Hallar “x”, si AC es diámetro ; A y B, centros de los arcos EB y FB.

A) Rr / (R + r)
B) 2Rr / (R + r)
C) 0,5Rr (R + r)
D) Rr / (R - r)
E) Ninguna

Solución
Sea O, centro de la semicircunferencia
AC R + r
AO = OC =
2
R+ r
También : OT =
2
y OM = OT - MT
r\KA R + r
OM = ------------ x

MO, es mediana del A AMC ; luego, con el Teorema para esta línea
2
AC
AM +M C = 2 ( OM ) +

(R+ r)
( R + x ) 2+ (r + x)2 = 21 ^ + r - x +
/
0, 5Rr
Efectuando : x =
(R + 0

Rpta: ( C ).

20) Hallar “x” , si los radios de las semicircunferencias son R y r.


4Rr( R - r )
A)
(R + r f

2Rr(R - r)
B)
( R + r )2

4Rr( R + r)
C)
(R -r)2
Rr
D) 4(R -r)

E) N.A.

430
Solución
- Siendo M, O y Q centros;
con los trazos:
MQ = r + x ; OQ = O T - Q T
-> OQ = R - x

y OM = OA - MA A r M O
—> OM = R - r u-

En el triá n g u lo M OQ ; el
semiperímetro p, es :

OM + OQ + MQ ( R - r ) + ( R - x ) + (r + x)
2

Aplicando el Teorema de Herón :

Efectuando : x = ? R r ^-R—
(R+ r)
Rpta: ( A ).

21) En la figura, “E” , “T 1 y “F" son puntos de tangencia.

o B E

Demostrar, que: AF = r ^ 2

Solución
- Llamemos “R” al radio de la circunferencia.
Se traza AM , MO y la perpendicular
MH a OA. A
H
Del gráfico:
Teorema de Euclides, en el triángulo AOM: R a

AM2 = AO2 + O M 2 - 2( AO ) ( OH )...(1 ).


Pero en el A A F M : A M - R2 + A F ;

Reemplazando valores en (1 ) :

R2 + ÁF2 = r2 + ( R + r )2 - 2( r ) ( R )

AF = r ^ 2 I.q.q.d

22) En la figura, AB, BC y AC, son diámetros. Hallar el radio de la circunferencia.

b ab
A) a + b B> i T b c) ^

a + b ab
D> 7 T b

Solución

Se unen los centros “M” con “Q" y


“M” con “O”. Se traza la perpendicular T

MP al diámetro mayor.
Los radios MT y ME.
El radio de la mayor semicircunferen-
AC
cia, mide : - r - = a+b
a+b

Si: uO” , “Q" y “M”, son centros. Del gráfico :

QM = a + x ; OM = OT - MT ; OM = a + b - x y OQ = OA - QA OQ = b

A d em ás:
OP = OC - BC - PB ; OP = a - b - x

En el triángulo QOM, el Teorema de Euclides:

QM = OQ + OM + 2 ( OQ )( OP )

Luego:
( a + x )2 = b2 + ( a + b - x ) 2 + 2b( a - b - x )

De donde :

ab
x=
a +b

Rpta: ( E ).
432
23)
Hallar la longitud de la cuerda AE,
si ñ es tangente común a las me­
nores circunferencias.
a,b y c radios

Solución
AE = ?

- Se tiene, con los trazos indicados:


AE = 2 ( AH ) ...............( I ).

- A AHO:

AH = a - OH ..... ( II ).

- Siendo: OH = BT = PB - PT = PB - c

- Para hallar PB, usamos el Teorema


de Euclides, en el A POQ :

OQ =PO + PQ -2PQ .PB

( a + b )2 = ( a + c )2 + ( b + c )2 - 2( b + c )PB

c2 + ac + be - ab
De donde : PB =
b + c

Entonces : BT = PB - c = c 2 + ac + be - ab _ q BT = — — — = OH
b + c b +c

/
ac - ab
- Reemplazando, en ( I I ): AH = a¿ -
\
b + c

_ 2a^bc
Efectuando : AH =
b + c

- Finalmente, en ( I ): Rpta

433
24) En la figura, O y M , son centros. Hallar la longitud de la tangente BE. R y r : radios.

A) ^ R( R + r )

B) ^ 2 R ( R + r )

C) ^ 2R ( R - r )

D) V R ( R - r )

E) ^ 2 R ( R - 2 r )
R —n
Solución
- Es más conveniente hallar la longitud
de B F , segunda tangente desde B.

- Con los trazos indicados: R


A MFB : BF = BM - MF o
A OMB , Teorema de Euclides: r
r

BM = OM + OB - 2 ( OB )( OP )
(R-r)
BM = ( R - r )2 + R2 - 2Rr

y , con MF = r , en ( I ) :

BF = ( R - r )2 + F¡2 - 2Rr - r2 BF = 2R( R - 2 r )

Rpta: ( E ).

25) En el triángulo ABC, de la figura : BC = a


AB = c y AC = b; G es Baricentro
y O es Circuncentro.

Demostrar, que:

434
Solución

Sea M , punto medio de AC.


OM 1 AC y por propiedad
del Baricentro:

BG = - ( B M ) y

Se trazan los radios OB y OC. •*t


«

En el A MBO , por el Teorema de


S tew art:

OG . BM + BG . GM . BM = OB . GM + OM . BG
Es decir:
1 2 1 2 2
OG . BM + - BM . - BM . BM = OB . — BM + OM . - BM
3 3 3 3

Simplificando BM :
1
OG + - BM = - - OB + - OM
9 3 3

R
Es d e c ir: OG = — + - OM - — BM o .
3 3 9
h2
Por otro lado, en el A OMC : OM = R2 ----- (II).

Y , en el A ABC, con el Teorema de la mediana :

oETS2 + —
2BM b = a2 + c 2 —> STñ2 a +o b
BM = -------------------- (III)
2 2 4

Reemplazando ahora, ( I I ) y ( I I I ) , en ( I ) :

R2
2 b2 2 ar2 + cd
2 b2
OG = + — R2
3 3 4 9 2 4

Efectuando:

2
'
ac + c2 N
OG - B l . ÍR * - i í
2 +
3 3 6 y
18

De donde:

435
En la figura, 0 1 y 0 2 son centros de las circunferencias C, y C2 , respectivamente .
0 , 0 2 = 2 . Hallar la longitud de una cuerda AE del círculo C2, sabiendo que el punto medio
de AE está en Cr

C.: A )/Í4

C)
/'
/14
D) 4
E) N.A.

Solución
AE = ?

Sea H el punto medio de A E .


Luego : AE = 2AH.
H € C r También : 0 2 H 1 AE

Por lo tanto, AB será diámetro


de Cr

Encontraremos primero 0 2B ,
luego H 0 2 y después AH.

En el A A B 0 2, por el Teorema
de la mediana para 0 , 0 2 :

( 0 2B)2 + / 2 2 = 2 ( 2 2 ) + ^ o 2b = 2 / 2
A

En el mismo triángulo, con el Teorema de Euclides; para el A 0 2 B :

( A B ) 2 = ( A 0 2 )2 + ( 0 2B )2 - 2( 0 2B )( H 0 2 )

22 = ( / 2 ) 2 + ( 2 ^ 2 f - 2 ( 2 / 2 )(H02 ) -> H02 -

Ahora, en el A A H O ,, con el Teor. de Pitágoras : ( AH )2 + ( H 0 2 f = ( A O ,)

3 /2 I VT4
(AH)2 + “ (V T J -> AH = • • AE =
\

Rpta: ( B ).
Dos circunferencias de radios “R” y V , se interceptan según un ángulo de 120°. Se traza
una tangente común externa AB ( A y B son puntos de contacto ). Hallar el radio de la
circunferencia que es tangente a las dos primeras y tangente a AB.

2R r Rr 4 Rr Rr V r
A) B) O) D) E)
4(/R + /F) 18
( + / r )' ( /R + -/r)

Solución
O y P centros.
Llamando V el radio pedido, se sabe, por propiedad, que : AE = 2 JW x y EB = 2 ^ rx

S i: m y ñ, son dos rectas tangentes a las circunferencias, en su punto de intersección “C",


se define el ángulo entre dichas curvas, como el mismo que forman m y ñ .

Con los trazos indicados, como:


OC 1 rñ y PC 1 ñ .*. OCP = 609

En el triángulo OQP, Teor. Pitagoras:

OQ + PQ = OP (1)

En el triángulo OPC, Teor. Euclides:

OP =O C +PC - 2 x OC x CH,
PC r
Siendo CH =
2

OP =O C + PC - OC . PC ....( 2 )

Sustituyendo ( 2 ) en ( 1 ) :

OQ + PQ = OC + PC - OC . PC

Luego: ( R - r )2 + ( 2 / R x + 2 ^ frx ) = R 2 + r2 - R . r

Rr
Efectuando : x =
4(/R +/7)'

Rpta: ( D ).

Sea ABC, en triángulo cuyos lados son a, b, c. Se divide cada lado del triángulo en n
segmentos iguales. Sea S la suma de los cuadrados de las distancias de cada vértice a cada
uno de los puntos de división del lado opuesto, distintos de los vértices.

Demuestre que : es un número racional.


a2 + b2 + c 2

( III Olimpiada Iberoamericana de Matemática ) 24 ABRIL - 01 MAYO, 1988. Lima - Perú.

437
Solución
Consideremos: B

Ib = ( X 1 )2 + ( X j ) 2 + ( X 3 )2 + . . . + ( xn1 )2

Donde, con el Teorema de Stewart hallamos


cada ( x¡ )2:
2( b ^ 2 <n - 1\

c
i
* ( x j 2b = a “ + c b - b .í b ^
^n ) ^ n ; , n ; k n J

De donde:
a2 r2 h2
( X, f = (1).
n n n
** En forma análoga:

/
( x2 ) b = a + c b - b.
n j \ n / n / n
/ \
n- 2 2b
( x 2 )2 = a2 +c (n “ 2) . . . . (2)
v n / V n n

Luego, por inducción :

/ 2 f n —1
\
\2 n - ( n - 1)
( Xn- 1 ) = a [ — + c
/ n
Entonces:
2 2
X h = ~ ~ [1 + 2 + 3 + ... + ( n - 1 ) ] n ------ [ ( n —1) + ( n — 2 ) + ... + 1 j +
n n

^ - [ 1 ( n - 1 ) + 2 ( n - 2 ) + 3 ( n - 3 ) + ... + ( n - 1 ) [ n - ( n - 1 ) ] ]
1

a2
c

c2
c

c
1

Xb = 4-
n 2 n 2
2

- | n - 1 + 2n - 22 + 3n - 32 + ... + ( n - 1 )n - ( n - 1 )2 J

&> = ( y
b2 f r
— 2' | n [ l + 2 + 3 + ... + ( n - 1 ) ] - 12 + 22 + 32 +. . . + ( n - 1)

438
n - 1> b2 ( n - 1 )n ( n - 1 )n( 2( n - 1) + 1)
2 2 o1
Sb=( a¿ + cr 1
2 y n2 2 6
>

2 . -2 U n - n
ib = ( - u2
b ( n 2 - 1)
2 , 6n

Así, también en la división de BC y A B :

2 2 n2 - 1
- a
/ 6n /

V - 1
Ic = ( a2+b! - c
/ v 6n /

Ahora : S = £ a + £ b + Xc

S = ( a2 + b2 + c2) ( n - 1 ) - ( a2 + b2 + c2) n2 - 1
\ 6n /

(n2 - l )
S = ( a2 + b2 + c2 )
6n

De donde :
S ( n - 1)( 5n - 1)
, es, efectivamente, un número racional
a2 + b2 + c2 6n

439
CAPITULO 15

RELACIONES METRICAS EN LA
CIRCUNFERENCIA Y POTENCIA
o TEOREMA DE CUERDAS.- En toda circunferencia, el producto de las longitudes de los
segmentos sobre cuerdas secantes, es constante.

PA . PB = PC . PD

D em ostración

- Por semejanza de triángulos. Veamos


Se trazan AC y BD.

a AD ' * CB ^
C = -----= B y Á = ------ = D

D - AACP ADBP:

PA PC
PA . PB = PC . PD
PD " PB
l.q.q.d

II) TEOREMA DE LA TANGENTE.- El cuadrado de la longitud de una tangente es igual al


producto de longitudes de una secante entera y su parte externa, trazadas desde el mismo
punto, a la circunferencia.

PT = PA . PB

441
Demostración

- Se trazan TB y AT. Entonces :

Á = — = B Í P . Luego : A ATP ~ AT BP

PT PB
ATP = TBP. A s í : — - = —
PA PT

De donde : PT = PA . PB

III) TEOREMA DE LA SECANTE.- El producto de las longitudes de una secante y su parte


externa, es constante.

Para el gráfico adjunto :

PA . PB = PC . PD

D em ostración

Se trazan AC y BD.
Luego, en el cuadrilátero ABCD:
Á = BDP y C = DBP
( Propiedad del cuadrilátero inscrito )

PA PC
- A PAC - A PDB :
PD “ PB

PA . PB = PC. PD.... I.q.q.d.

IV) RECTAS ISOGONALES.-Son aque­


llas que, partiendo del vértice, forman
ángulos congruentes con los lados de o
un ángulo. o

Se dice que son simétricas respecto a


la bisectriz del ángulo, porque forman
ángulos co n g ru e n te s con d icha
bisectriz.
*Q
Por ejemplo, OP y OQ f se llaman
isogonales respecto a los lados
OA y OB.

- Lossegm entosquetienenunextrem o
O y los o tro s sobre las rectas
isogonales, se llaman isogonales.

442
V) TEOREMA.- En todo triángulo, el produc­
to de las longitudes de segm entos
isogonales que parten de un vértice, me­
didos uno hasta el lado opuesto y otro
hasta la circunferencia circunscrita, es
igual al producto de longitudes de los
lados que concurren con ellos en el vérti­
ce mencionado.

Así, para el A ABC de la figura adjunta:

BD . BF = AB . BC

D em ostración

Se traza C F . Entonces: B

BC

A ABD AF BC

Luego : AÓB = FCB

De la sem ejanza:

BD AB
-> BD . BF = AB . BC
BC " BF

l.q.q.d.

VI) TEOREMA.- En todo triángulo, el producto de las longitudes de dos lados, es igual al
producto de las longitudes de la altura al tercer lado y el diámetro de la circunferencia
circunscrita.
En efecto; sea el A ABC, de circuncentro O y
circunradio R.En el capítulo de circunferencias
( Capítulo 9 ), se ha demostrado en un proble­
ma, que : FBC = ABH.
Es decir, BH y BF son segmentos isogonales
respecto al ángulo ABC. Luego, por el teorema
a n te rio r: AB . BC = BH . BF

AB . BC = BH . (2R)

VII) SEGUNDO TEOREMA DE LA BISECTRIZ.- En todo triángulo, el cuadrado de la longitud de


una bisectriz es igual a la diferencia de producto de longitudes de los lados que concurren
con dicha bisectriz y los segmentos medidos del pie de la bisectriz, a los vértices del lado
opuesto.
443
Así, para el A ABC, tenemos
a) Bisectriz interior: b) Bisectriz exterior:

BD = AB . BC - AD . DC BE = EA . EC - AB . BC

Demostración
Se pueden demostrar, usando el Teorema de Stewart, pero lo haremos con el Teorema de
isogonales.
a) BD y BF son isogonales respecto
al ángulo ABC. Entonces, según el
teorema para estas líneas: B
BD . BF = AB . BC

Es d e c ir: BD . (BD + DF) = AB . BC

BD + BD . DF = AB . BC

Pero, por el teorema de cuerdas :


BD . DF = AD . DC

En lo anterior : BD + AD . DC = AB . BC

BD = A B . B C - A D . DC .......... I.q.q.d

b) En este caso, los segmentos isogonales son BE y BF. Entonces: BE . BF = AB . BC

Es decir : BE . ( EF - BE ) = AB . BC

BE = BE . EF - AB . BC ... ( I ).
%

- Ahora, por el teorema de la secante :

EF . EB = EA . EC

ó: BE . EF = E A . E C . . . ( I I ) .

Reemplazando ( I I ) , en ( I ) :

BE = EA . EC - AB . BC
I.q.q.d

444
VIII) TEOREMAS DE PTOLOMEO.- En todo cuadrilátero inscrito ( ó inscriptible ) en una
circunferencia, se cumplen:

1o) El producto de las diagonales es igual a la B


suma de productos de longitudes de lados
opuestos.

AC x BD = AB x CD + BC x AD

2 o) La razón de diagonales es igual a la razón de


la suma de productos de longitudes de los
lados que concurren en los extremos de cada
diagonal. ( Teorema de Viette ).

AC _ AB x AD + BC x CD
BD AB x BC + AD x CD

D em ostración

1o) Se toma un punto P de AC ( tal que


ABP = a = DBC.

- Entonces:

AABP - ADBC:
AB AP
BD CD

.*. AB x CD = AP x B D ( I ).
AD BD
A ABD - A PBC : AD x BC = PC x B D (II).
PC BC

- Sumando ahora, miembro a miembro ( I ) y ( I I ) :

AB x CD + AD x BC = ( AP + PC ) BD

AB x CD + AD x BC = AC x BD ........... I.q.q.d.

2o) Para demostrar este teorema, usaremos la fórmula del área de un triángulo

AB x BD x AD
aabd -
4R

( Esta se demuestra en el capítulo de Areas )

- Entonces, para el cuadrilátero ABCD:

^ABCD “ ^ABC + ^AOC

445
^ABCD = ^ABD + ^BCD
Es decir:
^ABC + ^ADC = ^ABD + ^BCD
Usando la fórmula en mención

AB x BC x AC AD x CD x AC
+ ---------------------- +
4R 4R 4R 4R

- Cancelando 4R y factorizando términos comunes:

( AB x BC + AD x C D ) AC = ( AB x AD + BC x CD )BD
De donde:
AC _ AB x AD + BC x CD
BD AB x BC + AD x CD
l.q.q.d

P O TE N C IA

DEFINICION: La potencia de un punto respecto ai centro de una circunferencia es el


producto de las distancias dirigidas de dicho punto a dos puntos cuales
quiera de la circunferencia siempre y cuando los tres puntos se encuentren
en línea recta.

POR CONVENCION Se considera la potencia (+) cuando el punto es exterior a la


circunferencia y a la potencia se le antepone el signo (-) cuando el
punto es interior a la circunferencia.

1) POTENCIA DE UN PUNTO EXTERIOR : Se puede obtener de las siguientes formas:

A) El cuadrado de la tangente trazada por dicho punto a la circunferencia..

p ° ' a (0) = AD

B) El producto de una secante por su parte externa

PotA(0) = AC . AB

C) FORMULA G E N E R A L: h . ODA : AD = d2 - R2

Luego :

d Es la distancia del punto al centro “O”


446
2) POTENCIA DE UN PUNTO DE LA CIRCUNFERENCIA
De la fórmula obtenida:

P °tA(o i = d2 - R 2 Pero d = R

P o t A, o , = R2 - R2

POtA,0) = 0

3) POTENCIA DE UN PUNTO INTERIOR : Se puede obtener de las siguientes formas

A) El producto de segmentos de cuerda


trazados por el punto, anteponiéndo­
le el signo menos.

PotA = - B A . AC = -D A . AE
A(0)

B) FORMULA GENERAL: Usando la distancia del punto al centro “O”.

PotA = -M A . AN = - (R - d) (R + d) = (d - R) (d + R)

PotA(0) = d* - R2

4) POTENCIA DEL CENTRO : Es el mínimo valor de la potencia

De la fórmula, se tiene:

P °tA = d2 - R2 pero d = 0

447
I

EJE R A D IC A L
Es el lugar geométrico de todos los puntos que tienen igual potencia con relación a los centros
de dos circunferencias.

PROPIEDADES:

1) El eje radical es una recta perpendicular al segmento que une los centros de las dos
circunferencias.

* Se afirm ará que I es el eje radical *

Por definición :
PoU t = PotA
a ( E) a (F|

d,2 - R2 = d22 - r2

d ,2 - d22 = R2 - r2

Como los radios son valores constantes, entonces: d ^ - d22 = Cte.

En el triángulo EAF usamos el teorema de la proyección de la mediana :


d ^ - d , 2 = 2ÉF . MH = Cte.

En donde observamos que para cualquier punto “ A ” de HI” se forma un triángulo EAF donde
EF y MH no varían, por lo tanto “ I ” es el eje radical.

2) El eje radical de dos circunferencias secantes pasa por la cuerda común.

Pot = Pot = 0
<F) ( E)
Luego, por A y B
pasa el eje radical
Pote . = PotBici = 0
(F ) 1E)

448
El eje radical se puede construir empleando una circunferencia auxiliar

\ i / V.. Eje,
A

E.R

Se traza la circunferencia auxiliar de centro G.

El eje radical de E y G es eje,


PotA „ = PotA
a (E}
^4
a |G}
El eje radical de F y G es eje2
PotA_
A(F) = PotA
a (G)

Igualando PotA = PotA


a {E) a(F
( F)

Por lo tanto A pertenece al eje radical de las circunferencias de centro E y F, luego se traza
una perpendicular al segmento que une los centros o de lo contrario se traza otra
circunferencia y se ubica otro punto, uniéndose estos 2 puntos sé determina el eje radical.

Como casos particulares tenemos:

E.R.

CENTRO RADICAL : Es el punto de intersección de los ejes radicales de tres circunferen


das tomadas de dos en dos.

Los ejes son : Eje, -> de ( E }' F )

Eje2 -> de(F>'G )


CD
m
o.

Eje3 -»
“0

“0

P °tA ^
II
o

II
o

_>
m

-n

A(G)

A, es el Centro Radical.

449
PROBLEMAS RESUELTOS

Las diagonales de un paralelogramo ABCD, miden AC = 12 y BD = 8. La prolongación de D B ,


corta en E a la circunferencia circunscrita al triángulo ACD. Hallar BE.

A) 4 B) 3 C) 6 D) 4,5 E) 5

Solución

EB = x = ?

Como ABCD, es paralelogramo:


V

x \
BM = MD = — = - = 4.
2 2 / \ A
\
4C
AH AC 12 .
y AM = MC = ------ = — — 6.
y 2 2

Por el Teorema de cuerdas:

D M . ME = A M . MC

4( 4 + x ) = 6 . 6

Efectuando: x = 5

Rpta: ( E ).

En un cuadrado ABCD , AB = 2 J~5 ; M, biseca B C ; con centro en D, se traza el arco AC,


cortando a AM en el punto E. Hallar EM

V?
A) 1 B) 2 C) 0,5 D) E) Ning. anterior

Solución
EM = ?

M, es punto medio de BC.

Por el Teorema de la tangente

MC = MA . ME

/ 5 2= MA . ME

De donde: MA . ME = 5 ............( I ).
2 2 ^
Pero, en el A ABM, con el Teorema de Pitágoras: MA = /5 + (2/ 5 ) MA = 5

Reemplazando en ( I ) : 5ME = 5 - > ME = 1 ó EM = 1

Rpta: ( A ).
3) En la figura, AC y BD son diámetros de
la circunferencia mayor. “C”, es punto de
tangencia.

Si: BE = 1 ; AN = 2,5 y TD = 2

Hallar NC.

A) 0,5 B) 10 C) 5,5
D) 2 E) 2,5

Solución

Sea R, el radio del círculo mayor. Por el teorema de cuerdas en la menor circunferencia :
OE . OT = OC . ON

.\ ( R - 1 ) ( R - 2 ) = R ( R - 2,5 )

De d o n d e : R = 4

Luego: NC = 2 R - A N = 8 - 2 , 5

NC = 5,5

Rpta: ( C ).

4) En un paralelogramo ABCD, de diagonales AC = 10 y BD = 8, la circunferencia circunscrita


al triángulo ABC es secante a*BC y tangente a*CD en “D”. Hallar CD.

A) 3 B) 3 ^ 2 C )2 / 3 D) 3 / 3 E) 2 / 2

Solución

Se traza la figura de acuerdo


al enunciado:

Incógnita: CD

Por ser paralelogramo:


BM = MD = 4
CM = AM = 5

Teorema de la Tangente, desde HC”:

CD = CA . CE x2 = 10 . m ( 1)
Teorema de C uerdas: AM . EM = MB . MD

5 ( 5 - m ) = 16
m = ? :

Sustituyendo en (1 ) : x = 3 J~2.

Rpta: ( B ).

451
I

5) En la figura :

“O” es centro de la mayor circunferencia.


T y B son puntos de tangencia.
AB = 9 , BC = 4.

Hallar TB.

A) 5 B) 6 C) 7
D) 8 E) N.A.

Solución
* Con los trazos indicados:

* OT contiene al centro O’ de la menor


circunferencia. Luego, OT es diámetro
y OBT = 90°.

* Para la circunferencia mayor, como


OB 1 ET, entonces : EB = TB.

* Teorema de cuerdas: EB . BT = AB . BC

( TB )2 = 9 x 4 —» TB = 6

Rpta: ( B ).

6) En la figura :
O es centro del círculo , cuyo radio se
quiere conocer, sabiendo que:
AE = 4 , EB= 6

CÉO = 90°y OE = 3.

A )^ 5 B) 3 / ? C) 4

D) 5 E) 3 ^ 6

Solución

Prolongamos CE hasta cortar a la


circunferencia en D.

Como O es centro y OE ± C D ,
entonces ED = EC.

Por el teorema de cuerdas:

EC . ED = EA . EB

452
( EC )2 = 4 x 9 -» EC = 6

Finalmente, en el A CEO : r2 = ( OE )2 + ( EC )2

r2 = 32 + 62 = 3y[5

Rpta. B

7) En una circunferencia de centro “O” y diámetro AC = 12, (acuerda MF corta a AO e n “E"

Hallar EM, si AE = 1 y MC = 3AF.

A) 1,1 B) 1,8 0)2,2 D) 2,4 E) 3,2

Solución
* Angulo in te rio r:

AF + MC a* + 3 a e
E=

E = 2a°

'—

* Angulo c e n tra l: AOF = AF


AÓF = cc°

* A OEF, Isósceles : EF = EO = 5

* Teorema de cuerdas : EM . EF = AE . EC

EM x 5 = 1 x 1 1
EM = 2,2

Rpta: ( C ).

8) Hallar FM, si L, es longitud del lado del cuadrado ABCD.


M, es punto de tangencia.

A )t/5

B )f L/5

C>T^
D )f L/5
D
E) N.A.

453
Solución
FM = ?

- Se tiene:

FM = BM - BF (*)
- En el A BAM:

- • Por otro lado, con el Teorem a de la Tangente :

BT = B M . BF
\2
L
BF = — J~5
2 ) 10v

- Reemplazando lo obtenido, en ( I ) :

FM = - J 5 Js
2 10v
Rpta: ( B ).

9) En el gráfico adjunto :

B y E, son puntos de tangencia. AB = 15 y


ED = 8.

Hallar AD.

A) 23 B) 20 C) 18
D) 17 E) 19

S olución
AD = ?

Datos : AB = 1 5 , ED = 8

- Por el Teorema de la Tangente:

* Circunferencia m e n o r: AB2 = AD . AC

+ Circunferencia m a y o r: DE2 = DA . DC

♦ Sumando miembro a miembro : ÁB2 + DE2 = AD . AC + DA . DC

o mejor : ÁB2 + DE2 = AD . AC + AD . CD

454
Es decir : AB2 + DE2 = AD ( AC + CD )

AD

• • ( Propiedad)

- Reemplazando datos: 15® +8® = AD AD = 17

Rpta: ( D ).

10) Los lados de un triángulo miden 13 ; 14 y 15. El circunradio, mide :

A) 8 B) 8,42 C) 8,125 D) 8,25 E) N.A

Solución
- R = ?

- Por te o re m a :
1 5 x 1 3 = h( 2 R ) (I)

- Con el teorema de Herón :


. . 13 + 14 + 15 _
donde: p 21

_2_
h = — ^ 2 1 (2 1 - 1 5 )(2 1 - 1 3 )( 21 - 1 4 ) -> h = 12
14
65
En ( I ) : 1 5 x 1 3 = 12 ( 2R ). De donde : R = — = 8,125
O

Rpta: ( C ).

11) En la figura, O y B son centros. BC = 5

Hallar EF.

A) 1
B) 0,5

/* ■ [*
/ D )2
E) N.A.

Solución
EF = ?

- Como AB es diámetro: AÉ B = 90°

- En el arco O E C : EF = FC

455
I

EC
EF = (I)

- A ABC : AC = AB + BC

AC = 10 2 + 5 2 -> AC = 5 / 5

Se completa la semicircunferen­
\\
cia de centro B . Por el Teorema j
1

D
de la secante:

AC . AE = AD . AO

5 5 . AE = 1 5 x 5 AE = 3 / 5

- Entonces: EC = AC - AE EC = 5 -J~S - EC = 2 / 5

_ 2/5
- Finalmente, con EC, en ( I ) EF = EF = / 5

Rpta: ( C ).

12) En la figura:

A, B y C, son puntos de tangencia.


Los radios miden 12 y 2 4 . 0 , es
centro. Hallar EF.

A) 7 B) 9 C) 8
D) 6 E) 5

Solución
EF = ?

- Por propiedad : AC = 2 ^R~r

AC = 2.7 12 x 24 = 2 4 / 2

- Se prolonga AO hasta H.

En la menor circunferencia, por el


Teorema de la Tangente :

ÁC2 = AF . AH -> (24/2 Y = AF ( AF + 24 ) AF = 24

- Si Q es centro de la mayor circunferencia; al trazar OQM, aplicamos el Teorema de la


secante:
OA . OE = OM . OB ( 0 F + FA ) . ( OF + FE ) = 60 x 12

456
( 1 2 + 2 4 ) ( 12 + FE) = 720
De donde : EF = 8

Rpta: ( C ).

13) En la figura:

AC es diámetro; “E", punto de tangencia y


“B”, centro.

Demostrar, que:

BH = HE

Solución
- Se completa la circunferencia:

LLamando: BH = x , HE = y F

Se tiene : BF = BE = x + y

EN = BE

Por el Teorema de Cuerdas:

En C, : DH x HQ = FH x HE
En C2 : DH x HQ = BH x HN

De donde: FH x HE = BH x HN
( 2x + y ) y = x ( x + 2y )

x = y BH = HE
Iq.q.d.

14) En la figura, AC es diámetro

Si: EF = 3 , FG = 2

Hallar GH.

A) 5 B) 4 C) 4,5
D) 4,8 E) N.A.

Solución

Se completa la circunferencia:
Como ÁC es diámetro, ABC = 90°

457
Con los trazos indicados :
HQ = HF HQ = 2 + x
Por el Teorema de Secante:
EA . EB = EQ . EF ... (1 )

ABGH, inscriptible: entonces


por el Teorema de Secante:
tC
EA . EB = EH . EG ... ( 2 ).

De(1 ) y ( 2 ):
EQ . EF = EH . EG

(7 + 2 x ) . 3 = ( 5 + x ). 5 ; x = 4

Rpta: ( B ).

15) En la figura:

ABCD es un cuadrado, cuyo lado tiene


longitud L .
D, es centro del arco AC. CM = MD.
Hallar BE.
M
B ) ^ 3 C ) |/Í0

D o )W s E» I

Solución

BE = ?
B

Se prolongan BM y A D , hasta L
__
'" x fc,

su corte en P. / e\ 2

M
- En el A BAP :
\
AB 1 r
MD II AB y MD =
D

MD es base media relativa a AB. Por eso : DP = AD DP es radio.

- En el A BAP : BP = L2 + ( 2L )2 BP = L ^ 5

- Por el Teorem a de la Tangente : BC = BP . BE • • L2 = L / 5 . BE

De donde : BE = 7 - / ^
5

Rpta: ( A ).

458
En la fig u ra :

O es centro del semicírculo.


OECF es un cuadrado, cuyo lado se quiere
conocer, sí FM = a

A) aj~2 B) a / 3 C )a /6

E) N.A.

Solución

Incógnita ; x ( longitud del lado del cua


d ra d o ).

Prolongamos CF hasta D. Luego, como


O es centro y OF _L C D , entonces:
FD = CF = x

Al trazar OC, en el AOFC : OC = x J 2

OA = radio = OC = xj~2 . V, en el A AOF, con el Teorema de Pitágoras: AF = x

Finalmente, Teorema de cuerdas : CF . FD = FM . FA

x2 = a x ^ 3 = a/3

Rpta: ( B ).

En la fig u ra :

AC 1 BD ; AH = a , HB = b, HC = c yHD=d
Demostrar, que:

1°)

2°)

( Teoremas de Arquímedes ).

Solución

-Con los trazos indicados y el Teorema de Pitágoras en el ABM O:

BO = MO + MB

' c - a f f b + d
• » R2 = +

459
Efectuando:

4R2 = a2 + b2 + c2 + d2 - 2ac + 2bd ... ( I ).

- Por otro lado, según el Teorema de cuerdas


AH . HC = BH . HD -> ac = bd.

- Entonces, al reemplazar en ( I ):

4R2 = a2 + b2 + c2 + d2 l.q.q.d

2o) Se deja para el lector.

18) El trapecio ABCD, está inscrito en una circunferencia^ El centro es interior al cuadrilátero)
Hallar la longitud del radio, si: BC 11 AD ; BC = 4 0, AD = 48 y la altura del trapecio, mide 22

A) 30 B) 25 C) 26 D) 27 E) 28

Solución

R = ?

Se trata de un trapecio isósceles ;


ya que, s i :

BC I AD AB = CD
AB = CD.

- Con los trazos adicionales:

AOMC -> OC = OM + MC

R2 = OM + 2O2 ........( I )

- C álculo de OM:

nM CP CH + HP 22 + HP HP
OM = FC = -----= ---------------= -------------- -> OM = 11 + — (II)

- Se observa : TH = BC = 40 AT = HD = ü ^ = 4

- Por el Teorema de cuerdas :

AH . HD = CH . HP 4 4 x 4 = 22 x HP • • HP = 8

- Reemplazando esto último, en ( I I ) : OM = 15

y, en ( I ) : R2 = 152 + 202 R = 25

Rpta: ( B ).

460
19) En la figura:

A A y B son puntos de tangencia.


HF = 6 y FP = 8.

Hallar EF.

A) 5 B) 4 C) 3
D) 5,2 E) 4,8

Solución
- Datos: HF = 6 y FP = 8

- Incógnita : EF = x

- Con las variables indicadas, y :

* Teor. de la tangente:

PA = PH . PE
a2 = 1 4 ( 8 - x ) .... (I)

* Teor. de cuerdas:

mn = 6x .. (N )

* En el A APB, Teor. de S tew art: PA . FB + PB . AF = AF . FB . AB + PF2 . AB

a2n + a2m = m n ( m + n ) + 8 ( m + n )

De donde : a2 = mn + 8 . . ( I I I ).

Reemplazando ( I ) y ( I I ), en ( I I I ):

14 ( 8 - x ) = 6x + 8

Resolviendo esto último : x = 5,2 EF = 5,2

Rpta: ( D ).

20) En la figura :

UE” y “F" son puntos de tangencia.

Demostrar, que:

461
Solución
- LLamando a:
BE = b ; AB = a ; AC = c ;
CF = d ; AP = m y AQ = n.

Se demostrará que : b2 + c2 = a2 + d2
Se tiene por Teorema de Secante:

AB . AP = AC . AQ /. am = e n (1 ).

Por Teorema de Tangente:

BE2 = BA . BP -» b2 = a( a - m ) .............( 2 ).

CF2 = CA . CQ -> d2 = c( c - n ) ........... ( 3 ).

Efectuando ( 2 ) - ( 3 ) :
b2 - d2 =a( a - m ) - c( c - n ).
b2 - d2 = a2 - am - c2 - en

Pero : am = en .*. b2 - d2 = a2 - c2
b2 + c2 = a2 + d2 .......... l.q.q.d.

En un triángulo ABC, se trazan: BD , Bisectriz Interior y B M , mediana.


Si: BD = DM y AB . BC = 9. Hallar AC.

A) 6 B) 6 / 2 C) 6 / 3 D )8 E )5

Solución
Se traza el gráfico con el enunciado:

Incógnita: AC = x

Por el 2o Teorema de la Bisectriz : B

BD2 = AB . BC - AD . DC; es decir:

'-(i-X H

Rpta: ( A ).

En un triángulo ABC la longitud de la bisectriz interna de| Á , es media proporcional entre los
segmentos que ella determina en el lado BC. Demostrar que, entre las longitudes de los lados
del triángulo, se cumple :

b + c = a^ 2
Solución

- Dato : AD = BD . DC

- Pero, según el segundo Teorema de la

bisectriz : AD = be - BD . DC

- Entonces: BD . DC = be - BD . DC

« • 2BD . DC = b e (I)

Por el primer teorema de la b ise ctriz:


BD c
DC ~ b

Usando propiedad de proporciones :


BD _ c BD ac
BD =
BD + DC b + c b + c b + c

ab
- En forma análoga : DC =
b + c

ac ab
- Reemplazando lo hallado, en ( I ) : = be
(b + c) ’ (b + c)

2a
- Cancelando b e : = 1
( b + c )'

- De donde, efectivamente : a^¡~2 = b + c ó b + c = a - /2 .......... I.q.q.d.

23) Hallar la longitud de la diagonal de un trapecio isósceles cuyas bases miden 14 y 50cm.,
y lados no paralelos 30cm. cada uno.

A) 40cm B) 36cm C) 32cm D) 44cm E) 60cm

Solución
Todo trapecio isósceles, es ins
criptible en una circunferencia:
- - 1¿ —
Por el Teorema de Ptolomeo:

AC . BD = AB . CD + BC . AD

x2 = 3 0 ( 3 0 ) + 1 4 ( 5 0 )

x2 = 1600

• 9 x = 40

Rpta: ( A ).
463
En un triángulo ABC, se traza la bisectriz interior BD. Sobre BC se ubica el punto “E”, de
modo que BÁC = BDE.
Sea EF 11 ÁC , ( “F” en AB ).
S i : BE. BF = 16 y DE = 3 . Hallar BD.

A) 5 B) 6 C) 7 D) 4 E) N.A.

Solución
Se traza la figura con los datos:

Incógnita: BD
EF 11 ÁB BFE = Á

De! Dato: BDE = Á .-. BFE = BDE ;

Esto indica que FBED es un cuadriláte­


ro inscriptible.

Por el Teorema de Ptolomeo:


BD . EF = 3BF + 3BE

Por el Teorema de Vietta :


BD BE . BF + 3 x 3
EF ~ 3BF + 3BE

Multiplicando m . a . m ; y simplificando:

BD2 = B E . B F + 9 -» BD2 = 16 + 9 BD = 5

Rpta: (A).

Sobre ei arco BC, de la circunferencia circunscrita a un A ABC, equilátero se toma el punto


"Q". Demostrar que :

AQ = BQ + QC

Solución

En el cuadrilátero ABQC, Teorema de Ptolomeo

AQ . BC = AC . BQ + AB . QC

AQ . L = L . BQ + L . QC ->

( Teorema de Chadú ).
I.q.q.d
En la fig u ra :

el A ABC es equilátero. P, un punto cualquiera


de! arco BC.
B
L , es longitud de AC.

Hallar : PA + PB + PC

A) 2L2 B) 3L2 C) 4L2


D) 5L2 E) 6L2

Solución

x = PA + PB + PC = ?

Por el Teorema de Chadú, demos­


trado anteriormente : PA = PB + PC

Entonces:

x = ( PB + PC )2 + PB + PC

x = 2( PB + PC + P B . PC ) . . ( ! )

En el cuadrilátero ABPC, por el se­


gundo Teorema de Ptolomeo:

AP AB . AC + PB . PC
BC ” AB . PB + AC . PC

AP L . L + PB . PC AP L2 + PB . PC
L “ L . PB + L . PC ^ L “ L( PB + P C )

De d o n d e : AP( PB + PC ) = L2 + PB . PC

Pero : AP = PB + PC. Entonces : ( PB + PC )2 = L2 + PB . PC

Efectuando : PB + PC + PB . PC = L2

Reemplazando esto último, en ( I ) : x = 2L2 —»

( PROPIEDAD).

En un A ABC, B = 60°, I es incentro.


Si O es el circuncentro del A AIC y AB + BC = 12.
Hallar OB.

A) 6 B) 12 C) 4 D )4 /3 E) 6 / 3

465
Solución
Dato : AB + BC = 12

Incógnita: OB.

En el A ABC, por ser “ I ” incentro B 60

ABC
A IC = 909 +

A IC = 120

Luego : AEC = 2 A IC

AEC = 240°

AOC = AIC = 120°

Entonces, en la circunferencia circunscrita al A AIC, si R es circunradio :

AC = L j -» AC = R / 3 .

Observamos que el cuadrilátero ABCO es inscriptible porque ABC + AÓC = 180


Entonces, con el Teorema de Ptolomeo :
0

OB x AC = OC x AB + OA x BC -» OB x R ^ 3 = R x AB + R x BC

Cancelando “ R ” : OB x = AB + BC OB x / 3 = 12

Rpta: ( D ).

En la figura, E y D son puntos


de tangencia.

Demostrar, q u e :

Solución

Por el Teorema de la tangente, en la mayor circunferencia : CE = CA.CB (1 ¡

Según el g rá fic o : CE = DC ,
CA = DA - DC
CB = DB - DC

Sustituyendo estas tres equivalencias, en (1 )

DC = ( DA - DC ) ( DB - DC )
Efectuando : DC2 = DA . DB - DC(DA + DB) + DC2

Es d e c ir: DC ( DA + DB ) = DA . DB

Arreglando en forma conveniente:


DA + DB 1 DA DB 1
o:
DA . DB ’ DC D A . DB D A . DB DC

1 f 1
De donde : + I.q.q.d.
DB DA DC

29) En la figura :

PotE = 2 PotE ^
OI L{ Q )

H a lla r: —
H

1 1

A >2
B) 1 C)
V2
1
E) N.A.

Solución

Por definición, en el dato : Pote ^ = 2 Pote ^


S O ) fc( Q )

2 2
EA = 2EB

EA = y¡2 EB (I).

- Si se trazan OA y Q B : A Q B E ~ AOAE

r EB
Entonces : — = —
R EA

r EB 1
Con lo de ( I ) : p “ ^ eb
R y[2

Rpta: ( C ).

30) En la figura f AB = 15 , BC = 14 y AC = 13
Hallar : PotB
'i O)

A) 8 B) 64 C) -8
D) -64 E) 21
f

467
Solución

- Por definición : PotR = BE .......... ( I ).


°{ o > v 7

- Pero, si p, es e! semiperímetro del A ABC, sabemos por propiedad, que : BE = p - AC

BE = i - - ■14 + -15 - 13 BE = 8

- Reemplazando en ( I )
Po,B,o, = 64

Rpta: ( B ).

31) En la figura:
B
AB = 5 , BC = 8 y BD = 6.

Hallar la potencia del punto D


con respecto al centro O de la
circunferencia.

A) 4 B) 2 C) 3
D) -3 E) -4

Solución

Pot D
(Ol

- Por definición : PotDj = - AD . D C ( I ).

- En el A ABC, por el segundo teorema de la bisectriz

BD = AB . BC - AD . DC
62 = 5 x 8 - AD . DC

A D . DC = 4

Reemplazando, en ( I ) : ^ otD(0) = “4

Rpta: (E).

32) En el siguiente gráfico, Hallar:


PotA + Potr
A( o ) o)

A) 4R (R + r) B) 4R (R - r)
C) 2R (R + r) D) 2R (R - r)
E) 4Rr

468
Solución
Se tienen :

PotA = AB y
A{0) y

Pote = CB
°10)

Luego:

PotA + Potc = AB + CB (I).


M|0> ° I 0)

Siendo; en el A MBO : MB2 = MO2 - OB2 -» MB = J ( R + r f - r2

MB = ^/ R2 + 2Rr

Luego : AB = AM + MB —> AB = R + V r2 + 2R r

CB = MB - MC -> CB = V r2 + 2 R r - R

Entonces, reemplazando en ( I ) :
2 /
PotA + Potc = ( R + J R2 + 2 R r + •J r 2 + 2 R r - R '
A(0| o» v * > \

P°* a ^
A<0) L'(0) = 4R (v R + r );
Rpta: ( A ).

En la figura :

B O e f son el Circuncentro e Incentro del A ABC

Demostrar, que:

O I = R( R - 2 r )

O I : Distancia del circuncentro al Incentro


Solución

Se prolonga Bl hasta F.

Sean; Bl = m , IF = n

Se tiene:

Pot, _ = OI - R2
(O)
Poti = -mn
(0)

469
En el capítulo de circunferencias se demuestra que : AF = IF.
( El lector puede comprobarlo al analizar los ángulos del A AIF ).

Entonces : AF = n

Se traza el diámetro FM AMF = ABF = a

AMAF - A B Q I : — = — -> - = — h> mn = 2 R r (2).


IQ Bl r m

—2
Reemplazando ( 2 ) en ( 1 ) : OI = R2 - Rr —> OI2 = R ( R - 2 r ) .
V

r
v

POLIGONOS REGULARES Y LONGITUD


DE LA CIRCUNFERENCIA
POLIGONOS REGULARES.- Son aquellos polígonos convexos que tienen sus lados y
ángulos, respectivamente congruentes.
Todo polígono regular puede ser inscrito y circunscrito a dos circunferencias concéntricas.
O: Centro de la circunferencia
R: Circunradio
r— n — "I ^
Ln: Longitud del lado , para el
polígono regular de u n " lados.
an: Longitud del Apotema.( ó apn ).
A COB : Elemento fundamental
del polígono.
a n: Medida del ángulo central o
del arco que subtiende cada la­
do del polígono.

Cálculo de la longitud del Lado

En el A COB con la Ley de Cosenos : Ln = R ^ 2 ( 1 - Cos ccn )

Cálculo del Apotema

Casos Particulares

1) Triángulo Equilátero:

L3 = R / 3 ;
R

471
2) Cuadrado: 3) Hexágono Regular

L4 = r /2 ; l6 = R;

_ R /2 R /3
B =
a
ae = 4 -

4) O ctógono Regular:

~ A 45"
R

L8 - R yj 2 - ,f2 ; a8 - —- yj 2 + *J~2

5) Dodecágono Regular:

L-J2 — R-J 2 — ; a 12 - — ^~2 + yfs

6) Decágono Regular:

^-10 ~ * 2 ( ~ ’ S l ° ” * 4 * V ^ +

472
V

7) Pentágono Regular:

Ls =■§-V10-V20
R
a5 = T ( / 5 +1

Propiedad: Los lados del pentágono,


exágono y decágono, regulares for­
man un triángulo rectángulo; así:

En resum en:

Polígono an Ln an
Regular

R
Triángulo 120° R /3
2

R /2
Cuadrado 90° R /2
2

R ^3
Hexágono 60° R
2

Octógono 45°

Dodecágono 30° Ry¡2-


I
i
I
(X |c\l
Ü

Decágono 36° S j l O * J20


I

4 *

Pentágono 72° f V 10 ">/2Ó

473
DIVISION DE UN SEGMENTO EN MEDIA Y EXTREMA RAZON

Dividir un segmento en media y extrema razón, quiere decir dividir la longitud del segmento
en dos partes diferentes, siendo la mayor media proporcional entre la parte menor y el total.
A la porción mayor, se le llama “sección áurea” del segmento total.
AP PB
A s, : 7 b ' w
( definición) I* x ~
B

- Si AB = a, entonces, para hallar x : x2 = a ( a - x )

x2 + ax - a2 = 0

( fórmula de la sección áurea de


- Resolviendo : un segmento de longitud a ).

\
f/5 - 1
♦ A la cantidad : , se la conoce como “ número á u re o " ( 0,61803 ).
V /

Propiedad.- El lado del decágono regular tiene longitud igual a la sección áurea del radio
de la circunferencia circunscrita.

En efecto. Sea ABCD ... , el decágono


regular inscrito en la circunferencia de
centro O y radio R. Demostremos que:
\
r ^ 5 -1
L,o = R
v j

Para ello, extraemos el A AOB:


Tracemos la bisectriz BZ del ZABO
Entonces, se o b se rva :
A ZBA ->■ isósceles : BZ = AB
BZ = L 10

AOZB isósceles: OZ = BZ

OZ = L10 ZA = OA - OZ

ZA = R - L 10

En el A ABO, por el Primer Teorema de


AB ZA
la Bisectriz :
OB OZ

474

küL R - L 10 ( L 10 f = R ( R - L,n)
= 10
R 10

De donde : L,o = R I.q.q.d


/

¿Qué es núm ero áureo?

El número áureo (O ) constituye uno de los valores más importantes y significativos en el


campo de las Matemáticas ya que, aparte de representar un determinado valor, define con
curiosas propiedades cualquier proporción. Es decir, de entre todas las particiones asimétricas
que se pueden hacer en un segmento lineal, es el número O el que representa la operación
lógicamente más sencilla, la que dé la impresión de armonía lineal y de equilibrio en una
desigualdad más satisfactoria, entre el resto de las combinaciones posibles. En el gráfico
inferior se exponen dos construcciones en las que se establece una proporción entre dos
segmentos, el AC y el CB, dentro de uno mayor (AB), de tal modo que la parte mayor AC es
a la menor CB como lo que la suma de las dos es a la mayor

El valor del número es siempre el de la fracción decimal 0,61803398875... Sin embargo,


se trata de un número indefinido de cifras, puesto que <J> es inconmesurable. Leonardo da
Vinci fue el primero en denominar sección áurea a esta proporción. Aunque tenga un valor
indefinido, es perfectamente representable por una longitud exacta; ya que, al pertenecer a
la categoría de los números algebraicos de segundo grado, puede representarse
euclidianamente, es decir, por medio de la regla y el compás. No ocurre lo mismo con otros
números trascendentes, como sería el caso de k (p/).
Por todas estas características el número áureo se considera desde la Antigüedad impres­
cindible en el estudio teórico de las formas, contornos y volúmenes, en el examen matemático
de la estructura en general, y se aplica en las creaciones del arte decorativo, plástico y
monumental. Aparece además constantemente en el crecimiento de los seres vivos. Todas
estas connotaciones le hicieron erigirse en número mágico, utilizado por científicos (Kepler),
artistas ( Fidias, D urero) y ocultistas. En el siglo XVI, el veneciano Fra Luca Paccioli di Borge
lo adornó con la denominación de Divina Proporción.

Los siguientes gráficos muestran dos formas de obtener las otras longitudes, conociendo
una de ellas.

/ /// 1■
/ / 1
C / 1a
\ Y/ 1
/\ *i AC: Sección
Aurea de AB.
Q a/2 P b

* Dado a ; cómo hallar b y c. * Dado c ; cómo hallar a y b.

ARTICULO PUBLICADO EN LA REVISTA


MUY INTERESANTE AÑO 1 N° 6 PAG. 32

475
¿Cómo Eratóstenes m idió la longitud de la Circunferencia de la tierra?
En el siglo III a. de J.C., un matemático griego, nacido en Cirene, Eratóstenes, midió la
longitud de la circunferencia de la tierra y su resultado tuvo un error de solamente 1 ó 2%.

& rayos
del sol

Alejandría
Syene 800Km

Se le informó que todos los años, al mediodía del 21 de junio, el sol se reflejaba en la
superficie del agua de un pozo en Syene. En la misma fecha y a la misma hora, los rayos
solares proyectaban en un obelisco distante 800km. al norte, en la ciudad de Alejandría, una
sombra, de manera que los rayos formaban con el obelisco un ángulo de medida:
a = 7°12' = 7,2°

Tierra

a
............ ..
Centro

Eratóstenes concluyó que esto era posible sólo si la tierra es redonda, ya que los rayos
solares son paralelos.

Como el arco SA, ( distancia entre Syene y Alejandría ) tiene longitud de 800km; con una
Regla de Tres:

7,2° -------------------» 800km.

360

De donde:

L = 40 OOOkm.

fue la longitud de la circunferencia terrestre hallada por Eratóstenes.

476
PROBLEMAS RESUELTOS

El radio de la circunferencia inscrita en un hexágono regular, es 3.


Hallar la longitud del radio de la circunferencia circunscrita al mismo polígono

A) y[3 B) 2 / 3 C)3/3 D) 4 E) 2
Solución
Según el gráfico dado por
enunciado OH = 3
En el A AHO ( 30° y 60°):
OA
/ 3 = OH

- J~3 = 3
2 v
De donde: R = 2 >/~3
Rpta: ( B ).

Hallar el perímetro de! triángulo que se forma al unir los puntos medios de tres lados no
consecutivos de un hexágono regular de circunradio “R”.

A) 2R b) | r c) | r d) | r E)¡R

Solución

- Como: ABCD = 180 AD, es diámetro.

En el trapecio ABCD : x = AD + BC
2 D

Donde : BC = L6 = R

x= 2R + R = 3 r
2 2

9R
Como el triángulo es equilátero, su perímetro : 3x =

Rpta: ( E ).

Sobre el arco AB, de la circunferencia circunscrita a un hexágono regular ABCDEF, de lado


“a”, se toma un punto “ P” :
Hallar:
— 2 —2 — 2 — 2 —2 —2
y = PA + PB + PC + PD + PE + PF

A) 6a2 B) 9a2 C) 12a2 D) 15a2 E) 18a2

477
Solución
Se observan, en los triángulos rectángulos:

2 2 2 1
PA + PD = AD

PB2 + PE2 = BE2 (+)

A
y = AD2 + BE2 + FC2

I = 12a2

Rpta: ( C ).

Hallar el perímetro del polígono que se obtiene al unir, en forma consecutiva, los puntos
medios de los lados de un octógono regular, de circunradio “R”.

A)R/2 Q) 4 R y [ 2 C) 3 yj~2 D) E) N.A.


2

Solución
Se obtiene otro octógono regular de lado x.

Como: ABC = 90° ; AC = L4

En el triángulo ABC, ( base media ):

X = AC = U = R / 2
2 2 2

Perímetro = 8x = 4R <J~2

Rpta: ( B ).

En un triángulo ABC, equilátero, inscrito en una circunferencia de radio “R" , “M” y wN"son

puntos medios de AB y AC, respectivamente.

Hallar MN.

A )f/7 B) R ^/T C)2R/7


B

D) f / 1 7 E) 4

Solución

Se construye el gráfico:
AM = 60 AM = L6 = R

MB
MAB = = 30’ MAN = 90°

En el A MAN : MN = ( AM )2 + ( AN )2

R /7
MN = (R) + MN =
/
Rpta: ( A ).

6) En un octógono regular ABCDEFGH, de circunradio “ R”, hallar la distancia del vértice A, al


punto medio del lado D E .

A) 5 / 1 0 “ + 2 / 3 B )- ^ 1 0 + 3 / 2 C) 10 + / 3

D) R ^ 1 0 + 3 / 2 E) N.A.

Solución
- Incógnita: AM

OM = a8 = — ^ 2 +

En el triángulo ADE , por base media:

AD = 2( OM ) /. AD = R >/ 2 + / 2

En el triángulo rectángulo ADM :

( AM )2 = ( AD )2 + ( DM )2
\
( AM )2 = R-J 2 + / 2 +

AM = - ^ 1 0 + 3 / 2

Rpta: ( B ).

7) En una circunferencia de centro “Q” y radio ^J~2 metros, se trazan los diámetros AB y DE,
perpendiculares entre sí. La recta que une el punto “A” con el punto medio “O” de B C , lado
del hexágono regular inscrito, corta a DE en “N”. Indicar cuanto mide QN.

J6
A ) ^ m B) —— m q ^ m D )^ m E)
7 7

479
Solución
- Sea el gráfico, con los datos:

Como CB = L6 = J~2

OB = OC = V 2

60“

Se traza OH 1 QB

En el A OHB ( 30° y 60°)

HB = <
^ r = ^ T - y OH = ( H B ) / 3 = - ^

AA QN - AAHO

/2 _ /6
x =
/ 2 + | / 2
4 4

Rpta: ( D ).

La hipotenusa de un triángulo rectángulo ABC, mide AC = 2. Se traza la bisectriz interior BD


Si BD = AB. Hallar AB.

A) 2 - / 2 B )2-/3 0 /2 - 1 D )/2 -/2 E )^ //2 - 1

Solución
AB = x = ?
B
B, es el centro de una circunferencia
de radio AB = BD, donde:

AD = Le ya q u e ABD = 45

Trazamos BH 1 AC

« •

Por relaciones métricas:

AB2 = ( AC ) ( AH )

\
* = ( 2 ) ( f / 2 - / 2 x = / 2 - /2

Rpta: ( D ).
En ur. cuadrado ABCD, de lado ua”, tomando como centros los vértices “A” y UD" y con radio

“a" se trazan los arcos BD y AC, secantes en “F”.


-s

Hallar la distancia entre los puntos medios de AF y FD.

A )a (/3 -l) B)a/3 C )a (^3 + l) D )a( J~3 + 2 ) E)a(V3-2)

Solución
Sea el gráfico:

Incógnita: MN

En el triángulo AFD -» Equilátero


Luego:

AF = FD = 60° ( ángulo c e n tra l).

Angulo Inscrito:
OA°
AFM = NFD = —— = 15°
2

Entonces, por arco que subtiende :

FM = FN = L12 = a ^ 2 - / 3

En el A MFN, isósceles :

MN = MFy[2 MN = a ^ 4 - 2 / 3 ó, MN = a ( / 3 - 1)

Rpta: ( A ).

En una circunferencia de radio R, hallar la longitud de la cuerda que subtiene un arco de


144°.

A )jV l°+ ^ B )5-/l°W 2 0 C )7 (^5 +1) D )y (/2 + l) E) N.A.

Solución

Sea AB la cuerda, tal que m AB = 144°.

Si se traza el diámetro AC: m BC = 36°


-*• OM = a10.

En el A ABC:

AB = 2( a10) AB = 2^ 5 - ^ 1 0 + ^ 2 0

_> AB = + /2 Ó

Rpta: ( B ).

481
11) En un A ABC , Á = 18°, C = 27° y m AC = ( V"5 + 1). Hallar m BC.

A) 1 B)/2 C) 2 D) 0,5 E)

Solución

m BC = ?
Dibujamos la circunferencia
circunscrita al A ABC.
R circunradio.

Como Á = 18° BC = 36 <*♦»)

C = 27° AB = 54'

Luego: ABC = 90

Entonces, AC es un lado del cua­


drado inscrito en la circunferencia:

/5 + 1
ÁC = L, /5 + 1 = R ^2 R =
&
También, BC = L10 ya q u e BC = 36

R
Para el decágono re g u la r: L10 = — ( ^J~5 - 1)

BC = S ( / Í - 1 ) BC = f 2
60
Rpta: ( B ).

12) Hallar la longitud del lado de un pentágono regular, sabiendo que una diagonal mide “a”

A) 2a B )a (/5 -l) C ) J ( / 5 + 1)

E) N.A.

Solución B
Es fácil probar que en el pentágono regu­
lar, todas las diagonales son congruen­
tes. Como “B”, equidista de UE” y “D "; “B”,
es centro de una circunferencia de radio
“a”, pasando por “E" y “D” . En dicha
circunferencia:

482
ED = L10= | ( / 5 - l )

( Lado del pentágono regular en función de la d ia g o n a l).

Rpta: ( D ).

13) En un triángulo ABC, acutángulo, B = 75° y m AC = 12, se trazan las alturas AQ y CH


Hallar HQ.

A) 6 ^ 3 " B) 6 ^ - / 2 C) 6 ^ 2 - y[2 0 ) 6 ^ 2 - y[3 E) N.A

Solución
AHQC es un cuadrilátero inscriptible

BAQ = 15 HQ = 30

Luego : HQ = L12 = R ^ 2 - / 3

Pero : R = — = — = 6
2 2

HQ = 6 ^ - / 3

Rpta: ( D ).

14) El cuadrilátero ABCD, se inscribe en una circunferencia de radio

S i: AB = ^ 1 2 - 6 / 3 ; BC = ^ /12 - 6 ^ 2 y AD = 3 J 2 .

Hallar la medida del menor ángulo que forman AB y DC.

A) 35,7° B) 15,7 C) 17,5 D) 27,5 E) 37,5

Solución
- Sea el gráfico:

Como : AD = 3 ^ 2 = J T q - J 3.

ÁD = R / 3 ÁD = L3

Entonces: AD = 120 -í'-

También : BC = ^ 1 2 - 6 ^ 2

BC = / 6 . J 2 - f 2 BC = R ^ 2 - / 2

483
Entonces: BC = L8 y BC = 45

AD-BC 120° - 45°


Luego : x =

x = 37,5°

Rpta: (E).

15) Hallar la longitud del lado de un dodecágono regular, sabiendo que el radio de la circunfe
rencia inscrita en él, mide 0,5cm.
Solución
Dato : r = 0,5

Incógnita : L12

Sabemos que, si UR” es el Circunradio:

l 12 = r

ap

Luego:

L,2 ...2 V 2 - V 3
a p i2 yj2 + J~3

L1Z = 2( 2 - / 3 ) • ap12

Siendo : aPi2 = r

Entonces : L12 = 2( 2 - ^ 3 ) . r

Con el dato: L12 = 2( 2 - J ~3 ) . 0,5 L12 = ( 2 - ^ 3 ) cm

16) En una circunferencia de radio ^ 2 + ^ 3 , se inscribe el triángulo isósceles ABC, tal que
B = 120°.
Luego, se dibuja, interiormente, el cuadrado BCPQ. Hallar AQ.

A) 1 B) 2 C) 3 D) 4 E )5

Solución
Para la circunferencia de radio UR”:

AB = L6 = R

484
Para la circunferencia de centro “B”
y radio AB = BQ:

B
ÁQ = L12 = A B ^ 2 -

••• AQ = R ^ 2 - / 3

- J(2 + ^ ) ( 2 - / 3 )

AQ = 1

Rpta: ( A ).

Dado un nonágono regular ABC..., AB + BD = 4. Hallar BG.

A) 1 B) 2 C) 3 D) 4 E )5

Solución
Del gráfico: c
ABD = DEG = GHA

AD = DG = GA = n

En el cuadrilátero ABDG. Teorema


de Ptolomeo:
BG . AD = AB . DG + BD . AG
BG . n = AB . n + BD . n

De donde :
BG = AB + BD

BG = 4

Rpta: ( D ).

Hallar la relación entre los radios de las circunferencias, inscrita y circunscrita a un octógono
regular.
Solución
Sea el gráfico:
Sabemos, que:
OM = ap8
i 1

r _ J2 +J2
R 2

4 85
19) Hallar la relación entre las longitudes de los lados de dos octógonos regulares, inscrito y
circunscrito, a la misma circunferencia.

Solución
“Los polígonos regulares de igual
número de lados, son semejantes”

Luego, como se pide:

MN
; de la semejanza, podemos
AB
escribir:

razón de razón de
lados apotemas

MN OH
AB ” ON

Así: _ 2 MN = + ^2
• •

AB R AB 2

20) Hallar el perímetro de un heptágono regular ABCDEFG, s i : -r— + = —


At Au o

A) 6 B) 5 C )4 D) 3 E) 12

Solución

S i: AC = m y AE = n

1 1
El Dato : — + —
n m
*\

Como : ABC = CDE ; Entonces

CE = AC CE = m

ABCD = AGFE entonces:

AD = AE AD = n

En ACDE, Teorema de Ptolomeo:

AE . CD + AC . DE = AD . CE

nx + mx = nm

n+ m _ 1
x( n + m ) = nm
nm x

4H 6
1 1 1
De donde : — + — = —
m n x
( Propiedad, para todo heptágono re g u la r)

Comparando con el dato : x = 6

Perímetro = 42

Rpta: (A).

ABCDEF, es un hexágono regular circunscrito a una circunferencia de radio V tangente a


BC en “Q". AQ, corta a la circunferencia en MM”. Hallar AM.

Solución
AM = ?
Según el gráfico, “T” es punto de
tangencia en AB.
Por el Teorema de la Tangente:

AT2 = AQ . AM ........ ( I ). D

Trazamos AS, perpendicular a BC.

A E W O : EW = 5 E -» E W = (
/3 J3
n —— ^

ED = 2EW = —r J~3, ( lado del hexágono )


3 y
AB
Entonces : AB = y AT =

AT = (II).

Además en ASB : AS = OQ AS = r
CM

SB = —^~3 ; Luego : SQ = SB + BQ = - H +L SQ = •
CO

2 3 3

En A ASQ, Teorema de Pitágoras : AQ = AS + SQ

AQ2 = r2 + ' f - V 5 AQ = - Í 2 1 ( I I I ).

Finalmente, al reemplazar ( I I ) y ( I I I ) en ( I ) :

AM . , jh
21

487
22) El triángulo equilátero ABC, está circunscrito a una circunferencia de radio Mr”, tangente a

AB en M y a BC en N. Si “Q” es el punto medio de MN, h a lla r: BQ2 + AQ2.

Solución
“O” baricentro del triángulo ABC B
BO = 2( OH ).

BO = 2r -> BQ = r / I

w
/
/
/
/ / r N
A //
A AHO ( 30° y 60° ) : AH = r / 3 /
/t 1 \ y* * 0
/A
/
/ / V ' ' 60" X
/

V V
A AHQ : AQ2 = AH2 + HQ2

AQ = ( r / 3 )2 + ( 2 r ) AQ = 7r2

Luego:

BQ + AQ = r2 + 7r2 = 8r2

23) En un cuadrado ABCD, se inscribe una circunferencia de radio “a”.


Si "P”, es cualquier punto de la circunferencia.
H a lla r:

PA + PB + PC + PD .
B
Solución

Triángulo APC. teorema de la mediana

AC
PA +PC = 2PO +

(2a/2)
PA + PC = 2a2 +

PA + PC = 6 a2

2 n
Análogamente : PB + PD = 6a2

Luego: PA + PB + PC +PD =l2a2

24) Hallar el radio de la circunferencia inscrita en un pentágono regular, inscrito en una


circunferencia de radio 4cm.

488
Solución
Se tiene:

OM = aP¡
í i

r =

Siendo R = 4

r = ( , [ 5 + 1)cm.

25) Hallar el perímetro del triángulo formado al unir tres vértices no consecutivos de un
hexágono regular circunscrito a una circunferencia de radio V .

Solución

Sea el hexágono ABCDEF


y FBD el triángulo.

Sabemos que ABC = 120°


y C = 120°. D

Siendo el triángulo BCD, isósceles:


CBD = 30°

Luego: ABD = 90°

MBDN : Rectángulo BD = MN = 2r

El perímetro del triángulo FBD = 6r

26) ABCD..., es un octógono regular circunscrito a una circunferencia de radio 1cm

Hallar AD.

Solución

Del gráfico:

AD 1 DE

AD = PQ = Diámetro

AD = 2

27) Conociendo el lado del polígono regular de n lados ( wLn” ) y el radio de la circunferencia
circunscrita UR”. Hallar una fórmula que permita calcular el lado de otro polígono regular de
2n lados ( “L ^ ” ).

489
Solución
Se construye el gráfico.

Considerando AB = Ln:
•2n

AM = MB AM = L,n B
L_i
N
En el triángulo AOM, por Teorema •2 n

de Euclides:
\ s
\
N
V
AM2 = OA2+ OM2- 2 x OM x ON ... (1 ) \
s

o
Donde: OA = OM = R

y; ON = an = 4 R 2 - L2

Reemplazando en ( 1 ) :

L |n = R2 + R2 - 2R x ^ y ¡ 4 R 2 - L2

de donde : L2 n = ( fó rm u la ).

Nota.- Por ejemplo, si se nos pidiese calcular el lado del polígono regular, de 24 lados

L24 = L2 x 12 = 2 fí2 - R -J~4R2 - L22

24 = ^2R2 -R ^ 4 R 2 -^ R iJ T -j3

L24 = R ^ 2 — -J 2 + -J~3

- Análogamente:

L 16 " L 2 x 0
= - /2 R 2 ^ R ^ p T ^

*-16 = f í j 2 - j 2 + y[2
- También:

L 32 - L 2 x 16

*-64 - *-2 x 32 = R \l 2 - \| 2 + -J 2 + -J~2 + J~2

490
Se observa que, para potencias de 2, se puede encontrar fácilmente la longitud del lado
del polígono regular. Para un polígono regular de:

4 lados -» L4 = L 22 = R y¡~2

8 lados L0 = L = R- J 2 - -¿2

16 lados -» L16 = L

32 lados = = R ^ 2 - ( 7 l^ 2 + /2
32 ~~ *"25

64 lados *“ S4 ~ L26 = R i/ 2 - 2 + ^ 2 + y¡2 + / 2

2k lados —»

Por ejemplo, para L12e:

L i28 = L2? = R-J 2 - J 2 + J 2 + )/ 2 + a/ 2 + / 2

El Núm ero k .- El número n se define como la razón de la longitud de una circunferencia,


a su diámetro.

Q
Si C, es la longitud de cualquier circunferencia, de radio R : — • = n , ( constante ).
2R

Cálculo de t i .- Si se inscribe un polígono regular de n = 2k lados, en una circunferencia de


radio R; haciendo “k” cada vez mayor, se tendrá un polígono de mayor número de lados y
su perímetro se aproximará cada vez más a la longitud de la circunferencia ( C ).
El perímetro Pn, de éste polígono:

Pn = nLn , donde n = 2k lados ,

Entonces, s i : n °o Pn C

491
P e ro : Ln = L2k = FlJ 2 --J 2 + -J 2 + ... +

( k -1 ) radicales.

/. Pn = nLn = 2K . R ^ 2 - ^ 2 + ^ 2 + ... + / 2

( k - 1 ) radicales
Luego:
_ 1n
71 =
2R 2R

Es d e c ir:

n = 2( k - 1 ) ^ 2 - ^ 2 + ^ 2 + ... + f 2

( k - 1 ) radicales.

o mejor aun :

71 = 2 k + 1 J 2 - J 2 + J 2 + ... +

k radicales

Los antiguos egipcios calcularon el valor de n , hasta , para k = 14 ; dando


n = 2 14 + 1 = 2 15 = 32 768 lados.

obteniendo entonces : 71 =3,1415926535140

L o n g itu d de una circu n fe re n cia .- Como : ^ = 71

492
Es d e cir: " La longitud de toda circunferencia es proporcional a la longitud de su radio.H

Usualmente , la longitud C , de una circunferencia , se deja indicada en función de n.


Por ejemplo para una circunferencia de radio 5 cm : C = 10 n cm.

Si D , es longitud del diámetro de una circunferencia :

Longitud de un arco.- La longitud de un arco, cuyo ángulo central tiene medida a® , se


evalúa:
Longitud del arco AB.

PROBLEMAS

En la fig u ra :

O es centro de los arcos AD y BC.


AB y DC, son diámetros. Hallar el perímetro de la
figura sombreada.

29 23
A ) — n cm B) — n cm C) 14 ti cm.

D) 15ti cm. E) N.A.

Solución

X = L © + Lnr
BC + LÍ
ABd + L DC
(D -

/ 30
Donde : L^g = 2 k x 12 cm x - 271 cm
V 360' /

' 30 7 K
Lb£ = 271 x 21 Cm x cm
^ 360 /

AB y DC, forman una circunferencia de diámetro 9cm

493
l Í b + L dc = 2 n x ? ~ p - = 9 K cm

Reemplazando, en ( I ) :
7 71
x = 2 ncm + cm + 9 Tccm

29
x = — 7i cm

Rpta: ( A ).

Un tramo de vía férrea curvilínea está formado por dos arcos sucesivos. El primer arco
corresponde a un ángulo central de 20° con un radio de 2500 pies, y el segundo corresponde
a un ángulo central de 25° con un radio de 3000 pies. Encontrar la longitud total de los dos
arcos.

A ) 2180 B ) 2182 C ) 2184 D ) 2186 E ) 2188

Solución
Los arcos pueden ser del mismo
sentido o de sentido contrario.
/ 25° ' y 2500
20 \
L1 = 2 k ( 3000 )
\ 360° >

L, = 2 k ( 2 5 0 0 ) 20
360° /
s *
\ i
i i
L L = 25000 * = 2181,66 v -r
'* 25'
2 36

L, + Lg = 2182

Rpta: ( B ).

Un arco con radio de 8m. mide 3m. ¿Qué diferencia existe entre la longitud de este arco y
la de otro, del mismo valor angular, de 6m. de radio?

A) 4m. B) 2,5m. C) 1,2m. D) 0,75m. E) 0,5m.

Solución
Sean los arcos :

Por semejanza : L =3

^ ¡- = - -> Lx = 2 , 2 5 A*
3 8 x 8

L - L = 3 - 2,25 = 0,75

Rpta: ( D ).
Sobre el diámetro “ 2 R “ de un semicírculo se toman segmentos cualesquiera AB, BD, DE,
EF y FG, los cuales se utilizan como diámetros de otros semicírculos, según la figura.
Calcular la longitud total de A C ,, BC2 , DC3 , EC4 , FC5 G.

A) 2 rr R

B) Falta datos

C) tcR

D) S « R

E) Ninguno

Solución
Llamando , L , L , , ...

Las longitudes de las semicircunferencias

L = 7lR

/ AB \
BD
Li = K ,9 Loc = 71
2 J , 2 J
/ > / \
DE EF
Lo = 71 , L4 = n
\ 2 / 2

FG
Le = 7t
j

Sumando L, + L2 + L3 ... se tiene :

n
L, + L2 + l_3 + L4 + L5 = - | ( AB + BD + DE + EF + F G )

K
= — ( 2R ) = tiR
2 v '

'• L, + *-2 + >-3 + L4 + L5 = L

“La suma de longitudes de todas las semicircunferencias menores, es igual a la longitud de


la semicircunferencia mayor” .

Entonces, la longitud total:

L1 + L2 + l_3 + L4 + L 5 + L = tcR + tcR = 2 tcR.

Rpta: ( A ).

495
5) Hallar la longitud de la figura, si: m EF es de 60° y los demás arcos son semicircunferencias
de igual radio.

A )5 k R

B )4 tcR

7CR
C) 17_6~

19jiR
D> —

E) Ning. anterior

Solución
Se tendrá : L,otal = 5 ( L ® ) + (1 ).

Como : AB = L6 = R

Esto indica que el radio de AB es — .


2

tcR
Luego: *-ab “
( 2 ).

También:
/
60 O \ tcR
L^f = 2 tcR (3).

Reemplazando en (1 ) :
/ rcR ' TtR
Ltotal = 5° 17nR
+ ocal
V J

Rpta: ( C ).

6) Hallar la distancia “ x u que recorre un ü yo - yo ”, cuya cuerda tiene longitud L, hasta el


instante en que la cuerda esté tensa, manteniendo fijo el extremo A.

V 7C2 - 4 - 2
A) 71
*

;--ia
cuerda
■Jn2 - 4 - 2 total

2L A*.-'"" ^ #
yj K2 - 4 - 2 í*hiw.íj.r «I.KÍÍt-f¿"íw. >
C) • — X — '

7C

D) Ning. anterior.

496
Solución
Sea “ r ,l, el radio del arco que describe
ia cuerda, en su posición inicial:
2nr 21
= L r=
n
En el A ABA

2 4L2
A "B = ^ ( A A “ )2 - ( A B ) A "B = J L - ; A“B = - J n 2 - 4
K n

Ahora : x = A"B - r ; sustituy. valores:

L yn tt—- 4
x= — 2L ;
a -------- = - f y k2 - 4 - 2
71 V J

Rpta: (A).

La figura muestra un cuadrado de lado L, una circunferencia y cuatro semicircunferencias


Hallar el perímetro de la región sombreada.

5 ,
B A )3

B)27cL

0 ) 1 «L

8 ,

Solución

Se analiza una parte de la figura ;


el perímetro pedido equivale a :

x = * 8 ( L £ ) + 4 ( L - ) .......... ( I )

Como : OM = OF = MF

A OMF, equilátero.

Entonces : m ZOM F = m ¿M OF = 60°


y m Z.FOP = 30°

Análogamente :
m L MOE = 30° m Z.EOF = 30

497
í m ZOM F ^ L (' 60
Luego : L¿p = 2 rc x MO x = 2 7C X — X
[ 360° , 2 l^ 360
71L
l Óf -

O
co
O
( m ZE O F >
También : L£p = 2 k x OF x = 2 tc x — x
l 360° > 2 l 360°

i \
'/ 71L i ' ni
Finalmente, reemplazando en ( I ) : x = 8 + 4
V 6 , { 12

Rpta: ( A ).

ABCDEF, es un hexágono regular, de lado “ L


A , es centro de los arcos BF y CE.
BC y EF, son diámetros. Hallar el perímetro de
la figura sombreada.

kL
A) 4 * /3 )

E) N.A.

Solución
Se pide . x = Lgg + + L|g + L^g (I)

( m ZBAF > f 120° \


S ie n d o : LBF = 2 tc . AB . = 2 rc x L x
l 360° ) ^ 360° J
L '
= — x 2n x = — x 2 tc x LFE
/ 2 v. 2 /

\
' m ZCAE 60
L^ q —2 7i . AC . = 2 tcL / 3 .
v 360' / 360° ,

Entonces; en ( I ):

2 . tcL t i, rr 7CL TtL


x = —tcL + — + —L J 3 + (5 + / 3 )
3 2 3 v

Rpta: (B)
Hallar el perímetro de la figura sombreada

Si el radio OA, mide R. m L AOB = 60°.


Además : O, es centro de EF. AE y FB,
A diámetros.

A ) - |n R ( 6 - / 3 ) B)|^R(3 + / 3 )

C)^R (3 + ^ 3 ) D) f * R ( 3 - V 3 )
o B

E )f *R (6 -/3 )

Solución
x - La^ + LF¿ + + Lawb.......... ( I ).
A
Debemos encontrar los otros radios,
en función de R.

Sean P y r, centro y radio de la cir­


cunferencia.

A OFP (30°, 60° ) : OP = 2PF o F


OP = 2r *" R—

Entonces : OP + PM = OM
R
2r + r = R -+ r = —

Además : OF = r^¡3 = — >/~3 y FB = R - r f é —> FB = R -


3 3
AE y FB, forman una circunferencia de diámetro F B .

Por otro lado : Lgp = r*J~3 )

Finalmente, reemplazando lo hallado, en ( I ) :

R (6 -/3 )

Rpta: ( E ).
499
La figura muestra tres circunferencias de
igual radio V , ortogonales entre sí, dos a
dos.
Hallar el perímetro de la figura sombreada.

nr
A)

Ttr
B)
~3~
2nr
C)

nr
D)

E) N.A.

Solución

Por ser ortogonales las circunferencias:


OA 1 PA ; OC _L QC,
Q BlP B ;O E ±P E ,

OAPE, es un cuadrado.

La suma de las medidas de los ángu


los internos de APBQCO, es :
1 8 0 ° ( 6 - 2 ) = 720°

P e ro :
A + B + C = 3 x 90° = 270

L AOC + ¿CQB + L APB = 720° -2 7 0 ° = 450


450'
y ¿ AOC = ¿ CQB = ¿ APB = = 150

Entonces : L EOC = L AOC - L AOE = 150° - 90°


L EOC = 60° L EOF = 30°

/ \
ZEO F Tcr
Luego: Lee = 2
L^p r = 2 7cr Perímetro = 3 ( Lfp )
V 360° / 360'
7tr
Perímetro =

Rpta: ( D ).
11)
La longitud del lado del triángulo equilátero
ABC , es 2u.
AB y BC : diámetros. M, es punto medio
de AC. B, es centro de los arcos EMF y AC.
Hallar el perímetro de la región sombreada.

2n
A ) ^ ( 2 + / 3 ) u B) ^ ( i + ^ 3 ) u
3 3
271 2n
C)4f(3 + / 2 ) U D )^(2 + ^ ) u

E) N.A.
Solución

X“ + ^AC + ^"CF + ^EMF ~ ?

Como AB es diámetro, entonces


m Z A E B = 90°
AAEB = A AME
m Z ABE = m Z ABM = 30°

Luego : m Z AOE = 60°


\
60 71
- Ahora : L ^ —LCF
r í —2 71 x 1
360' / 3

60 2 ti f 120 2 k ^J~3
L ac = 2 k x 2 y lemf - 2 k -Z2
\ 360' J \ 360'

. _ ^ 71 271 7C 2 * 4 3
- Entonces : x = Lea + L ac + LCp + Lemf - — + — + — + -

271
X = ( 2 + / 3 )u

Rpta: ( A ).

12) La longitud del lado del cuadrado


ABCD , es 6cm.
A -> centro del arco BD.
D centro del arco AC.
BC. diámetro.
Hallar la suma de longitudes de los arcos:
^EF + ^FM
A) Ttcm. B) 37rcm. C) 27tcm

D) — k cm F } — ti cm
3 ; 3

501
Solución

Si se trazan AO , OF y AF
A ABO = A AFO ( LLL )

-¿AFO = Z A B O = 90c

Entonces , si Z BAF = a°

Z FOC = a°

V Z EOB = a °

Sea: Z FAD = p° a° + p° = 90P,


en el vértice A.

Z EOF = 2p°

A AMD, equilátero, ya que AM = AD = DM o

Z MAF = 60° - p°

Luego:
' 2p Kp
Lgp' = 2 ti x 3. L^ =
, 360 J 30
» •
/
60° - p° N
-> l^ - = Í L ( 6 0 - P )
360 /

Rpta: ( C ).
CAPITULO 1 7
v

AREAS DE LAS REGIONES PLANAS


CONTENIDO

I. INTRODUCCION: Postulado de la Unidad. Expresiones básicas: Rectángulo, Romboide,


Triángulo, Rombo, Trapecio.
Ejemplos Resueltos.

II. REGIONES TRIANGULARES: Teorema de Herón; Area en función de: Circunradio, Inradio,
Exradios . Expresión Trigonométrica. Casos Particulares (Triángulo rectángulo).

III. Relaciones Fundamentales en el Triángulo.

IV. Comparación de Regiones Triangulares - Propiedades.


Problemas Resueltos.

V. REGIONES CUADRANGULARES.- Casos y Expresiones.


Comparación de Regiones Cuadrangulares.
Propiedades.
Interpretación Geométrica del Teorema de Pitágoras.
Problemas Resueltos.

VI. REGIONES POLIGONALES.- Polígonos circunscritos. Polígonos Regulares. Sector Poligonal


Regular. Teorema: Polígonos semejantes. Teorema; Generalización en figuras semejantes.
Teorema aplicado en el triángulo rectángulo.
Problemas Resueltos.

VII. REGIONES CIRCULARES.- Círculo. Sector circular. Segmento circular. Zona circular. Aro
circular. Trapecio circular.
Lúnulas de Hipócrates.
Problemas Resueltos.

PROBLEMAS PROPUESTOS.

CLAVE DE LOS PROBLEMAS PROPUESTOS.


503
I.- INTRODUCCION
Una región triangular es la reunión de un triángulo y su interior.

Una región poligonal es una figura plana formada por la reunión de un número finito de
regiones triangulares en un plano, de modo que si dos cualesquiera de ellas se intersecan,
su intersección es o bien un punto o un segmento.

(a) (b) (c)

Observe el lector que una misma región poligonal se puede dividir de diferentes formas :
( figura a y c ) a fin de tener un conjunto de regiones triangulares.

El área de una superficie limitada, cualquiera, es la medida de su extensión, indicada por un


número positivo único, acompañado de la unidad adecuada (cm2 , m2 , etc.). De modo que
la hablar del área de una región triangular, región cuadrangular, región trapezoidal, región
poligonal, etc., estaremos haciendo referencia a la medida de la superficie limitada por un
triángulo, cuadrilátero, trapecio, polígono, etc. Para simbolizar el área de una región
cualquiera, comunmente se usan las letras mayúsculas A o S , siendo por ejemplo: SpQR, el
área de una determinada región triangular PQR.

1.1. P ostulado de la Unidad.- El área de una región cpadrada, se expresa por el cuadrado de
la longitud de su lado.

a
s

504
V

O bservaciones:
* En adelante, para abreviar, haremos referencia al área de un triángulo, área de un polígono,
etc., entendiendo, desde luego, que se trata del área de la región correspondiente.

* Es a partir del postulado de la unidad del área (área del cuadrado), que se demuestran las
fórmulas básicas para el cálculo de área de las diferentes regiones elementales: rectángulo,
triángulo, trapecio, etc.

* Dos regiones cualesquiera, que tienen igual área, se llaman equivalentes, independiente­
mente de la forma de uno con relación al otro. Así, por ejemplo, para la figura adjunta diremos
que el triángulo y cuadrilátero, son equivalentes.

* Dos triángulos congruentes ( en general dos polígonos congruentes ), son equivalentes.

Veamos a manera de aplicación del postulado de la unidad de áreas, la demostración de la


expresión para el cálculo del área de un rectángulo, así como la deducción para el romboide
y triángulo.

1.2. Teorema.- El área de todo rectángulo es igual al producto de sus dos longitudes.

Demostración :
Con la figura adjunta, donde A 1 = b2 , A2 = a2 y S’ = S, por ser rectángulos congruentes,
el área de la figura total es:

S + S’ + A 1 + A2 - StotaI
i--------q
— a i
p........ ......
Luego : 2S + b2 + a2 = ( a + b )2 T
2S + b2 + a2 = a2 + 2ab + b2 s b
Ai
Cancelando a2 y b2 : 2S = 2ab
a S a2
De donde : S = a . b , como queríamos i i □
demostrar. - - b ----- .... a -i

505
1.3. Teorem a.- El área de todo romboide es igual al producto de la longitud de un lado y la altura
respectiva a dicho lado.

^ABCD “ k • h

* Nota.- Puede expresarse también el área del romboide mediante el producto de la longitud
de CD y la altura trazada a dicho lado. ( Distancia entre AB y CD ).

D em ostración :

Como : A BHA = A CED, entonces AH = DE, por lo que HE = AD = b.

Además SAHB - SDEC 9 _ q


ABCD “ HBCE

Siendo : SHBCE = HE x BH = bh

^ABCD “ kh

1.4. Teorem a.- El área de todo triángulo es igual al semiproducto de la longitud de un lado y la
altura respectiva.

D em ostración :

Trazando paralelas para formar el paralelogramo ABCD; puesto que los triángulos ABC y
DCB son congruentes, sus áreas equivalen a la mitad de la del paralelogramo:

_ Ü ABDC _ b . h
ABC

b .h
ABC
A

506
* Demuestre el Lector las expresiones básicas para el cálculo de áreas del trapecio y rombo,
así como las deducciones para los triángulos rectángulo y equilátero.

1.5. Teorem a.- El área de todo trapecio es igual al producto de la semisuma de las longitudes de
las bases y de la altura.

1.6. C orolario.- El área de todo trapecio es igual al producto de la longitud de su mediana y la


altura.
Siendo m y h las longitudes de la mediana y la altura, respectivamente:

1.7. Teorem a.- El área de todo rombo es igual al semiproducto de las longitudes de las
diagonales.

•£H
(AC).(BD)
4

' c

* Nota.- También, para el rombo ABCD :


S = ( CD ) . ( BH )

507
1.8. C orolario.- El área de todo triángulo rectángulo es igual al semiproducto de las longitudes
de los catetos.

1.9. C orolario.- El área de todo triángulo equilátero es igual a! cuadrado de la longitud del lado,

43
multiplicado por el factor

1.10. C orolario.- El área de todo cuadrado se puede expresar como la mitad del cuadrado de la
longitud de su diagonal.

EJEMPLOS

1) En un trapecio ABCD, se conocen las longitudes de las bases: BC = 15cm. y AD = 27cm


P esun punto de AD, tal que al unirlo con C, resultan dos regiones equivalentes. Hallar PA

Solución
B 15
Del gráfico, se tiene:

^ABCP “ ^PCD
_ ( 2 7 - x)h
h=
2
De donde, fácilmente hallamos :

Rpta: x = 6 -> AP = 6cm

508
2) Un terreno que tiene forma de un trapecio rectángulo cuyas bases son : AB = 200m. t
DC = 180m. y la altura AD = 120m., ha de dividirse en tres parceles equivalentes, de modo
que sus dueños puedan, sin salir de sus propiedades respectivas, ir por agua a un pozo P,
situado en la base superior DC y a 75m. del punto D. Hallar la diferencia entre las longitudes
de los segmentos AR y GB de la base inferior.

A) 35 m B) 30 m
C) 40 m D) 25 m
E) 20 m

Solución
D — 75 - P 105
Por ser equivalentes, los trapecios :

^AOPR = ^GPCB
120
( AR + 75 ^ f GB + 105 >
—> . 120 = . 120
CM

^ 2 j
De donde, al simplificar y d e sp e ja r: B
AR - GB = 30

Rpta: ( B ).

3) En un trapecio de bases 3 y 5 se traza la mediana determinándose dos trapecios cuya


relación de área, es:
1
A) 1 B> 9 C> 2
D)
25
E)
16

Solución
- Sea ABCD, el trapecio.
B
3 + 5
Luego : MN = — - — = 4

- Se pide :

^3 + 4 '
. h
’MBCN _ v
\
AMND '4 + 5
.h
/

Sm bcn _ J_
S amnd ^

Rpta: ( B ).

509
4) En la figura, hallar el área de la región sombreada.

y í) 65u2
B) 60u2 Bh 10u He
C) 75u2 6u
D) 80u2 D -í-
t

E) N.A. 4u
*
Solución *iE

Se pi de: SABD
f
O bservam os:

^ABD + ^BCD + ^AED “ ^ABCE f

Luego :
\
e 10x6 15x4 f 10 + 15
b ABD + -----~----- + -----~----- . 10
\ T
CE

Efectuando : SABD = 65u2.

Rpta: ( A ).

5) La longitud del lado de un cuadrado ABCD es 6cm. Se construye exteriormente el triángulo


equilátero CED y se traza A E . Hallar el área del triángulo AED.

Solución
Del gráfico :
AD . EH
AED (1 )

Siendo AD = 6cm. y en el ADHE,


DE
EH se opone a 30°, luego : EH = —

EH = 3. Sustituyendo en (1 ) :

6x3
Rpta: SAED S AED = 9 CrT>2

6) Calcular el área de un triángulo equilátero, sabiendo que el radio de la circunferencia inscrita


mide 2m.

A) 6 / 3 m B )1 2 /3 m C) 8 / 3 m D) 9 ^ 5 m E) 10 /3 m

510
Solución
Sea el triángulo ABC
B
En A IMC : MC = 2 / 3 ,
( se opone a 60° )

AC = 2MC AC = 4 / 3

/3
Luego : SABC = AC

ABC

Rpta: ( B ).

En un triángulo equilátero ABC de 4m. de lado se trazan los segmentos : MN paralelo a AC


y ND pareleio a B A , de tal manera que se forma el rombo AMND.
¿Cuál es el área del rombo?

A) 2 / 6 m B) 4 m: C) 2 / 3 m D)3^3 m E)4/3 m

Solución

Como MN es paralelo a AC, el A MBN es equilátero. Luego, BM = MN = AM. Esto indica que
el lado del rombo AMND mide 2m.

Luego : B

AMND = 2(Samd) = 2( AM2 )

SAmnd = 2 ( 2 ^ )

AMND = 2 / 3 m

Rpta: (C)

En un triángulo rectángulo un cateto mide 4m. y la altura sobre la hipotenusa 2,4m.


¿Cuál es el área del triángulo?

A) 9,6 m B) 8 m2 C) 7 m2 D) 6 m2 E) 5 m:

511
Solución
Consideremos el triángulo PQR. Q
Luego:

_ PR . QH
PQR

S = PR(2,4)
PQR ------- -

PQR = ( 1,2 ) P R (1 )

Para hallar PR, previamente calculamos PH, en el A PHQ, con el teorema de Pitágoras

PH = y/42 - ( 2 , 4 ) -4 PH = 3,2

Luego, en A PQR, sabemos por relaciones métricas :

PQ = PR . PH -> 42 = PR . ( 3,2 ) /. PR = 5, sustituimos esto en (1 ) :

SPQR = ( 1, 2 )( 5 ) ®PQR — '

Rpta: (D)

9) En un triángulo rectángulo ABC, el ángulo recto es C. Por D punto medio de A B , se levanta


una perpendicular que corta al cateto CB en el punto E. Se sabe que : AB = 20m. y
AC = 12m. Entonces, el área del cuadrilátero ADEC, es :

A) 37,5 m B) 58,5 m2 C) 48 m D) 75 m: E) 24 m2

Solución

Del gráfico :

^ADEC ” ^ACB " ^EDB (1 )

En A ACB, con el Teorema de Pitágoras


B
hallamos CB = 16

L u e g o : SACB _ A C . CB 12x16
^ A C B “ 9 6 m 2
(2).

Para el SEDB, hallamos antes ED a partir de la semejanza entre los triángulos EDB y
ACB :

ED DB ED 10
— = —» — = — ED = 7,5
AC CB 12 16

512
ED . DB _ ( 7 , 5 ) ( 1 0 )
Ahora : S eos SEDB = 37,5 m2 (3 )

En seguida con ( 2 ) y ( 3 ) en ( 1 ) : SADEC = 96 m2 - 37,5 m2 = 58,5 m2

Rpta: ( 8 )

En un triángulo ABC, isósceles, con AB = BC, la altura que parte de B mide 8m. y el
perímetro 32m.

El área de! triangulo, es :

A) 126 m: B) 64 m2 C) 48 m D) 142 m: E) 56 m2

Solución B
Con la figura adjunta :

2a + b = 3 2 ........... ( 1 ).

En el A BHC :
/
a2 = 82 +

De donde : a2 - — = 64
4

ó : 4a2 - b2 = 256 -» ( 2a + b ) ( 2a - b ) = 256

Con ( 1 ) : ( 32 ) ( 2a - b ) = 256 2a * b = 8 (2).

De (1 ) y ( 2 ), hallamos : a = 10 y b = 12

Luego : S ABC
,„ „ = A 9_'.BH = 1 2 x 8 = 48

SABC = 48 m2

Rpta: ( C ).

Los lados no paralelos y la base menor de un trapecio isósceles son congruentes entre sí
y miden 10m. Si la base mayor mide 22m., el área del trapecio, en metros cuadrados, es :

A) 128 B) 320 C) 144


D) 160 E) 220

Solución
Según el gráfico : m = 6

Luego : h2 = 102 - m2 = 102 - 62

h = 8

513
'1 0 + 22 ^
• • El área : S = .8
\ /

S = 128.

Rpta: (A)

12) El área de un cuadrado inscrito en un semicírculo es al área del cuadrado inscrito al círculo
entero, como :

A) 1 :2 B) 2 : 5 C) 2 : 3 D) 3 : 5 E) 6 : 4

Solución
( a ) En el semicírculo, consideramos el triángulo rectángulo

x
x2 +
\ 2 j

De a q u í: x2 = — R2

Luego, el área : S- = — R2 (1 )
1 5

( b ) En el círculo : y = R

Siendo el á r e a :
s, = y2 S2 = 2R2 (2)

Luego, de ( 1 ) y ( 2 ) :

R
1 _
2
2R 5

Rpta: ( B ).

13) Una de las bases de un trapecio mide 10m., su altura 4m. y el área 32m2. Calcular la longitud
de la paralela a las basec, trazada a un metro de distancia de la base dada.

Solución
Llamando “a” la longitud de la base desconocida, con el dato del área :

\
x 4 = 32 -> a = 6
V J

Luego, tenemos la figura adjunta, donde la incógnita es EF

514
A s í:

^A£FD + ^EBCF “ ^ABCO


Es d e c ir:
( EF + 10 > ' EF + 6 ^
X 1+ í x 3 = 32
l J
CM
, 2

De donde, fácilmente hallamos

Rpta: EF = 9.

14) Los lados de un rombo son dos radios y dos cuerdas de una circunferencia de 16cm. de
radio. El área del rombo, en cm2, es :

A) 128 B) 1 2 8 /3 C) 256 D) 512 E) 5 1 2 / 3

Solución
Del gráfico, notamos que los triángulos AOB y BOC son equiláteros.

Luego:
B
ABCO

\
S - ^ 16‘
ABCO "*
/

® ABCO = 128 ^

Rpta: ( B )

15) En un cuadrado de 6m. de lado se inscribe un rectángulo de 8m. de diagonal, con la condición
de que sus lados sean paralelos a las diagonales del cuadrado.
El área del rectángulo, es :

A) 2 m2 B)2 / 5 m C )4/5 m D)4m: E) Ninguna anterior.

Solución
Al graficar observamos que los vértices
del cuadrado ABCD son los respectivos
vértices de los ángulos rectos de cuatro
triángulos rectángulos isósceles.

El área S del rectángulo :

S = ( a / 2 ) ( b / 2 ) -» S = 2ab...( 1).

Siendo, por dato, la longitud del lado del


cuadrado 6 metros : a + b = 6 .
515
Elevando al cuadrado, para tener el producto de a y b :
( a + b )2 = e2 -> a2 + b2 + 2ab = 3 6 ........... ( 2 ).

Además en el triángulo rectángulo sombreado :

( a / 2 ) % ( b ^ ) 2 =82

De donde : a2 + b2 = 3 2 ( 3 ).

Finalmente, con (1 ) y ( 3 ) en ( 2 ) :

32 + S = 36 S = 4 m2.

Rpta: ( D ).

Si la base de un rectángulo aumenta en 10% y el área no varía, es porque la altura disminuye


en :

A) 9% B) 10% C) 11% D) 11— % E) 9 — %


9 11
Solución
3--- L_
Inicialmente se tiene el rectángulo :

Luego, al aumentar la base en un 10% s

Entonces : 1 c
( 1,1b ) x = bh

_ 10u
De donde : x = — h
11

Es decir, la altura disminuye en :

10 1 J L
1-
11 11
s
/

Equivalentemente :

_1_ _1_
x 100 % = 9 — % 1 r
11 11
1,1b
Rpta: ( E ).

Un terreno tiene forma rectangular y se sabe que su perímetro mide 46m., siendo su diago­
nal igual a 17m.

¿Cuál es el área del terreno?

A) 90 m2 B) 72 m2 C) 120 m2 D) 60 m2 E) 80 m2
Solución

Del dato : 2(a + b) = 46

a + b = 23 (1 )

Además : a2 + b2 = 2 8 9 ............... ( 2 ).

Para obtener el área S = ab, elevamos


ambos miembros de la expresión (1 ) al
cuadrado : (a + b)2 = 232

Desarrollando : a2 + b2 + 2ab = 529

Con ( 2 ) 289 + 2ab = 529

ab = 120 S = 120 m2

Rpta: ( C ).

La base de un triángulo isósceles es Si las medianas dibujadas hacia los lados


congruentes se cortan perpendicularmente; entonces, el área del triángulo, es :

A) 1,5 B) 2 C) 2,5 D) 3,5 E) 4

Solución

Sea el A ABC, con AB = BC. Entonces :


AN = C M y AH = HC. El A AOC es B
isósceles : AO = OC.

_>/2
OH = — -> OH =

Siendo O, el Baricentro del A ABC

BH = 3( OH ).

• • BH = - J 2

Luego :
1
SABC = "2 ( AC )( BH )

ABC = 1,5

Rpta: ( A ).

517
19) En un triángulo ABC se conoce que la altura BH y la mediana BM trisecan al ángulo ABC
Hallar el área del triángulo ABC, si HM = 1 m.

A) V 3 m B) 2 m C ) 2 / 2 m' D) 2 / 3 m E ) 3 j3 m

Solución

- Del gráfico :

A ABM isósceles

AH = HM = 1

Luego : MC = AM = 2 1 h 1 -
“—4
En el A BHC, por el primer teorema
BC MC BC 2
de la bisectriz C = 302
BH HM BH 1

HC
Luego BH = BH = / 3
7 ? /3

- Finalmente

Rpta: ( D ).

20) Hallar el área de un rombo ABCD, en el cual M es punto medio de B C ; AM corta a BD en


el punto R , RM = ~2 y el ángulo BRM mide 45°.

A) 12 B) 48 C) 24 D) 36 E) 18

Solución

S = AC . BD (1 )
ABCD -----------------

Del gráfico : R, es Baricentro del A ABC


AR = 2( RM ).

AR = 2 / 2

- En el A AOR : AO = OR = .AR
7?
AO = OR = 2

Entonces : BO = 3 ( O R ), para el A ABC


BO = 3(2) BO = 6. Luego: BD = 12.

51H
Además : AC = 2( A O ) —> AC - 2( 2 )
AC = 4.

Sustituyendo los valores de AC y BD, en (1 )

c _ 4 x 1 2 _ p4
ABCD “ ----- -------

Rpta: ( C ).

519
II.- REGIONES TRIANGULARES

Veamos a continuación, diversas expresiones para el cálculo del área de un triángulo,


derivadas en su mayoría de la fórmula básica y dependiendo de los datos considerados.

11.1. Teorem a de Herón de A lejandría.- El área de todo triángulo, es igual a la raíz cuadrada del
producto del semiperímetro y su diferencia con cada lado.

D em ostración : B
En efecto, para el AABC, se tiene

s = bi i hb
(1).
2

Pero, sabemos que, siendo p el se­


miperímetro del triá n g u lo :
2
= -V p (p -a )(p -b )(p -c) ............( 2 )
b

Al reemplazar ( 2 ) en la expresión ( 1 ) : S = ^ p ( p - a )( p - b ) ( p - c )

2. En fu n ció n del c ircu n ra d io R.- El área de todo triángulo es igual al producto de las
longitudes de los tres lados, dividido por cuatro veces el circunradio.
D em ostración
B En A ABC : S = ( 1 )-
2
Sabemos, además que : h ( 2R ) = a . c
a.c
2R (2).

Reemplazando la expresión ( 2 ) en la (1 )

3. En fu n ció n del in ra d io r.- El área de todo triángulo es igual al producto del semiperímetro
y el inradio.
b
D em ostración :

* Por adición, tenemos :

^ABC - ^AIB + ^BIC + ^AIC

Luego: SABC - AB.r


------------ +
BC. r AC . r
+ ------------

520
Esto e s :

' AB + BC + AC
ABC
V

pero :

AB + BC + AC
= p. ( semiperímetro )

De d o n d e :
^ABC _ P • r

.4. En fu n ció n de un ex-radio.- El área de todo triángulo es igual al producto del ex-radio relativo
a un lado y la diferencia entre el semiperímetro y dicho lado.

D em ostración :

Sea el A ABC, donde AB = c , BC = a , AC = b


y ra el ex-radio relativo al lado B C .

Se tiene : SABC - SABEC - SBEC

Es d e c ir. SABC - SAEC + SAEB - SBEC

h ■ra c . rB ^ a ■ra
ABC

ABC
j

Siendo : a + b + c = 2p, el perímetro del A ABC,

De a q u í: b + c = 2p - a , reemplazando en la expresión ( 1 )

' 2 p - 2a
ABC
\ /

* En forma análoga : ^ abc ~ ( P " ^ )ri

II.5. En fu n ció n del in ra d io y lo s ex-radlos.- En todo triángulo, el área es igual a la raíz


cuadrada del producto del inradio y los tres ex-radios.

521
Demostración :
* En efecto, por deducciones anteriores
tenemos, para el área :

S = p.r

S = (p-a)r a

S = ( p - b ) rb

S = ( p - c ) rc

Multiplicando miembro a miembro estas expresiones :

S4 = p( p - a ) ( p - b ) ( p - c ) r . ra . rb . rc

i
Esto e s : S< = S 2 . r . ra . rb . rc

De donde : S = ^ r . ra . rb . r£

11.6. Expresión T rig o n o m é trica .- En todo triángulo, el área se puede expresar como el
semiproducto de dos lados, por el seno del ángulo comprendido entre ellos.

* Para un triángulo ABC, cualquiera :

* La demostración se deja al lector.

Casos P articulares. A partir de las expresiones generales, deduciremos a continuación


fórmulas para el caso de triángulos rectángulos.

11.7. En todo triángulo rectángulo el área


se puede expresar como el producto
de los exradios relativos a los catetos.

D em ostración :

* Desde luego, sabemos que, para el


área S de la región triangular ABC :
S
S = ( p - a )ra ->

Además : = p_ 5
<b

522
Sumando miembro a miembro estas expresiones :
8 8
— + — = 2 p - a - b , siendo : 2 p = a + b + c
'8

>1
1 1 C ra . rb
Luego: (1 )
V ra rb v ra + rb /

Ahora, recordemos que “en todo triángulo rectángulo la suma de exradios relativos a los
catetos es igual a la longitud de la hipotenusa" ( Propiedad ). Por lo que : ra + rb = c.

Al reemplazar en (1 ) :

11.8. En todo triángulo rectángulo el área es igual al producto del inradio y e! exradio relativo a
la hipotenusa.
Para demostrar esto, recordemos que :

s 2 = r . r , . rb rc

Pero, por lo a n te rio r: S = ra . rb

Luego : S2 = r . S r

S = r.r

11.9. En todo triángulo rectángulo, el área se puede expresar como el producto de las longitudes
de los segmentos que determina la circunferencia inscrita sobre la hipotenusa.

Sea el A ACB, donde AT = m y TB = n,


son las longitudes de los segmentos en
mención. Demostraremos, que :

^ACB - m • n

Del gráfico tenemos : AC = m + r y CB = n + r

Por lo que : SArn = AC ■■CB -> 2 SACB = ( m + r ) ( n + r )

esto es : 2 SACB = m . n + (m + n + r ) r (1 )

Además : p . r = S ACB (m+ n+ r)r = S ACB (2 )

Sustituyendo ( 2 ) en ( 1 ) : 2 SACB = m . n + SACB

SACB = m . n ........... Como queríamos demostrar,

523
11.10. En todo triángulo rectángulo, el área puede expresarse como el producto de las longitudes
de los segmentos que determina en la hipotenusa, la respectiva circunferencia ex-inscrita.

* Así, para el A ACB

J EV s

S = q. í

D em ostración :

Por ser tangentes trazadas desde un mismo punto : AD = AE = q, BE = BF = t,


y además : CD = CF -> b + AD = a + BF

Con lo anterior : b + q = a + £. De lo que : q - í - a - b.

Elevando ambos miembros de esta última expresión , a fin de obtener el producto entre q
y t :
(q- O 2 = (a-b)2

Desarrollando : q2 + í 1 - 2q? = a2 + b2 - 2 a b ( 1 ).

Siendo, para el A ACB : a2 + b2 = AB = ( q + £ )2 ........... ( 2 )

Reemplazando ( 2 ) en (1 ) :
q2 + l 2 - 2q£ = ( q + O 2 - 2ab

De donde : ab = 2q¿

ab
= q.t Esto es : S = q . t

11. Sea ÁCB un triángulo rectángulo ( ver figura ), recto en C. Se dibuja la circunferencia ex­
inscrita relativa a uno de los catetos, la cual es tangente a la prolongación de la hipotenusa,
en F.

Demostrar q u e :

F A b

524
III.- RELACIONES FUNDAMENTALES EN EL TRIANGULO

Consideremos un triángulo ABC cualquiera, de inradio r, circunradio R, ex-radio ra , rb , rc


y alturas ha, hb, hc . Entonces :

111.1. La inversa del inradio es igual a la suma de las inversas de los ex-radios
Demostremos esto. Sabemos que para el área :

S = p .r -> - = — (1)*
r s

S S . S
Además podemos e s c rib ir: ~ = p - a ; — = p - b y — = p -c
rb

Sumando miembro a miembro estas últimas tres expresiones y factorizando S

\
1 1 1
—+ — + — = 3 p - ( a + b + c)
v ra rb r(c )

/
1 11
Esto e s : S —+ — + — = P
*b c )

De d o n d e : 1 1 1 p (2).
—+ — + — = —

1 -1 L -L + i
Luego, comparando (1 ) y ( 2 ) : r ra rb rc

** Ejemplo : Hallar el área del triángulo, en el cual los ex-radios miden

r = 2 cm., r. = 3 cm. y r = 6 cm.

S o lu c ió n :
1 1 1 1
* Cálculo de r: - = —+ —+ —
r ¿ ó o

De a q u í: r - 1 cm.

Luego : S = ^ r . ra . rb . rc = 1 ) ( 2 ) ( 3 ) ( 6 ) cm:

S = 6 cm2

* Nota.-La demostración de las siguientes relaciones son similares. Se dejan al lector


como ejercicio.

2. La inversa del inradio es igual a la suma de las inversas de las alturas:


1 = _1_ J _ + _[
r ha hb hc

525
Ex-radios en función de alturas :

1 1 ±
1 _ 1
r. “ N> ' h.

1 1 1
1 1

*> ’ h . he ' hb

1 1 1 1

*c " h . hb ~ h c

Ht.4. Además recordemos la relación de S te in e r:

r. + rb+ re = 4R + r
M.5. Cabe adicionar fa relación resultante de combinar III.1 y III.2

ra rb *c ha Ní N

526
IV.- COMPARACION DE REGIONES TRIANGULARES - PROPIEDADES

IV.1. Las áreas de dos triángulos son entre sí como los productos de sus bases y respectivas
alturas.

D em ostración :
Q
* Para la figura adjunta,
b .h
como : S ABC

m .n
PQR

S ABC . b h I
entonces : m .n
SpQ R

IV.2. Si dos triángulos tienen congruente un lado, sus áreas son entre sí como las respectivas
alturas.
N
B
* Para el gráfico, se tendrá :

IV.3. Si dos triángulos tienen una altura congruente, entonces las áreas son entre sí como sus
respectivas bases.

Como ejem plo:

S AED AD
Sdeb DB

IV.4. Dos triángulos que tienen congruentes un lado y las respectivas alturas, son equivalentes

D e m o s tra c ió n :

* Según gráfico, por IV.1

5 ABC = b • b
= 1
S jvm b *b

ABC - S TVM

527
IV.5. En todo triángulo, una mediana cualquiera determina dos triángulos parciales equivalentes

D em ostración :
B
En efecto, sea BM una mediana del
A ABC. Entonces, por tener congruen­
tes AM y MC y la misma altura
desde B :

IV.6. En todo triángulo, al unir los puntos medios de los tres lados se determinan cuatro triángulos
parciales equivalentes.

D em ostración :

* Por ser congruentes los triángulos MBN, AMP,


MNP y NPC, se tendrá :

IV.7, En todo triángulo, al trazar las tres medianas se determinan seis triángulos parciales
equivalentes.

B
En efecto, por IV.5, en el gráfico adjunto X = Y,
U = V , W = Z . Bastará probar que X Z . Para
la mediana BM :
c _ c
ABM ~ BMC

X +Y +V = Z +W + U

Cancelando V y U : 2X = 2Z X = Z

Luego :

IV.8. En todo triángulo, si se une el baricentro con los tres vértices se determinan tres triángulos
parciales equivalentes.

* Siendo G el baricentro del A ABC,


al trazar GA, GB y GC, tenem os:
IV.9. En todo triángulo, al unir el baricentro con los puntos medios de los tres lados, se determinan
tres regiones equivalentes.

* Así, para el A ABC, donde G es baricentro


y M, N, R son puntos medios de los lados :

ABC
S1 - S2 - S3 -

IV.10.AI unir el baricentro de todo triángulo con los puntos medios de dos lados y dichos puntos
entre sí, se determina una región triangular cuya área equivale a la doceava parte del área
total. D
b
* En el gráfico :

tV.11 .Si dos triángulos ( dos polígonos, en general), son semejantes, entonces las áreas son entre
sí como los cuadrados de cualquier par de elementos homólogos.

D em ostración :

* Sea k, la razón de semejanza en los triángulos ABC y A’B'C’.


S abc _ _ bh B*
Como :
S a ’B'C" ~ b ’ h’ - B

5 ABC = ^ . i U k 2
S a ’b c b* h'

Luego : ABC
S a ’B'C’
_
(a 'f
, \2
( b* )
2
c
(O
, \2
( h ‘ )2

* En la relación anterior podemos considerar también medianas, inradios, circunradios, etc

IV.12.Si dos triángulos tienen un par de B Q


ángulo congruentes, las áreas son
entre sí como los productos de los
lados que forman dichos ángulos.
* Sean los triángulos ABC y PQR, con
Z B = Z Q; entonces :
R
Sabc _
AB BC
S PQR PQ QR

529
IV.13.En los triángulos que tienen dos ángulos suplementarios, uno en cada triángulo, las áreas
son entre sí, como los productos de los lados que forman dichos ángulos.

* Para los triángulos ABC y MNL,


s i : a + <{>= 180°, entonces : B M

S abc _ CA . CB
®MNL MN . NL

Nota.- Las propiedades que no


han sido demostradas se dejan
al lector como ejercicio.

PROBLEMAS RESUELTOS

1) Los lados AB y BC de un triángulo ABC tienen longitudes 8 y 9 centímetros, respectiva­


mente. Una semicircunferencia de radio 6cm .es tangente a AB y BC, teniendo su diámetro
sobre AC. Hallar el área del triángulo.

Solución B

Trazando BO y luego los radios


a los puntos de tangencia :

^ABC ” ^AOB + ^BOC


8x6 9x6
+ --------
ABC

Rpta: SABC = 51 cm2.

2) En un triángulo ABC, se traza la bisectriz interior B D .

S abd AB
Demostrar q u e : ( Propiedad, para todo triángulo ).
s dbc BC

Solución
B
Como los triángulos ABD y DBC tienen :
Z ABD = Z DBC, entonces, por IV. 12:

S abd _ AB . BD SABD _ AB
c or* nr» > donde : q
ODBC ODBC
nnr t3 U

530
El área de un triángulo ABC es 22cm2. Sobre las prolongaciones de los lados BA y BC se
toman longitudes AE = 2AB y CF = 3BC. Hallar el área del cuadrilátero ACFE.

Solución
Del gráfico, como los triángulos EBF
y ABC, tienen en común el Z B :

SEBF BE . BF
S abc ®A . BC

22 + X (3n)(4a)
Luego :
22 n.a

22 + X = 12( 22 ) , de donde X = 242

Rpta: SACFE = 242 cm2

El área de un triángulo ABC es 72 cm2. Por el baricentro G se trazan paralelas a AB y BC,


que interceptan a AC en los puntos E y F, respectivamente. Hallarel área del triángulo EGF.

Solución

B En el A ABC, como G es baricentro, al trazar la


mediana BG M , sabemos que BG = 2 GM
Si GM = m, entonces BG = 2m.

Como los triángulos EGF y ABC son semejan­


tes, d o nde GM y BM son m e d ia n a s
homologas :

EGF GM EGF _ m
ABC BM‘ 72 cm2 ( 3m )

Rpta: SEGF = 8cm2

Hallar el área de la región triangular correspondiente a un triángulo isósceles, en el cual la


base mide 16cm. y el circunradio 10cm., siendo el triángulo obtusángulo.

Solución
B
Como el A ABC es isósceles,
la prolongación de la altura
BH pasa por el centro O de la
circunferencia circunscrita.

Siendo AC = 16 .*. HC = 8 y
en el A OHC, con el Teorema 10
de Pitágoras, hallam os: o

531
OH = ^/ l O2 - 82 , OH = 6.

Luego: BH = OB - OH = 10 - 6 BH = 4

A C . BH 16x4
En seguida : ABC

S ABC = 3 2 Cm2

6) El área de un triángulo rectángulo ABC ( B = 90°), es 24cm2. Exteriormente se dibujan los


triángulos equiláteros AEB y BFC. Trazar EF y hallar el área del triángulo EBF.

Solución
Haciendo AB = c y BC = a, el dato
AB . BC
ABC
= 24 cm2

L uego:
a .c
= 24 cm2

Es d e c ir:
a . c = 48 cm2 .......... (1 )

La prolongación de FB forma con AB un ángulo de 30°, por lo que BH será perpendicular


a AE :

' C '
_ B F . EH a .c 48 cm2
EBF EBF , con (1 ) . SEBF -

Rpta: S EBF
™ = 12 cm2.

7) Hallar el área de un triángulo equilátero, sabiendo que las distancias de un punto interior a
los tres lados, son de 2, 3 y 4cm.

Solución
Sea ei A ABC , P el punto en mención y
a = 2cm, b = 3cm, c = 4cm, las distancias a
los lados.

Al trazar PA, PB y PC :

^ABC “ ^APB + ^APC + ^BPC

L .h L .c L .b L .a
= + + -------

532
De donde : ( Propiedad ) ( * )

Luego, con los valores dados : h = 9cm

Además, sabemos que : h = ■— / 3 -> L = ~^j~3h

L = —/ 3 ( 9 c m ) = 6 / 3 cm
3

Y el área : ABC = il/3 = ( e / 3 cm fÉ


RPta: S abc = 27/3 cm

O Otra alternativa de demostración para esta propiedad que el estudiante debe haber visto en
el capítulo de congruencia de triángulos.

8) En un triángulo ABC, se traza la mediana BM y luego MF perpendicular a BC ( F en BC ).

Si MA" dista 8 / 3 cm. de BM, MF = 6cm. y el ángulo MBC mide 30°, hallar el área del
triángulo MFC.
B
Solución
Del gráfico :
S - ^ - o
MFC “ MBC BFM (1)-
En el A BFM ( 30° , 60° ) :

BF = 6 / 3 y BM = 12

Luego :

BF . MF ( 6/ 3 ) ( 6 )
BFM BFM = 1 8 / 3 cm ..(2)

Además como BM es mediana del A ABC : SMBC = SABM

BM . AE 12( 8 / 3 )
Siendo : S ABM

S ABM = 4 8 / 3 cm2, entonces también : SMBC = 4 8 / 3 cm (3).

Finalmente con lo hallado en ( 2 ) y ( 3 ) , al sustituir en ( 1 ):

s mfc = 4 8 / 3 cm2 - 1 8 / 3 cm2

Rpta: ;mfc = 3 0 / 3 cm

533
9) El área de un triángulo es “S”. Si se prolongan los lados en un mismo sentido y una longitud
igual a la del lado prolongado, hallar el área del triángulo que se forma al unir los extremos
de dichas prolongaciones.

Solución

Sea ABC el triángulo dado y A’B’C’ el


de los extremos de las prolongacio­
nes.

Del g rá fico :
SA'B'C' = S + X + Y + Z ( 1 ).
A*
* Los triángulos A’B’A y ABC tie­
nen un par de ángulos suplementa­
rios ( Z A’AB' + Z BAC = 180°), luego,
por IV. 13 :

S A'AB' _ ( A A ')( A B ') X ( b )( 2 n ) .


SABC ( AC)(AB) S ( b )(n ) "

Análogamente, al comparar los triángulos BB’C’ y A’CC’ con ABC : Y = 2S , Z = 2S.

Sustituyendo en ( 1 ) : SA.B.C. = S + 2S + 2S + 2S

Rpta: SA.B.C. = 7S.

10) El área de un triángulo es A. Si se unen los puntos medios de los lados se determina otro
triángulo de área A r Si se unen los puntos medios de los lados de éste último se obtiene un
tercer triángulo de área A2. Si se repite indefinidamente este proceso , hallar :
A + A t + A2 + ..., en función de A.

Solución

_A
Se tienen A. =
4
Ai
Ao = —> Ao —
4 16
_A_
Ao = A 2 —> Ao —

64

Luego : £ = A + A 1 + A2 + A3 + ..

^ . A A A
) = A + H H h ...
^ 4 16 64

Esta es la suma de los infinitos términos de una progresión geométrica, de primer término:
1
t . = A y razón : r = -r < 1.
1 4
534
Como sabemos, por Algebra, dicha suma se evalúa mediante la fórmula :
ti
I =
1- r

Entonces : 1
1-

Rpta: X - g A.

11) Uno de los lados de un triángulo tiene longitud í. H allarla longitud del segmento paralelo a
dicho lado e interceptado por los otros dos, que determina figuras equivalentes.

Solución B

Sea EF || AC, donde :


AC - i y. SEBF = SAEFC = S

Como los triángulos EBF y ABC,


son sem ejantes:

S ebf _ E F 2
0 ABC AC

S EF2 _ tJ 2
Es d e c ir: , de donde al simplificar S y d e sp e ja r: E F = —^ —

12) ¿De qué naturaleza es aquel triángulo en el cual, conocidas las longitudes a y c, de dos
lados, el área es máxima?. Hallar el valor de dicha área.
Solución

* Considerando la figura adjunta, de acuer­


B
do a la expresión trigonométrica para el
área :

S = sen B ............( 1 ).

Ahora bien, como a y c son constantes y el único variable es sen B, el área S será máxima
cuando sen B sea máximo, esto es : sen B = 1. Por lo que B = 90°. Es decir, se trata de
9 C
un triá n g u lo rectángulo. Y al reemplazarlo en ( 1 ), el valor del área máxima : S = — 1—

13) En un triángulo rectángulo ABC, las longitudes de los ca te to s: AB = 3cm. y BC = 4cm. Se


dibuja el triángulo isósceles BDC ( BD = D C ), equivalente a ABC, interceptando BD a AC
en el punto E. Hallar el área del triángulo BEC.
535
Solución
Del gráfico, como los triángulos BDC y
ABC son equivalentes y tienen la misma
base B C , entonces DH = AB = 3.

Además AD resulta paralelo a BC:


AD = BH = 2 y de la semejanza entre
los triángulos BEC y DEA :

,EP _ BC EP BC
EQ ” AD QP-EP AD

Luego :
EP 4
=- -> EP = 2
3 - EP 2

BC . EP ( 4 )( 2 )
Entonces : SBEC = — - — • = — ^

Rpta: SBEC = 4cm2.

14) Los radios de dos circunferencias exteriores miden 3 y 8cm., respectivamente, siendo las
tangentes interiores comunes perpendiculares entre sí. Hallar el área del triángulo que
forman las dos tangentes anteriores y una tangente exterior común.

Solución

El triángulo formado, ABC, es recto en


B, y las circunferencias son exinscritas
relativas a los catetos.
Luego, de acuerdo a II-7, tenemos :

S ABC = ra . rc = ( 3cm ) ( 8cm )

Rpta: SABC = 24 cm2.

15) Lahipotenusa AC deun triángulo rectángulo isóscelesABC,tiene longitud" i' ”. Exteriormen-


te se construye el cuadrado ACDE. BE y BD cortan a AC en los puntos P y Q. Hallar el
área del triángulo PBQ.

Solución

Del gráfico : SPB0 _ ( 1 ).


2

Para hallar PQ : A PBQ - A EBD

536
PQ 2 B
Luego: ed 9.
( + -
2
PQ 2
PQ = —
3 3

Como, además BH = — ,

sustituyendo en (1 ) :

f. í
—x —
= 3 2
PBQ ----- 1-----

f.2
Rpta P8Q
12

16) Hallar el área de un triángulo, sabiendo que las longitudes de las alturas son 12cm., 15cm
y 20cm.

Solución
Sean : ha = 12 , hb = 15 y hc = 20

Para los exradios del triángulo, usaremos las relaciones III.3 :

1 J_ _1 1 1 1 1
+ —» ra - 30
hb + hc 15 20 12 30

2 1 1 j_ _i_
—+— -> Tk = 15
rb hb 12 + 20 15 15

1 1 1 1 1 1 1 1
—+ — -> rc “ 1 0
hb 12 + 15 20 ~ 10

Luego, el inradio :
1 1 1 1
— r= — + — +
r ra rb rc

1 1 1 1 1
— + *+*
■ -■ = _ ■ — r= 5
r 30 15 10 5

Y para el área con la expresión 11.5 :

S = ijr. ra . rb . rc = ^ 5 x 30 x 15 x 10 = 150
Rpta: S = 150 cm2.

537
17) En un triángulo ABC, de área 26 cm2, AB = 8cm y BC = 10cm. La mediana AM y la bisectriz
interior BD se interceptan en el punto P.
Hallar el área del triángulo BPM.
Solución
Para el A ABC, como AM es mediana :
_ S ABC
ABM S ABM = 13Cm 2

Siendo : S ^ p + SBPM = SABM

3 ABP + ^BPM = 13cm 2........... ( 1 )•

Además, por el problema n + 2 ,


S abp _ AB __ 8^
Sabemos que : S ABP = 8 S
'-’BPM (2).
'BPM BM ” 5

13
Sustituyendo ( 2 ) en ( 1 ) : - S BPM + SBPM = 13cm2 -> BPM = 13 cm2
5

R P ta ; S BPM = 5C m 2*

18) Las longitudes de los lados de un triángulo son 5; 6 y 7 centímetros. Hallar las longitudes
del inradio y circunradio.
Solución

En este tipo de problema se trata de relacionar


las fórm ulas de áreas. Así, por Herón de
Alejandría :

S = >/ 9 ( 9 - 5 ) ( 9 - 6 ) ( 9 - 7 )

S = 6 / 6 cm

5 + 6 + 7
P= r P= 9

C álculo del in ra d io r : Sabemos : p .r = S

Luego : 9 r = 6 y¡
^ 66 /. r = —j — cm

Cálculo del c irc u n ra d io R :


abe _

A partir de : ü

Sustituyendo valores numéricos : 5 x 6 x 7 = q J q , de donde : R = cm

538
19) Halle el área de un triángulo isósceles ABC, sabiendo que AB = BC = 30 cm. y que la
— AE 1
perpendicular a BC, trazada en su punto medio M corta a AB en E y que - —

B
Solución
Según el gráfico, como AB = 30 ,

Luego : 6a = 30 a = 5

Entonces EB = 25 y en el A EMB,
hallamos en seguida EM = 20 (Teorema
de Pitágoras ).
Los triángulos ABC y EMB, tienen en
común el Z B ; por lo que :

S Ab c ab • BC EM . BM ( 20)(15)
(1 ) S ie n d o : SEMB =
Semb EB . BM

SEMB = 150 cm2 y con : AB = BC = 30 , EB = 25 y BM = 15

S abc _ ( 30)(30 )
al sustituir en (1 ) : 1 5 Q c m 2 " ( 25 ) ( 1 5 )

De donde : SABC = 360 cm2

20 )

Se tiene un terreno constituido por cuatro


zonas : tres de ellas son cuadradas, de
áreas 26 H a ., 18 H a ., 20 Ha. y la cuarta es
triangular.
Hallar el área de esta última zona.

En este problema a pesar de conocer las


longitudes efe los tres lados no es'convenien-
te usar el teorema de Herón, por los radica­
les.
Tracemos la altura BH.

Luego : SABC = AC • BH .......... (1 ).


2
Para calcular BH, hallamos previamente AH,
con el Teorema de Euclides en el A ABC :
( / 2 0 ) 2 = ( . / Í 8 f + ( / 2 6 ) 2 - 2 ( / 2 6 ) AH

539
12
De donde, al despejar : AH =
/2 6

Ahora, en el A A H B : B H = ^ a B2 - AH2 -> BH= 18 (2 )


y[26

Finalmente, como AC = 26 , con ( 2 ) en (1 ) :

J26 18
-------- X —= =
ABC V26

Rpta: ABC
= 9 Ha.

21) En un triángulo ABC, de lados AB = 6, BC = 10 y AC = 14, se ha inscrito una


semicircunferencia, cuyo diámetro se encuentra contenido en el lado AB.
Hallar la longitud de! radio de esta semicircunferencia. ( UNI - 87 - 1).

Solución

* Por á re a s :

^ boc + ^ coa ~ ^ACB

10. r 14.r = s
ACB

(i)
12

C álculo del área SACB, Calculemos previamente el semiperímetro, para hacer uso del
Teorema de Herón :
10 + 1 4 + 6
p = ----------------- -+ p = 15

Luego :

S acb = V 15( 15 - 1 0 ) ( 15 - 1 4 ) ( 1 5 - 6 )

» •
ACB = 1 5 / 3 , en (1 ) : r =

Ftpta: r=j/ 3

22) El inradio de un triángulo mide 4cm. y la circunferencia inscrita determina sobre uno de los
lados, segmentos de longitudes 6 y 8cm. Hallar el área del triángulo.

540
Solución
B
Sea el A ABC, con las longitudes conocidas
sobre el lado BC. Luego, por tangentes
iguales trazadas desde un mismo punto, com­
pletamos el gráfico adjunto. Relacionemos
fórmulas de áreas, para el A ABC. Así, con
expresión del inradio :

S = p .r —» S = (m + 6 + 8 ) . 4

S = 4 ( m +14 ) .......... (1 ).

Además por Herón : S = y¡ p ( p - a ) ( p - b ) ( p - c )

Siendo : p - a = m, p - b = 6 y p - c = 8

Luego, en la expresión de Herón : S = m + 14) ( m ) ( 6 ) ( 8 ) ............( 2 )

En seguida, igualando los segundos miembros de ( 1 ) y ( 2 ) :

4( m + 14 ) = m + 14 )( m )( 6 )( 8 )

Elevando al cuadrado y luego de simplificar, hallamos : m = 7.

Finalmente, para el área, reemplazamos este valor de m en ( 1 ) :

S = 4(7+14)

Rpta: S = 84 cm2.

3) El área de un triángulo ABC es 24 cm2. M, N y P son puntos de los lados AB, BC y AC,
respectivamente, de modo que AM = BM, NC = 2 BN y AP = 3 PC. Hallar el área del
triángulo MNP.

Solución
Dato : SABC = 24 cm2

Del gráfico :
^MNP = ^ABC " ( ^AMP + ^MBN + ^NPC ) *** ( ^ )*
* Por tener común el Z A :
S Amp _ AM . AP _ n . 3f
S Abc 2n . 4 f

De d o n d e : S AMP = ± ( 24 cm2 ) -> AMP


= 9 cm2 ............( 2 )
8

^MBN _ _ rc . í
* Para el Z B : BA . BC 2n . 3?
ABC

541
1
Despejando : SMBN = - SABC = i (2 4 cm2 ) -» SMBN = 4 cm2 (3 )

SNPC CN . CP _ 2 1 . f
Con el Z C :
ABC C B . CA 3t.4i

Luego : SNPC -- —
6 S ABC ^ SNPC = 4 cm2 (4 )

Sustituyendo el dato, ( 2 ), ( 3 ) y ( 4 ) en ( 1 )

MNP = 24 - ( 9 + 4 + 4 )

R P ta : S MNP = 7 Cm 2'

24) En un triángulo PQR , la mediana QM corta a la ceviana interior PE en el punto A. Siendo


ER = 2EQ y el área del A QAE , 2 cm2. Hallar el área del triángulo PQR.

Solución
Considerando el gráfico adjunto, los triángu­
los AER y QAE, tienen la misma altura desde
el vértice A.
Luego, las áreas son entre sí como sus
bases : SAER = 2 SQAE
Y = 4

Además, por ser AM mediana del A APR,


^ apm = ®amr = ^ y en A PQR . SP0M = SMQR
—» Z + X = X + Y + 2 Z=Y+2->Z =6

También, para los triángulos PQR y QPE , que tienen la misma altura desde P, como :
QR = 3 ( E Q ) : S pqr = 3 (S QPE)

Esto es : SP0R = 3( Z + 2 ) S ^ = 3( 6 + 2 )

Rpta: SPQR = 24 cm2.

25) En el interior de un triángulo rectángulo ABC, recto en B, isósceles, se toma el punto P y se


trazan PA, PB y PC, siendo : Z PAB = Z PBC = Z PCA .
Si PB = 5cm. , hallar el área del triángulo ABC.

Solución
Del gráfico, como : Z BAC = Z BCA

Ó' = ó

Los triángulos PAC y PCB son semejantes.

542
Luego :

PA _ pC _ a / ^ PC
, de d o n d e ; ^ = J~2
PC 5 a 5 v
. PA «—
PC = 5 / 2 y luego: ^ - = V 2 -» PA = 10
PC
B
A dem ás: Z ABP = 90° - P en A APB,
Z A P B = 90°. Con el Teorema de Pitágoras

AB = PA + PB

a2 = 102 + 52 a2 = 125
a .a
S ie n d o : S ABC

Rpta: SABC = 62,5 cm2

Tomando como diámetro el lado CD de un cuadrado ABCD se dibuja interiormente una


semicircunferencia. Luego se traza AE tangente a CD. Siendo AB = i , hallar el área de la
región que encierra el triángulo ABE. ^^
' £
Solución n -

Prolongamos AE hasta N. Por tangentes


desde un mismo punto :

A -> AE = AD = £ ; N -> EN = NC = n

Luego, en A ABN :

AB = t , BN = ( i - n ) y AN = ( t + n )

Por P itágoras:
/
AN = AB + BN O

( i + n ) 2 = ?2 + ( i - n )

£
De d o n d e : n = —
4 \

Ahora bien , los triángulos ABE y ABN tienen la misma altura trazada desde el vértice B ,
luego :
/
^abe _ AE ^ 5
ABE ABN (1).
ABN AN \

Siendo : A N = £ + n = / , + — AN = — t
4 4

543
y s ABN

Sustituyendo en (1 ) :
/ \

ABE

v 4 J

c _ 3 ,2
Rpta: S abe *

27) Sobre los lados AB, BC y AC de un triángulo ABC, se tienen los puntos E, F y L,
respectivamente, de modo que : EF 11 AC y FL 11 AB.

Demostrar :

solución
Del gráfico, los triángulos ABC, EBF
y LFC son semejantes , por lo que :
S ABC_
ao/"* _ S coc _ ^LFC
°EBF
2 =2 “
AC EF LC

Extrayendo raíz cu adrada:

V S ABC _ V S e BF _ LFC
AC EF LC

Con propiedad de proporciones :

V s a b c _ v ^ e b f + V S LFC
, siendo : EF = AL .\ EF + LC = AC
AC EF + LC

De donde : ^ SABC = ^J S^ep + ^ S l f c

28) Demostrar que si por un punto interior a un triángulo se trazan paralelas a los tres lados, éstas
determinan tres regiones triangulares de áreas Sv S2 y S3, que con relación al área total S,
cumplen :

Solución
De la figura, observamos que los triángulos ABC,
EDP, PMF y NPQ son semejantes, luego :
S _ Si _ S2 _ S3

AC EP PF
2 ~ 1=2
NQ

544
Al extraer raíz cuadrada :
y fs _ s7 _
AC- " EP " PF NQ

Con propiedad de proporciones :

>[$ _ & + V~^2~ + s ¡e, = AN y PF = QC EP + PF + NQ = AC


AC EP + PF + NQ

Luego : Js = + /§

Los lados de un triángulo ABC, miden : AB = 21 cm., BC = 10 cm. y AC = 17 cm. La


circunferencia exinscrita relativa a BC es tangente a este lado en el punto F y a las
prolongaciones de AB y AC en Q y T. Hallar el área del triángulo QFT.

Solución
Sa b e mo s que si “p ” es el
semiperímetro del A ABC, entonces:
AQ = AT = p ,

como : p = t 1Z = 24

AQ = AT = 24

Luego :

BQ = BF = AQ - AB = 24 - 21
BQ = BF = 3
y CT = CF = AT - AC = 2 4 - 17
-» CT = CF = 7

^QFT “ ^AQT ‘ ^ABC ' ^FBQ ' ^FCT <1 >-

Para hallar el SAQT , previamente calculamos el SARr


ABC , por el Teorema de Herón :

Sabc = V(24)(24“17)(24“21X24"10) = V(24H7)(3)(14)


SABC = 84 cm2 .................................................. ( 2 ).

En seguida, como los triángulos AQT y ABC tienen en común el Z A :

AQT _ ( A Q ) ( A T )
Sabc (AB)(AC)

S a q t . ( 24 )( 2 4 )
SAQT = 135,53 cm2 . . . ( 3 ) .
84 ( 21)(17)

545
Los triángulos FBQ y ABC tienen dos ángulos suplementarios ( Z F B Q + Z ABC = 180°),
por lo q u e :

FBQ _ (BQ)(BF) S fb q _ ( 3 )( 3 )
FBQ
= 3,6cm2 .......... ( 4 )
SABC " ( BA )( BC ) 84 ( 21)(10 )

También, los triángulos FCT y ABC tienen dos ángulos suplementarios


( Z FCT + Z ACB = 180°), luego :

FC T (CF)(CT) ;F C T _ ( 7 )( 7 )
SFCT = 24,21 cm2 (5).
SABC (CB)(CA) 84 ( 10 ) ( 1 7 )

Finalmente, sustituimos ( 2 ), ( 3 ), ( 4 ) y ( 5 ) en ( 1 ) :

Rpta: SQFT = 23,72 cm2

30) Hallar la expresión para el área de un triángulo, en función de las longitudes de sus medianas.
( Teorema de Herón de Constantinopla o el Joven ).

Solución
Sean : AN = ma , BM = mb , CQ = mc . Tracemos por C, paralela a AN hasta cortar a la
prolongación de BM en P.
2 2
El área del A GPC, es 2S 1 y las longitudes de sus lados son : CP = — ma , GP = — mb

y GC = — mc . Además este triángulo es semejante al que tiene por longitudes de lados ma,
O

n \D y' rnC . Llamando Sm


rn
el área de éste último y* haciendo : m = ma + mb
2
+ mc

Se tiene : Sm = ^ m ( m - ma )( m - mb )( m - mc )

/
—m
B qpc 3 /
A dem ás: S Gpc - — S m
m ( ma )

Esto es : 2S. = — Sm . Siendo el área del


1 9
\
S,
i ^
A ABC, equivalente a 6S1 :
P*

31) interiormente a un AABC, se toma el punto P y se trazan las perpendiculares de longitudes


xa , ^ y xc a los lados BC, AC y AB, respectivamente.
Siendo ha, hb y hc , las longitudes de las alturas del triángulo, demostrar que :

+ *b + c _
= 1

546
D em ostración

Se tiene . S BPC + SAPC + SAPB - SABC (1) .

B Los triángulos BPC y ABC tienen la misma


base B C , luego por IV .2 , sus áreas son entre
sí como las alturas :

BPC _ xa
ABC

De donde : S BPC = ) s ABC

xb
Análogamente : SAPC = ( ^ )• SABC
b

APB = ( ^ ) S ABC
h
/ \ \ / \
Xo xb
Sustituyendo en (1 ) : ABC + ABC + ABC = s ABC
V ha j / v hc y

Simplificando S ABC ' h hi h

Dado un triángulo ABC, sobre AB y BC se toman los puntos E y F, respectivamente, de


modo que EF sea paralelo a AC. Luego en AC se ubica un punto cualquiera P y se trazan
PE y P F . Hallar el área del triángulo EPF, sabiendo que las regiones EBF y ABC son de
4 y 25 cm2, de área.

Solución
SEBF = 4 cm2
D a to s:
S abc = 2 5 cm 2

Incógnita : S EPF

Del gráfico, los triángulos EPF y EBF


tienen en común el lado E F .
Luego, las áreas serán entre sí como
las respectivas alturas:

—E-p- = , de donde : S EPF y ( 4 cm2) (1)


S ebf x \ X

Por otro lado, los triángulos EBF y ABC son semejantes. Luego :
✓ \2
(
a

Sabc ^ y I
-r = -------- , ( relación de alturas, al cuadrado ).
b EBF v x ;

547
25 x +y
esto es :
/
3
Al extraer raíz cuadrada y d e sp e ja r: — (2 ).
x 2

Sustituyendo, en seguida ( 2 ) en ( 1 ) : S EPF ( 4 cm2)

Rpta: SEPF = 6 c m 2

33) En un triángulo ABC se traza la ceviana interior AE y luego EF paralelo a CA. ( F en AB )


S i : SABC = 96 cm2 y SFBE = 54 cm2. Hallar SAEB

Solución
* Los triángulos AEB y ABC tienen la
misma altura trazada desde A, entonces:
B
Saeb = BE
Sabc BC (1 )

Además como los triángulos FBE y ABC,


son sem ejantes:

FBE BE ‘FBE _ BE
. . . . (2)
ABC BC ABC BC

Igualando los primeros miembros de las


expresiones ( 1 ) y ( 2 ) :

AEB FBE —»
ABC ABC

sustituyendo datos : SAEB = J ( 96 cm2 54 cm2 j

Rpta: SAEB = 72 cm2

34) En un triángulo acutángulo ABC, de ortocentro L, sobre la prolongación de la altura BH se


toma el punto Q, de modo que AQ y QC sean perpendiculares.
S i : SABC = 1 6 c m2 y SALC = 9 c m 2 . H a lla r: S AQC

Solución

Se tienen :
o _ A C . QH
AQC “ ----------------- (1 )

548
AC . BH
ABC (2).
2

A C . LH
ALC (3).

Siendo los triángulos AHL y BHC


semejantes ( Jf LAH = f HBC )
entonces:
AH LH
AH . HC = BH . LH
BH HC

Además en el A AQC, por relaciones


m étricas:

QH = AH . HC y con lo anterior:

QH = B H . LH ............( 4 ) .

A fin de relacionar las expresiones de áreas, efectuemos operaciones convenientemente


Así, multiplicando miembro a miembro ( 2 ) y ( 3 ) :

AC . B H . LH
^ABC • ^ALC , y con (4):

AC .Q H
^ABC ‘ ^ALC

AC . QH ^
esto es . SABC . SALC Con ( 1 ) . SABC . SALC = <SAQC)2

o m e jo r: Propiedad

Sustituyendo datos :

Rpta: SAQC = 16 cm2 )( 9 cm2 ) = 12 cm;

35) La longitud del lado del cuadrado ABCD es


“ a T o m a n d o como diámetros los lados
AD y CD se han d ib u ja d o dos
semicircunferencias , trazándose luego las
tangentes BE y BF. D

Hallar el área de la región triangular EBF

549
Solución

Prolongamos BE y BF hasta interceptar a


los lados AD y CD en los puntos P y Q.
Luego trazamos BD, cortando a PQ en M.
Por ser tangentes trazadas desde un mismo
punto : BE = BC = a = BA = BF,
PE = PD = x = DQ.

Los triángulos PBQ y EBF, son isósceles


semejantes. Luego :

S ebf _ B E2
(1 )•
PBQ BP

* C álculo de x : En el A BAP, con el teorema de Pitágoras :

BP = AB + AP ( a + x )2 = a2 + ( a - x )
a
De donde : x = —
4

Entonces : PQ = x / 2 =
4 2 8

y como : BD = a J 2 BM = ^ a / 2 , con lo que en seguida


8

( a j2 ] í ? a / 2
00

PQ . BM 4 7a£
V / (2)
PBQ PBQ “
32

Además :B P = a + x = a + — —> BP 5a
(3 )
4

Finalmente, con ( 2 ) y ( 3 ) en (1 ) :

EBF
' 7a2 ^
í 5a l
( 32 > , 4 ,

7a¿
Rpta: SEBF =
50
La circunferencia inscrita a un A ABC, es tangente a los lados en los puntos P, Q y M,
respectivamente. Siendo r y R , inradio y circunradio del triángulo , respectivamente.
Demostrar q u e :
S PQM r
S a BC 2R
B
D em ostración

Siendo I el incentro del A ABC, se tiene :

SpQM “ SqiM + SM|p + Sp|Q d )

Como los ángulos Z MIQ y ZC , son suple


mentarlos , se tiene :

S q im IQ • IM
Q IM “ ABC
ABC BC . AC BC . AC

Análogamente :
2 j.2
ABC
S mip = AB . AC SabC ’ SplQ " AB . BC

en (1 ) :

r2 ^ c* ^
BC . AC SabC + AB . BC SabC + AB . AC SabC

1 1 1
^ - r 2 Cí + +
PQM “ 1 ABC ^ BC . AC AB . AC AB . BC /
Efectuando la expresión dentro del paréntesis:
AB + BC + AC
SpQ M - f2 S A BC
A B . BC . AC / (2)

Ahora bien, recordemos que :


( AB + BC + AC ' 2 SA B C
r = SA B C —> AB + BC + AC —
/
AB . BC . AC = S AB . BC . AC = 4R S ABC
y • ABC
4R

Sustituyendo estas últimas dos expresiones, en ( 2 ) :


\
2S A B C
S rqm - r2 ^ABC ^ r - 4R SABC }

De donde, efectivamente :
$pqm „ J_
S abc 2R

?>51
B
En la fig u ra :

área ( ABC ) = S
AE = 2EB , BF = 2FC
y GC = 2AG.

H a lla r: área (PQR)

Solución
Se tie n e :
^PQR = $AEC ’ ^AEPQ “ ^AQG ' ^QRCG (1 )
Cálculo del S AEC • B

Los triángulos AEC y ABC, tienen la misma


altura que parte de C , luego :
s aec _ AE
ABC AB

AEC _ 2c
S aec - “ S (2 )
3c
C
Cálculo de las áreas AEPQ , AQG y QRCG :

Por la naturaleza de la figura : SEBP = SAQG = SRFC

Para hallar SEBP, necesitamos conocer la relación entre BP y BG. Para ello, con el teorema
de Menelao en el A ABG :
A E . B P . GC 2 c . B P . 2b BP 3
= 1
E B . PG .CA c . PG . 3b PG 4

Luego : BP BP 3
(n )
BP + PG 3 + 4 BG 7

En seguida, como los triángulos EBP y ABG tienen en común el ¿ ABG

^ EBP = S§§E_ = , con (n) y el gráfico :


S abg ae ■SG SABG AB BG
SEBP _ c 3 1
SEBP “ ~ SABG (3 )
S abg 3c 7

Pero, por tener la misma altura desde B :


ABG _ AG SABG = _b_ 1
S ABG - — S (m)
ABC AC S 3b

2
Sustituyendo en (3): SEBP = ~ -> EBP S ... ( 4 ) .
{ 3 21
_1_
Entonces AQG S (5 ) y SAEPQ ^ABG " ^EBP " ^AOG
21

Saepq = g - S — S- — S
21 21

esto e s : S AEPQ (6 )
21

Análogamente S QRCG S , con ( 2 ) , ( 5 ) y ( 6 ) ,


21

en (1 ) : SPQR = - S — S- — S- — s
21 21 21

Rpta

En la figura, M, N y Q son puntos de


tangencia. Si R eselcircunradiodel A ABC
y r3 exradio relativo al lado B C ; demostrar,
*

que:
S m NQ _ ra
S ABC 2R

Solución

Llamemos a , b , c , longitudes de los


ladosdelAABC y peí semiperímetro.
Se tiene :

^MNO ~ ^MNQE ‘ ^MEQ

^MNQ ” ^MNE + ^NEO ’ ^MEO


Dividiendo cada término entre S ABC

Smnq _ SMNE SNEQ MEQ


(I).
ABC ABC ABC ;A B C

Además, por tener, respectivamente, un ángulo congruente :


2
SMNE „ EM . EN S MNE _ ( ra )
(H )
$ abc ^A • BC S abc ac
y:

Sneq _ EN . EQ NEQ _ ( ra )
(III)
Sabc ^ A x CB 'A B C ab
También ; Á + MÉQ = 180

Smeq _ EM . EQ °M E Q (U
_ _____ ( I V) .
S abc AB • AC S ABC bc

Sustituyendo ( I I ), ( I I I ) y ( IV ), en ( I ):

°M N Q ( ra )2
_____ , ( ra f ( ra )

S ABC ac ab bc

Efectuando : ^ MNQ = ^ ra ^ ( b + c - a ).
ABC abe

abe
Pero :b + c - a = 2 ( p - a ) y S ABC abe = 4R S ABC
T r"

Luego : ^ mnq _ , Í j L )__ . 2 ( p - a ) ; además : S ABC = ( P - a )L


ABC 4R S A B C

Entonces MNQ ( ra ) 2(P-a )


Sabc 4FÍ( P ” a ) r¿

MNQ
De donde : 2p ....... Lq .q •el•
>ABC
D

39) Del gráfico, marcar lo correcto :

A) X = V Y . Z B) Y = J X . Z

C) Z = , / X . Y D) X2 = Y 2 + Z2

E) N.A.

Solución

* A ABD, AC es bisectriz Según el prob. N°2 de esta sección Y AD


(1 ).
Z AB

X AD ■/~X _ AD
* A ADE - A ABC , (2 )
AB ■J~Z ~ AB

Y _ y[X
De ( 1 ) y ( 2 )
Z = J z

Rpta: (B).

554
40) Del gráfico , demostrar
que :

s r

Si

Solución

Si R es el circunradio del A ABC, sabemos por los problemas números 36 y 38 de esta


sección , que :
S r S! r,
ABC 2R ABC 2R

Al dividir miembro a miembro, queda efectivamente :

— = L
1 *i

41) Del gráfico, demostrar


que:

s3
S i + —i _ S2 , S<
<3 **2 u

Solución

Del problema anterior, tenemos S 1 = $MNQ y _§3 NPQ


*1 r r3 r

Sumando m.a.m. : S3 MNQ + s NPQ S i + S3 MNPQ (U


«1 r3 r3 r

S2 S mn P l( S4 MQP
También : ~ - : y
r4

MNP + S MQP $2 t $ 4 _ b MNPQ ............ ( 2 ).


Sumando m.a.m. : + S4
r2 r4

De (1 ) y ( 2 ) : Si + _JL
21 S = £ 2_ + 2 ± .......... I.q.q.d.
ri r3 *4 r4

555
42)
En la figura adjunta, demostrar que la
inversa del área del triángulo en la
circunferencia inscrita al /. ABC, es
igual a la suma de las inversas de las
áreas de los triángulos en las circun­
ferencias exinscritas.

Es d e c ir:

Solución
Por el problema anterior, número 36, sabemos que :
^abc _ 2R_ . SABC = 2R_ SABC _ £ R
S 2 rc S3 ra S4 rb

Sumando miembro a miembro todas estas expresiones :


/ \ /
1 1 1 ^ ( 1 1 1 ^
+ ---- -f ---- = 2R +—
ABC
ls 2 S3 S4 > V rc ra rb J
Escribiendo el equivalente en el paréntesis del segundo miembro :

ABC
1
— + 1
+ —1 = 2R ^ 1
V. r y
1
/

1 1 1 2R
Esto e s : SABC b ---- +
V S2 S3 S4 ,

1 1 1 - . -ABC. f p0r el problema N° 36


ABC
— + — + —
/ Si

1
De donde : _L +JL + _! ................ l . q . q . d -

R
43) C a lcu la r el área del pentágono
ABCDE, si el lado del cuadrado PQRS
es “a”.
M
PABR ha sido trazado con centro en
“S".
PS y RS son diámetros
55(r
Solución
Con los trazos indicados:

■U

“ D ” centro del cuadrado PQRS

^ = ^ ( AaQDC - Aa q jb ) ......................... ........ (1 ).

( a ) AaQDC = ?? Por tener Z común ( LQM = DQC = a ) :

a aQ DC _ CQ . QD
A aqlm LQ . QM

a aqdc _ 2 ^ v ' 2 y

— . 3 —J~2 . ~J~2 3 - J 2 .- /5
2 4 4 4 2

a aq d c = — ( 5 - /5 ) .......( 2 ) .
4 0' v '

( b - ) \ QJB = ? ?
A QJB - A QL M

a aq jb
( Q B )2
. ( 3 ).
:
I

ro

A aq lm
Q

* C álculo de QB = ?? : Construyendo el cuadrado RTVS de lado “a” y trazando RV (centro


“S” ) :

QV = Diagonal del rectángulo PQTV = a


En la semicircunferencia de centro “S” ( Teor. de la tangente ) :

( QR )2 = QB . QV

557
a2 = QB . a f s

QB = | ^ 5 • Reemplazando en (3 )

AQJB

3a
AQJB (4)
100

De ( 2 ) y ( 4 ), en (1 ) : X = 2 (5 - / 5 ) - 3
40 100

Rpta: X =— (19- 5J 5 )
100' V 1

558
r
V. REGIONES CUADRANGULARES

A manera de introducción para este apartado veamos algunos problemas que nos muestran ,
propiedades particulares y generales, para luego indicar las fórmulas correspondientes a
los cuadriláteros.

V.1. En un trapecio ABCD, de bases BC y AD , las diagonales AC y BD se cortan en el pun­


to N.

Demostrar que : SABN = SCND

Solución
De la propiedad IV .4, como los triángulos
ABD y ACD son equivalentes por tener
la misma base AD e igual altura h :
®ABD = ^ACD

^ABN + ^AND = ^CND + ^ANO

S.PM = SrNjrt | Para todo trapecio.

V.2. En un trapezoide ABCD, las diagonales AC y BD se interceptan en el punto P


Demostrar que : SABP . SPCD - 9 Q
” BPC • APD

Solución

B Considerando el gráfico adjunto, observa­


mos que los triángulos ABP y BPC tienen
la misma altura trazada desde el vértice B.
Luego, sus áreas serán entre sí como las
respectivas bases ( IV.3 ) :
S ABP _ AP
Sbpc PC (1 ).

Análogamente para APD y PCD :


D
S APD _ .......... ( 2 ) .
SpCD PC

9 c - Q C En todo cuadri­
De (1) y (2) ABP _ ^APD • •
ABP ' PCD “ BPC ’ APD
látero convexo.
BPC PCD

V.3. En un trapecio ABCD, de bases BC y AD, las diagonales AC y BD se interceptan en el


punto E.
Demostrar q u e : ( 1 ° ) S AEB = V SBEC. s AED

(2 o ) J s ABCD = V s BEC + V s AED

559
Solución
( 1o ) Del problema V . 1 . , sabemos que B
^ - Q
AEB “ CEO

Y con el problema a n te rio r:


q _ q q
AEB ’ CED “ BEC ‘ AED

(^AE b )2 = ^BEC ' ^AED


D
^AEB ” V ®B E C • S AED

( 2o ) Para el trapecio : SABCD = SBEC + SAEB + SCED + SAED

S ie n d o : SCED = SAEB ^ABCD “ ^BEC + ^AEB + ^AED

Con lo demostrado : SABCD - SBEC + 2 SBEC . S^ ed + $ aed

S ABCD ~ ( ij$BEC + V S AED ) V ABCD = ^[§BEC + V S AED

* Estas dos relaciones son válidas para to d o trapecio.

V.4. El área de un cuadrilátero convexo es 40 cm2. Hallar el área del cuadrilátero que tiene por
vértices los puntos medios de los lados del primero.

Solución

Del gráfico . SMNEF - SPQRT - ( SMQN + SFTE + SPMF + SNRE) (1 )•

Con la propiedad IV.6. :

APQR -> SMQN= % ^

Efectuando la suma indicada :

T + sFTE PQR + S p T R
MQN “

Esto es : PQRT
MQN + s FTE “
(2 ).

En forma análoga : SPMF + SNRE PQRT (3 ).

_ ^P Q R T
Ahora, reemplazando ( 2 ) y ( 3 ) en ( 1 ) : 'M N E F ( Propiedad)

m )
Para el problema :
S PORT = 4 0 Cm2

Rpta: SMNEF = 20 cm2

* Nota.- Además, recuerde el lector que el cuadrilátero MNEF es un paralelogramo.

L5. El área de un cuadrilátero convexo es 60 cm2. Hallar el área del triángulo que tiene por
vértices los puntos medios de tres lados del cuadrilátero.

Solución
Sea el cuadrilátero ABCD en mención y
MNR el triángulo.
Sabemos que MNPR es un paralelogramo,
por lo que :
o _ S MNPR i 1^
b MNR - .................. I'I'

Además, del problema anterior :


S
- °ABCP ........... ( 2 ) .
MNPR
2

Reemplazando ( 2 ) en ( 1 ) : ( P ropiedad).

En el problema ^ abcd ~ cm2

R P ta : S MNR = 1 5 C fT |2

1.6. En un cuadrilátero convexo ABCD, M es punto medio de la diagonal AC. Siendo SBDC > SABD,
demostrar que :

B
Demostración

Del gráfico :

BMD ~ ^BOC " ( ^BMC + ^DMC ) ••• ( ^ )

Siendo M punto medio de AC :


Sabc o _ Sadc
BMC - y
i .
s DMC “

L uego:

BMC + SDMC - S ABC + S ADC

SBMC + SDMC _ ^ABC + S ADC _ S abCD = ^ABD + S BDC


2

561
—» _ S ABD + B bdc (2 )
^BMC + ^D M C

sustituyendo ( 2 ) en ( 1 ) : S BMD -” ^ BDC


f Sadi*»
ABD + ^ S
°BDC
\

De donde, en efecto : S BMD = ^ bdc $ abd

V.7. En todo trapecio el área es igual al producto de la longitud de uno de los lados no paralelos
y la distancia del punto medio del lado opuesto, al primero.

Demostración
* Consideremos el trapecio ABCD, donde
M es punto medio de AB y MH 1 CD.
Demostraremos q u e :

S ABcd = CD . MH

Desde luego, sabemos que :

ABCD
= MN . C Q (1 ) , donde MN
es la mediana y CQ la altura del trapecio.

Siendo: ZM HN = Z D ; AMHN - ACQD, luego: — = — ; MN . CQ = CD . M H (2 )


CD CQ

Reemplazando ( 2 ) en ( 1 ) : SABCD = CD . MH

V.8.
En todo trapecio, el área del triángulo que tiene
por vértices: al punto medio de uno de los lados
no paralelos y los vértices del lado opuesto, es
la mitad del área total.

* Así, para el trapecio ABCD, se tiene :

ABCD
D MCD

Demostración

Llamando d la distancia de M a C D , se tiene : S—


MCD
= (C D J . d d )•

Pero, por lo a n te rio r: ( C D ) . d = S ABCD (2 )

De ( 2 ) en (1 ) : SMCD = A|£P. . Como queríamos demostrar.

562
V.9. En todo cuadrilátero circunscrito ( o circunscriptible ) a una circunferencia, el área es igual
al producto del semiperímetro y el radio de dicha circunferencia.
La demostración es sencilla. En el gráfico :

^ABCD = ^AOB + ^BOC + ^COD + ^AOD


Luego :

O AB . r + ---------
--------- B C . r + C D . r + ---------
AD.r
abcd 2 2 2 2

' AB 4- BC + CD + AD '
ABCD - . r ,

V /
siendo la expresión en el paréntesis el
D
semiperímetro p : ABCD
= p. r

V.10. Teorema de Brahm a - Gupta.- El área de todo cuadrilátero inscrito o inscriptible, es igual
a la raíz cuadrada del producto de las diferencias del semiperímetro con cada lado.

D em ostración
Así, para el cuadrilátero ABCD, donde :

p = a + k + 0 + d , demostraremos
2
que :

ABCD = V ( p - a ) ( p - b ) ( p - c ) ( p - d )

En efecto, sabemos que Z DCE = Z A y Z C D E = Z B, por lo que los triángulos ABE


y CDE son semejantes.
o
’ABE _ S abe _ $ cde _ a 2 - c2
Luego :
CDE cde

/ 2 ? N
ABCD _ a ~ c a¿ - c
De donde : s ABCD - CDE ( 1)
ScDE C2

C álculo del área SC0E : Para el A CDE, haremos uso del teorema de Herón :

\
m + n- c n+ c - m
cde - (2)
/ j

m n
De la sem ejanza‘entre los triángulos CDE y ABE : ^ +
b + m

De lo que deducimos : ( m + n ) = c ( ^ + ^ ). ; ( m - n ) = 5 .(.j__ Ü


a - c a +c
563
Luego:
m + n+ c /

a - c\ /

m + n- c
a - c /
/ (*)
m + c - n a + c + d -b
a + cV

n+ c - m / a + b +c - d

a + c

Con ( * ) en ( 2 ) , ordenando :

/ /a + b + c - d
b + c + d - a a + c + d - bY a + b + d - c
'CDE “
2
- cr2 \
(a¿
)'

o en función del semiperímetro p del cuadrilátero ABCD :


/ \
CDE 2 2 V (p ~ a )(p - b )(p - ° ) ( p - d ) ( 3 )•
\ a¿ - c¿ /

Finalmente, sustituyendo ( 3 ) en (1 ) :

abcd = V (p-a)(p-b)(p-c)(p-d )

V.11.Teorem a de Leudesdorf.- El área de todo cuadrilátero bicéntrico es igual a la raíz cuadrada


del producto de las longitudes de sus cuatro lados.

Dem ostración

Por el teorema anterior, tenemos :

S = v(P _ a )(p -b )(p -c)(p-d) ( 1)


V con el teorema de Pitoth
a+c = b+d

a+b+c+d
S iendo: P =

Luego: p a+c
P b+d

Entonces: p - a = c, p-b d,
p-c = a y p-d b

Sustituyendo en (1 ) : S = ^ abcd

564
V.12. Teorema.- En todo cuadrilátero, el área se puede expresar como el semiproducto de las
longitudes de sus diagonales y el seno del ángulo formado por ellas

D em ostración

( a ) Si el cuadrilátero es convexo :
Considerando ABCD, por el problema
V.5. .anterior, sabemos que :
c _4 c
ABCD “ H MNR (1 )•

Por la expresión trigonométrica para el


área :

SMNR = 1 MN . MR. ( sen Z RMN ).... ( 2 )

Como MN y MR son bases medias en los triángulos ABC y ABD, luego :

MN = — y MR = § 2 .. Además MÑ II AC y MR I BD , por lo que Z RMN = a

Sustituyendo en ( 2 ) : S MNR = — AC . BD. Sen a (3 )


8

Finalmente, de ( 3 ) en ( 1 ) :

( b ) Si el cuadrilátero es cóncavo :

B Sea ABEF el cuadrilátero, donde BF y AE


son las diagonales.

* Se deja la demostración al lector


S ugerencia:
^ABEF “ ^A B E " ^A F E

V.13. C orolario.- En todo cuadrilátero de diagonales perpendiculares, el área es igual al


semiproducto de las longitudes de dichas diagonales.

'A
M

_ F A .M G _ EM . FN
FMAG EFMN

565
COMPARACION DE REGIONES CUADRANGULARES

Las áreas de dos paralelogramos son entre sí como los productos de sus bases y respectivas
alturas.

Si dos paralelogramos tienen congruentes un lado, entonces las áreas son entre sí como sus
respectivas alturas.

Si dos paralelogramos tienen congruentes una altura, las áreas son entre sí como son sus
respectivas bases.

Dos paralelogramos que tienen congruentes un lado y sus respectivas alturas, son
equivalentes.

Las áreas de dos trapecios son entre sí como los productos de sus medianas y alturas.

Si dos trapecios tienen sus medianas congruentes, las áreas son entre sí como las alturas.

Si dos trapecios tienen sus alturas congruentes, las áreas se encuentran en la misma relación
que las medianas.

Dos trapecios que tienen medianas y alturas, respectivamente congruentes son equivalen­
tes.

Si dos cuadriláteros son semejantes, las áreas son entre sí, como el cuadrado de la
respectiva razón de semejanza.

( * Demuestre el lector cada una de estas relaciones, con un gráfico en cada caso.
Deduzca así mismo las relaciones para la comparación de áreas en dos rombos).

PROPIEDADES

En todo paralelogramo una diagonal determina dos regiones equivalentes.

A s í' B

En todo paralelogramo al trazar las dos diagonales se determinan cuatro regiones equiva­
lentes.

* En el paralelogramo ABCD , para el


B C A BCD, como CO es mediana :
\

Además BO es mediana del A ABC


s, = s2

Luego: R = £ = R = R = ^ABCD
1 2 3 4 —

En todo paralelogramo, el área de la región triangular deteminada al unir un punto cualquiera


de un lado con los vértices correspondientes al lado opuesto, equivale a la mitad de la del
cuadrilátero.

D em ostración
B
Consideremos el paralelogramo ABCD 4
y P un punto cualquiera del lado B C .
Se tienen :

^APD = ^ y ^ABCD “ ^
!
SAPD = ^ abcd

Al trazar los segmentos que unen los puntos medios de los lados opuestos de todo
paralelogramo, se determinan cuatro regiones equivalentes.

-U- Así, en el paralelogramo ABCD,


Y Demuestre el lector que :
X

W
~ff~
A

Si se une un punto interior a un paralelogramo con los cuatro vértices, se determinan cuatro
triángulos, donde la suma de áreas correspondientes a aquellos que tienen por bases dos
lados opuestos del cuadrilátero, es igual a la suma de las otras dos.

D em ostración

Sea Q punto interior, cualquiera,


al paralelogramo ABCD.

Del gráfico :

AB . QH + ------------
CD . QF
^AQB + ^CQD

Siendo AB = CD, se puede e s c rib ir:

AB . ( QH + QF ) _ AB . H F SABCD
^AQB + ^CQD

Fifi
V

Esto e s : ABCD
^AQB + ^CQO (1 )

En forma a n á lo g a :
- °ABCD (2 ).
^ bqc + ^AQD

De ( 1 ) y ( 2 ) : ^AQB + ^CQD " ^BQC + ^AQD

6.- Si por un punto cualquiera de una diagonal de un paralelogramo se trazan dos paralelas a
los lados, entonces los paralelogramos obtenidos son equivalentes.( Teorema de Gnomon).

* Sea ABCD el paralelogramo y “O”


el punto en mención. B

Demuestre el lector que ;

7.- Toda recta que pase por el punto medio de la mediana de un trapecio e intercepte a las bases,
determina dos regiones equivalentes.

B P
Sea E punto medio de la mediana MN del
trapecio ABCD. Si P y Q son puntos cuales­
quiera de las bases, entonces :

c _ c ABCD
ABPQ " QPCD

D em ostración

Se tienen : SABPQ = ( ME ) . h y SQPCD = ( EN ) . h

Siendo : ME = EN 9 - 9
ABPQ ~ QPCD

8.- El segmento que une los puntos medios de las bases de todo trapecio determina dos
regiones equivalentes.

Así, para el trapecio ABCD,


H y T son puntos medios
de las bases.

S abht = S thcd = ^B C D

56?
El área de la región que encierra el cuadrilátero que tiene por vértices los puntos medios de
los lados no paralelos y dos puntos cualesquiera de las bases de un trapecio, equivale a la
mitad del área de dicho trapecio.

Considerando el trapecio ABCD, don­


de M y N son puntos medios de los
lados no paralelos y además P y R son
puntos cualesquiera en las bases :

c _ ^ABCD
MPNfR “ g

La demostración de las dos últimas propiedades se dejan como ejercicio al lector)

INTERPRETACION GEOMETRICA DEL TEOREMA DE PITAGORAS.- Los antiguos grie­


gos conocían el Teorema de Pitágoras de la manera siguiente : “En todo triángulo, la suma
de las áreas de los cuadrados construidos sobre los catetos es igual al área del cuadrado
construido sobre la hipotenusa".

Así, para el A ABC, donde M, N y Pson


las áreas de las regiones indicadas ,
demostraremos, q u e :

M+N = P

Demostración

Del gráfico : Z EAC = Z BAL, luego A EAC = A BAL , por lo que SEAC = SBAL (1 ).

Además, para el A EAC, la longitud de la altura trazada desde el vértice C es igual a AB. Por
, c A E . AB
lo que : SEAC = ----- ------

De lo que deducimos que : SABDE = 2 SEAC ............( 2 ).

En forma análoga, para el A BAL, la longitud de la altura que parte del vértice B es igual a
AH. L u e g o :

e A L . AH
BAL “ «
^ALQH ~ ^ ^BAL...... ( ^ )*
En seguida, de ( 1 ), ( 2 ) y ( 3 ) :
R
^ALQH = ^ABDE ......... ( 4 )■
Análogamente se demuestra que :

^HQTC = ^BCRF ........ ( ^


Finalmente, sumando miembro a miem
bro las relaciones ( 4 ) y ( 5 ) :

^ALQH + ^HQTC = ^ABDE + ^BCRF


De donde : SALTC = SABDE + SBCRF

E N TER ATE:

Actualmente, se conocen m ás de 360 formas distintas de demostrar eí "Teorema de


Pitágoras". ( Ver el Libro de E. Loom is; "The Pythagorean Proposition”, NCTM, Í 9 7 2 ),
La demostración de James Garfield {1831 -1 8 8 1 ), vigésimo Presidente de los Estados
Unidos, consiste en lo sig u ie n te ;
Sea ABC, un triángulo recto en C, en el cual se demostrará que :
*...
D e m o s tra c ió n : ♦

12) Se toma un punto P de la prolongación Q


—1
de C B , tal que BP = AC y por él se eleva i
una perpendicularidad a CB, cortando i
1
t \ I
en Q a la perpendicular a AB trazada / V
por B. / C v *b

2e) Se concluye: A BPQ '3 A ACB. Por lo


tanto: PQ = a ; BG = c y por condición:
BP = b.

3a) Area BPQ = área ACB = abJ2 y área


ABQ = c2 / 2 .

49) Como el área del trapecio es lá suma de


áreas de los tres triángulos:

a + l \ ab c 2 ab
a + b)=— + — + —
; 2 2 2

De donde : a2 + b2 = c2

570
V

PROBLEMAS RESUELTOS

1) El área de un trapezoide es 18 cm2. Hallar el área del cuadrilátero que se forma al trazar
paralelas a las diagonales del trapezoide, por los cuatro vértices.

Solución
Sea el trapezoide ABCD y A’B’C’D’
el obtenido con las paralelas.

D a to :

S ABCD = 18 Cm2

Incógnita :
q
A'B'C'O'

Como AA’B’C y ACC’D \ son parale-


logramos , sabemos por la propiedad D'
N 3 que . SAA.B.C = 2 (S ABC) y
®acc'd* = 2( SACD) .

Sumando miembro a miembro y antecediendo el factor común en el segundo miembro

^ aa’B’c + ^ acc'D’ ” ^ ( SABC + 3 acd )

esto es :

y con el dato : SA,B.C.D, = 2 ( 1 8 cm2 )

Rpta: 36 cm2

2)
B
Sobre el lado AB del A ABC, se toman “n”
puntos que determinan ( n + 1 ) segmentos
congruentes y por cada uno de ellos se
trazan paralelas a AC. Luego por cada
punto de intersección con BC se trazan
paralelas a AB.
Siendo “S” el área de la región indicada.
Hallar el área total ABC.

Solución

Los triángulos adyacentes a BC tienen por área zL , equivalente a la mitad del área de cada
2
paralelogramo.

571
En (a figura tenemos el análisis de las regiones por filas :

Sabc = ( 1 + 2 + 3 + ... + n ) S + ( n + 1 ) S

El primer paréntesis es la suma de los primeros “ n " números naturales :

_ n( n + 1) f n + 1> f n+ U
ABC “----- + S = ( n + 1 )S
^ 2 j k 2 )

Rpta s
b ABC -“ ( n +p -----
1> s

Hallar el área de la región sombreada, si el


área del paralelogramo ABCD es K.

M y N : puntos medios.

Solución

EMFN es un paralelogramo
Del gráfico :

ABCD
ABMN

C „ K
ABMN ~ —

Además :

o _ ^ABMN K
^NEM “ — :— • «
NEM =
8

y finalmente ® EM FN “ ^ ^NEM = 2

K
Rpta: SEMFN
4
Hallar el área de la región sombreada,
si el área del paratelogramo ABCD
es “Z", donde M, N, P y Q son puntos
medios.

Solución
Dato . SABCD - Z

* Del gráfico, CM es paralela a PA y


BQ lo es a ND.

En A ABL, ME es base media

s¡ ME = a, entonces AL = 2a

Deducimos luego que, en A ATD : LT = AL = 2a y como el A TDP es congruente con


el A MBE , TP = ME = a.

Luego : SEFTL = LT . CH SEFTL = 2a ( CH ) .......... (1 )

Además : SAMCP = AP . CH SAMCP = 5a ( CH ) .........( 2 )

Pero, el área del paralelogramo AMCP equivale a la mitad de la del ABCD

^amcr = Z , sustituimos esto en ( 2 ) :

- = 5a ( CH ) a ( CH ) = (3 )
2 10

Finalmente, con ( 3 ) en ( 1 ) : S EFTl = 2 .


10

Rpta: SEFTL = £
5

En la figura , M y N son puntos


medios de AB y AD. Los lados del
rectángulo ABCD, m id e n : AB = 8
cm y AD = 12 cm.

Hallar el área del cuadrilátero NPOQ.

573
Solución
B
Del g rá fic o :

^NPOQ “ ^NOP + ^NOQ (1 )

S ie n d o :
.c; = SAM0N 6x4
°N P O ~ = ----- ------
4 4

NPO = 6 cm2 (2 )

Para el SNOQt debemos hallar previamente EG. De la semajanza de los triángulos NOQ y
CQD:
EQ ON EQ
EQ = 2
QF “ DG 6 - EQ 8

Luego :
s _ ON . EQ 4x2
NOQ S NOQ = 4 C tT l2 (3 )

Finalmente , con ( 2 ) y ( 3 ) , en ( 1 ) :

S npoq = 6cm 2 + 4cm 2

Rpta: S NPOQ
, , ^ = 10cm 2.

Hallar el área de la región sombreada,


siendo O, P y Q centros de los círculos,
de radios 9 , 1 6 y 4 centímetros.

Solución
C álculo de MN y NT :

Sabemos que la longitud del segmen­


to tangente a dos circunferencias tan­
gentes exteriores, de radios R y re s

A s í: MN = 2^/ 9 ( 1 6 ) -» MN = 24

y NT = 2 ^ 1 6 ( 4 ) NT = 16

Para el área de la región sombreada : X = MOPQT " S m OQT


o m e jo r. X - SM0PN + SNPOT - SMOQT

Siendo MOPN, NPQT y MOQT, trapecios

r 9 + 16 ^ '1 6 + 4 9 + 4
X = . 24 + . 16 - . 40

ro
y

Rpta: X = 200 cm

En la figura adjunta , se sabe que el área


del paralelogramo ABCD es “Z”. AM = MB
«

y BN = NC. Hallar el área de la región


sombreada. AE = ED.

Solución

Al trazar MN y MQ , como MBNQ


es un paralelogramo , MR = RN y
NF = FQ, deducimos que P es
baricentro del A MQN. Luego, sucesi­
vamente tenemos :

1 e
SpQF ~ 7:( Smq N ) - — ~2 MBNQ
\ /
1
PQF — ( s MBNQ )
12

r 1 s 1
y a continuación : PQF - “ • ^ABCD ( S ABCD )
12 48

Rpta: S PQF
48

En la figura adjunta , se tienen dos


circunferencias tangentes exteriores ,
donde AB y CD son tangentes comu­
nes.

Si :
S A E B + S CFD = 17CmZ

Hallar : S OEO'F

575
Solución
Con los trazos señalados , por la propiedad indicada en el problema V .1 ., para todo
trap ecio:

OABO’ ^O E O ' = S ABE


OEOT ^OEO + ^OFO’
OCDO’ ® 0F0'
= s CFD

^O EO 'F = ^A B E + ^C F D

Rpta ^O EO 'F
- 17 cm2.

B M 1
El área del paralelogramo ABCD, de la
figura, es U.
BM = MN = NC.

Hallar el área de la región sombreada.

Solución

D a t 0 : S ABCD = U

3a . h = U
B M— a —n — a - C
a .h = H

Como : A OMB s A OFD

( ZOBM = ZODF, OB = OD
y Z MOB = Z FOD ).

Entonces : OM = OF

Análogamente : A AOE = A CON OE s ON


Con lo que AOMN = AOFE
Luego, la región propuesta equivale a la indicada en la solución
^ - c . c
sombreada ~ AMND AOD d )•

( a + 3a 1 ( U ^ ABCD _ U
Siendo : SAMND h = 2ah = 2 y s AOD ~
l 2 ; 3 J

Sustituyendo en (1 ) :

sombreada - u - -
3 4

Rpta: S sombreada U
12

B
Hallar el área de la región sombreada, si
E es un excentro del triángulo rectángu­
lo ABC, cuyo inradio mide 4 cm. y P, Q
son puntos de tangencia.

Solución
Con los trazos indicados, donde rB
es el exradio relativo a BC, se
tiene :

S rqE - SpFEO - SPFE , siendo.

S pfeq = Srbq + ^ bfeq

en lo a n te rio r:
S pqe - Spbq + SBFEQ - SPFE

e P B . BQ BQ + EF 'i PF . EF
b PQE = -----«------ + Dr —
V

q _ 4x4 , 4 + ra ( 4 + ra )r;
b PQE------ ^— + ra
\ /
Rpta: SPQE = 8 cm

El área de un triángulo ABC, es S. Se trazan AA', BB' y CC' perpendiculares a una recta
exterior, relativa a AC.

Hallar el área del triángulo que tiene por vértices los puntos medios de dichas perpendicu­
lares.

577
Solución
Según el gráfico : S, + S2 = S

Incógnita : X + Y

AM = A’M , BN = B'N , CP = C’P

Sabemos que en todo trapecio, el


segmento que une los puntos medios
de las bases determina regiones equi­
valentes ( Propiedad N° 8 ).

Luego :
Trapecio AA’B’B : X + Z = U + S t .................. ( 1 ).

Trapecio BB’C’C : Y + W = V + S2 .................( 2 ).

Trapecio AA'C’C : X + Y + U + V = Z + W ... ( 3 ).


#

Sumando miembro a miembro estas tres expresiones :

2X + 2Y + U + V + W + Z = U + V + W + Z + S1 + S2

2( X + Y ) = s 1+ s2

Rpta x + y = Si + S 2 = -
2 2

Un rectámgulo y un cuadrado tienen igual perímetro.


Demostrar que el área del cuadrado es mayor que la del rectángulo

Solución
Por ser iguales los perímetros

4L = 2( a + b )
a + b '
L = L
/

Siendo las áreas : Srect = ab ,

a + b
Scuaa = L2 = \ /

Ahora bien, de la aritmética sabemos que para cualquier par de números positivos distintos,
la media aritmética es mayor que la media geométrica.

Así, con a y b : U J l > Jgb


2
Elevando al cuadrado ambos miembros de la desigualdad, para tener las expresiones de las
2
areas : > ab
V
De donde, efectivamente :
^cuad. > ^rect.

13)
Dado el rectángulo ABCD, I es incentro
del triángulo ABC.

Demostrar que el área del rectángulo


ABCD es el doble del área del rectángu­
lo QIPD.

Solución
* Se tiene : SQIPD = IQ . I P ( 1)

Siendo : IQ = AT y AT = AE

Además : IP = NC y NC = CE
V

Luego : IQ = AE y IP = CE . Por lo
que al reemplazar en ( 1 ) :

QIPD
= AE . C E .................. ( 2 ) .

Pero, por el teorema II.9., sabemos que para el triángulo rectángulo ABC :

SABC = AE . CE

Y como : SABCD = 2 (S abc) -» ABCD = 2( AE . CE ) ............( 3 ) .


S4Brn

Finalmente, de ( 2 ) en ( 3 ) : SABCD = 2( Solpn)


QIPD

14) En un trapecio , las longitudes de las bases son a y b . Hallar la longitud del segmento
paralelo a las bases, limitado por los otros dos lados y que determina dos figuras
equivalentes.

Solución
Consideremos el trapecio ABCD, don­
de BC = a , AD = b, EF el segmento
paralelo a las bases, siendo además h
y H alturas de AEFD y ABCD.
Datos : a y b. Incógnita : x.

Se tienen :
/
AEFD h ( i )•
V /

a + x
EBCF s = j ( H ~ h )...( 2 ).
v
\
y s ABCD -» 2S = H (3 )
\

579
/

b+ x ' a + b > a +b
De (1 ) en ( 3 ) : 2 h= (4 )
V / 2 ( b + x)

' b + x ' ' a + x '


D e (1 ) = ( 2 ) : h= (H-h)
l 2 J

CsJ
a + x
Operando convenientemente : (b + x)h + (a + x)h = (a + x)H
H a + b + 2x

a 4- b a + x
Finalmente, de ( 4 ) = ( 5 ) : 2 (b + x ) ~ a + b + 2x

a2 + b2
De donde : x =

Otra form a : Prolongando AB y DC hasta su punto de intersección en Q :

Sbqc + S *2
A EOF - A BQC -> = -T
BOC
Q
2
de donde : 1+ (1). /
t v
V

BQC
B ¿ -s= .a~ \c
También, A AQD - A BQC :
2
SBQC + 2 S = b _ , y de aquí :
BQC
/ \
1+ 2 (2).
\ BQC >

D e (1 ): - 1, sustituyendo en ( 2 ) 1+ 2 - 1
BQC /

a + b
Despejando : x =

15) Sea ABCD un Cuadrado cuyo lado mide 6m. Si se trazan los segmentos paralelos AF y CE,
distantes2m .,entresí,dem odoque F e DC y E e AB.entoncesel área del paralelogramo
que se forma, es :

' yTT33 - 9 ' ' - 3 + /1 5 3 ' ' /Í5 3 - 3 '


A) m B) m C)3 m
v / v j
/
' -2 + /1 3 3 ' - 2 + / Í 5 3 "
D) m E)3 m 1
\ v y

580
Solución

Al trazar AH _L CE , A AHE - A EBC,

Con BC y AH homólogos, y como :


BC = 3( AH ) Si AE = x -+ EC = 3x

Luego EB = 6 - x

En A EBC : ( 6 - x )2 + 62 = ( 3x )2

Desarrollando y ordenando luego de


sim plificar: 2x2 + 3x - 18 = 0

r- . J 153-3
Ecuación que nos conduce a : x = jl-------------
2

/l5 3 - 3 '
Luego : SAECF = ( AE ) ( CB ) = .6

\
' / T 53 - 3
AECF = 3
/

Rpta: ( C ).

En la figura, ABCD es un rombo, P punto


medio de BC, AP = 9m. y DP = 13m
Hallar el área del rombo.

Solución
Sabemos que : SARf-n = 2 ( SAPD) ............(1 ).

En el A APD, trazamos la mediana PM.

Luego PM //'Á B y PM = AB = í = AD. Por el


teorema de la mediana en dicho triángulo :

2?.2 + — = 92 + 132 -+ i = 10

Con el Teorema de Herón para el SAPD hallamos


9 + 13 + 10
previamente : p= ------------------- = 16

*Q1
Entonces: SAPD = / 16( 16 - 9 ) ( 1 6 - 1 3 ) ( 1 6 - 10)

S apd = 1 2 / 1 4 cm:

Sustituyendo en ( 1 ) :

Rpta: SABCD = 2 4 ^ /T Í cm2.

17) En un trapecio ABCD, de bases BC y AD, BC < AD, se toman M y N puntos medios de
AB y CD respectivamente. MC y NB se cortan en P. MD y AN se interceptan en Q.
S i : S BPC = 1 0 cm 2 y S AQD = 14 cm2. H a lla r: S MPNQ

Solución

Al trazar MN, en el trapecio MBCN,


sabemos que SMBP = SNCP = S,
y en trapecio AMND :
9 - q _ c
AMQ “ NOD ” 2

Además, para el trapecio ABCD, por


v .8 . 2 ( S c m d ) = S ABCD

Con el g rá fico :
2( 8 ^ 8 2 + SMPNQ) = 2S 1 + 2S 2 + SBPC + ^ M P N Q + S AQD

De donde, al cancelar S 1 y S2 :

R Pt3 : S MPNO = 2 4 Cm2

18) En un cuadrilátero inscriptible ABCD, AB = 7cm. ,B C = CD = 15cm. y AD = 25cm., se


traza AC. Hallar el área del triángulo ABC.

Solución
Con el Teorema de Brahma-Gupta,
para el cuadrilátero ABCD :
_ 7 + 15 + 15 + 25 _ o í
p— — Ol

S abcd = / ( 3 1 - 7 ) ( 3 1 - 1 5 ) ( 3 1 - 1 5 ) ( 3 1 - 2 5 )

S ABCD = 1 9 2 Cm2

Luego : SABC + S ADC - Q


“ ABCD

®ABC + ^A D C “ ^^ ClTl2 (1 )
Además, como B + D = 180

^ABC _ 7 x 1 5
S ADC 25x15

CQO
25
AB C (2 )
ADC

25 s
+ — e
Reemplazando ( 2 ) en (1 ) ; SABC ABC = 192 cm2

Rpta: SABC = 42 cm2

El área de un paralelogramo ABCD, es S. Se toman M y N puntos medios de AB y AD,


respectivamente. CM y BN se interceptan en el punto R. Hallar el área del triángulo MBR.

Solución

Del gráfico, los triángulos MBR y


MBC tienen la misma altura que parte
del vértice B, luego :
S mbr _
MR
S MBC MC

O ( MH
MR ^loo M\
sMBR “- [ m c Jb M
mbr BC
MBC '
l MC ;

Tomamos P y Q, puntos medios de BC y CD :

O 1 io \ _ 1 ( S ABCD "l . S ABCD s^


S M BC - 2 { ^M B C Q ) - 2 { 2 )~ 4 S MBC (2).
4

AT
Además en el A ABT, como AQ es paralelo a MC : MR =
2
RC RC MR _ 1
AT = TL = RE = EC = MR =
2 RC ” 4
MR 1 MR = 1
(3)
MR + RC 1+ 4 MC " 5 ......

/
C on(2 ) y ( 3 ) e n ( 1 ) MBR
\ /
p.
$*
Í s
t |
; ^
Rpta:

( Teorema de C lairaut) Dado el triángulo


ABC, sobre los la d o s se constru­
yen ABDE y BCGF, paralelogramos cua­
lesquiera. Se prolongan ED y GF hasta
su intersección en el punto P. La prolon­
gación de PB corta a AC en L y se
prolonga BL una longitud LR = PB.

583
Luego por los vértices A y C se trazan paralelas a L R , determinando el paralelogramo
ACQM.

Demostrar q u e :

Demostración :

Prolongamos MA hasta interceptar a


ED en H.
R
I \ X

Como AAEH = A BDP ( AE a BD, f


' <
»
\
\

4 EAH a 4 DBP y ÁH a BP) ,


entonces AHPB <> AEDB.
Además los paralelogramos AMRL y
AHPB , tienen : LR = BP y la mis­
ma altura d.
S amrl = S ahpb

Luego: S AMRL AEDB


Análogamente se tendrá :
S lroc = S BFGC

Entonces : SACQM = SAMRL + SLRQC


S ACQM = S AEDB + S BFGC

En un triángulo ABC, las cevianas interiores AF y CE se cortan en el punto P, siendo


además EF paralelo a AC. Hallar el área del triángulo ABC , si las regiones triangulares
EPF y APC, tienen por área 9 y 16 cm2,. respectivamente.

Solución

SABC = ?*

Por la propiedad deducida en el problema B


V.3, sabemos que en trapecio AEFC de la
figura adjunta :

AEFC EPF + i s APC

V ^AEFC - V 9 + >/T6 -> SAEFC - 49 cm'

Además : A EBF - A ABC

Sebf _ EF
_________

S ABC AC
n. S ABC “ S AEFC _ EF , S ABC “ 4 9 _ EF
° • o ^— 2 o ” 2 ( *)
^ABC AC b ABC AC

Siendo los triángulos EPF y APC, semejantes :


:2
S EPF _ EF E F^ = 9 ....................... ( 2 )
S APC AC2 ÁC2 16

Sustituyendo ( 2 ) en (1 ) : ^ abc _4^ _ _9_


ABC 16

Rpta: SABC = 112cm 2

B M

Dado el cuadrado ABCD, donde M y N


son puntos medios de BC y CD.
N
Hallar el área de la región sombreada, si
AB = a.

Solución

Sabemos que AM y BN son perpendiculares


Llamando X el área pedida , con el gráfico : B

^ “ ^AEF " ^AOP (1 )•

Para determinar SAEF, hallamos AE y EF

En el A ABM, con el teorema de Pitágoras


2
-» AM2 = a2 + I ±
/

por relaciones métricas : AB2 = AM . AE


2 .
Luego : a2 = ^~5 . AE, de donde : AE = — a.y~5

De otro lado F es Baricentro del A BCD , ( BN y CO medianas de dicho triángulo )

Entonces : OF = —( O C ) = —r A C ^ l ( A C ) = l ( a / 2 )
v /

585
a Í2 _ AC _ a j 2
Es d e c ir: OF = —^— . Y como AO = -LL1 = *“ , se tendrá :
6 2 2
g /2 3 /"j? 2 _
AF = AO + OF = — ^— -» AF = — a^^2 . Esto nos permite hallar EF,
2 6 3

en el A AEF : EF = AF - AE
2 ,
_ 2 /5
EF = -a /2 - -a /5 ; EF =
v \ IG

En seguida, para el S AEF ‘


\
AE . EF 2
AEF -
2 V /
2 2
AEF — a¿ (2 )
15

Ahora para hallar el SA0P aprovechemos que los triángulos AOP y ABD tienen la misma
altura trazada desde A. Luego las áreas SA0P y SABD serán entre sí como las bases OP
y BD :
S aop OP
(3).
ABD BD

Siendo P Baricentro del A ABC, ( AM y BO, medianas en dicho triángulo ) entonces :


/ \
BD BD OP 1
OP = —( OB ) = - -> OP = — , esto es : ------ = —
3 } 3V / 6 BD 6

sustituyendo en ( 3 ) tenemos :

2
** = S AOP = 7:(S ABd ) S AOP (4)
^ABD 6 b 6v /
12

Fir tímente, con ( 2 ) y ( 4 ) en (1 ) :

X = A aa
15 12

Rpta: X= -
20
B M
/
23)
En el cuadrado ABCD, M y N son
\ / f
K / N
puntos medios de BC y CD.
Si AB = a, hallar el área de la región
sombreada.
/ / ^
*
I.
y i\ D

5K6
Solución
El área de la región sombreada es igual a la suma de las áreas de las regiones AMC y BND,
menos el área de la región OPEF. Siendo :
s
= — ( s ABCD
AMC ABC ) - 2
2 v \

S amc - “ ( SABcd ) ” AMC ~ = SBND

Además, por el problema a n te rio r: SOPEF


20

Lue9 ° : ® sombreada ” ^AMC + ^BND " S qPEF

s sombreada = —
a +
20

Rpta:

Las diagonales de un trapezoide miden 20 y 34 cm. y el segmento que une los puntos
medios de dos lados opuestos mide 21 m. Hallar el área del trapezoide.

Solución
Sea el trapezoide ABCD , donde :
AC = 20cm., BD = 34cm. y MN = 21cm.
Tomando Q, punto medio de BC, al
trazar MQ y NQ, según el problema
visto en V.5, sabemos que :

ABCD ™ SMQN ) (1).

Siendo, en el A A B C : MQ = AC

MQ = 10 cm.

BD
y en el A BCD : QN = QN = 17 cm

Luego, con el Teorema de Herón, para el SMQN , donde


10 + 17 + 21
P= = 24

Smqn = \ 2 4 ( 24 - 1 0 )( 2 4 - 17 ) ( 2 4 - 2 1)

SMQN = 84 cm2

Remplazando en ( 1 ) : SABCD = 336 cm2 Rpta


587
25)

En la figura adjunta AB y DC son


tangentes comunes a las circunferen­
cias de radios R y r.

Hallar la expresión para el área del tra­


pecio ABCD. (O y P : centros)

Solución
Con los trazos indicados, sabemos
—A
que AB = DC = 2 / R t
rt
E il
Al trazar la tangente interior común a •ii H
i
ambas circunferencias , como :
MA = ML = MB y ND = NL = NC 0 L 1“

MN = AB —» MN = 2 ^ r 7.
“MN es mediana del trapecio ABCD”.
Luego, para el área, falta determinar
la longitud de la altura B H .

En el A EAB : EB = OP EB = (R + r ) y AB = 2 / R r , por relaciones métricas:

AB = EB . BH
^ 2
Sustituyendo lo a n te rio r: ( 2^ r7 ) = ( R + r ) BH

4Rr
De donde : BH =
(R + r)

Luego : SABCD = MN . BH, ( área del trapecio en función de la mediana y altura )

4R r
Con lo h a lla d o : s abcd = 2 /R f.
(R + r )

Rpta:

26) En un trapezoide ABCD, las diagonales AC y BD se interceptan en el punto P. Si el área


del cuadrilátero que tiene por vértices los baricentros de los triángulos ABP, BPC, PCD y
APD, es 2cm2. Hallar el área del cuadrilátero que se obtiene al trazar paralelas a las
diagonales AC y BD, por los vértices A, B, C y D.
Solución

Según el gráfico, sean Et F, G y H,


baricentros de los triángulos ABP,
BPC, PCD y APD, respectivamente.
M y N son p u n to s m e d io s de
AP y PC. A’B’C’D' es el cuadrilátero
que determinan las paralelas.
Por propiedad del baricentro, en cada

triángulo : BE 2 BF . Esto indi-


BM 3 BN
ca que se cumple el Teorema de
Thales • • EF AC.
EF 2
Entonces los triángulos EBF y MBN son semejantes, donde :
MN 3

_ AC EF EF
Siendo MN = luego: ^
AC (1 )

GH
Análogamente se demuestra que GH 11 AC y I . F G / / B D / / EH y
AC V
FG EH
BD BD

Concluimos en seguida que “ los cuadriláteros EFGH y A’B’C’D’ son paralelogramos


semejantes", con razón de semejanza :

EF EF _ 1
A ' B* AC 3

^EFGH _ EF
y • 'V
S a ’B’C'D' \ A 'B ' / V 3 y 9

S efgh _ _L p Qr datQ S EFrH = 2cm 2


S a ' B ' C ' D ' 9 E F G H

Rpta: SA<B.C>D. = 18cm2

El área de un trapezoide ABCD, es 9 cm2. Se traza la diagonal AC y se determinan P y R,


baricentros de los triángulos ABC y ADC, respectivamente. Luego se traza BD y se ubican
Q y L, baricentros de los triángulos BCD y ABD. Luego de trazar PQ, QR, RL y LP, hallar
el área del cuadrilátero PQRL .

589
Solución
En el gráfico, T, M y N son puntos
medios de A B ,B C y A C .C o m o P y
Q, son baricentros de los triángulos
ABC y BCD, respectivamente, por
propiedad de este punto notable :
PM 1 QM
AM ~ 3 ~ DM
Se cumple el Teorema de Thales.

Luego PQ // AD, los triángulos PMQ


AD
y AMD, son semejantes /. PQ = — -

CD AB
Análogamente se demuestra que PL 11 CD y PL = ----- , QR 11 AB y QR = ----- , LR 11 BC

BC
y LR = — . Esto indica que “el cuadrilátero PQRL es semejante al ABCD, donde la razón

1„
de semejanza es

Entonces;
\2
^ pqrl _
S PQRL “ -JT ( S ABCD ) “ 7T ( 9 c m 2 )
S ABCD /

R Pta : S PQRL = 1C m 2

28) La longitud del lado de un cuadrado ABCD es 10 cm. Tomando como diámetros los lados
AD y CD se dibujan interiormente dos semicircunferencias secantes en el punto E. M y
N son puntos medios de AD y CD . BM y BN interceptan a las semicircunferencias en
P y Q. Hallar el área de! cuadrilátero PBQE.

Solución
Del gráfico, notamos que el cuadrilátero PBQE B
es simétrico : BP = BQ y EP = EQ.
Luego : BE 1 PQ y:
S _ B E . PQ / 1\
PBQE ' 1 /

Además E es el centro del cuadrado /* -

B E .E D .^ .H Ü BE = 5 / 2 . . . ( 2 ; .
2 2
Para determinar PQ , como BM = BN y
BP = BQ, entonces PQ es paralelo a MN.
590
En seguida:
PQ BP „ M N . BP
A PBQ ~ AM BN -> PQ == - (3 )
MN BM BM

Siendo MN = 5 / 2 , ( A M D N ) ,

En A M A B :B M 2 = 10* + 52 -» BM = 5 / 5 y BP = BM - PM -4 BP = 5 / 5 - 5

5 /2 Í 5 / 5 - 5 )
Al sustituir en ( 3 ) : PQ = v .L .I L
5 /5

PQ = / 2 y 5 - / 5 J. Luego, con este valor y ( 2 ), en (1 ) :

5 /2 . / 2 ( 5 - / 5 )
PBQE ~

Rpta: SPBQg = ( 25 - 5 / 5 * ) cm2 Rpta.

29) Demostrar que el área de todo triángulo acutángulo es igual al producto del circunradio y el
semiperímetro de su triángulo órtico pedal

D em ostración
Consideremos el A ABC, donde O es B
el circuncentro y PQH el triángulo
órtico pedal.

Sabemos que los radios OB, OA, OC


de la circunferencia circunscrita al A
ABC son perpendiculares ( * ) a
PQ ,PH y H Q , re s p e c tiv a m e n te .
Luego, las áreas de los cuadriláteros
POQB, POHA y HOQC :

e O B . PQ OA . PH
op oQ B «------•b POHA--------- «-----

OC .H Q
y 'HOQC
S ie n d o :
^A B C " SpO Q B + SpOHA + ^HOQC

OB . PQ OA . PH OC . HQ
S abc = ----- : ----- + r + -------: -----
\
' PQ + PH + HQ
C o m o : OB = OA = OC = R -» R
ABC
v /

Llamando p’ = PQ + PH + HQ e| sem¡perímetro del A PQH : .*.

591
(*) Para d e m o s tra r que OB es
perpenicular a PQ, prolongamos BO
hasta E. Luego : ^ PQB = ^ BAC,
por s e r el c u a d rilá te ro APQ C
inscriptible.
Además como ^ BEC = ^ BAC
¿ PQB = BEC .

Siendo, en A BCE :
¿ EBC + ^ BEC = 90°

entonces : EBC + PQB = 90e

Por lo que el ^ BNQ = 90°

30) En un trapezoide ABCD , M y N son puntos medios de las diagonales AC y BD ,


respectivamente. Las prolongaciones de los lados AB y DC se interceptan en el punto P.
Demostrar que : S BMDP ” ^AN C P

Solución
Del g rá fico :
S BMDP ^BMDC + ®BCP

BMDP (^BMC + ^CMü) + ^BCP

S ABC + S ACD + S BCP


BMDP
J

x S a b c + S A C D , +

BMDP BCP
\

D Es decir: S BMDP ABCD + S BCP d )


V J

En forma análoga : SANCP = SABCN + S ^ p ANCP " ( SABN + SBCN ) + SBCP

/
ABD + BCD ABD + s BCD
ANCP + s BCP ANCP + s BCP
\

S ancp " ^ ABCD + SBcp (2 )


2

Luego, de (1 ) y ( 2 ) : ( Propiedad)

* Notar que ( Propiedad)

592
En un trapezoide ABCD , M y N son puntos medios de las diagonales AC y BD,
respectivamente. Las prolongaciones de los lados AB y DC se cortan en el punto P.
Demostrar q u e :

_ S ABCD
MPN “

Solución
Por el problema anterior

ANCP
= S ABCD + s BPC (1 )

También, con el gráfico :

^ANCP - ^APM + ^AMN + ^MPN + ^NPC


D
como : SAPM = ^ apc ( S AMN - ^ANC
2
y sNPC = s pnd - sCND
en lo a n te rio r:

ANCP
= ^ E S . + S ans. + SMPN + SPND -
2 2

= S APC , S ANC + « + S BPD - S CKD


MNP
^.
ANCP

ANCP
Sabc + $qpc +, $anc + s .MPN + ^bpc + Sbcd - s CNO

Y como SCND = , al reemplazar en lo anterior, agrupar y cancelar términos

semejantes, llegamos a :
_ S (2).
^ANCP “ ^ + ^ ANC + SMPN + SBpc

Igualando ahora los segundos miembros de las expresiones ( 2 ) y ( 1 ) :

S ABC , S ANC . o , e S ABCD + s


— “— + — "— + b MPN + b BPC - ---- 2 BPC

Cancelando SBPC, despejamos luego SMPN

o _ ^ABCD ^ABC ^ANC


^MPN ~----------- «----------- ^—

e _ S abcd S abc S abcn " S ABC


^ mpn 2 2 2

593
o _ S abcd S abcn
MPN ------------—

g
Y com o: SABCN = — ( problema anterior).

( S abcd )
_ S abcd
MPN -

_ s ABCD
MPN -

32) En un cuadrilátero ABCD, AB = 3m., BC = 4m. y CD = 12m. H allarelvalordeA D , sabiendo


que el área de la región ABCD es máxima.

Solución B
- AD = ?, cuando S ABCD —> maxima

' SABCD»será máxima, cuando SABC y SACD


lo sean. L u e g o :

- SABC, máxima, implica : a = 90°

Entonces : AC = 32 + 42 AC = 5
D
“ S ACD, máxima , implica : p = 90°

AD = AC + CD AD = + 12

Rpta: AD = 13

33)

Hallar Sx, si el □ ABCD, tiene área 72u2.


Los lados del □ ABCD , han sido
trisecados.

Solución
( a ) PROBLEMAS PREVIOS:

a.1) Sea el DABCD. P, R, Q y T se ubican sobre


los lados de modo que PA = 2PB, CR = 2RB,
DQ = 2QC y TD = 2AT.

Demostremos, q u e :
* OQ = 2( OP ).
Veam os: A PBR - A ABC, porque 2 2 = 2 2 = 1 y tienen en común el B.
AB CB 3

E ntonces: PR = 2 2 y PR // AC (1 )
3

-A n á lo g a m e n te : A TD Q ~ A A D C .y a q u e : 1 2 = 2 2 = 2 y comparten el D.
AD CD
2AC ™
Luego : TQ = ------- y TQ AC (2 )

- De( 1 ) y ( 2 ) : PR || TQ y TQ = 2PR

- Esto indica que AOPR - AO Q T OQ = 2 (O P ) y OT = 2( O R ), tal como queríamos


demostrar.
a.2) R ecordar, según se demostró en un problema del capítulo HLíneas Proporcionales ”
( Capítulo 1 0 ) , que :

AP DQ „
Si • — = ------- = 2
• PB QC

NL
Entonces : PL = LQ y = 2
LM
D

a.3)
Es fácil concluir, de los resultados anteriores, que si
se trisecan los lados del □ ABCD, entonces :

Pl = |J = JH y
QT = TU = UF

(b) RESOLUCION DEL PROBLEMA PROPUESTO

- S iM ,G , N y L, son puntos medios de


los lados del OABCD, entonces GL y
M N interceptarán a los lados del □ ITU J
en sus puntos medios.
Luego:
S x - S r r u j - 2( SVZWE)

( Propiedad, prob. n° V.4 )

(OC).

- Sabemos, que OMGNL y DVZW E,


son paralelogramos.

595
ZE
Entonces ZO =

Pero ZE = GZ ZO = ZO = En forma a n á loga, OV =


o 3 o

OW y OE = 9 L . De aquí, es fácil concluir que los cuadriláteros VZWE y MGNL,


3 3
1
son semejantes; con razón de semejanza —. A s í:

\2
q vzwe =
-> SvzwE = g- ( S MGNL )•
SfoGNL /

S iendo: SMGNL = A^ CD S VZWE -“ A ( s°ABCD )

Reemplazando, esto último en ( a ) :

sx = -g ( S ABCD ) ¡Propiedad!

Con el dato : SABCD = 72u2

Rpta: S„ = 8u2
A
VI.- REGIONES POLIGONALES

VI.1. P olígonos C irc u n s c rito s .- En todo polígono circunscrito a una circunferencia, el área se
puede expresar como el producto del semiperímetro y el radio de la circunferencia inscrita.

D em ostración

* En efecto. Sea ABCDE...... un polígono


D
convexo de n lados circunscrito a una
circunferencia de centro O y radio r.

Luego:

® polígono ” ^A O B + ^B O C + ^C O D +

( “ n " sumandos )

AB . r BC. r CD. r
5 = + ------------ ------------ +
1
polígono 2 2 2

f AB + BC + CD +
r , donde la expresión dentro del paréntesis
polígono

significa el semiperímetro p, del polígono :

Vl:2. Polígonos Regulares.- En todo polígono regularel área es igual al producto del semiperímetro
y apotema.

D em ostración
* Consideremos un polígono regular de
wn" lados. Siendo y an, las longitu
des del lado y apotema , respectiva­
mente :

^ p o líg - " n ( ^A aob )•


regular

ln ^ ( Elemento fundamental del polígono regular)


O
R
í;n • an
Luego : políg. = n
regular

597
4 s

regular

p, del polígono, .*

VI.3. Sector P oligonal regular.- Es la región del plano que encierra una línea poligonal regular,
llamada base y los radios a los extremos, de la circunferencia circunscrita a dicha línea.

Teorema.- El área de todo sector poligonal regular es igual al producto del semiperímetro
de la base y el apotema.

D em ostración

Sea el sector poligonal ABC...., de wm” lados, originado por una poligonal de apotema an y
longitud de lado l n. El área S, del sector poligonal será “m” veces el de la región AOB. Luego:

donde la

expresión en el paréntesis es el F
semiperímetro p' de la poligonal.

VI.4. Teorema.- Las áreas de dos polígonos semejantes son entre sí como los cuadrados de
cualquier par de elementos homólogos.

D em ostración

* Sean, por ejemplo, los polígonos se-


B mejantes ABCDEF y A’B'C’D’E’F con
razón de semejanza K. Tracemos to
das las diagonales correspondientes al
vértice de un par de ángulos congruen­
A D tes, a fin de tener triángulos semejan­
tes.

Luego :

E x = _y = _z = w _ = k2
X' y Z' W*

598
Con Propiedad de proporciones :
X + Y + Z + W
X' + Y ‘ + T + W ‘

De donde , llamando S y S’ , las


áreas totales :X + Y + Z + W = S ;
y X’ + Y '+ Z’ + W ’ = S’
A

Esto e s :
2 2 2
S AB BC CD
s' A 'B '2 B 'C '2 C D '2

VI.5. G eneralización.- Para cualquier par de figuras cerradas semejantes, las áreas de las
superficies que determinan, son entre sí como los cuadrados de sus elementos homólogos.
Así por ejemplo, para las figuras
semejantes a d juntas:

Aprovecharemos esto en
el siguiente teorema. A B’ A B

VI.6. Teorem a.- Si los lados de un triángulo rectángulo son líneas homologas de figuras
semejantes construidas sobre ellos, entonces la suma de las áreas de las regiones
construidas sobre los catetos es igual al área de la región apoyada en la hipotenusa.

D em ostración

Sea el A ABC, donde X, Y, Z representan


las áreas en mención. De la generaliza­
ción anterior podemos e scrib ir:
X Y Z
2 2 2
AB BC AC

Con propiedad de proporciones :


X + Y Z
2 2 2
AB + BC AC

Siendo : AB2 + BC2 = AC2 X+Y = Z

( * Usaremos este teorema, más adelante en la demostración de la relación para las lúnulas
de H ipócrates).
599
PROBLEMAS RESUELTOS

1) Determinar las fórmulas para el cálculo de áreas de los polígonos regulares :

a) Hexágono b) Octógono c) Dodecágono, en función del radio R de la


circunferencia circunscrita.

Solución
a) Hexágono re g u la r:

A AOB es equilátero.

S 6 = 6 ( S iA 0 B ) = 6 ' r 2 V 3 1

expresión en la que R puede sustituirse por d6.

b) O ctógono re g u la r:

Del gráfico

S8 - 8( SAuhin )
AMNO

/ O M . NH
-> S8 = 8 = 4R( NH )... (1 )

ON
Siendo en el A NHO : NH =
V2
R
NH = , en ( 1 ) :
IX

S8 = 4R 9 9

c) D odecágona re g u la r:

Con el A ODC. elemento fundamental


de este polígono : S12 = 1 2 (S aODC)

es d e c ir:
OC . DE
S « = 12 = 6 R (D E )... (1 )

OD R
Del A O E D : DE = D E ^ ,

600
sustituimos en (1 ) : S12 = 6R

2) El lado de un hexágono regular es V . Se prolongan los lados en un mismo sentido y una


longitud igual a m£M. Hallar el área del polígono que tiene por vértices los extremos de dichas
prolongaciones.

Solución
Sea ABC F el polígono original y
A’B'C* F el de los extremos de las
prolongaciones . Notamos que el A
AA’B’ , es recto en A \ por; AB’ = 2AA*
y Jf A’AB1 = 60°

Luego: A’B’ = l -J~3 , es la longitud del


lado del hexágono regular A'B'C’ F.

Por el problema anterior el área será:

S '-fíí/lf J í

Rpta

3) Hallar el área del hexágono que tiene por vértices los puntos medios de los lados de un
hexágono regular, cuyo lado mide “a”.

Solución
Sea ABC el hexágono mayor, de lado a. La
longitud del lado del otro es x.

Con el gráfico :

A ABC x = ,( Base m e d ia ).

Siendo AC = = R ^ 3 = a^~3
D

a /3
Luego : x = *

a — E
ó 2 rñ
Entonces, para el área Sx : Sx = —x ó

3 fa j3 '
^ = f a 2V 3 Rpta
s x = 2
V

601
Hallar el área de un octógono regular de 1m. de lado.

Solución
Sabemos, por el problema N° 1, que el área S8 del octógono regular, en función del radio R

de la circunferencia circunscrita, es : Sfl = 2R2 y[% .


1
Siendo : - R ^ 2 - ^ 2 = 1m R=

1 2/2
sustituyendo en S8 : S«8 = 2
IT - 7 ? 2- J 2

S8 = 2 ( 1 + / 2 )m2 Rpta

Un hexágono ABCDEF tiene sus ángulos congruentes y sus lados son tales que :
AB = CD = EF = a y BC = DE = FA = b. Siendo a > b . Calcular el área de dicho
hexágono.

Solución N
Cada ángulo exterior del hexágono mide: v \ \
a/ 60°'' a
360° // \ \
= 60°. De modo que al prolongar g/60»-a >60\ p
6
los lados como indica la figura, se obtie­
nen los triángulos CND, EPF, AMB y
MNP, equiláteros.
Luego: * \ \
(.60*____6_0°_\,
S hexágono A — b

V3 V3 V3 V3
hexágono = ( 2a + b )2

De donde :

hexágono = ( a2 + 4ab + b2 ) Rpta

El lado de un hexágono regular mide 9m. Determinar la longitud del lado de otro hexágono
regular, cuya área es 4/9 de la del primero.

Solución
Como estos polígonos son semejantes, las áreas se encontrarán en la misma relación que
los cuadrados de sus elementos homólogos. Llamando X, la longitud del lado del segundo

. . x2 4
hexágono : — = —

Rpta: X = 6 m.
7) Los lados de dos pentágonos regulares miden 17 y 8 cm., respectivamente. Hallar la longitud
del lado de otro pentágono regular, cuya área es igual a la diferencia de las áreas de los
dos anteriores.

Solución
Llamando Sv S2 y Sx las áreas; por semejanza :

172 82 t

Con propiedad de Proporciones :


S-t - S
172 - 82

Y como : Sx = S, - S2 , por dato, entonces : í \ = 172 - 82

Rpta: f = 15cm.

8) En una circunferencia de radio R, se inscriben los triángulos equiláteros ABC y DEF, siendo
sus lados respectivamente paralelos. Hallar el área de la estrella formada.

Solución
Del gráfico: Sestrelta = SABC + 3S, (1 )

Siendo: AB = R-J~3

ABC

ABC ” — R^ (2 )
4

Para el A EMP; equilátero, se tiene :

AM = EM = MP = EP = PB

Luego : AM + MP + PB = AB

_ AB _ R ^ 3
MP + MP + MP = AB -> MP =

2 ¡ 3
' R /3 f /3 e R2 / 3
entonces : SEMP = MP ^ — _> S t = (3)
4 1 12
V /

Finalmente, con ( 2 ) y ( 3 ) en (1 ) :

s estrella - t4 r2/ 3 + 3 12
V /

^estrella “ ^ Rpta

603
Hallar la relación entre las áreas de los octógonos regulares, inscrito y circunscrito a una
misma circunferencia.
Solución

Sean A B C H y A ’B’C ’ H’ los


octógonos regulares inscrito y circuns­
crito, respectivamente, a la circunferen­
D' cia de centro O y radio R.
Sabemos que todos los polígonos regu­
lares de igual número de lados son
semejantes. Luego, las áreas estarán
E’ en la misma relación que los cuadrados
de c u a lq u ie r p a r de e le m e n to s
homólogos. En la figura, ON y OC son
apotemas de los polígonos, inscrito y
circunscrito.

Siendo : ON = — ^ 2 + / 2 y OC = R

ON S _ 2 + /2
Rpta
S‘ OC S7 “ 4

Se tiene un hexágono regular ABCDEF, de lado "a". Sea M punto medio de B C . Se dibuja
el simétrico del hexágono dado, según un eje que contiene a FM . Hallar el área d la región
común a los dos polígonos.
A* es simétrico de A, respecto al eje FM, sí y sólo si FM es mediatriz de AA'.

solución
El área de la región sombreada,
equivale al doble del área del cua­
drilátero ABMF y ésta a su vez
7 0
está formada por las regiones trian­
gulares FAB y FBM. Así, con los
gráficos adicionales:

Siendo : BF = a ^ , entonces
BF . AH
FAB

a / 3 Í f
/ V3
Es decir : SFAB =
BF x BM _ . V 5 ( |
Además : SpBM =

s
^FBM - a2^
:—

En seguida : SABMF = SFAB + SFBM

a2 ,[3 a
ABMF “ +

ABMF Luego : Spedida - 2 S ABMF

Rpta /3

11) El área de un octógono regular ABCDEFGH , es 8 cm2 . M, N, P, Q, L, T, I, J, son puntos


medios de los lados AB, BC, CD, DE, EF, FG, GH y H A , respectivamente. Hallar el área
de la región común a los cuadriláteros MPLI y NGTJ.

Solución
El área de la región pedida correspon­
de a la del octógono sombreado.
Por semejanza con ABCDEFGH :

X UV
8 ( i)
CD

S iendo: UV = ( UP ) / 2 = ( NIC ) / 2

_ (BC) UV _ / 2
UV =
BC 2

UV J 2 , UV
CD 2 ' LUeg° : W 2

En (1 ) :
8

Rpta: X = 4cm2

12) Interiormente al cuadrado ABCD, de lado “a", se construyen los triángulos equiláteros ABK,
BCL, CDM y DAN. Determinar el área de la región encerrada por el dodecágono cuyos
vértices son los puntos medios de los cuatro segmentos KL, LM, MN y N K , junto con los
punios medios de los ocho segmentos AK, BK, BL, CL, CM, DM, DN y A N , demostrando
previamente que el polígono es regular.

605
Solución
B
Para demostrar que el dodecágono
es re g u la r b a s ta rá p ro b a r que
PQ y PT son congruentes.
En efecto, el A NEM es isósceles, Q
punto medio de MN y P punto medio
de ME por ser el triángulo EMF
equilátero. Luego, por el Teorema de
los puntos m e d io s:
EN D
A NEM PQ = —

EF
A EMF PT = — , y como:

EF = EM = EN PQ = PT

Además MNKL es un cuadrado y el dodecágono regular se halla inscrito en la circunferencia


inscrita a dicho cuadrado.
En el A MDN, elemento de un dodecágono regular, de radios DM = DN = a, tenemos :

MN

GH MN
Entonces R = ;
R = | > / 2 - ^ 3 , es el radio para el polígono en cuestión
2 2 2
Como sabemos, por el problema N°1 : S 12 = 3R2
2
. Luego :

S i 2 = f a 2( 2 - ^ 3 ) Rpta

13) Hallar el área del octógono regular estrellado inscrito en una circunferencia de radio R,
uniendo los p u n to s:

a) De dos en dos.
b) De tres en tres.

Solución
a) ABC H, son los vértices respecti­
vos. Trazando AC, CE, E G , se
obtiene la estrella para este caso.
Siendo ACEG y HBDF cuadrados,
evaluaremos el área de la región en­
cerrada por la estrella sumando al
área del cuadrado ACEG. los cuatro

606
cuatro triángulos sobrantes equivalentes al MBN. Así :

^ e s tre lla ” ^A C E G + ^M B N ) (1 )

Siendo AC = R V"2» la longitud del lado del cuadrado inscrito :

SACEG = ( R ^ 2 ) = 2 R2 .............. ( 2 ) .

Para el A MBN, como MB = BN = AM = NC y MN = bABj~2 ,


entonces : AM + MN + NC = AC

MB + M B / 2 + MB = R^/2" MB( 2 + ^ 2 ) = R / 2 .

Luego : MB = R ( / 2 - i)

^ M B . BN MB R
En seguida: SMBN = ----- = — MBN “ ( 3 - 2 V 2 ) ... (3 )

Ahora, con ( 2 ) y ( 3 ) en ( 1 ) :
4R2
Sestrella
^ „ = 2R* + 2 (3 - 2/ 2 )

RPta- SeslreJla = 4 R2 ( 2 - ^ 2 )

b) En este caso la estrella se obtiene al


trazar A D .D G .G B , .... Calculare­
mos el área pedida, restando al
octógono regular ABC...H, los ocho
triángulos sobrantes equivalentes al
APB. Por el problema N°1, de esta
sección, el área del octógono ABC...H,
es 2 R 2 / 2 . Para el A A P B , como

AB = = R ^ 2 - yf~2 , entonces :

S apb = - • AP . PB = - . A®. .
2 2 / 2 / 2
G
R^2 - / 2
AB R
SAPB = -> SA P B = ( 2- / 2 )
Luego:
estrella “ ®ABC...H “ ® ( ^A P B )

607
= 2 R ^ -8x ^ ( 2 - / 2 )

14) Las áreas de dos polígonos regulares de n lados, uno inscrito y otro circunscrito a una misma
circunferencia, son respectivamente S y S \ Hallar las áreas de los polígonos regulares de
2n la d o s :

( a ) Inscrito a la misma circunferencia.


( b ) Circunscrito a la misma circunferencia.

( Problem a de G regory )

Sofución
(a) Sea ABC... el polígono regular de n
lados inscrito en la circunferencia de
centro O y radio R. A’B’C’... es el
polígono regular de n lados circunscri­
to a la misma circunferencia e inscrito
en la circunferencia de radio R \ B’
MC uno de los lados del polígono
regular de 2n lados inscrito en la cir­
cunferencia de radio R. Llamando X
el área de este último polígono, tene­
mos q u e :

( 2 n ) S A MOC = X (1)-

Los triángulos MOC y B’OC, tienen en común la altura C T, por lo que :

s am oc OM R , esto e s : s amoc _
(2).
OB‘ R* ^ A B 'O C R............
AB'OC

S"
Además : SáW0C = SAB'OC') = j
n /

ÉL (3 )
2n

2 n ( S AMOC ) ^ r
Reemplazando ( 3 ) en ( 2 ) : A MOC

ÉL R' S‘ “ R*
2n

Y con ( 1 ): (4).
S1 R*

608
También, como los polígonos ABC ... y A’B’C’... son semejantes

f _R ' JL —
S
R* J S* R1 S1

Sustituyendo esto último en la relación ( 4 ) :

7 , de donde :
s*

Rpta

( b ) ABC..., es el polígono regular de n


lados y área S, inscrito. ( lado i n y
apotema an) A’B’C’..., es el polígono
regular de n lados y área S \ circuns­
crito. ( Lado Lny apotema An) AMBN.
es el polígono de 2n lados, circunscri­
to y área X, hallada anteriormente.
( Lado ¿2n y apotema a ^ ).
M’N’G..., es el polígono de 2n lados,
circunscrito, de área Y, a calcular.
( apotema A2n)

Los polígonos M’N’G... y AMB..., de 2n lados cada uno son semejantes. Escribiremos la
relación de áreas en función de los cuadrados de sus apotemas.

Y ( A2n )
Así : (1 )
X
( a2n )

Siendo A ^ = OB = R. Tratemos de expresar a2n en función de los parámetros del polígono


ABC... y de las áreas conocidas. Previamente, de la semejanza entre los polígonos de n
lados ABC... y A’B’C’..., tenemos, para la relación de áreas :
2
_S
- , donde An = ON = R ,
S’ (A n )

(an ) _s
luego : (2 )
R2 S1

Si el lector no recuerda las fórmulas de an y f 2n, en función de R y £n, las puede hallar
fácilmente en los triángulos OTB y OPD, usando los teoremas de Pitágoras y Euclides,
respectivam ente:

= | V 4R2 ~ % ............ ( * ) •

y l 2n = v '2 R 2 - 2 R . an ....... ( * * ).
609
Haciendo analogía con ( * ) : a2r> 4R 2 - a2
2n
- i l

Para sustituir en (1 ) : ( a2n )2 = - j ( 4R 2 - f,2n)

1
y con ( * * ) : ( a j 2 = - 4 R2 - ( 2 R 2 - 2R . an )
4

( a 2n )2 = —( R2 + R . an ), sustituyendo aquí lo hallado en ( 2 )


\

R2 + R . R
( a2n )2 = 1 S‘ /
V

Ahora reemplazamos esto último en (1 ) :. Y ( A 2n )

X
( a 2n )

Y R
/ \
X 1 R2 + R2
2 s* /

De donde, luego de cancelar R2 y con lo hallado en la parte ( a ) para X

es la respuesta.

15) Conociendo las áreas Sn y S2n, respectivas a dos polígonos regulares inscritos en una misma
circunferencia, uno de n lados y el otro de 2n lados, hallar el área para el polígono regular
de 4n lados inscrito en la misma circunferencia.

Solución
Consideremos el gráfico adjunto, don­
de se indican los lados respectivos.
Los triángulos MOB y POB, tienen en
común la altura BP, entonces las
áreas serán entre sí como OM y OP:

MOB OM ’4n
, siendo S MOB
;POB OP 4n

2n
POB , OM = R y OP = a2n,
4n
sustituyendo en la relación anterior y

simplificando luego : s 4n R
(1 )•
S 2n a 2n

610
Así mismo, los triángulos AQO y AOB, tienen la misma altura AQ, luego :
S„ e _ s2n
aqo _ Q Q t d o n d e SAQO = ^ , SAQB = , OQ = a n y OB = R; al sustituir y
OB 2n ’ S aob = ^
AOB

’n n
simplificar en lo a n te rio r: (2 )
2n R
Los apotemas nos ayudarán a relacionar las expresiones ( 1 ) y ( 2 ). Como sabemos
2 2 / 2
a 2n = ~~ * 2 n ^ ^ 2n = V ^ ^ ■a n , al combinar estas fórmulas :

a2 n = ^ 4 R 2 - ( 2 R 2 - 2 R . a n ) -» a2n = ^ 2 R2 + 2R . an (3 )

Sn _ an n
De la expresión ( 2 ) : g p , despejemos : an - R
2n 2n )
\

n
para sustituir en ( 3 ) : a2n = — Í2 R 2 + 2 R R
2n

^ aquí- : R1 2S S 2S
y de — —— ,1o que reemplazamos en ( t ) : - n= _____ De donde
a 2n S n + S 2n $2n + $ 2n

Rpta

RESUMEN

Fó rm ulas para el c á lc u lo de áreas en reg io n es p o lig o n a le s

1.- CUADRADO:

2.- RECTANGULO Y ROMBOIDE

S = b. h

611
3.- ROMBO:

S = CD . BH

4.- TR APE C IO :

B
O S = ( m ediana) . h

* Además, si M es punto medio de A B :

S = CD . MQ

5.- TRIANGULO ( Fórmula básica )

6.- FORMULA TRIGONOMETRICA EN TRIANGULOS:

7.- TRIANGULO RECTANGULO:

612
8 TRIANGULO EQUILATERO

9.- FORMULA TRIGONOMETRICA EN CUADRILATEROS:

a) CO NVEXO S:

b) NO CONVEXOS:
B

10.- AREA DEL TRIANGULO, EN FUNCION DEL SEMIPERIMETRO p Y EL INRADIO r

S = p.r

a + b + c
P =

A
11.- CUADRILATERO CIRCUNSCRITO:

S = p.r

a + b +c + d
P =

613
12.- FORMULA DE HERON:

(P sem iperim etro)

S = V p ( p " ~ a" ) ( p - b) ( p - c)

13.- AREA DEL TRIANGULO EN FUNCION DEL CIRCUNRADIO R:

14.- AREA DEL TRIANGULO EN FUNCION DE UN EXRADIO:

S = ( p - a ). r

* También:

S = ( p - b ) . r,

S = (p - c ). r

15.- AREA DEL TRIANGULO EN FUNCION DEL INRADIO Y LOS EXRADIOS

S = ,/ r • ra . rb . r,

16.- CUADRILATERO INSCRITO O INSCRIPTIBLE:

a+b+c+d

Fórmula de Bramma-Guptha

s = J(p - a )(p - b)(p - c )(p - d)

614
17.- CUADRILATERO INSCRITO Y CIRCUNSCRITO:

S = y¡ abcó b _

( Fórmula de Leudesdorf)

18.- POLIGONO CIRCUNSCRITO:

S = p.r

p -> semiperímetro

19.- POLIGONO REGULAR

2a.Forma : Hallando
'A AOB " 2 ' ^n ' a^n

Reemplazando en ( I ): polig n ( ^ n ' aPn)

/ \
n . f.n
polig 2 ■ a Pn
\

t ^ p o lig . ” P ■ a Pn

615
p -4 semiperímetro
apn -? longitud del apotema

* Para triángulos rectángulos, se demuestran

20.- EN FUNCION DEL INRADIO Y EL EXRADIO RELATIVO A LA HIPOTENUSA: ( B = 90° )

S = r . r.

21.- EN FUNCION DE LOS EXRADIOS RELATIVOS A LOS CATETOS:

23.- CIRCUNFERENCIA EX-INSCRITA RELATIVA A LA HIPOTENUSA:

616
24 CIRCUNFERENCIA EX-INSCRITA RELATIVA A UN CATETO

S = AT . TC

RELACIONES UTILES EN LOS TRIANGULOS

R circunradio
r inradio
ra • rb - rc ■ e x r a d ío s

h a - h b ' h c - a lt u r a s

1°) r*a + brK +e r, = 4R + r

( Relación de S te in e r)

2o)
VIL- REGIONES CIRCULARES

C írculo.- Es la región del plano limitada por una circunferencia.

VII.1. Teorema.- E! área de todo círculo es igual al semiproducto de la longitud de su circunferencia


y el radio.

D em ostración

*• En efecto. Sabemos que el área del


polígono regular es igual al producto
del semiperímetro y apotema. Aumen­
tando indefinidamente la cantidad de
lados del polígono, su perímetro se
aproxima cada vez más a la longitud
de la circunferencia L.
Luego:
S p o lig = P• 3n ................. ( 1 )•
regular

Cuando n -> «, el área del polígono regular : -> Scírcul0

an —> R y p —> De modo que al sustituir en ( 1 ) :


2

S c írc u lo = {-^ 4^2


................ ( 2 )•

VII.2. C orolario.- El área de todo círcülo es directamente proporcional al cuadrado de su radio.


Para esto, reemplazando L. = 2 k R , en la expresión ( 2 ) :

c _ ( 2 tcR ) R
círculo -------------------

* Nota: Además, en función del diámetro D : Scírcul0 = _


4

VII.3. Sector C ircular.- Se denomina así a la porción de círculo limitada por dos radios

Teorema.- El área de todo sector cir­


cular es igual al semiproducto de su
respectivo arco y el radio.

sector
AOB

* La demostración se deja ai lector.


( Ver sector poligonal re g u la r)

618
VII.4. Corolario.- El área de todo sector circular de radio R y ángulo central a°t es

VII.5. Segm ento C ircular.- Es la porción de círculo limitada por una cuerda y su respectivo arco

* Para el área del segmento adjunto

S s e g m e n t o = S s e c t o r - St r i á n g u l o
AOS AC*

VII.6.Zona o faja circu la r.- Es la porción de círculo limitada por cuerdas paralelas.

(a) Las bases a un mismo lado (b) Las bases a diferentes lados del centro
del c e n tro :
* bases : BC y AD
* NE y MF : bases

Sz o n a -~ Ss e c t o r - Ss e g m e n t o - St r i á n g u l o = s O
zona se cto re s t r iá n g u lo s
leer *of he m of ASCO (AOS)U(COO) (BOC)U(AOO)

VII.7. Aro o Corona circu la r.- Se llama así a la


región del plano exterior a la menor de
dos circunferencias concéntricas e inte­
rior a la mayor.

= 71 ( R2 - r2 )
corona

619
VII.8. Trapecio circular.- Es la porción de corona circular, determinada al trazar dos radios
mayores.

t r a p e c io
= Ss e c t o r se c to r
ABOC AC* coo

an
( R2 - r2)
t r a p e c io
ABOC 360

LUNULAS DE HIPOCRATES

Al tomar los lados de un triángulo rectángulo como diámetro de semicircunferencias :

1°)

Demostración

Por el teorema VI.6


O i O — O
s e m ic ír c u lo s e m ic ír c u lo s e m ic ír c u lo

AB BC AC

(X + S se g m e n to
) + ( Y + Ss e g m e n to ) = s se g m e n to
+ Ss e g m e n t o + s A ABC
AB BC AB BC

De donde, efectivamente : X + Y = SA ABC


2°) W - Z = SA ABC

Demostración

Con el teorema VI.6 :


B

^ s e m ic ír c u lo ^ s e m ic ír c u lo ^ s e m ic ír c u lo
AB BC AC

( Z + U + ^segm ento ) + ( ^ + V + S seg ment0) -


AH HC

- W + Ssegmenl0 +
AH HC

ordenando convenientemente a fin de tener SAABC y simplificando:

( Z + U + V ) + Z = W

S aabc + Z = W S AABC = W -Z

** Considerando las circunferencias, cuyos diámetros son los lados del triángulo rectángulo

620
3o) W = X +Y + Z
i

D em ostración

Por las relaciones anteriores :

W - Z = S A ABC

X + Y = SAABC

De donde, al igualar los primeros


miembros:
W -Z = X + Y

W = X+Y+Z

PRO BLEM AS RESUELTO S

* Nota.- En algunos problemas donde no sea necesario resaltar el ángulo central del sector
circular al que hagamos referencia escribiremos las expresiones directas para el área, como
una fracción del círculo correspondiente. Así por ejemplo :

Un octavo de círculo Un sexto de círculo


Un cuarto de círculo
rcR2 rcR2
rcR2
8

Un doceavo de círculo
Un tercio de círculo
rcR2 rcR2
12

90' 1 45 1 6<F 1 .
Ya q u e : —, -------- = —, etc.
360° 4 ' 360° 8 360* 6

621
1)
B C

Hallar el área de la región sombreada, si


4cm los vértices del cuadrado ABCD son
centros de los cuartos de circunferencia
de igual radio.

A D

Solución

Se tiene : Stotal = 8 S , .......... ( 1 ).

Siendo : S, = 6 M
2

A 2 n

Reemplazando en ( 1 ) :

RPta; Siotai = 8 ( Jt - 2 )cm2

2) Sean las regiones A 1 y A2 limitadas por dos circunferencias de igual radio tal que :
A1 n A2 = 100 je m2 y : A, u A2 = 400 7tm2 . Hallar el radio de las circunferencias.

Solución

Del gráfico, con los datos :

A1 u A2 -» 2X + Y = 400 n m2 ... ( 1 ). A A

A1 n Y = 100 rc m2.......... ( 2 ).

Restando miembo a miembro :

2X = 300 rc m2 -> X = 150 ti m2

Luego : k R2 = X + Y

k R2 = 250 ti m2

R = 5-ylO m

622
3)
Los vértices de un hexágono regular
son los centros de seis circunferencias
iguales y tangentes ( según muestra la
fig u ra ).

Hallar el área de la región sombreada en


función del lado waMdel hexágono.

Solución
El área de la región sombreada es igual a la
del hexágono, menos los seis sectores. Cada

sector tiene radio — y ángulo central 120

El área de cada sector es :

c 1 ' a n2 na
S1 = —71
/ 12

Luego . ^so m b re a d a ^hexágono ® ^1

/ 2 N
na
sombreada = — a2 J~3 - 6
2 y \ 12 /

a
Rpta: Ssombreada ( 3 / 3 - 7t)
"2

4)
D La longitud del lado del triángulo equilátero
ABC es 2 a . Sobre las prolongaciones de AB y
AC se toman BD = a y CE = a, trazándose
luego los arcos DM y EM con centros en B y C,
así como DE con centro en A.
Hallar el área de la región encerrada por el
triángulo curvilíneo DME.

Solución
Del gráfico, el área X de la región
sombreada se evalúa como:

^ = ® se cto r ■ ^ triá n g u lo ' S , )


DAE ABC

\
7t( 3a )2 (2 a )2 ^ 3 na
X = - 2
/
623
De donde

Rpta: X =

5)
En la figura adjunta, AH, HB y AB
son diámetros y CH es perpendicular
a AB. Hallar el área de ia región
sombreada en función de CH.

Solución
Por diferencia, el área X será :

X =
AB AH HB

Expresamos las áreas semicirculares en función de los diámetros AH, HB y AB = AH + HB

n AB rc AH rcHB
X =
8 8 8

x=* AB - AH + HB
8

x = * ( AH + HB )2 - ( AH2 + HB
8
71
De donde : X = —( AH . HB ) ............( 1 ).

Ademas por relaciones métricas sabemos que : AH . HB = CH ............( 2 ).

Reemplazando ( 2 ) en (1 ) :

Rpta: X = -C H 2
4

6)

La figura muestra dos circunferencias


concéntricas de centro O. AB es una
cuerda de la mayor, tangente a la me­
nor.

Hallar el área de la corona en función de


AB.

624
Solución
* Se tiene : Scorona = rc( R2 - r2)

* En el A MOB : R2 - r2 = MB

Siendo : MB =

AB
R2 - r2 =

Sustituyendo ( 2 ) en (1 )
71
Rpta: ^corona A AB
4

7) Hallar la longitud del radio de una circunferencia interior a un círculo de radio R, que
determina dos regiones equivalentes.

Solución
1
* Del enunciado : Scircu,0 = - ( S círcul0)
menor mayor

Es decir : rcx2 = —( rcR2 )

De d o n d e :

_ o /2
Rpta: X = R

8) Sobre el radio OB de un cuarto de círculo AOB, de centro O, se toma el punto P, de modo


que PB = 2 - /2 cm. Con centro en P y radio PB se traza un arco de circunferencia que corta
a OA en el punto N.
Si AN = PB, hallar el área de la región encerrada por *AEÍ, AN y NB.

Solución

Del g rá fico :

S = Sseo tor - S sec tor - S triángulo (i)


AOB NPB NOP

Como AN = PB ON = OP y en ANOP,
isósceles , ^ NPB = 135° y ON = OP = 2
P 2 J2
Luego: OA = (2 + 2/ 2 )

625
Entonces:
ti AO
sec tor : =f(2 + 2/2) en (1 ):
AOB
135 tü
sec tor ( 2 / 2 2 ) = 3 tc Rpta: S = 2 ( n j 2 - 1) cm:
NPB 360

2x2
^triángulo = 2
NOP

9) Hallar el área de una zona, en círculo de radio R, sabiendo que las bases son los lados del
triángulo equilátero y hexágono regular, inscritos, situados a un mismo lado del centro.

Solución

Sean : AB = 6 y CD = l
, - 3
60'
AB = 60° y CD = 120 O

Luego : D

^zona — S seC(0r — segmento ~ ^triángulo (1 )•


COD AB COD

S ie n d o :

120p7tR2 tiR2
* ^ s e c to r
COD 360

_ 60°7cR2 R2 / 3
^segm ento “ ^ s e c to r ^triángulo
AB AOB AOB 360°

_ 7iR2 R2 / 3
segmento
AB

_ R2 / 3
S ingóle = ^ Sen 120° = ^

Al sustituir en (1 ):

7ER ti R2 R2 ^ 3 ) R2 / 3
zona
\ /

tcR
zona

626
En la figura adjunta se tiene un cuarto
de círculo con centro O y un semicír­
culo de diámetro A B .

Demostrar que las regiones: triangu­


lar AOB y lunular AMB, son equiva­
lentes.

Solución

Tenemos : Str¡á lo + S segmento ~ ^sector


AOB AB AOB

7i r
esto es : Striá (0 + S segmento ( 1)
AOB AB

También :
^lúnula + § segmento ^semicírculo
AMB AB AB

7t AB
'lúnula + S segmento , y c o m o : AB = r ^ 2
AMB AB 8

Entonces:

ti r
^lúnula + S segmento (2 )
AMB AB

Igualando ahora los primeros miembros de las relaciones (1 ) y ( 2 )


c
'-,,riángi segmento ~ ^lúnula + Ssegmento
triánaulo + ^seamento
AOB AB AMB AB

^triángulo - lúnula , como se quería demostrar.


AOB AMB

En la figura adjunta, AOBC es un


cuadrado de lado “a”.

AB y CD han sido trazados con


centro en O.

H a lla r el á re a de la re g ió n
sombreada.
Solución
Del g rá fico :
S som b = X + Y ..................... ( 1 ) .

S ie n d o :

X = ^cuadrado " ^sector


AOBC AOB

X = a 2 - — ..................( 2 ) .
4
D
También : Y = SseC(0r —^triángulo
COD OBC

45° K / r7r \2 a2
Y = a J 2 ) ---------
3 6 0 °v v 1 2

Y = IIH L - ........( 3 ) .
4 2
2 na 2 na 2
Reemplazando ( 2 ) y ( 3 ) en ( 1 ) : Ssomb a¿ + ------

a2
RPta: somb = Y

12)

Hallar el área de la región sombreada Sx,


sabiendo que la suma de las áreas S 1 , S2
y S3 es 100 cm2.

ABCD es cuadrado y D centro del arco AC.

Solución
* Dato : St + S2 + S3 = 100 cm2 ................. (1 )

* Incógnita: Sx

* Del gráfico , la longitud del diámetro del cír­


culo es igual a AD.

Luego :

ti AD ti A D
círculo S v + s* = ..(2).

tcAD tcAD
* Para el cuarto de círculo ADC : Ssec tor —> Si + So + So + S — (3 )
¿OC

628
Siendo iguales los segundos miembros de las expresiones (2 ) y (3 )t los primeros miembros
también deben serlo :

Sx + S’ = S1 + S2 + S3 + S ’

" S x ~ S 1 + S 2 + S 3

Con (1 ) :

Rpta: S„ = 100 cm2.


A

En el cuadrado ABCD , AB es diámetro


de la semicircunferencia y radio del
cuarto de círculo. Si AB = 2a.

Hallar el área de la región sombreada.

Solución
LLamando X el área pedida

X = ^ se cto r ^segmento (1 )
AED AM

Siendo

_ 45°7i( 2 a ) 7ia
s e c t o r
S sec tor
AED 360° AED
(2 )
Para el segmento circular AM, como X OfiM = 45° y OA = OM , en A AOM
/\
AOM = 90°.

Luego :
S segmento ^sector ^triánaulo
AM AOM AOM

Tía
segmento
AM

Reemplazando ( 2 ) y ( 3 ) en ( 1 ) :
\ 2
7t a na rca¿ a.2
X = + —
/

Rpta: X = — ( rc + 2 )

629
14)

La figura muestra dos semicírculos de diá­


metros OA = OB y un cuarto de círculo con
centro en O.

Hallar el área de la región sombreada.

Solución

Del gráfico, al trazar OP, BP y PA,


se observa que los segmentos cir­
culares BP, ONP, PA y OMP tie­
nen igual área. Luego, el área de
la región pedida es equivalente al
área del segmento circular AB :
7lR R
somb
4

R2
• •
somb ( n - 2)

15)

En la figura adjunta, AC y BD son diá­


metros perpendiculares del círculo de
radio R.

Hallar el área de la región sombreada.

Solución
Se tiene :

“ ^ c í r c u l o S s e c t o r

AB AOB

7CAB nR
Si =

S , - - R 2 * '• ^ t o t a i - 4 S¡

® total - rc R Rpta

630
16)

Tomando como diámetros los lados de un


hexágono regular se han construido inte­
riormente semicircunferencias.

solución

* Del gráfico, notamos que el área de la


región pedida, equivale a un círculo
de diámetro a.

7ta‘
Rpta: S =

17)

Hallar el área de la región sombreada, si


AOB es un sector circular de radio
/6 c m .

Solución
Se tiene :

somb ^sec tor S cuadrilátero


AOB cóncavo AOBM (1 )•

n(j6)
D onde: sector = K ( 2 ). ( Sexta parte del círculo )
AOB

Para el cuadrilátero cóncavo AOBM, como M es baricentro del triángulo AOB, entonces :

2
cuadrilátero o ^triángulo Ve = V3 (3).
cóncavo AOBM ^ v AOB 3\ /

De ( 2 ) y ( 3 ) en ( 1 ) :

RPta: S som b = ( * - / 3 ) cm

631
18) En un sector circular de ángulo central 60° y radio R se halla inscrita una circunferencia
Hallar el área de la región exterior a la circunferencia e interior al sector.

Solución
Dato : R

Del gráfico, el área de la región men­


cionada e s :
q _ ^sector “ ^circulo
somb ~~ AOB P

k R¿ 2
somb k r ( 1 ).

* C álculo de r.- En A ONP = ( 30°, 6 0 °)


R
OP = 2 . NP R - r = 2r (2)
3

tcR 'R
Sustituyendo ( 2 ) en (1 ) : Ssomb - K
\ 3 /

nR ‘
Rpta: Sgomb =
18

19) Se tienen tres círculos A, B y C, de igual radio, de modo que sus circunferencias son
secantes entre sí, con un punto común a las tres.
Si : A n B = 53jcm2 , A n C = 49 n m2 , B n C = 47 n m2 y A u B u C = 990 n m2.
Hallar la longitud del radio de los círculos.

Solución
Se tiene :

3 tc r2 = ( A u B u C ) - ( A n B) u
( A n C ) u ( B n C)

3 TU2 = 99071 - ( 5371 + 4971 + 47tc ) B

3 k r2 = 841 ti
29
De donde : r = m
V3

20)

B
Hallar el área de la porción de círculo
mostrada, sabiendo que los arcos AB y
BC miden 90° y 45°, respectivamente.
Solución
Del gráfico :
B

Ssomb - ^segmento ^segmento (1 )


ABC BC

D onde:

segmento ~ ^sector ^triángulo


ABC AOC AOC

R
R
135° n R2 V
segmento
ABC 360°

3tcR2 R2 J~2
segmento g 4 ■•* ( 2 ).
ABC

A dem ás:
45° rc R2 R.R
segmento ^sector ^triángulo Sen 45®
BC BOC BOC 360'

tcR R2 / 2
segmento (3 )
BC 8

Sustituyendo lo hallado en ( 2 ) y ( 3 ), en (1 ) :

3nR 2 R2 V~2 ^ ( tiR 2 R2 J 2 '


som b 8 4 8 4
V ) V /

rcR
Rpta: Ssomb

La figura muestra un cuarto de círculo y


un semicírculo.
*

AM = OM = 2 ^ 3 cm. y MN 1 OA.
B Hallar el área de la región sombreada.

633
Solución

Trazando ON :
A
A OMN : ON = 2(OM) = 4 / 3
2-Jz
¥ MNO = 30° y E\ x
M1 N
^ N O M = 60°
2^3 60
^ **
■ ^
A dem ás: MN = (OM)</3 0

MN = 6

El área X :

X SggcjQr SS6Cl0r
AON AME OMN (1 )

K{ 4 V~3 )
Donde: S sec tor = 871 (2 )
AON

71( 2 / 3 )
sec tor = 371 (3 )
AME

( OM )( MN ) ( 2 / 3 )( 6 ) ^ ^
^triángulo ~ 0 --------------------r - o y o ... ( 4 )
OMN ¿ 4

Sustituyendo ( 2 ), ( 3 ) y ( 4 ) en ( 1 ) :
X = 8 tc - 3 ti - 6 / 3

Rpta: X = ( 5 tí - 6y¡~3 )cm

En la figura adjunta, el área de la región


sombreada, limitada por la circunferen­
cia y las semicircunferencias, es “S”.
Hallar el área del hexágono regular
ABCDEF.
Solución
Del dato, en el gráfico :

2(S1 + S2) = S -> S1 + S2 = - ... ( 1 ).

Los triángulos AFE y AOE son con­


gruentes y AOE es equivalente con AOB:

®AFE = ^AOE ~ ®AOB ~ ^


Con las lúnulas de Hipócrates en el trián­
gulo rectángulo EAB:

EAB = s, + s;

2X = — -» X =-
2 4

Luego :

ABCDEF
- 2 (S efab) = 2 ( 3X ) = 6x —> SABCDEF - 6

Rpta: SABCDEF

Hallar el área de la región sombreada,


donde EF mide 60° y los demás arcos
corresponden a semicircunferencias de
igual diámetro.

Solución

Como los semicírculos tienen igual diámetro,


la figura anterior equivalente a la adjunta,
luego de trasladar la región correspondiente
al s e m ic írc u lo en CD a DE y el de
AB a AF.

El área de esta nueva región sombreada es:

som b “ hexágono ^semicírculo +s segm ento . . ( 1 ).


\ ABCDEF CB /
635
/
Donde : = 6
rR2y[3 '
^hexágono ~ ® ^ triá n ^ u lo
ABCDEF \ / \ J

ti CB 7cR
‘semicírculo ” Q
CB 8

_ TtR2 R2 / 3
^segmento ” ^seoíor ^triángulo
EF EOF EOF

Sustituyendo en (1 ) :

1
Csl
r R2 / 3 '

QC
f rcR2 R2 / 3 \
somb ~ 6 +

1
4 ) 8 ,^ 6 4 )

Ssomb = I R 2 V 3 - ^ r2 = 3 0 /3 - * ) Rpta
24

24)

1\
Hallar el área de la región sombreada,
6cm
donde los lados del cuadrado ABCD son
diámetros de las semicircunferencias.

Solución
* Llamando S, el área de un pétalo, el área
total será :
^total = ( ^ )■
-i
* Del gráfico : OM = OP = MP
f «
A OMP es equilátero y : ^ O M P = 60°,
)f n o p = 30° = x- E0M y ¥ E 0P = 30°-

Siendo . Si —^segmento + ^segmento + Ssector. *


OE OP EOP f
donde:
segmento — ^segmento , por lo que :
OE OP

S, = 2 segmento + S sector
V OP / EOP

636
S-t = 2 sec tor ^triángulo + ^ s e c to r
\ OMP OMP EOP
/
60° . k . 32 32 < ¿ 3 > 3 0 ° . 7t . 3 2
Si = 2
360' 360°

9^3 3 n _ 1571 _ 9 .^ 3
= 3 7i - (2 )

• • Reemplazando ( 2 ) en (1 ) :

' 1571 9/3 '


total = 4
V

Rpta: StotaJ = ( 15 n - 18-^3 ) cm:

En la figura adjunta, el A ABC es


equilátero, cuyo lado mide 2a. O es
punto medio de A C , EOF ha sido traza­
do con centro en B y AC es diámetro
de la semicircunferencia. Hallar el área
de la región sombreada.

Solución

Trazando OM, ON y BO , como


OM = ON = a, deducimos que los
triá n g u lo s AOM y NOC son
equiláteros. Luego, AM = MB = a,

BO = a / 3 y ^ MON = 6 0 °. En
seguida:

Ssombreada — ^sector + 2( S^) ... ( 1 ).


MON

S ie n d o : na
sec tor
(2 )
MON

Y para la región EOM “ ^ s e c to r S OMB


OBE

donde : SOMB = —( SABC ) , en lo anterior:

637
7c( a^[3 ) 1 ( 2 a ) 2/ 3
®i “ ^ s e c to r ” — ( S abc ) =
QBE 12

na2 a2 ,j~3
Si = (3 )

Sustituyendo ( 2 ) y ( 3 ) en (1 ) ;

na a2 / 3 '
°<5sombreada “- * ^a +*"9^

/

Rpta: S sombreada (471 - 3 ^ 3 )

26)

A, B y C son tres vértices consecutivos


de un hexágono regular inscrito en la
circunferencia de centro O y radio R.
Haciendo centro en A se han trazado los
arcos BO y CE.

Hallar el área de la región sombreada.

Solución
Como A, B y C son vértices de un
hexágono regular, deducimos que :

BC = 60° , CE = 120° = AE.


R
^O A B = 60° = X- EAC, OM =

AE = *3 = R / 3

La región sombreada se compone de


las regiones ONBC y OCTE. Eva­
luando por p a rte s :

ONBC sector - S seamento


BOC *% NO
\
/
ONBC = S sec tor sector Stránguio
BOC OAB OAB /

Obsérvese que los sectores BOC y OAB son equivalentes por tener igual radio R y ángulos
centrales de 60°, luego, lo anterior se reduce a :

638
R2 / 3
S O N B C - ^triángulo . Esto es : S ONBC “ -----^ ----- (1 ).
OAB

El área de la región OCTE es igual a la del sector EAC, disminuida en los triángulos OAC y
OEA, los que a su vez son equivalentes al triángulo OAB.
Así:
/
OCTE sector ^trián gu lo + ^ triá n ^ ilo
EAC V OAC

f
OCTE = S sec tor - 2 triangulo
EAC OAB j

,(R /3) ' R2^ /"^ 'j _ 71R 2 R2 / 3


b OCTE ” ---------1-------------- ¿ (2)
\

Finalmente, con lo hallado en ( 1 ) y ( 2 ) :

= R2 y r i tcr2 r2 / 3
total “= S
°O N B C + s, C T E
_r °O

R
R p ta : S totaJ= — (2n-f3).

La lo n g itu d del lado del triángulo


equilátero ABC es 2a. AB y BC son
diámetros de las semicircunferencias y
M punto medio de AC. Con centro en B
se han trazado los arcos EMF y AC.
Hallar el área de la región sombreada.

Solución

La longitud de la altura BM del trián-


AR
guio equilátero ABC.esBM = — J 3
2 y

BM = a ^ 3 = BE. Siendo O
centro de la semicircunferencia AEB,
de radio a, concluimos fácilmente que
BE = t y porloqueE B = 120°. Luego
AE = 60° = AOE y % ABE = 30°.

639
La región APMQC es un trapecio circular con centro en B, ángulo central 60° y radios
BA = 2 a , BM = a^J~3. El área respectiva :

60° n
APMQC - (2a)2 - ( a / 3 )
360°

na
APMQC “ (1)

Las regiones AEP y CQF son equivalentes. El área de la región AEP es igual a la del sector
AOE, disminuida en la región EOP.
De modo que podemos e s c rib ir:

^AEP - ^sector - S EOP (2 )


AOE

na
S ie n d o : ssec tor (3 )
AOE

V el área de la región EOP igual a la del sector EBP, menos la del triángulo EOB :

EOP = S
“ °sector - StriÁ
°triár>gulo
EBP EOB

n(a/3)2 a2 / 3
(4)
EOP “
12

Sustituyendo ( 3 ) y ( 4 ) en ( 2 ) :

na na2 a2 ^ 3
AEP
\

-/3 na
AEP = = SCQF
12

Finalmente el área de la región pedida, con ( 1 ) :

s**, = sAEP
.~ +' wsAPMQC + ^Q C F ” ^ A E P ) + ^A P M Q C

/
/3 na 2 7ta'
total = 2 +
12
\ /

to ta l

640
28)

La figura muestra tres circunferencias


de igual radio “R" ortogonales entre sí,
dos a dos.
Hallar el área de la región sombreada.

Solución
Se tie n e :
somb
= 3 (S MUTN) - 2 ( S lUTN
m ) (1).

En el hexágono MOEQFP, las medi­


das de los ángulos en M, E y F son
iguales a 90°.
Siendo la suma de las medidas de
todos los ángulos interiores.
180° ( 6 - 2 ) = 720°

Luego : Y MOE = Y EQF = Y MPF,


con medidas 150° cada uno.

Entonces: MNTE = 150°. Además los cuadriláteros ONQE y OMPT son cuadrados. Por lo
que Y EON = Y MOT = 90° y los arcos respectivos: ETN = MNT = 90°.De donde hallamos
NT = 30°.

NT = f-\2 -> NT = R )/ 2 - / 3

* C álculo del : SUTN ,egm(


^seamento + ^triánaulo
iangu
\ / UfN

UTN = 3 Ssec tor ^triángulo + ^triángulo


v NOT NOT UTN

' 30°tcR2 R x R
UTN = 3 sen 30 + NT
V 360'

R2 / 3
Esto es : S uj N = (2)

641
* Cálculo del SMUTN :

^MUTN “ 2( SMNT)

jtR 2 R2 ' O
9 - O , '0 \ %
MUTN “ ^ 2 */ u .. \ *
y

j iR
MUTN - R (3 )

Reemplazando ( 2 ) y ( 3 ) en ( 1 ) :

/
k R¿ 3R r2/ f
somb = 3 ^ - - r2 - 2 +
\ v /

RP^: = (ti - ) R2

29)

En la figura, M es punto medio del arco


circular BC. É M _ lB C y EÑ | | ÁM.
Demostrar que MN divide el área de la
región ABMC, en dos equivalentes.

Solución
De los datos : BE = EC. Tracemos
AE, interceptando a MN en el punto
P. Como el área de la región BME es
la mitad del área de BMC y el área de
ABE es la mitad de ABC, entonces la
región ABME es la mitad de ABMC.
Para el trapecio ANEM, sabemos que
los triángulos ANP y EPM son equiva­
lentes, por lo que las regiones ABMN
y ABME tienen igual área. (La mitad
del área ABMC).
Con esto demostramos que MN divi­
de el área de la región ABMC en dos
equivalentes : ABMN y NMC.

642
Hallar el área de la región sombreada, si
se sabe que la medida del ángulo AOB
y la del ángulo A'O ’B’ es 60°.
Los segmentos O' A, O 'B son tangen­
tes a la circunferencia con centro O y
radio R, y los segmentos O" A', 0 "B '
son tangentes a la circunferencia de
centro O ’.

Solución
A AOB, equilátero : AB = R. ^ A O ’B = 120 O

EnelAOAO’ : O’A = y en A AMO’,


3

^ O’AM o■ M - 2 A - M .
2 6
Llamando S, al área de la región común a los
círculos de centros O y O’ i
/ / \ A
V3
R. R
' 60° JtR2 R2 / 3 ^ ( n j3 )
120° K
S< = +
360° 4 360° 3
V i ^ J

„ 5 n R 2 R2 J~3
oí —— — 1 (1 )
18 3

Para el área S2 de la región común a los círculos de centros O’ y O”, por ser semejante a
la a n te rio r:

( o 1A ') ( O'A‘ )
—y So — . s1

w (OÁ)!

’a 1 _
Siendo O’A’ = rvA
O’A _
= jj

r R/3 I

So /
R
643
1
De donde : S2 = — S1
O
1
Luego : StotaJ = S, + S2 = S1 + - S,

\
5 tiR2 R2 / 3
total
= - S , ; con (1 ) : S total “
V 18
j

Rpta: S,otal= ^ - ( 1 0 ^ - 1 2 / 3 )

31) Un sector circular tiene por perímetro : L


El valor máximo del área del sector, es :

A) B) L2 C )7 t D)?c E)
16 16

Solución

- Llamando x, el radio; la longitud del


arco AB, será : = L - 2x

- El área del s e c to r:

1
s - *£-( ¿ab ) x

S = ¿ ( L - 2x )x

Efectuando : S = — - x2
2

Completando cuadrados en el 2o miembro :

L
x -----
16 \ 4

Luego, S será máximo, cuando | x - — sea mínimo; es decir, cuando


\

Por lo tanto ; S m áxim o


16

Rpta: ( E ).

644
En la figura adjunta , AB es diámetro y
AM = MC.
Hallar el área de la región sombreada, si el
radio mide : r = 10u.

A) 7CU2 B) 2nu2 C) 5 k u 2

D) 10rcu2 E) 20rcu2

Solución

Trazamos OM. Entonces, como


AM = MC : OM 1 AC

También : ACB = 90°

OMCB, es un trapecio.
Por propiedad:
área A MPC = área A OPB

Entonces , la región propuesta ,


equivale al sector COB :
f 36°
S s o m b r e a d a = 7C102 so m b re a d a
= 1071 U 2
V 360'

Rpta: ( D ).

645
PROBLEMAS PROPUESTOS

1) En un triángulo ABC, de Baricentro G, se trazan AG, BG, CG y se determinan M, N y Q,


Baricentros de los triángulos ABG, BGC y AGC, respectivamente. Unir estos tres puntos y
h a lla r:
área ( M N Q )
área ( A B C )

2 1 4 2 4
A >9 B )9 C )? D)ñ E>25

2) En un trapecio ABCD, de bases AD y BC, AD = 4 BC, exteriormente se construyen los


triángulos equiláteros BCE y CDF. Hallar el área del triángulo CEF, si el área del triángulo
ADF es 40 cm2.

A) 8 cm2 B) 10 cm2 C) 12 cm2 D) 20 cm2 E) 15 cm2

3) En un A ABC, los lados miden : AB = 13cm., BC = 14cm. y AC = 15cm. La prolongación de


la mediana AM intercepta a la bisectriz exterior del ángulo B en N.
Hallar el área del triángulo MBN.

A) 45 cm B) 48 cm2 C) 36 cm D) 64 cm: E) 49 cm2

4) Hallar el área del triángulo rectángulo MNE si MN y NE son diámetros y PQ tangente


Dato : PQ = n.

n n
A) B)
8

n
C) D) n:

E) 2n2

5) En un triángulo rectángulo de circunradio 8,5m. e inradio 3m., hallar el área.

A) 28 m2 B) 34 m C) 60 m2 D) 51 m: E) 54 nrv

6) En el gráfico adjunto :

TN = NL = 2 cm,

5 ( ED ) = 7 ( DL ) = 35 cm
646
Hallar el área de la región sombreada.

A) 12 cm2 B) 32 cm2

C) 24 cm2 D) 30 cm2

E) 36 cm2

7) Hallar la longitud del mayor lado de un triángulo, sabiendo que sus exradios miden 8; 12 y
24 cm., repectivamente.

A) 12 cm B) 18 cm C) 16 cm D) 20 cm E) 24 cm

8) En la figura, el área del triángulo PQR es 90 m2. PM = QM ; QN = NR y PE = ED = DR


Hallar el área de la región sombreada.

A) 30 m: B) 29 m2

C) 25 m: D) 26 m2

E) N.A.

9) En la figura, O y O* son centros de las circunferencias de radios 4 y 9 cm.


A, E, F, D, P, son puntos de tangencia. Hallar el área de la región sombreada.

A) 39 cm2 B) 78 cm:

C) 76 cm2 D) 82 cm

E) 58 cm2

10) En un trapecio ABCD, BC A D , se trazan las diagonales. Las áreas de los triángulos BCD
y ACD son de 20 y 60 cm2. Hallar el área del trapecio.

A) 100 cm: B) 80 cm: C) 70 cm2 D) 90 cm: E) N.A.

11) Dado el trapecio MNPQ : MN FP y NG PQ. Hallar el área de la región OPQG, si las
áreas de los triángulos NOP y FOG son 9 m2 y 4 m2.

647
A) 13 m2 B) 15 m2

C) 17 m2 D) 21 m2

E) 18 m2

12) Hallar el área de la región sombreada, si el área del rectángulo EFGH es 20 cm2. 0 y P son
centros de las circunferencias.

A) 4 cm2 B) 5 cm2

C) 8 cm2 D) 6 cm2

E) 10 cm:

13) Hallar el área de la región sombreada, si el área del romboide IJKL es 72 cm2. M y N : puntos
medios.

A) 9 cm2 B) 8 cm2

C) 10 cm2 D) 6 cm2

E) 12 cm2

14) Hallar el área de un trapezoide, sabiendo que las distancias del punto medio de un lado a
los puntos medios de los otros tres, son de 10,‘ 21 y 17 cm., respectivamnte.

A) 336 cm B) 236 cm2 C) 168 cm2 D) 84 cm2 E) N.A.

15) Los lados de un rectángulo miden 15 y 20 m., respectivamente. ¿Cuál es el área del
rectángulo de perímetro 308 m, semejante al rectángulo dado?

A ) 1452 m2 B ) 1456 m2 C) 4608 m2 D) 5806 m2 E) 5808 nrv

16) En la figura : AP 11 CE ; PE 11 CD

área ( APE ) = 16 cm2


área ( CED ) = 4 cm2
H a lla r: área ( PEC )

648
A) 12 cm2 B) 10 cm2

C) 8 cm2 D) 9 cm2

D E) 6 cm2

17) En el paralelogramo RSTU, SP = PT, TQ = UQ y área ( RSTU ) - 48 cm2


H a lla r: área ( TEQL ).

A) 4 cm2 B) 6 cm2

C) 5 cm2 D) 4,5 crrr

E) 5,5 crrr

18) Si el área del cuadrado DEFG es 1 cm2. Hallar el área de la región sombreada.

A)(2-/2)cm 2 B)(/2-1)cm 2

C) 0,75 cm: D ) - — cm
2

E) N.A.

19) En la figura, E es excentro del triángulo TUL . M, N y G son puntos de tangencia


área ( MUN ) = 2 cm2
área ( QPL ) = 6 c m 2

H a lla r: área ( NEP )

A) 8 cm2 B ) 2 /3 c m 2

C) 9 cm2 D) 7,2 crrr

E) 10 cnrv

649
20) En la figura , E y F son puntos cualesquiera, interior y exterior al triángulo JKL.
ÉÑ 1 JK, ET _L KL, FP _L KP y F Q X K Q
área ( PNM ) = 20 cm2
v ' K
área ( U T Q )
área ( R S F )

H a lla r: área

A) 19 cm2

C) 21 cm2

E) N.A.

21) Hallar la relación entre las áreas de los dodecágonos regulares, inscrito y circunscrito a
una misma circunferencia.

2 + J~3 2 + S2 2 + J~3 2-J3


A) g B)^ “ C )_ 4 D)~ 4 E)NA

22) Un segmento AB = 6a está dividido en tres segmentos iguales AC = CD = DB. En uno de


los semiplanos que determina AB se construyen : sobre AC un triángulo equilátero AEC;
sobre CD un cuadrado CFGD y sobre BD como hipotenusa un triángulo rectángulo
isósceles DHB. Trácense, enseguida, los segmentos EF y G H . Hallar el área del polígono
AEFGHB.

A)(6+/3)a2 B)(5+/3)a2 C )(7 + /3 )a 2 D)(4+/3)a2 E) N.A.

23) ¿Cuánto mide el lado de un dodecágono regular de área 1 m2?

A ) ^ 3 - 3 /3 m B ) ^ 6 - 3/3 m ni
O O u

D )-^6-2/3 m £ ) N.A:

24) El área de un octógono regular circunscrito en una circunferencia de radio V , es :

A) 2 ^ / 2 B) 2I-2 (y[2 - 1) C) Sr2 ( J~2 - 1) D) 2I-2 E) N.A.

25) El área del dodecágono regular inscrito en una circunferencia circunscrita a un octógono de
área s / i c m 2, e s :

A) 12 cm2 B) 12-/3 cm2 C) 1 2 ^ 2 cm2 D )6 c m 2 E J ie c m 2


650
26) El radio y el apotema de un polígono regular, son respectivamente R y a. Calcular el radio
de otro polígono regular equivalente al primero y de doble número de lados.

VaR /a R / aR
A)2/aR B )/a R C) D) E)
8

27) Tres circunferencias, de igual radio R, tangentes exteriormente dos a dos en los puntos A,
B y C, determinan el triángulo curvilíneo ABC, de área :

A) R2 ( 2 / 3 - n ) C) R2 ( n - 2 )

tcR
D) E) N.A.
8

28) En la figura : PQ = TL = 90° , PQ = 4 m

Hallar el área de la región sombreada.

A) 4(2 + 7i)m2 B) 3(2 + 7i)m:

C) 5(2 + 7i)m2 D) 6(2 + 7 t)m

E) 3(1 + 7t)m2

Hallar el área del cuadrilátero curvilíneo MNPQ, cuyos vértices son los puntos medios de los
lados del rombo.

A) 2ab - - |( a 2 + b2) B) 2ab - ^ (a2 + b2)

C) 4ab - 1 ( a 2 + b2) D) 2ab - (a2 + b2)

E) N.A.

30) Haciendo centro en cada vértice del


hexágono regular PQJDEF de han
trazado arcos, con radios iguales a la
longitud “a” del lado del polígono.
Hallar el área de la región sombreada.

A) a2 (2 n - / 3 ) B) a2 ( n - ^ 3 )

C) a2 (3ti - 2 / 3 ) D) a2 ( 2 7 t - 3 / 3 )
E) N.A.
651
31)

La figura muestra un cuarto de E


circunferencia y dos semicircun­
ferencias. Demostrar que las áreas
X e Y de las regiones sombreadas,
son iguales.

32) En la figura adjunta, EO, OF y EF son diámetros de las semicircunferencias. MP y MQ han


sido trazados con centros en E y F respectivamente. Hallar el área de la región sombreada,
si EF = 12 cm.

A) 25 Ticm2 M

B) 27 Ttcrn2

C) 21 7ccm2

D) 20 ncm2

E) N.A.

33) Hallar el área del cuadrilátero curvilíneo cuyos vértices son las intersecciones de los arcos
trazados con centros en los vértices del cuadrado MNLP.

A) £ (jt_2) B)^-(k + 3 -J 3 )

2
C ) a 2( 7 t - / 3 ) D ) - y ( 7t + 2 - V 3 )

M P
E) N.A.

34) O, M, P son centros de los arcos MN,


FN y MEN, respectivamente.

Hallar la relación y , entre las áreas

de las regiones indicadas.

A) 1
B) 1/2
C) 2/3
D) 3/4
E) N.A.

652
35) El A AOC es equilátero, AC diámetro de la semicircunferencia y O centro del arco AEC
Hallar el área de la región sombreada.

rc a
c) D)
8 " l6

E) N.A.

36) El lado del hexágono regular ABCDEF mide / 6 cm. El vértice A es centro de ios arcos BF
y CE.
Hallar el área de la región sombreada.

A) ( ti + ^ 3 )cm2 B) (k + 2 J 3 )cm2

C) (rc + 3<J~3)cm2 D) (2k + )cm2

E) N.A.

37) En la figura mostrada NR y MPL son cuartos de circunferencia con centro O y radios “2a1
y “a” respectivamente.
Calcular el área de la región
sombreada.
R

A) a2 (n - 0,7) B) a2 (n - 0,8)

C) a2 (71 - 0,4) D) a2 (tc - 0,3)

E) a2 (n - 0,5)

38) Hallar el área de la región sombreada, si el radio de la circunferencia de centro O, mide 2 cm

AB = CD = 90° y AD = 60

A) (27t + 3 - 3 / 3 ) c m 2

B) (3t i - / 3 ) c m 2

C) (2 ji - / 3 ) cm2

*1C D )A (2 n + 3 - 3 / 3 ) crrv
O

E ) ^ ( 2 ti + 2 - / 3 ) cm2

653
39) Hallar el área de la región sombreada, si AB y EF son diámetros

nR 71R
A) B)
3

R2
C) R2 D)

R
E)

40) Sobre el lado AC del triángulo equilátero ABC se encuentra el centro O de la semicircunferencia
tangente a AB y BC en P y Q. Además el centro del arco EOF es B. S i: AB = a, hallar el
área de la región limitada por la poligonal mixta EPMQFO.

2a

C )a2 ( n - ^ 3 )

E) N.A.

41) En la figura, el triángulo ABC es recto en B e isósceles. La semicircunferencia de centro O


es tangente a AB y B C . El arco BE ha sido trazado con centro en A.
Hallar la relación entre las áreas X, Y, Z de las regiones indicadas.

A) X + Y = Z B )/x Y = Z

C) 2( X + Y ) = Z D) X + Y = 2Z

E) No se relacionan

42) La longitud del lado del triángulo equilátero ABC, es ui n. M, N, P son puntos medios de los
lados y las semicircunferencias tienen su diámetro sobre cada lado. Hallar el área de la región
som breada!5


A )^ r/3 B)
24 24

7C^
C) D )^ r/3
36 48

E) N.A.

654
43) M, N y P son puntos medios de los lados del triángulo equilátero ABC. MB y BN son
diámetros. Los arcos MP y NP tienen sus centros en A y C.
Hallar el área de la región sombreada.

A) ( 21/ 3 - 8ti ) cm2

B) ( 1 8 / 3 - 5ti )c m 2

C) ( 1 6 / 3 - 3tc ) cm2

D) ( 21 / 3 - 5tc ) cm2
E) N.A.

44) En la fig u ra :
AF GH -» equilátero
FG y GH son diámetros y FH ha sido trazado con centro en G. Hallar el área de la región
sombreada.
Dato : FG = 2a.

A )-y (*-/3 ) B) a2 ( 7t - / 2 )

C) a2 ( ti - /3 ) D) 2a2 ( 7t - / 3 )
H
E) N.A.

45) El área de un trapezoide es “S”. Las prolongaciones de dos lados opuestos se cortan en el
punto P. Hallar el área del triángulo que tiene por vértices, el punto P y los puntos medios de
las diagonales del trapezoide.

A )S B )f C )| D )|s E ),|

46) En un triángulo ABC,de lados a, b, c, semiperímetro p y área S, el área del triángulo


exincentral, es:
abe d abe p abe p abe
A> s B)T I “ c )~ "4s~ E> N A

47) Calcular el área del pentágono ABCDE,


si el lado del cuadrado PQRS es 2a.

2 2

A ) ~ ( 35 - 1 3 / 5 ) B )^-(7-/5)

2 2
C )^-(35-1l/5) q) ^ - ( /5 - /3 )

E) N.A.

655
48) En el interior de un triángulo ABC, se toma un punto P y se trazan : PQ, PM y PT,
perpendiculares a los lados. Siendo R el circunradio del triángulo ABC y d la distancia de P
al circuncentro de ABC, demostrar que :
área ( QMT ) _ R2 - d2
área ( A B C ) 4R 2

49) Para el problema anterior, si el punto es exterior a la circunferencia circunscrita al triángulo


ABC, demostrar que:
área ( Q M T ) _ d2 - R2
área ( A B C ) 4R 2

50) Usando los problemas anteriores de demostración; para el gráfico adjunto, demostrar que
la inversa del área del triángulo en la circunferencia inscrita al A ABC es igual a la suma de
las inversas de las áreas de los triángulos en las circunferencias exinscritas.

656
CLAVE

1.- B 11.- D
2.- B 12.- B
3.- E 13.- B
4.- D 14.- A
5.- C 15.- E
6.- D 16.- C
7.- D 17.- C
8.- B 18.- A
9.- B 19.- A
10.- B 20.- C

21.- C 31.-
22 . - C 32.- B
23.- B 33.- B
24.- C 34.- A
25.- A 35.- A
26.- B 36.- C
27.- B 37.- A
28.- A 38.- D
29.- B 39.- C
30.- D 40.- A

41.- A
42.- B
43.- A
44.- C
45.- C
46.- B
47.- E
48.- *
49.- *
50.- *

657
CAPITULO 18

RECTAS Y PLANOS
GEOMETRIA DEL ESPACIO

Como sabemos, la Geometría Plana estudia las figuras planas, esto es aquellas que tienen
todos sus puntos en un mismo plano. Ahora bien, la Geometría del Espacio o Estereométria,
tiene por objeto el estudio de las figuras sólidas o del espacio, es decir, de las figuras cuyos
puntos no pertenecen todos a un mismo plano, sino al espacio tridimensional, por ejemplo
el prisma, la pirámide, el cono, la esfera, etc.

Si se piensa en una superficie llana, perfectamente lisa, sin espesor; que se extiende
indefinidamente en todas las direcciones, se tendrá una buena idea de lo que se supone sea
una superficie plana o simplemente plano. Por ejemplo, si la superficie del tablero de una
mesa perfectamente lisa, que nos da una idea aproximada de una parte de superficie plana,
la imaginamos de extensión ilimitada tendremos la idea de un plano. Queremos decir, que
la superficie lisa del tablero, extendida indefinidamente en todas direcciones, no es el plano,
sino una representación física de un conjunto de puntos que en geometría conocemos como
un plano.

Generalmente, un plano en el espacio ( o más exactamente, una parte del plano ) se


representa por medio de un paralelogramo. Este plano se denota por dos letras correspon­
dientes a vértices opuestos ( plano AB ), o también por una sola letra ( plano P ).
* Las numeraciones que siguen son sucesivas hasta ei capítulo 24.

1.- POSTULADO.- Todo plano contiene al menos tres puntos no colineales.


Esto nos indica que los planos son amplios.
Los puntos que están sobre un mismo plano se llaman Coplanarios.

2.- POSTULADO.- Dos puntos cualesquiera de un plano determinan una recta contenida en ei
plano.

Así, los puntos A y B pertenecen al


plano Q; ellos determinan la recta AB,
contenida en el plano.

(ABcQ

3.- POSTULADO.- El espacio contiene al menos cuatro puntos no coplanarios.


Esto nos indica que el espacio no es llano.

4.- POSTULADO.- ( De separación del espacio).- Los infinitos puntos del espacio que no están
en un plano dado, form andos conjuntos separados por dicho plano, llamados semi-espacios,
tales que :

(a) Todo segmento determinado por dos D


puntos de un mismo semi-espacio, no /
interseca al plano.
( Por ejemplo CD ). //
\

(b) Todo segmento determinado por dos p■


puntos, uno en cada semi-espacio,
interseca al plano.
Por ejemplo AB.

5.- TEOREMA.- Si una recta interseca a un plano que no la contiene, entonces la intersección
es un solo punto.

Hipótesis: r, es una recta que interseca


al plano M, que no la contiene.

* Tesis: La intersección de r y M es un
solo punto O.

Demostración

(1o.) Supongamos que la recta r interseca al plano M en dos puntos O y 0 ’( Hipótesis a u xilia r).

(2o.) Entonces, resultaría que la recta r, determinada por los puntos O y O', del plano M, está
contenida en M ( por el No. 2 postulado ).
660
(3o.) Pero, según la hipótesis, el plano M no contiene a la recta r; luego, nuestra suposición de que
r interseca al plano M en más de un punto es falsa y por lo tanto la recta r interseca al plano
M en el único punto O.

* Nota.- El punto de intersección de una recta y un plano se llama “traza” o “pie”, de la recta
en el plano.

DETERMINACION DEL PLANO

Así como en el caso de la recta, decimos que ciertos conjuntos ( puntos o rectas) determinan
un plano, si éste es el único que los contiene.

6.- POSTULADO DEL PLANO.- Tres puntos cualesquiera son coplanarios y tres puntos
cualesquiera no colineales determinan un plano.

Así, los puntos no colineales A, B y C,


determinan el plano H. ( El único que contie­
A
ne a los tres ).
!
>
t

i ( Cada uno de los puntos A, B y C, pertenece


c
al plano H y podemos e scrib ir:
/
A € H. B e H, C e H ).

7.- TEOREMA.- Una recta y un punto exterior a ella determinan un plano.

Demostrar este teorema es sencillo.


Si E es un punto exterior a la recta AB.
entonces A, B y E no son colineales, y
de acuerdo al postulado anterior No.6,
determinan un único plano Q.

8.- TEOREMA.- Dos rectas secantes determinan un plano.

Sean las rectas secantes OA y OB. Desde


que los puntos A, O y B no son colineales,
determinan el plano M.

9.- TEOREMA.- Dos rectas paralelas


determinan un plano.

Así, las rectas m y r, paralelas


determinan el plano S.

( m 11 r , m c S y rcS).

661
POSICIONES RELATIVAS EN EL ESPACIO

10.- POSICIONES RELATIVAS DE DOS PLANOS.-

Dos planos distintos pueden ser secantes o paralelos.

a) Planos Secantes.- Postulado: Si dos planos diferentes se intersecan, su intersección es una


recta.

Así, los planos distintos P y Q se


cortan en la recta AB.

La intersección de estos planos se­


cantes se llama traza de uno de ellos
sobre el otro.

( P n Q : AB)

b) Planos Paralelos.- Definición: Dos planos son paralelos si no se intersecan

Si los planos R y S son paralelos,


escribim os:

/
R S
//

(Rn S : ó)

11.- POSICIONES RELATIVAS DE UNA RECTA Y UN PLANO.-

Una recta, con relación a un plano, puede estar contenida en dicho plano, (por ejemplo AB).
Puede ser secante al plano, si su intersección con el plano es un solo punto. O, puede ser
paralela al plano, si no se intersecan.

* AB : Contenida en el plano P.
(XI c p )

* r : Secante al plano P, en el punto O.


( r n P : {O})

* m 11 P . La recta y el plano no se intersecan.


( m n P : <(> )

Í6 2
12.- POSICIONES RELATIVAS DE DOS RECTAS.-
Dos rectas diferentes en el espacio, pueden s e r :

a) Secantes, es decir se intersecan en un punto. Estas rectas son coplanarias.

b) Paralelas, es decir no intersecan y están en el mismo plano.

c) Alabeadas o cruzadas; si no se intersecan y no son paralelas, es decir no están en un mismo


plano.

En la siguiente figura, las rectas m, n y r están contenidas en el plano Q y la recta L e s


secante a dicho plano, en el punto O.

Las rectas n y r son secantes y las rectas m


y n son paralelas. Ninguna de las rectas m,
n y r pasa por el punto O. La recta L es
alabeada con cada una de las tres anterio­
res ( se cruza con ellas ).

13.- DEFINICION.- Angulo entre dos rectas alabeadas, es el ángulo determinado por dos rayos
respectivamente paralelos a las rectas dadas y cuyo origen es un punto cualquiera en el
espacio.
♦ X

Así, dadas las rectas alabeadas x e y;


—» M «->
si x » y' 11 y * entonces a° es la
medida del ángulo entre las rectas que
se cruzan x e y.

NOTA.- Si por el punto A trazamos una recta r, paralela a y , entonces el ángulo de cruce
entre x e y es el mismo que forma la recta x con r .

14.- RECTAS PERPENDICULARES.- Son aquellas rectas que cruzándose o cortándose deter­
minan ángulo recto.

14.1. Observación.- En algunos cursos se usa el término ortogonalidad como sinónimo de


perpendicularidad.
Algunos textos de Geometría distinguen lo siguiente:

* Rectas perpendiculares, aquellas que se cortan formando ángulo recto y;


* Rectas ortogonales, aquellas que se cruzan formando ángulo recto.

Nosotros de acuerdo con la interpretación dada en la Universidad Nacional de Ingeniería,


adoptaremos como rectas perpendiculares aquellas que cortándose o cruzándose forman
ángulo recto. Sin embargo, en algunos casos, para mencionar que dos rectas dadas son
663
y

"alabeadas y perpendiculares, sencillamente diremos que ellas son ortogonales , por ser
este último término más cómodo.
r

15.- HAZ DE PLANOS.- Definición: Es el conjunto de todos los pianos que contienen una misma
recta, la cual recibe el nombre de arista del Haz .

La figura muestra tres de los infini­


tos planos que contienen a la recta
L. Desde luego, es demostrable
que por una recta cualquiera del
espacio pasan infinidad de pla­
nos.

( P f l Q D R : L)

16.- TEOREMA.- Si dos planos distintos tienen un punto común, entonces tienen comunes todos
los puntos de una recta, y sóio ellas.

Probemos esto. Supongamos que los pla­


nos P y R tienen en común el punto A. Por
A tracemos dos rectas m y n contenidas en
el plano R.

Si alguna de estas rectas estuviera en P, el


teorema quedaría demostrado.
Si ninguna está en P, tomemos un punto C
sobre n y otro punto D en la recta m, en
distinto semi-espacio que B, respecto del
plano P. Esto es siempre posible, ya que el
punto A divide a cada una de dichas rectas
en dos semi-rectas, situadas en cada semi-
espacio determinado por P.

Entonces, la recta CD cortará al plano P en un punto B, de acuerdo con el postulado 5.


Con seguridad, este punto es distinto del A, por pertenecer al lado opuesto al vértice A del
triángulo ACD.

Por lo tanto, los planos P y R tienen, además del A, otro punto común B. La recta AB estará
situada en P y también en R, según el postulado 2; luego será una recta común a ambos
planos. Y, como por una recta y un punto exterior a ella no pueden pasar dos planos distintos,
los planos P y R tienen comunes todos los puntos de la recta AB y sólo ellos.

664
V

CAPITULO 1 9
v

RECTAS Y PLANOS, PERPENDICULARES


17.- DEFINICION.-
Una recta y un plano son perpendiculares, si se intersecan en un punto, y si toda recta en
el plano, que pase por el punto de intersección, es perpendicular a la recta dada.

En la figura adjunta, la recta (' y el


plano P se intersecan en el punto O.
Tres rectas de dicho plano pasan por
O. Luego, por definición, si t es per­
p e n d ic u la r m, r , x ; entonces la rec­
ta i y el plano P serán perpendicula­
res: f 1 P ó P i 7, siendo el punto
O, pie de la perpendicular en el plano. »

* Como la figura está en perspectiva, no se notan los ángulos rectos que forma 7 con las
rectas del plano.

17.1.-OBSERVACIONES:
* Si una recta interseca a un plano, sin serle perpendicular, se llama Oblicua al plano.
** Si una recta es perpendicular a un plano, será perpendicular a todas las rectas del plano,
aún cuando no pasen por el pie de la perpendicular, ya que se cruza con ellas en ángulo recto.
( Definición dada en el No.14 ).
t n la figura adjunta, por ejemplo, si la
recta f es perpendicular al plano Q en el
punto O; las rectas m, x, r, están conte­
nidas en Q y r no pasa por O, entonces
t será perpendicular a las tres y a todas
las demás rectas del plano, aunque no
pasen por O. Sin embargo, como 7 y r
son alabeadas y perpendiculares, dire­
mos que ellas son ortogonales. Para t y
m o 7 y x , simplemente diremos per­
pendiculares.

665
CONDICIONES PARA QUE UNA RECTA SEA PERPENDICULAR A UN PLANO

18.- TEOREMA.- Para que una recta sea perpendicular a un plano, es necesario y suficiente que
sea perpendicular a dos rectas secantes de dicho plano.
Casos:

a) Si las dos rectas pasan por el pie de la


perpendicular:

* Hipótesis: t 1 m y í 1 r, en el pun­
to O.

m y r , rectas secantes contenidas


en el plano P.

* Tesis : í _L P, en el punto O.

b) Si sólo una de las rectas secantes pasa por el pie de la perpendicular.

* H ipótesis: í l r , e n O ; í ' l x f / ' y x


son ortogonales)

r y x rectas secantes contenidas en el


plano P.

* T e sis: í ± P, en el punto O.

c) Si ninguna de las rectas secantes pasa por el pie de la perpendicular.

* Hipótesis: f, secante al plano Q, en el


punto O.

t í a y f JL b. ( ^ es ortogonal a am­
bas re cta s).

a y b , rectas secantes contenidas en el


plano Q.

* Tesis : 1 ± Q , en el punto O.

DEMOSTRACIONES

(a) Para demostrar que f 1 P, bastará probar que una recta cualquiera ñ , de dteho plano
( distinta de m y r ), es perpendicular a 1, en el punto O, pues con esto cumpliríamos la
definición de perpendicularidad entre recta y plano.

Para ello, trazamos una recta del plano P, que corte a m, ñ y r, en los puntos A, C y B,
respectivamente. R y FT son puntos de la recta tales que OR = OR’.
Trazamos RA, R B .......
666
Luego, como m y F son mediatrices
de! segmento RR’, en los planos RAR’
y RBR\ respectivamente, entonces:
AR = AR’ y BR = BR’, por lo que:
A RAB = A R'AB ( postulado LLL ),
de donde: ^ RAB = ^ R’AB.

y : A RAC = A R’AC ( Postulado


LAL ).

CR = C R \ Esto último indica que el


A RCR’ es isósceles, y como O es
punto medio de RR1, deducimos que
CO 1 RR1,

En consecuencia : 71 ñ
y : 71 P ,

tal como se quería demostrar.

(b)

Entonces:
L 1 x L X x \ por definición de án­
gulo entre rectas alabeadas.
Luego, este teorema se reduce al caso (a),
siendo las rectas r y x’ las que pasan por el
pie de la perpendicular.

(c) Este caso se demuestra trazando por el punto O:


allá y b' 11 b
De donde : L _L á' y L 1 b ’

19.- TEOREMA DE LAS TRES PERPENDICULARES

Si desde el pie de una perpendicular a un plano, se traza otra perpendicular a una recta
cualquiera dada en el plano; entonces, toda recta que pasa por un punto cualquiera de la
primera y el punto de intersección de las dos últimas, es perpendicular a la recta dada en el
plano.

* Hipótesis: Sea 7 1 H, en el punto O ; F es una recta del plano H. Se traza OB i r y se toma


un punto cualquiera P, de la recta L

* Tesis : P B 'l F ( a = 9 0 °)

667
Demostración:
La recta r por hallarse en el plano H, es
perpendicular a f . También, por hipó­
tesis, F es perpendicular a OB; luego,
7será perpendicular al plano que de­
terminan las rectas / y O B .(T e o re ­
ma 18, caso b ). Entonces, r será
perpendicular a la recta PB de éste
plano :
- I
r jl PB a = 90°

EXISTENCIA Y UNICIDAD DE PLANO PERPENDICULAR A UNA RECTA

TEOREMA.- Por un punto dado pasa un plano, y sólo uno, perpendicular a una recta dada.
Este teorema comprende dos casos, según el punto esté en la recta o fuera de la recta dada.

1er. CASO.- Si el punto dado está en la recta dada.

A
* Hipótesis: Sea O un punto de la recta AB.

* Tesis: Por O pasa un plano H, y sólo uno,


perpendicular a AB.

D em ostración:

Sea P un punto fuera de la recta AB,


determinándose el plano ABP.

Sea Q un punto situado fuera del


plano ABP, determinándose el plano
ABQ.

En el plano ABP hay una recta OC


perpendicular a la recta AB en el pun­
to O.

Asimismo, en el plano ABQ hay una


recta OD perpendicular a la recta AB
en el punto O.

Luego, las rectas OC y OD, determi­


nan el plano H.
i—>
Entonces: Plano H 1 AB en el punto
O. ( Por el teorema 18 ).
Y H es único, porque si existiera otro
plano Q, perpendicular a AB en
0 ,e n to n c e s las in te rs e c c io n e s
i—>
y OE de los planos H y Q con un
i—>
plano cualquiera R que pase por A B ,
serían dos rectas en el plano R,
perpendiculares a AB en un mismo
punto O. Como sabemos, esto último
es imposible.

2do. CASO.- Si el punto dado está fuera de la recta dada.

* Hipótesis: Sea O un punto situado


fuera de la recta AB.

Tesis: Por O pasa un plano H, y sólo


uno, perpendicular a AB.

D em ostración:

La recta AB y el punto O, determinan un plano


R, en el cual existe una recta OC, perpendi-
cular a AB que pasa por O.

Sea F un punto situado fuera del plano R. La


recta AB y el punto F determinan un plano P,
en el que hay una recta CD, perpendicular a
AB, en C.
Luego, las rectas CO y CD determinan un
plano H. Y como: CO l AB, CD 1 AB, en-
tonces, el plano H es perpendicular a AB,
pasando por el punto O, ya que se cumple la
condición de perpendicular entre recta y pla­
no, dada en el No. 18.

Además, H es el único, porque si suponemos


que existe un segundo plano T perpendicular
a AB, que pasa por O, las intersecciones
OC 1 OE de H y T, respectivamente, con
un plano cualquiera R que pase por AB,
serían dos rectas en el plano R, perpendicu­
lares a A B , desde un mismo punto exterior O;
lo cual, como sabemos, es imposible.

669
2 1 COROLARIO.- Todas las perpendiculares a una recta, por un punto dado, están contenidas
en un plano perpendicular a la recta, trazado en dicho punto.

* El p jnto O se sitúa en la recta r; m, n,


x sor algunas de la infinidad de rectas
perpendiculares a r, que pasan por O.
Todas ellas están contenidas en el pla­
no P, perpendicular a r .

* El punto O es exterior a la recta s. Por


O pasa un plano R perpendicular a s.
q, 7, t son algunas de la infinidad de
^ y
perpendiculares a s y que pasan por O;
todas contenidas en el plano R.

PLANO MEDIATRIZ.- Se llama plano mediatriz de un segmento, al plano perpendicular a


dicho segmento, trazado en su punto medio ( Llamado también plano m ediador).

22 . - TEOREMA.- Todo punto situado sobre el plano mediatriz de un segmento, equidista de los
extremos de dicho segmento.

En la figura adjunta, R es el plano


mediatriz del segmento AB. B
Si P es un punto cualquiera de dicho
plano y M punto medio de A B , enton­
ces PM 1 AB y el triángulo APB es
isósceles PA = PB.

EXISTENCIA Y UNICIDAD DE RECTA PERPENDICULAR A UN PLANO

23.- TEOREMA.- Por un punto dado, pasa una recta, y sólo una, perpendicular a un plano dado.
Este teorema comprende dos casos, según el punto esté en el plano o fuera del plano dado.
*

1er. Caso.-Si el punto dado está en el plano dado.

X - Hipótesis: Sea O un punto en el


plano R.

- Tesis: Por O, pasa una recta y


sólo una, perpendicular al plano
R.

fí70
Demostración:

Sea m , una recta en el plano R, que pasa


por O.

Sea S el plano perpendicular a m e n O.


( No.20, le r.C a s o ).

Los planos R y S se intersecan en una


recta AB.

Sea 9, una recta en el plano S, perpendi-


cular a AB, en el punto O ( * ).

- Entonces, ya que el plano S _L m, y 1 está contenida en S ílm

- Luego:
ílm implica que f .L R , por el
-

y por ( * ): t1± AB No. 18, caso ( a ).

- Además, f es única, porque de existir otra recta t ,


diferente de perpendicular al plano R en O,
entonces las dos rectas t y /' determinarían un
plano T, secante con R en una recta EF. Y tendría­
mos así, en el plano T, dos perpendiculares f y í'
O
a EF, en el mismo punto O, lo cual es imposible.

2do.Caso.- Si el punto dado está fuera del plano dado

Hipótesis: Sea P, un punto situado


fuera del plano R.

Tesis: Por P pasa una recta x, y


sólo una, perpendicular al plano
R.

( La demostración se deja como


ejercicio al le c to r).

TEOREMA.- Si dos rectas paralelas son secantes a un mismo plano y una de ellas es
perpendicular al plano dado, entonces
la otra recta es también perpendicular
al plano.
m
D em ostración:

Sean ^ y í 2 dos re c ta s p a ra le ­ n
las,secantes al plano P. La recta ¿1 es
perpendicular a P en el punto O.
671
Luego, si m y ñ son dos rectas de dicho plano que pasan por O, s e rá : ^ 1 m y í 1 1 ñ.

Entonces, como ?2 *1 el ángulo de cruce de ?2 con m y ñ es el mismo entre ~


pA y dichas
rectas ( No. 14 ).
Por lo tanto: ^2 ± m y 12 ± ñ, de donde 12 p - ( No.18, caso C ).

PERPENDICULAR Y OBLICUAS A UN PLANO

25.- TEOREMA.- Si desde un punto exterior a un plano se trazan a éste el segmento perpendi­
cular y varios segmentos oblicuos:

1) El perpendicular es menor que cualquier oblicuo.

2) Los segmentos oblicuos cuyos pies equidistan del pie del perpendicular, son congruentes.
0

3) De dos segmentos oblicuos, cuyos pies no equidistan del pie del perpendicular, es mayor
aquel cuyo pie dista más.

Graficamos cada caso: / P

1- - Hipótesis: P es un punto exterior al /


/ / .

plano M; PA perpendicular al plano M / g"


en A ; PB una oblicua a M, en B. /* \
_

- Tesis : PA < PB
_ ¿mA
D em ostración:

ComoPA 1 M , por hipótesis, entonces al trazar BA : BA 1 PA. Luego, en el triángulo PAB,


PB es hipotenusa y PA cateto : PA < PB.

2-
- Hipótesis: P es un punto exterior al plano
H; PA 1 H en A, PB y PC, segmentos
oblicuos al plano H en B y C, respectiva­
mente, tal que AB = AC.

-T e s is : PB = PC

D em ostración:

Siendo PA 1 H , por hipótesis, entonces PA 1 BA y AC. Luego, los triángulos rectángu


los PAB y PAC son congruentes por tener en común el cateto PA y de la hipótesis :
ÁB s AC.
Entonces : PB = PC.
672
3-

Hipótesis: P es un punto exterior al plano


Q. PA 1 Q en A ; PE y PF oblicuas a
Q t tal que AE > AF.

- Tesis : PE > PF

La Demostración se deja como ejercicio al lector. Para ello tome un punto B en A E , tal que
AB = AF y únalo con P.

26.- DISTANCIA DE UN PUNTO A UN PLANO.- Se llama distancia de un punto a un plano, a la


longitud del segmento perpendicular trazado desde el punto al plano.

Así, en la figura adjunta, si PO es per­


pendicular al plano R, entonces la lon­
gitud del segmento PO es la distancia
del punto Pt al plano R.

673
CAPITULO 2 O

RECTAS Y PLANOS PARALELOS


27.- DEFINICION.- Dos planos son paralelos, si no se intersecan.
Análogamente, una recta y un plano son paralelos si no se intersecan.

LZl ¿~U P//Q r//s

28.- PARALELISMO ENTRE RECTAS EN EL ESPACIO.- En Geometría del Espacio, para


afirmar que dos rectas son paralelas no es suficiente con probar que las rectas no se cortan,
pues cabe la posibilidad de que las rectas sean alabeadas. Entonces, se debe dem ostrar
que: ( 1o ) las rectas son coplanarias y ( 2o ) las rectas no se intersecan.

29.- TEOREMA.- En el espacio, por un punto exterior a una recta, sólo puede trazarse una
paralela a la recta dada.

Dem ostración.-Sea ría recta dada y


P un punto exterior a ella.
Luego, toda recta paralela a r , traza­
da por el punto dado P, tendrá que
estar en el plano Q determinado por r
y P ; según e lBV Postulado de Eucli-
des", en este plano hay una sola para­
lela a r, por el punto P.
675
30.- TEOREMA.- Dos rectas perpendiculares a un mismo plano, son paralelas entre sí.

D em ostración:

Sean m y ñ , dos rectas perpendiculares al


plano R, en los puntos O y P. Sea m’ una recta
paralela a m, trazada por el punto P. Como
sabemos, de acuerdo con el N°29, m' es
única y por el N°24, m* es perpendicular a R,
en el punto P. Además, según el N°23
1er.caso, por el punto P del plano R, sólo se
puede trazar una perpendicular a dicho pla­
no. Y como ,ñ 1 R por hipótesis; entonces
nV y ñ son una misma recta. Con lo que
queda probado que ñ 11 m .

31.- TEOREMA.- Todo plano que corta a una de dos rectas paralelas, también corta a la otra.

D em ostración:

Sean m y r dos rectas paralelas. Sea Sun


plano que corta a m en el punto A. Probe­
mos que S cortará también a r .
En efecto, el plano S y el que determinan las
paralelas tienen un punto en común A.
Luego tendrán en común los puntos de una
recta x. Además, en el plano de las parale­
las, como x corta a m , también cortará a r
en un punto B. El punto B es común a la
recta r y al plano S, con lo que queda
demostrado el teorema.

32.- TEOREMA.- En el espacio, dos rectas paralelas a una tercera, son paralelas entre sí.

H ipótesis: Sean m, n y r,
tres rectas, tales que:

m || ñ y 7 n

- Tesis: m

676
Demostración;

Sea O un punto de la recta n y tracemos el plano Pf perpendicular a ñ por dicho punto


Luego:

m ñ y como ñ i P , entonces m 1 P
( Por el N° 24 )
i ñ y de ñ 1 P, entonces 7 i P

y , de acuerdo con el N°30, por ser m y f dos rectas perpendiculares a un mismo plano
P, se concluye que m 7.

33.- PARALELISMO ENTRE RECTA Y PLANO.- Según demostramos en el siguiente teorema:


“para que una recta no contenida en un plano dado, sea paralela a dicho plano, es necesario
y suficiente que sea paralela a una recta contenida en el plano".

34.- TEOREMA.- Toda recta no contenida en un plano y paralela a una recta de este plano, es
paralela al plano.

- H ipótesis: Sea r una recta no conte­


nida en el plano P y paralela a la recta
x, contenida en dicho plano.

- Tesis: r || P. O

D em ostración:

Supongamos que la recta r no es paralela al plano P, es d e c ir, que lo interseca en un


punto O.
Las rectas paralelas r y x , determinan un plano S secante a P, según la recta x. Luego,
como hemos supuesto que F interseca a P en O; este punto pertenecerá a los planos S y P;
por lo tanto el punto O será un punto de la recta x, intersección de dichos planos. Así, se
llegaría a que las rectas r y x tienen en común el punto O, contradiciendo la hipótesis de
que r 11 x. Entonces, la suposición de que r interseca a P, es falsa , y en consecuencia:
P.

35.- TEOREMA.- Si una recta y un plano son paralelos, ella es paralela a la intersección de este
plano con cualquier otro que contenga a la recta y corte al plano dado.

Hipótesis: Sea r una recta paralela al


plano P y Q un plano cualquiera que
contiene a r y se interseca con P
según la recta m.

- Tesis: r m.
677
D em ostración:

Como la recta r es paralela al plano P, entonces r no cortará a m. Siendo además r y m


coplanarias, entonces r m.
*%*

36.- TEOREMA.- Si dos rectas se cruzan, por una de ellas pasa un plano, y sólo uno, paralelo a
la otra.

- H ipótesis: Sean m y r dos rectas


alabeadas.
( Por comodidad grafiquemos r con­
tenida en plano P y m secante a dicho
plano en el punto O ).

Tesis: Por m pasa un plano paralelo


a r , y sólo uno.

D em ostración:

• a

En efecto. Sea r' paralela a r , trazada por el


punto O. Luego, las rectas m y r\ determinan
un plano Q, el cual con seguridad no contiene
a r . Por lo tanto : Q r , ya que r

* Y este plano Q es único, porque de


existir otro plano Q' paralelo a r , pa­
sando por m , la intersección de este
otro plano, con P, sería una recta r"
que contiene al punto O. Así, en el
plano P, por el punto O se tendrían
dos paralelas a la recta r : r' y r". Lo
cual contradice al “ V Postulado de
Euclides”

37.- TEOREMA.- Toda recta paralela a dos planos secantes , es también paralela a la
intersección de dichos planos.
( La demostración se deja como ejercicio al lector ).

38.- PARALELISMO ENTRE PLANOS.- Dos planos son paralelos cuando no tienen punto
común.

39.- TEOREMA.- Dos planos perpendiculares a una misma recta, son paralelos entre si.
D em ostración:
Sean P y Q, dos planos perpendicu­
/
lares a la recta ( en los puntos A y B,
respectivamente. Si P y Q tuvieran un ¿A
punto común R, este punto sería exte­
rior a í y por él pasarían dos planos
perpendiculares a contradiciendo
el teorema 20, 2do.caso. B
/ q 'S
Por lo tanto P y Q no pueden tener
algún punto común; en consecuencia
son paralelos.

40.- TEOREMA.- Si un plano contiene a dos rectas secantes, paralelas a un segundo plano,
entonces ambos planos son paralelos
entre sí.

H ipótesis: á y b son dos rectas se­ a-


cantes contenidas en el plano P, tales
que á Q y b Q.

Tesis: P Q

D em ostracion:

Supongamos que los planos P y Q no son paralelos; tendrían entonces en común una recta
r ( Postulado 10.a ). Luego esta recta interceptaría, con seguridad , a alguna de las rectas
á ó b (o, a ambas). Si asumimos, porejemplo, que r intercepta a á en un punto C, entonces
este punto por pertenecer a r pertenecerá al plano Q. Lo que indicaría que la recta á y el
plano Q tienen en común el punto C, contradiciendo la hipótesis de que á y Q son paralelos.
i* Por lo tanto, los planos P y Q no se intersecan y en consecuencia son paralelos.

41.- TEOREMA.- Toda recta secante a uno de dos planos paralelos, corla también al otro.

H ipótesis: P y Q son dos planos


paralelos; t , una recta que corta a P
en el punto A. T \ , ,
Tesis: i corta a Q. w
D em ostración:
/ ^

Sea V la recta paralela a trazada por un punto C del plano Q. Luego, por el teorema 31,
como el plano Q corta a 7 \ cortará también a su paralela 1. Así, la recta i corta a los dos
planos P y Q.
679
42.- TEOREMA.- Todo plano que corta a uno de dos planos paralelos, corta también al otro

H ipótesis: P y Q son dos planos


paralelos; R es un plano secante a P.

- Tesis: R corta a Q.

D em ostración: Sea r, una recta del plano R, que corta a P en el punto A. Luego, por el
teorema anterior esta recta corta también al plano Q, en un punto tal como B; entonces, los
planos R y Q tienen en común el punto B y tendrán comunes todos los puntos de una recta.

43.- TEOREM A.-Toda recta perpendicular a uno de los planos paralelos, es también perpendi­
cular al otro.
* ( Demuestre el lector este teorema ).

44.- TEOREMA.- Portodo punto exteriora


un plano dado, pasa un plano y sólo
uno, paralelo al primero.

- H ipótesis: Sea P plano dado y A, un


punto exterior a dicho plano.

- Tesis: Por A pasa un plano Q, y sólo


uno, paralelo a P.

D em ostración: Por A , trazamos dos rectas paralelas al plano P. ( paralelas a dos rectas m’
y n’, de P ). El plano Q, determinado por estas rectas, es paralelo a P, ( Por teorema 40 ).
Y este plano es único, porque de existir otro plano que pase por A, cortará a Q y también
a P, según el teorema 42.

45.1 COROLARIO.- Todas las rectas paralelas a un plano dado, trazadas por un punto exterior,
están contenidas en el plano paralelo al primero y que pasa por dicho punto.

* La figura muestra algunas de la infini­


dad de rectas paralelas al plano P, que
pasan por el punto exterior A. Todas e-
llas están contenidas en el plano Q,
paralelo a P.

RRÍ)
V

45.2 COROLARIO.- Por dos rectas alabeadas dadas, pasan dos planos, y sólo dos, paralelos
entre sí, y que contienen cada uno, una de dichas rectas.

La figura ( a ) muestra dos rectas alabeadas m y r , estando r contenida en el plano P y


m secante a P.
La figura ( b ) indica los planos M y R, únicos, paralelos entre sí, que contienen a m y r ,
respectivamente.

46.- TEOREMA.- Dos planos paralelos a un tercero, son paralelos entre sí.

D em ostración: Veamos. Si los dos planos paralelos al tercero, no fueran paralelos entre sí,
entonces tendrían en común un punto, por el cual pasarían dos planos paralelos al tercero,
contradiciendo el teorema 44. Por lo tanto, los dos planos son paralelos entre sí.

47.- TEOREMA.- Si dos planos paralelos se interceptan por un tercer plano, las rectas de
intersección son paralelas entre si.

D em ostración:

P y Q son dos planos paralelos


cortados por el plano R, según las
rectas m y n.
Como m y ñ están en un mismo plano
R y no se cortan, por situarse en
planos paralelos, entonces: m ñ.

48.- TEOREMA.- Los segmentos de rectas paralelas, comprendidos entre planos paralelos, son
congruentes.
H ipótesis: m y n son rectas paralelas
interceptadas por los planos paralelos P
y Q, según AC y BD.

Tesis: AC = BD

D em ostración: Las rectas m y n, deter-


minan un plano en el que AB | f C D . El
cuadrilátero ABDC tiene sus lados o-
\ puestos paralelos; entonces es un para-
lelogramo. De donde: AC = BD.

681
49.- TEOREMA DE THALES EN EL ESPACIO.- Tres o más planos paralelos determinan sobre
dos rectas secantes cualesquiera, segmentos proporcionales.

Hipótesis: Sean m y F , dos rectas


que interceptan a los planos paralelos
P, Q y R en los puntos A, B, C, D, E
y F, respectivamente.
AB DE
TeS¡S: B C = ÉF

Demostración: Sea m' la recta que


pasa por el punto A y es paralela a m ,
interceptando a los planos Q y R en
los puntos E’ y F . Luego, m' y F
determinan un plano secante a P, Q y
R; de tal manera que BE' y CF1 son
paralelos, en el que usamos el teore-
AB AE'
ma de Thales: -

Pero, por el teorema anterior: AE’ = DE y E’F = EF, por lo tanto:


AB DE
BC “ EF

50.- TEOREMA.- Dos ángulos no coplanarios que tienen sus lados respectivamente paralelos,
son congruentes o suplementarios, y los planos que los contienen son paralelos.
Son congruentes, si tienen sus lados dirigidos en el mismo sentido o en sentido contrario,
y suplementarios si sólo un par de éstos se dirigen en el mismo sentido.

Veamos el gráfico de cada caso:

1 °)

HIPOTESIS: Sean los ángulos AOB y


CMD con ÓA MC y OB MD, en
igual sentido.

* TESIS : AOB s CMD

2°)

* HIPOTESIS : OÁ NE y OB NF
con sentido opuesto.

* TESIS : AOB = ENF

682
* HIPO TESIS: OA DE ( igual sentido)
y OB NF ( sentido opuesto ).

* T E S IS : AOB + ENF = 180

Demostración del ter.caso:

Sea E un punto de OA y K un punto


de MC, tales que OE a MK. Tam­
bién, sea F un punto de OB y T un
punto de M D , de modo que OF a M T .

Tracemos los segmentos:


OM, EK, FT, EF y TK
Como OE II MK y OE a MK ( por
hipótesis y primer paso ), entonces
el cuadrilátero OEKM es un
paralelogramo.*. OM y EK son con­
gruentes y paralelos ( a ).

También OF 11 MT y OF a MT ( por hipótesis y primer paso ), entonces el cuadrilátero


ÜFTM es un paralelogramo OM y FT son congruentes y paralelos ( b ).

Luego, de ( a ) y ( b ): EK 11 FT y EK a FT .*. el cuadrilátero EKTF es un paralelogramo;

de donde EF a TK.

Finalmente : A EOF a A KMT ( postulado LLL ).

Entonces : AÓB a CMD

Nota.- Demuestre el lector los otros dos casos.

683
V

P R O B LE M A S R ESU ELTO S
( Capítulos 1 8 ,1 9 y 20 )

1) Se tiene una circunferencia de centro 0 y diámetro 12 cm. Por 0, pasa una recta L
perpendicular al plano de la circunferencia. F, es un punto de L, tal que OF = 8 cm. Hallar la
distancia de F a cualquier recta tangente a la circunferencia.

A) 10 cm B) 20 cm C) 13 cm
D) 15 cm E) 16 cm

Solución
12cm _
radio = — r— = 6cm

Sea m, una recta tan­


gente a la circunferen­
cia, en el punto T.

OT JLm

Según el Teorema de las tres perpendiculares, será F T lm Luego, FT es la dis-


tanciade F a m . En A FOT:

FT = 6 + 8 ->

Rpta: (A)

2) En una circunferencia de centro 0, se inscribe un triángulo ABC, recto en B. Se eleva bf


perpendicular al plano ABC, de modo que BF = AC.
Si AB = 6 y BC = 8, hallar OF

A) 5 B) 5 ->¡2 C) 5 V3

D) 10 V2 E) 5 V5

Solución
* X=?

AC = 62 + 82

6S5
OB = OA = OC = radio = 5 F

FB _L plano ABC
x
FB 1 BO

En a OBF, Teorema de Pitágoras:

X2 = BF2 + O B 2 X2 = 102 + 52 = 125 C

Rpta: (3)
3) Indicar verdadero (V) o falso (F):

* Tres puntos determinan siempre un plano.


* Dos rectas determinan siempre un plano.
* Si una recta es paralela a un plano, será paralela a todas las rectas contenidas en dicho
plano.
* Si una recta es perpendicular aun plano, será perpendicular a todas las rectas contenidas
en dicho plano.

A) VVVV B) VFVF C )FV FV


D)FFFF E )FFFV

Solución

* (F) Tres puntos NO COLINEALES determinan un plano.


* (F) Dos rectas secantes o dos rectas paralelas, determinan un plano.
* (F) Si una recta es paralela a un plano, será paralela a infinitas rectas paralelas de dicho
plano, pero no a todas.
* (V) Teoría.

Rpta: (E)

4) En un cubo, cuyas aristas tienen longitud "a” cada una, hallar la distancia de un vértice al
centro de una cara opuesta.

aV3

686
Solución D
Sea O, centro de la cara AFEH y B
un vértice de la cara opuesta
BCDG. Hallemos BO = X.

Como BA 1 AF y BA 1 AH
BA 1 plano AFEH

Luego: BA _L AO

AE 42
AO = OE =

En A OAB: X2 = AB2 + AO

'a V T
—>
v /

Rpta: (C)

ABCD, es un cuadrado de lado “a”. Por B, se eleva BE perpendicular al plano ABCD, tal
que BE = a. Si “O” es centro del cuadrado y “H" punto medio de C D , hallar el área de la región
triangular EOH.

V2 V2
A) B) C)
8 8
5a2
D) 8 E)
~ ~ 8 ~ ~

Solución

Al prolongar HO hasta P

H P X AB y AP = PB = —
y 2

Teorema de las tres perpendiculares

EB 1 plano ABCD
y EP X PH
BP 1 PH

EP es altura del A EOH. Luego:


687
/ \
a
S e o h = \ (OH)(EP) = 1 (EP) (1)
12,

/
a
En el A EBP: EP2 = E B 2 +B P 2 -> EP2 = a 2 +
<2 j

EP = - V 5 (2 )
2

/ \
a
(2), en (1): SE0H - —
2 ) \

Rpta: (C)

6) ABC, es un triángulo equilátero de lado “L” por B, se eleva BR perpendicular al plano


ABC,demodoque: BR = L/2. Setrazan luego R D y R C . Hallarel áreade la región triangular
ARC.
L‘
A) L2 B) 2 L2 C)

D) E)

Solución
SARC = 7
Con el teorema de las tres perpendiculares

RB 1 plano ABC
y RH X AC
BH X AC

Luego: . SAñc = ~ ( AC ) ( RH )

ARC = l ( L ) ( RH ) (1 )

En A AHB (notable): BH = -^-V3

En A RBH, teorema de Pitágoras: RH2 = RB2 + BH

688
Rpta: (E)

El sólido ABCD de la figura, se llama tetraedro. Las D


regiones triangulares ABC, ABD, ACD y BCD se
llaman caras; los puntos A, B, C y D, son los vértices
y las aristas: AB, BC, CD, AD, A C y B D . Se llama
altura del tetraedro, a la distancia de un vértice a la
cara opuesta. (Por ejemplo DH ) \ ■

Si las caras del tetraedro son triángulos equiláteros,


el sólido es regular.
En un tetraedro regular, cuyas aristas tienen longitud “L", hallar la longitud de la altura.

Solución

Como el tetraedro es regular: AB = BC = AC = AD = CD = BD = L.

Es fácil demostrar que H es el centro del A ABC.


- El efecto: A AHD b A CHD = a BHD. Por tener igual hipotenosa ( L ) y el cateto
común DH.

HA = HB = HC , ( H es circuncentro del A ABC )

L
AH = D

En A AHD: DH2 = AD2 - AH2

DH2 =L2 - DH = - V 6
IV 3 3

Rpta: (D)

689
8) Hallar el máximo número de planos que determinan “n” puntos del espacio.

n (n -1) n (n -fl) n
A) - B) } C) -
; 2 2 ' 3
d n (n -1 )(n -2 ) n (n -1 )(n -2 )

Solución
* El máximo número de planos que determinan “n" puntos del espacio, ocurrirá cuando no
existan tres en línea recta ni cuatro coplanares:

Cn = n! = n (n -1 )(n -2 )
3 3! (n - 3)! 6

ya que cada tres puntos no colineales se determina un plano.

Rpta: (D)

9) Hallar el máximo número de planos que determinan “n” rectas del espacio.

n (n -t) n(n + 1)
A) n B) —^ C )-^ ~ D)2n E) n2

Solución
* El número máximo de planos que determinan “n” rectas del espacio es:

Cn = n! = n (n - 1)
2 2! (n - 2)! 2

y esto ocurrirá cuando no hayan tres rectas coplanares, ya que cada dos rectas, secantes
o paralelas, se determina un plano

Rpta: (B)

10) Hallar el máximo número de planos que determinan 10 rectas y 12 puntos del espacio.

A) 265 B) 385 C) 120 D) 340 E) 165

Solución
* Las 10 rectas determinan, como máximo:

10 10 ! 10 !
Cl° = — ---- :— — = ----- — = 45 planos
2 2 ! (10 - 2 )! 2! 8!

' Los 12 puntos, como máximo, determinan:

12 12! 12!
Cl2 = — ---- ^— — = ------^ = 220 planos
3 3! ( 1 2 - 2 ) ! 3! 9!

690
* Como cada recta determina con un punto exterior a ella, un plano; el máximo número de
planos que determinan las 10 rectas con los 12 puntos, es: 10 .12 = 120
Sumando los resultados parciales: 45 + 220 + 120 = 385 planos

Rpta: (B)

11) La figura muestra un cubo. M, N, R y Q, son puntos medios de las aristas AB,,AF,EF y DE ,
respectivamente. Hallar la medida del ángulo de cruce entre las rectas MN y RQ.

A) 45° B) 30°

C) 90° D) 75°

E) 60°

Solución

Como B F / / M N y D F / / R Q BFD dá el ángulo de cruce entre MN y RQ

( BFD , mide a )

BD = BF = FD, por ser diagonales de


los cuadrados y congruentes.
A BFD, equilátero

Rpta: (E)

12) En un tetraedro ABCD (no necesariamente regular), B y Q son Baricentros de las caras
ABCyACD, respectivamente. Demostrar que BQ y DB son secantes, en un punto que
llamaremos “R" y que se cumple:

Solución

Como las prolongaciones de BG y BQ concurren en el

punto medio de AB ( M ), entonces B Q y D G estarán


cotenidas en el plano MBD y por lo tanto serán secantes.
Ahora, en el A MGD, con el Teorema de Menelao
para la transversal BQ

MQ DR GB
= 1 (a)
Q D ' R G ' BM

MQ _ 1 GB 2
Pero, por propiedad del Baricentro:
QD ” 2 BM 3

691
c n 1 DR 2 .
En ( a ) : —. ----- . — = 1 —»
2 RG 3

En forma análoga se demuestra que

13) Sea “R” un punto exterior al plano que contiene a un rectángulo ABCD.

Probar que se cumple: ra2 + RC2 = RB2 + RD2 R

Solución
/ / ü ',\ \
Llamemos M, al punto de intersec­ _ _

ción de las diagonales del rectán­


gulo. Se traza RM

Por el teorema de la mediana:


BD
A BRD -> RB2 + RD2 = 2RM2 + (a )

AC
A ARC -> RA2 + RC2 = 2RM2 + (P)

- Pero, AC = BD. Por lo tanto ( a ) = ( P ). Entonces: RA2 + RC2 = RB2 + RD2 I.q.q.d

14) ABC, es un triángulo equilátero de lado 6 cm., contenido en un plano P. Se elevan:


BQ 1 P y CR _L P . de modo que B Q = 6 cm y CR = 3 cm . Hallar el área de la región
triangular AQR.

A) 9 V 6 cm 2 B) 9 cm2 C) 12 cm: D) 9V 2 cm2 E) 18>/6 cm2

Solución
SAQR = *>
1
Se tiene: SAQR = - ( A Q ) ( R T ) (1 )

En A ABQ: AB = BQ = 6

AQ = 6V2 (2 )

Se traza RF _L QB

En A ACR: AR2 = 32 + 62 AR=3rV5] AR = RQ

En A RFQ: r q 2 = 32 + 62 RQ = 3*^5 J A ARQ, Isósceles

692
Luego: AT = TQ =

y en A ATR: RT2 + AT2 = AR2 -> RT2 + (3^ 2 f = (3 V 5 )2

RT = 3 t¡3

- Sustituyendo ( 2 ) y ( 3 ), en (1 ): SAQR = (6 V 2 )(3 -/3 )

Rpta: (A)

BAC, es un triángulo recto en A. AB = 6 y AC = 8. Por su Incentro I, se eleva IH perpen


dicular al plano ABC, siendo IH = 3. Hallar HC.

A) 8 B) 10 C)11 D) 112 E )7

Solución

HC = x = ?

- Sean F y T, puntos de tangencia


de la circunferencia inscrita en el
A ABC y los lados BC, AC.

.-. IT = IF = r
(inradio)

Por el teorema del Pitágoras en A ABC : BC = 10

Con el teorema de Poncelet: BC + 2r = AB + AC

10 + 2r =6 + 8 r= 2

Luego: AT = r = 2 y TC = 8 - 2 = 6 = CF

Por el teorema de las tres perpendiculares: H F 1 BC . Por ser

Hl ± plano ABC y IF 1 BC •

En AHFC: x2 =HF2 +CF2 x2 = H F 2 + 6 2 ...................... (1)

EnAHIF: HF2 = 3 2 + r2 -+ HF2 = 32 + a : (2 )

De ( 2 ), en ( 1 ): x2 = 62 + 32 + 22

Rpta: ( E )

693
4-¥
16) En la figura, ST es intersección de los planos H y R

AC n H: { E } ; B g R
ABnH:{Q}

AC//R

G € H y G es Baricentro del A ABC.

c. AE 5 . n AQ
Si — = — ; hallar: —
EC 3 QB

A) 8 B) 9 C) 7 D) 12 E) 10

Solución

AQ * AE 5 5n 3n
= •?
QB EC 3

Sean: A£ = 5 n y EC = 3 n
Mn E
* Con las prolongaciones hechas:
FB//ÁC
BG = 2 GM (propiedad del
Baricentro)
AM = MC AM = 4 n y ME = n
T
FB
A FGB ~ A EGM -> FB = —
n 2r 2

AQ AE AQ 4n + n
AEQA - A FQB
QB “ FB QB n
2
Rpta: (E)

17) Un paralelepípedo rectangular recto, es el sólido formado por 6 regiones rectangulares


llamadas caras. Dos caras opuestas cualesquiera son congruentes entre sí y ubicadas en
planos paralelos. Llamemos a, b y c, las longitudes de tres aristas concurrentes en un vértice.
Si d, es longitud de una diagonal del paralelepípedo, demostrar que:

d2 = a2 + b2 + c2

Solución

- La figura adjunta muestra un paralelepípedo rectangular recto (semejante a una caja


rectangular).
- A, B, C, . . . son los vértices.

AB , BC , CH,... son las aristas

6 9 4
CF , HG , BE , AD , son diagonales del sólido (unen dos vértices de caras diferentes).

Sean: AF = a, EF = b y GF = c. Además: CF = d.

En el A CHF, CH 1 HF por ser CH 1 plano AHEF:

d2 = HF2 + C H 2 .........(1) <- (Teor. Pitágoras)

Siendo: CH = c y en el A HAF: HF2 = a2 + b2

Al sustituir en (1 ):

18) En un cubo, la longitud de una diagonal en términos de la longitud “a” de una arista es:

A) 2a B) aV2 C)aV6 D)aV3 E) 3a

Solución
Por el problema anterior:

d2 = a2 + a2 + a2 para el cubo
d2 = 3a2

(Fórmula)

19) Dos aristas opuestas de un tetraedro, son perpendiculares entre sí. Demostrar que por una
de ellas es posible pasar un plano perpendicular a la otra.

Solución
Sesa ABCD el tetraedro, donde las aristas opuestas AC y DB son perpendiculares
entre sí, según dato. D

Tracemos BR 1 AC

Entonces: plano BRD 1 AC ,

ya que: AC 1 BD por hipótesis


AC _L BR por construcción

20) Para el tetraedro del problema anterior, demostrar que:

Solución

Del gráfico anterior: DR 1 AC

A ARD -> AD2 = AR2 + RD2

A BRC -> BC2 = BR2 + RC2

« Q S
Sumando miembro a miembro:

AD2 + BC2 = AR2 + BR2 + RD2 + RC2

AD2 + BC2 = AB + CD
9 /—
21) En el gráfico mostrado ABCD es un cuadrado de centro “O” y lado — 2 m , QN = QM,

QF es perpendicular al plano que contiene a ABCD y “F” se encuentra en dicho plano.


Calcular PF, s i : AQ = 8m , QC = 10m y m / O P H = 30°.

A )2/3 m

B)/3 m

C) 4 6m

2 /5 m
D)

• E)2/2 m

Solución

- T. Proyección de la mediana ; A AQC :

Q 102 - 82 = 2 ( 9 ) ( O H )
OH = 2

- A O P H ( 3 0 ° , 60o ):
OP = 4

- A FHO ( 45°, 45°):


FO = 2 4 2

- APFO : x = 2 /2

Rpta: ( E ).

22) Dadas tres rectas alabeadas, x, y, ¿.contenidas en planos paralelos, ¿Cuántas rectas son
posibles de trazar, de modo que intercepten a las anteriores?

A) 1 B) 2 C) 3 D) Infinidad E) Ninguna

Solución

Consideremos la fig. 1, que cumple con la condición de que x, y, z se ubiquen en planos


paralelos. ( P, Q, R )
696
Para dar solución al problema, tomemos un punto cualquiera A, sobre x.

Luego, el plano H, determinado por A y la recta y , intercepta a 2 en un punto M. Es claro


que H intercepta a z, porque la intersección de H y R es una recta y ’ paralela a y ( según
*

teorema 35 ) y de hecho y ' interceptará a z.


Así, la recta AM soluciona el problema, puesto que también intercepta a y, en el punto N.
Como el punto A es arbitrario, existirán infinidad de rectas que cumplen la condición; basta
variar dicho punto sobre x .

ABC, es un triángulo equilátero de lado 4 cm. Por A, se traza AE perpendicular al plano ABC,

tal que AE = V^3cm . Si R es punto medio de BC , hallar la medida del ángulo con que
se cruzan los segmentos AB y ER .

A) 37° B) 45° C) 60° D) 53° E) 30°

Solución

- Se traza por R, paralela a AB , cortando AC en N, x dá la medida del ángulo pedido.

Sea AQ J_ RN

Según el Teorema de las tres perpendiculares: EQ ± QR

6 9 7
A RNC, equilátero: RN = 2

A AQN, notable ( 30°, 6 0 °):

AN
NQ =

^ 2
En A EAQ, Teorema Pitágoras: EQ2 = (VT3^ + (-^3)

EQ 4
Luego: — - = — En A EQR:
QR 3

Rpta: (D)
24) A A \ B B \ C C y DD' son aristas laterales de un cubo. (ABCD es una cara). Hallar la distancia
del punto medio M de B'C’ a la diagonal A'C, si AB = a.

a
A )f/2 B) a
C> 2

D)fV2 E) N. A.

Solución

MH 1 A'C

Incógnita: MH.
M 2_C

En A A'B‘M: ( A’M f = a2 +
V2;

A'M = - V 5
2

Como A MC'C s A A'B'M : MC = A'M

A A'MC Isósceles. Luego:

A'C aV3
A'H = HC =

698
Finalmente, en A A' HM: MH2 = A'M 2 - A ‘H2

MH = -|V 2

Rpta: (A)

¿Cuál es el lugar geométrico de los puntos de un plano H cuya distancia aun punto P, situado
a 2 metros del plano, varía entre 3 y 4 metros?

A) una circunferencia B) un círculo C) un sector circular

D) una corona circular E) un segmento

Solución
La respuesta es D

PO = 2, es la distancia del punto P al plano H.

PA = 3 y PB = 4

OA y OB son radios de dos círculos concéntricos.

Se observa que:

699
CAPITULO 21

ANGULOS DIEDROS
51.- DEFINICION.- Un ángulo diedro es la reunión de una recta y dos semiplanos no coplanarios
que tienen dicha recta común. La recta se llama arista del diedro y la reunión de la arista con
cualquiera de los dos semiplanos, es una cara del diedro.

Así, en la figura adjunta, la reunión de


los semiplanos P y Q , y su arista AB
es un ángulo diedro. La recta AB es la
arista de! diedro y las caras : P y Q.

Podemos denotar el diedro como


P-AB-Q ó sencillamente AB, si no hay
otro diedro con la. misma arista.

52.- ANGULO PLANO O RECTILINEO DE UN ANGULO DIEDRO.- Es el ángulo que forman dos
rayos perpendiculares a la arista en uno de sus puntos y situados uno en cada cara.

En la figura adjunta, OS y OT son dos


rayos perpendiculares a la arista AB, estan­
do OS en la cara R y en N. Luego, SÓT
es un ángulo plano del diedro R-AB-N.

Es evidente que todo diedro tiene infinidad,


de ángulos planos, Cada uno de ellos es en
realidad la intersección del ángulo diedro
con un plano perpendicular a la arista.

53.- MEDIDA DE UN ANGULO DIEDRO,- Es la medida de cualquiera de sus ángulos planos.

54.- CONGRUENCIA DE DOS ANGULOS DIEDROS.- Se dice que dos diedros son congruentes,
si sUs ángulos planos son respectivamente congruentes.

55.- CLASIFICACION DE LOS DIEDROS.- Tal como en geometría plana, los diedros se
clasifican según su medida ( agudos, obtusos, etc. ) o su posición ( consecutivos,
adyacentes, opuestos por la arista). De igual modo se tienen diedros complementarios y
suplementarios. ¡

i 701
D. AGUDO D. RECTO D. OBTUSO D. LLANO

D. CONSECUTIVOS DIEDROS ADYACENTES

DEFINICION.- Dos planos secantes son perpendiculares, si contienen un ángulo diedro


recto. En caso contrarío se llaman oblicuos.

Planos Perpendiculares Planos oblicuos

P1 Q M

TEOREMA.- Si una recta es perpendicular a un plano, entonces todo plano que contenga
a la recta será perpendicular al plano dado.

Hipótesis: Sea OC una recta perpendicular al plano Q, en el punto O, y P un plano cualquiera


JtX
que contiene a OC.

Tesis: P 1 Q
Demostración: Sea AB, la recta de inter­
sección de los planos P y Q. Entonces,
*—>
como por hipótesis: O C l Q , luego:
<r~> <—
>
OC X AB. ( Definición de recta perpendi-
cular a un plano). Sea OD una recta del
4->
plano Q, perpendicular a AB. Así, COD
es un ángulo plano del diedro C-AB-D,
determinado por los planos P y Q. Pero,
*—> <*•>
CO es perpendicular a OD ( ya que
<—) ♦
CO 1 Q ). En consecuencia, el diedro C-
AB-D es recto y el plano P i plano Q.

58.- TEOREMA.- Si dos planos son perpendiculares, entonces toda recta situada en uno de ellos
y perpendicular a su interscción, es perpendicular al otro plano.
(—)
- Hipótesis: Sean P y Q dos planos perpendiculares, y CD su intersección.
Sea AB una recta contenida en P y perpendicular a C D , en el punto B.
<-»
- Tesis: AB 1 Q

Demostración: En el plano Q, tracemos BE 1 C D ; luego, como AB 1 CD por hipótesis,


el ABE es ángulo plano del diedro:
P-CD-Q.

Siendo: plano P X plano Q, por hipótesis,


entonces el diedro P - CD - Q es recto-
A

ABE es recto.
4—>
De donde AB es perpendicular al plano Q,
4 ” > 4 ->

ya que AB X CD y AB X BE.

59.- TEOREMA.- Todo plano perpendicular a las caras de un diedro, es perpendicular a la arista
del diedro.

Hipótesis: Sea el diedro P - AB - Q y R el plano perpendicular a las caras, según las rectas
MN y NS.

Tesis: R X AB

Demostración: Por un punto O. del plano


4-> 4-> <— > 4->
R. trazamo's CE X MN y OF X NS.
Luego, por el teorema anterior:
4-* 4^
OE X P y OF X Q. Así tenemos que
OE y OF son perpendiculares a AB.
( por estar contenida en P y Q ).
703
<4 <-4 <-4
Entonces, como AE3 es perpendicular a dos rectas secantesde! plano R,( OE y OF ), será
<~4 __ <-4
perpendicular al plano **• AB 1 R ó R i AB.

SEMIPLANO BISECTOR: De un diedro, es el semiplano que lodivideen dosdiedros


congruentes.

60.- TEOREMA.- Todo punto situado sobre el semiplano bisector de un diedro, equidista de las
caras del diedro.
En la figura adjunta, S es el semiplano bisector del diedro M - AB - N.

P es un punto cualquiera de S .
Al trazar PQ 1 M y PR i. N, el plano
QPR es perpendicular a las caras del
diedro también lo será a la arista AB,
en el punto 0 .( teorema anterior).
<4 *4 <4 o
Así, OQ 1 AB y OR 1 AB; luego, GOR
es un ángulo piano de! diedro y su
bisectriz. De donde, por geometría plana,
en el plano QPR :

PQ = PR~j

NOTA: La demostración de las siguientes proposiciones se dejan como ejercicio al lector.

61.- Por una recta de un plano, se puede trazar a éste un plano perpendicular, y sólo uno.

62.- Por un punto cualquiera pasan infinidad de planos perpendiculares a un plano dado.

63.- Todo plano perpendicular a una recta situada en otro plano, será también perpendicular a
este plano.

64.- Por una recta, que no sea perpendicular a un plano, sólo pasa un segundo plano
perpendicular al primero.

65.- Todo plano perpendicular a la arista de un diedro, es perpendicular a las caras del diedro.

66.- Dados dos planos perpendiculares, entonces toda recta perpendicular a uno de ellos, en
cualquier punto de su intersección, estará contenida en el otro plano.

67.- Si dos planos son perpendiculares, entonces una recta perpendicular a uno de ellos, trazada
por un punto cualquiera del otro, estará contenida en este último.

704
r
CAPITULO 2 2

PROYECCIONES EN EL ESPACIO
68.- Se llama proyección de un punto sobre un plano, al pie de la perpendicular trazada del
punto al plano.

En la figura adjunta, A' es la proyec­ A


ción del punto A sobre el plano P.

La perpendicular AA’ se llama


proyectante y P es el plano de pro­
yección. Si B es un punto situado en
/
el plano P, la proyección de B es él
mismo.

69.- La proyección de una recta sobre un plano, es el conjunto de puntos del plano que son
proyecciones de los puntos de la recta.

Porejemploen lafigura: A’, B\ C’ ..., son


las proyecciones de los puntos A, B,
C,... respectivamente.
Q’ B* a;
D’ E’ La proyección de la recta L sobre el
plano Q es la línea C’ B’ D’ E’, que según
Q
demostramos a continuación, es una
recta.

70.- TEOREMA.- La proyección de una recta sobre un plano no perpendicular a ella, es una recta.

- Hipótesis: Sea r, una recta no per­


pendicular al plano P.

Tesis: La proyección de r sobre P es


una recta.

Demostración: Sea A un punto de la


recta r y A’ su proyección sobre el
plano P. Luego, r y A* determinan un
plano Q, perpendicular al P, por ser
AA' 1 P ( según teorema 57 ).

705
Sea r \ la recta de intersección de los planos P y Q. Entonces, si por cada punto de r
trazamos perpendiculares al plano P, todas ellas estarán contenidas en el plano Q y los
pies de éstas perpendiculares, que son proyecciones de los puntos de F sobre el plano P
serán puntos de r' . Por lo tanto, la recta r’ es la proyección de r sobre el plano P.

71.- OBSERVACIONES.- Si una recta es paralela a un plano, su proyección sobre dicho plano
es una recta paralela a ella; si está contenida en el plano, su proyección es la misma recta,
y si es perpendicular al plano su proyección es un punto. En éste último caso diremos que
la recta está proyectada “de punta".

? i

AB 11 P ; CD c P ; EF _L P

72.- PROYECCION DE UNA FIGURA CUALQUIERA.- Si F es una figura cualquiera en el


espacio, y P un plano, entonces la proyección de F sobre P es el conjunto de todos los
puntos que son proyecciones de los puntos de la figura F sobre el plano P.

Por ejemplo, la proyección de una línea curva es generalmente una línea curva; la de un
triángulo, es un triángulo, etc., pudiendo ocurrir que sea un segmento, en el caso de que la
línea o el triángulo estén contenidos en un plano perpendicular al plano de proyección.

706
ANGULO ENTRE RECTA Y PLANO

73.- DEFINICION.- Se llama ángulo entre una recta y un plano al ángulo determinado por la recta
y su proyección en dicho plano.

Así, en la figura adjunta, la recta AB


intercepta al plano P en el punto O, y

su proyección en dicho plano es A* B’;
luego, a es el ángulo entre la recta AB
y el plano P.

74.- TEOREMA.- El ángulo entre una recta y un plano es menor que el ángulo que forma la recta
con cualquier otro rayo que parte del punto de intersección y está contenido en el plano.

Hipótesis: Sea OA una recta, y O A ’ su proyección sobre el plano P. Sea OR cualquier otro
rayo en el plano P.

Tesis: AÓA' < AÓR

Demostración:

AA’ es perpendicular al plano P, por ser la


proyectante. Tomemos un punto B de O R ,
tal que OB = OA’... ( 1 ) y tracemos AB.
Luego, como AA' 1 P y AB es una oblicua
al plano, trazada desde el mismo punto A:
AA’ < AB ... ( 2 ), entonces, en los triángu­
los AOA’ y AOB, se tiene: AÓA' < AÓB
( por relaciones ( 1 ) y ( 2 ) ), con lo cual
queda demostrado el teorema.

LIN E A DE M A X IM A P E N D IE N T E

75.- TEOREMA.- Sean P y Q, dos planos secantes según la recta AB. Entonces, las rectas del
plano P, que forman un ángulo máxi­
mo con el plano Q, son perpendicula­
res a AB.

Hipótesis: OE y OF son dos rectas


contenidas en el plano P, y OF 1 AB,

OFO' = a y OÉO' = p

Tesis: a > p

707
Demostración: Como O’ es proyección de O sobre el plano Q 0 0 ' ± Q. Luego, por
hipótesis OF _L AB; entonces, con el Teorema de las tres perpendiculares : O'F _L ^ 5 .
Además O ’E es proyección de OE sobre Q. Y, siendo en el A O ’FE: O 'F < O'E; existe un
punto R, tal que O ’R = O’F.
De donde : A O’RO = A O’FO ORO' = 0 F 0 ‘ O R O 1= a , observándose en el
A OER, que : a = p + EÓR ........... ( Teor. del Z externo ) .

a>p —

76.- DEFINICION.- En el gráfico anterior, si Q es un plano horizontal, la recta OF sé llama Línea


de máxima pendiente del plano P, respecto al Q.

P R O YEC C IO N ES DE REGIONES PLANAS

77.- TEOREMA.- El área de la proyección de una región triangular, sobre un plano, es igual al área
de dicha región triangular multiplicada por el coseno de! diedro que forman el plano del
triángulo proyectante y el plano de proyección.

(1o) Hipótesis:

Sean P y Q dos planos secantes en


E F ; ABC es un tr) [ingulo contenido
en P, tal que AC EF y A’B’C’ su
proyección en Q . a mide el diedro
P-EF-Q.

- Tesis: área A A’B’C’ = (área A ABC). Cos a .

Demostración: Como a es un ángulo plano del diedro P-EF-Q , entonces BN 1 EF y


B' N 1 EF. Además, según hipótesis : AC || EF AC | | q , por lo que A’C’ = AC y
B Ñ _L ÁC, BT'J _L Á7^

Si por H se traza paralela a N B d i c h a paralela estará contenida en el plano NBB’ y se prue


ba fácilmente que B’H’ = ( BH ) Cos a .

( A'C'KB'H')
Luego: área A A’B’C’ =

( AC )( BH )
ésto es: área A A’B’C’ = Cos a

De donde : área A A'B’C’ = ( área A ABC ) Cos a

708
s

( 2 o)

Si el A ABC no tiene algún lado para­


lelo a EF, trazamos BM || EF.

Luego:

área A A’B’M’ = ( área A ABM ) Cos a


área A B’M’C’ = ( área A B M C ) Cos a

De donde, al sumar miembro a miem­


bro:

área A A’B’C’ = ( área A ABC ) Cos a

78.- TEOREMA.- El área de la proyección de una región poligonal, sobre un plano, es igual al área
de dicha región por el coseno del ángulo que forman el plano del polígono proyectante y el
plano de proyección.

* Este teorema es fácil de demostrar, teniendo en cuenta que un polígono cualquiera se puede
descomponer en triángulos.
** Así mismo, el teorema es válido para todo tipo de región plana, sea cual fuere su forma.

M IN IM A D IS T A N C IA E N T R E D O S R E C T A S A L A B E A D A S

79.- La mínima distancia entre dos rectas que se cruzan en el espacio es la longitud del segmento
perpendicular a ambas.

a) ( Según hemos demostrado en el teorema N°36, si dos rectas se cruzan entonces por una
de ellas pasa un plano paralelo a la otra. Esto nos sugiere una forma de determinar la mínima
distancia entre tales rectas ).
A M B
Sean, por ejemplo, AB y CD dos
rectas alabeadas ; luego, por &D
pasa un plano P, paralelo a AB. En
seguida, proyectamos la recta AB
sobre P, determinando el plano Q
perpendicular a P. Si por el punto de
intersección N, de las rectas A’B’ y
CD, trazamos la perpendicular al pla­
no P, ésta se hallará en el plano Q,
cortando a AB en M. Entonces,

como MN 1 P y M N 1 A'B'

Siendo A'B*' A § , tam bién:

709
Es decir, hemos demostrado la existencia de un segmento MN, perpendicular a AB y ÜD,
teniendo sus extremos sobre dichas rectas. MN es la mínima distancia entre AE? y £ d .

Nótese que cualquier punto de la recta AB, equidista del plano P. De manera que para
determinar la mínima distancia entre X b y CD, basta calcular la distancia de un punto
cualquiera sobre AB, al plano P : AA’ = MN = BB’ = ....

b) En la práctica, a veces, es convenien­


te proyectar ambas rectas en un plano
perpendicular a una de ellas. Como
en la figura adjunta, donde el plano Q
es perpendicular a la recta AB. El
punto O es la proyección de AB sobre
Q y t'D' la proyección de CD en Q.
( b'D' es la misma recta de intersec­
ción entre P y Q ).
Luego, si trazamos OE 1 P, OE es­
tará contenido en Q y : OE _L
OE 1 . Es decir, OE nos da la
mínima distancia entre y CD.

Por último, si en el plano Q, se conocen las longitudes O D \ OC’ y C’D \ será fácil calcular
OE.
( Grafique el lector, el segmento que conecta perpendicularmente a AB y CD ) .

* NOTA: Desde luego, existen infinidad de segmentos que tienen sus extremos en ambas
rectas alabeadas; sin embargo, el de menor longitud es aquel perpendicular a las dos.
A M B

MN < MR

80.- OBSERVACIONES.-

1) Se dice que una figura plana se proyecta


en verdadera magnitud (V.M.) sobre un
plano, cuando el plano que la contiene es
paralela al plano de proyección.
En lafigura adjunta, el segmento AB no es
paralelo a plano P ;

710
luego, A‘B* no está en V.M. ( A'B' < AB ) .
El plano que contiene al A CDE es paralelo al plano P. Entonces C’D’E’ está en V.M.

2) Se dice que un plano se proyecta “de canto*’, sobre otro plano, si le es perpendicular.

Sean por ejemplo los planos per­


pendiculares P y Q, según la recta
r. Luego, diremos que el plano Q
se proyecta “de canto” en el pla­
no P.

3) Si dos planos son perpendiculares entre sí y perpendiculares a un tercer plano, entonces dos
rectas, cualesquiera en los dos primeros, tienen sus proyecciones perpendiculares entre sí,
en el tercer plano.

Sean P y Q, dos planos perpendiculares


entre sí, y perpendiculares además al plano
R; p está contenida en P y q en Q.
x es la proyección de p en R; y es la
proyección de q en R.
ln rí Luego: x l y , por ser intersecciones de
planos respectivamente perpendiculares.
Nótese que e! hecho de que las proyeccio­
nes x e y, de las rectas p y q , sean
perpendiculares no significa que p y q lo
sean.

4) a) Si dos rectas perpendiculares se proyectan sobre un plano paralelo a una de ellas, entonces
sus proyecciones son perpendiculares entre sí.

Sean las rectas a y b, de modo que:


á X b, y b es perpendicular a la inter­
sección de M y S, por lo que el piano T,
paralelo a b, será perpendicular a M y
S. Luego, en T : á ' l b ' .

b) Si dos rectas perpendiculares a y b se proyectan sobre un plano P, según las rectas


¡' y b' .y dichas proyecciones son perpendiculares entre sí, entonces por lo menos una
de las rectas: á ó b , es paralela al plano P.

711
\

Consideremos el gráfico adjunto, y


supongamos que á no es paralela al
plano P.

Sea S el plano determinado por las


rectas b y b’.

Luego: b' 1 a' ( por hipótesis )

y b ' l proyectante OE,

por lo tanto: b 'lR (1).

Asimismo: á 11 b' y á' _L proyectante OE a ' l S , por lo que á ' i b , ya-quetTestá


contenida en S.
A s í : b 1 á ( por hipótesis ) y b 1 a* ( por lo anterior).
De donde : b l R ........... ( 2 ).

Entonces, con ( 1 ) y ( 2 ): b 11 b' , y por lo tanto b 11 P .

c) Si dos rectas a y b , se proyectan sobre un plano P paralelo a una de ellas y sus


proyecciones á ‘ y b' forman ángulo recto, entonces las rectas a y b son perpendiculares
entre sí.

* Consideremos el gráfico adjunto y supo


niendo que b es paralelo al plano P,
tenemos:

b b' y como b ' l R ,


blR . Luego : b _L á , a = 90°

5) Si una recta y un plano son perpendiculares, y se proyectan sobre un plano paralelo a la


recta (o perpendicular al plano), entonces la recta se proyecta en “ verdadera magnitud ”
y el plano “de canto", siendo además ambas proyecciones perpendiculares.

Por ejemplo, en la figura adjunta, la


recta r y el plano P son perpendicu­
lares entre sí . S es un plano de
proyección paralelo a F . Luego, r se
proyecta según r ' ( r ' 11 r ) y la recta
p1 es la proyección “ de canto ”, del
plano P en S, donde P ' i f ' .

712
\
Cabe señalar que toda recta contenida en P y perpendicular a S se proyecta “de punta” en
dicho plano. Así mismo, todo segmento de recta contenido en P y paralelo a S, se proyecta
en “verdadera magnitud-” sobre S.

6) Si dos rectas alabeadas se proyectan sobre un plano perpendicular a los planos paralelos
que las contienen, entonces sus proyecciones son rectas paralelas y la mínima distancia
entre dichas rectas alabeadas es igual a la distancia entre las proyecciones paralelas.

En efecto, según la figura adjunta, sean q y r las rectas alabeadas situadas en los planos
paralelos Q y R. ( según el N°45.2, Q y R son únicos ). Sea P un plano de proyección
perpendicular a Q y R : q' y r‘ son las proyecciones “de canto” de Q y R, siendo además
proyecciones de q y r , así como de todas las rectas no perpendiculares a P, contenidas
en Q y R. ( Las perpendiculares se proyectan “de punta” ).
Luego : q' es paralela a r 1 y d , distancia entre ellas, es la distancia entre los planos Q y
R , y en consecuencia la mínima distancia entre las rectas alabeadas q y r.

NOTA: Como ejercicio , grafique el lector, el segmento que conecta perpendicularmente a


q y r.

7) Si una recta y un plano son paralelos, y se proyectan sobre un plano perpendicular a la recta,
entonces dicha recta se proyecta de “punta" y el plano “de canto”.

En la figura: m II Q y P es un plano de
proyección perpendicular a m ; por lo
tanto P 1 Q .
El punto A y la recta F son proyeccio­
nes de m y Q en P, respectivamente.
* Todos los segmentos contenidos en
Q y perpendiculares a m, se proyec­
tan en “ verdadera magnitud ”.

713
8) Si una recta r es perpendicular a un plano P, entonces toda recta perpendicular a r se
proyecta en forma paralela sobre P.

En efecto, según el gráfico:


r 1 P y m _L r ,
Por lo tanto :
m || P.

Luego, m ’ es la proyección de m
sobre P : m' m.

(*) Nótese que todo segmento per­


pendicular a r se proyectará sobre P
en su verdadera magnitud. Además,
todo plano que contenga a r se pro­
yectará sobre P, “de canto”.

9) Si A , B y C son tres puntos de una recta y A ’ , B’ y C’ sus proyecciones sobre


cualquier plano, entonces :

AB A1B
BC B 'C ‘

* En particular, si B es punto medio de


AC, B’ lo será de A ' C \

10) PROBLEMAS SOBRE MINIMA DISTANCIA ENTRE RECTAS ALABEADAS.

En algunos problemas de éste tipo, no es sencillo graficar el segmento que indica la mínima
distancia entre rectas que se cruzan, pero simpre es posible determinar la longitud de dicho
segmento. Para ello, el lector debe leer detenidamente los apartados números 79 y 80.

* M E T O D 0 1 Proyectamos ambas rectas en un plano perpendiculara una de ellas. Esta recta


se proyectará como un punto. La mínima distancia pedida queda determinada por la longitud
del segmento perpendicular trazado desde dicho punto, a la proyección de la otra recta ( ver
7 9 - b ).

Por supuesto, todos los segmentos paralelos al plano de proyección, se proyectarán en él,
en su verdadera magnitud, (por el N°80.8, éstos segmentos han de ser perpendiculares a la
recta que se proyecta de punta).
Veamos el siguiente ejemplo:

* En un plano Q, está contenido el triángulo equilátero ABC, de lado 2 tJ~3 cm. Por el vértice
C, se levanta CD perpendicular a Q, siendo CD = 4cm. Hallar la mínima distancia entre las

714
rectas que contienen a los segmentos AC y BD
Solución
Del gráfico, sea P un plano perpendi­
cular a AC. Luego: P 1 Q , por el
teorema 57. Plano de
proyección
Proyectamos el conjunto sobre P: D
O Proyección de AC.
B'D’ -> Proyección de BD.
OD' -» Proyección de C D .

Como CD _L AC .*. CD 11 OD1 y


OD’ = CD = 4. { CD se proyecta en
su verdadera magnitud}.

La longitud del segmento OR ( OR J_B‘D ' ), dá la mínima distancia entre X c y B Ó ,

pudiéndose hallar en el A D’O’B’, teniendo en cuenta que OB ’ es la proyección en verdadera


magnitud de la altura BH del A ABC: OB’ = BH , donde:

_ 2 /3
BH = /3 = 3 • • OB' = 3

1 1 1 1 1 1
Luego : + + OR = 2,4cm
( OR f ( OD’ f ( OB' f ( OR* )

* METODO 2.- En éste caso:

(a) Formamos un plano que contenga a una de las rectas y sea paralelo a la otra, ( según teorema
36, éste plano es único ).

(b) Proyectamos el conjunto sobre un plano perpendicular al formado en ( a ).

(c) El plano formado en ( a ) y la otra recta, se proyectan según dos rectas paralelas. La
separación entre éstas rectas dá la mínima distancia pedida.
Así, para el gráfico:

715
* m y n son las rectas alabeadas.
* Q 11 m ; P es el plano de proyección perpendicular a Q.
* m ’ , proyección de m en P.
* ñ \ proyección de Q en P ( es la misma proyección de I T ).
* d: mínima distancia pedida.

Ejemplo:

La figura adjunta, muestra un cubo de


arista “a”.

* M es punto medio de B C .

* N es punto medio de C D .

Hallar la mínima distancia entre


AD y fm .

Solución

ComoAG II BD y MN II BD,entonces: AG II MN. Luego, el plano AGD es paralelo a MN.


Proyectamos el conjunto sobre un plano P, perpendicular al AGD. Así tenemos que P 11AHG
y BCDE, ya que ambos planos son perpendiculares al AGD.

M'N' Proyección de MN ; A 'U 1 Proyección del plano AGD ( y de toda recta


contenida en é l ).

La distancia entre M'N' y A‘D” soluciona el problema y es igual a la cuarta parte de la


longitud de la diagonal del cuadrado A’ H’ D’ E’ .

aJ~2
Respuesta : — A—

716
* Ejem plo:

En la figura, la arista del cubo tiene


longitud “a”.
Hallar la mínima distancia entre las
rectas BD y CF.

Solución

* Proyectamos la figura, sobre un plano R, perpendicular a BD.

* §D se proyecta según el punto M.

* Todas las perpendiculares a § D , se proyectan en V . M . , de modo que : C’G’ = CG = a f é


y C’H’ = CH = a.

* MN, es la mínima distancia pedida y se puede hallar en el A C’MO:


1 1 1 1 1 1
+
\2
(M N )
(MN)2 (C'M )2 (OM)2
&

Rpta: MN =
H 6

717
CAPITULO 2 3

SIMETRIA
I.- SIMETRIA CON RELACION A UNA RECTA

8 1 DEFINICION.- Dos puntos A y A' son simétricos con relación a una recta L, si dicha recta es
mediatriz del segmento AA’. La recta L se llama eje de simetría.
Así mismo, dos figuras F y F’ son simétricas con relación a un eje, si para cada punto de
F, existe en F su correspondiente simétrico.

Figura (1 ). Figura ( 2 ).

La fig. 1 muestra dos puntos A y A’ , simétricos entre sí con relación a una recta L . El
segmento AA’ está contenido en el plano P, perpendicular a L .

En la fig.2 se observan dos triángulos: ABC y A’B’C’, simétricos con relación al eje L. Los
planos P, Q y R que contienen a los segmentos AA1, BB1 y CC' son perpendiculares a
L , por lo que según el teorema N°39 : P 11 Q 11 R -

719
82 .- TEOREMA.- Dos segmentos simétricos con relación a un eje, son congruentes entre sí

Demostración: Sean los segmentos


AB y A’B’, simétricos con relación al
eje L. Consideremos que los segmen­
tos AB, A’B’ y la recta L no son
coplanarios ( fig.a). Los pianos P y Q
son perpendiculares a L y contienen
a AA' y BB' respectivamente.

En la figura ( b ) se indica la demostra­


ción: se proyecta AA’ sobre el plano
Q, según CD. Luego :
A BO’C s A B’O’D

( postulado LAL)
BC = B'D

Además, como AC = A'D', entonces


los triángulos rectángulos BC A y B’DA’
son congruentes:

AB = A'B

NOTA: Demuestre el lector el caso en


queAB y A'B* seancoplanarioscon
el eje L .

83.- COROLARIO.- Dos figuras simétricas con relación a un eje, son congruentes entre sí.
* En efecto, consideremos los triángulos simétricos ABC y A’B’C’, de la figura 2 en el apartado
N°81. Aplicando el teorema anterior:
AB = A'B’ , BC = B ' C , AC = A'C'. Por lo tanto : AABC s A A’B’C’.

84.- DEFINICION.- Sea P un plano perpendi­


cular a una recta L en un punto O, y sea
A un punto situado en dicho plano. Deci­
mos que el punto A gira alrededor de L ,
( eje) si describe una circunferencia ( ó un
arco de ella ), con centro O y radio O A .
En la figura adjunta, el punto A ha girado

360° alrededor del eje L. (Nótese el


sentido del giro).
El plano P se llama “piano de giro” del

punto A, con respecto a L .

720
* Asimismo, se dirá que una figura gira un ángulo ct° alrededor de un eje, cuando todos sus
puntos describan un arco con tal medida y en el mismo sentido, en planos perpendiculares
al eje.

85.- PROPIEDAD FUNDAMENTAL.- Dos figuras simétricas con relación a un eje, pueden
superponerse mediante el giro de 180°, de una de ellas, sobre el eje.

86.- EJE DE SIMETRIA DE UNA FIGURA.- Una figura admite un eje de simetría cuando todos
sus puntos son, dos a dos, simétricos con relación a dicho eje.

Por ejemplo, un cuadrado admite cin­


co ejes de simetría; todos ellos pasan
por su centro;

Dos contienen a las diagonales;


dos contienen los puntos medios de
lados opuestos, y el quinto es perpen­
dicular ai plano del cuadrado.

SIMETRIA CON RELACION A UN PUNTO

87.- DEFINICION.- Dos puntos A y A’ son simétricos con relación a un punto O, si dicho punto O,
está situado en el medio del segmento AA\
El punto O se llama centro de simetría.
Así mismo, dos figuras F y F’ son simétri­
cas con relación a un centro, si para cada
punto de F, existe en F* su correspondien­
te simétrico.
721
D
8.- CONCLUSIONES.-
* Dos figuras simétricas con respecto a un punto, son congruentes, pero dispuestas en sentido
inverso. ( Ver gráfico anterior).

* Dos figuras planas simétricas con re­


lación a un punto situado en el plano
que las contiene, se pueden superpo­
ner girando una de ellas 180° alrede­
dor de un eje que pase por el centro de
simetría y perpendicular al plano.
( figura adjunta).

89.- CENTRO DE SIMETRIA DE UNA FIGURA.- Una figura admite un centro de simetría, cuando
todos sus puntos son, dos a dos, simétricos con relación a dicho punto.
Por ejemplo, todo paralelogramo (romboide, rombo, rectángulo, cuadrado) tiene su centro
de simetría en el punto de intersección de las diagonales.

III SIMETRIA CON RELACION A UN PLANO

90.- DEFINICION.- Dos puntos A y A’ son simétricos con relación a un plano P, si dicho plano es
perpendicular, en su punto medio, al segmento AA'.
De modo análogo, si a cada punto de una figura F, le corresponde un punto simétrico en otra
figura F \ con relación a un plano P, se dice que F y F‘ son simétricos respecto a P.

9 1 PLANO DE SIMETRIA DE UNA FIGURA.-Así como en los casos anteriores, una figura
admite un plano de simetría cuando todos sus puntos, dos a dos, son simétricos respecto a
dicho plano. Por ejemplo, un cuadrado admite cuatro planos de simetría, todos ellos
perpendiculares al plano del cuadrado: dos contienen a las diagonales y los otros dos pasan
por los puntos medios de lados opuestos.

722
CAPITULO 2 4

ANGULOS POLIEDROS
92.- SUPERFICIE PIRAMIDAL.-La figura
1, muestra una poligonal contenida
en el plano H, siendo A uno de sus
extremos.

El punto O es exterior al plano H.

Luego, si el rayo OA se traslada a lo


largo de la poligonal, manteniendo fijo
el origen, se genera una superficie
piramidal.

O * es la generatriz de la superficie; el
punto O es el vértice y la poligonal
recibe el nombre de directriz.

En este caso la superficie generada


es abierta.

Si la directriz es un polígono, como en


la figura 2, la superficie piramidal ge­
nerada es cerrada.

93.- ANGULO POLIEDRO.- Llamado también ángulo sólido o anguloide, es la figura determi­
nada por una superficie piramidal cerrada. La fig. 2 muestra un ángulo poliedro.

Los rayos OA, OB, ÓÓ, ..... se llaman aristas; el punto O es el vértice; tanto las regiones

planas determinadas por dos aristas consecutivas, como los ángulos AÓB, B ÓC , , son las
caras y los ángulos diedros determinados por caras consecutivas son los diedros del
anguloide.

Según el número de caras ( 3, 4, 5,.... n ) un anguloide recibe el nombré de ángulo triedro,


ángulo tetraedro, ángulo pentaedro ángulo eneaedro.

94.- ANGULO TRIEDRO.- Es el ángulo poliedro de tres caras. Sus elementos son :

723
* Tres caras : BÓC = a , AÓC = b ,
AÓB = c.

* Tres diedros: OA, OB, O C ( o simple­


mente A, B, C).

El triedro de la figura adjunta se deno­


ta: OABC.

95.- CLASIFICACION DE LOS ANGULOS TRIEDROS.- De acuerdo a sus caras un triedro


puede s e r :

a) Escaleno, si sus tres caras son desiguales ( a * b * c ).


b) Isósceles o isoedro, si tiene dos caras congruentes.
c) Equilátero, si sus tres caras son congruentes (a = b = c)
d) Rectángulo, si una cara mide 90° ( a = 90° a b, c * 90°)
e) Birectángulo, si tiene dos caras de 90° (a = 90°, b = 90° , c * 90°)
f ) Trirectángülo, si cada cara mide 90° (a = b = c = 90°)
9

La siguiente figura muestra triedros rectángulo, birectángulo y trirectángülo; respectiva­


mente, de vértice O, V, y Q.

RELACIONES ENTRE CARAS


96.- TEOREMA.- En todo triedro una cara
es menor que la suma de las otras dos
y mayor que su diferencia.

- Hipótesis: Sean A Ó B . A Ó C y B Ó C
las caras del triedro OABC, con
AÓB mayor que las otras dos y
AÓC > B Ó C .

724
- Tesis: AÓB < AÓC + BÓC y

AÓB > AÓC - BÓC

Demostración : Sea E un punto contenido en la cara AOB y situado en AB,tal que :


AÓE = AÓC. Además el punto C puede ser tal que OE = OC.
En consecuencia A AOE = A AOC ( postulado LAL ). Luego: AE = A C (1 ).
En el triángulo ABC se cumple: AE + EB < AC + BC. De donde, cancelando AE yAC por
ser iguales : EB < B C ( 2 ).
Esto implica que EÓB < BÓC, yaque los triángulosOEB y BOC tienen OE = OC, OB común
y EB < BC.
Así: EÓB < BÓC, sumando AÓE = AÓC a uno y otro miembro:

AÓE + EÓB < BÓC + AÓC

• • AOB < BOC + AOC

Asi mismo podemos escribir: AÓC < AÓB + BÓC .parala cara AOC. ó AÓB + BÓC > AÓC

AOB > AOC - BOC

97.- TEOREMA.- En todo ángulo poliedro, la suma de las caras es mayor que 0o y menor que
360°.

- Hipótesis: Sea el ángulo pentaedro OABCDE.


( En general un ángulo eneaedro ).

- Tesis:
0o < AÓB + BÓC + CÓD + DÓE + EÓA < 360°

Demostración:

ABCDE, es un polígono determinado


por el plano secante a las caras del
anguloide.

Por el teorema anterior:

Triedro A : EÁB < OÁE + OÁB

Triedro B : ABC < OBA + OBC

Triedro C : BCD < BCO + OCD.

Triedro D : CDE < CDO + ODE

Triedro E : AÉD < AÉO + OÉD

Sumando miembro a miembro :


EÁB + ABC + BCD + CDE + AÉD < OÁE + OÁB + OBA + OBC + BCO +

725
Donde el primer miembro representa la suma de ángulos internos del polígono ABCDE y
el segundo miembro es la suma de ángulos en los triángulos, menos los ángulos en el vértice
“ O".
180° ( 5 - 2 ) < 5 x 1 8 0 ° - suma de ángulos en “ O "

Luego: suma de ángulos en “O" < 360°.

0o < AOB + BOC + COD + DOE + EOA < 360

NOTA: Según éste teorema, si a, b y c son las caras de un triedro, entonces:

0 °< a + b + c < 360

98.- DEFINICION.- Se llama simétrico de un triedro SABC, al triedro SA'B'C’, determinado por los
rayos SA*, SB’ y S C \ opuestos de SA, SB y s 3 t respectivamente.

99.- DEFINICION : Dado un triedro OABC y un punto cualquiera O* del espacio , se trazar
07?, O'E? y 0 'C \ rayos perpendiculares a las caras OBC, OAC y OAB, respectivamente
de modo que O’A* y O A* tengan sus sentidos hacia un mismo semiespacio con respecto a
plano OBC; asimismo O'B' y oS a un mismo lado del plano OAC y O’C y 0(5 de ur
mismo lado del plano OAC.

El triedro O’A’B'C’ se llama polar o suplementario del triedro OABC.

726
A’

OBC^J

TRIEDROS
SUPLEMENTARIOS
* La figura 1 indica como deben graficarse las aristas del triedro polar.
* La figura 2 muestra los triedros suplementarios.

100.-TEOREMA,- Si el triedro O ’A’B’C’ es suplementario del triedro OABC, entonces el triedro


OABC es también el suplementario del triedro O’A’B’C’.

Demostración:
Bastará probar que O A , OB y OC son, respectivamente, perpendiculares a los planos
O ’B’C ’, O ’A’C ’ y O ’A’B’, cumpliendo así con la definición dada.

Bien, en los gráficos anteriores:

O 7? 1 OBC ( hipótesis ) 0 (5 1 O 7^ ............ (1 ) .

O B 1 ± ’ OAC ( hipótesis ) O ? X O7!? ............ ( 2 ) .

Luego, de (1 ) y ( 2 ) : 0 (5 será perpendicular al plano O ’A'B’, que contiene a O'A^ y 0 ' B \

De modo análogo se concluye que:


OA 1 O 'B 'C ' y O B I O 'A 'C L

101. - TEOREMA. - D a d o s dos trie d ro s


suplementarios, cada cara de uno de
ellos es el suplemento del correspondien­
te diedro del otro.

Demostración:

Considerando la figura adjunta, donde se


muestran las caras OAB y OBC del triedro
OABC y la cara O ’A ’C ’ de su triedro suple­
mentario O ’A’B’C ’, trazado con vértice en
O ’, tenemos que :
Las prolongaciones de C ’O ’ y A’O ’ inter­
ceptan a los planos OAB y OBC, en los
puntos M y E; respectivamente.

7 9 7
El plano determinado por MC' y EA‘ es perpendicular a los planos OAB y OBC( porser
m 6‘ j L OAB y ÉA' ± OBC por lo que también será perpendicular a OB ( teorema N°59),
en el punto R. Luego : OB _L y OBI MRE es un ángulo plano del diedro OB.
Entonces, en el cuadrilátero O'MRE.

MÓ'E + MRE = 180°, pero: MÓ'E = A'O'C'?

diedro OB + cara A’O ’C’ = 180°

Análogamente se demuestran los otros casos.

** En conclusión, si a continuación graficamos los triedros OABC y O’A’B’C’, donde

caras a, b, c caras a’,b\c’


* Triedro OABC * Triedro O’A’B’C’ i
diedros A, B, C diedros A \ B \ C

Se tendrá: a + A’ 180° también: A + a’ = 180°


b + B’ 180° * B + b’ = 180°
c + C’ 180° C + c’ = 180°

102.-TEOREMA.-En todo triedro, la suma de los diedros está comprendida entre 180° y 540°.

Para la demostración, consideremos un triedro OABC y O’A’B’C’, su triedro polar. Llamando


A, B, C, los diedros del primero y a’, b \ c’ las caras del segundo ; de acuerdo al teorema
N- 97, tenemos que:
0o < a’ + b’ + c* < 360°

Pero: a’ = 180° - A , b’ = 180° - B , c* = 180° - C

Luego:
0o < ( 180° - A) + ( 180° - B)+ ( 180° - C)< 360

-540° < - A-B-C < - 180°

De donde
180° < A + B + C < 540

728
CONGRUENCIA DE ANGULOS TRIEDROS

103.- DEFINICION.-Dos ángulos triedros OABC y XYZW, son congruentes, si sus caras y ángulos
diedros son respectivamente congruentes, de modo que a caras congruentes se opongan
diedros congruentes y recíprocamente.

Es decir, si las caras: a = y , b


z , c = w, y los diedros: A = Y ,
B = Z , C = W.

104.-TEOREMA.- Si dos triedros son congruentes, sus triedros polares también lo son.

Demostración: Sean O’A’B’C’ y X’Y’Z W , triedros polares de los triedros congruentes


OABC y XYZW, respectivamente. Según el teorema N° 101, sabemos que:
a ' = 1 80° - A , y’ = 1 8 0 ° -Y.
Pero, como A = Y, por hipótesis, entonces: a’ = y’

También: b’ = 180° - B , z’ = 180° - Z. Siendo B = Z, b’ = z’

Análogamente: c’ = x\ Así tenemos que las tres caras de los triedros polares son
respectivamente congruentes.

Para probar que los diedros son también congruentes: A' = 180° - a , Y ’ = 180° - y . Como
a = y, entonces: A’ = Y’

Así mismo: B’ = 180o* - b , Z ’ = 180° - z ; con: b = z B’ = Z ’


.•

Además, so-llega a que C’ = W ’

En consecuencia, los triedros O ’A’B’C’ y X’Y’Z ’W ’ son congruentes.

105.-CASOS DE CONGRUENCIA DE TRIEDROS.- Los casos ae congruencia de triedros son


análogos a los de congruencia de triángulos.

1o) Postulado: “Dos triedros son congruentes si tienen dos caras y el ángulo diedro compren
dido, respectivamente congruentes”.

Para la figura adjunta, el postulado


indica que s i : * V
a=y
c=w
y B= Z

Entonces:
triedro OABC == triedro XYZW

729
2o) Teorema: Dos triedros son congruentes si tienen una cara y los ángulos diedros adyacentes,
respectivamente congruentes.

- Hipótesis: Sean los triedros OABC y


XYZW, tales que a = y, B = Z , C = W.

- Tesis: Triedro OABC = triedro XYZW

Demostración:

Sean O’A’B'C’ y X’Y ’Z ’W ’ , triedros


suplementarios de OABC y XYZW,
respectivamente.

Probemos que O’A’B’C’ y X’Y’Z'W ’ son congruentes.

Veamos: b’ = 180° - B , z’ = 180° - Z ( por teorema N°101 )

Pero: B = Z , ( hipótesis ). Entonces: b’ = z’

También: c’ = 180° - a , Y’ = 180° - y . Como : a = y , ( hipótesis ) .


A’ = Y*

Así tenemos que los triedros O ’A’B’C ’ y X’Y’Z’W ’ son congruentes, ya que tienen dos caras
y el diedro comprendido, respectivamente congruentes, cumpliendo el Postulado N° 105.
Entonces, según el teorema N° 104, los triedros OABC y XYZW serán también congruentes.

3o) Teorema: Dos triedros son congruentes si tienen sus tres caras respectivamente congruen­
tes.

4o) Teorema: Dos triedros son congruentes si tienen sus tres diedros respectivamente con­
gruentes.

NOTA : Las demostraciones de los casos 3o y 4o se dejan como ejercicio al lector.

106.-TEOREMA.- Dos diedros simétricos son congruentes entre sí. (La demostración de éste
teorema es sencillo y se deja como ejercicio al lector).

RELACION ENTRE CARAS Y DIEDROS DE UN TRIEDRO

107.- TEOREMA.- Si dos caras de un triedro


son congruentes, entonces los diedros
opuestos a dichas caras son con­
gruentes.

- Hipótesis: Sea el triedro OABC, con


las caras a y c congruentes.

- Tesis: A = C

730
D em ostración

Trazam os OÍ?, bisectriz de la cara A Ó C.


o
Luego, se determinan dos triedros OAEB y
O EBC, congruentes, ya que:
a = c ( hipótesis ) .

AÓE = EÓC y EÓB común; tienen sustres


caras respectivamente congruentes. Por
tanto, a caras congruentes, en triedros
congruentes, se oponen diedros congruen-
A

tes: A = C (se oponen a EOB).

1 0 8 .-T E O R E M A .- Si dos diedros de un triedro son congruentes, entonces las caras opuestas a
dichos diedros son congruentes.

H ipótesis: Consideremos el triedro


OABC, donde los diedros B y C son
congruentes. B

- Tesis: AÓC s AÓB

D em ostración:

* Trazam os A E , perpendicular al plano


BOC y luego, EM y EN , perpendiculares

a ÓB y O C , respectivamente.
Luego, por el teorem a de las tres perpendi­
culares: AM _L OB y AN _L O C .

AM E y AÑE son ángulos planos de los

diedros B y C AME = AÑ E. Luego


A AEM = A AEN , entonces EM = EN y
AM = AN.

En seguida:
A O M E s A ONE O M = ON y
finalmente:
A O M A s AONA.

De donde: AÓB = A Ó C , tal como se quería


demostrar.

1 0 9 .-T E O R E M A .- Si un triedro tiene dos diedros desiguales, las caras opuestas son desiguales
y al mayor diedro se opone mayor cara.

731
110.-TEOREMA.- Si un triedro tiene dos caras desiguales, los diedros opuestos son desiguales
y a mayor cara se opone mayor diedro.

* ( Demuestre el lector éstos teoremas ).

PROBLEMAS RESUELTOS
(C apítulos 21,22, 23, 2 4 )

1) ¿Cuántas de las siguientes proposiciones no son falsas:


I. Por una recta perpendicular a un plano, pasan infinidad de planos perpendiculares
al primero.
II. Poruna recta paralela a un plano, pasan infinidad de planos paralelos al primero.
III. Dos planos perpendiculares auna misma recta, son paralelos entre sí.
IV. Dos planos perpendiculares a un tercer plano, son paralelos entre sí.
V. Dos rectas perpendiculares a un mismo plano, son paralelas entre sí.
A) 1 B) 2 C) 3 D) 4 E) 5
Solución
I. Verdadera II. Falsa III. Verdadera
IV. Falsa V. Verdadera
Rpta: 3 verdaderas
Veamos por qué:

m // H
R // H
R es único P // Q

P1H y Q lH
No necesariamente P y Q m l P y r_LP
son paralelos. m //r

732
Indicar, verdadero (V) o falso (F):

I. En todo triedro, la suma de las medidas de los tres diedros exteriores es mayor
que 0o y menor que 360°.

Si A, B, C, son las medidas de los diedros de un triedro, entonces: B + C <


180° + A.

A) VV B) VF C) FF D) FV

Solución
I. Sea el triedro O - ABC, con
eA, e8 y ec, como medidas
de sus diedros exteriores. Se
sabe que, para los diedros
A, B y C:

180° < A + B + C < 540 O ... ( 1 )

Pero: A = 180° - e En ( 1 ):
B = 180° - e B 180° < (180° - eA) + (180° - eB) + (180° - ec) < 540'
C = 180° - e -360° < - (eA + eB + ec) < 0o

De donde:

verdadera (V)

Considerando el mismo gráfico; las caras del triedro polar, miden: (180° - A), (180'
- B) y (180° - C). Deben cumplir: (180° - A) - (180° - B) < (180° - C).

De donde: verdadera (V)

Rpta: (A)

ABC, es un triángulo recto en B. ACD, es un triángulo equilátero contenido en un plano


perpendicular al plano ABC. Si AC = 8, hallar BD.

A) 8 B) 8>/2 C) 4 V¡3 D) 4 V6 E) 8 V 3

Solución
BD = ?

* Plano R ± plano ABC

* Se traza DM 1 plano ABC

DM estará contenido en el plano R

DM 1 BM

* A ABC: BM = — = - ->

733
* A ADC, equilátero: DM = -^5-V3 = —V3
2 2
* Teorema de Pitágoras, en el A BMD: BD2 = BM2 + DM

BD2 = 4 2 + ( 4 V 3 ) BD = 8
Rpta: (A)

4) Dado un diedro R - EF - Q y un punto P interior, se trazan PA, PB y PC, perpendi­


culares a la cara R, la arista EF y a la cara Q, respectivamente. Si
PB PC
PA = Hallar la medida del diedro.
42
A) 75' B) 60 C) 45' D) 30° E) 90°

Solución

Sea: PA = PB = —
— PC = a
4z
.. PA = a, PB = 2a y PC = a V 2

- Como: EF 1 PA y EF 1 PC

EF X plano PAC

y PB estará contenido en dicho plano.

Se observa que sonnotables lostriángulos rectángulos PAB y PCB:


PB PB
mZPBA =30°, porque PA = — vm Z P B C = 45°, porque PC = — :
2 X 42
Entonces: diedro EF = m Z A B C = m Z P B A + mZPBC

diedro EF = 30° + 45° = 75°


Rpta: (A)

5) Se tiene un triángulo ABC, en el cual AB = 13, BC = 15 y AC = 14. Se eleva por


B, BF perpendicular al plano ABC, siendo BF = 12. Hallar la medida del triángulo diedro
que determinan los planos AFC y ABC.

A) 45° B) 30° C) 60° D) 37° E) 53°

7 3 4
Solución

En el plano ABC, se traza BH 1 AC-


Por el Teorema de las tres perpendiculares
FH 1 AC

FMB. es un ángulo plano del diedro


que determinan los planos AFC y ABC.

Teorema de Herón. en el

13+14+15
A ABC : p = ----------------- = 21

2 ^ , , 4

BH = — J 21(21 1 3 ) ( 2 1 —1 4 ) ( 2 1 — 1 5 ) = - . / 21(8)(7)(6)
14 7

Luego, el triángulo rectángulo FBM es isósceles (BF = 12 = BM)

Rpta: (A)

Dos caras de un triedro miden 84° y 114°, respectivamente. Hallar los valores enteros,
mínimo y máximo de la medida de la tercera cara.

A) 30° y 162° B) 31° y 161° C) 29° y 163°


D) 29° y 161° E) N. A.

Solución
Sea X, la medida de la tercera cara. Se deben cumplir:
X > 114° - 84° -> X > 30° ( I )

También:

X + 84° + 114° < 360° X < 162° ( II )

- De ( I ) y ( II ):
mínimo valor entero = 31°
máximo valor entero = 1 6 1 °
Rpta: (B)

Desde un punto “E” exterior a un plano H, se trazan EA, EB y EC* (A, B y C sobre

H). Si estos segmentos forman ángulos de 60°, 90° y 45°, respectivamente y AB ± BCi
hallar el área de la región triangular AEC, sabiendo que AB = 2 cm.

735
A) 2V 5cm B) 2>/6cm C) 6 cm D) 2 -/Í3 cm E) 2 V t5 cm

Solución
SAEC = 7

A ABE: BE = AB V3 = 2 -y/3

A EBC: BC = BE = 2^¡3

A ABC: notable, porque :

BC = AB>/3 ACB = 30°

y AC = 2 AB = 2 ( 2 ) = 4

BC 2 1/3
Al trazar BH X AC : BH = = V 3 (en A BHC)

Teorema de las tres perpendiculares: EH 1 AC

En el A EBH: EH2 = BE2 +BH2 = ( 2 ^ 3 f + (V3) f

Finalmente, SAEC = -(A C )(E H ) = - ( 4 ) ( v r 15) = 2 / T 5

Rpta: (E)
/
8) La figura muestra un cubo de arista “a". M es
A
punto medio de AB y N de BC.

Hallar la mínima distancia entre las rectas que


contienen a MN y CH

V2 V2
A) B) C) D) E)
4

Solución

Como M N / / A C . - . M N es paralelo al plano ACH


Toda recta perpendicular al plano ACH será tam
bién perpendicular a MN. Es particular. 1
plano ACH, por ser BD 1 AC y B B 'I AH- Si

BD intersecta a MN y AC ei> los puntos E y F,

respectivamente, entonces EF dá la mínima dis-

736
tancia entre MN y CH, que es la misma que hay entre MN y el plano ACH. En el
cuadrado ABCD, F es el centro:

BD
EF = BE = — -» EF = — = - ( B F ) EF = — = 1 (BD)
2 2 2 2v 2 , 4

Es decir:

Rpta: (B)

9) ABCDEF, es un hexágeno regular contenido en un plano P. Se eleva por A, AH X P,


de modo que los planos HBT y P formen un diedro de 45°. Si:
área ( A AHF ) = 8 cm2, hallar área ( A HBC )

A) 8 cm2 B) 16-s/2cm2 C) 16 cm D) 24 crrr E) 8>/2cm

Solución
H

Se traza A T 1 B C .\HT_LBC, según el


Teorema de las tres perpendiculares.
a

HTA = 45e (ángulo plano del ángulo diedro


que determinan los planos HBC y P.

BC HT
Se tienen: área ( A HBC ) =

AF HA
y área ( A AHF ) =

área(AHBC) HT
Dividiendo miembro a miembro, y como BC = AF :
área(AAHF) HA

HT
Pero : área ( A AHF ) = 8 cm2 y = ,¡ 2 -> área(AHBC) = 8 ^ 2 cm
HA
Rpta: (E)

10) Por el centro Q, de un cuadrado ABCD, se eleva QF perpendicular al plano que lo


contiene. Si AB = a, hallar la longitud de QF para que los planos BFC y CFD determinen
un diedro de medida 120°. (Diedro B - FC - D)

V2
A) a B) aV2 C) D) E) 2a

737
Solución
QF = x = ?

BHD es un ángulo plano del diedro B - FC - D


m Z BHD = 120°

m z B H Q = m Z D H Q = 60°

En el cuadrado ABCD:

BQ = QC

En el A BQH (notable):

a>/2 V3
QH = BQ = --------

Por relación métrica, en el triángulo rectángulo FQC:

1 1 1 1 1 1
+ — +* 2
2
(QC) (QH)2 X 'a h /T r aVe]
l 2 ; l 6 J
Rpta: (D)

11) O, es circuncentro de un triángulo isósceles ABC, ( AB = BC ). Se eleva, por O, OF

perpendicular al plano ABC. Si AB = 2 >¡7 ¡ AC = 4-^3 y AF forma con el plano ABC


un ángulo de 45°, hallar la medida del diedro que determinan los planos AFC y ABC.

A) 60° B) 64,5° C) 75° D) 45° E) 71,5°

Solución
X = ? {mZOHF}

- O -> circuncentro del A ABC


OA == OB = R -» circunradio.

- En A AHB: BH2 = AB2 - A H 2

BH2 = ( 2 V 7 f - ( 2 V3 ) 2
BH = 4
- En A AHO: HO = BH - R - * HO = 4 - R

Teor. de Pitágoras: AH2 + H Q 2 = AQ2

( 2 V 2 ) + ( 4 - R ) 2 =R —>

7 3 8
Entonces: OH = 4 - R y como OF = OA, por ser el A AOF
isósceles:

37
Finalmente, en el A HOF: a = ------- = 18,5°

.-. x = 9 0 ° - 1 8 , 5o
Rpta: (E)

12) Se tiene un hexágeno regular ABCDEF, contenido en un piano P. Por el centro “O” del
hexágeno, se eleva OH _L P. Si AB = 4 y OH = 2 ^ 6 , hallar la mínima distancia entre
AB y HE.

A) 4 B) 4V 3 C) 4 V6 D) 4 ^ 5 E) 4 V 2

Solución

Sea T, un plano perpendicular a AB-


Se hacen las proyecciones sobre T.
H'1 H
Todas las perpendiculares a AB, se
proyectan en verdadera magnitud.
Por eso:
H'O1 = HO = 2 V 6 y
A’ / 7a
A'E' = AE = 4>/3
Como AB se proyecta según el
punto A' la figura 2, dá la forma de
hallar la mínima distancia “x” entre
AB y HE.

Se observa: E'O' = O’A' = 2^3

A E'O'M': E’H’ = 6 (Teor. Pitágoras)

4V3
- A E'RA' ~ A E'O'M’:
2^6

• ■

4 V3
Rpta: (E)

13) Se tiene un triángulo rectángulo ABC, recto en B. Por el vértice A, se eleva AE


perpendicular al plano ABC. Si AE = 8 y BC = 6, hallar la mínima distancia entre AB y EC

A) 10 B) 4,8 C) 5 D) 4,5 E) 6
739
Solución

- Se proyecta la figura sobre un plano R,


perpendicular a AB.

- AB se proyecta como un punto (A'B')


y AE así como BC, en verdadera
magnitud por ser perpendiculares a
ÁB-
- El segmento de longitud x, dá la míni­
ma distancia entre AB y EC .

Por relación métrica, en el triángulo


rectángulo contenido en el plano R:
1 1 1
—— = + » x = 4,8

Rpta: (B)

14. En una circunferencia de centro 0, radio 2 cm se tiene la cuerda AB, de modo que
mAB = 45°. Se eleva O E, perpendicular al plano de la circunferencia, siendo OE = 2
cm.
Hallar la mínima distancia entre AE y OB.

V3
A) - V 3 B) C) - V 2 D) E)
3

Solución
Se proyecta el conjunto sobre un plano R
perpendicular a OB. Este segmento se
proyecta como un punto (O1) y todas las
perpendiculares a él, en verdadera mag­
nitud. Así: O'E’ = OE = 2. Si AH 1 OB.
entonces en R: 0 ‘A' = AH.

Pero, en el A AHO : AH = = V2
V2

0 ' A ' = V2

La distancia de O’ a A‘E‘ dá la mínima


distancia entre AE y OB : x .

7 4 0
1 1 1
+ (en A A 'O 'E 1)
Por relación métrica: x2 22 (V2)

De donde:

Rpta: (A)

15) La arista del cubo adjunto, mide “A". El


punto O es centro de la cara ABCD.
Hallar la mínima distancia entre GO y FD

a
A) B) - V 3 C)
3 B
7
a / \o
és
D) - V 2 E) - V 2 *
2

Solución
Si se proyecta el conjunto sobre un plano
perpendicular a FD , esta recta se proyec­
H* E’ tará como un punto (F' D’) y la distancia de
este punto a G ' 0 \ (proyección de G O ) ,
dará la mínima distancia pedido, (x), según
el gráfico adjunto.
En el triángulo rectángulo del cual x es
altura:

1 1 1 .... (Relación Métrica)


+
2 2 \

x a

De donde: Rpta: (B)

16) El tetraedro O - ABC, es trirrectángulo en el vértice O


OQ 1 plano ABC.

Demostrar, que :

741
V

Solución

OQ debe estar en un plano perpendicular al B


ABC.
Veamos:
Se traza OR 1 AC. Por el Teorema de las

tres perpendiculares: BR _L A C . El plano BOR


es perpendicular al plano ABC. En BCR estará
OQ _L plano ABC.

Por relación métrica, en los triángulos rectán


gulos:

1 1 1
A BOR + ----- ( I )
(OQ)Z (OR)2 (OB)2

1 1 1
A AOC + ----- ( II )
(OR)2 (OA)2 (OC)

( II ) en ( I ):
1 1 1 1
+ +
I.q.q.d
(OQ)2 (OA)2 (OB)2 (O C )2

17) AB y CD son segmentos alabeados y perpendiculares. AC es perpendicular a AB y CD.

Si: AB + CD = 256, hallar la distancia entre los puntos medios de AC y BD.

A) 16 B) 8 C) 4 D) 12 E) 14

Solución

AB2 + CD2 = 256 (dato).

* Sean: M, punto medio de AC y N, punto


medio de BD Incógnita: MN.

* Si H es punto medio de AD
AB
NH = y NH II AB, en ABAD

CD
MH = y MH II CD, en AACD

742
* Por dato: AB 1 CD MH 1 NH

* En el triángulo rectángulo NHM, teorema de Pitagoras:


2

<
CD
O

Q
2 AB2 + CD2
MN2 = MH2 + NH2 MN = + = ■■ ■ "■■■ ■
, 2 , ^ 2 ) 4

m m 2 - 256
Con el dato: MN -

Rpta: (B)

18) m y r son dos rectas alabeadas que se cruzan con un ángulo de 60°. A e m, B e r
y AB = 4 cm es la mínima distancia entre m y r. Sobre m se toma el punto E y en
r el punto F, de modo que AE = BF = 3 cm. Hallar la medida del ángulo con que se
cruzan AB y EF.

A) 60° B) 30' C) 45° D) 37° E) 53°

Solución

Grafiquemos r en un plano S paralelo a


m. m' es la proyección de m sobre S.

El plano R 1 plano S.

Se observa:

A BFHf equilátero:

( I )

X, dá la medida del ángulo de cruce

entre AB y EF , ya que EH II AB.

Como EH = AB —» ( II )

Con ( I ) y ( II ), en A EHF, recto en H:

Rpta: (D)

19) La mínima distancia entre dos aristas opuestas de un tetraedro regular, de arista con
longitud “L”, es:

L.V3 LV6
A) L B) - V 2 C) D) Í V 6 E)

7 4 3
Solución

C onsiderem os el T etraedro regular ABCD.


Hallemos la mínima distancia entre AC y BD-

Si OE 1 AC AE = EC y luego BE 1 AC
El plano BED es perpendicular a AC. Enton­
ces, si EF i BD, EF dá la mínima distancia
entre AC y BD, por ser EF perpendicular a
ambas rectas.

L r- L
Así: DE = EB = - ^ 3 yDF = FB = -
2 2
En el A EFB, Teorema de Pitágoras:
2 2
EF2 + FB2 = EB2 -> ÉF2 + ' L '- 7 3 ' —>
2) ^2 , 2
Rpta: (B)

20) En un triedro H - ABC, se sabe que: m AHC = 90°, m AHC = m BHC = 60°. Hallar

la medida del triángulo que forma HB con el plano AHC.


A) 45 B) 30° C) 37° D) 60° E) 53°

Solución H
Se traza BQ 1 plano AHC.

HQ es la proyección de HB en
dicho plano.

Incógnita : m ZBHQ = ?

Se trazan QE 1 HA y QF 1 HC

Sea: HB = ( . Luego:

A HFB -> HF = — ahfb -> HE = -


2 2

EHFQ es un cuadrado. Luego: HQ = HF V2 HQ = - > /2


t 2

Entonces, en el A HQB, recto en Q:

m Z BHQ = 45' Rpta: (A)


744
21) En un triedro H - ABC, el diedro C mide 90° y las caras: a = b = 45°. ¿Cuánto mide
la cara c?
A) 45' B) 90' C) 60' D) 30° E) 75'

Solución
a = ?

Sea F, un punto de HC
Se trazan: FE 1 HC y FP 1 HC

estando E en HA y P en HB
*

EFP es un ángulo plano del diedro C

m Z E F P = 90°
Si HF = I , entonces:

A HFP FP = i. y HP = f j 2 FP = FE en el

A HFE —> FE = t y HE = l'4 2 A EFP: EP = >42

En conclusión: EP = HP = HE = <42

A EHP, equilátero

Rpta: (C)

22) Dado un triedro O - ABC, la arista OC forma con la bisectriz qe de ía cara opuesta
un ángulo que mide la mitad de dicha cara. Demostrar que:

Soiucion

Recordemos, según el Teorema n


- 107 de este capítulo, que Hsi dos
caras de un triedro son congruen­
tes, entonces los diedros opuestos
a dichas caras son congruentes”.
Luego:

Triedro O - AEC:
cara EOC = cara AOE diedro E - OC - A = diedro E - OA - C ( i)

Triedro O - EBC:
cara BOE = cara EOC diedro E - OC - B = diedro E - OB - C (H )

7 4 5
Sumando miembro a miembro, lo de ( I ) y ( II ):
diedro E - OC • A + diedro E - OC - B = diedro E - OA - C + diedro E - OB • C
\
Es decir:

23) La figura, muestra un tetraedro OABC, en el


cual el triedro O es trirrectángulo. Se dis­
tinguen: OAB, OAC, OBC caras cateto
ABC -> cara hipotenusa.
Demostrar que: “El área de cada cara cateto
es media proporcional entre los áreas de la
cara hipotenusa y de su proyección sobre di­
cha cara hipotenusa”.

Solución

S ea OH 1 B C . E n to n c e s : AH ± BC
(Teor. de las, tres perpendiculares).

Plano AOH -L plano ABC. Luego, si


OR _L plano ABC, OR estará conteni­
do en el plano AOH.

ARB, BRC y ARC, son proyecciones de


las caras AOB, AOC y BOC, respectiva­
mente, sobre la cara ABC. Luego:

^BRC (^RDT.)
BOC COS CX
Pero: ^BOC ~ (^ARr.)
ABC COS CC

Dividiendo miembro a miembro, se cancela cos a : BRC _ BOC

BOC ABC

De donde: l.q.q.d

24) Para el anterior problema, demostrar que: “La suma de los cuadrados de las áreas de
las caras catetos, es igual al área de la cara hipotenusa”.

Solución

El problema anterior, se demostró: (SB0C)2 = (S abc) ( S b rc )

Análogamente, se puede escribir:

(SA0B)2 = (SABC) ( S ARB) y ( s A0C)2 = (SABC) (SARC)


7 4 6
V

Sumando miembro a miembro estas tres relaciones y factorizando luego SABC, en


el segundo miembro:
(S bqc)2 + ( S a o b )2 + (SAOC)2 = SABC
„ (Snnr + S4
'~ B R C BH +' 'S4Br)
ARB “'A R C
N_____________ /
^ABC
De donde:
l.q.q.d.

—^ ^
25) Se tienen los rayos AX y BY que se cruzan formando un ángulo que mide 60° y cuya
perpendicular común es A B . Sobre AX se ubica el punto “P” y sobre BY el punto “Q”; tal que
AP = AB = BQ = 4cm. Calcular PQ.

A) 4V2 cm B) 4V3 cm C) 4 / 7 cm D) 8 cm E) 8V2 cm

Solución
Incógnita: PQ
—> —>
Trazamos BZ // AX

Luego: m ZBY = 60

P R //A B y AB 1 Plano ZBY

=> PR _L Plano ZBY


[ \ PRQ:
PQ = 4V 2

26) Se tienen los rayos AX y BY que se cruzan formando un ángulo recto y cuya perpendicular
—»
común es A B . Sobre AX se ubica el punto “C” y sobre BY el punto “D”, tal que:
AC2 + BD2 + AD2 + BC2 = 18cm2. Calcular CD.

A) 2 cm. B) V2 cm C) V3 cm D) 2V 3 cm E) 3 cm

Solución

Incógnita: CD = m

* a2 + b2 + c2 + d2 = 18

- Trazamos É Í H Ñk

=* Z ZBY = 90°

BD ! □ H => BD i BC

£, DBC : m2 = b2 + d2 . (1)
BZ - ! □ ABD y

Á X //B ^
7 4 7
Cs DAC : m2 = a2 + c2 .................... ( 2 )

( 1 ) + ( 2 ):

2m2 = a2 + b2 + c2 + d2

m = 3 Rpta

27) En el interior de un triedro trirectángulo O-ABC, se toma un punto P y se trazan: P E , PH y


PF, perpendiculares a las caras, con longitudes: PE = a, PH = b y PF = c. Entonces:

a+b+c
A) OP = B) OP = J a 2 + b2 + c 2 C) OP = ^ a b c

D) OP = 2 V a2 + b 2 + c 2 E) N.A

Solución

Por H, F y E, trazamos perpendiculares a las a


ristas del triedro.

Trazamos O E ; con el Teorema de Pitágoras, teñe


mos:

AO EP -> OP = a2 + OE (1 )
A ORE:

OE = OR + ER

OE = c2 + b2 (2 )

- Reemplazando ( 2 ), en ( 1 ): OP = a2 + c2 + b2

de donde: (fórmula)

Rpta: (B)

28) En la región interior a un triedro triregtángulo de vértice O, se toma el punto P. Hallar OP;
sabiendo que las proyecciones de OP sobre las caras del triedro, tienen longitudes 3 ^¡2 ,

4yÍ2 y 1 2 V 2 cm., respectivamente.

A) 13 B) 17 C) 15 D) 12 E) 18

Solución
Consideremos el gráfico adjunto, donde el triedro es O-ABC
Incógnita: OP = x.

7 4 8
Con las distancias elegidas a, b y c; usando
el Teorema de Pitágoras:

x2 = a2 + b2 + c2 ..................( 1 )

AO M E -> OM + EM = OE

b2 + a2 = 288 (2)

A OMH -4 OM + MH = OH
b2 + c2 = 1 8 ....................... ( 3 )

AO R F OR + RF = OF
c2 + a2 = 3 2 ................ (4 )

Sumando miembro a miembro, las expresiones ( 2 ), ( 3 ) y ( 4 ):

2a2 + 2b2 + 2c2 = 338

a2 + b 2 + c2 = 169

- Finalmente, reemplazando en ( 1 ) : x = 169 2 x = 13


Rpta: ( A )

29) Hallar la longitud del menor camino para ir del punto A al B, tocando un punto P de la recta m
A) 14 B) 10 C) 12 D) 11 E) 9
Solución
A?

tB
- Se ubica el punto A ’, simétrico de A, respecto a m .
Íi 2
m Como A’P = AP, entonces A 'B dá la longitud
h—
8
pedida. Enseguida se traza A ’R i l m .
En el A A’RB, con el Teor. de Pitágoras:

( A ’B ) 2 = ( A’R )2 + ( RB )2

( A’B Y = 82 + 62

A ’B = 10 .-. AP + PB = 10

Rpta: ( B )
A’ 8 R
30) En la cara ABCD, de una caja rectangular de vidrio, interiormente, en el punto P, se encuentra
una hormiga que quiere alcanzar un trozo de pan en el punto Q de la cara ADEH. Hallar la
longitud de la menor trayectoria que debe seguir la hormiga, cruzando AD, hasta alcanzar
su comida.
749
A) 12 B) 15 C) 4 ^ 5 D) 10 E) 13

Solución

- Colocando la cara ABCD en el mismo plano


que ADEH; al trazar PQ, ubicamos el punto
R que debe tocar en AD la hormiga para la
trayectoria PRQ mínima.

En el A PSQ : PQ = PS + SQ
PQ2 = 122 + 52
-> PQ = 13

es la rpta: (E)

31) Hallar la longitud del menor camino para ir del punto P al punto Q, tocando primero un punto
E de AB y luego un punto F de AD, para ir finalmente a Q.

A) 8 B) 10 C) 12 D) 11 E) 15

Solución
B £
- Se grafican: f Í5

P’ -» simétrico de P, respecto a AB t
2,75
Q* simétrico de Q, respecto a AD r
D
P 'Q ’ determ inalospuntosE yFen AB y AD,
respectivamente.

Entonces : EP’ = EP y FQ’ = FQ

PE + EF + FQ = P’Q’, es la incógnita

- En el A P 'C Q ': ( P’Q’ )2 = ( P’C )2 + ( Q’C )2

( P’Q’ )2 = 82 + 62

P’Q’ = 10

PE + EF + FQ = 10

Rpta

32) En el ángulo POQ de 60° se traza la bisectriz ó k y se consideran los puntos A y B en la


bisectriz siendo OA = 6 y O B = 10. Hallar el menor camino para ir de A hasta B pasando
—► —)
primero por OP, luego por OQ y finalmente se liega al punto B.

750
16 B) 12 C) 14 D) 32 E) 18

Solución
Se ubican:
—>
A’ -» simétrico de A, respecto a OP.
—>
B’ -> simétrico de B, respecto a OQ.

Se unen A’ y B’:
AM = A ’M y BN = B’N

La longitud de la trayectoria AMNB dá la respuesta


al problema y es igual a la distancia A’B \

Con el A A ’O B \ podemos hallar A’B’. Para ello,


trazamos la altura A 'H l O B '. Siendo:
K

A'H = — V3 -> A'H = 3>/3 ;OH = — -> OH = 3


2 2

VA A A’HB' -» ( A’B' )2 = ( A’H )2 + ( B’H )2 =

= (3 ^ 3 f + W 2 = 196
A’B’ = 14
Entonces: AM + MN + NB = 14
Rpta: ( C )

Por el centro O, de la circuferencia inscrita a un trapezoide AEDC, se ha elevado la


perpendicular OB, al plano del cuadrilátero (verfigura). Luego se trazan B A ,B E ,B D y BC.
Demostrar, que:

A BE + C B D = ABC + EBD

Solución

Trazamos los radios a los puntos de tangencia:


OP 1 AE , OH 1 E D . Luego, por el teorema
de las tres perpendiculares:
BP 1 ÁE y BH 1 ED

A BOP 2 A BOH, por tener común cateto BO


y OP = OH

BP = BH ..................... (1 )

A BPE = A BHE, por (1) y además EP = EH.

Entonces: PBE = EBH ..................... (2 )

751
- Análogamente, se prueban que:

PBA = ABT (3 )

NBC = TBC (4 )

NBD = DBH (5 )

Así, sumando miembro a miembro ( 2 ) , ( 3 ) ;


(4) y (5):

PBE + PBA + NBC + NBD = EBH + ABT + TBC + DBH


t 1-----------?------------1

De donde: ABE CBD = EBD + ABC

l.q.q.d.

34) Por el centro O, de la circunferencia circunscrita a un cuadrilátero ABCD, se eleva la


perpendicular OS al plano que lo contiene. Se trazan, luego: S A , S B , SC y S D . Demostrar,
que:

Solución

- Se trazan los radios OA, OB, OC y OD.

- Los triedros A -OBS y B-OAS, tienen sus caras:


O Á S = OBS, S Á B = SBA y O Á B = OBA, ya
que los triángulos rectángulos AOS y BOS son
congruentes y los triángulos SAB y AOB son
isósceles. Entonces, los triedros en mención (A-
OBS y B-OAS), son congruentes.

Luego; llamando A 1 y B1 los diedros de dichos


triedros, con aristas AS y BS, respectivamente:
A, = B,

En forma análoga, se demuestra: A2 = ü0 , C„ = D~ y C. = B4. Efectuando la suma miembro


a miembro:

A 1 + A2 + Cg + C4 B! + D2 + D3 + B4

Es d e c ir:
t
t í
i
diedro SA + diedro SC = diedro SB + diedro SD

iqqd

752
/

CAPITULO 2 5

POLIEDROS
SOLIDO.- Un sólido, es una figura que encierra una región del espacio mediante superficies.

POLIEDRO.-Un poliedro, es un sólido formado por polígonos, que constituyen las caras. Los
vértices del sólido son los de sus caras y las aristas del poliedro son los lados de los
polígonos.

Los poliedros pueden ser convexos y no convexos o cóncavos.


B C * Caras: BGC , FGE , ABGF ,

* Vértices: A, B, C , ....

* Aristas: AB, B C ,....

* Diagonales: BE, F C ,....


(Fig. 1) (unen dos vértices ubicados en diferentes caras)

POLIEDRO CONVEXO.- Un poliedro se llama convexo, si determina sobre una recta secante
a su superficie, como máximo, dos puntos de intersección.

POLIEDRO NO CONVEXO O CONCAVO.- Es aquel que determina sobre una recta secante
más de dos puntos de intersección.

El nombre del poliedro depende del


número de caras y puede ser: tetraedro
(4 caras); p e n ta e d ro (5 caras); (Fig: 2)
hexaedro (6 caras), etc. Por ejemplo,
la figura 1 muestra un hexaedro con­
vexo y la figura 2, un decaedro no
convexo.

Propiedades.- Si “V” , “C” y “A”, representan los números de vértices, caras y aristas de un
poliedro:

1o) Las medidas de los ángulos, en todas la caras, suman: 360° ( V - 2 ).

753
2o) La suma del número de caras y vértices, excede en dos al total de aristas:
(Teorema de Euler).

Por ejemplo, para la figura 1, anterior: C = 6 ;V = 7 y A = 11. Se verifica que:

C+V = A+2=13 ( 2o propiedad )

Para la propiedad N°1: suma de medidas de los ángulos en los 2 triángulos y ios 4
cuadriláteros:

2 x 180° + 4 x 360° = 1800°; lo cual verifica:

360°( V - 2 ) = 360°( 7 - 2 ) = 360° x 5 = 1800°

POLIEDROS REGULARES.- Son aquellos que tienen por caras, polígonos regulares. Se
demuestra que sólo existen 5 poliedros regulares: tres formados por triángulos equiláteros;
1 por cuadrados y otro por pentágonos regulares.

POLIEDRO Forma de
C V A
REGULAR las caras

Tetraedro Triángulos 4 «e 4 6
Equiláteros

Octaedro Triángulos
co

6 12
Equiláteros /*
\
Hexaedro Cuadrados 6 8 12

Dodecaedro Pentágonos 12 20 30
\ /
Icosaedro Triángulos \
20 12 30
Equiláteros

Nota.- Se llaman conjugados, aquellos poliedros regulares en los que el número de caras
de uno, es igual al número de vértices del otro. (Uniendo los centros de las caras de uno, se
obtiene el otro). Son conjugados: Octaedro y Hexaedro; Icosaedro y Dodecaedro.
754
PROBLEMAS RESUELTOS

1. Indicar verdadera (V) o falso (F)

p) Sólo existen 5 poliedros regulares.


q) Si se unen los centros de las caras de un dodecaedro regular, se obtiene otro dodecaedro
r) El conjugado de un octaedro regular, es un cubo.

A) FFV B) VVF C) VFV D) FVF E) VVV

Solución

p) Verdadera, por teoría. (V)


q) Se obtiene un icosaedro regular.
La proposición dada es falsa. (F)
r) Teoría: (V)

Rpta. C

2. El área de la superficie de un icosaedro regular, de arista “a”, es:

A) a2V3 B) 20a2V3 C) 5a2>/3 D) 4a2V3 E) 5aJ

Solución

Son 20 triángulos equiláteros.

S = 20 a 2 — ' = 5a2 V3
\ 4 /

Rpta. C

3. ¿Cuánto suman las medidas de los ángulos en todas las caras de un dodecaedro regular?

A) 5 400° B) 5 480° C) 6 400° D) 6 800° E) 6 480°

Solución

El dodecaedro regular está formado por 12 pentángonos regulares. La suma de las medidas
de los ángulos en cada cara, es 180°(S - 2) = 540°. Como son 12, entonces:
12. 540° = 6 480°.

Rpta. E

7 5 5
4. El área de la superficie total de un cubo, es igual al cuadrado de la longitud de su diagonal,
por:

A) 1 B) 3 C) 2 D) 6 E) 4

Solución

* Se sabe, para un cubo de arista UX” y diagonal “d”

V3 = d V3

* S.....
total
= 6 □ = 6x2 = 6
\ V3,

Stótal = 2d2

Rpta. C

5. La figura muestra un cubo PQRS, en un cuadrilátero que tiene sus vértices en cuatro aristas
del cubo.

Demostrar, que: PA + RG = QH + SF

Solución

PQ y RS son paralelos, por estar contenidos en el mismo plano y no intersectarse.


Análogamente QR y ps son paralelas. Por lo tanto, PQRS es un paralelogramo.

OM es mediana de los trapecios PAGR y QHFS. Así:

„ . PA + RG ... QH -f SF
QM = ------------- y OM = ---------------

PA + RG QH + SF
Entonces:

756
6. Un poliedro convexo está formado por 8 triángulos, 2 pentángonos y 5 hexágonos. Hallar
el número de vértices.

A) 64 B) 30 C) 32 D) 16 E) 36

Solución

* Las medidas de los ángulos en todas las caras, suman

8 triángulos * 8(180°) = 1 440

2 pentágonos -+ 2 (540°) = 1 080 (+)


5 hexágonos -> 5 (720°) = 3 600° j

V ¿
Es decir: caras = 6 120

Fero, por fórmula: 360° (V - 2) =6120°

De donde: V = 19

7. Un poliedro está formado por 6 triángulos. 4 pentágonos y 7 cuadriláteros convexos. Hallar


el número de aristas.

A) 17 B) 66 C) 22 D) 35 E) 33

Solución

Cada arista es un lado para dos polígonos, en caras adyacentes. Por lo tanto, si sumamos
los números de lados de los 6 triángulos, 4 pentágonos y 7 cuadriláteros, tendremos el doble
del número de aristas. Así:

2A = 6 . 3 + 4 . 5 + 7 . 4

A = 33

Rpta. E

8. Para el poliedro del problema anterior, los números de caras y vértices, son respectivamente:

A) 17; 17 B) 18; 17 C) 17; 18 D) 18; 18 E) N.A.

Solución

- El número de caras, es igual al número de regiones poligonales que forman el poliedro


(6 triángulos, 4 pentágonos y 7 cuadriláteros) C = 6+4+7

757
Con el teorema de Euler:

C+V =A+2 -» 17 + V = 33 + 2

Rpta. C

9. En un octaedro regular, de arista “a”, hallar la distancia del centro a una cara.

A) B )|V 6 C )f E) N.A

Solución
OF = ?
Las diagonales de un octaedro regular son congruentes.
(Esto es fácil demostrar). B

AO = OB =

OM =

1 1 1
En AMOB: + 2
(OF)" (OB)2 (OM)

1
(OF f a V2
V2 ;
Rpta. B

10. Hallar la distancia entre los baricentros de dos caras de un tetraedro regular, de arista “a”

a a 2
C )-a D) 4 a E) N.A
A> 3 B> 2

Solución B

Sea ABCD, el tetraedro.

P, baricentro de ABD
Q, baricentro de DBC

A PBQ - A MBN :

PQ BP
MN “ BM o)

758
%

AP a RP 9
- Siendo: MN = — = - (E n A ABC) y — = - { Baricentro}
2 2 d M 3

- En ( I ): PQ = —
a/2 3 3

Rpta. A

11 Hallar el área de la proyección de un octaedro regular sobre un plano perpendicular a una


arista. Cada arista del octaedro mide “a”.

A ) íiI B ,íi# C ) ^ D)a- E) N.A

Solución

* Sea R. un plano perpendicular a la arista AF, AD y FC se proyectan en verdadera


magnitud

MZNQ. es un rombo.

AC V2
ZH = BO = AO =

MN . ZQ . aV2 a2 V2
El área: S MZNQ

Rpta. A

12. Un poliedro convexo está formado por 4 triángulos y 5 cuadriláteros. Hallar el número de
diagonales de este sólido.

A) 36 B) 26 C) 16 D) 20 E) 10

7 5 9
Solución

Si “V”, es el número de vértices, el número de diagonales del sólido, es

#DIAG = C% - A - # DIAG (I)

SOLIDO CARAS

Donde:

Co = combinatoria de todos los vértices, de dos en dos.

V!
C2
V=
2! (V - 2)! (II)

# aristas 4 A + # aristas5 □
A = número de aristas =

A = 4-~- + 5 ' 4 = 16-> A ^ 16


2

(Se divide por 2, ya que cada arista se cuenta dos veces)

De otro lado:

# DIAG. = # Diag. de 5 □ = 2 . 5 = 10 (IV )

CARAS

El número de vértices “V” se obtiene con el teorema del Euler:

C+V = A+2

9 + V = 16 + 2

9! 9 .8
* En ( I I ) : c2= = 36
2! 7!

Un posible gráfico
En ( I ):

# DIAG. = 3 6 - 1 6 - 1 0

SÓLIDO

# DIAG. = 10

SÓLIDO
Rpta. E

760
13 Hallar la distancia entre los baricentros de dos caras adyacentes de un octaedro regular de
arista “a”.

a
B )f/2 C |V 2 E) N.A
A> 2

Solución B

Sean : G Baricentro de ABD

R Baricentro de BCD

A MBN - A GBR :

GR _ BG
MN ” BM ( i)

AC aV2
Siendo: MN =

BG 2
v ------ = — { Propiedad del Baricentro}
y BM 3 1 ;

En ( I ):

Rpta. C

14. ¿Cuántas diagonales tiene un icosaedro regular?

A) 66 B) 30 C) 24 D) 32 E) 36

Solución

Se hacen las combinaciones de todos los vértices, de dos en dos, y a ésta cantidad se restan
el número de aristas y el número de diagonales en las caras:

# DIAGONALES = C1
22- 30 - 0
ICOSAEDRO i
# diagonales
en las caras

12!
# DIAGONALES - 30
( 2 !) (12 - 2 )!
ICOSAEDRO

# DIAGONALES = 66 - 30 = 36
ICOSAEDRO
Rpta. E
7 6 1
15 ¿Cuántas diagonales tiene un dodecaedro regular?

A) 160 B ) 190 C )1 0 0 D ) 200 E) 250

Solución

# DIAG. = C f * # aristas - #diag. de las caras


DODECAEDRO

20 !
# DIAG. - 3 0 - 12(5)
2! (18!)
DODEC

# DIAG. = 100
DODECAEDRO

(20 vértices)

Rpta. C

16 En un octaedro regular, de arista “a”, hallar la distancia del centro de una cara, al centro del
poliedro.

a a
B) | V 6 C) | V 2 E) N.A.
A> ?

Solución

Sea el octaedro de la figura. 0, su centro y G el de la cara BCD

OG = ?

En el A BCD (equilátero), se tiene:

BG = f (BM) = | *V 3 =^V 3
,2 i 3

GM = -^-(BG) = {G es Baricentro}

AD a
Además se observa: OM =

BE V2
y se sabe: BO =

762
En el A BOM, Teor. de Stewart.

OG2 ( BM ) = BO2 . GM + OM2 BG - BG . GM. BM

/ \ / \
= ^ a-v/íP • aV3 1 fa
OG2 £2^ 3 . - V 6 . ^
3 3 6 2

C\J
l 3 v 2 , 6

De donde:

Rpta. B
* Nota.- Se verifica que OG 1 plano BCD

Hallar el área de la superficie de un dodecaedro regular de arista a .

3 2
A) T a ( 3 j5 + 5 B) - | a 2 (3V5 + 5 C) + 5

D) - y ( 3 V5 + 5 E) N.A.

Solución

Son 12 pentágonos regulares, de lado “a”. Se sabe, que para un pentánono regular

= — ^ 1 0 - V2 0

.-. a = - J 10 - V 2 0 R= , 2a —
2 V V i O - V2Ó

R a (V5 + 1)
Y, el apotema: ap. = — v5 + 1 = — ,
4 2 V 1 0 -V 2 Ó

Luego, el área de una cara:

S, = 2 { ( s ) ( ap5 )

\
5 2a (V5 + 1
S, = 2 ^10 - >/2Ó ; [ 2^/10 - V 20 ;
V

763
a2
S, = T (3 ^ + 5

3
S DODECAEDRO = 12 S = —a2
1^ ° 1 2 ■
4- 5)
'

Rpta. C

18. Hallar el área de la proyección de un tetraedro regular, de arista “L”, sobre un plano
perpendicular a una arista.

A) L2 B )L 2 V2

U
D)

Solución

* Sea ABCD el tetraedro y R un plano perpendicular a la arista CD

CD se proyecta como un punto y por ser AB 1 CD , AB se proyectará en verdadera


magnitud, (todas las perpendiculares a C D , se proyectan en verdadera magnitud).

* Lueqo: Sproyección = —
2
.( A 'C ‘) .h (I)

* Siendo: A'C’ = AM = ^ V 3 OD

y h = BO = altura del tetraedro.

Ve
h= L (n i)

Con ( I I ) y ( I I I ), en ( I ):

764
1 L Ve
s = 1 . ^ 4 3 .i
proyección 2 2 6

proyección
V2

Rpta. E

En cubo de arista “L”, hallar el área de la proyección obtenida sobre un plano perpendicular
a una diagonal.

A) L2 V~2 B) L2 / 3 C) 2 L2 V ^

D) 2 L2 V 5 E) L2 V~3 n

Solución

Proyectamos el cubo sobre un plano T, perpen­


dicular a la diagonal FC. Todas las perpendicu­
lares a FC , serán paralelas a T y se proyectarán
en verdadera magnitud. La proyección resulta la
región correspondiente a un exágono regular, de
V\ "‘V
\ ' y\ LD
lado “x”. Además, “x” es la distancia de D a C F . \» FA3\
En el ACDF: k
✓ ¿A.\• -JE
DP = x ; DF = LV2

Por relación métrica:

1 1 1
+ ----- 1 1 1
(DP)2 (CD)2 (DF)2 +
(LV 2)

2 i2
De donde: x2 =

Y, el área de la proyección:

S = - x2 V3 = - V3 . S = L2 V3
2 2 /

Rpta. B

7 6 5
20) Hallar el número de segmentos que unen los puntos medios de todas las aristas de un
icosaedro regular.

A) 345 B) 435 C) 543 D) 534 E) Ning. anterior.

Solución

- El icosaedro regular tiene 30 aristas. Luego, hay que combinar 30 puntos medios, de dos en
dos:

c 2° = •„,? !? '. ^ - 435 segmentos


2!( 2 8 !)

Rpta: ( B )

Uno de los primeros capítulos de la "GEOMETRIA DESCRIPTIVA"(CURSO UNIVERSITA­


RIO DE INGENIERIA), consiste en obtener las representaciones de proyecciones de sólidos
en planos dados.

Por Ejemplo, en la siguiente figura se muestran las proyecciones del sólido prismático sobre
los planos H (plano horizontal) y F (plano frontal), perpendiculares entre sí.

Obtener la proyección en el plano P (plano de perfil).

El problema inverso es más interesante: dadas las proyecciones, obtener el sólido. Por
ejemplo, en cada uno de los siguientes casos, se tienen las proyecciones en H y F de algún
sólido. Hallarlos.

7 6 6
a)

b)

PROBLEMAS PROPUESTOS

1) Se da un cubo de arista a. Tomando como referencia un vértice, construya un tetraedro


regular uniendo los vértices no adyacentes con rectas contenidas en las caras.

Construya un octaedro regular uniendo los centros de cada cara del cubo. La razón entre el
área total del tetraedro y el área total del octaedro, es :

A) 1/3 B) 1/2 C )1 D) 2 E) 3

2) Hallar el área total de un tetraedro regular, siendo la suma de las longitudes de sus aristas
36 cm.

A )3 6 c m 2 B )6-v/3cm 2 C) 24 cm2 D )3 6 > /3 c m 2 E )2 4 V 3 c m 2

3) Al unir los puntos medios de las aristas de un dodecaedro regular, se obtiene un poliedro
formado por:

A) 12 pentágonos B) 12 triángulos y C) 12 triángulos y


20 pentágonos 12 pentágonos

D) 20 triángulos y E) Ning. anterior.


12 pentágonos

4. Uniendo los puntos medios de las aristas de un icosaedro regular, se obtiene un poliedro
formado por:

A) 20 triángulos B) 20 triángulos y C) 12 pentágonos y


12 pentágonos 12 triángulos

D) 12 triángulos E) Ninguna anterior.


20 pentágonos

5) Un poliedro convexo está formado por2 triángulos, 3 cuadriláteros y “x” polígonos de 11 lados
cada uno.
7 6 7
Hallar el valor mínimo de V .
A) 1 B) 2 C) 3 D) 4 E) 5

6) Hallar la relación de áreas de un octaedro regular y un tetraedro regular, sabiendo que la


diagonal del octaedro es igual a la altura del tetraedro.

A) — B) -1 C) — D) — E) Ning. anterior.
3 9 3 2

7) Las aristas de un cubo miden 15cm. cada una Si una mosca puede desplazarse sólo sobre
las aristas, y parte de uno de los vértices; el máximo recorrido que puede hacer al volver a
su punto de partida, sin pasar dos veces por la misma arista, es:

A) 1,80m. B) 0,60m. C) 0,75m. D) 0,90m. E) 1,20m.

8) La arista de un octaedro regular mide “ a Hallar el área de la sección determinada al


interceptar la superficie del poliedro con un plano paralelo a una cara, que pasa por el punto
medio de una arista.

A) 3a2V3 B) - | a 2 -J3 C) ^ a 2 ^ 3 D) | a 2 & E) Ning. anterior.

9) En un cubo de arista Ha”, el área de la sección determinada al intersectar su superficie


mediante un plano perpendicular a una diagonal, en su punto medio, es:

2 o

A) a2 B ) — >/3 C) 3a2 V¡3 D) —a2 V3 E) Ning. anterior.


4 4

10) Uniendo tres vértices de un cubo, se obtiene un tetraedro regular.


El área del tetraedro es al área del cubo, como:

A) 2 :V¡3 B) V3 : 2 C) 2 : 3 D) V2 : V3 E) Ning. anterior.

11) La figura, muestra un tetraedro regular B

ABCD; AB = 14>/3 , R e AB y M biseca C D .

Una hormiga quiere alcanzar su comida ubi­


cada en M, partiendo de R siguiendo la tra­
yectoria más corta. Hallar la longitud de tal
trayectoria, si: RB = 20.
A) 25 B) 28 C) 29 D) 30 E) Ning. anterior

12) La suma de las medidas de las caras de un poliedro convexo, es 3600°. Si el número de
aristas excede en 2 al doble del número de caras; hallar el número de caras.

A) 8 B) 9 C)10 D) 7 E) 11

7 6 8
13) En un poliedro convexo, el número de caras, más el número de vértices, y más el número
de aristas, es 28. Si los ángulos en todas las caras suman 1800°; hallar el número de caras.

A) 13 B) 7 C) 9 D) 10 E) 8

14) ¿Cuántos triángulos pueden formarse con 30 palitos de fósforos, siendo cada uno un lado?

A) 12 B) 18 C) 20 D) 10 E) Ning. anterior.
B
15) La figura muestra un octaedro regular
de arista “ a Gv G2 y G3, son
baricentros de las regiones triangula­
res ABF, BCD y AED, respectivamen­
te.

Hallar la longitud del segmento que es


intersección de las regiones BDEF y
G 1’ G 2 y G 3-

3
A) Ia B) a

16) Si se unen los baricentros de las caras de un icosaedro regular, se forma:

A) Un dodecaedro B) otro icosaedro C) Un octaedro D) Un hexaedro

E) Ninguna anterior.

17) En la figura: B

La arista del tetraedro regular tiene longi­

tud 4 t¡33 cm. M, biseca AD. E, biseca


BD. FC = 3 BF. Hallar la longitud del
segmento que es intersección de las re­
giones AEF y BMC.

A) 11 cm. B) 10 cm C ) 12 cm D) 13 cm. E) Ning. anterior.


CLAVE

1. D 7. E 13. E
2. D 8. D 14. C
3. D 9. D 15. A
4. B 10. A 16. A
5. B 11. C 17. A
6. A 12. A
769
CAPITULO 2 6

PRISMA Y TRONCO DE PRISMA


SUPERFICIE PRISMATICA:
Se llama superficie prismática, a aquella que genera una recta (generatriz), al deslizarse
paralelamente a su posición inicial, a lo largo de una poligonal o polígono (directriz).

Si la directriz es una poligonal, la superficie prismática es abierta. Si es un polígono, la


superficie es cerrada.

Superficie prismática abierta.


r : generatriz n : generatriz
ABCD : directriz P Q R S T : directriz
B C
PRISMA

Un prisma, es el poliedro determinado al inter­


ceptar una superficie prism ática cerrada,
mediante dos planos paralelos entre sí.

La figura adjunta muestra un prisma.


Las regiones poligonales ABCDE y A ’B'C'D’E’
son paralelas y corresponden a polígonos
congruentes. Estas dos caras son las “bases"

771
del prisma y la distancia entre ellas es la altura del sólido. Las demás caras son regiones
paralelográmicas, llamadas "caras laterales"; sus intersecciones se llaman "aristas
laterales". Todas las aristas laterales son paralelas y congruentes.

CLASIFICACION DE LOS PRISMAS.- Se clasifican en: recto, oblicuo y regular.

a) Prism a Recto.- Es aquel cuyas aristas laterales son perpendiculares a las bases. Las caras
laterales son regiones rectangulares, y las aristas laterales son congruentes a la altura.

b) Prism a O blicuo.- Tiene sus aristas laterales oblicuas a las bases.

/
/
/

Según sus bases sean regiones triangulares,cuadrangulares, pentaginales, etc., los prismas
se llaman triangulares, cuadrangulares, pentagonales, etc. Por ejemplo, la figura (a) muestra
un prisma recto triangular.

c) Prism a Regular.- Aquel prisma recto, cuyas bases corresponden a polígonos regulares. ( En
cualquier otro caso, el prisma no es re g u la r).

SECCIONES DE UN PRISMA
Una “sección” de un prisma, es la región determinada por la intersección del prisma con un
plano.

Una “sección transversal” de un prisma, es la sección del prisma con un plano paralelo a la
base.

Una “sección recta” de un prisma, es la sección del prisma con un plano perpendicular a las
aristas laterales. Por ejemplo, la sección PQR, en la siguiente figura.

Nota.- Si B y SR, son las áreas de la


base del prisma y de la sección recta,
respectivamente; entonces: SR=Bcosp
donde p, es la medida del ángulo diedro
que forman los planos que contienen a la
base del prisma y a la sección recta.

PARALELEPIPEDO.- Aquel prisma cuyas bases son regiones paralelogramicas.

C lasifica ció n de Paralelepípedos.- Se clasifican en:

a) Paralelepípedo recto: sus aristas laterales son perpendiculares a las bases. Las caras
laterales son regiones rectangulares.
772
b) Paralelepípedo oblicuo: tiene sus aristas laterales oblicuas a las bases.Las seis caras son
regiones paralelográmicas.
c) Paralelepípedo rectangular: aquel paralelepípedo recto cuyas bases son regiones rectan­
gulares. Llamado también rectoedro.

d) Cubo:- Es un paralelepípedo rectangular que tiene todas sus aristas congruentes.


m

e) Romboedro.- Aquel paralelepípedo que tiene por bases regiones romboédricas.

Paralelepípedo Oblicuo. Paralelepído rectangular (rectoedro)


d : Longitud de la diagonal

d2 = a2 + b2 + c2

FORMULAS

I) S uperficies lateral y total de un prism a.- La superficie lateral de un prisma es la suma de


las superficies de todas sus caras laterales. La superficie total del prisma es la suma de
superficie lateral y de las dos bases. A dichas superficies se refieren las áreas lateral y total.

Teorema.- El área lateral de un prisma oblicuo es el poducto del perímetro de una sección
recta por la longitud de una arista lateral.

Así, para el prisma de la figura:

longitud de la arista lateral.


P Perímetro de la sección recta.

El área lateral: SL = P . a

Area total.- Si B, es el área de cada base; el


área total será:

S( = S l + 2 B

II) Volum en de un prism a.- El volumen de un prisma es el producto del área de una base por
su altura.

Si h, es longitud de la altura del prisma:

V = B.h |

773
También, el volumen de un prisma, es el producto del área de una sección recta por una arista
lateral. Así, llamando SR, al área de una sección recta:

V = ( SR ) . a~|

O bservaciones.- En la figura anterior, a°, es la medida del ángulo que forman las aristas
laterales con las bases.

Es evidente que, en un prisma oblicuo, a < 90 y h < a. Además, el área de la sección recta,
es menor que el área de la base: SR < B.

En un prisma recto: a = 90 ; h = a y SR = B.

De lo anterior, se deduce que, el área lateral de un prisma recto, es el producto del perímetro
de una base por una'arista lateral. Asimismo, el volumen es igual a producto del área de una
base por la arista lateral.

Si a, b y c, son longitudes de tres aristas concurrentes de un paralelepípedo rectangular,


entonces su volumen será:

V = a . b .c

Si se extiende (se desarrolla) la superficie lateral de un prisma, a partir de una arista lateral,
(Por ejemplo: PQ), de modo que todas las caras laterales queden coplanarias, se dice que
se ha desarrollado dicha superficie.

P M

TRONCO DE PRISMA

Se obtiene al interceptar la superficie


lateral de un prisma, con un plano parale­
lo a las bases.

Las caras laterales son trapecios.

El volumen es igual al producto del área


de una sección recta y la longitud del
segmento que une los centros de grave­
dad de las bases del tronco. (CG1).
Fig. 1
(Las secciones rectas del tronco son las
misma que el prisma original).
Existen fórmulas sencillas para evaluar el
volumen de un tronco de prisma de base
triangular. Así, para el tronco de la figura 2; el
volumen V, se evalúa:

V = ( área AEC ). f hi + h2 + h3 '

Fig. 2
También, para la misma fig.2.:

s
V = ( área de una sección recta ). AF + EQ + CR >
\

Si el tronco de prisma es recto (originado de un prisma recto), y de base triangular, las caras
laterales resultan trapecio rectángulos, (fig.3).
En este caso:

V = B •
V

a, b, c: longitudes de las aristas laterales.


B : área de la base del prisma recto original

También, se pueden presentar gráficos como


en la figura 4.; donde B es el área de la base,
del tronco de prisma recto.

V=B —
3

b=0 b=0;c=0

A veces, es frecuente tener troncos


originados al interceptar la superficie
lateral de un prisma con dos planos,
como en la figura 5; donde A B , CD y
EF son aristas del tronco.

^ f AB + CD + EF N
/ *
área de una
V=
\ sección recta j v /
PROBLEMAS

1) El trapecio ABCD, es base de un prisma recto. BC / / A D , BC = 2cm, y AD = 7 cm. Hallar el


volumen del prisma, sabiendo que su proyección sobre un plano perpendicular a B C , tiene
área 12 cm2.
A) 6 0 cm3 B) 64cm3 C) 54cm3 D) 81 cm3 E) 72cm3

Solución
Sean H, altura del prisma y h altura de la
base trapezoidal. La proyección del pris­
ma, sobre el plano P, perpendicular a B C ,
es un rectángulo de lados H y h.
El área de la proyección, es :
h . H = 12 cm2 (1 )
El volumen del prisma:

V = ( área de la base ) x H

V -Ü lU h a H -> V = -xh-H
2 2

Con (1 ) : V = —■x 12 V = 54 cm3


2
Rpta: ( C ).

2) La base de un prisma recto, es base de un tetraedro regular de altura 2 V 6cm y el área la


teral del prisma es igual al área total del tetraedro. Hallar el volumen del prisma.

A) 27cm3 B) 54cm3 C) 18cm3 D) 36cm3 E) N.A.

Solución
- Sea la figura
Ve
- Se sabe, por fórmula: h =
h*2,/6
Entonces: 2<j6 = —
3
- De otro lado, según dato:

área lateral área total


del prisma del tetraedro

3aH = 4a2 V3

y, el volumen del prisma: H = 2V 3


77fí
v = a2 — X H 62 x 2 J3
4
V = 54 cm3
Rpta.: ( B )

3) Hallar el área lateral de un prisma oblicuo, cuya sección recta es un hexágono regular de
área 24 J~3 u2 . La altura del prisma es 3 ^ 3 u y las aristas laterales forman ángulos de 60°
con la base.

A) 144 u2 B) 348 u2 C) 834 u2 D) 438 u2 E) N.A.

Solución
- Sea la figura:

- El triángulo rectángulo AHB es de 30° y 60°.


El cateto AH = 3 V3 por dato, luego la hipotenusa
AB = 6 u.

- Para hallar “a”, longitud del lado de la sección


recta:

Por dato: área = 24^3

Es decir: 5 a2 >/3 = 2 4 ^3
2
a = 4

- El área lateral: SL = 6 x A B x a = 6 x 6 u x 4 u = 144 u2

Rpta.: ( A )

4) Hallar el volumen de un prisma oblicuo triangular, sabiendo que el área de una cara lateral,
es 5cm2 y la distancia de la arista opuesta a ésta es 10cm.

A) 50cm3 B) 20cm3 C) 32cm3 D) 40cm3 E) 25cm3

Consideremos el prisma de la figura, donde la


cara lateral ABCD tiene área 5 cm2, según
enunciado. PQ = 10cm., es la distancia de la
arista EFal plano ABCD. PQ se encuentra en
una sección recta del prisma, tal como MPR.

Se tiene : SABCD = 5 cm2

AB . MR = 5 cm2 (1 )

777
El volumen de! prisma:
\
' MR x PQ
V = ( S mpr) x AB -> V= x AB
\ j

ó: V = - (A B .M R ).P Q —> V = ¿ ( S ABCD) . P Q ( fórmula)


2

Con los datos: V = — ( 5cm2) ( 1 0cm ) V = 25 cm3

Rpta.: (E)

Hallar el volumen del prisma regular hexagonal, de la figura. El plano que pasa por las aristas
EF y GH, forma diedros de 45° con las bases y la sección determinada tiene área 6 ^6 .

A) 36 B) 40 C) 46 D) 50 E) 56

Solución
Incógnita: V = ?
Como E J l P y J H l H G ; entonces,
con el teorema de las tres perpendicu­
lares:
E H l H G . Luego, EHJ es un ángulo
plano del diedro que forman la base y
el plano diagonal. Si “h” es longitud de
la altura del prisma y “a”, longitud del
lado de la base, se tendrán.:

(Fig. 2)

En el prisma, MNmide igual que la diagonalmayor de la base: MN = 2a.

EH hV¡2
De la figura 1: EH = h->/2 y la figura 2: EQ =

El área de la sección EFNGHM, es: 2 ( SEFNM) = 6>/6

EQ
Luego. S EFNM = 3-JE ; Es decir: ( EF + MN ) = 3-/6

hV2
Esto es: ( a + 2 a ) = 3-Jg ; ah = 4 ^ 3 (1 )
- Pero, de la figura 1: h = a ^ 3

- Entonces sustituyendo en ( 1 ): a . a V3 = 4<j3 -» a = 2 h = 2^¡3

- Finalmente, el volumen:
x x
V = Bh = ' —a 2 V3 h* - x 22 V3 2 V3
V 2 \ 2

De donde: V = 36
Rpta.: ( A )

6) Un prisma recto tiene como base un trapecio isósceles cuyos lados no paralelos miden 13cm.
y cuyas bases miden 10cm. y 20cm. Hallar el área de la sección plana cuyo plano forma un
ángulo de 60° con la base y pasa por la mayor base del trapecio.

A) 300 cm1 B) 350 cm: C) 360 cm: D) 400 cm: E) 450 cm;

Solución
Sea EFDA, la sección.

En el gráfico: AH = QD y HQ = EF

20-10
AH = AH = 5
EHB-
En el A AHB:

BH = V ab2 - a h 2 = V ( 13 )2 “ ( 5 )
BH = 12

- En el A EBH ( 30°, 60° ) : EH = 2 BH EH = 24

< 10 + 20
- Luego: área EFDA = Í E F + A D U = x 24
l 2 J ^ 2 )

. área EFDA = 360 cm2


Rpta.: ( C )

7) La base de un tronco de prisma oblicuo triangular, tiene área 12. Hallar el volumen del sólido,
sabiendo que las aristas laterales están inclinadas 60° respecto a la base y tienen longitudes
3; 4 y 5 respectivamente.

A) 2 4 ^ 3 B) 23-J3 C) 22 y¡2 D) 21 >/2 E) 20 V3

Solución
- Incógnita: V = ?

779
(Fig. sgte.)
Del gráfico, donde hv h2 y h3 son alturas del
sólido:

h, = |>/3
5 ; h, = 2^3
- y h ,

- Cálculo del volumen:


r \
V = B ; B área de la base
\
- Pero: B = 12 ; luego:

v =l - V 3 +2>/3 + - V 3 •12 V = 24^3


3 ,2 2 J

Rpta.: ( A )

8) Hallar el volumen de un tronco de prisma recto, cuyas bases son un triángulo equilátero FED
y un triángulo rectángulo isósceles ABC.
Además una cara lateral es un rectángulo de lados 3>/2 y 6; siendo los mayores lados las
aristas laterales.
A) 35,1 B) 51,3 C) 31,5 D) 53,1 E )6

Solución
Incógnita: V

El sólido es, como indica la figura:


El A ABC, isósceles, recto en B.
El triángulo EFD, equilátero.
Luego: EF = FD - 3 ^ 2
y : AB = BC = 3

Para hallar EB, se traza F H 1 BE.


Entonces: BH = AF = 6 ; FH = AB = 3
En el A EHF, HE = 3. Por lo tanto : EB = 9

' EB + AF + CD '
El volumen: V = S ABC
/

3x3
V = 31,5
\ /
Rpta.: ( C )

9) En un tetraedro ABCD, AB = 10. El área de la proyección de éste sólido, sobre un plano


perpendicular a AB, es 24. Hallar el volumen del tetraedro.

A) 90 B) 80 C) 70 D) 60 E) 50
780
Solución

Consideremos el gráfico que sigue, donde el


triángulo EFG es la proyección del tetraedro
sobre el plano P; el volumen Vx pedido, es:

V
vx
= VAEFGDC - VBEFGDC

\ \
AE+CF+DG ' B E + CF + DG
•S EFG
\ / /
Factorizando S EFG
^ , se tiene:

Vx = .S EFG « •

(10)( 24)
Con los datos: Vy
a = Vx = 80

Rpta.: ( B )

AA'; BB‘ y CC1, son aristas laterales de un prisma. “M”, es punto medio de AA' y “E" es un
punto de C C ' . Hallar CE para que los volúmenes AMB’BCE y EMA’C’B’, sean entre sí como
5 a 3; si BB' = 12.

A) 4 B) 5 C) 2 D) 4,5 E) N.A.

Solución
Sea la figura siguiente:
Incógnita: CE = x
- Se tiene, por dato:
VAMB'BCE _
V,EMA'C'B'

Que, es lo mismo que:

AM + CE + BB1
( Sección re c ta )
3 5
A 'M + C‘ E + 0 3
( Sección re c ta )
3

Luego:
6 + X+12 5
6 + 12-x + O 3

De donde: x = CE = 4,5
Rpta.: ( D )
781
11) En un tronco de prisma recto, de aristas laterales 3; 4 y 5 cm. (Prisma Triangular), la base
es un triángulo cuya área se desconoce.
Hallar el volumen del sólido si la otra base mide 12 cm2 y está contenida en un plano que for­
ma un diedro de 60° con el plano de la base desconocida.

A)24>/3 cm3 B) 24 cm3 C ) 1 2 cm3 D)12V3 cm3 E) N.A.

Solución

- Por propiedad:
Fórmula del área de la
Bt = B2 x Cos60 proyeción de una región
plana.

V3
B, = 12 x

B1 = 6*>/3 cm'

\
3+4+5
- Luego, el volumen: V =

- Sustituyendo valor de B1t se tiene: V = 24->/3 cm'

Rpta.: ( A )

12) Se tiene un cubo ABCDA’B’C’D’, cuya longitud de aristas es “a”. Un plano intersecta a las
aristas AA1, B B \ CC1y DD1 en los puntos P, Q, R, M respectivamente, de modo que AP =
A’P y CR = 2C’R. Hallar el volumen del tronco ABCD - MRQP.

7a- 5 a- 11a
A) B) C) D) E) a
12 12 12 12

Solución

El volumen buscado será:


B’ C’
v = ( s abcd) . oh

Donde:

S abcd _ a

PA + RC
OH =

782
OH = — a
12

Luego :

7 3
V = — a
12
Rpta.: ( A )

Hallar el volumen del sólido de la


figura, si ABCD es un cuadrado; AEFD
y BEFC son trapecios isósceles con­
gruentes; triángulo AEB - » Equilátero.

Solución

Incógnita: V = ?

Se trata de un tronco de prisma con aristas


laterales AD, BC y EF;

Triángulo FPQ es una sección recta. En el


trapecio AEFD, se puede hallar PD, si se traza

EH_LÁD, / . A H = PD = — EF

/. PD = — . Luego, en triángulo rectángulo FPD:

FP = JFD 2 - PD2 = - V T 5
y 2

Para hallar el área FPQ; se deduce :

FM = -5-VTí
2

Entonces:

S rfq = \ PQ -FM -> SPFQ VTi

Y, el volumen del tronco:

' AD + BC + EF '
V = ' Spfq V = - a 3VTi
v J 6

783
14) En un tronco de prisma triangular recto, la base ABC es un triángulo equilátero de lado 6 y
las aristas laterales son: AA, = 8 ; BBt = 5 y CC, = 4
Hallar el volumen del tetraedro de base A1B1C1 y de vértice B.

A) 5 / 3 B) 6 5 / 3 C) 1 2 / 3 D) 4 5 / 3 E) 2 7 / 3

Solución

Incógnita: V = ?

Se tiene, en la figura:

= V Total ' V a b c C í A,

"8 + 4 + 5 ' •
"8+5+0'
Vba ^ a ^ABC .S ABC

3 • "
3

Pero: SABC = 62V3 / 4;

4 r2>/3
Luego: VBA)BlCi = ~ 6 —

Vba ^ tC, - 1 2 V 3

Rpta.: ( C )

15) El prisma ABCA’B'C’, se corta por un plano determinando los puntos M, N y P, sobre las
aristas laterales A A \ BB' y C C \ respectivamente, de modo que AM = 2MA’ y BN = PC’.
Si el volumen del prisma es V, hallar el volumen del tronco ABCMNP.

A) - V B) 1 V C) - V D) L V
5 4

Solución

- Incógnita: V ABCMNP

Si se llama “S* el área de la sección recta


del prisma y también:

V
vto ta l
= V
v

V
3(a . S ) = V -> a . S = (1)

Por otro lado:

V ABCMNP = ¿ (A M + BN + C P ) . S

784
1 5
VABCMNP = - * ( 2 a + m + 3 a - m ) . S
3

Con (1 ) : V ABCMNP ~ g V

Rpta.: ( D )

Probar que, todo rectoedro, el volumen es igual a la raíz cuadrada del producto de las áreas
de tres caras concurrentes.
Solución

Sean A, B y C las áreas de las caras rectangu­


lares que concurren al vértice N.

x, y, z : longitudes de las aristas.

Se quiere demostrar que el volumen V, se


puede expresar así:

V = vA B C
Area de los rectángulos:
xy = A
yz = B
xz = C
Multiplicando m.a.m.:
( x y z ) 2 = ABC -> V2 = ABC

v = Va b c
l.q.q.d.

Las bases de un paralelepípedo recto, son rombos cuyas regiones tienen áreas S-j. Las áreas
de las secciones que determinan los planos diagonales, son S2 y S3. Hallar el volumen del
romboedro.

Solución
Incógnita: V. Se diseña el gráfico:
El volumen, es:

V = S t AG ..................... (1 )

Los datos S2 y S3:

AG . GH = S„
y: ED . EF = S3
Multiplicando m.a.m.:
AG . ED . GH . EF = S2 . S3

785
Pero: ED = AG y ^ = S í ; luego:

S, S
( AG )2 . 2S, = S2 . S3 -> AG = (2)
2S 1

Finalmente, reemplazando ( 2 ) en ( 1 ):

18) Hallar el volumen de un prisma regular hexagonal, circunscrito a una esfera de radio “r” .

A) 4I-3 -J3 B) 2r2 V2 0 )2 1 ^ 7 3 D) 3r3V2 E) Sr3 V3

Solución
yf
y
La superficie esférica, es tangente a las
r caras del prisma en sus centros.
0 h
Volumen del prisma:
r
V = B .h ( i )
L Cálculo de h:

Del gráfico : h = 2r .............. ( 2 )

Cálculo de B:
Para la sección mostrada

2 r V3
L =

Area de la base: B = —L V3 , ( fórmula para un hexágono re g u la r)

/ \
3 2rV3
■J3 -> B = 2r 2 V3 (3)
B = 2V

Sustituyendo ( 2 ) y ( 3 ) en ( 1 ):

V = ( 2 r 2 V3 ) ( 2 r ) V = Ar3^

Rpta: ( A )

19) Se tiene un prisma triangular recto cuya base es el triángulo ABC. Se traza un plano paralelo
a B C , que pasa por el incentro de la base ABC y corta en M a AC, en N a AB y en P y Q a
las aristas opuestas a AB y AC.

7KH
Hallar el volumen del prisma en cm3si: CM = 3 cm., BN = 5cm, AM = 6cm. y la arista lateral
mide 10 cm.

A) 820 B) 540 C) 540 D) 360 E) 120

Solución

- Incógnita: V Q

Para hallar el volumen, falta calcular el área de la base: V = ( SABC ) . h (1 )

Como MN || BC :

AN = AM AN = 6
NB MC 5 3

Los triángulos BNI y C M I,


son isósceles.

Luego : NI = BN y CMI
NI = 5 y MI = 3. Entonces:
MN = 8. Se concluye que el
triángulo AMN, es recto en M.

También : ACB = 90°.

En el T.R. ACB : BC2 = AB2 - AC2 = 152 - 92 -> BC = 12

Luego:
AC x BC 9x12
ABC -

ABC = 54 (2)

Sustituyendo el dato “h" y ( 2 ) en ( 1 ) :

V = 10x54 —» V = 540 cm3

Rpta: ( C ).

787
20) Hallar el número de caras “O", número de vértices “V” y total de aristas “A", de un prisma,
en función del número de lados “n” en una base.

Solución
Para el número total de caras, se cuentan un” caras laterales más las bases;
Luego:
C = n+2

Hay Mn" vértices en cada base.


Total V = 2n

Se tienen un” aristas laterales y “n" aristas en cada base.


En to ta l: A = 3n

21) Hallar el número de diagonales de un prisma, conociendo el número de lados ( n ), en cada


base.

Solución
Desde cada vértice de la base superior, se pueden trazar diagonales a ( n - 3 ) vértices de
la base inferior. Luego el número de diagonales del prisma, será: n ( n - 3 ).

22) Hallar el volumen del poliedro de la figura:


B C
Triángulos: BNC y ENF equiláteros y con­
tenidos en planos perpendiculares entre sí.

C u a d r il á te r o s : ABNF y ENCD;
paralelogramos; BC = 4u y EF = 2u

A) 20 u3 B) 18 u3 C) 16 u3
D) 14 u3 E ) 12 u3
F E
Solución
- Incógnita : V

De los datos:
B C
Plano AFED || plano BNC.

Plano ABCD || plano ENF.

Luego: Plano ABCD perpendicular a


los planos AFED y BNC.

Si, por C, se traza CP II N F ; CP


estará contenido en el plano ABCD. El
volumen total es la suma de los volú­
menes de los prismas ABCPFN y
FNCPDE.

788
CH: altura del prisma ABCPFN y del triángulo equilátero

Entonces: CH = 2 2 ^ “3

» • CH = / 3

FR ; altura del prisma FNCPDE y del triángulo AFP, equilátero:


Ap .________ ._
Entonces : FR = — . ^ 3 FR = 2 y 3

Finalmente, como ya se dijo:

Vt = VABCPFN + VFNCPDE

V = ( área AFP ) x CH + ( área CPD ) x FR

\
V = . y[3 + .2/3
\ / v J

V = 18u3

Rpta : ( B ).

Hallar el volumen de un prisma recto, de base triangular, sabiendo que una arista básica mide
4 y el área de la proyección del sólido, sobre un plano perpendicular a dicha arista, es 10.

A) 5 B ) 10 C ) 15 D) 20 E) 25

Solución
- Icógnita: V

Considerando la figura, donde AB = 4;


y P es un plano de proyección per­
pendicular a AB ; se tiene:

QR = CD y RN = CH

Donde CH = altura del triángulo ABC.

Cálculo del volumen:

V = área ABC x CD

V = 1 . AB . CH . CD ó
2

V = 1 . A B . R N . QR
2
789
«

1
Es decir : V = — x AB x área RQMN.

Con los d a to s :
4x10
V = V = 20

Rpta: ( D ).

24) Hallar el volumen de un rectoedro, sabiendo que las diagonales de las caras miden
^ 3 4 ; ^/~58 y J 74 , respectivamente.

A) 105 B) 201 C) 501 D) 205 E) 150

Solución

- Incógnita: V

Se construye la figura

Se tienen:

a2 + C2 = 34 (U

a2 + b2 = 58 ( 2 ).

b2 + C2 = 74 (3).

Sumando (1 ) , ( 2 ) y ( 3 ) :

2 ( a 2 + b 2 + C2 ) = 166

a2 + b2 + c2 = 83 ( 4 )

Sustituyendo ( 1 ) e n ( 4 ) : b = 7 ;

Luego:
a = 3 y c = 5

El volumen:
V = a .b .c V = 105

Rpta: ( A ).
M

25) La figura muestra una caja en forma


de un prisma regular pentagonal.

Una “astuta” hormiga, parte de “A”, en


busca de su comida en “E”, siguiendo
la trayectoria ABCDE, de menor longi­
tud posible, debido a que la cara AN EM
está rociada con un insecticida.

790
Hallar la longitud de dicha trayectoria.

A) 51 B) 13 C) 42 D) 28 E) 3

Solución

M
A'
r

i N N’
12

Se desarrolla la superficie la te ra l, a través de AN. La menor trayectoria de A a E , es A E .


( La recta que los une ).

Luego, en el A ANE :
2 2 2
AE = AN + NE

AE2 = 5 2 + 122 ; AE = 13

Rpta: ( B ).

Dado un cuadrado ABCD, de lado 6, se elevan AE y CF, perpendiculares, al plano que lo


contiene. Si AE = 4 y CF = 5. Hallar el volumen del tetraedro FEDB.

A) 120 B) 136 C) 114 D) 110 E) N.A.

Solución
V = ?

El volumen pedido : V = VFEDB

V = Vtotal " VFCDB - VEBDA ............ ( 1 )•

Además, si se traza AC :

Vtotal = VACBFE + VACDFE

Sustituyendo en (1 ) :

V = VACBFE + VACDFE “ VFCDB “ VEBDA


( 2 ).

( Todos son troncos de prisma ), donde:

791
\

CF + AE + O
VA C B F E “ •S abc = i 1-18 = 54

' CF + AE + O '
VACDFE - • S aco = I 1.18 = 54
\ /

V,FCDB BCD “ .1 8 = 30
v /
/
V,EBDA - . SBDA 1.18 = 24
\ V

Reemplazando, en ( 2 ) :
V = 54 + 54 - 30 - 24 ;

V = 54

Rpta: ( E ).

El volumen del prisma regular ABC A ’B’C’ , es 18 u3. Se toman M y N , puntos medios de
las aristas laterales BB' y C C \ respectivamente.
Las prolongaciones de A'N y A C , se cortan en P y las de A ’M y AB, en Q.
Hallar el volumen del poliedro MBCNPQ.

A) 14 B) 12 C) 10 D) 8 E )6

Solución
Si las aristas laterales del prisma
ABCA’B’C’, tienen longitud 2a y las
bases área S; resulta, del dato:

S ( 2 a ) = 18

aS = 9 ....... (1 )
El volumen pedido:

v - Va* apq “ va‘ACBMN (2 )

Donde: NC es base media del triángulo A*AP y : BM es base media del triángulo A*AQ
Luego; BC es base media del triángulo PAQ.
Entonces: SPAQ = 4 ( SABC ) ó SPAQ = 4S

A s í : Yaa 'Pq = —( SPAq )( A A ' ) = —( 4S ) ( 2a )

Vaa'PQ = 3 ( a S ); con( 1) :
V AA'PQ = 2 4 (3)

- Además: A’ACBMN es tronco de prisma.

f AA' + CN + BM '
- Luego: VA<ACBMN = ( SABC )
\ /

V A'ACBMN
a- = (S ) = —( a S )
v / 3 ;

Con (1): VA>ACBMN = 12 .......... ( 4 ) .

- Finalmente, reemplazamos ( 3 ) y ( 4 ) , en ( 2 )

V = 2 4 - 12 -> V = 12 u3

Rpta: ( B ).

IMPORTANTE:
El "Principio de Cavalieri", es la clave para el cálculo de volúmenes y con él se demuestra
la fórmula para el prisma, teniendo en cuenta el P ostulado de la Unidad de Volum en,
que es el paralelepípedo rectangular. Así, secuencialmente, se mencionan :

El Postulado de la Unidad: El volumen


de un paralelepípedo rectangular es h
el producto del área de la base (B) y
la altura (h).

P rincipio de C avalieri.- Dados dos cuerpos sólidos, cuyas bases son equivalentes y
éstan contenidas en el mismo plano.
Si todo plano paralelo al anterior in­ A7
1
terseca a ambos sólidos y determina 1t
secciones transversales con áreas 1
1
11
iguales; entonces, los cuerpos tienen 1
el mismo volumen. I
A7
* m
8

793
CAPITULO 2 7
v.
VÁV.

CAPITULO 2 7
PIRAMIDE Y TRONCO DE PIRAMIDE

DEFINICION.- Pirámide, es el poliedro obtenido al interceptar una superficie piramidal


cerrada, mediante un plano.

- La figura muestra una pirámide, de


vértice “O”; base ACDEF y caras latera­
les AOF, O F E , e t c .

- “h”, es la Ingitud de la altura del sólido.


( Distancia del vértice, al plano de la
b a s e ).

- Si “V”, es el volumen de la pirámide,


cuya base tiene área “ B ”, se calcula
así:

J
La suma de las áreas de las caras laterales ( todas, regiones triangulares), es el área lateral
de la pirámide. El área total del sólido es igual al área lateral, más el área de su base.

La pirámide se llama regular, si la base es un polígono regular y la altura cae en el centro de


la base. En cualquier otro caso, la pirámide es irregular.

Una pirámide se menciona según el


número de lados de la base, indican­
do primero el vértice. La figura 1 mues­
tra una pirámide irregular pentagonal
O - ACDEF y la figura 2, una pirámide
regular cuadrangular O - ACDE.

En la figura 2:
ON es apotema de la pirámide.

a, es la medida del diedro que forma


la cara latera! con la base.

795
<J>, es medida del ángulo que forman las aristas laterales con la base. ( Se cumple: a > <¡))

PROPIEDADES
1. Si dos pirámides triangulares, tienen
un mismo triedro o dos triedros con­
gruentes, uno en cada sólido, enton­
ces los volúmenes son entre sí como
los productos de las longitudes de las
aristas que determinan dicho triedro.

Para el gráfico:

( VAEFQ, se lee: volumen del sólido AEFG ).

2. Si se intercepta la superficie lateral de cual­


O quier pirámide con un plano paralelo a la
base, se obtiene una pirámide parcial se­
m ejante a la original.

Se cumple:

a) Las áreas son entre sí, como los cuadra­


dos de las longitudes de las alturas o de
cualquier par de líneas homologas.

Así, llamando S1 y S2 las áreas de las


superficies referidas a la pirámide parcial y
total, respectivamente:

( Sn y S2, pueden representar áreas totales, áreas laterales, áreas de las bases ó áreas de
caras hom ólogas).

b) Los volúmenes son entre sí como los cubos de cualquier par de líneas homólogas

V, _ ( h, )3 _ ( O E ) 3 _ ( E Q ) 3 _
V. ( OA )3 ( AM )'

796
V

DESARROLLO DE LA SUPERFICIE LATERAL.- Resulta una región poligonal.

TRONCO DE PIRAMIDE

Un tronco de pirámide, es el sólido


que se determina al cortar la superfi­
cie lateral de una pirámide, con un Pirámide
plano, sea o no, paralelo a la base. deficiente
Se llaman bases del tronco, a la base
de la pirámide original y a la región
que genera el plano secante. Pirámide
El volumen se calcula restando al
volumen de la pirámide original, el de
la pirámide deficiente.

TRONCO DE PIRAMIDE DE BASES PARALELAS.- En este caso, las regiones poligonales


que representan las bases del tronco, están contenidas en planos paralelos. Estos polígonos
son semejantes.

La distancia entre dichos planos es altura


del sólido. En este caso, el volumen se
evalúa así:

V =- ( a + B + /A B

A y B son áreas de las bases

“ h “ es la longitud de la altura.

797
TRONCO DE PIRAMIDE REGULAR.- Es el tronco de pirámide regular, de bases paralelas.
Las caras laterales son trapecios isósceles. El segmento que une los puntos medios de las
bases de cada cara lateral, representa un apotema del tronco. ( Ejm.: MN ).

El volumen se calcula con la fórmula


anterior.

PROBLEMAS RESUELTOS
1) Hallar el área lateral y el volumen de una pirámide regular hexagonal, sabiendo que las caras
laterales forman diedros de 45° con la base y las aristas básicas tienen longitudes “ a “.
Dar como respuesta el volumen.
4 3
A) a3
C> 3 3
E
Solución
- Incógnitas SL y V.

OM es el apotema de la base.

Se calcula su v a lo r:

OM = — J~3
2v

En el T.R. EOM ( isósceles )

EM = ( OM ) J~2 ; EM = | / 6 (1 )•

y: OM = EO = ( 2 ).

Cálculo del Area Lateral:


' A B _E M '
SL = 6 ( S aeb) = 6
, 2 ,

798
V

C álculo del Volum en


1
V = “ (área de la b a s e ). EO
w
1
V = - ^ f - a 2/ 3 •-/3
3 2v

Rpta: ( D ).

2) Hallar la fórmula para calcular los volúmenes del tetraedro y octaedro, regulares, en función
de la longitud “ a “ de la arista.

Solución r-

a) Tetraedro regular:

Se sabe que, la longitud de la altura


del tetraedro regular, es:

EH = - J 6
3V

Luego, el vo lum en:

1 Í3
V = - r ( área A ACD ) . EH v - l
O 3
\

( fórmula)

b) Octaedro re g u la r:
Del gráfico:
Voctaedro = 2 V EACDG

Voctaedro 2 . [ ~r( área ACDG ) . EO ]


D

Voctaedro = 2

( fórmula)

3) Hallar el volumen del sólido que se obtiene al unir los centros de las caras de un cubo, de
volumen V.

V V V
A) 2V B) V C) D) E)

799
Solución
Sea el gráfico del problema : Al unir los centros de las caras del cubo se forma doble
pirámide:

Se observa, del gráfico:

’OPQR “ — ( ^ABCD )

1
y M N = —.altura delapirám ide M - O P Q R .

Volumen pedido:

V = 2 ( V M - OPQR )
/
a¿ a
t
V, = 2 ( S 0PQ R . M N ) = 2
\ 3'~2'2 j

V
Vx = Vx =

Rpta: ( E ).

4) Hallar el volumen del solido que se forma al unir los puntos medios de las aristas de un cubo,
de volumen “ V

V
B )|V D )|V E )|V
A> 2 c )fv

Solución
Sea la figura:
El volumen Vx, pedido, equivale al del
cubo, menos el de las ocho pirámides:

Vx = Vcubo - 8 1í Vpirámide *
1

V„ = V - 8 [ 1 . ( S bac) .(EA)]

' 1 ' 1 a a ^ f a '



V X= V - 8
3 , 2 ' 2 ' 2 > , 2 y
3
= V - — ; pero a3 = V , luego:

V
V =
V‘ = V - ? 6

Rpta: (E )

800
V

5) Se tiene un pirámide E - ABCD, cuya


base es un trapecio, siendo BC || AD;
BC = 6 y AD = 10. Hallar el volumen,
sabiendo que el área de la proyección
de la pirámide, sobre un plano per­
pendicular a B C , e s 1 8 .

A) 108 B) 96 C) 13

D) 50 E) 10

Solución

- En el gráfico, P es un plano de proyección, perpendicular a B C .


BC y AD se proyectan como puntos. La altura de la pirámide se proyecta en su verdadera
magnitud ( RH = EF ). MN tiene igual longitud que la altura del trapecio ABCD.

1
Para el volumen : V = — ( SABCD) . EF
3

^ BC + AD
V-l x MN EF
3 v

Como E F = RH :
w 1 M N x RH
V = — x /( dBC
/-' ah \
+ A D ) x --------------
3 v ; 2

MN x RH 0
Pero ------------ SMRN

V - — x ( BC + AD ) x SMRN

1
Con los datos : V = — x ( 6 + 1 0 ) x 18
O

De donde: V = 96

Rpta: ( B ).

6) Hallar el volumen de una pirámide regular cuadrangular, sabiendo que el lado de la base tiene
longitud “ a “ y el plano que pasa por una arista básica y la base media de la cara opuesta,
forma un diedro de 45° con la base.

a3 a3 a3 a3 a3
A )T b j— C )T D )T E )-

801
V

Solución
Sea la figura del problema:
Sea E - ABCD, la pirámide y MNCD
el plano que forma diedro de 45° con
la base ABCD. ( Figura 1 ).

Si trazamos EP 1 AB y EQ _L CD,
en el triángulo PEQ se puede hallar la
longitud de la altura EH, de la pirámi­
de. ( Figura 2 ).

EH = 3 ( GH )

Siendo : GH = HQ = -

3a
EH =

Luego, el volumen de la pirámide :

1
3
\
^(a2) 3a
J

Rpta: ( A ).

7) Hallar el volumen de una pirámide E - ABC, cuyas caras laterales forman diedros de 45° con
la base ABC.

S i : AB = 13 c m . ; BC = 15 cm. y AC = 14cm.

A) 110 cm B) 115 cm C) 118 cm3 D) 112 cm3 E) 120 cm3

Solución
- AB = 13 ; BC = 15 ; AC = 14

C álculo de la altura EH :

En el gráfico:
EQH = EPH = EÑH = 45

Luego:
AE H Q = A E H P = A EHN

HQ = HN = HP

802
Es decir, H equidista de los lados del triángulo ABC. H es el incentro del triángulo y r el
inradio.

Luego : EH = r

Para hallar SABC, con fórmula de Herón:

13 + 14 + 15
P= = 21 /. p = 21

SABC = V 2 1( 21 - 1 3 ) ( 2 1 - 14 ) ( 2 1 - -!5) ;

SABC = 84

También : SABC = pr; sustituyendo valores resulta:

r = 4 EH = 4

El volumen de la pirámide:

V = - x SABC x EH = - x 84 x 4 = 112
3 3

Rpta: ( D ).

Las áreas de las bases de dos pirámides semejantes, son entre sí como 4 es a 9. Hallar la
relación de sus volúmenes.
8
A) B) D) E) Ning. anterior
27 C> 9 27

Solución
Siendo las áreas S1 y S2; las alturas h, y h2; se sabe que se debe cumplir
\2
1 __

\
h2

\
1 4 hi _ 2
Luego :
9

Luego, para los volúmenes

V1 _ ül V, _ 8
V V h2 / Vo 27

Rpta: ( B ).

En qué relación se encuentran los volúmenes de los sólidos parciales que determina el plano
mediatriz de la altura de una pirámide.
1 1 1 1 1
A> i B> i C> 4 ° > 5 E> 7

H (K 5
Solución
- Sea la figura:

Del gráfico, se pide -^L , para h = ^

Se sabe que : = JíL


Vtotal H
/ \3
H
V V1 _
V,total H Vtotal

Es decir: V, = ¿ V,ota| V2 = | v , otat

Lueg° : v2 “ 7

Rpta: ( E ).

10) El número de caras de una pirámide es 10. Hallar la suma de los ángulos internos de todas
las caras.

A) 3600° B ) 2880° C ) 2800 D ) 2400 E) Ning. anterior.

Solución
Si “ n “ es el número de lados en la base, entonces el número de caras laterales será “ n “
El total de caras ( incluida la base ), será n + 1, Por dato : n + 1 = 10 .*.■ n = 9.

El número de vértices en la base : 9.

El total de vértices, incluyendo el de la pirámide : V = n+1 ; V = 9+1=10

La suma de los ángulos en todas las caras, se sabe por fórmula :

360° ( V - 2 ) = 360° ( 10 - 2 ) = 2880

Rpta: ( B ).

11) Estabecer relaciones entre el número de caras “ C número de vértices “ V número de


aristas “A”, de una pirámide.

Solución
- Suponiendo “ n “ lados en la base:
A = n aristas básicas + n aristas laterales.

A = 2n O 2 = n

804
V = n vértices en la base + 1
V = n+ 1 ó n = V- 1

C = 1 base + n caras laterales ;


C = n+ 1 ó n = C- 1

Luego :

Obsérvese, que: V = C

El volumen de un tetraedro ABCD , es 30 u3. Sobre AB, AC y AD se toman los puntos


UM V N U y - R respectivamente. S i : AM = MB ; AN = 2 NC y 2 AR = 3 RD, hallar
el volumen del sólido BCDRMN.

A) 28 u3 B) 24 u3 C) 22 u3 D) 20 u3 E) 18 u3

Solución
Incógnita: VBCDRNM

Las pirámides AMNR y ABCD, tienen


en compun el triedro A. Luego:
Vamnr _ AM . AN . AR
Vabcd AB . AC . AD

Sustituyendo datos y valores asigna­


dos por el enunciado:

VAMNR a . 2 b . 3n w _ 3
0 _ Qh , v amnr = 6u
30 U 2 a . 3 b . 5n

Pero : V BCDRNM = VABCD - V AMNR

Luego . ^ bcdrnm - ^4 u3

Rpta: ( B ).

Hallar el volumen de una pirámide regular hexagonal, circunscrita a una esfera de radio
“ r u, sabiendo que las caras laterales forman diedros de 60° con la base.

A) r3 C )6rV 3 D) 7 r3 E) 3 r3 ^[3

Solución

Incógnita: V

La superficie esférica es tangente a las caras del poliedro

El punto de tangencia con la cara EAC, se ubica sobre el apotema AM y con la base, en el
centro “ O u.

«05
Por dato: AMO = 60

Si se traza MP ; en el triángulo MOP,


puede hallarse:
OM = r / 3
Luego, este valor, en la base:
. OM __.
CM = —— ; CM = r CE = 2r
/3

En el T.R. AOM : AO = OM / 3 ;
AO = 3r
Por otro lado, área de la base:

B = f(C E )2. / 3 B = |( 2 r ) 2/ 3 ; B = 6 r2 / 3

Finalmente, el volumen:

V = B( A O ) = 1 . 6 r 2 7 3 ( 3 r ) V = 6 r3 / 3
3 v ;
Rpta: ( C ).

14) Hallar el volumen de un tetraedro ABCD, en el cual AB = 4; CD = 6. La mínima distancia entre


X § y CD, es 5 y además estas rectas se cruzan con un ángulo de medida 30°.
A) 10 B) 20 C) 30 D) 40 E) 50

Solución
Incógnita: V
La figura 1, muestra el tetraedro y la figura 2, el procedimiento de solución
B

(Fig. 2)

806
Para graficar la mínima distancia entre AB y CD, se proyecta el conjunto sobre el plano P,
perpendicular a CD:

ST, es la mínima distancia entre dichas rectas. Por dato : ST = 5.

Como AR CD , entonces el ángulo de cruce entre AB y CD , es el mismo que forma

AB y AR : RAB = 30°.

Además : RM = BE , donde BE 1 AR.

AB
E n e lT .R . A E B : BE =

BE = 2 -> RM = 2
Para el volumen:
1
V = — x área ACD x BH
O
w 1 CD x AQ _u
V = — x --------------- x BH
3 2
Pero : AQ = RS y BH = MN ; Luego:
1 CD x RS . -Kl
V = —x x MN
3 2
Es d e c ir:
1 ' RS x MN ^ 1__,, _ .,c .
V = - x CD = — CD(area RMS)
3
M e jo r:
w 1 „ RM x ST
V = — x CD x -------------
3 2
Sustituyendo datos:

v/ 1 x 6
V = — e x --------
2 x 5
3 2
V = 10

Rpta: ( A ).

B
El volumen de tetraedro ABCD, es V
AP = 2 P B ; BR = RD y BQ = QC

Hallar el volumen del poliedro EFCQRD.

A) y V B )|v C )|v

V
°> 2

807
Solución
Incógnita: Vx = ?
El volumen pedido, es :
V
vx
= VAPEF - VAPQRDC (^1 )r
.................................. V

C\J | CO
Llamando h la altura de la pirámide ABCD, trazada desde B ; como por dato: AP =

2
entonces la altura de la pirámide APEF, trazada desde “ P ”, será —h.

Para hallar el VAPEF, falta relacionar las áreas AEF y ACD. Para ello, se recurre al teorema
de Menelao en el triángulo ABC:
AP BQ CF , .
— x x — = 1, sust. valores :
PB QC FA
CF
2 x 1x — = 1 FA = 2 CF
FA
Análogamente: EA = 2AD
Luego, CD es base media del triángulo EAF.

^ aef ~ 4( SACD)

2h
v (saef)
Entonces: APEf. =
vabcd ( S aco ) ( h )

^£E L = 8 -> W = § V (2).

Ahora, se calcula VAPQRDC :

VAPQRDC = V ABCD - V PBQR ( a )'*


'

Como las pirámides PBQR y ABCD tienen en común el triedro B

VpBQR ®P ^ ^
VABCD
.o ^n = BA X BC X BD ; de donde :

w
BPD r
xA BR
D n
xA BQ
C )V J W e> L / \
pbqr = ~ ~ • V ; Sust. en ( a )
BA x BC x BD

\/ _ \/ BP BR BQ
APQRDC - ABCD ’ ^ X ^ X ^ X V

Sustituyendo equivalentes :
1 1 1 w
V = V - —x —x —xV
APQRDC 3 2 2

^APQRDC “ ^ ^
Finalmente, sustituyendo ( 2 ) y ( 3 )en ( 1 ) :
fl 11 7
Vx = - V - — V ;Vx = - V
x 3 12 x 4
Rpta: (E).

El volumen de un tronco de pirámide cuadrangular regular es 74 cm3 . Si su altura mide


6 cm y el área de una de las bases es 16 cm2
Hallar el área de la otra base, en cm2.
A) 4 B) 3 C) 2 D) 1 E) Ning. anterior.

Solución
Se tiene, por fórmula:

v = ^ ( a + b + / á b )

Donde: V = 74 , A = 16 , h = 6.

6
Luego : 74 = — ( 16 + B + J l 6 B )

Resolviendo : B = 9

Rpta: 9 cm2 ... ( E ).

Hallar el volumen de un tronco de pirámide regular, cuadrangular, de áreas básicas 16m2


y 64m2 circunscrita a una esfera.

Solución
4 -i
Incógnita: V = ?

Areas básicas: A = 16m2.


B = 64m2
Lados de las bases: 4 y 8

Para hallar “ h " , se usa la sección


EFMN ,
donde EF = MN y con el teorema de Pitoth:

EF + MN = 4 + 8

EF = MN = 6

Además: EP = QN = 2

En el T.R. EPF : h2 = EF2 - EP2 =

= 62 - 22 ; h = 4 / 2

809
Ahora, recordando la fórmula del volumen

V = £ (A + B + / A B )

Sustituyendo valores:

_ 4/2
V = — £— ( 16 + 6 4 + V 1 6 x 64 ) m3
3

V = 448-*— m,3

Rpta: ( D ).

18) Se tiene una pirámide regular O - ABCD, de volumen “V”. Un plano que contiene a CD,
intercepta a la cara opuesta en su línea media MN. Hallar el volumen del sólido AMNBCD.

A)fv B )fv C» ? V D )fv

Solución
- Incógnita : Vx = ? = VAMNBCD

El sólido AMNBCD es un tronco de


prisma triangular, de aristas laterales
ÁB, MÑ y CD y bases AMD y BNC.
El triángulo PQE, de la figura 2, es una
sección recta del tronco.

El volumen pedido:

AB + MN -f CD ^
Vx = (á re a APQE )
/
F ig.2 es decir :
o

/ f a + — +a
área A POE 2
Vw =
v /
v

h ah
Donde: área A POE =
( p£) 2 = T
5a2 h
Vx = (1 )■
24
De otro lado, se tiene por dato, que:
VOABCD = V, es decir
1
( a2)h = V ; a2h = 3V

810
Reemplazando en ( 1 ):

Vx = - V
x 8

Rpta: (E).

Demostrar que: Si a un tronco de pirámide de bases paralelas y con áreas en las bases B
y b, se le corta por un plano paralelo a las bases, determinando sobre la altura del tronco,
segmentos de longitudes m y n, respectivamente, el área de la sección “X”, obtenida, es:
2
m ^ B + r \J b |
X =
m +n
/

Donde “ m “ es la distancia entre las regiones de áreas b y X.

Solución
Sea el gráfico de acuerdo al enuncia
do:
Con el gráfico, donde:

MNPQ || AECD || A 'E ’C'D’; por pro


piedad :
B b X
OA OA' OM

Extrayendo raíz cuadrada a todo:

JB /b _ /X
(D
OA " OA' - OM

y[B _ OA
D e(1): J x ~ OM

Por propiedad de proporciones:

/ B - / X O A-O M . / B - / X _ AM
/X " OM Jx ~ OM

De donde : ^ ~ = V Z . . . ( 2 ).
AM OM

/x OM
Otra vez de (1 ) :
yfb OA'

Por propiedad de proporciones:


■{X- Jb QM - OA1 . yfx -Jb A 'M
/b OA' ’ /b OA'

811
Luego :
. y fx ~ y [b /b
(3 )
A 'M OA'

Según la expresión (1 ) , los segundos miembros de ( 2 ) y ( 3 ) son ¡guales , luego los


primeros deben serlo :

J b -/X /X -/b
AM A'M

JB - /X _ AM
De donde
/X - J b ~ A'M

AM n
Pero, en la figura, por Thales :
A'M m

J B -J x n
Entonces
/X - /F m

Efectuando y despejando X
\
f + r\^fb
X =
m+ n
\ /
l.q.q.d

Nota.- Si el plano pasa por el punto medio de las aristas laterales del tronco, a la sección
determinada se llama base media del tronco y su área, por ser m = n , queda:
\2
' JB + Jb
X=
\

E - ABCD, es una pirámide regular, de


base cuadrangular. MNQP es un pla­
no secante a la superficie lateral.

Demostrar, que:

Solución
Una forma de solucionar este problema es relacionando volúmenes de pirámides triangula
res que tienen un mismo triedro:

Ve m n q _ EM x EN x EQ
(a )
VEabc ^ x x ^
V

Vempq _ EM x EP x EQ (b )
VEAcd x x

Como la pirámide EABCD es regular:

EA = EB = EC = ED = a

vy '■ V EABC = VEACO = V


V

De modo que al sumar miembro a


miembro ( a ) con ( b ) se pueden
escribir a s í :

EMNQ + VEMPQ _ E M x E Q( E N + EP)


V

VEMNQP EM x EQ( EN + EP )
Luego v/ — 2
V
De donde:

V EMNQP V . EM x EQ ( EN + EP ) ............( 1 )

Análogamente
f
Venqp EN x EP x EQ (c).
VEBCD EB x ED x EC

Vemnp EM x EN x EP
y • (d)
V,EABD EA x EB x ED

S iendo: V EBCD = V-.™ = V ; al sumar las expresiones ( c ) y ( d ), se puede escribir así


” ' EABD

v enqp + vemnp _ EN x E P (E Q + EM )
V " a3
Pero
rciu V + VV EMNP = V
VENQPT v EMNQP

V . EN x EP ( EQ + EM ) .......... ( 2 )
Luego : V EMNQP

Así, igualando (1 ) y ( 2 )

EM x EQ ( EN + EP ) = EN x EP ( EQ + EM )

De donde, fácilmente se llega:


1 1 1 1
+ +
EP EN EM EQ

813
r
V

CILINDROS Y TRONCOS DE CILINDROS


Superficie C ilíndrica.-

Es la superficie generada , al deslizarse una recta ( generatriz ) , a lo largo de una curva ,


( directriz ) , manteniéndose paralela a su posición inicial.

Fig. 1

Fig.2

En la figura 1: r, es la generatriz de la superficie cilindrica y c 1, la directriz. Como c1 no es


cerrada, la superficie obtenida es abierta.

En la figura 2: c2 es una curva cerrada; Luego, la superficie generada es cerrada.

C ilindro.- Es el sólido obtenido al


interceptar una superficie cilindrica
cerrada, por medio de dos planos _____
paralelos. Tí ■’B ’ N
Las regiones que determinan dichos < llfljlm
planos, son las bases del cilindro y la 9 J¡y//®
distancia entre ellos es la altura. Las / M t M i l ¡IJ __
bases son congruentes. / ~ j ¡ [ [[ [J jif fi 1
Si “B”, es el área de una base y “h” ¿ ' W ljíM n ¡im r /
longitud de la altura; el volumen del ^ x /ii/
sóido se evalúa:

V = B. h

815
- En la figura, eLsegmento de longitud g, es la generatriz del cilindro.

- La sección recta del cilindro, es la intersección del sólido con un plano perpendicular a las
generatrices. ( Todas las generatrices del cilindro, son congruentes ).

- El cilindro es oblicuo, si las generatrices son oblicuas a las bases.

- El cilindro es recto, si las generatrices son perpendiculares a las bases .En este caso :
g = h y además, las secciones rectas son congruentes a las bases.

- Si “ C ", es el perímetro de una sección recta, entonces el área de lasuperficie lateral, se


expresa :

V, el área to ta l:

- Si SR, es el área de una sección recta, el volumen :

CILINDRO DE REVOLUCION

Se genera al girar una región rectangular, una vuelta, alrededor de un eje que contiene a un
lado.

Las bases son círculos y la altura mide igual que la generatriz . Es también llamado cilindro
circular recto.

Fórm ulas :
Area la te ra l: - Area T o ta l: St = SL + 2B

- Volumen : En este caso : B = rtr2

D esarrollo de la su p e rfic ie lateral.- Es la región rectangular, obtenida al extender


( desarrollar) la superficie lateral, de modo que los lados del rectángulo sean la generatriz
y las circunferencias de las bases, del cilindro de revolución original.

816 27tr
Nota.- En el caso de un cilindro oblicuo, el desarrollo puede resultar romboide o rombo

TRONCO DE CILINDRO
Se obtiene al intersectar la superficie lateral de un cilindro, con un plano no paralelo a las
bases.

En la figura 1, OO1 es el eje del tro n c o ; g y


G , son longitudes de dos generatrices
opuestas. ( g < G ).

Las secciones rectas del tronco son las


mismas que del cilindro original.
El volumen se puede evaluar, así:

V = (área de una sección re c ta ). OO*

Donde: OO* =

Si el tronco se deriva de un cilindro de revolución, su volumen es :

v = 7cr2 f g ± Q

( figura .2 .)

Si una generatriz es nula, el sólido se


llama “cuña cilindrica”. Por ejemplo,
en la figura 3 .:

' 0+ G
V = n r2

• •

817
Otras posibilidades

h y H : alturas (h<H)

B : área de la base.

Tronco de cilindro de revolución, con


dos bases elípticas.
área de la sección recta : n r2

( Si g 1 = 0, se trata de una cuña ci­


lindrica ).

Algunos “desarrollos” de las superficies laterales de troncos de cilindro, son

(a)
t

T
G

g
X
2rcr
1) Un cilindro está lleno de agua hasta la mitad. Se suelta un pedazo metálico y el nivel del agua
sube en 3,5 cm. Si el diámetro del cilindro es 8 cm. ¿Cuál es el volumen del pedazo?

A ) 1 7 5 cm3 B) Faltan datos C) 88 cm3 E) 0,226 t.

Solución
La variación es debida al trozo metá­
lico, y su volumen es : • *
variación
82 ) ( 3 # 5 ) =
( * . _ 175

Rpta: (A)
Trozo
metálico
2) ABCD, es un rectángulo. Se traza BH _L A C . Si V, y V2, son los volúmenes oe los sólidos
obtenidos al girar la región triangular ABCD, alrededor de AB y B C , respectivamente.
V1 AH
H a lla r: , s i:
HC 25

2 25 2
A ,f C) D)
25 E>3
819
Solución
Con el gráfico

Alrededor de AB :
V1 = k ( BC )2 . AB

Alrededor de B C :
V2 = k ( AB )2 . BC

Luego :

^ _ ; t ( B C ) . AB
Vo AB (I)-
V2 ti( A B )2 . BC

Por otro lado, en el A ABC, según relación métrica:

( BC f HC
(AB) 2 AH

(BC] 25 BC 5
Con el dato :
( AB ): AB 2

V1 _ 5
Reemplazando esto último, en ( I ) :

Rpta: ( A ).

AB y CD , son generatrices opuestas de un cilindro circular recto y O punto medio de B C .


Siendo E un punto de C D , tal que OE 1 A E , CE = 8 cm. y ED = 9 cm. Hallar el área total
del sólido.

A) 138 Ticm2 B) 207 ncm2 C) 206 ncm2 D) 204 ncm2 E) 276 ncm2

Solución
Sea r el radio de la base:
r 8
A O C E ~ A EDA- * g = ^

r = 6

El área total:

ST = SL + 2?tr2

ST = 27tr ( CD ) + 2711^ =
=2 ti x 6 ( 1 7 ) + 2 tt x 62 .*. ST = 276 TC

Rpta: (E ).
V

4) En un vaso que tiene la forma de un cilindro recto de revolución, la altura es el doble del
diámetro de la base. Si el vaso contiene un líquido que ocupa las 3/4 partes de su capacidad,
determinar el ángulo que debe inclinarse desde su posición normal hasta el instante en que
el líquido esté por derramarse.

A) 30 B) 37 C) 45 D) 53 E) 60

Solución
* Incógnita : a

* Llamando V el volumen del vaso; el

líquido ocupa los —V.

V
Luego, el volumen vacío será — y

corresponde a la cuña ABC.

Al trazar AF D E , formando el cilin­


dro ABCF, su volumen será el doble
V V
de ABC, es d e c ir: 2 . — = —

BD
Como el volumen de ABCF es la mitad del total: AB =

Luego, en el A ABC : AB = BC a = 45

Rpta: ( C ).

5) El desarrollo de la superficie lateral de un cilindro oblicuo, de bases elípticas, es un rombo


de diagonales 12cm. y 16cm., respectivamente. Hallar el volumen del sólido.

1152 ti 3 576 tc 3 100 3 1152 3


A) 1152 7c cm3 B) — - — c rrf C) — r — cor* D) cmJ E) “T — cm
5 5 TC

Solución
- Volumen : V = tc r2 . AB

B B’
Sección recta
(círculo de radio r)

A'B=12
AB’=16
(desarrollo de la
superficie lateral)
A’

821
- En el A B M B ', del desarrollo : BB’ = 10
Luego, por ser rombo : AB = BB’ AB = 10 ( Longitud de la generatriz )

A’H, es igual a la longitud de la circunferencia de una seción recta. En el A ABA’ :


AB x A’H = A’B x AM

48
10 x A’H = 1 2 x 8 -+ A’H =

48 24
- P e ro : A’H = 2nr —> 2rcr =
5n

- Finalmente, el volumen: V = rcr2 . AB

24 w 1152 3
V = 71 x 10 -+ V = cm3
V 5n j 571

Rpta: ( E) .

6) Hallar el volumen de un cilindro oblicuo, de base circular; sabiendo que la generatriz mide
igual que el diámetro de la base y la distancia del centro Q de una de dichas bases , a los
extremos de un diámetro AC de la otra, son 9 y 13 cm. ; respectivamente.

A) 60 k cm3 B) 64 n cm C) 60 7i cm3 D) 72 n ^/’T íc m 3 E) N.A.

Solución
- Si r , es radio de la base: g = 2r.

QO = g = 2r, ( Q y O : ce n tro s).

- En el A AQC, por el teorema de la


mediana:

AC
AQ +Q C = 2Q O +

(2r)
132 + 9a = 2 ( 2 r ) 2 +

r = 5 y AC = 10
Q
Luego, para hallar la longitud de la altu­
ra, h; usamos el teorema de Herón en el
A AQC:

p _ 13 + 9 + 10 _ 1 6 ( semiperímetro )

h= 16(16-13)(16-9)(16-10)

822

12
h = _ z .J l4
5 v
- Entonces, el volumen del cilindro:
V = ti r2 h
10 . .

V = ti x 52 x — cm3 -» V = 6 0 tc/ T 4 cm'

Rpta: ( C ).

7) Hallar el área total de un cilindro de revolución, en el cual la diagonal axial mide 17cm. y la
distancia de un punto de la circunferencia de una base, al centro de la otra, es -y/241 cm.

A) 164 Ticm2 B) 148 rccm2 C) 152 Ttcm2 D) 172 rccm2 E) N.A.

Solución

- S T = 2 7 i r h + 2 7 i r 2 ............( I ).

- Consideremos el gráfico adjunto.


Si AB y CD , son diámetros opues­
tos, se llama diagonal axial del cilin­
dro, a la diagonal del rectángulo ABCD,
AD = 17.

También, por dato : AM = ^ 2 4 1

- Por el Teorema de Pitágoras :


A ACM -> h2 + r2 = 241 . (II).
AACD h2 + 4r2 = 289 (III)

- Restando ( I I I ) - ( I I ) : 3 ^ = 48 r = 4
h = 15

- Reemplazando en ( I ) :

ST = 2n x 4 x 15 + 27t x 42

ST = 152 71 cm2

Rpta: ( C ).

8) AC» es una generatriz del cilindro circular recto de radio r, en la figura adjunta. CB, es
diámetro. E, un punto de AC- Hallar la longitud mínima de la trayectoria ABE sobre la
15
superficie lateral del cilindro, sabiendo que r =
71

AE = 12 y EC = 8
A) 32

B) 52

C) 42

D) 62

E) 36

Solución A’
Se pide el valor mínimo de la curva
12
ABE.
20 E
En la figura adjunta, la región rectan­
gular ACC'A’, es desarrollo de la su­ 8
perficie lateral del cilindro. ri \ / r
15 B 15 C’
Se tiene: CC’ = longitud de la circun­
ferencia de la base.
15
CC’ = 2 7t r = 2 ti x — =30
n
CB = BC' = 15

La trayectoria curva AB, sobre la superficie del cilindro, equivale a la longitud del segmento
AB, en el desarrollo.

La trayectoria curva BE, equivale a la longitud del BE.

A ACB ( AB )2 = 152 + 202 AB = 25 AB + BE = 42, es la longi­


tud de la mínima trayectoria
ABC* E ( B E )2 = 1 5 2 + 8 2 BE = 17 curva ABE .

Rpta: ( C ).

Hallar el volumen del cilindro circular


recto de la figura. AB y CD son diá­
metros. AB 1 CD. La sección DBC
tiene área 32rc cm2. y forma un diedro
de 60° con la base.

A) 128 7 t / 6 c m 3 B )1 2 8 jt/3 c m 3

C) 1 2 8 /2 cm3 D) 128 7t ^ 2 crrv

E) 128 7i cm3
Solución
Se tiene: AREA DEL ' AREA DE LA
SEMICIRCULO CDH x Cos 60
SECCION DBC

AREA DEL 2 ^
32 je cm x —
SEMICIRCULO CDH

tcOH
= 1671 OH = 4 / 2

En el A BHO: BH = ( OH ) / 3 BH = 4 / ?

Entonces, el volumen del cilindro:

V = Jt ( OH ) . BH

V = k (4 /2 f .4 /6

V = 128 tc / 6 cm3
Rpta: ( A ).

Un cilindro oblicuo está circunscrito a una esfera de radio “ r ” y las generatrices, inclinadas
60°, respecto al plano de la base.
H a lla r: ( a ) El área lateral del cilindro.
( b ) El área de cada base.
( c ) El volumen

Solución
La sección recta , es un círculo de
radio r , igual al de la esfera. ( Un
círculo máximo de la esfera ).

La longitud de la altura del cilindro:

En la sección ABCD:
F
ACHD CD = . ( CH) .

/ 3

CD = (2r)
/3
D
f longitud de la
generatriz >

825
r
- Area L a te ra l: SL = 2 n r ( C D ) = 27tr 4 ^ 871 r2 J 3 Rpta
- 3

- V o lu m e n : V = 7cr2 ( C H ) = 7 c r 2( 2 r ) -> V = 2 ^ ^ Rpta.

AREA EN BASES: El área de una base se encuentra a s í :


área de la sección recta = ( área de la base ) cos 30°

rc r2 = ( área de la base ) x —»

11) En un tronco de cilindro circular recto, la generatriz mínima es nula y los planos de las bases
forman un diedro de 60°.
Hallar el volumen del sólido, sabiendo que la suma de las áreas de las bases es 487icm2.

A) 48 ti cm3 B) 128 ti cm3 C) 64 tc cm3 D) 128 ti cm3 E) N.A.

Solución
- Consideremos el gráfico adjunto:
EC Generatriz máxima.

m Z EAC = 60°

- El volumen pedido:
V = 7i r2 . E C ............ (I)

Si r es el radio de la base ;
en el A ACE, se tiene:
EC = ( AC ) / 3

EC = 2 r / 3 . . . ( I I ) .

- Area de la base c irc u la r: B- = 7t r2


- Para hallar el área de la región elíptica:

B1 = B2 . cos 60°

tc r2 = B2 x j B2 = 2 k r2

Por dato: B1 + B2 = 48 ti cm2


-» rcr2 + 2 n r 2 = 487tcm2
r = 4 cm.
- Entonces, en ( I I ) : EC = 8.y~3 cm

- Y, en ( I ) : V = 7i(4cm)2 . 8<J~3 cm

Rpta: ( D ).
826
Hallar el volumen del tronco de cilin­
dro circular re c to ;
si AB = 40 ; CD = 20, m Z ABC = 45°
y m Z BHA = 90° .
( P y Q: Centros ).

A) 720 Jt

B) 750 ti
> H

C) 780 tí
D
D) 8 1 0 ti

E) 700 7t
Solución
Del g rá fic o :

A AHB HM
45

1 ACHD HN
20

MN = H M - H N
MN = 2 0 - 10
MN = 10

- El volumen del tronco de cilindro :


AB + CD
V = ( área de la sección recta ) .
\
o ( AB + CD ^ c2 f 40 + 20 ^
V = k r I ------ I= ti x 5*1 -----------

V = 750 7t
Rpta: ( B ).

Hallar el volumen del tronco de cilin­


dro circular recto circunscrito a la es­
fera de radio r, sabiendo que el eje AB
de la elipse , forma un ángulo de 45°
gon la generatriz máxima B C .

A) 7t r» / 2 B) 2it r3 ( / I + 1)

C) 2n r3 D) n r3 ( / ¥ + 1)

E )*|3 (/2 -1)


Solución
- El radio de la base, es igual al de la esfera : r

- El volumen del tronco, se evalúa:


AD + BC N
V = (área de la base)
/

V = ( ixr2) -.(i).
/

En la sección ABCD , por el teorema de


Pitoth :
AD + BC = AB + CD

AD + BC = 2r J~2 + 2 r .. ( I I ).

Reemplaz. ( I I ) en ( I ):

' 2 r / 2 + 2r '
v.(„=) V = Jtr3 ( J~2 + i )
v j
Rpta: ( D ).

14) Hallar el volumen de la “cuña cilindrica”


^ B circunscrita a la esfera de radio r, siendo
AB y AC ejes de las elipses ; AB = AC y
AB ± AC.

A) 7ir3 ( 5 / 2 + 7 ) B) 7tr3 ( 5 + / 7 )

C ,H Í(5 ^ +^ ) D)í £ ( 5>f 2 - 7 )

E ) ^ ( 5 / 2 + 7)

Solución
B I
- En la sección BAC:

AH = r ( J 2 + 1 ) ;

BC l_l generatriz
BC = 2 r ( / 2 + 1 ) y AH = máxima

AH
- El radio de la sección recta, es :
±
- Luego, el volumen :
828
' O + BC
V = ( área de la sección recta ) .
\

1
íl B2C1J

<
X
V = 7C —

^ 2 J _

/
/ 2 + l f 2 r(V 2 + l)
V = kr
V /

Efectuando:

V -íll(5 /2 +7)

Rpta: ( E )
CAPITULO 2 9

CONO Y TRONCO DE CONO


S uperficie Cónica. Es la superficie que se genera, al deslizarse una recta, ( generatriz), a
lo largo de una curva (directriz), pasando siempre por un punto fijo del espacio.

Figura 1 Figura 2
La figura 1, muestra una superficie cónica abierta. La directriz C , , es una línea y V es la
generatriz.

La figura 2 , muestra una superficie cónica cerrada , debido a que la directriz C2 es una
curva cerrada.

C O N O .- Es el s ó lid o o b t e n i d o al
interpceptar una superficie cónica cerrada,
mediante ün plano.
La figura muestra un cono. El punto uO”
es vértice, “B" el área de la base y “h”,
longitud de la altura.
La base puede ser cualquier curva cerra­
da.
El volumen V, se expresa a s í :
CONO DE REVOLUCION.- Se genera, al girar la región correspondiente a un triángulo
rectángulo, una vuelta, alrededor de un eje que contiene a un cateto.
El otro cateto genera la base ( círcu lo ) y la
hipotenusa ( generatriz) , genera la super­
ficie lateral.

Area lateral : SL = n r g |

Area Total : ST = SL + B I ;

( En este caso : B = k r2 I )

D esarrollo de la S uperficie Lateral.- Es un sector circular que tiene por radio la generatriz
del cono y por arco, la longitud de la circunferencia de la base del cono:

Cono de o
'evolución

Se verifica que: 2 n r = 2 n g . a
v 360 j

PROPIEDADES

Si se intercepta la superficie lateral de


un cono, mediante un plano paralelo a
la base, se obtiene otro cono parcial,
semejante al original.

En este caso, se cumple que :

a) Las áreas son entre sí como los cua­


drados de las alturas o como el cua­
drado de las generatrices. Así: (Fig. 3)

S, y S2, pueden ser áreas laterales, tota


les o áreas de las bases de los conos.
832
Los volúmenes son entre sí como los cubos de las alturas o como el cubo de las generatrices
( Fig. 3 )

V, _ ( h, )3 _ ( OE )3
V ( h2 )3 ( OA )3

TR O N C O DE CO NO

Fig. 1 Cono
deficiente
Se obtiene al interceptar la superficie lateral
de un cono, con un plano cualquiera, para­
lelo o no a la base.

TRONCO DE CONO DE REVOLUCION

Se genera al girar, una vuelta, la re­


gión correspondiente a un trapecio
rectángulo, alrededor del eje que con­
tiene al lado perpendicular a las ba­
ses.
Las bases son círculos y la altura del
trapecio es altura del tronco.
Fig. 2
Para la figura 2 :
9 -> Longitud de la generatriz,
h -> longitud de la altura,
R y r -> radios de las bases (círculos)

rch
Volumen V = — ( r2 + R2 + R r )

Area lateral : SL = 71 ( R + r ) g

Area Total : ST = SL + rcr2 + rtR2|

El “ desarrollo" de la superficie de un tronco de cono de revolución , es un trapecio


circular.

833
PROBLEMAS RESUELTOS

1) La superficie lateral de un cono de revolución se intersecta por un plano paralelo a la base,


determinando un cono parcial. Si las áreas laterales del cono parcial y tronco de cono, son
entre sí como 4 es a 5 ; hallar la relación de volúmenes del cono parcial al cono total.

A)
64
C)
8
D)
11
125 27 25

Solución
Por dato:
^Cono Parcial 4
^Tronco de Cono 5

Scono parcial _ 4
^Cono Total 9

Por propiedad:

^Cono Parcial
^Cono Total H

- Entonces, la relación de volúmenes:

VCono Parcial 8
VCono Total / 27

Rpta: (C)
H34
2) Dado un cono de revolución, de vértice E, y volumen 54 cm3., se traza un diámetro AC en
el círculo de la base. Hallar el volumen del tronco de cono que se determina al trazar un plano
paralelo a la base, por el baricentro de la región triangular AEC.

A) 19 cm3. B ) 1 9 7tcm3 C) 36 cm3. D) 38 cm3 E) 36 cm 3

Solución
- V
v TRONCO
= •?

V
v CONO TOTAL
= 54 cm3

* Como G, es baricentro del


h 2
AAEC: H = 3

* Se tendrá :
V
y TRONCO
= V
v CONO TOTAL
- vVCONO PARCIAL

V
v TRONCO
- '■54rm
>‘+ ^ "1
3 - vVCONO PARCIAL (I)

* Por otro lado, de la semejanza entre los conos :


VCono Parcial VCono parcial _ ( 2 \3
—»
VCono Total 54 cm3 \ 3 /

v cono parcial = 16 cm3. Reemplazando en ( I )

V tronco = 3 8 C fT l3

Rpta: ( D ).

3) Un cono de revolución, se llama equilátero, si la generatriz mide igual que el diámetro de


la base. Hallar el volumen de un cono equilátero, conociendo el radio r de la esfera inscrita
en él.

A) 2 7i r3 B) ti r3 CJnr3 / 3 D)3rtr3 E) 9 tc r3

Solución
- Sea el cono de la figura :
AE = EC = AC
A AEC, equilátero

m Z ACE = 60°

y m Z MCO = m Z OCE = 30

- El volumen :

V = ^ x n( MC )2 x EM
835
V = ^ x n ( r / 3 )2 x ( 3 r )

• *

Rpta: ( D ).

4) Sobre la superficie lateral de un cono de revolución se toma un punto distante 6 , 1 6 y 10


cm. de la altura, la base y el vértice , respectivamente. Hallar el área total del cono.

A) 854 k cm2 B) 324 it cm2 C) 864 ti cm2 D) 824 n cm2 E) 844 k cm2

Solución
- Sea P el punto en mención.
Datos: PF = 6 , PQ = 16 y PE = 10

En el A EFP: EF = PE - PF
EF = 8

- A EOC - A E F P :
OC EO R 24
R = 18
FP " EF 6 8

También:
EC EO JL 24
g = 30
EP EF 10 8
- Luego, el área total
ST = ji R2 + n R g

ST = 7 tx1 8 2 + 7tx18x30
ST = 864 k cm2
Rpta: (C).

5) La figura, muestra un cilindro circular recto, inscrito en un cono de revolución. E! cono parcial,
de vértice E , y el cilindro , son equivalentes.
¿Qué fracción del volumen del cono total, es el volumen del cono parcial?
25
A)
64
27
B)
64
9
C)
16
36
D)
125
1
E)
4

836
Solución
- Sean : —> Volumen del cono total.

<
H
o
VCP —» Volumen del cono parcial


a.
- Se pide
= ?
A

9

' vCT

I : V CONO — r2 h = n r 2 ( H - h)
PARCIAL 3 v ;

h 3
H 4
3 3

' h ^ ' 3 ' 27


—> VCP
CL

- Luego :
II

VCT 1v H J ^4 , VCT 64

Rpta: ( B ).

6) Hallar el volumen de un cono de revolución, conociendo los radios R y r, de la base y de


la esfera inscrita, respectivamente.

Solución
1
V = — x 7t R 2h = ?
O

* Para hallar h :

A MTE - A COE

EM TM h- r r
EC “ OC VR2 + h R

2rR
h=
(R 2 - r 2 )

- Entonces :

V = 7" x 71 R2 h
O

Rpta:

7) Hallar el área lateral de un tronco de cono de revolución, circunscrito a una esfera. La


generatriz del tronco mide 4 cm.

A) 4 7i cm2 B) 2 7i cm2 C) 8 7 ic m 2 D) 16 cm E) 16 7i cm2

837
V

Solución
- El área lateral del tronco de cono tiene
fórmula:
SL = J i ( r + R ) g (I).

- En la sección ABCD, por el Teorema


de Pitoth:
BC + AD = AB + CD

2r + 2R = 4cm + 4c m

Luego: r + R = 4 cm.

- Reemplazando, en ( I ) :

SL = 7t ( 4cm ) ( 4cm ) SL = 16 tc cm2

Rpta: (E).

8) El desarrollo de la superficie lateral de un cono de revolución, es un sector circular de radio


15 cm. y arco de medida 216o. Hallar el volumen del cono original.

A) 328 k cm3 B) 326 n cm3 C)364rccm 3 D )3 5 4 rccm 3 E) 324 7ccm3


Solución
La generatriz del cono mide igual que el
radio del sector en el desarrollo de la
A' superficie lateral.
g = 15

216
- l a ¿ a ' = 2 K g x

360 /
/
271 r = 2 k x 15 x 216 ‘

V
360 /
r = 9

- En el cono : h2 = g2 - r 2 = 1 5 2 - 9 2 h = 12

- Finalmente, el volumen V del cono:

V = — x ji x r2 x h = | x kx g2 x 12 V = 324 n cm3
O O

Rpta: ( E ).

9) El desarrollo de la superficie lateral de un tronco de cono de revolución, es un trapecio circular


de área 30 n cm2. Hallar el volumen del tronco de cono, si la altura y la generatriz miden 3
y 5 cm., respectivamente.

A) 31 tí cm3 B) 30 rc cm C) 27 n cm3 D) 33 ti cm E) 32 rc cm:


838
Solución
V = ?

Area L a te ra l:
n ( R + r ) g = 30 ti

tu ( R + r ) 5 = 30

R+r = 6 (I)

En el A EFC : EC = 5 y
"desarrollo" de la
EF = 3 -> FC = 4
superficie lateral
R - r = 4 ............( II ).

R = 5 y r = 1, de ( I ) y ( I I ).

n
El volumen : V = - h ( R2 + r U Rr).
ó
K
V = - x 3 ( 5 2+12+ 5 x 1 )

V = 31 ti cm3

Rpta: ( A ).

Hallar el volumen de un tronco de cono de revolución, sabiendo que los radios de las bases,
miden 8 y 12 cm., respectivamente y que el área de la superficie lateral es igual a la suma
de áreas de las bases.

71 71
A) 3264 - r cm3 B) 3268 — cm 3 C) 3624 - cm D) 2624 ^ cm3 E) N.A
b

Solución
52
Dato : k ( 8 + 12 )g = 7t ( 82 + 122 ) g =

Sean O y M, centros de las bases. Se


traza ME, paralelo a la generatriz NF.

AEOM:

h2 = g2 - OE
/ 2
52 N
h2 = - 42
\ 5 j
48
h =

Luego, el volumen del tronco :


71 7t 48
V = - x h [ R2 + r2 + Rr ] = - x — [ 82 + 122 + 8 x 12 ]
3 3 5
q ^Q
V = 3264 ^5 cm3
Rpta: ( A ).

11) Hallar el volumen de un tronco de cono de revolución, cuyas bases tienen radios 4 y 9cm.,
respectivamente. El área total del cono, es 266 7c cm2.

A) 532xtcm3 B) 523 tí cm3 C)352rccm3 D) 235 7tcm3 E) 502 7tcm3

Solución
4 J
- V = ?; r = 4 y R = 9

- Area L a te ra l:
S. = k ( R + r )g

- Area Total :
ST = SL +- k r2 + 7t R2

- Por dato :
ST = 266 rccm2

7t ( R + r ) g + n r2 + rc R2 = 266 rt

7r(9 + 4 ) g + 7: x 42 + ti x 92 = 266 xc -■> g = 13

- Luego, en el A AHB : h2 = g2 - 52 = 132 - 52 -> h = 12

- Finalmente, el volumen

V = -- x h ( R2 + r2 + R r )
u

V = -£ x 12 ( 92 + 42 + 9 x 4 )

V = 532 7c cm 3
Rpta: ( A ).

12) Hallar el área de la superficie del sólido que se obtiene al girar 360a, la región poligonal
ABCD, alrededor del diámetro AD.
t N

AB s BC = CD
A) 2 n r2 B) ti r2.y~3 C) 3 ti r
r
D) 3 ?t r2/ ! E) 2 ti r2 / 3 r = radio

oi .
Solución
\
- Como ABCD, es un semihexágono cv
regular: m AB = r = m BC = m C D . D
Se obtienen dos conos congruentes
entre sí y un cilindro de revolución.

Para cada cono :


^ la t e r a l = k BH x AB
CONO

SLATERAL x r = — r2 J*3
CONO 2 v

Para el cilindro :

■la t e r a l ~ 2jcBH x BC — 2 tt x —^ 3 x r — kt ^ 3
CILINDRO 2

/ \
Luego : S to ta l - 2 ^ la te r a l + S l a t e r a l - 2 ^ -r2 ^¡~3 i + Ttr V 3
l CONO y CILINDRO

S TOTAL = ^2 TTr2/ 3
u 1 *

Rpta.: ( E )

Un trapecio isósceles con su ángulo agudo de 60° gira alrededor del eje que pasa por uno
de sus lados no paralelos. Hallar el volumen del cuerpo de revolución, si las bases del
trapecio miden 6 y 20, además los lados no paralelos miden 14.

A) 1992 k B) 1900 ti C) 2000 ti D) 1946 7t E) 1000 7t

Solución

Sea ABCD. el trapecio.


BC = 6 ; AD = 20 y A B = CD = 14

VABCD, es el volumen del sólido que


genera el trapecio.
¿\D
A
Su valor se calculará así: eje >

^ A B C D ~ ^ABH O
- V’ BHC
„..„ ; ó también: VABCD = VABHE + VAED - VBHC

ABHE , genera un tronco de cono de radios en las bases AE y BH ; altura HE.

AED y BHC, generan conos de radios en las bases también AE y BH respectivamente.

HE
Luego :VABCD = k BH + AE + BH . AE + ! ( AE2 ) ( E D ) - | Í B H 2 |( CH )
l 3

Reemplazando valores :

V ABCD - 71^
(3 / 3 ) + [ 10/ 3 ) + ( 3 / 3 ] ( i o / 3 ) + —[ 10/ 3 ) . 1 0 - —(3 / 3 ) . 3

841
De donde:
V ABCD = 1946 7t

Rpta: ( D ).

14) Se dan dos esferas tangentes exteriormente y cuyos radios miden 1 dm. y 3 dm. El volumen
del cono recto circunscrito a ambas esferas, es:

A) 81 dm3 B) 18 ti dm3 C) 18 dm3 D) 81 ti dm3 E) N . A

Solución
* Volumen V = ?

C álculo de la altura h :
ABLP - ABTM
BP PL
BM MT
h- 7 _ 2 BP = 2
h= 9 y
h- 3 ~ 3 BL = ■/?

C álculo del radio r ( de la base )


OP BO
A BOC - A BLP PL ~ BL 1 = 3 /3

1 o 1 i— o
El volumen :V = — 7c r2 h = “ te ( 3 ^ 3 )2 ( 9 )

Rpta: ( D ).

15) El volumen de un cono de revolución, es 36 rc cm3. ABC, es un triángulo equilátero inscrito


en la circunferencia de la base del cono. El A ABC, está circunscrito a la vez a una
circunferencia cuyo círculo es base de un cilindro recto inscrito en el cono. Hallar el volumen
del cilindro.

A) 27 rc cm B) 18 rc cm3 C) k cm3 D) 16 7i cm3 E) 12 7t cm3


2
Solución
- Considerando el gráfico
Vcono = 36 rc cm3
1
- x It Ra H = 36 71cm3
u

R2 H = 108 cm3 ............( I ) . /


/

- Volumen del cilindro :


i
V * dro = ..........( " ) .
842
R
Pero , r = — • • En el A EOD,

OD
MF es base media de OD :M F =

EO
Entonces: OM = h=—
2

( R > í H
En ( I ) : Vcilindro K é
l 2 , l 2 )

Vcilindro 4 x n R2 H
8

rc
Con lo de ( I ) : V cilindro — x 108cm3
8
27 3
Vcilindro — n cm (En el plano de la base)

Rpta: ( C) .

Los volúmenes de los sólidos generados por la rotación de un triángulo rectángulo, alrededor
de los catetos e hipotenusa, son Vv V2 y V3, respectivamente.
Demostrar, que:
? / \? / \

1l +
1 1 '
l V3
1
J
Solución
Consideremos el A ABC, recto en B

843
Se tienen

* Alrededor de BC : V1 = ^ ( AB )2 . ( BC ) ..........(1 )

K
* Alrededor de AB : V2 = - ( BC )2 . A B ( 2 ).
3

* Alrededor de AC ( dos conos de radio común BH )

V3 = ~ ( BH )2 . AH + ^ . ( BH )2 . HC
O J

ó : v3 = ^ ( BH f . ( AH + HC ) v 3 = I * ( BH )2 . AC (3 )

.1 -1 1
De ( 1 ) y ( 2 ) : Vl n ‘ ( AB )2 . BC

1 1
V2 * ( BC ) . AB
Luego :
/ \2 2 2
1 1 ' 1 ' 9 1 i 9 1
+
U , , V2 , TC2 ( AB f . BC TI2 ( BC )2 . AB

Factorizando :
/ \2 f( 1* ^\2 9 1 1 1
í 1 - +
.. ■ A + ------
, V1 > ^2) K ( A B . BC ) ( AB f ( BC f
- Pero, por relaciones métricas, en el triángulo rectángulo ABC :
1 1 1
AB . BC = AC . BH y + ------
( AB f ( BC f ( BH f

- Reemplazando en la anterior expresión :

1 1
+
V. / V TC ( AC . BH ) (BH)

Es d e c ir:
-i2 \2
_l_ 1 1 1
+ +
V Vo
v2 J 7t ( B H ) . AC \ V1 ) V / n ( BH ) . AC

Finalmente, con lo de (3):


/ S 2 ✓ 2\

1 ^
1
+
[v2,
íiv3
1 J1 I.q.q.d
i Vi,
844
V

ESFERA Y TEOREMAS DE
PAPPUS-GULDIM
AREA DE LA ESFERA
La superficie de la esfera es el lugar geométrico de los puntos que equidistan de otro
punto llamado centro.

TEOREMA:
El área engendrada por un segm ento de recta AB que gira alrededor de un eje,
situado en el mismo plano, sin cortarse, es igual al producto de la proyección EF del
segmento sobre el eje, por la longitud de circunferencia que tiene por radio la porción de
mediatriz del segmento, interceptada por el eje.

x AB y xy están situados en un mismo plano.


Eje
Sea:
£ > á b = área generada por la rotación de

AB.

EF = proyección de AB sobre xy.

ML = mediatriz de A B .
Luego:
S AB = s L. Tronco de cono.

= tc(AE + BF)AB = 2 k MN . AB

Como:AMNL - AAHB
H

AH AB
y

845
TEOREMA
El área engendrada por una p o lig o n a l re g u la r que gira alrededor de un eje situado en
el mismo plano, que pasa por el centro sin cortarla, es igual al producto de la longitud de la
circunferencia que tiene por radio el apotema de la poligonal, por la proyección de la poligonal
sobre el eje. ( TEOREMA DE ARQUIMIDES ).

SABCD = área generada por la rotaión de la poligonal ABCD.

^ abcd = ^ áb + ®§c + ®CD

ABCD
= 2 71 ap . EL + 2 7i ap . U + 2 n ap . JF

ABCD = 2 71 ap ( EL + LJ + JF )
Eje
S abcd = 271 a P • EF

c;

ZONA ESFERICA :
Es la porción de superficie esférica limitada por dos planos paralelos. Los planos
determinan dos circunferencias que son las bases de la zona, la distancia entre los planos
es la altura de la zona.

TEOREMA.-
El área de una zona es igual al producto de la circunferencia máxima, por la altura de
la zona.

846
La zona, se puede considerar generada por la rotación del arco AB alrededor del
diámetro.
Si en el arco AB se inscribe una poligonal regular cuyo número de lados aumenta
indefinidamente, e n to n ce s:
ap R

De acuerdo al teorema anterior : ®AB “ ^zona “ •h

Casquete E

Si la zona tiene una base, se llama CASQUETE ESFERICO

^C asq = ^ 7t R.h

En el A ABL : AB = 2 R.h ® C asq - 71 AB

TEOREMA
El área de una esfera es igual al producto de la circunferencia máxima por el diámetro.
En efecto* la su p e rficie de la esfera se puede considerar generada por la rotación de
una semicircunferencia que gira alrededor de su diámetro.

Como: h = 2R SESF = 2 n R( 2R ) SEs f ~ 4 71 R2 \

HUSO ESFERICO

Es la porción de superficie de la
esfera, c o m p r e n d i d a entre dos
semicircunferencias máximas que tie­
nen un diámetro común.

Si la semicircunferencia EAF gira 360°


alrededor de E F , se genera la super­
ficie de la esfera.

Luego:
847
%

360° -> 4 jcR2


a HUSO

7t R¿
HUSO
a
90°

VOLUMEN DE LA ESFERA
TEOREMA:
El volumen dei sólido engendrado por un triángulo que gira alrededor de un eje que
pasa por un vértice, sin cortar al triángulo y situado en el mismo plano, es igual a la tercera
parte del producto del área generada por el lado opuesto al vértice situado sobre el eje, por
la altura relativa a este lado.

1er. Caso: El eje xy se contunde con uno de los lados.


El volumen generado por el triángulo
ABC, es la suma de los volúmenes de
los conos de revolución generados
por los triángulos rectángulos ADB y
BDC.
A
Sea : VABC = Volumen generado por
el A'A B C

1
V A = — jiBD
1
. AD + — rcBD . DC *
ABC 3 3

B
Vabc = - r e B D 2 (AD + DC) = - tc B D 2.AC
3 ' 3

Pero : 2 S ABC = AC BD = BC . h
1
V A = — jiBD . BC . h
ABC 3

y como: 7t BD. BC = área lateral del cono, BC


generado por B C .

Luego:

2do. Caso: El triángulo sólo tiene un vértice común con el eje.


Prolongando BC hasta D sobre xy.

=V - V - 1 h _ l q _ .h
V a A “ 3 • BD ■n 3 • DCD
ABC ABO ACD

848
v aA - « ( SBD SCD )h
ABC

TEOREMA
El volumen engendrado por un sector poligonal regular que gira alrededor de un eje que
pasa por el centro, sin cortarlo, es igual a la tercera parte del producto del área generada por
la poligonal regular, por el apotema ( h ).

Sea:
VABCDo = Volumen generado por
el sector poligonal regu­
lar ABCDO.

VABCDO - V A + V A + V A
AOB BOC COD

VABCDO 3 \ AB + ^BC + S CD ) h

SECTOR ESFERICO.-
Es el volumen engendrado por un sector circular que gira alrededor de un diámetro
situado en el mismo plano y que no lo corta.

TEOREMA:
El volumen de un sector esférico es igual a la tercera parte del producto del área de la
zona correspondiente, por el radio de la esfera.
Si a la poligonal inscrita en el arco AB se aumenta indefinidamente su número de lados:

ap -> R
Y:

VSect. = T SAB • R
Est. ¿

849
= —( 2rcR . h )R
3 }

^Sect. = —
o rcR2 h
Esf. ¿

TEOREMA:
El volumen de la esfera, es igual a la tercera parte de su área por el radio.

El volumen de la esfera es igual al volumen de un sector esférico engendrado por la


rotación de un semicírculo que gira alrededor del diámetro : h = 2R

Vesf = 3 * R 2 ( 2 R ) -> VEsf = — . rcR


3

CUÑA ESFERICA:

Es la porción de volumen de la esfera limitada por dos semicírculos máximos que tienen
el mismo diámetro.
Si el semicírculo EAF gira 360° alre­
dedor de EF, se engendra el volumen
de la esfera.

Luego:

360° ->
? * rj
a V
Cuña

ANILLO ESFERICO:
Es el sólido generado por la rotación de un segmento circular que gira alrededor de un
diámetro exterior.

TEOREMA:
El volumen de un anillo esférico es igual a la sexta parte del volumen de un cilindro que
tiene por radio la cuerda del segmento y por altura la proyección de esta cuerda sobre el
diámetro.

El volumen del anillo es igual a la diferencia del volumen del sector esférico OBM menos
el volumen generado por el triángulo AOB.
850
VAn i l l o = ! * R 2 h - 4 s A B - O L

VAn¡l|0 = | ; t R 2 h - ± ( 2 * O L . h ) O L

^Anillo ~ 2 n ^ R2 - O L 2 1 = —Tth AL2


V > 3
( ----2 >
AB 1
—> ^Anillo ■" ^ 7t AB . h
^Anillo — g ^ 4
V )

Corte de un
anillo esférico
TEOREMA:
SEGMENTO ESFERICO:
Es la porción de volumen de la esfera comprendida entre dos planos paralelos

TEOREMA:

El volumen de un segmento esférico es igual al volumen de una esfera que tiene por
diámetro la altura del segmento, más el volumen de un cilindro de igual altura y que tiene por
base la semisuma de las bases del segmento.

El volumen del segmento esférico, es igual al volumen del anillo AMB, más el volumen
del tronco de cono generado por el trapecio AEFB.

851
Segmento
esférico
de una
base

1 7ih
VSEG.E - h+ 3 a2 + b2 + ab

K 7t h
v s e g .e = ^ t h2 + (a * b)2] h + " y [ a2 + b2 + ab ]

VSEG e = 3a2 + 3b2 j


6 L
Fórmula del seg.
a6- +
"( >2 ) esférico de 2 ba­
VSEG. E “ TT7^ 3 + .h
6 ses.

Si el segmento esférico tiene una base:

V = — 71 h3 + 71f h ó V = — 7i h2 ( 3R - h ).
o 2 3

TEOREMAS DE PAPPUS - GULDIM:


1 El área generada p o r una fig u ra que gira alrededor de un eje coplanar y exterior, es igual
al producto de la longitud o perímetro de la figura, por la longitud de la circunferencia que
describe su centro de gravedad.

Eje

Corte de la superficie
obtenida

852
Sea:
C = perímetro de la figura que va a girar.
S = área generada por la rotación de la figura C.

X = distancia del centro de gravedad de C, al eje.

S = C. 2 k X

El volum en engendrado por la rotación de una figura que gira alrededor de un eje coplanar
y exterior, es igual al producto del área de la figura, por la longitud de la circunferencia que
describe su centro de gravedad.
Sea:
A = Area de la figura que va a girar.

V = Volumen obtenido por ia rotación de A.

X = Distancia del centro de gravedad de A, al eje.

V = A .2kX I

Eje Eje

Corte del volumen


obtenido

853
PROBLEMAS

El lado de un rombo mide “a”; una esfera de radio “RMes tangente a todos los lados del
rombo, la distancia del centro de la esfera al plano del rombo es “b". Hallar el área del rombo.

A) 2 ti V R 2 - b 2 B) R2 - b C ) 2 a ^ R 2 - b2 D) R3 E) b2

Solución
Sea la figura

En el A ABO : r = R2 - b2

Cálculo del área del rombo:


A rombo = a x 2r
Sustituyendo el valor de r :

^ rombo ~ ^
V r2 - b

Rpta: ( C ).

En una esfera de radio “R”, una zona esférica de altura R / 4, es equivalente a un Huso
Hallar el ángulo correspondiente al Huso.

A) 15 B) 30 C) 45 D) 60’ E) 90

Solución
Incógnita: a

Se tiene :
tcR
= 2 tcR
zona

TC R
huso
x a
90'

Areas equivalentes, quiere decir que son iguales :


Por enunciado:
o _ c
huso zona

nPr ttR
Luego : —— .a - —
y 90° 2

De donde : a = 45°

Rpta: ( C ).

3) Hallar el volumen, en m3, de un segmento esférico de una base, cuyo casquete tiene área
40 n m2 y el radio de la esfera mide 10 m.

n K
A) 500 | B) 400 | C) 300 | D ) 200 - E) 100

Solución
- Sea la figura : H

Por dato : Scasquete = 40*

Es d e c ir: 2 n R( HM ) = 40rc

Sustituyendo valor de R y
despejando HM :
HM = 2 ; entonces;

^segmento = 3 *F 12 ( H M ) 3**10* ( 2 ).

rc 3
Vsegmento
, = 400 —
g
nrr

Rpta: ( B ).

4) En una circunferencia de radio R = 2 7~3cm., se tiene una cuerda AB, siendo m AB = 120°.
Hallar el volumen del anillo esférico que se obtiene al girar 360°, el segmento circular AB,
alrededor de un eje diametral paralelo a A B .

A) 12 7i cm3 B) 24 ti cm3 C) 27 ti cm D) 32 7i cm E) 36 7t cm

Solución

R = 2 /3

- A AMO : m ¿ AOM = 60
R
• 1
AM = - / 3

_ 2 V~3
AM =
Luego : h = 6 ya que EF AB

- El volumen :
1
^anillo “ 0 K A® .h

1
= 36 n cm3
Van.,,0 = 6 7t ( 62) (6) V anillo

Rpta: ( E) .

5) La figura muestra una esfera inscrita en un octavo de esfera de radio R


Hallar el radio de la esfera.

A)R(/2 -1)

B>f(V2-l)

D )-|(/2 +1)

E )?(V 3-1)

Solución
- Sea M, el centro de la esfera.

- S, es el punto de tangencia de ambas


superficies esféricas.

- E, G y H, son puntos de tangencia,


de la superficie esférica, con los pla­
nos BOC . AOB y AOC, respectiva­
mente.

- OM es diagonal del cubo formado,


cuyaaristatiene longitud igual al radio
x de la esfera.

- En el plano AOW ; OS = R y MS = x.

- Pero, para el cubo: OM = x

- Entonces : OM + MS = OS

i i

Rpta: ( E)

856
Hallar el área de la superficie del sólido que se genera al girar la figura sombreada, alrededor
del eje diametral C D , si m BC = 120° y r = dato.

1 2
A) j Kr

3 2
s

5 2
C) ¿ K ' 2

9
D )-7 ir¿
2

7 2
E) ñ nr
Solución
Se hacen los trazos que a continua­
ción se indican:

Sean: Ss^ = área del casquete que

genera BC.

^ bc = area de 'a superficie lateral


cónica que genera B C .

La superficie pedida será:

total BC BC

Es d e c ir:
s ,o,ai = 2 7i r ( CH ) + 71 ( BH ) ( BC )

3r
total
= 2 Jtr(

9 2
S = —r c r
. t o t al 2

Rpta: ( D ).

Calcular el volumen del segmento


esférico mostrado, si su altura mide 1 ,
mientras que AB = 2^~2

11n 1071 7 71
A) B) C)

D) 3 7t E) N.A.

857
Solución
Se sabe :

V = (3 R - h ) (1 ) /yj?/y-
3 h=1
' / h~
Donde: h = 1
i \* y x
— *— \
En el A BAC, Por una relación métrica: i

( 2 / 2 )2 = 2 R x 1 ; R = 4
\
\ /
Sustituyendo en (1 ) : \

tr ( 1 ) 2 11 n
V = (3x4-1) V =

Rpta: ( A ).

Se tienen dos esferas concéntricas; se traza un plano secante a la esfera mayor y tangente
a la esfera menor, determinando un círculo de 16itm2. Calcular el área del casquete menor
formado en la esfera mayor sabiendo que el radio de la esfera menor es 3m.

A) 16 Jim2 B) 18 n m: C) 20 n m2 D) 25 k m: E) 36 ti m:

Solución
- Sea la figura:

- Por dato, se conoce el área del círculo


en el plano secante:

A = 7t r2 = 16 rc; de donde:

r = 4

- En el A 0 0 , E : R = 5

h = R- 3 = 5- 3 = 2
Cálculo del área del casquete ( Ax) :

A = (2jcR )h=(2x7rx5)2
A
A¥ = 20
A
ti m2

Rpta: ( C ).

Pintar el piso de un reservorio de agua


que tiene forma de Una semiesfera, cues­
ta $80.
¿Qué costo implicará pintar el exterior, si
el pago por cada m2. de pintura es el
mismo?

A) $80 B) $40 C )$100


D) $160 E) N.A.
Solución
- Como el costo es proporcional al área de la superficie pintada:
área del piso $80

área del casquete

Efectuando la regla de tres simple:

área del casquete


?= x $80
área del piso

2nx
9 - x $80
Ttr J
? = $ 160
Rpta: ( D ).
radio r

10) Una cuerda de radio r = 2m m .se


enrolla fuertemente, obteniéndose
una esfera de radio R = 3cm.
Hallar la longitud de la cuerda,
suponiendo que no hay espacios
libres en el enrollamiento.

A) 8m. B) 10m. C) 6m.

D) 9m. E) 12m.

solución
Si L , es la longitud de la cuerda ( forma cilindrica )
Se tendrá :

Volumen del cilindro = Volumen de la esfera


( cuerda ) ( ovillo )

4R
7c r2 L = — rt R3 -» L =
3rs

4(3cm y
- Reemplazando datos : L =
3( 2 m m )

cm
L = 9 j ; recordando que: 1cm = 10mm :
mm

L = g ( 10mm) = 9000mm -» L = 9m
mm

Rpta: ( D ).

859
11) P, es un punto exterior a una esfera de centro “O”. Se trazan todas las rectas tangentes a
la superficie esférica, desde P, formándose un cono equilátero, cuya base es un círculo
menor de la esfera. Hallar la relación de volúmenes, del cono a la esfera.

9 7 5 1 3

A) 32 B) 32 C ) 32 D) 4 E) 16

Solución

- Por ser el cono equilátero :


A APB, equilátero.

- Sea R, radio de la esfera.


R ¡-T
AOMB MB = — V 3

APMB PM = (MB)>(r3

PM = [ y / 3 )/3 = | r

- Luego :

Vcono f • (MB)2 • PM (MB)2 . PM


^esfera —7tR 3 4R 3
3

í ñ / 3 f . ¿ R Q
Vcono _ V 2 ) 2 _ 9
Vesfera 4 R 3 32

Rpta: ( A ).

12) Se funde una bola de plomo de radio 8cm. para obtener luego bolitas del mismo material, con
radio 1cm. cada una. ¿Cuántas bolitas, como máximo, se obtendrán?

A) 8 B) 16 C) 64 D) 32 E) 128

Solución
- Sean “n” bolitas:
/ =1 cm
© ^ r
- Como los volúmenes, de la esfera
original y la suma de las pequeñas, o
deben ser iguales:
/
o
Vesfera - n V1 ESFERA
MAYOR PEQUEÑA/
"n” bolitas
4 3 R
• • —nR3 = n - r c r n=
3 V 3 j r

860
Reemplazando d a to s:

( 8 c m )'
n= —> n = 64
( 1cm y

Rpta: ( C ).

En la figura:
AB = 30° y BC = 90°
Hallar el área de la superficie que genera el perímetro de la región sombreada, al girar una
vuelta, alrededor del diámetro C D .

tcR
A) (5+2/2)
2

jtR 2
B) (3 + 2 / 2 )
~

D) rt R2

E ) ^ (5+2/2)

Solución
AB, genera una zona esférica.
BC y AC, generan superficies cónicas.

Se observa que BC es lado del cuadrado y AC lado del triángulo equilátero, inscritos

BC = L BC = R / 2

AC = L. AC = R / 3

También se observa que:

r /3 3 R
OB = R ; AH = ; HC = - R y OH = -

3
Luego : = n . AH . AC = — n R2 corte de
2
la
superficie
Sgc = ji . OB . BC = R2 / 2

S20na = = 2 ir R . OH = 2 * R . |

= * R2

861
- Area total de la superficie generada :

S x = S ÁC + S BC + S AB
- Sustituyendo valores hallados :

Sx = + tcR2 +7lR2

Sx = ^ { 5 +2j2)

Rpta: ( A ).

14) Dos planos paralelos, distantes 14cm., determinan sobre una esfera, círculos de áreas
225:tcm2 y 2537ccm2 , respectivamente. Hallar el volumen de la esfera, sabiendo que los
anteriores planos están a ambos lados del centro.

A) 4913-^-cm3 B )4 1 9 3 -|c m 3 C )4 9 3 1 -|c m 3 D) 9 4 1 3 -|cm 3 E )3 9 4 1 -^cm 3


o o o ó ó

Solución
- R radio de la esfera.

Para los círculos :


71a 2 = 225tí az = 225

7ib2 = 253 ti b2 = 253

Con el Teorema de Pitágoras :


AO M E -> R2 = a2 + y2 ....... ( I )

AO H F -> R2 = b2 + (14 - y)2 . . . ( I I )

- Restando miembro a miembro :


O = a2 + y2 - b2 - (14 - y)2

O = 225 + y2 - 253 - (14 - y)2

De donde: y = 8

- Reemplazando en ( I ) : R2 = a2 + y2 = 225 + 64 R = 17

w _ 4 q 3 - 4913 71 cm
^esfera — 3 ^ ~~ 3

Rpta: ( A) .

15) Hallar el volumen de la esfera inscrita en un cilindro circular recto de volumen V.

V V
a > 2 B) C ,fv D)fv E ,|V

862
Solución
El radio de la base del cilindro es igual
al radio de la esfera y la altura igual al
diámetro.
Vciltndro = V ( d a t o ).

Ttr^h = V ti r2 ( 2 r ) = V

V
n r3 = (I)

Para la esfera :

Vesfera

/
V
C o n ( I ) : Vesfera
\

“ El volumen de la esfera es los — del volumen del cilindro ”


O

Rpta: ( C ).

Hallar el radio de una esfera en la cual, dos círculos menores están contenidos en planos que
forman un diedro de 120° y cuyas circunferencias tienen un punto común. Las áreas de los
círculos anteriores, son 9n cm2 y 16tc cm2 .

A) j / ¡ i í B) 2 / T T i C) v m D)-Jrñ E) N.A.

Solución
H
Consideremos el gráfico :
m / ABC = 120°

Para hallar los radios de los círculos


menores:
\2
AB
7t = 1671
l 2 /
AB = 8

tam bién:
/
7t = 9 71
\ /

BC = 6
BC 6 BC (—
En A BHC : BH = —r~ = — = 3 y HC = — <¡3 = 3 ^

863
- En A A H C : AH = AB + BH = 8 + 3 = 11

ÁC2 = AH2 + H C 2 = 112 + ( 3 / 3 )2 -> AC = 2 / 3 7

^ R # 2 í___
- Entonces : QC = R = —* / Í ^
¿ O

Rpta: ( A ).

17) Hallar el volumen del segmento esférico de una base, en una esfera, cuya superficie tiene
área 36 ti m2. El área de la superficie total del segmento, es 11 tc m2.

8 6 7 9
A) 2 Jim 3 B) — Jtm3 C )-Jim 3 D)-rcm 3 E) - n m 3
3 5 3 4

Solución
1 ?
* El volumen del segmento: V 0 = - J t l r ( 3R - h ) ......... ( I ).
O

* Para la superficie esférica : 4% R2 = 36 n R = 3

* La base del segmento, tiene área : 71 a2 y , el casquete : 2tc R h

Por dato : tc a2 + 2 tc R h = 11 tc

a2 + 6h = 11.( I I )

* En el A AMO :

ÁM2 + OM2 = OA2

a2 + ( R - h )2 = R2

a2 + ( 3 - h ) = 9 ......... ( I I I ).

* Restando ( I I ) - ( I I I ):
6h - ( 3 - h )2 = 2
h2 - 12h + 11 = 0 -» h = 1

* Finalmente, en ( I ) :

^ s e g m e n to = J K X 1 ( 3x3- 1 ) -»

Rpta: ( B ).

18) En un cono equilátero de generatriz “L”f hallar el área del casquete menor determinado en
la esfera inscrita, por la superficie lateral.

A> L2 B> * La c> 4 D)€ E)f


864
Solución
Incógnita : Scasquele ( m e n o r)

Como el cono es equilátero :


AB = L
El radio de la esfera :

r = L m3
r^r ; Es d e c ir: r = —V
/3

La altura del casquete : EM = —

EM = ^ ^ 3 ^casquete = 2 71 r . E M

7ÜJ
casquete
12

Rpta: ( D ).

En un recipiente que tiene la forma de un cilindro circular recto de altura igual al radio.se
deposita arena, adoptando ésta la forma de una semiesfera cuyo círculo máximo coincide
con la base del cilindro igual al radio de su base. ¿Qué fracción del volumen del recipiente
no está ocupado?

1 1 2
A) ~2 B> 3 C> 3

Solución

Con el gráfico adjunto , el volumen V desocupado, será

Vx = V cilindro - V semiesfera (i).

siendo : Y = 7t r2 h = n r2 . r

V cilindro = k r^ (II).

V = — K r3
semiesfera 3 (III).

Con ( I I ) y ( I I I ) , en ( I )

V = k r 3 - — 71 r3
3

Vx = ^ ( V cil,ndro)

Rpta: ( C ).

S65
20) Un plano “R” que pasa por una recta tangente en “A”, a una circunferencia máxima de una
semiesfera, forma 15o con el plano “P" que contiene dicho círculo. Calcular la relación entre
las áreas del casquete y la zona esférica determinadas en la semiesfera por un plano UQ" que
pasa paralelo al plano “P" por el extremo “ET del diámetro AB en el círculo determinado por
el plano “R” en la semiesfera.

A) 1 B) 2 C) 3 D)
E> 2

Solución
- Sea la figura ; llamando 4 r al diámetro

- En el A A B C ( 1 5 ° , 7 5 o ) :
AC
BH = — = r

Area del casquete :


A, = [ 2 7 t ( 2 r ) ) ( E F )
A, = 4k r ( r ) ........... ( 1 )

Area de la zona :
A2 = [ 2 TC( 2 r ) ] ( OE )
A2 = 4 tc r ( r ) ........... ( 2 )
Ai
uego. ( 1 ) - ( 2 ) : j = 1

Rpta: ( A ).

21) Las generatrices de un cono circular resto están inclinadas 60° respecto al plano de la base.
El volumen del cono, es V. Hallar el volumen de una semiesfera cuyo círculo máximo está
apoyado en la base del cono y cuya superficie es tangente a la superficie lateral del cono.

A)fv B)§V C)fv D )fv


E» ? V

Solución
- Sea R , radio de la base del cono

A EOC : EO = R / 3

R r~r
A OHC : OH = - V 3

- El volumen del cono :

- R 2 (EO ) = V
3 v

| R 2( R / 3 ) = V

H 6 P
tcR3 = V / 3 ........... ( I )■

- El volumen de la semiesfera : V

Vxx = —
2 — rcOH
\ 3 j

- v « = i' r 3 2 ?

3V
Con lo de ( I ) : Vx = V ^ 3 .

Rpta: ( C ).

22) Calcular el área de una esfera sabiendo que las áreas de los círculos menores paralelos,
distantes 3m. y situados a un mismo lado del centro tienen valores rcm2 y 16rcm2.
A) 44rc m2 B)48rc m C) 52rc m D) 62rcm2 E) 68rc m2

Solución
- Sea la figura :
7üm
- Por dato : A, = rc m2
O sea : 16rcm2

rc r f = rc m2

- También, por dato :


A2 = 16 rc m2

O sea:
rc rf = 16 rc m2 r2 = 4

- Por otro lado, en el A ONF :

R2 = r,2 + í 3 + ,] R2 - r | '

R = fn ( radio de la esfera )

- Cálculo del área de la esfera :

A esf. = 4 rc R2 = 4 rc ( / T 7 )

A esf = 6 8 rc m 2

Rpta: ( E) .

23) Hallar el volumen del sólido engendrado al girar el hexágono regular ABCDEF, 360°
alrededor del eje CD.

8R7
A) 9 rc a:

B) 7i a3

9 3
O) 2 713

1 3
D) - n a 3

1 3
E) o 7 13

Solución
- Incógnita: V = ?
El volumen pedido, es ; según el Teor. de Pappus :

V = (área ABCDEF) . 2 x ;

Donde : x = Distancia del centro O, al eje.

x = apotema del hexágono.

- Recordando que, el apotema mide :

V = ' | a 2/ 3 ) . 2 * | / 3

V = - T ía 3
2
Rpta: ( C ).

24) El lado de un cuadrado ABCD, mide 10. Hallar el volumen del sólido engendrado al girar el
cuadrado, una vuelta, alrededor de un eje coplanar que pasa por el punto MD”, haciendo un
ángulo de 8o con C D . exteriormente al cuadrilátero.

A) 800 * J ~2 B) 700 rc .^2 C) 800 * / 3

D) 500 * E) 600 r c /2

Solución
- Incógnita : Volumen engendrado

- En el AB C D : BD = 1 0 / 2

BD
y:O D = — = 5 / 2

- En el AODH : HDO = 53

OH = 4 / 2
868
- Según fórmula del Teorema de Pappus :

V = 2 n ( OH ) . ( AD )

V = 2 k (4 / 2 M 1 0 2 )

V = 8 0 0 jt/2

Rpta: (A).

25) Hallar la relación de volúmenes de los


B sólidos generados al girar las regiones
MBN y AMNC, alrededor del eje MN, si
AM = MB y BN = NC.

1 1 1
AK 2 3
O)

1 t

LU
D)I
Solución
B
V1
Incógnita:

Sean: VMBN. volumen que genera MBN.


VAUNr
AMNC
volumen que genera AMNC

x1; x 2 y x 3 distancia de ios centros de


gravedadde las regionestriangularesMBN,
MNC y AMC, al eje de giro.

Por propiedad del baricentro para cada región, es fácil deducir, que:
d d 2
X i = - ; X 2 = - y X 3 = —d
2 3 3
Además la altura del triángulo ABC es el doble del triángulo MAC pero tienen la misma base,
luego:

ABC ABC
AMC y ; ®mbn “ ® mnc

Aplicando el teorema de Pappus - Guldim:

Vmbn - SMBn * VMBN


4 3

• » vmbn — • d • ^ abc ( 1 ).

^AMNC = ^AMC + ^MNC


869
V<MNC = ^ i £ . . 2 « . í d + 5 4 a £ . 2 » . í

VAMNC ~ . S ABC ( 2 ).

V
Se pide: — MBN , relacionando (1 ) + ( 2 )
VAMNC

V,MBN 1

VAMNC ^
Rpta: ( D ).

26) Usando el teorema de Pappus - Guldim, deducir la posición del centro de gravedad, de

A) Una semicircunferencia
B) Un semicírculo
C) Un cuarto de circunferencia
D) Un cuarto de círculo

Solución
a) Sea ACB, una sem icircunferencia de
centro 0 y radio r.
í\
Al girar ACB, 3609 alrededor del eje AB,
j A o B
se genera la superficie esférica, cuya
área es:
N
S¿B = ( Longitud de ABC ) . ( 2 OG )

Siendo “G", el centro de gravedad del arco

Así: 4 7t r2 = ( ti r)(2 ti . OG )

De donde:
2r
OG =
TC
b) Si “G”, es centro de gravedad del semicírculo;
su giro alrededor del eje A B , determina una
esfera.
Según Pappus:
Vs¡eneraao = (área del semicírculo) (2ji . OG)
Reemplazando sus equivalentes:
U "A o B
r 1
2 >
4 2 Ttr
— Ttr = ( 2 tc.O G )
3
W70
De donde:
4r
OG =
3 ti

c) Sea AB un cuarto de circunferencia,


de centro 0 y radio r. “G”, es el punto
que indica la posición de su centro de
gravedad.
Al girar AB, 360Qalrededor del ejeOB,
se genera media superficie esférica. \
Según el teorema de Pappus:

S ^ = ( Longitud de ÁB ) ( 2rc . y )
/ \
4 nr ' 2;cr '
Reemplazando equivalentes: 0 , v OG
, 4 i 72

2r
De donde: OG = V2
71

2r
O: y =
TC

d) Al girar el cuarto de círculo AOB, 3605 alrededor de OB, el volumen generado corresponde
a media esfera.
Por Pappus:

Vgenerado área del cuar


(2 tc . GH )
to de círculo

Reemplazando sus equivalentes:


2 '
2 3 7i r
— Tcr = ( 2 tc. GH)
3 \ / A

4r H B
GH =
3 71

871
r \

ISOMETRIAS
1 DEFINICION.- Una aplicación del plano, en sí mismo, es una regla que asocia a cada punto
del plano, otro punto del mismo plano. Si P es un punto del plano y P' el punto asociado
mediante una aplicación, escribiremos:

P* se llamará el valor de la aplicación en el punto P. De otro modo, podemos decir que P’


corresponde a P según la aplicación o que P es aplicado en P\
Las aplicaciones se denotarán con letras mayúsculas. Así, diremos que si F es una
aplicación, entonces F(P) = P \

2.- DEFINICION.- Dos aplicaciones del plano en sí mismo, F y G, son iguales, sí y sólo sí, para
todo punto P del plano :
F(P) = G(P)

3.- APLICACION CONSTANTE.-Sea Q,


un punto dado del plano.
Si a cada punto P se le asocia el punto
Q, se obtiene la aplicación constante.
Q es el valor constante. En este caso,
escribiremos:
F ( P 1) = F ( P 2 ) = F ( P 3 )= ... = Q. f¡g. 1
Donde F representaría a la aplicación
constante.

4.- APLICACION IDENTIDAD.- Se representa con la letra I. A cada punto P asocia el mismo
punto P.
I ( P ) = p , para todo punto P.

5- REFLEXION A TRAVES DE UNA RECTA.-


Sea L una recta. Se dice que el punto P* es
reflexión del punto P, a través de la recta L, sí
y sólo sí, L es mediatriz del segmento PP’¡
esto es, L intersecta a PP’ en form a
perpendicular y en su punto medio, (verfig.2).
873
%

Si Rl es la reflexión a través de la
recta L, podemos e scrib ir:
RL( P ) = p-

Ejemplo.-En la figura 3, adjunta, el A’


A A ’B'C' es la reflexión del A ABC,
a través de la recta L.
Luego; A’ = RL( A ) ; B! = R J B )
y C’ = Rl(C)

6.- REFLEXION A TRAVES DE UN PUNTO.- Sea O un punto dado del plano. Se dice que el
punto P’ es la reflexión del punto P a través del punto O, sí y sólo sí, O es punto medio del
segmento PPL Es decir, O está sobre PP’ y d (Ó,P’) = d (0,P).
d (0,P), se lee: distancia del punto O al punto P.

Fig. 4
P P‘
O- -c
o
Si R representa la reflexión a través del punto O, escribiremos: R( P ) = P’

Ejemplo.- la fig.5 muestra la reflexión del A ABC, a través del punto O.

C*

B’
oy —

A1

fig. 5
R(A) = A * , R(B) = B’ , R(C) = C \ donde R representa la reflexión a través del punto O
Ejemplo.-
La figura es un rectángulo,
d (A.C) = 10u . Si R l es la reflexión a
través de la recta L, hallar la suma de
las áreas de las regiones BCB’ y CDD\
siendo B‘ = RL(B) y D’ = RL(D).

A) 100u2 B)25u2 C) 10u2


D )50u2 E) 200u2

Solución
Sean: d( A,B ) = a y d( B.C ) = b
Luego:
bx b b2
área A BCB’ =
2 “ T
H74
Entonces:
área A BCB’ + área A DCD’

b“2 a_ 2
— +—
2 2
+ a 2

) o

Pero, en el A ABC: b2 + a2 = ( 1 0u )2
b2 + a2 = 100u2 ..........( I I ).

Sustituyendo ( I I ), en ( I ) :
área A BCB’ + área A DCD’ =

j ( 100u2) = 50u2

Rpta: ( D ).

Ejemplo. - ABC, es un triángulo equilátero, cuyo lado tiene longitud 12u. A’B’C’ es el trián
guio obtenido al reflejar el A ABC respecto al punto O, su ortocentro. Hallar el área de la re
gión común al A ABC y A A’B’C’.

A) 24u2 B) 4 / 3 u: C) 16 / 3 u2 D) 24 / 3 u2 E) N.A.

Solución
El ortocentro es el punto de intersección de
las alturas.
EFGHIJ : Región hexagonal ( regular) co­
mún al A ABC y A A ’B'C'.
AB
EF = EF = 4

Scomún = 6 ( SAE0F) ; a EOF es equilátero.

S . u n = 6v ( \42' & A ) /
w común

Srnmnn = 24 Jy 1 u2
com ún

12 T
Rpta: ( D ).

DILATACION.-Sea rcualquier núme­


ro real positivo. La dilatación en un P’=F(P)
factor r, es la aplicación Fr, con refe­
rencia a un punto O, que a cualquier
punto P, le asigna el punto Fr( P )
ubicado sobre el rayo de vórtice O, que
pasa por P, a una distancia de O igual d(O.P) fig. 9
s

a r veces la distancia de O a P.
El resultado de hallar F ( P ), también se escribe rP : Fr( P ) = r P

Una dilatación también recibe el nom­


bre de transformación de semejanza.
La figura 10 muestra las dilataciones a*
del punto A, respecto a O, factores 2
y 0,5:
FJ A ) = 2A

0 .5
( A ) = ¿A

Ejemplo.- Sea AOB un triángulo rectángulo, recto en O. Si la aplicación F es una dilatación


respecto a O y d( O,A ) = d( 0 ,B ) = 2, h a lla r: d( F3 ( A ), F4 ( B )).

A) 5 B) 10 C) 8 D) 14 E) 12

Solución
La figura 10, indica el modo de hallar
las distancias: F o (A )
d(O, F3( A ) ) = 6 fig: 11
y
d ( O, F4 ( B ) ) = 8 3d(0;A)=6

Luego, con el Teorema de Pitágoras:


x2 = 62 + 82

x = 10 _> d ( F 3( A ) , F 4( B ) ) = 10
4d(0;B)=8
Rpta: ( B ).

Ejemplo.-OBA es un triángulo rectángulo, recto en B, d( O,A ) = 15u. y d ( 0 , B ) = 12u.


A’ y B’ son dilataciones de A y B, respecto a O, factores 4/3 y 25/12, respectivamente. Hallar
el área de la región AA’B’B, sabiendo que el ángulo AA’B’ es recto.

A) 96u2 B) 84u2 C) 90u2 D) 81 u2 E) N.A.

Solución

Del gráfico: d( O,A’ ) = 20 y d (0 ,B ') = 25


fig - 12
Luego, con el Teorema de Pitágoras en los
triángulos OBA y OA’B \ se obtienen:

d(A ,B ) = 9

y d ( A ’B’ ) = 15

El área S : S = área A OA’B’ - área A OBA


876
d ( 0 , A ' ) x d( A \ B * ) d( 0 , B ) x d( A,B )
s=
2 2
20x15 12x9

2 2
S = 96 u2
Rpta: ( A ).

8.- ROTACION.- Sea O un punto dado en el plano y P un punto del mismo, tal que d (0,P ) = d.

La rotación del punto P, respecto a O,


un número de grados A, se logra to­
mando sobre la circunferencia de cen­
tro O y radio d, un arco PP' tal que el j □----- -
ángulo POP’ tenga medida A. ( El
arco PP’ tiene igual medida ).

La aplicación que asocia P’ a P es llamada una rotación ( antihoraria) por A, respecto a O,


ó relativa a O.
Se denota esta aplicación como GA. Luego, en la figura 13 : GA( P ) = P \

Ejemplo: p:
*\

Q ¿p
La figura 14 muestra la rotación del
✓ \
segmento PQ, 80° respecto al punto /
/ \
\

¿80V
O, obteniéndose el segmento P’Q’. Q’
\ ' /
fig. 14

Una rotación horaria respecto al punto O, se escribe G,A y asocia el punto P” al punto P, tal
que el ángulo P” OP tenga la misma medida que A.

P” = G .A (P)

Ejemplo.- En la figura, O, P y Q son puntos de una recta, d ( O.P ) = 8 y d ( P,Q ) = 7


La aplicación G es una rotación respecto al punto O. *

Si: P' = Glpo( P ) y Q1 = Ginpc( Q )

Hallar d ( P ’,Q’ )
O Q
A) 15 B) 23 C) 16 fig. 16
D) 17 E) 20

877
Solución
La fig. 17 corresponde a la solución:
Z POP’ mide 12°
Z QOQ’ mide 102°

P’OQ’ mide 90°

d ( 0 ,P ’) = d ( O tP) = 8 y
d (0 ,Q ’) = d (OtQ) = 15

Con el Teorema de Pitágoras , en el


A P’OQ’ : x2 = 82 + 152

x = 17 -> d( P ', Q’ ) = 17

Rpta: ( D ).
15 1

9.- OBSERVACIONES:
1o) Una rotación de 180° respecto a O, es lo mismo que una reflexión a través de O.

G¡180= (p ) = R(p )
/—J 8°'
-F- A__
También: G 180. ( P ) = R(P) P’ O
fig. 18
2 o) Conviene asociar una rotación con un número en lugar de un ángulo. Sea x un número entre
0o y 360. Sea Gx la rotación en un ángulo de x grados. Luego, siempre es posible escribir
x en la forma :
x° = 3 6 0 °. n + w°

Siendo n un número entero y w un número tal que 0 < w < 360. Luego : Gy0 = G wr

* En particular : G0* ~ G360c — I

Ejemplo.-Si x = 510, escribiremos : x = 3 6 0 + 150

Gx: = G150> .

P = Es decir, una rotación de 510'


es lo mismo que una rotación
de 150°.
fig. 19

Ejemplo.- Si x = -60 -» x = -360 + 300

G„0 = G -60' = G 300'

878
10.- TRASLACION.-La figura 21 muestra
una dirección en el plano y una distan­
"A . R
cia d = d ( 0 ,R ).

La flecha indica un par ordenado


fig- 21
( 0 , R ) de principio O y extremo final R.

La traslación ( determinada por la dirección y la distancia), es una aplicación que a cada punto
P, asocia el punto P’ ubicado a una distancia d de P, en la dirección dada OR.
Si T representa la traslación definida anteriormente, entonces escribirimos: P* = T( P )

, 'T ( P ) = P'
/
T(B)
La figura 22 muestra la traslación
del punto P y del A ABC.

fig. 2 2

Ejemplo. -ABCD es un rectángulo cu­


yas diagonales se cortan en Q. M es
punto medio de AB, T es una trasla­
ción en la dirección mostrada, tal que
T( M ) = Q.
¿Qué fracción del área de ABCD es el
área de la región triangular que deter­
minan T( D ) , T( C ) y Q?
1 1 1
A> 2 B> 4 C) 8

D) E)
12 16
Solución
La traslación T está definida por la dirección
dada y la distancia d(M,Q).
Sean: D’ = T( D ) y C’ = T( C ).
d (B,C) = h y d ( D , C ) = b
h
El área ABCD es : SABCD = b . h

El área de la región común a ABCD y


D’C’Q, es :
'h ' b .h b .h
Sv Sv =
\ 2 ) 12 / ~T~ ~ ~ Q ~ ~

b ... s x = -?4§cd = S ABCD )


1 x 8 8V
fig. 24
Rpta: (C )
879
11.- DEFINICION.- Sea F una aplicación del plano en sí mismo . P es un punto fijo para F, sí
y sólo s í : F( P ) = P.

Por ejemplo, la rotación a través de un punto dado O, tiene como punto fijo el mismo punto
O. La reflexión a través del punto dado O, tiene por punto fijo O.
La reflexión a través de una recta deja fijos todos los puntos de dicha recta.

12.- DEFINICION.- Si F es una aplicación y P un punto cualquiera del plano, F(P) se llama la
imagen de P bajo F.

13.- DEFINICION.- Sea F, una aplicación del plano en sí mismo. P y Q, dos puntos cualesquiera,
distintos, del plano. Diremos que F es una ISOMETRIA, sí y sólo sí, la distancia de P a Q,
es igual a la distancia de F( P ) a F( Q ).

De la definición anterior, se deducen:

1o) Son isometrías: La rotación, la traslación, la reflexión a través de un punto, la reflexión a


través de una recta y la aplicación identidad.

2o) La imagen de un segmento, a través de una isometría, es otro segmento congruente al


original.

3o) La imagen de cualquier figura geométrica, a través de una isometría, es otra figura
congruente a la original.

EJEMPLOS.

1) El lado del cuadrado ABCD, tiene


longitud 2 ^ 3 u.
Luego de reflejar ABCD a través de L,
hallar el área de la región común con
su imagen.
A) 4u2 B) 2u2 C )2^3u2

D )4 /3 u : E) N.A.

Solución
La región común a ABCD y su imagen, es : D’CDE

En el A CDE, con ángulos notables 30° y 60°:


880
„ , c n , d(C D ) 2 /3
d ( E 'D ) = ~ ^ ¡T ~ ~ ÍfT fig: 26
• • d ( E.D ) = 2

Luego, el área común:

2 (d (D ,C )x d (E ,D ))
® d ‘c d e “ ^ (® ed c ) -

SDCOE = d( D,C ) x d( E,D ) = 2 / 3 x2

• *
®DCDE
u2
« s -2 7 / é
Rpta: ( D ).
2 a /
a&

r
h- a t

ABCDEF es un hexágono regular cuyo lado tiene longitud 2u. M es el punto medio del lado
EF. __
Se refleja el hexágono a través de la recta que contiene a B M . El perímetro de la región común
que encierran ABCDEF y su imagen a través de dicha reflexión, es :

A) 12u. B) 10u. C) 8u.


D) 6u. E) 9u.

Solución
Del gráfico, se observa que:

d ( A,B) = d (A , F) = d ( A \ B ) = d( A’.F’)
= 2u y d(F,M) = d(F,M ) = 1u.

Perímetro = 10u.

Rpta.: (B)

En la figura, d(0,P ) = 2. G es una rotación de 90°


respecto al punto O.
F es una reflexión a través de L y H es una reflexión
i a través de O.

Si: P’ = G ( P ) , Q = F(P’)
fig. 29 y M = H(P’) , hallar la distancia de M a PQ.

A) 2 B) 1 C) / 3 D) / 2 E) 1/2

881
Solución

El gráfico adjunto muestra la solución L


a las aplicaciones planteadas. Como 30’
d(0,P') = d (O, P) = d(0,Q )=d(0,M ) =2,
entonces P \ P, Q y M están sobre la P’ 's30‘ o /

<?/ 2
misma circunferencia. 4

; < ;.H
*? \ |
* Incógnita : d ( M,H )

Se observa:
MPQ = MP'Q = 30°
fig. 30
También: A MOP -» PM O = 45°

d ( P.M ) = 2 / 2

Y, en A MHP :

d( P , M ) 2J~2
(M ,H) — ¿ = —* _

d(M ,H ) = / 2

Rpta: ( D ).

4) ABCD, es un rectángulo: AB = 6u. y BC = 10u. Se refleja ABCD, a través de una recta que
contiene a la diagonal A C . Hallar el área de la región común al rectángulo dado y su imagen.
A) 40u2 B) 42,8u2 C) 46,8u2 D) 40,8u2 E) N.A.

Solución
- Sea A’B'C’D’, imagen de ABCD, a
través de la reflexión respecto a X c .

- Incógnita : SAFPF ( área AFCE )

- Se tiene:
Se d e m u e s t r a
CD que AFCE, es un
S afce = AE
paralelogramo.

Si AE = x , como CD = 6

* ^AFCE = (^

- Cálculo de x:
- Por propiedad de la mediatriz : CD1 = CD ; AB’ = AB
882
AD' = AD y CB’ = CB o A ’B’C’D’ = oA BC D . Esto demuestra que AFCE es un
paralelogramo FC = AE = x.

También : A ABF = A FD’C AF = FC = x

Luego, en el A ABF, por el Teorema de Pitágoras:

AF = AB + BF x2 = 62 + ( 10 - x )2 , efectuando : x = 6,8

• • Reemplazando en ( I ) : SAFCE = 6 ( 6,8 ) u2

S afce = 40.8 u2

Rpta: ( D ).

En la figura: T, es una traslación, con dirección, sentido y módulo indicado


Si G, es una rotación de 8o,respecto a B,y :

C = T(B)

D = G(C)

Hallar la distancia AD.

Solución

Para hallar C, se traza BC O P , tal que BC = OP = 10

Para hallar D, se rota C, 8o respecto a B BD = BC = 10

m Z DBH = m Z HBC + m Z CBD


m Z DBH = 45° + 8o = 53°

B 6 H
f— 5^2

Si trazamos DH 1 AB : A DHB ( 53°, 3 7 ° ) : DH = 8 y BH = 6 AH = 15

A AHD : AD = AH + DH = 152 + 82

Dado el sistem a de corrdenadas


cartesianas XY, P(a;b) un punto del
mismo y la recta y = x.
883
V

Demostrar que P \ reflexión de P, a través de dicha recta , tiene coordenadas


P’ ( b ; a )
Solución

Se trazan PH jl OY
P 'A ± OX

OH = b y HP = a
( a + ó = 45°)

- A O A P ’ = AOHP

Luego: OA = OH
P'A = HP

OA = b y P’A = a P’ = ( b ; a ) I.q.q.d.

Nota.- Aunque la demostración se ha hecho cuando P está en el Primer cuadrante, la


propiedad es válida en cualquiera.

7) Sea : A ( 3 ; 4 ) un punto del plano


cartesiano.
Sea B, reflexión de A , a través de la
recta y = x.
Hallar AC, Si C es reflexión de B a
través del eje x.

Solución
AC = d ( A,C ) = ?

Del gráfico:

d ( A , C ) = ^ ( 4 - 3)2 + ( - 3 - 4)

Rpta: d ( A,C ) = / 5 Ó = 5 / 2
* Si el lector no recuerda los conceptos básicos de Geometría Analítica, vea el capítulo 32

8) Un cuadrado de longitud de lado L, se rota 90°, respecto a un vértice. Hallar la distancia entre
los centros del cuadrado dado y su imagen.

Solución
90°
Sea ABCD , el cuadrado , de
B
centro O. r%\ .■* / / r-~ -„ ■ ■•• ■»
\ ✓ ./
En el A B’BD : *X ,^
sf t / \Q '' \
f \ »
/ \
/ A \
B'D 2L / X \
00 ' =
B
\
A D
h* L -H
Rpta:

884
ABCD, es un rectángulo de centro O. AB = 15 y BC = 20. Sean: E, reflexión de B, a través
i
de AC. F, reflexión de E, a través de O. Hallar BF. r

Solución
BF = ?

En el A EBF, por el Teorema de los


puntos medios: BF = 2 (H O ).... ( I ).

C álculo de HO:
En el A ABC, por el Teorema de
P itágoras:

AC = 152 + 202 AC = 25
25
AO = OC =

y, por relación métrica: AB = A C . AH


152 = 25AH AH = 9
25
Entonces : HO = AO - AH = -9

Reemplazando lo último, en ( I )

Rpta: BF = 7

En la figura: OP = 3 y PQ = 2

E, reflexión de P, a través de OA.

F, reflexión de Q, a través de O B .

corta OA en M y OB en H

H a lla r: PM + MH + HQ.

Se traza el gráfico según enunciado y


luego O E ; OF.

Por propiedad de la mediatriz :


OE = OP -» OE = 3
o ) 38 B
OF = OQ -» OF = 5
ii
Mi
•4
Además: EM = PM y HF = HQ F
PM + MH + HQ = EF

885
- Extrayendo el A EOF, hallaremos EF,
trazando EV J_ O F .

A EVF, Teorema de Pitágoras :


— 2 2 2
EF = EV + VF

— 2 27 169
EF = — + -----

EF = 7

Entonces : PM + MH + HQ = 7

Rpta

11) En la figura: AB = 20

T qp y Tmn, s°n traslaciones definidas, por O P y M ^ , respectivamente

OP = 1 y MN = 4

S i : T0P( A ) = E y T MN( B ) = F
Hallar EF.

Solución

- Para hallar E, se traza por A : AE O P . t a l q u e A E = OP = 1

- Para F : BF MN, de modo que BF = MN = 4.

20

- Si prolongamos AE y BF hasta R : AR _L BR y ene! A A R B ( 3 7 ° , 5 3 ) : AR=1€ y BR= 12

• • ER = 15 y RF = 8. Luego : EF = ER + RF

EF = 152 + 8 2 Rpta

886
#

CAPITULO 3 2

INTRODUCCION A LA GEOMETRIA
ANALITICA
1 Introducción.- El presente capítulo es una introducción a la Geometría Analítica en el plano.
Aquí, los puntos y las rectas no son objetos indefinidos sino que están definidos en términos
de números reales llamados “coordenadas” o “pares ordenados”. Fue el matemático filósofo
inglés René Descartes (1596 -1 6 5 0 ) quien explotó la idea de darle a la Geometría euclidiana
un tratamiento algebraico consistente y sistemático.
Al término del presente capítulo, el estudiante podrá graficar cualquier punto en un sistema
de coordenadas cartesianas, hallar la distancia entre dos puntos del plano cartesiano, así
como deducir la ecuación de una recta.

2.- Sistem a de C oordenadas R ectangulares ( C artesianas ).- Consideremos dos rectas


orientadas X’X y Y’Y, perpendiculares entre sí en un punto.
El conjunto de estas dos rectas recibe el nombre de "Sistema rectangular de coordenadas
cartesianas", en memoria de René Descartes, por ser el primero que lo ideó. Se definen:

II I

o X
III IV

* Recta X’X, horizontal, llamada “eje de las abcisas” o “eje de las X”, o simplemente “eje X”.

* Recta Y’Y, llamada “eje de las ordenadas”, o “eje de las Y”, o simplemente “eje Y". Los dos
ejes son llamados “ejes coordenados”.

* El punto O, intersección de los ejes, se llama “origen de coordenadas”.

* En el eje x se considera positivo el sentido hacia la derecha. En el eje Y se toma por positivo
el sentido hacia arriba. Además:
OX , es el semieje positivo de las X.
O X \ es el semieje negativo de las X.
00"
OY, es el semieje positivo de las Y.
OY’, es el semieje negativo de las Y.

Las cuatro regiones 1,11,111 y IV, se llaman cuadrantes, numeradas en sentido contrario al
movimiento de las agujas del reloj.

A bscisa y Ordenada de un p u n to del Plano.

f
y

Sea P, un punto referido a un sistema rectangular de coordenadas.

Tracemos : PA 1 eje x y PB 1 eje y.

Si se conocen las longitudes de los segmentos OA y O B , el punto P queda perfectamente


definido. La longitud de OA medida con una unidad elegida, se representa por x y se llama
abcisa del punto P: la longitud OB medida con la misma unidad o con cualquier otra, se
representa por y, llamada ordenada del punto P. Los números x e y determinan el punto P
y se llaman coordenadas cartesianas de P, escribiéndose del siguiente modo: P( x ; y ), que
se lee: Punto P de coordenadas x , y.

S ignos de las C oordenadas.- Las abscisas son positivas a la derecha del eje Y, porque se
miden en el semieje OX positivo, y negativas a la izquierda, porque se miden en el semieje
negativo OX’.
Las ordenadas son positivas en la región superior al eje de las abcisas, medidas en el semieje
positivo OY, y negativas en la región inferior, porque se miden en el semieje OY'.

Se tendrá, entonces:
IY
II Cuad. I Cuad.
x- x+
y+ y+

)
X- x+
y- y-
III Cuad. IV Cuad.

Ejm.-
Graficar los puntos A ( 3; 5 ), B (-4; 2 ), C (-2; *3 ), D ( 3; -6 )
Solución
Y
-_ 4

ó i X
i

c' [

Se elige un sistema de ejes coordenados y las unidades respectivas para los ejes X’X e Y'Y
( es cómodo usar papel cuadriculado ).
Se observan:
A, pertenece al I cuadrante.
B, pertenece al II cuadrante.
C, pertenece al III cuadrante.
D, pertenece al IV cuadrante.

Ejm.- Graficar los puntos P ( 0; 3 ), Q ( 8; 0 )


Solución

El punto P se ubica a 3 unidades del origen, sobre el semieje positivo OY.

El punto Q se ubica a 8 unidades del origen, sobre el semieje positivo OX.

Ejm.- Detemrminar el área de la región triangular ABC, s i : A (-3; 0 ) , B ( 0; 4 ) y C ( 5; 0)

Solución
Se grafican los puntos en un plano
cartesiano.

Luego: AC = 3 + 5 = 8
OB = 4
1 1
S _ = - (AC)(OB) = - (8)(4) = 16 unida­
ABC 2 ' M ' - 2
des cuadradas.

889
DISTANCIA DE UN PUNTO, A L ORIGEN DE COORDENADAS.-
La distancia del origen O, al punto
P( x ; y ) , es:

OP = yj x2 + y2

DISTANCIA ENTRE DOS PUNTOS.- Dados dos puntos P( x1; y1) y Q( x2; y2)t la distancia
entre ellos se escribe: dp^ y se calcula por la fórmula:

Nota.- La distancia entre los puntos P y Q, también se acostumbra escribir: d ( P, Q ), PQ

PQ

Ejm.- Hallar la distancia entre los puntos P ( 3 ; 4 ) y Q (-5; -11 ).

Solución
- Por fó rm u la :

po = y ( 3 - -5 ) + ( 4 - -11)

p q = i ¡ { e f + ( 1 5 )2

Rpta: PQ = 17

* Observar que cada componente


va con su propio signo. Q(-5 ; -11)

Ejm.- El punto P( n ; n ), está en la mediatriz del segmento AB, siendo A (2; -3) y B (-4; 9)

Hallar el valor de n.
890
Solución

Sea el gráfico :
Como P es un punto de la mediatriz
del AB, entonces por propiedad :
PA = PB

Y ahora, usando la fórmula de distan­


cia entre dos p untos del plano
cartesiano:

(n _ 2 )2 + (n - -3 ) =

Rpta: Efectuando: n = 7

DIVISION DE UN SEGMENTO EN UNA RAZON DADA

Sea P ( x ; y ) un punto del segmento


ÁB , donde A ( x1; y t ) y B í x ^ y , , ) ,
tal que :
AP
= r
PB

Entonces, las coordenadas ( x ; y ) del


punto P, se evalúan así:

X, + r x 2 .. y i + f y2
y -
r+ 1 r+ 1

D em ostración.- Por semejanza de los triángulos.


AC AP x - x1 xt + rx 2
= r X =
PD PB x2 - x r + 1

PC = AP y - yi _ r _ yi +ry2
y= r+1
BD “ PB y2 - y

PUNTO MEDIO DE UN SEGMENTO.


Si M, es el punto medio del segmento AB,
sus coordenadas se calculan p o r :

x 1 + x2
x =
Y

yi + y2
y=
Ejem plo. Dado un triángulo ABC, donde: A( 3 ; 8 ), B (9 ; -2 ) y C (-4; -1); hallar la longitud
de la mediana CM.
Solución
- CM = ?

- Con el gráfico:

- Coordenadas de M :
3+ 9
x = -------- -+ x

8 + -2
y = y *3
2
M ( 6; 3 )

- Entonces, con fórmula de la distancia:

CM = 6 - "4 f + ( 3 - '1 f

Rpta: CM = ^ 1 1 6

COORDENADAS DEL BARICENTRO DE UN TRIANGULO.


Si G( x ; y ), es la posición del Baricentro de un triángulo ABC, tal que: A( x,; y,), B(Xg; y2) y
C( X3 ; y3), entonces :

AREA DE UNA REGION TRIANGULAR.- El área de la región triangular, que determinan los
puntos A(x,; y,). B(x2; y2) y C(x3; y3),
se evalúa por el valor absoluto de :

*1 yi 1
x2 y3 1

X3 y3 1

Ejem plo.- Hallar el área de la región triangular ABC, si :

A ( 3 ; 7 ) , B ( 2 ; 9 ) y C (-2 ; 3 )
892
Solución
Se tie n e :

• -
3 7 1 19 g 2 1 2 9
1
s =± 2 9 1 3 -7 +1
2 ~ 2 m 3 1 "2 1 "2 3
’ 2 3 1

1 1
S=“ [ 3 ( 9 - 3 ) - 7(2 + 2 ) + 1 ( 6 + 1 8 ) ] = “ [1 8 - 2 8 + 24]

Rpta: 7u2

Ejem plo.- Hallar el área de la región triangular PQR , siendo P(-2; 9) Q(4; 5) R(6; -2)

Solución
Se tiene:

"2 9 1 5 1 4 1 4 5
s =— 4 5 1 -9 +1
2
1
) "2 1 6 1 6 "2

s = i [ ( - 2 ) ( 5 + 2 ) - 9 ( 4 - 6 ) + 1( - 8 - 3 0 ) ] = [ - 20 + 18 - 38 ]

S = \ [-40] S = - 20 ; como es negativo , se debe tomar valor absoluto

• • El área será : IS I = I - 2 0 1= 20 u Rpta

Nota.- Como puede verse, para usar este método es necesario que el lector sepa cómo
resolver un determinante.

METODO PRACTICO PARA HALLAR EL AREA DE UNA REGION TRIANGULAR.

Se procede del siguiente modo:

Se ordenan los elementos de los pa­


res ordenados, en filas, repitiendo al
final la primera:

xi yi
y2
X3 y3
X1 y.

893
2°) Se efectúa ta suma de productos indicada :

I —> D

Es decir : D = x1y2 + Xgy3 + x3y,

I = y , x 2 + y2x3 + y3x,

3o) Finalmente, el área S, de la región triangular correspondiente, será: S = —| D —I|u 2


2 1

Ejem plo.- Hallar el área de la región triangular ABC, con el “método práctico”, si: A (2 ; 8 )
B (-4; 4 ) C( -4; -6 )

Solución
Se tiene:

I <— D

D = 2( 4 ) + ( - 4 ) ( - 6 ) + ( - 4 )( 8 ) D = 0

I = 8 ( -4 ) + 4( -4 ) + ( - 6 ) ( 2 ) I = -60

Entonces: S = — ID —11 = — 0 - 6 0 = — I 6 0 1= 30 S = 30u2 ... Rpta


2 i i 2 2 1 1

AREA DE UNA REGION POLIGONAL.- Sea A 1A2A 3...An, un polígono cuyos vértices,
nombrados en sentido antihorario, tienen coordenadas:
A ,( xt ; y i ) , Ag( Xg; y2) ,
^3 ( ^3* Y3 ) » *** I An ( Xn, yn)
A1 An
Entonces, el área de la región poligonal
correspondiente, es el valor absoluto A An -1
de la expresión.
*1 Y^ 1
x2 Yz 1
s = — *3 Ys 1
2 • #
# #
yn 1

894
METODO PRACTICO.- Se procede como en el caso de una región triangular, colocando los
pares ordenados en filas y repitiendo la primera:

I <— —* D

Ejem plo.- Hallar el área de la región pentagonal cuyos vértices, son


(-6; 16 ) , ( 16; 6 ) , ( - 1 0 ; -4 ) , ( 12; 12 ) y ( 20; -8 )

Solución
Hacemos un gráfico aproximado.

Elegimos un punto in ic ia l:
( x i ; y i ) = (12; 12)

Luego, para usar el “método práctico”,


ordenamos los otros puntos teniendo
en cuenta el sentido antihorario :

I <— —>

Siendo:

D = 12( 16 ) + ( -6 ) ( - 4 ) + (-1 0 )(-8 ) + 20( 6 ) + 16( 12 ) -» D = 608

I = 12( -6 ) + 16( -10 ) + ( - 4 )( 20 ) + ( - 8 )( 1 6 ) + 6( 12 ) -» I = 368

895
el área : S = — | D —11 = — 608 - 368
2 1 1 2

S = 119761 = 488 S = 488 u2 .......... Rpta


2 1 1

LA RECTA.- La expresión:

y = mx + b | ... ( I )

es la ecuación de una recta, que corta


al eje de ordenadas en el punto ( o ; b)

a medida del ángulo de in


clinación de la recta.

m Pendiente de la recta.

CARACTERISTICAS DE UNA RECTA:

a) A ngulo de In clin a ció n .- Aquel que forma la recta con la dirección positiva del eje de
abscisas, medido en sentido antihorario. Toma valores comprendidos entre 0o y 180°.

b) Pendiente.- Es la tangente trigonométrica del ángulo de inclinación de la recta. En el grá­


fico anterior:
~m = tg g° |

Ejm. La ecuación de una recta, es : x - y + 4 = 0

H a lla r: 1°) La pendiente.


2°) La medida del ángulo de inclinación

Solución
Se escribe la ecuación, como en ( I ) :
y = x+4
Entonces, comparando:
m = 1
• •

tga° = 1 —» a = 45

Rptas:
1o) La pendiente, es m = 1
2o) El ángulo de inclinación, mide : oc° = 45

O bservaciones:
1) - Si el ángulo de inclinación de una recta, es agudo, su pendiente resulta positiva : a < 90
m = tg a° > 0

896
2) - Si el ángulo de inclinación de una
recta, es obtuso, su pendiente es ne­
gativa :
a > 90

m = tg a° < 0 a

3) La pendiente de una recta paralela al eje


X, es c e r o :

L OX .% m = 0

La ecuación de esta recta, es

y = b |

4) - Las rectas paralelas al eje Y, (rectas verticales), no tienen pendiente.


Para el gráfico adjunto:

* Medida del ángulo de inclinación : 90°

* Ecuación de la recta :

V y

5) - Recta que pasa por el origen de coordenadas.

En este caso :
b = o

La ecuación : y = mx

- En particular, si m = 1, la ecuación queda:


y = x, siendo tg a = 1

a = 45°

897
6) - La expresión : — + 1~ = 1 1 es llamada forma simétrica de la ecuación de una recta. En
a b

este caso un gráfico puede ser el siguiente :

7) - En algunos casos es conveniente escribir la ecuación de una recta del modo

Ax + By + C = 0 | ( Forma ge n e ra l)

8) - Si una recta pasa por los puntos (x1; y ^ y (x2; y2) , entonces su pendiente m = t g a , se puede
evaluar como:

(x2 i y2)

<y2 - y i )

Ejem plo.- Hallar la pendiente de la recta que pasa por los puntos ( 3 ; 5 ) y (-2 ; -4 )

Solución
Se tiene:
5 - ~4 _ 5 + 4
m =
3 - "2 ~~ 3 + 2

. 9
Rpta: m= —
5

9) En general, si P ( x1; y1) y Q ( x¿; y2) son


dos puntos de una recta L, entonces la
ecuación de diha recta, puede hallarse
al d e sp e ja r:

y - yi = y2 - yi
X - X -| — X-)

898
O,de: y ^ L = y2JLyi
x-x2 x2 -

Siendo la pendiente : m = tg a = ------“


x2 - x1

Ejm. Escribir la ecuación de la recta que pasa por los puntos P ( 7 ; 2 ) y Q (1 0 ; 5 )


Solución

- Si A ( x ; y ) , es un punto genérico de la recta ; entonces:


y - 2 5 -2
x - 7 ~ 10- 7

Efectuando : y = x-5

Como el coeficiente en x dá la pendiente : m = 1

10) Si P0 ( x0; y0), es un punto de paso de una recta con pendiente m; entonces la ecuación de
dicha recta, e s :

y - y0 = m ( x - xo) I ( Ecuación p u n to pendiente )

Ejm. Hallar la ecuación de una recta con pendiente 2 y que pasa por el punto ( 3 ; - 7 )
Solución
Según lo anterior, la ecuación se logra de:
y * ( -7 ) = 2 ( x - 3 )
Rpta: y = 2x - 13

11) Y Dos rectas paralelas tienen igual pen­


L diente.
Así, para el gráfico adjunto , si L1 11 L2
entonces a 1 =

tgo^ = tg<x2

x Luego :
/
A /
Y
ANGULO ENTRE DOS RECTAS.-
Sean las rectas L1 y L j, de pendiente:
m, = tg a 1 y : m2 = tg ct2.

Entonces, para hallar la medida del


ángulo entre dichas rectas, se deduce
del gráfico adjunto, que:
a = o tj-a ,
899
Como, generalmente son conocidos m1 y m2 , aplicamos la función tangente a ambos
miembros:
jg a ^ t g ^
tga = tg ( a 2 - a 1)
1 + tg a 2 . tg a 1

ítu - m<
ó también : tg a = — £------- 5-
1 + m., . m2 ( *) ■

RECTAS PERPENDICULARES- Si en ( * ) ,a = 90°, el denominador del segundo miembro


debe amularse.
Entonces : 1 + m1 . m2 = 0

m1 . m2 = - 1 ( ** )

Ejm. Hallar la ecuación de la recta L, que pasa por el punto ( 3 ; -4 ) y es perpendicular a otra
recta de ecuación : y = 2x + 5

Solución
Sea m la pendiente de la recta L cuya
ecuación se quiere hallar.

Como la pendiente de la otra recta, es


2 ; entonces, por la propiedad ante­
rior:

2m = -1 m= - —
2

Usando la ecuación punto pendiente,


para L :

y-(-4) = “ (x-3)

De donde : 1 5
y= —x -----
2 2

DISTANCIA DE UN PUNTO A UNA


RECTA.- La distancia d, de un punto
P( x1; y1), a una recta L, de ecuación:
Ax + B y + C = o , s e evalúa con la
fó rm u la :

900
\

ECUACION DE UNA CIRCUNFERENCIA.- Sea C ( h ; k ) , centro conocido de una


circunferencia de radio conocido r.
Entonces, la ecuación de cualquier
punto P ( x ; y) de dicha circunferencia,
se obtiene al e s c rib ir: d (P.C) = r

Usando la fórmula de distancia entre


dos puntos :

i j ( x - h f + ( y - k )2 = r

elevando al cuadrado :

( x - h )2 + ( y - k )2 = r2 ! ....(I).

Ejemplo.- Escribir la ecuación de la circunferencia de centro ( -2 ; 5 ) y radio 4.

Solución
En este caso ,h = - 2 , k = 5 y r = 4

De modo, que, al reemplazar en ( I ) :


( x - -2 )2 + (y - 5 )2 = 42

Rpta: ( x + 2 )2 + ( y - 5 )2 = 16

CIRCUNFERENCIA CENTRADA EN ELORIGEN.- Si en la ecuación (I), el centro C, coincide


con el origen, entonces : (h; k) = (0; 0),

La ecuación queda: x2 + y2 = r2 1 Y

Ejm. Hallar la ecuación de la circunfe­


rencia cuyo centro es el origen de
coordenadas y pasa por ( 5 ; 12 )

Solución

Centro : C = ( 0 ; 0 ) ecuación: x2 + y2 = r2 (U

Como ( 5 ; 12 ) , pertenece a la circunferencia; entonces reemplazando en (1 )

52 + 122 = r2 —> r = 13

En ( 1 ) mismo, la ecuación será :

x2 + y2 = 132

Rpta x2 + y2 = 169

901
\

PROBLEMAS

1) Dados tres vértices de un paralelogramo ABCD: A(3; 5), B(5; -3) y C(-1; 3). Determinar las
coordenadas del vértice D, opuesto a B.

A) (3; 1) B) (3; -1) C)(-3;-1) D) (-3; 1) E) N.A.


Solución

- ABCD : Paralelogramo.

Sea D( -x; y ), donde x > 0


D
De la congruencia entre los trián­
gulos sombreados, podemos de­
ducir fácilmente que :

x - 1 = 2 /. x= 3

3 - y = 2 .-. y= 1

Luego : D( -3; 1 )

Rpta: ( D ).

2) La mayor base de un trapecio isósceles une los puntos (-2; 8) y (-2; -4). Uno de los extremos
de la otra base tiene coordenadas ( 3 ; -2 ). La longitud de la base menor, es :

A) 8 B) 6 C) 9 D) 10 E ) 12
Solución

Incógnita: CD
B (-2 ; 8) Y
Del gráfico : QA = 2 BH = 2
H 3----- C Luego : C ( 3 ; 6 )

Entonces:
n
X CD = ^ ( 3 - 3 ) 2 + ( 6 - ( - 2 ) 2 )

Q3 D (3 ; -2)
2 CD = 8
A (-2 ; -4)
Rpta.: (A)

3) El área de un triángulo es 4u2; dos de sus vértices son los puntos : A( 2; 1 ) y B( 3; -2 ); el


tercer vétice C está situado en el eje x.

Determinar las coordenadas del tercer vértice C.

A) ( 5 ; 0 ) B) ( 4 ; 0 ) C) ( 3 ; 0 ) D) (-5 ; 0 ) E) (-4 ; 0 )
902
\

Solución

^A B C “ ^

Como : AB = ^ (3 - 2)2 + ( -2 - 1)2 Y

AB = V 1 + 9 = /T Ó A (2 ; 1)

La distancia "d”, del vértice C ( x , ; O )


a la recta A B , se halla :
1
S „c= ó (AB)d B (3 ; -2)

8
4. u m o d=
Vio -(1 >*
De otro lado, la ecuación de t ñ :

y - 1 _ 1 - (- 2 )
3x + y - 7 = 0
x - 2 2 -3

Usando la ecuación de la distancia Hd" del punto C ( x1; 0 ) a la recta AB

3 xt - 1(0 ) - 7 3X t - 7
d= d= (2).
/To
Luego, de (1 ) = ( 2 ) :
3x, - 7 8
V io Vio

1
* Si 3x. - 7 = - 8 xt = - 3

* Si 3x, - 7 = 8 Xi = 5
1
Hay dos respuestas : —;0 )

Cj>( 5 ; 0 )
Rpta: ( A ).

4) Hallar las coordenadas del


circuncentro del triángulo ABC , si
A (0 ; 0 ), B (3 ; 8) y C ( 7; 5).

Solución
Sea P(x; y), circuncentro del triángulo,
centro de lacircunferecia circunscrita.
Se debe cumplir : PA = PB

J 7 7 7 = ^ ( x - 3 )2 + ( y - 8 ) 2
on*
De donde : 6x + 16y - 73 = 0 ........... ( 1 ).

También : PA = PC

4X2 + y2 = ^ ( x - 7 )2 + ( y - 5 ) :

Luego : 7x + 5y - 37 = 0 .......... ( 2 ).

Resolviendo (1 ) ; ( 2 ) , hallamos :
227 289
x = y =
82 82

' 227 289 '


Rpta:
\ 82 82 /

5) Hallar la ecuación de la mediatriz del segmento AB, donde : A ( -4 ; 3 ) y B ( 2 ; 9 )


Solución
- Para este tipo de problemas hay dos métodos. Veamos :

1er. M étodo :
- Sea L, mediatriz de AB.

- Las coordenadas de M, punto medio


de AB, son:
“ 4 + 2
En x -+ ------------= -1

En y -> i ± l = 6

M (-1 ; 6 ) , es un p u n to de paso de la recta L.

- La pendiente de L, es el inverso negativo de la pendiente de la recta AB

9 -3
Siendo : mAB = mAB ~ ^

Para la pendiente de L : mL . mAB = -1 mL . 1 = -1 -» mL = -1

Finalmente, con M (-1 ; 6 ) y mL = -1, usamos la forma punto pendiente, para la ecuación
de L : y - 6 = -1 ( x - (-1 ) )

De donde : y = -x + 5

2do. M étodo.- Si P ( x ; y ) , es un
punto genérico de L; entonces, por la
propiedad de la mediatriz :
PA = PB

904
J ( x - - 4f + ( y - 3 ) 2 =>/ ( x - 2 ) 2 + ( y - 9 ) 2

Elevando al cuadrado ambos miembros y simplificando, queda : y = -x + 5

6) Hallar la distancia del punto ( -4 ; 3 ) , a la recta L : y = 2x + 5

Solución
Se debe escribir la ecuación de L,
en su forma g e n e ra l:
Ax + By + C = 0

Se tiene :
y = 2x + 5
2x - y + 5 = 0
A = 2 , B = -1,C = 5

- Luego, en la expresión para la distancia d, con x1 = -4 ; y1= 3 :

A x, + B y, + C | 2( - 4 ) + ( - 1 ) ( 3 ) + 5
d=
J A2 + B V 22 + ( - 1 )

-8 -3 + 5 -6
d= d= | / 5 Rpta
Js J 5

7) Los vértices de un triángulo son los puntos A ( 3 ; 6 ) , B( -1 ; 3 ) y C ( 2 ; -1 )


Calcular la longitud de la altura trazada desde C.

A) 2 B) 3 C) 4 D) 5 E) 6
Solución

Se pide la distancia de C, a la recta


que contiene a A B , para ello hallamos
la ecuación de X § :
y -6 6 -3
________ _ 3x 15
x - 3 3 - (-1) y " 4 + 4

3x 15 .
o : ------- y + — = 0

o, m ejor: 3x - 4y + 15 = 0, es la forma
paramétrica de la ecuación de AB.
Ahora, hallamos la distancia del punto
C( 2 ; -1 ), usando la fórmula :
A x1 + B y1 + C
d=
para A = 3 , B = -3
/ a + B

905
\

y, = -1 , C = 15 , x, = 2,

3 x 2 + ( - 4 )( - 1 ) + 15 25
• • d= -> d= 5
i / 3' + ( ~ 4 )

Rpta: ( D ).

8) Dada la recta. L1 : 5x + 3y - 3 = 0.
Hallar la ecuación de la recta que pasa por M ( 2 ; 1 )

1o) Paralela a Lv

2o) Perpendicular a L ,.
\ L2
Solución \
\
L1 : 5x + 3y - 3 = 0 \
Pendiente m1, de L1
\
5 \
m, = 3 , \
\
X
5 \
ya que L1 : y = - - x + 3

5
1o) Sea l_2 11 L, : entonces, la pendiente de : m2 = m1 m2 = -
3

Con la pendiente y el punto de paso M( 2 ; 1 ), usamos la forma punto-pendiente para la


ecuación de :
y ■y0 = m ( x - x0 )

y- 1 = - f ( x- 2 )

5 13
De donde : y = - — x + — , es la ec. de L0 .
3 3 2

2o) Sea l_2 1 L1( desde el punto M.

Como debe cumplirse : m 1 x m3 = -1


-5 3
x m3 = -1 —> m3 = —
5

Entonces, para L3; con la forma punto pendiente :

y -1 = f (x-2)

y = T3 X + i1 - —
6

. 3 1
Rpta y ——x —
5 5

90R
9) Los lados de un triángulo están en las rectas x + 5y - 7 = 0 ; 3 x - 2 y - 4 = 0 ; x - y = 0
Hallar su área.

Solución
Se grafican las rectas.

L, : x + 5y - 7 = 0 -d ).
L , : 3x - 2y - 4 = 0 ..(II).

L,:x-y = 0 .......... ( II I ) .

- Se obtienen los vértices :

{A} = L, n L, De ( I ) y ( I I ) : x = 2 ; y = 1

A = (2 ;1)

{Bj = l , n l De ( I ) y ( I I I ): x = y = ■£

' 7 . 7 '
B =
<6 ’ 6 ,

ÍC} = L , n L, De ( I I ) y ( I I I ) : x = 4 ; y = 4
» • C = (4;4)
Entonces, para el área :

I D

I = 1x — + — x 4 + 4 x 2 -» 1= —
6 6 6

D = 2 x —+ —x4 + 4x1 D = 11
6 6

17 o = —
17 u 2
S = —| D —I| = — —> S Rpta
2 1 2 6 12

10) Hallar la ecuación de la recta bisectriz del ángulo agudo que forman las rectas

L1 : 3x - 4y + 6 = 0

l~2 : 24x - 7y - 177 = 0

9Ü7
Solución
Con la fórmula de la distancia de un
punto a una recta :
24x-7y-177|

V 24' + ( - y )
y, por propiedad de la bisectriz :
d, = d2
Luego :
|3x - 4y - 6| _ |24x - 7y - 177|
/2 5 ~ /6 2 5
Efectuando : 13x - 9y - 49 = 0
Es la ecuación de la bisectriz.

11) Los vértices de un cuadrado son: A ( 0 ; -3 ) , B ( b1; b2), C ( 3 ; 4 ) y D ( d1; d2).


Hallar el área del rectángulo cuyos vértices son los puntos B, P, D, Q, donde P = ( d , ; b2)
y Q = ( b 1; d a)
A) 58 B) 21 C) 19,5 D) 29 E) 25
Solución
Sea el gráfico
Ecuación de L

y _4 4-(-3)
x - 3 3-0

y = 3 x ’ 3

Luego: tg ó = — = ctg a

El área del rectángulo, puede


hallarse c o m o :

S = j ( PQ )( BD ) Sen 2a

Siendo: PQ = BD = AC = ,l ( 3 - 0 ) + 4 - 3 = J 58

'3 '

Sen 2 a =
2tga \ 7 , 11
1 + tg a 3 29
1+
V7

Reemplazando en (1 ): S = -1( ^ 5 8 ^ ~58 j . 11 = 21 Rpta: (B)


29
9<)8
\

12) La ecuación : x2 + 6x + y2 - 9 = O, es la ecuación de una circunferencia de centro ( h; k ) y


radio r.

H a lla r: E = -------
h+ k

Solución
Se tiene : ( x - h )2 + ( y - k )2 = r2 (1 ).
Desarrollando y ordenando, queda :
x2 - 2hx + y2 - 2ky + ( h2 + k2 - r2 ) = 0 ............( 2 ).

Comparando ( 2 ) con lo propuesto :


x2 + 6x + y2 - 9 = 0 ............( 3 ).

Deducimos, que : -2h = 6 -+ h = -3


-2k = 0 -> k = 0
h2 + k2 - r2 = -9 -> (-3)2 + O2 - r2 = -9

r = 3 /2

- Finalmente, reemplazando en E :

E = 3^ -+ E = -J~2 .......... Rpta.


-3 + 0 v
Otra form a: completando cuadrados en la ecuación dada :
x2 + 6 x + y2 - 9 = 0
Es decir ; escribiendo lo mismo como :

( x + 3 ) 2 + ( y + 0 ) 2 = 9 + 32

Entonces : h = - 3 , k = 0 y r = <JTq = 3 ^ 2

E=- ^ - = -/2 Rpta.


-3 + 0 v

13) En la figura : AB = 7 y OB = 24. A


Hallar la ecuación de la circunferencia
inscrita en el A ABO.

Solución
- OH y HP, dan las coordenadas del centro P.

- En el A ABO, por el Teor. de Pitágoras:


OA2 = OB2 + AB2 o C X

OA2 = 242 + 72

909
OA = 25
Ahora, con el Teorema de P oncelet:
OA + 2r = AB + OB

25 + 2r = 7 + 24 r = 3

Al trazar PB : Z PBA = Z PBO = 45

PB OX

Adem ás: PB = r / 2 = HC = 3 / 2

OB 24 i—
En el A OCB (45°, 45o): OC = BC = - = - ^ = 1 2 / 2 = HP
f2 yf2

Entonces : OH = OC - HC = 12/ 2 - 3 - /2 OH = 9 / 2

Entonces, las coordenadas del centro P, de la circunferencia, son :


( OH ; HP ) = ( 9 / 2 ; 1 2 -/2 ) y su ecuación:

( x - 9 / 2 )2 + ( y - 1 2 -/2 f = 9 Rpta
CAPITULO 3 3

VECTORES
Un vector es una magnitud que queda
determinada exactamente si se cono­
cen de él su dirección, sentido y mó­
*0
dulo.

En la figura 1, el vector AB, trazado


desde A ( llamado origen ), hasta B
(extremo), se indica colocando una
flecha en el extremo y se denota por Fig. 1.
ÁB.
Es también común denotar el vector con una letra minúscula colocada en el extremo ó al
costado: v.
A, también es llamado punto de aplicación del vector.
Entonces,para el vector de la figura 1:

* L, es la recta que define la d ire cció n de v.


* El sentido de v es de A hacia B.
* El m ódulo ó norm a de v , es la longitud del segmento AB y se denota p o r :

También pueden escribirse, simplemente v ó AB.

Dado un vector á , el vector opuesto, - á , es el que tiene el mismo módulo y dirección, pero
sentido contrario ( fig.2 )

Los vectores son iguales (equipolentes), si


están ubicados en rectas paralelas o en la
misma recta y tienen sus sentidos y módulos,
respectivamente iguales. Por ejemplo, para la
figura2,tenem os:á = b. Por!otanto,si á =3u,

e n to n ce s: = 3u. ( La u se lee : unidades ).

911
De lo anterior, deducimos que un vector se
puede trasladar en forma paralela a su
dirección, manteniendo el sentido, como
indica la figura 3.

fig. 3

SUMA DE DOS VECTORES - Se lla­


ma suma á + b, de los vectores
á y b , al vector que tiene por origen

el mismo que á y extremo el de b ,


luego de hacer coincidir el extremo de
á con el origen de b, trasladando fig. 4
alguno de los vectores o los dos, como
en la figura 4.

Este método de graficar el vector a + b, es conocido comoREGLA DEL TRIANGULO, y el


vector obtenido al sumar a y b , es conocido como VECTOR RESULTANTE de ellos.

Si hacemos coincidir el extremo de b con el


origen de a y trazamos el vector con origen
el de b y extremo el de a , estaremos
graf¡cando el vector b + a. ( Ver Fig.5 )

fig. 5 De lo anterior, podemos enunciar la propie­


b
dad conmutativa de la suma o adición de
dos vectores a y b :

a + b = b + a

La suma de muchos vectores se efectúa mediante la aplicación sucesiva de la Regla del


Triángulo, tal como se indica en la figura 6, donde se representa el vector resultante:

r=á+b+c+d

912
DIFERENCIA DE DOS VECTORES.-
La diferencia de los vectores á y b ,

se esbribe a - b, y representa otro


vector c , tal que sumado con b , resul­
ta el vector á. Es d e c i r , para restar
dos vectores, se suma al vector
m inuendo el opuesto del vector
sustraendo :

c = á - b = á + ( - b )

La fig.7 muestra la forma práctica de graficar el vector c = á - b . Aquí se han hecho coincidir

los orígenes de á y b - El vector c = á - b , une la punta de flecha de b con la de á .

- En el caso de que á = b, el vector á - b se llama ve cto r nulo o cero y se representa por


Ó ó O.

PRODUCTO DE UN ESCALAR POR UN V E C T O R E l producto de un escalar (número real),


por un vector á , es otro vector r á , de la misma dirección que á , pero con módulo | r | veces
el de á y sentido igual al de á si r es positivo y opuesto al de á si r es negativo. Si r = 0,
r á es el vector nulo.

fig. 8

PROPIEDADES.-Si a, b y c . s o n vectores ; m y n, dos escalares:

1) á + b= b + á
( Prop. conmutativa de la suma )

2)
( Prop. asociativa de la suma )
3) ma = á m
( Propiedad conmutativa del producto de un escalar y un v e c to r).
4) ( mn ) á = m ( ná )
( Prop. asociativa del producto por un e sca la r).

913
( m + n )á = má + na.
( Prop. distributiva, respecto a la suma de escalares ).

m ( á + b ) = má + nb
( Prop. distributiva, respecto a la suma de vectores ).

VECTOR UNITARIO.- Es todo vector de módulo unidad.

Si á es un vector de módulo distinto de cero, á * 0, el vector H = es un vector unitario

de la misma dirección y sentido que á a


A
Todo vector á , se puede representar , V "

-Y "
por el producto de un vector unitario
\
¡I, de la misma dirección y sentido que
aquel, multiplicado por el módulo de o
á, que es un escalar.

Entonces : fig. 9

VECTORES UNITARIOS RECTANGULARES.- Un sistema muy importante de vectores


unitarios son los que tienen por direcciones
las correspondientes a los ejes de un siste-
Vi
t ma de coordenadas cartesianas.
fig. 10

o
fig. 11

La fig. 10 muestra el caso en el plano Y


y la fig. 11, en el espacio.

Aquí trataremos sólo el caso del plano

VECTORES COMPONENTES.- Todo vector á, en el plano, se puede representar con su


origen en el de coordenadas.

Sean (a1 ; a2) las coordenadas del


punto extremo del vector á, en la
figura 12. Los vectores a, i , a2 j , se
llaman vectores componentes rectan­
gulares o simplemente vectores com-
ponentes de a, según las direcciones x , y , respectivamente

Entonces :

y el módulo :

LEY DEL PARALELOGRAMO.- Si a y b son dos vectores, el vector R = a + b, se llama


la resultante de ellos.

Para grafícar R , se han trazado paralelas graficando un paralelogramo de lados

El vector R se obtiene trazando la diagonal desde el origen común de los vectores á y b .


Usando la Ley de cosenos de la Trigonometría, se demuestra que el módulo del vector R,
se puede ca lc u la r:

2
R = J a 2 + b + 2 a b Cosa

Siendo a , la medida del ángulo que determinan los vectores es tal como se indica en la fig. 13
PROBLEMAS RESUELTOS

1) En la siguiente figura :

á = 5
b

b = 12

Hallar el módulo de la resultante de a y b .


Solución :
5 + b es el vector que tiene por origen el de á y extremo el mismo que b .

á + b , es la longitud de la hipotenusa del triángulo rectángulo obtenido . Luego, haciendo


uso del Teorema de Pitágoras :

á + b +

a + b = 52 + 122

.4
á + b = 169

Rpta: á + b = 13

2) ABCD, es un cuadrado de lado 2


unidades de longitud.
Hallar :

MA A? MA + AP

Solución :

* El módulo de MA es la longitud del segmento MA.


En el A ABM : ( MA )2 = ( AB )2 + ( BM )2

( M A ) 2 = 22 + 12 MA = / 5

Rpta MA = / 5

Análogamente; en el A ADP ( AP )2 = ( AD f + ( PD )2
( A P ) 2 = 22 + 12

AP = / ?

Rpta AP

916
Para hallar el módulo de MA + AP, usamos la “Regla del Triángulo” para la suma de dos
vectores. A s í : MA + AP = MP.

E ntonces: MA + ÁP MP
t
Siendo, en el A MCP : MP = J 2

MP « V 2

1
Rpta: MA + AF5 =

B Para el rectángulo ABCD, de la figura

H a lla r:

á + b

Solución

á+ b = 9

Q
Hacemos coincidir el extremo de a
con el origen de b , trasladando b de
modo que c 3 = b , como en la figura
de la solución.
Luego, en el A ACQ : AQ = á + b.

La longitud del segmento AQ, la pode­


mos hallar en el A APQ, usando el
Teorema de P itágoras:

( AQ Y = ( AP )2 + ( PQ )2
( A Q ) 2 = 82 + 62 -> AQ = 10

Por lo t a n t o : AQ = 10 Rpta: = 10

Según el gráfico adjunto


Hallar el módulo de :

á+b

fig -1

917
Solución
El triángulo formado por los vectores
a y b , es isósceles.
6 6u
= 3^2

a+ b
Trasladamos á como en la figura 2.

fig. 2
Rpta: á + b = 6u

5)
B

La longitud de la arista del cubo de la figura,


mide 4 unidades. Hallar el módulo de :
Á § + GE

Solución

Incógnita : AB + GE

Como los segmentos BD y GE son


congruentes y paralelos , entonces
los vectores
BD y GE serán iguales : BD = GE.
Luego : ÁB + GE = AB + BD

Es decir Ja resultante de AB + GE, es


la misma resultante de AB + BD
En el gráfico de la solución notamos
que :
ÁB + BD = Á D , haciendo uso de la
“Regla del triángulo”. Por lo tanto :

AB + GE AB + BD AD = AD = 4^]~2 , por ser A D , hipotenusa del A AED

6) Cada cuadrado pequeño de la figura, tiene longitud de lado 1 unidad


H a lla r:

a+b+c+d

mu
a^

7
Solución
Para hallar el vector resultante de los
vectores á , b , c y d , trasladamos
b colocando su origen en el extremo
de a; el origen de d lo hacemos
coincidir con el extremo de b y el
origen de c con el extremo de d .
Luego, el vector r que va del origen de
á al e x t re m o de c, es
r = á + b + d + c, resultante pedida.
En el triángulo rectángulo de catetos
3 y 5 , hallamos el módulo de r :
2 2 2
= 3 + 5 = /3 4

Rpta á + b + c + d = /3 4

7) En la figura adjunta:
o
AB 2
BC “ 3
S i:
b = má + nc
siendo m y n escalares
H a lla r: m . n

B) C) D)
25 25 25

Solución

Del gráfico : AB = b - á = c - b

AB 2
Como, según dato : AB = - B C
BC 3 3
2 - - 2 -
De donde : b + —b = a + —c
3 3

• •

2
Comparando con el enunciado : m = — y n
5 5

Luego : m . n =
25

Rpta: ( B ).

En el hexágono regular ABCDEF, de la


figura :
f 1'
BC + + = r AB + n
\ 3 /
Hallar: n + r

11 10
A) B) C) -3
3
D) -4 E) N.A

Solución
La expresión propuesta se puede es
cribir así:
/ 1
BC + CE = rA B + n —
3 J

Ahora, con el gráfico tratemos de ex­


presar BC + CÉ , en términos de
ÁB y EF.

Se tiene, con los trazos efectuados :


BC + CE = BE = 2BO = 2 (-Á É T + ~OÁ)

Es d e c ir:
BC + CÉ = 2 A § + ( - 2 ) O A

ó, m ej or : BC + CE = - 2 A B + ( - 2 )EF, ya que OA =

Luego, comparando esta última expresión para BC + C E, con la (1 ) :


r = -2
1
n — - = -2 n= —
3 3
5 11
Por lo tanto : n + r = - 2 = -
3 3
Rpta: ( A ).

9) Para el gráfico adjunto , demostrar


que:

AC AC

Solución
B
Del gráfico, podemos escribir
AE = BE - BA

v EC = BC - BE

AE
A dem ás: AE = EC
EC
Reemplazando las dos primeras expresiones en la tercera

BE - b K = — ( § 5 - B E )
EC
AE AE
BE + = BA +
EC EC
Es d e c ir:
( AE + EC ) § t = EC . BA + AE . BC
Pero : AE + EC = AC.

Luego :
AC . BE = EC . BA + AE . BC

De donde EC . q
BA/T + -----
ae Tal como se quería demostrar
AC AC

10) En la figura adjunta, ABCD es un


paralelogramo y M, punto medio de AD.
S i:
M(5 = nBC + r D C ; siendo n y r escalares
( números).
n
H a lla r:

1 1 1 1
A) g B) - - E) N.A.
C>4 ° ) - 4

921
Solución

Del gráfico : AM + MQ = AQ ........... ( 1 )

Siendo : ÁM = - Á D = - B C ( 2 ).
2 2
AQ AM AQ
Y, el A AQM semejante al A BQC :
QC BC QC

AQ = —QC AQ = - A C . Luego : AQ = - A C
2 3 3

Pero : Á ^ = Á $ + D $ . Entonces : Á S = AD + DC )

ó mejor aún : AQ = —( BC + D C ) ............( 3 ).


3
ya que BC = ÁD.

Reemplazando las expresiones ( 2 ) y ( 3 ) en la (1 ) :

- bc + m3 = - ( b6 + dS )

Despejando m 2? :
1
MQ = — BC + - DC
6 3
Comparando con :
MQ = nBC + rDC, que es lo propuesto,

tenemos : n = y r= —
6 y 3

Se pide : — = —— = —-
r 1

Rpta: ( B ).

11) En la figura, ABCD es un parale­


logramo.
AM = M B ; A N = ND y RD = 2RC.

S i:
MN = mQD + nBR, donde m y r son
escalares; h a lla r: m + n

A) 9/2 B) 2/9 C) 4 A
D) 3 E) 11/2

922
Solución
Del gráfico :
MN + ÑD = MD
Siendo Q , Baricentro del A ABD :

MD = - Q D

MD = — QD. R e e m p la z a n d o

esto, en la a n te rio r expresión :


N D
MN + Ñ 5 = - Q D
2

De donde : MÑ = —QD - ND ( i )•
2

Ahora, debemos expresar ND en términos de alguna combinación de MN, QD ó BR,para


reemplazar en (1 ). Veamos : BC + CR = B R .

Pero : BC = ÁD = 2 Ñ 5 y c B = ~ c B = - B ^ = - ( 2 M ^ )
3 3 3

Entonces; reemplazando en la anterior expresión que relaciona BÜ, CR y

2ÑD+ - ( 2 M ^ ) = B^

ó : 2N D +-M A =
3

Siendo, además : MA = MN - AÑ ; entonces :

2ÑD + - ( M N - Á N ) = BR

Y, como también : AN = ND, podemos e scrib ir:

2 ñB + - ( m n - nd )=

Es d e c i r : — ND + — MN = BR
O O

Despejando ND : ND = — . B R MN

Ahora, reemplazando esto último, en (1 ) :

l 4
í
MN = — QD - —BR - —MN
2 2 /
923
Efectuando : MN - — MN - - Q D - - B R

De donde: MN = 3Q D - —BR
2
3
Comparando con la expresión propuesta en el enunciado : m = 3 y n = —

Se pide : m + n = —

Rpta: ( A ).

12) Demostrar que si á y b son dos vectores no paralelos, no nulos, y : rá + kb = 0 ; enton­


ces : r = k = 0
D em ostración

De rá + Kb = 0, se deduce que : rá = - K b . Supongamos que r * 0. Luego; podemos

despejar: á = - r ü ] b
r )

Lo cual indica que á || b, contradiciendo la hipótesis de que a y b no son paralelos.


Entonces, nuestra suposición de que r * 0 es falsa. Por lo tanto : r = 0. Y, de kb = 0,
también : k = 0
Q
Ejem plo: Para el gráfico adjunto :
r 2- 5-
b = —a + —c
7 7
QM
Hallar el valor de :
MT

Solución

Sea : = k. Entonces : QM = k ( MT )
MT

En forma ve cto ria l: QM = k M T (1 ).

Pero, del gráfico : QM = b - á y M T = c - b

Reemplazando estas dos relaciones en ( 1 ) :

b -á = k (c -b ) có b - á = k c - kb

Usando la expresión dada en el enunciado para b :


\
2- 5- - . '2 - 5-
—a + —c - a = k c - k —a + —c
7 7 /

924
Efectuando:
( 2k - 5 ^ f 5-2k >
á + c = 0
l 7 ) ^ 7 j

Como a y c, no son paralelos, entonces por la propiedad anterior

2 != _ 5 =0 y i ^ = 0

De ambas : k = —
2
QM 5
Rpta
MT 2

H
Hallar el módulo de la resultante de los
vectores mostrados , si el cubo tiene
arista con longitud 2u.
( HM = EM )

Se p id e : R , d o n d e :

R = AC + BG + MF (I)

Sea N, punto medio de FG

Entonces : HN = MF

De otro lado : CH = BG.

Luego, en ( I ) :
R = ÁC + CH + HN

Pero, según el gráfico el segundo miembro de esta expresión, es : AN R = AN

R = AN = V 2* + 12

Rpta R =

En la figura :A E = (1/2)EB, EM = MF
y CF = FG

S i: G

ÁM = mAB + nBC + rMG


925
H allar: m + n + r
11 10
A) 1 B) C) D)
11 E>3
Solución

Debemos expresar el vector AM, en


función de A B , BC y M G , para luego
comparar con el dato y hallar m, n y r.

Se observa : EM = AM - AE
1
EM = AM - —AB (1 )•
3

También : EF = EB + BC + CF

ó, m e j o r : EF = —AB + BC + FG , ya que EB = —AB y CF = FG


3 3

Por otro lado : FG = MG - MF

Luego, reemplazando en la última expresión de EF : EF = —AB + BC + MG - MF


3
1 1
ó, también : EF = —AB + BC + M G EF , ya que MF = —EF
3 2 7 2

De donde : - E F = - A B + BC + MG
2 3

4 — P —^ p
Por lo tanto : -A B + -B C + -M G (2)
9 3 3

Entonces, como : EF = 2EM

Reemplazando las expresiones (1 ) y ( 2 ), en ésta última :


\
4 —* p —* 2
— AB + —BC + —MG = 2 AM - —AB
9 3 3 3

1 1
Luego : AM = - Á 5 + - B C + - M G
y 9 3 3

5 1 1
Comparando con el dato :m = — ,n = — , r = —
9 3 3

5 1 1 11
Finalm ente: m + n + r = —+ — + —= —
9 3 3 9

Rpta: ( C ).
926
15) Para el A ABC de la figura adjunta ,
AN y BM son medianas. Como sa­
bemos, ef punto Q se llama Bari­
centro.

Demostrar, usando pro p ie d a d e s


vectoriales, q u e :
AQ = 2QN

Solución

* Tenemos : BQ = kBM ............( 1 ) ; para algún escalar k.

* Por otro lado, sabemos que, si BM es mediana del A ABC, se cumple la relación vectorial

• BM = —BA + —B C (2).
2 2
* Además, del problema anterior, se ha demostrado que :

BQ = — . BA + — . BÑ
AN AN
1
Pero, como BN = — BC; la última expresión se puede escribir:

BQ = — . BA + — - . BC (3 )
AN 2 AN

* sustituyendo las conclusiones ( 2 ) y ( 3 ) , en la expresión (1 )

— . BA + . BC = k - B A + - B C
AN 2 AN \ 2 2
o , m ej or :
O
<

( QN k
{ AN 2 )
. BA +
2AN
k]
2
BC = 0

Pero, como BA y BC no son paralelos, la anterior expresión será cierta, sí y sólo sí

-A Q -J L .o
AN 2 2 AN 2

Es d e c i r :
QN k AQ k
— =- y
AN 2 2 AN 2

Por lo tanto, igualando los primeros miembros de estas últimas dos relaciones :
QN AQ
AN " 2 AN

De donde, efectivamente :

927
* Observación: Hemos demostrado la propiedad del Baricentro en cualquier triángulo. V,

como AQ = 2QN, entonces AN = 3QN ; luego.al reemplazar en : = — , se obtiene


AN 2
2
k = ■
— , lo cual era de esperar.
3

33.2. VECTORES EN EL PLANO CARTESIANO


1.- Como sabemos, un vector es una magnitud con dirección, sentido y módulo o longitud,
conocidos.
En el plano cartesiano XY, el vector a = ( a , ; a2), se puede representar tomando un punto
de partida P, que va a ser el origen del vector, y a partir de él hacer un desplazamiento paralelo
al eje x, una distancia dirigida a,, hacia la derecha si a, > 0 ó hacia la izquierda si a 1 < 0, para
luego continuar con un desplazamiento paralelo al eje Y, una distancia dirigida a2, hacia
marriba si a2 > 0 ó hacia abajo, si a 2 < 0, determinándose el punto de llegada Q. Luego, el
vector á , se representa por P(5.

ii Y Q i
[ Y p a-17 P
a2 ! /
V a

ai Q

0
o X
X
^1 > 0 a 1< 0
fig. 1 a2 > 0 fig. 2 82 < 0

Yi
Y- p ai [ Q

V '"Q a N ja
a2: V
! 3p
üN j
a1 p

0
X 0
X
a^> o <o
fig. 3 < 0 fig. 4 a2 > 0

928
- La elección del punto P, es arbitraria.

2.- Radio Vector.- Es aquel cuyo origen coincide con el origen de coordenadas.

En la figura 5, si las coordenadas del


punto Q, son ( at ; a2) :
a = (a, ; a2)
V A
Entonces, el módulo, longitud o la
norma del vector á , es la distancia OP Q (a ! ; a2)

y se simboliza á , evaluándose a s í :

“ V ( a 1 f + ( a2 ) o X

fig. 5
* Se puede notar además, que :

ai = sen a

a2 = eos a

3.- Y En la figura adjunta , i es un vector de


S£ origen P( x1, y1) y extremo Q( x2, y2 ).
La recta L dá la dirección del vector. El
(x2 ;y2) sentido para á es de P hacia Q y su módulo
o norma es la longitud o la distancia entre
los puntos P y Q.

Luego, escribiremos simbólicamente :


x a =
fig. 6
Las componentes de á son sus proyecciones (positivas o negativas, según su sentido) sobre
los ejes cooerdenados. Así, para la fig. 6 :
a = ( x2 - x, ; y2 - y , )

Es decir, “Las componentes del á se obtienen restando las componentes del punto P a las
del punto Q”. De modo que :
á = PQ = Q - P = ( x2; y2) - ( x , ; y , ) = ( x2 - x, ; y2 - y , )

Ejm .1: Dados los puntos P( 2; 3 ) y Q( 5; 7 ).

H a lla r: PQ

Solución

Se tendrá : PQ = Q - P = ( 5; 7 ) - ( 2; 3 )

PQ = ( 3; 4 )
929
4.- Módulo, norma o longitud, del vector á es la distancia entre los puntos P y Q, de la fig. 6
En símbolos : a =. d( P,Q )

= v ( * 2 - x i )2 + ( ya - y i )

Ejm .2: Hallar el módulo de PQ, para el ejemplo 1 anterior.

Solución

PQ = ( 3 ; 4 ) PQ = ^ 3 2 + 4 2 = 5

5.- Dos vectores se llaman iguales, si tienen iguales el módulo, la dirección y el sentido.
Así, en la figura 7 : PQ = R S .

Es evidente que las rectas que u


dan las direcciones de p 3 y RS,
son paralelas entre sí.

( Notar que P, Q, S y R son


vértices de un paralelogram o). K R fig. 7

Ejm.3: En la figura 7, anterior, se conocen las coordenadas P( 2; 3 ) , Q( 5; 8 ) y R( 6; 9 ).


Hallar las coordenadas de S.

Solución

Como : RS = PQ
Entonces : S - R = Q - P
Luego : S= R+Q-P
S = (6;9) + (5;8)-(2;3)

S = ( 9 ; 14 ) Rpta.

6.- Si r es un número real positivo, entonces rá es otro vector r veces mayor que á, de igual
dirección y sentido.

Así, dado á . = ( a 1; a2 ) , entonces | á | = ^ f + ( a2 )2

Luego, las componentes del vector r á ,


son : rá = r( a1 ; a2 ) = ( ra1 ; ra2 ) y el módulo de rá :

= V ( ra 1 ^ + ( f a 2 ^

ra = i r2 ( 3 1 f + r2 ( a2 f

ra =J 1-2 ( ( a i f +( a 2 f )

930
ra = r i ( a l ) 2 + ( a 2 f
Esto es : ra = r

Aquí r representa un escalar


( número re a l)

fig. 8
Ejem plo 4 . - Dado el vector á = ( 8 ; 15).
Hallar las componentes y el módulo del vector 2 á .

Solución

2a = 2( 8 ; 15 ) = ( 16 ; 30 ) 2á = ( 16 ; 30 )

2a = 2 = 2 y[ ¥ T :\ ¥ = 2 x 1 7 = 34

7.- Si re s un número negativo entonces


rá esotro vector) r | veces mayor que
á , de igual dirección y sentido opues­
to que á.

El módulo de rá será : ra r

Ejm.5.- Dado el vector á = ( 3 ; 4 )

Hallar las componentes y el módulo del vector - 3á

Solución

-3á = - 3 ( 3 ; 4 ) = ( - 9 ; - 1 2 )

y el m ó d u lo : -3 á = -3 = 3 = 3/3 + 42 = 1 5

8.- Si dos vectores tienen la misma medida y


dirección pero sentidos opuestos, uno de
ellos es el negativo del otro. Así, en la
Q
figura 10, adjunta :
SR = -P Ü

-á es el vector opuesto
del vector á . fig- 10
931
VECTORES PARALELOS
De las menciones en los números 5 al
8, podemos decir que: Dos vectores V*

á y b , son paralelos, sí y sólo sí,


existe un número real r, único, tal que
b = rá .
Entonces, escribiremos : á Ib

Ejm.6.- Dados los vectores: á = ( 2 ; 7 ) y b = ( n ; n + 3 )


Hallar el valor de n, si a 11 b .

Solución

Como á I b a = rb , para un r único

(2 ;7) = r(n ;n+3)


( 2 ; 7 ) = ( r n ; r n + 3n)

De donde : r n = 2 .................... (1 )
r n + 3n = 7 ........... (2)
5
Reemplazando ( 1 ), en ( 2 ) : 2 + 3n = 7 -» n =
3

Nota.- Si reemplazamos n = — , en ( 1 ),

6
se obtiene : r = —
5

Entonces : a = —b
5

Ejm.7.- Dados : á = ( 3 ; 4 ) , b = ( —I — ) , c = ( 8 ; 6 )

¿ Cuáles son paralelos ?

Solución
Planteamos la condición de paralelismo entre dos vectores:

Si a | b —> 3 r único, tal que : á = rb


r
De donde : —= 3 —> r = 12
4 Como e! r obtenido en ambos casos, es
el mismo; entonces : á b.
—= 4 r = 12

Si á 3 r único, tal que : á = rc

( 3 ; 4 ) = r( 8 ; 6 ) ( 3 ; 4 ) = ( 8 r; 6 r)

De donde : 8r = 3 -> r= -
8 Como los valores de raqui obtenidos, son distintos
2 entre sí, entonces a No es paralelo a c .
6r = 4 —> r = —
3

Es evidente que, por ser b á , b no es paralelo a c .

Ejm.8.- Sea M, punto medio del seg­


mento AB. B

D em ostrar, usando pro p ie d a d e s Yi


vectoriales, que las coordenadas de
M, se obtienen :
A +B X
M=

Solución

Como M, es punto medio de A B :


AM = MB

Entonces, por ser además AM MB,


podemos e scrib ir:
= M § ( Igualdad de vectores. Ver apartado 5 ).

Es d e c ir:
M-A = B-M ( Definición dada en el apartado 3 )

■ • 2M = A + B -» m = A + B .......... I.q.q.d

Ejm.9.- Hallar las coordenadas del punto medio M, del segmento AB, si A(2; 3) y B(6; 7)

Solución

A + B _ ( 2 ; 3 ) + (6;7)
Por lo anterior: M =

933
\

M
(2+ 6;3+7) ( 8 ; 10) f8 10 >
,2 = 1 /
Rpta: M = ( 4 ; 5 ).
i
Ejm.10.- En la figura adjunta: A (3 ; 6)
y C (15 ; 12)

S i: AB = 2BC, hallar las coordena­


das de B.
o X
Solución
Este problema se puede resolver con la teoría del capítulo anterior, Introducción a la
Geometría Analítica, pero lo haremos vectorialmente.

Es evidente, que AB BC, dada la figu­


ra. Entonces, como AB = 2BC { Relación
de longitudes
ÁB = 2BC { Relación vectorial
por ser AB BC

Luego : B - A = 2(C - B) B - A = 2C - 2B

3B = 2C + A

Reemplazando d a to s : 3B 2( 15 ; 12 ) + ( 3 ; 6 )
3B ( 30 ; 24 ) + ( 3 ; 6 )
3B ( 30 + 3 ; 24 + 6 )
3B ( 33 ; 30 )
1 < 33 30 \
B = - ( 3 3 ; 30) =
V /
Rpta: B = ( 11 ; 10 )
B

Ej m. 11 En la figura adjunta :
M, es punto medio de AB.
N, es punto medio de BC.

Si, A (7;7) M(10; 12) y N (15;9) Yf

Hallar las coordenadas de C. o


X
Solución

- De la Geometría Elemental, sabemos que MN AC y AC = 2MN. Entonces, podemos


escribir, vectorialmente :
AC = 2MN

C -A = 2 [ N - M ]

934
C = A + 2[ N - M ]
C = (7;7) + 2[(15;9)-(Í0;t2)]
C = ( 7 ; 7 ) + 2 [ ( 15 - 1 0 ; 9 - 1 2 ) ]
C = ( 7;7 ) + 2( 5 ; -3 )
C = ( 7 ; 7 ) + ( 10 ; -6)

Rpta: C = ( 17; 1 )

Ejm.12.- M, N y Q, son puntos medios B


de los lados del A ABC.

Demostrar que, en coordenadas :

M+N+Q = A+B+C
Y*
Solución
o
X
Con fórmula del punto medio
A + B
M=
2
B+ C
N=
2
A + C

- Sumando miembro a m iem bro:


2A + 2B + 2C
M + N+ Q =

M+N+Q = A+B+C l.q.q.d.

PRODUCTO ESCALAR DE DOS VECTORES.- El producto escalar, (conocido también


como producto interno), de dos vectores á = (a1; a2) y b = ( b , ; b2), se denota por á . b
y se e v a lú a :
á . b = a1b1 + a2b2

O bservaciones:

1o) á . b , es un escalar ( número re a l), no es un vector.

2o) El producto escalar de dos vectores, cumple la propiedad conmutativa. Es decir


á .b = b .á ; hecho que puede verificarse fácilmente.

3o) El producto escalar, cumple la propiedad distributiva :

935
4 o) á . á = a, x a1 + a, x % = ( a, )2 + ( )

) á . a se llama cuadrado escalar del ve cto r


a .a =
a y resulta igual al cuadrado del módulo.

5o) ( a . b )c * á ( b . c j, no se cumple la propiedad asociativa entre tres vectores

6o) Si r es un número re a l:

rá . b = ( rá ) . b = á .( r b )

Ejem plo 13.- Dados los vectores : á = ( 2 ; 3 ) , b = < -1 ; 2) y c = ( 5 ; 7 ), h a lla r:

1°) á .b

2o) á.( b + c )

3o) á .b c

4o) á. b . c

5o) á .á

Solución

1°) á . b = ( 2 ; 3 ) . (-1 ; 2 ) = 2 ( - 1 ) + 3 x 2 = -2 + 6 = 4

2o) á (b .c ) = ?

* Prim era form a.- Efectuando previamente la suma indicada :

á.(b+c) = ( 2 ; 3 ). ((-1 ; 2 ) + ( 5 ; 7 ))

= ( 2 ; 3 ) . (-1 + 5 ; 2 + 7 )
= ( 2 ; 3 ) . ( 4 ; 9 ) = 2 x 4 + 3 x 9 = 8 + 27

á.( b + c ) = 35

* Segunda form a.- Con la propiedad distributiva

.( b + c ) = a . b + a . c

= ( 2 ; 3 ) . ( -1 ; 2 ) + ( 2 ; 3 ) . ( 5 ; 7 )
= 2 (-1 ) + 3 x 2 + 2 x 5 + 3 x 7
= -2 + 6 + 10 + 21 = 3 5

• • a .( b + c ) = 35

936
3o) á.b ]c = [ ( 2 ; 3 ) . (-1 ; 2 ) ] ( 5 ; 7 )

= (2(-1) + 3 x 2](5;7)
= [ -2 + 6 ] ( 5 ; 7 )
= 4( 5 ; 7 ) = ( 20 ; 28 )

• • [ a . b ] c = ( 20 ; 28 )

4 o) a[b.c] = ( 2 ; 3 ) [ (-1 ; 2 ).( 5 ; 7 ) ]

( 2 ; 3 )[-1 x 5 + 2 x 7 ]
( 2 ; 3 ) [ - 5 + 14]
( 2 ; 3 ) 9 = 9 ( 2 ; 3 ) = ( 1 8 ; 27)

b . c ] = (18;27)

5o) á .á = 2 = 22 + 32 -> á . á = 13

ANGULO ENTRE DOS VECTORES


Sea a, medida del ángulo que deter­
minan dos vectores á y b .( Ver figura
adjunta). Entonces, usando la ley de

cosenos entre las longitudes

y á - b , de los lados del triángulo,


tenemos:

á - b + - 2 Cos a

Pero, por la observación ( 4o ) :

á - b = (5 -b ).(á -b )

Reemplazando en lo a n te rio r:

( á - b ) . ( á - b ) = |á + - 2 Cos a

Desarrollando el primer miembro :

á . ( á - b ) - b . ( a - b ) = |á |2 + - 2 Cos a
- |2
a.á-a.b-b.a-b.b = a + - 2 i Cos a

937
Recordando que a . b = b . a :

á .b - á .b - + - 2 Cos a

Simplificando : - 2 ( a . b J = - 2 Cos a

Cancelando -2 :

Es d e c ir: “ El producto escalar de dos vectores, es igual al producto de sus módulos, por el
coseno del ángulo entre ellos

- De la anterior fórmula ; se puede despejar cos a :

Ejm.14.- Dados los vectores: a = ( 3y¡~2 ; ^ 6 ) y b = ( <J~3 ; 3 j


Hallar la medida del ángulo entre ellos.
Solución
Se evalúan, previamente :

á . b = ( 3 / 2 ; / 6 ) . ( / 3 ; 3) = 3 / 2 x ^ 3 + 3 / 6 = 6 / 6

3/2) + (/6 ) = V 18 + 6 = / 2 4 = 2 / 6

= J (/3 f + 32 = yj 3 + 9 = / Í 2 = 2 / 3

- Entonces, usando la fórmula ultima :

á .b 6 /6
Cos a = Cos a =
2 /6 x 2 /3

_ /3 ' / 3 '
Cos a = a = Arceos • • a = 30°
V /

YA

o X

938
VECTORES PERPENDICULARES.-Si dos vectores á y b son perpendiculares entre sí,

( á I b ) , entonces el coseno del án­


gulo que forman es cero , ya que : b\
eos 90° = 0

Luego :
V*
á .b = á b eos 90°
0
Implica :

á .b = a b x 0 -»

Ejm.15.- Hallar n , si los vectores: á = ( 2 ; n ) y b = ( 3 ; 4 ), son perpendiculares entre


sí.

Solución

- Como á 1 b, entonces : á . b = 0

Luego : ( 2 ; n ) . ( 3 ; 4 ) = 0

2 x 3 + 4n = 0
3
Rpta; n = -

O bservación.- El hecho de que el producto escalar de dos vectores sea cero, es condición
suficiente para afirmar que ellos son perpendiculares entre sí.

Ejm .16.- Si á y b son dos vectores paralelos y tienen el mismo sentido, hallar á . b,

sabiendo que á = 5 y = 3

Solución

á .b = eos a

Como tienen el mismo sentido : a = 0

á .b = eos 0o

/
á .b = 5 x 3 x 1 //

Rpta: á . b = 15

O bservación.- Si dos vectores son paralelos y del mismo sentido, su producto escalar es
igual al producto de sus módulos.

939
E jm .17.-S iá y b , son dos vectores paralelos y de sentidos opuestos, hallara . b , sabiendo

que: = 2 y = 7

Solución
- En este caso , a = 180

a .b = eos 180

a .b = - 2 x 7 = - 1 4

O bservación.- Si dos vectores son paralelos y de sentidos opuestos, su producto escalar


es igual al negativo del producto de sus módulos.

ORTOGONAL DE UN VECTOR.- Dado el vector a = (a1; a2) se define el vector ortogonal


de á , como aquel perpendicular a él y que tiene su mismo módulo, ( á 1 , designa al vector
ortogonal de á ), y de modo que :
Y*
á1 = ( -a2 ; a , )

- Por cierto: á 1 = ( a 1 ; a 2) 1

y a .á1 =0 “ oí X

Ejm.18.- Hallar el ortogonal del vector á = ( 3 ; 5 )

Solución

á = (3,5 ^ = (-5:3)

Ejm.19.- Dado el v e c to r: á = ( 2 ; 3 )

H a lla r: ( á 1 )

Solución

(a1 ) , es el vector ortogonal del á 1

- Primero se halla : á 1 = ( 2 ; 3 ) 1 =(-3;2)


/. á 1 = (-3 ; 2 )
- Luego :

(a 1 )1 = ( - 3 ; 2 ) X = (-2 ;-3 )

940
O bservación.- Si se aplica sucesivamente la definición de vector ortogonal, a un vector dado
a :

i * 1 )1 - - -
Yt
- i i \± -i
a = -a
/

/ . \±
a =a ' (S W
\ /
i
a1 = a , donde k = número natural
-
y

4k veces

ECUACION VECTORIAL DE UN RAYO


Sea Rab, el rayo de origen A y que
pasa por el punto B.
Rab, es el conjunto de todos los pun­
tos P, del plano, que cumplen :
X''
AP = r AB , r > 0 (I) p


Si r = 0 -» P coincide con A.
Si r = 1 P coincide con B. r
0 X
Si o < r < 1—> P está entre A y B.
Si r > 1 -> P está, sobre el rayo, a la derecha de B

De ( I ) : P - A = r AB

P = A + rA§,r>0

E ntonces:
RAB = { p ( x ; y ) / P = A + r Á § , r > o } (II).

( Ecuación vectorial del rayo de origen A, y que pasa por B )

Ejm.20.- Hallar la ecuación vectorial del rayo de origen A ( 2 ; 3 ) y que pasa por B( 5 ; 4 )

Solución
Según ( I I ):
RAb = { P( x ; y ) / p = A + r Á§ , r > 0 }
941
• *
r ab = { p( x ; y ) 1p = a + r [ B - A ], r > 0 }

RAb = í p ( x ¡ y ) ^ ^ = ( 2 ; 3 ) + r [ ( 5 ; 4 ) - ( 2 ; 3 ) ] , r > 0 }

Rpta: R AB = { ^ ( x » y ) 7 p = ( 2 i 3 ) + r ( 3 ¡ 1 ) * r ^ 0 }

E jm .2 1 S e a RA, el rayo de origen A,


paralelo al vector á .
A = (-1 ; 3 ) y á = ( 3 ; 1 ) , hallar la
ecuación vectorial de RA’

Solución
En este caso, se tienen como datos el
origen del rayo y el vector á , paralelo
a él. A este vector á , se le llama
“vector direccional”.

Se procede a s í: Tomamos un punto


P, genérico, del rayo RA. Luego, como
Ra l i a y tienen el mismo sentido
podemos p la n te a r:
rá , r > 0
Es d e c ir: P - A = r á . r > 0

De donde : P = A + r 5 , r > 0

- Ahora, diremos que :


R = {P = ( x ; y ) / P = A + r á , r > 0 }

Es la ecuación vectorial del rayo RA, conociendo el origen A y un vector direccional á.

- Reemplazando d a to s :

Rpta: RA = { P = ( x : y ) / P = ( -1 ; 3 ) + r ( 3 ; 1 ) , r > 0}

Ejm.22.- Para el rayo RA, del ejemplo anterior, ( n° 21 ) , h a lla r:

a) Las coordenadas de P, para r = 2.


b) El valor de n, si el punto Q( 6 ; n ), pertenece al rayo.
c) Indicar, si el punto C( 14 ; 9 ) pertenece al rayo.
11
d) Indicar, si el punto D( 1 ; — ) pertenece al rayo.
O
Solución
Se tiene :
Ra = { P = ( x ; y ) / P = ( - 1 ; 3 ) + r ( 3 ; 1 ) , r > 0 }
942
a) Si r = 2 :
P = (-1 ; 3 ) + 2 ( 3 ; 1 ) -> P = (-1;3) + (6;2)

Rpta: P = ( 5 ; 5 ).

b) Si Q( 6 ; n ) e RA, entonces :Q = ( - 1 ; 3 ) + r ( 3 ; 1 )

Luego : ( 6 ; n ) = ( -1 ; 3 ) + r ( 3 ; 1 )
(6;n) = (-1 ; 3 ) + ( 3 r ; r )
( 6 ; n ) = (-1 + 3r ; 3 + r )

De donde : 6 = -1 + 3r ..........( 1 ).
y : n = 3 + r ...........( 2 ).

- Para hallar n,de la expresión ( 2 ) despejamos : r = n - 3 y reemplazamos esto en (1 ) :


6 = -1 + 3 ( n - 3 )

16
Resolviendo : Rpta: n = —

c) El punto C(14 ; 9), pertenece al rayot sí y sólo sí, para un único r , se cumple :
C = (-1 ; 3 ) + r( 3 ; 1 )

Veamos : ( 14 ; 9 ) = (-1 ; 3 ) + r ( 3 ; 1 )
( 14; 9) = (-1 ; 3 ) +( 3 r ; r )
( 14 ; 9 ) = (-1 + 3 r ; 3 + r )

De donde :f 14 = -1 + 3r -» r = 5
9 = 3 +r -» r = 6

Como se obtienen dos valores diferentes de r, entonces el punto C (1 4 ; 9 ) no está contenido


en el rayo.

11
d) Si D(1 ; — ) e RA, entonces; debe c u m p lir:

D = (-1;3) + r(3;1),
para un único r. Veamos :

( 1 ; y ) = (-1 ; 3 ) + r ( 3 ; 1 )

( 1 ; " T ) = (-1 + 3 r ; 3 + r )

De donde: f 1 = -1 + 3 r -> r = -
O
11 2
T * 3*' -> r = 3
943
Como el r obtenido es el mismo, D pertenece al rayo

ECUACION VECTORIAL DE UNA RECTA

Sea L, una recta que pasa porel punto


P0 y además paralela al vector V,
llamado un vector direccional de L.
Entonces, L es el conjunto de puntos
P = ( x i y ) que cumplen :

Pb? = rV , r e R I ( I ).

Si r = 0 P coincide con P0.


Si r > 0 P está a la derecha de P0 ( Sobre L )
Si r < 0 P está a la izquierda de P0 ( Sobre L )

* Luego, de ( I ) : P - P0 = r V , r e R

P = Pn + r V , r e R

Es d e c ir:
L = { P = ( x ; y ) / P = P + r V , re R
□ ( I I ).

Ejm.23.- Hallar la ecuación vectorial de la recta que pasa por los puntos A(2; 3) > B(5; 7)

Solución
En este caso, podemos to m a r:

V = ÁB y P0 = A

V = B - A = (3;4)

Reemplazando en ( I I ) :

L = {P = ( x ; y ) / P = A + rV , r e R }

Rpta: L = { P = (x ;y ) / P = (2;3) + r(3;4),reR }

Ejm.24.- Hallar la ecuación param étrica de la recta L, del ejemplo 23, anterior.

Solución

Como P = ( 2 ; 3 ) + r ( 3 ; 4 ) , r e R

Entonces : P = ( 2 + 3 r ; 3 + 4 r ) , r e R

( x ¡ y ) = ( 2 + 3 r ; 3 + 4 r ), r e R
944
x = 2 + 3r
y = 3 + 4r, r e R

Es la forma paramétrica de la ecuación de la recta L. r se llama parámetro

ECUACION DE UN SEGMENTO.- Sea AB, un segmento de extremos conocidos A y B


Sea P = ( x ; y ), un punto genérico,
elegido para determinar una ecuación
de AB.
B
Entonces :

AP = r AB , r e [ 0 ; 1 ... ( i ). P (x;y)
Ya /A
- De ( I ): P - A = r ( B - A )
P= A + r(B -A ) ,r e [0;1] o
X
Luego :
AB = { P = ( x ; y ) / P = A + r ( B - A ) , r e [ 0 ; 1 ] } ............( II ).

- La expresión ( I I ) , es una ecuación del segmento AB

- Se observa, que :

* Si r = 0 —> P =A
* Si r = 1 -> P =B
1 A + B
* Si r = - , punto medio de AB

Ejm .25.- Hallar una ecuación del segmento AB, de extremos A( 3 ; 5 ) y B( 7 ; 6 )

Solución
Y.
En ( I I ), con los datos
B
de A y B :
P (x;y)

o
X

Rpta. AB = { P = ( x ; y ) / P = ( 3 ; 5 ) + r ( 4 ; 1 ) t r e [ 0 ; 1 ] }

945
PROBLEMAS RESUELTOS

26) Hallar el valor de n, para que los vectores: á = ( n - 4 ; 2 ) y b = (1;n+1),sean


perpendiculares entre sí.

2
B) 0 C)1 E) 2
A> 3
Solución

Si a L b, entonces: a . b = 0

Luego : (n - 4; 2 ) . (1; n + 1) = 0

Efectuando: n - 4 + 2(n + 1) = 0
2
3n - 2 = 0 n =
3
Rpta: ( A) .

27) En un triángulo ABC, recto en B, se conocen: A = (1; 3) B = (a; a + 1) y C = (a + 2; a + 5).


Hallar el valor de a.

2
A) 3 D) E) 5
B> 5 C> 3
Solución
Se puede usar la fórmula de distancia entre dos puntos para hallar a, pero usaremos la
propiedad de vectores perpendiculares.
Como : AB _L BC, entonces: ÁB . BC = 0

Luego : ( a - 1 ; a - 2 ) . ( 2 ; 4 ) = 0
C (a+2;a+5)
Efectuando : 2(a -1 ) + 4(a - 2) = 0

De donde : a = —
O

Rpta: ( D ). B (a; a+1)

28) Hallar el valor de a, para que los vectores: ü = ( a - 5 ; 4 ) y v = ( 2a ; -1 ) sean paralelos

B) D) E)
A> 9 C> 8 8

Solución

Si u 11 v , entonces para algún t e IR


ü = tv

( a - 5; 4 ) = t ( 2 a ; - 1 )

946
De donde : a - 5 = t ( 2a ) .......... ( 1 ),
4 = -t t = -4

Reemplazando el valor de t, en ( 1 ) :
a - 5 = -4( 2a )
5
a = 9
Rpta: ( A ).

29) Hallar las coordenadas de los vértices del paralelogramo EFGH, en el cual A , B y C, son
los puntos medios de los lados GH , HE y EFf respectivamente, siendo :
A = (2; 3) , B = (13; 8) y C = (18;3)
Solución
- Se tiene, del gráfico :

= B -> H + E = 2B .... (I).

También :
HE = AC E - H = C - A ..(II).
Sumando miembro a miembro, las
relaciones ( I ) y ( I I ):

2E = 2B + C - A
2E 2(13; 8 )+ (18; 3) - (2; 3)
2E (26; 16)+ (18; 3) - (2; 3)
2E (26+ 1 8 -2 ; 16 + 3 - 3 )

{ 42 16
2E = (42; 16) -> E= —» E = (21 ; 8 )
I T ' 2 ;

- De ( I ) : H = 2B - E -+ H = 2( 13 ; 8 ) - ( 21; 8 )
H = (26; 1 6 )-(2 1 ; 8)

Del gráfico; para hallar F : E * ^ = C

F = 2 C - E = 2(18; 3 )-(2 1 ; 8)

F = (36; 6 )-(2 1 ; 8)

Por último, también de la figura :


G + H
= A G = 2A - H

G = 2(2; 3) - (5; 8) = (4; 6) - (5; 8)


De donde :
947
30) Dados los vértices de un A ABC :
A(2; 3 ), B(5; -1) y C(9; 2), hallar la medida del ángulo interno del vértice A

Solución
Del gráfico :
AB . AC = AB x AC x co sa (I).
Siendo :
AB = B - A = (3; -4)

Á 5 = C - A = (7; -1)

AB = AB = ^3*+(-4 f =

AC = AC - =

Reemplazando, en ( I ) :

3 x 7 + ( - 4 )(-1 ) = 2 5 / 2 eos a

1
25 = 2Sj~2 eos a —> eos a =

Rpta

31) Sean ú y v , dos vectores perpendiculares entre sí. Demostrar, que


- |2
( ü + V f = U + V

Solución
Se tie n e :
( ü + v ) = ( G + v ) . ( G + v )

- \2
( G + v ) = G . G + G . V + V . U + V . V

Pero : u . v = v . G = 0, ya que D i v

E ntonces:

G . G + V . V

2 2 .
(ü + v ) 2 = G + V ••••••*»«• 1

* Análogamente, se demuestra que, si Q


G 1 v
v ;; 1

^ 2 t 2
Entonces: ( G - v ) = u + V

948
Las rectas L 1 y L 2 = { P } son perpendiculares entre sí, se intersectan en el punto (4; 5) y
forman con el eje Y un triángulo de área 16u2.
H a lla r: mt - m2 ; siendo m1 y m2, pendientes de L1 y L 2, respectivamente. (m 1>0).

8 15
B)! C )i
D) E) 2

Solución

Sean : L, n L2 = { P }
Y*
Y n L, = { B }

Y n L2 = { A } (o;a) A

Por dato :
área ( A APB ) = 16u2 (o;a-8) B
Luego ;
O X
AB x 4
------ = 1 6
r> -+ AB = 8

Entonces, si A = ( o ; a )

B = (o;a-8 )

Como : PA ± PB , se tendrá : PA . PB = 0

Luego : (-4 ; a - 5 ) . (-4; a - 13) = 0

Efectuando : 16 + (a - 5)(a - 13) = 0

a2 - 1 8 a + 81 = 0

( a - 9 )2 = 0

De d o n d e : a = 9

Entonces, las pendientes :


_ 5 - ( a - 8 ) _ 5-1
m
1 4-0 4

-1
nrii — 1 mo — -» m2 = - 1
mi

Finalmente : m, - m2 = 1 - ( “ 1 ) = 2

Rpta: (E )

949
33) Sea ABCD, un rectángulo, una de cuyas diagonales tiene por extremos los puntos A = (3; 4)
y C = (9; 16). Si los lados de mayor longitud son paralelos al vector (1; 1); determinar las
* coordenadas de los vértices B y D.

A) B = ( 13; 12) B) B = ( 0; 7 ) C) B = ( 7; 0 ) D) B = ( 6; 4 ) E) N.A.


D = ( 0; 7 ) D = ( 12; 13 ) D = ( 12; 13 ) D = ( 0; 2 )

Solución

* Sea : ñ = ( 1; 1)

Como ÁB ñ, entonces B (b ,;b 2 )


para algún t e IR :
V

AB = tñ C (9; 16)

B-A = t ñ
( b, - 3 ; b2 - 4 ) = t ( 1 ; 1 )

De donde : bt = 3 + t
(1 )•
b2 = 4 + 1 D

Por otro lado : ñ _L b S . Luego : ñ . BC = 0

Es d e c ir: (1; 1 ) . (9 - b,; 16 - b2) = 0

Efectuando : 9 - b, + 16 - b2 = 0

b, + b2 = 25 (2).

Reemplazando (1 ) en ( 2 ) : 3 + t + 4 + t = 25 t = 9

Volviendo a la expresión ( 1 ) ; b1 = 12 y b2 = 13

Entonces, para B : B = ( 12 ; 13 )

Enseguida, hallamos D, teniendo en cuenta que :


Á C ^B C D-A =C-B
D = A + C -D
D = (3; 4 )+ (9; 1 6 )-(1 2 ; 13)

D = (0; 7)

Rpta : ( E ) f B = (12; 13)


D = (0; 7)

34) En la figura, ABCD es un rectángulo.


Hallar las coordenadas del vértice C.

A) (9; 11) B) (10; 11) C )(1 0 ;1 0 )

D) (9; 9) E) (9; 10)

950
Solución
* Primero hallaremos B, luego A y finalmente C.

* Para B : FB = tÉ F , donde t e IR y ÉF=(-1-"2;2-1)

=*(1 ; 1 )

Luego : (9 - 1 ; n - 2 ) = t( 1; 1 )

• • ( 10; n - 2 ) = t( 1; 1 ) -> (1 0 ; n - 2 ) = ( t ; t )

De donde: t = 10 y n-2 = t n=12

Entonces : B = ( 9 ; 12 )

* C álculo de A :

Para algún r e IR : AB = rEF

ÁB = r( 1; 1 )
B -A = ( r ; r )
(9; 1 2 ) - A = ( r ; r ) A = ( 9 - r ; 12-r)

Por otro lado, como DA 1 EF DA . EF = 0

Siendo: DA=A-D = (9-r-3;12-r-4) -> DX=(6-r;8-r)

En : DA . EF = 0
( 6 - r ; 8 - r ) . ( 1; 1 ) = 0 -» 6-r+8-r = 0

Obteniéndose : r = 7

Entonces : A = ( 9 - r; 12 - r )
A = (2;5)

Finalmente, para obtener C : BC = Á&


C -B = D -A
C = B+D-A
C = (9;12)+(3;4)-(2;5)

C = ( 10; 11 )
Rpta: (B)

35) El pentágono ABCDE, de la figura, es irregular.


A = ( - 4 I 4 ) I B = ( x ; m ) , C = ( 1 6 ; n ) f D = ( x ; 0 ) I E = ( t ; - 2 ) , P = ( x ; 7 ) , Q = ( 5 ;y )
AQ = 3QD ; 2AP = 3PC
3EQ = QB.
9 r '
x •y
Hallar el valor de m.n.t B
A) 1
1
B)
45
1
C )8
1
D)
16

E) N.A
Solución
Usando las relaciones entre las longitudes, tenemos :

AQ = 3QD -» AQ = 3QD
Q - A = 3( D - Q )
Con los datos :
(9 :y -4 ) = 3(x-5;-y)

De donde : 9 = 3( x - 5 ) —> x = 8
y - 4 = -3y -> y = 1
*
También : 2AP = 3PC 2AP = 3PC
2( P - A ) = 3( C - P )
A s í: 2( x + 4 ; 3 ) = 3 ( 1 6 - x ; n - 7 )

Luego: \ 2(x + 4) = 3 (1 6 - x ) x = 8 confirma lo anterior


2(3) = 3( n - 7 ) n = 9

Además, el último dato : 3EQ = QB 3EÚ = QB


3( Q - E ) = B - Q

3( 5 - 1; y + 2 ) = ( x - 5; m - y )

Es d e c ir: 3( 5 - 1 ) = x - 5 3( 5 - 1 ) = 8 - 5 t = 4
3 (y + 2 ) = m - y —> 3 ( 1 + 2 ) = m-1 m = 1

Finalmente, hemos encontrado:


x = 8 ; y = 1 ; m = 1 0 ; n = 9 ; t = 4

x.y 8 x 1 1
Se pide la expresión :
m.n.t 10x9x4 45

Rpta: ( B ).
36) ABCD, es un paralelogramo. M, punto medio de BC y N, de CD. Conociendo las
coordenadas de A, M y N, hallar las de B.

1 1 1 1
A W A + M -N ) B )-(A + M + N ) C)—( A+4M-2N ) D )-(A + 2 M -N ) E) N.A.
O O ó o
Solución
- Datos : A, M, N.

- Incógnita : B

- Se tie n e :
= 2MN , ya que en el A BCD ,
MN BD y BD = 2MN, por el teore­
ma de los puntos medios.

Entonces : D - B = 2( N - M )
D = B + 2( N - M ) .......... (1 ).

También, sabemos que , para todo paralelogramo :


A+C —B+D —> C = B+ D*A (2 )

- Por ú ltim o :. B + C -= M

- Reemplazando aquí, lo de ( 2 ) : B+B+D-A=2M


2B + D - A = 2M

- Con lo de ( 1 ): 2B + B + 2( N - M ) - A = 2M

Rpta. ( C ).

37.- Hallar la ecuación general de la recta L, de ecuación ve cto ria l:


{ ( x ; y ) = (1 ¡ 4 ) + t ( 2 ; 5 ) , t e I R}

A) 5x + 3y + 3 = 0 B) 5x - 3y + 1 = 0 C) 5x + 2y + .3 = 0
D) 5x - 2y + 3 = 0 E) 5x - 2y + 6 = 0
Solución
Se tiene : ( x ; y ) = (1 ; 4 ) + ( 2 t ; 5 t )
( x ; y ) = (1 + 2t ¡ 4 + 5 t )

- De donde : x 1 +2t
t 6 IR
y 4 + 5t

x - 1
= t
- De estas relaciones, despejamos t : 2
y -4
= t

953
Luego, igualando los primeros miembros :
x - 1 y - 4

- De donde :

Rpta: ( D ).

38) Dados los vectores a y b , demostrar que :

Solución

- Sean : a = ( a , ; a2) y b = ( b , ; b 2)

- Entonces : a - b = ( a1 - b1 ; a2 - b2 )

Luego : ( a - b )X = ( -a2 + b2 ; a, - b 1)

= ( -*2 ; a1) + ( b2 ; - b1)


- vi
( i - b ) = ( -a2 ; a , ) - ( -b2 ; b , )

a-
Es d e c ir: ( á - b ) = á x - b 1 .......... I.q.q.d

39) Dados los vectores á y b , de módu­


los a y b, respectivam ente; demos­
trar que un vector en la dirección de la
bisectriz del ángulo entre á y b , es :

/ ^ — i
á b
V =

Va b
/

Solución

a .v
Llamando v, el módulo de v , se tienen, para á y v : cos a = y , para b y v
av

b.v
cos a =
bv

a .v b .v
Igualando los segundos miembros de ambas expresiones :
av bv

954
á.v b.v
Cancelando v :

á.v b .v n
Luego : --------------— = 0

Es d e c ir: v . = 0
v
- \J-
a b
Entonces, podemos e le g ir: v =

Nota.- Es importante observar que, v , tal como se ha elegido, tiene el mismo sentido que la
bisectriz. A d em ás:
a
— , es el vector unitario en la dirección y sentido de a .
a

— . es el vector unitario en dirección y sentido de b


b 7

.a b
Si llamamos u, al vector : ------ —
a b

a b
Es decir : u ---------
a b

Entonces : v = r.l
ü

Dados los puntos :A = ( 2 ; 3 ) , B = ( 7 ; 1 5 ) y C = ( 5 ; 7 ). Hallar una ecuación vectorial


de la bisectriz del ángulo CAB.

Solución
i
La bisectriz indicada es un rayo con Y (7; 15) B «r'p (x;y)
origen en el punto A y vector direc-
cional v , el cual se puede hallar se­
gún el problema anterior, como : * C (5;7)
\1
AC AB A (2;3)
v = ( i)
\ AC AB y
X
Siendo :

AS ( 5 - 2 ; 7 - 3) (3 ;4 )_
AC 5 V /
V(5-2)2+ (7-3):

( 7 - 2 ; 15 - 3 ) _ ( 5;12) / 5 12
AB 13 ^ 13 ’ 13
V ( 7 - 2 )2 + ( 1 5 - 3 ) :

955
- Reemplazando, en ( I ):

3 . 4 5 12
v = «

5 ’ 5 13 *'13
i
f 3 5 4 12
v = «

v 5 “ Í3* 5 ~Í3 /
i
14 "8 r 8 14 '
V = v =
65 * 65 ^ 65 ' 65 /

Este es un vector direccional de la bisectriz. Pero, como v = — ( 4 ; 7 ), podemos elegir


65
el (4; 7). Así, sea: ; ü es también un vector direccional de la bisectriz.

- Entonces, la ecuación de la bisectriz, del ángulo CAB, tiene forma vectorial; para cualquier
punto P( x ; y ), tal que: A ? = t v , t > 0
ó también : A ? = rü , r > 0

- Por simplicidad, elegimos la segunda. Luego :

P - A = r ü , r> 0
P = A+ rü ,r> 0

Rpta:

41) Hallar la ecuación de la recta L, cuyos puntos equidistan de las rectas


L, = { ( x ; y ) = ( 1 ; 3 ) + t ( 5 ; 1 2 ) , t e R )

Y L2 = { ( x ; y ) = ( 1 ; 3 ) + r ( 4 ; 3 ) , r e R )

Además, L tiene pendiente positiva.

A) 9x + 7y - 12 = 0 B) 9x + 7 y - 15 = 0
C) 9x - 7y + 12 = 0 D) 9x + 7y - 13 = 0

Solución

L, 0 L , : { ( 1 ; 3 ) }

Como los puntos de L, equidistan de


L1 y l_2 , L contiene a la bisectriz del
ángulo entre dichas rectas. Además,
por tener L, pendiente positiva, el
gráfico será el indicado. v 1 tv 2 y v,
son vectores direccionales de Lv L2 y
L , respectivamente. Sabemos que v ,
se puede h a lla r:
956
/ 1
(4 ; 3) (5:12)
V =

V
^ 42 + 32 ^ 5 2 + 122

/ \1

io!

CNJ
4 4
3 ^
V =

»
1
1 5 ’ 5; ^ 13 ’ 13 ;

\ i \
27 ‘ 21 ' 21 27
V =
\ 65 ’ '6 5 ' / v 65 65 /

Entonces, la pendiente m, de L, será :


27
9
m -= 65 m= —
21 7
65
Luego, la ecuación en forma punto-pendiente, de L , ya que pasa por (1 ; 3 ) ,

y " 3 = y ( x -1 )

De donde :

Rpta: ( C )
CAPITULO 3 4

MAXIMOS Y MINIMOS EN GEOMETRIA


( Introducción al Cálculo Diferencial)
Existen problemas, en geom etría, física y en álgebra, que hablan de calcular una magnitud
en términos de otra conocida o de evaluar el valor máximo o mínimo de dicha magnitud; por
ejemplo, calcular el valor máximo del volumen de un cono inscrito en una esfera de radio
conocido V . Estos son, generalemente, problemas que se estudian en Análisis Matemático.
Sin embargo, creemos que el estudiante preuniversitario debe estar en capacidad de
resolverlos. Para ello, el objetivo del presente capítulo es que el lector conozca lo que es la
“derivada de una función” , concepto que necesariamente depende de conocimientos previos
como: función, límite de una función, incremento de una función y gráfico de una función en
un sistema de coordenadas cartesianas, los cuales damos a conocer a continuación:

1. Idea de Función - Hay magnitudes relacionadas, entre sí, de modo que el valor de una viene
determinado por el valor de la otra. En este caso se dice que la primera es función de la
segunda.
E jem plos:
1. El área de un círculo depende de la longitud de su radio. Diremos entonces que el área
del círculo es función del radio. Al escribir la fórmula del área A = k r2
r es la variable independiente.
A es la variable dependiente ( depende de r ).
7i es una constante ( un valor fijo ).

2. El área de un triángulo depende de las longitudes de la base y su altura respectiva. En

la fórmula : A = — . b . h
2
b y h son variables independientes (a menos que se relacionen entre sí por alguna
otra expresión).
A, es la variable dependiente de b y h.
1
— , es una constante.

2. O bservaciones:
* En general, si a cada valor de x le corresponde un valor, y sólo uno, de otra variable y, se
dice que y es una función de x. Esta relación se escribe :
959
y = f(x)
que se lee: “ y igual a f de x ”, lo cual quiere d e c ir: “ y es una función de x

* No es objetivo de este capítulo , desarrollar toda la teoría de funciones ( la cual es muy


amplia ) , sino usar de aquella , sólo lo que nos permite ver las aplicaciones posteriores
con mayor claridad.

* De modo algo más “formal”: Se llama función de un conjunto A en un conjunto B, a toda


relación f de A en B que cumple la condición de que cada elemento de A está relacionado
con un único elemento de B. En este caso se escribirá f : A -» B.

Por ejemplo, sean; A = { 5 ; 6 ; 12 ; 7 } , B = {2 5 ; 36 ; 144 ; 49 } y sea f : A -> B


una función definida como f ( x ) = x2.
Para cada x e A, existe en B un único elemento que es su cuadrado.

Luego: f ( 5 ) = 52 = 25 ; f ( 6 ) = 62 = 36
f (7 ) = 72 = 49 ; f ( 1 2 ) = 122 = 144

Al conjunto de valores que toma x, se llama dominio de f y se escribe:


Dom ( f ) = A . Al conjunto de valores que toma f ( x ) se llama Rango de f : R ( f )= B.

3. R epresentación G ráfica de F unciones


Se puede graficar cualquier función utilizando un sistema de coordenadas cartesianas
rectangulares.

Función Lineal.- Sea la ecuación de primer grado, con dos incógnitas: 3x + y = 5


Esta ecuación puede escribirse a s í :
y = 5 - 3x

Dando valores a x (variable indepen­


diente ), hallamos las valores respec­
tivas de y ( variable dependiente ) ;
siendo y una función de x . A s í, escri­
bimos: y = f ( x ).

Si x = 1entonces y = 2
Si x = 2entonces y = -1
Si x = 0entonces y = 5

En el sistema de ejes marcamos los puntos (1 ; 2 ), (2 ; -1 ), ( 0 ; 5 ),... y al unirlos tendre­


mos la gráfica de la función ( en este caso una recta ).

* En general, la expresión ^y^^m x^^J se llama función lineal, porque su gráfica


corresponde a una recta.

Función C uadrática.- Grafiquemos la función y = x2 . Dando valores a x , tenemos la


siguiente tabla y la respectiva gráfica:

960
I

Una Parábola con vértice en el origen de coordenadas.

X y ( x ;y )

-2 4 (-2 ; 4 )

-1 1 H ; 1 )

0 0 (0 ;0 )

1 1 ( 1 ; 1 )

2 4 (2 ;4 )

3 9 (3 ; 9)

En general, la expresión: y = ax2 + bx + c j es llamada función cuadrática y repre­

senta, en el plano cartesiano, una parábola cuyo eje es una recta paralela al eje Y, cortan
do a dicho eje en el punto y = c.

Nota.- Es fácil reconocer si una gráfica corresponde al de una función, ya que por definición,
para un valor dado de x, debe existir un único valor de y. Es decir, los pares ordenados que
representan puntos de la función deben ser tales que no existan dos con un mismo x.
Así, son gráficas de funciones, por ejemplo:

961
Obsérvese que una función puede ser tal que dos valores de x produzcan un mismo valor
de y.
No son gráficas de funciones, por ejemplo:

Ya que, para un mismo valor de x, existe más de un valor y.

4. Idea de Lím ite de una Función

Sea la función y = f ( x ) = x2. Supongamos que la variable x toma los valores :


{ 2,9 ; 2 ,99 ; 2,999 es decir; x se aproxima al valor 3.

La función y = f ( x ) toma entonces los valores (2 ,9 )2 ; (2 ,9 9 )2; (2 ,9 9 9 )2; ... acercándose


cada vez más al valor ( 3 )2 = 9

Haciendo una tabla:

X f (x )

2,9 8,41
2,99 8,9401
2,999 8,99401
4
é
# é
3 9

962
Diremos: f ( x ) se aproxima a 9 como límite cuando x se aproxima a 3 , y escribiremos:
lím f(x) = 9
Se lee: “el límite de f ( x ), cuando x tiende a 3 , es 9”
x —> 3

Como el conjunto de valores asignados a x eran menores que 3, diremos que x se aproxima
a 3 por la izquierda:

2,9
2,99 2,999

Si ahora, asignamos a x los valores {3,01 ; 3,011 ; 3 ,0 1 1 1 ;...} mayores, pero muy próximos
a 3, los respectivos valores de y = f ( x ) serán mayores pero próximos a 9. En este caso
diremos que x se aproxima a 3 por la derecha:

Si x tiende a 3 por la derecha: x -» 3+

Entonces; para el ejemplo : ------


3 \ \
lim f ( x ) = lim f ( x ) 3,01x
x->3" x->3+ 3,011

A x í í r ^ X ^ ^ x ^ 3+ ^ X ^ se ,,aman limites laterales de f ( x ). Si son iguales:

lim f( x ) = lim f( x ) = lim f( x )


~ . O - ' _ o + V 7 V 7

5. Funciones C ontinuas y Funciones D isco n tin u a s

Sea la gráfica de una función y = f ( x ) :

Se observa que, a valores cercanos de! valor a, del dominio, corresponden valores de la
función cercanos al valor f ( a ). Se dice, entonces que la función es continua en el valor a,
Si esto ocurre para todos los valores del dominio, se dice que la función es continua en todo
su dominio. Luego, una función y = f ( x ) , es continua en x = a , si cumple:
1o) f ( a ) está definida.

963
Ahora, observemos la gráfica de otra función y = g ( x ) :

g(b) t1
1
»
1
}
1
g(a)
i1 i
a .
/ °

Presenta una interrupción en el punto x = a y para valores próximos de a no se obtienen


siempre valores próximos de g(x) próximos a g(a). Se dice que g(x) es discontinua en x = a.

Adem ás: lim g ( x ) * lim g ( x )


x —»a~ x-> a +

yaque: lim g( x ) = g( a ) y l¡mg(x)=g(b)


x -> a x-» a +

6. Incremento de una Función


Fijemos la idea con la función:
y = f ( x ) = x2

Si inicialmente la variable x toma el valor 2, la función y toma el valor y = f ( 2 ) = 22 = 4.

Incrementemos la variable x en 0,1.


Es decir, hagamos que x tome el nuevo valor de 2,1. La variable y, tomará el valor :
y = f ( 2, 1 ) = (2 , 1 )2 = 4,41.

Al valor 0,1 llamaremos incremento de la variable x, siendo el valor 0,41 incremento de la


función ( incremento de la variable y ). Dichos incrementos se denotan así:
Ax ; Ay

( A x, se lee “ delta x “ ; A y , se lee “ delta y ” ).

Entonces, se llama incremento de una función, a la diferencia entre dos valores, uno inicial
y otro final.

Luego, en el ejemplo : A x = 0,1


A y = 0,41

Ahora bien, como : y = f ( x ) = x2 ( 1 ).

Al incrementar la variable x en Ax, se tendrá:


y + Ay = f ( x + Ax ) = ( x + A x )2 ( 2 ).

Efectuando (2) - (1 ):
f ó r m u la d e l in c r e m e n to d e

u n a fu n c ió n .

964
Que en este caso resulta :
A y = ( x + A x )2 - x2
Ay = 2x(Ax) + (Ax)2

En el ejemplo : x = 2, y A x = 0,1 . Reemplazando en la última expresión :

A y = 2( 2 ) ( 0,1 ) + ( 0,1 )2

A y = 0,41, que coincide con el resultado obtenido al principio.

Geométricamente, se ilustra la interpretación en el siguiente gráfico.

Ejem plo.- Hallar el incremento A y , de ia función y = f ( x ) = x2 + 2 , en términos de x


y del incremento A x.
Solución

Se tiene : Ay = f( x + A x ) - f ( x )
Ay = (x + A x ) 2+ 2- (x2 + 2)

Efectuando : Ay = 2x ( A x ) + ( A x )2

Ejem plo.- El área de un cuadrado es 16. Si su lado se incrementa en 0,1, hallare! incremen­
to del área.

Solución
El incremento A S del área , será : A S = ( 4 + 0,1 )2 -1 6 , ya que la longitud original del
lado es 4.

Luego: A S = 2 ( 4 ) ( 0,1 ) + ( 0,1 )2

DS = 0,81

Increm ento R elativo de una Función.-


Se define como el cociente del incremento de la función entre el incremento de la variable,
es d e c ir:
A y , Luego, usando la fórmula de A y :
Ax

A y _ f( x + A x ) - f( x )
AX AX
965
Ejem plo.- Hallar el incremento relativo de la función y = f ( x ) = x2

Solución
*
Se tiene : A i = '< » * A * ) - f ( * )
Ax Ax

Ay ( x + Ax)2 - x 2 2x(Ax) + (Ax)2


Ax Ax Ax

Ay
= 2x + A x, es la repuesta.

LA DERIVADA DE UNA FUNCION.- Dada la función y = f ( x ), se llama la derivada de y


Ay
respecto a x , a l límite de T T , cuando A x tiende a cero; se denota por 21. , y se lee
Ax dx
a s í: “ derivada de y respecto a x "

Entonces : lim Ai
dx Ax-*0 Ax

dy = )jm f(x + A x ) - f ( x )
o, también ;
dx A x —>0 Ax

8. O bservaciones
1o) Para hallar la derivada de una función, se siguen 3 pasos importantes:

a) Encontrar el incremento de la función: A y = f ( x + A x ) - f ( x ) .


Ay
b) Dividir por el incremento A x ; para obtener :

c) Hallar el límite cuando A x tiende a cero. El valor obtenido , según definición , es la

dy
derivada :
\ dx

2o) Otros símbolos usados para denotar la derivada de y respecto a x , son:


dy
f * ( x ) ; fx ; y ; Dx ( y ) , y’ ; siendo los más utilizados ^ y f ’ ( x ).

3o) La derivada de una función es el incremento relativo de la función respecto a su variable


independiente cuando el cambio en la última es tan pequeño como se desee. ( tiende a cero).

4o) Se llama diferenciación al proceso usado para hallar la derivada.

dy
Ejm. 1.- Hallar , para la función y = f ( x ) = x2
Solución
Se procede según la 1a. observación. Veamos

a) Hallamos el incremento A y , de la función:

Ay = f(x + A x ) - f ( x )
A y = ( x + A x )2 • x2
A y = x2 + 2x( A x ) + ( A x )2 - x2
Al simplificar:
Ay = 2 x ( A x ) + ( A x )

b) Se divide A y , por A x

Ay 2x(Ax) + (Ax)
Ax Ax

Factorizando A x en el 2o miembro:
Ay Ax(2x + A x )
AX AX

Simplificando A x , en el numerador y denominador del 2o miembro :


Ay
Ax = ( 2x + A x )

C) Se pasa al límite, cuando A x 0 ( tiende a cero ).


Ay
lim lim (2 x + A x ) = 2x + 0 = 2x
Ax 0 A X AX —» 0

Entonces : = 2x
dx

Así : ( x2) = 2x
dx

Ejm. 2.- Hallar f ’( x ) , si y = f ( x ) = x3.


Solución

a) Hallamos Ay = f(x + A x ) - f ( x )
A y = ( x + A x )3 - x3

Desarrollando : A y = x3 + 3x2 ( A x ) + 3x ( A x )2 + ( A x )3 - x3

Al sim plificar: A y = 3x2 ( A x ) + 3x ( A x )2 + ( A x )3

b) Dividimos por A x : A Z = 3 x 2 ( A x ) + 3 x ( A x )2 + ( A x )3
Ax Ax

Ay
A x = 3x2 + 3x ( A x ) + ( A x )
967
c) Pasamos al límite, cuando A x tiende a cero:
Ay
!¡m = lim f 3 x 2 + 3 x ( A x ) + ( A x )2 )
A x —> 0 AX A X —►0 1 ’

lim = 3x2 + 3 x ( 0 ) + ( 0 ) 2 = 3x2


Ax —>o Ax \ \ /

dy
Es d e c ir: f * ( x ) = ^ = 3x2

9. Observación®*:

q¡ lim f(x) = L y lim g(x) = M


' x-*a x->a

Entonces:

a) lim ( f( x ) ± g ( x ) ) = L ± M
X —►3

b) lim [ f ( x ) g( x ) ] = LM
' x->a J

1 M L_
c) lim
x-»a g(x) M

d) lim [ k f ( x ) ] = kL , siendo k una constante


x — > a

2) La derivada de la función f ( x ) en un punto a, está dada por

f.. . . lim f(a + A x ) - l ( . )


' > A x-»0 AX

E j m . - S i f ( x ) = x2, hallar f * ( 3 ).

Solución
Según se hayó anteriormente: f * ( x ) = 2x, para la función f ( x ) = x2

Luego: f ’ ( 3 ) = 2 ( 3 ) = 6, es la respuesta.

3) El proceso de encontrar una derivada por incrementación y paso al límite, es generalmente,


laborioso. De modo que, se han establecido fórmulas que permitan más rápidamente, hallar
la derivada de una función dada, las cuales damos a conocer. Asimismo, propiedades del
cálculo diferencial.

10. Reglas básicas para ca lcu la r derivadas

I) Derivada de una constante.- Si y = f ( x ) = c ( constante ).


dy _ d , x_ n
entonces:
968
II) Derivada de una constante por una función.
Si y = a f ( x ) , siendo a constante :
dy d / u ,x d f( x )
— = — ( a f ( x ) ) = a — 7— 1
dx dxv ; dx

III) Derivada de xn : — — - = nx"*1 ( n es un número racional cualquiera ).


7 dx

*- Caso particular.- Si a es constante :

= anx 0' 1
dx

IV) Derivada de una suma.-


df(x) dg( x )
dx dx

V) Derivada de un producto.-
d
^(f(x ).g (x ) = f ’ ( x ) . g ( x ) + f ( x ) . g 1( x )

VI) Derivada de un cociente.-

d í f(x) ) f '( x ) . g( x ) - f( x ) . g '( x )


dxU(x)J (g(x))2
g(x) * o
VII) Derivación en cadena.-
Si y es una función de u, la cual a su vez es una función de x :
dy dy du
- - •

dx du dx

dy
Ejm.- Hallar ^ , si y = ( x3 + 1 )10

S o lu c ió n Hacemos u = x3 + 1

Luego ; y = ( x 3 + 1 )10 = u 10

A s í , y es una función de u, siendo u una función de x.

Entonces, aplicando la regia de derivación en cadena :

dy dy du
dx du dx du dx
969
Efectuando; haciendo uso de las reglas ( I I I) y ( IV ) :

dy dx-
= (10 u9 ) +
V dx dx dx

dy
= 1Ou9 ( 3x2 + 0 ) = 30x2 u9
dx

Finalmente, sustituimos u = x3 + 1 :
dy
^ = 30x2 ( x3 + 1 )9

Nota.- En particular, si y = f ( x ) , se tendrá:

_d_ ( y )n = n ( y )rh1 dy
dx dx

VIII) Derivada de Funciones Trigonométricas:

Funci ón Derivada

sen x c os X

cos X - sen x

tg x sec^x

ctg x - csc2x

sec x s e c x.tg x

CSC X - esc x.ctg x

s en ( f ( x ) ) f ’ ( x ) c os ( f ( x ) )

c os ( f ( x ) ) -f ’ ( x ) s en ( f ( x ) )

tg ( f ( x ) ) f ’ ( x ) sec2 ( f ( x ) )

ctg ( f ( x ) ) -f ’ ( X ) CSC2 ( f ( x ) )

sec ( f ( x ) ) f ’ ( x ) . sec ( f ( x ) ) . tg ( f ( x ) )

CSC ( f ( x ) ) -f ’( x ) . esc ( f ( x ) ) . ct g ( f ( x ) )

IX) Derivada de funciones exponenciales y logarítmicas.


( Cuadro en la siguiente hoja ).

970
Función Derivada

ex ex

ef(x) f ‘( x j e 1**)

t
Inx —

f(x)
ln f ( x )
f(x)

a* ax Ina ; a > o

af(X) f ’ ( x ) . af (x >. ln a

1
l°9a x x Ina

f ’( x )
loga f (x) f( x ) Ina

Derivadas de Funciones Trigonométricas inversas:

Función Derivada

1
sen-1x
V l-x 2

-1
COS'1X
V i - X2

1
tg*1x
1 + x2

-1
ctg'1x
1 + X2

f*(x)
sen_1(f(x))
V1 - f2(x)
INTERPRETACION GEOMETRICA DE LA DERIVADA
Supongamos que y = f ( x ) es la
ecuación que determina la función.
y = f(x)-^
Sabemos que esta ecuación repre­
senta una curva en elpiano de coorde­
nadas XY. Sea P( x ; y ) un punto sobre Q (x + Ax;y + Ay)
la curva representativa de la función.

Si incrementamos x en Ax, quedará


incrementado y en A y y se tendrá un
segundo punto Q ( x + A x ; y + A y )
como se muestra en la figura :

Tracemos la cuerda PQ. Por P una


horizontal y por Q una vertical. Estas
dos rectas se cortan en R.

Ay
De la figura, podemos extraer el siguiente resultado : tan 8 =

Pero tan 0 es la pendiente de la cuerda PQ. O sea que la pendiente de la cuerda PQ es el

incremento relativo de y respecto a x : mPQ =

Si hacemos A x muy pequeño, permaneciendo P fijo, el punto Q se mueve sobre la curva


de tal manera que se aproxima al punto P y la cuerda PQ tiende a confundirse con la
tangente a la curva en P.
Entonces:
Ay dy A A , ,
rripT = hm —- = — = tan<J> (* ).
A x —> 0 Ax dx

Interpretación G eom étrica


Geométricamente, la derivada de ia función determinada por y = f ( x ) respecto a la va­
riable independiente x, representa la pendiente de la recta tangente a la curva de la función
en el punto ( x ; y ).

Si se conoce un punto sobre una curva, puede encontrarse, entonces, la ecuación de la


tangente a la curva, ya que se sabe que la ecuación de la recta que pasa por ( x1 ; y , ) y tiene
pendiente m, es:
y - y, = m ( x - x , )

y la pendiente m es la derivada de la función evaluada en el punto de tangencia ( x , ; y1).

Ejem plo:

Encontremos la ecuación de la recta tangente a la curva representativa de la función


determinada por la ecuación y = x2 en el punto P( 2 ; 4 ).

972
Solución
Sabemos que ta derivada de y = x2,
respecto a x, en el punto P( x ; y ) es:

^ = 2x = f ’ ( x ).
dx

En el punto P( 2 ; 4 ) la x toma el valor


2 por ser la abcisa del punto , luego
f ’ ( 2 ) = 2 ( 2 ) = 4.

Por la ecuación ( * ) este valor corres­


ponde a la pendiente de la tangente
en P( 2 ; 4 ).

Aplicando la ecuación : y - y1 = m ( x - x , )

se tiene: y - 4 = 4 ( x - 2 )

O sea : 4x - y - 4 = 0 que es la ecuación que se pide

Nota.- Si la función es lineal, esto es, si su representación gráfica es una línea recta, ésta
coincide con la tangente en cualquier punto; luego, la pendiente de la recta es la derivada
de y , respecto a x.

E jercicios
1. Hallar la expresión de la derivada para cada una de las funciones de x :
1
a) y = 2X3 + 5x2 + 10 b) y = —
' 1 x
x2 + 1
c) y = d) y = sen 2x
~x - 1

e) y s eos x3 0 y - 1 + eos 3 x
Soluciones
a) y = 2x3 + 5x2 + 10
d f 0 3 d / c 2
É l = J L ( 2x3 + 5x2 + 10 ) 2x + — I 5x + (10)
dx dx dx dx dx

dy
= 2 .3 x 3 " 1 + 5 .2 x 2 ~ 1 + 0
dx

dy
Rpta = 6 x 2 + 10x
dx

dy d ( -1 -1-1
b) x = —1x
X dx dx

= — Y
-2 = ___
“ 1
Rpta 2
dx V x ; X¿
973
c) y = * 2 + ^ ; hallamos usando la regla V I ; derivada de un cociente
x - 1 dx

Sea y = - M - , para f ( x ) = x2 + 1 ; g ( x ) = x - 1
g(x )
Luego : y ’ = f '.( x ) - 9( x ) - f ( x ) • 9 ' ( x )
( g ( x ))2

Siendo : f ’ ( x ) = 2 x ; g * ( x ) = 1

dv 2 x ( x - 1) - ( x2 + 1) . 1
Entonces : y ’ = —L = ------------------------------- í—
dx ( x - 1)

Efectuando : = 2x2 - 2 x - x 2 - 1
dx (x-1)2

Es decir : y’ = x ~ 2x Rpta
(x-1)2

d) y = sen 2x

Según la regla V III:


d
( sen f ( x ) ) = f 1( x ) eos f ( x )

En el ejercicio propuesto: f ( x ) = 2x

luego : d y _ _d_ ( sen 2x ) = 2cos 2x. Rpta.


dx dx

e) y = eos x3

con la regla VIII

— = — ( cosx3 ) = — ( x3 ) senx3
dx dx' ' dx'1 ’

dy
^ = -3X2 sen x3 Rpta.

f) y = ^ 1 + eos 3 x 2 = (1 + eos 3 x 2 ) 2

Usando la regla V I I , derivación en cadena:

— = —(1 + eos 3 x 2 )2 . — (1 + eos 3 x 2 )


dx ^ 1 dx' }

974
1
dy 1
-6xse n 3x2 (1 + cos 3 x 2 ) 2 =
dx 2 ^ 1 + eos 3 x 2 dx

- 3 x sen 3 x 2
-R p ta
^ 1 + cos 3 x 2

2. Si f ( x ) = x3 + 2x2, hallar f ’( 2 ).

Solución
Se tiene: f ’( x ) = 3x2 + 2.2x
f ’( x ) = 3x2 + 4x

Luego: f ’ ( 2 ) = 3 ( 2 )2 + 4 ( 2 ) = 12 + 8

f *( 2 ) = 20 Rpta.

3. Hallar la pendiente de la recta tangente a la curva


y = x2 + 2 , en un punto de abscisa 1.

Solución
La curva y = x2 + 2 , tiene por gráfica:

La pendiente de la recta tangente a la


curva, en el punto P ( 1 ; y ) es igual
al valor que toma la derivada de la
función en dicho punto, es d e c ir:
f ’ (t).

Luego:
y = f ( x ) = x2 + 2 ->

La pendiente de la recta L, es :
mL = f ’(1 ) = 2( 1 )
mL = 2 Rpta.

4. Dada la función :
y = f ( x ) = ax2 + bx + c. I _™2
f% | l ar \ #
V

Hallar “ d ” , si T es punto de tan­ y


gencia y el punto P tiene coordena­
das ( 2 ; 5 ). 9

Solución

* ( 0 ; 9) e f ( x ) -» f ( 0 ) = 9 c=9 , j
45 ^ 1 0 B
* P= (2;5)ef(x) L d-
4a + 2b + 9 = 5 -+ 2a + b = -2

975
* La pendiente de la recta: f ’ (4) = tg 45°

Siendo: f *( x ) = ( ax2 + bx + 9 )
dx
f ’ ( x ) = 2ax + b

f ’ ( 4 ) = 8a + b 8a + b = 1 ( 2 )

De las ecuaciones (1 ) y ( 2 ) , se obtienen :

-3

Luego: f ( x ) = ax2 + bx + c
x2
f ( x ) = — - 3x + 9

Entonces, para T ; T = ( 4 ; y1)

Yt = —— - 3( 4 ) + 9 y, = 5

Finalmente, en el A ABT , isósceles :


AB = TB
d + 4 = y,

d = 1 Rpta

11. Funciones C recientes y D ecrecientes


* Consideremos la función: y = f ( x ) ,
cuya gráfica se muestra.

Se observa que, para el intervalo < x t;


x2>, conforme el valor de x crece de x 1
a x2 , el valor de y decrece desde y t
hasta y2, para todo x e < x, ; x2 >.
Luego, diremos que y = f ( x ) es una
función decreciente en este intervalo.

De otro lado, en el intervalo < ^ ; x3 > a medida que x crece desde hasta x3, el valor de
la función crece desde y2 hasta y3. Luego , se dirá que la función es creciente para todo
x g < x2 ; x3 >.

* Este hecho se puede expresar en el siguiente teorema: Si f ( x ) es una función derivable en


un intervalo < a; b >, entonces :

i) Si f ’ ( x ) > 0 f es creciente
V x g <a ;b>

ii) Si f * ( x ) < 0 f es decreciente


V x g <a;b>
976
Como puede verse , en el gráfico a n te rio r, para x e < x1; x2> , las tangentes a la curva
y = f ( x ) tienen pendientes negativas y para x e < X2 ; x3> las rectas tangentes a la curva
de la función tienen pendientes positivas:

pendiente < 0
f (x) < 0

•2 X

12. Valores M áxim o y M ínim o de una fu n ció n , en un in te rva lo dado

Sea la función y = g ( x ), cuya gráfica


se muestra a continuación.

Se observa que, para todo xe < a; b >:

* g ( a ) es el mínimo valor de la función.

* g ( b ) es el máximo valor de la función

Nota.- La teoría que conduce a la determinación de máximos y mínimos de una función da­
da, tiene un tratamiento extenso en el curso “Análisis Matemático T, el cual no es objeto de
nuestro tema. La aplicación a la Geometría Elemental se puede entender fácilmente con
algunos sencillos ejemplos.

Ejem plo 1.- El área de una cuadrado, en m2, se expresa por la función :
A = f ( x ) = x2 - 2x + 5, donde x es un número en el intervalo [ 0 ; 6 ].
( Es d e c ir: 0 < x < 6 )
H a lla r:

a) La gráfica de la función.

b) El valor de la función para x = 0

c) El mínimo.valor de la función.

d) El máximo valor de la función.

Solución
Se tie n e :

A = f ( x ) = x2 - 2x + 5 ;
0 < x <6

a) Gráfica:

977
V

b) Para x = 0 : A = f ( 0 ) = 02 - 2 ( 0 ) + 5 = 5

c) El mínimo valor de la función, se obtiene, según la gráfica cuando x = x , ; es decir, en el


vértice de la parábola. En este punto, la recta tangente a la curva es paralela al eje x, por lo
cual su pendiente será 0 ( cero ).
Luego :
f * ( X l) = 0
df(x)
Hallando f *( x ) ( x2 - 2x + 5 )
dx dx

df(x)
= 2x - 2 f ’ ( x ) = 2x - 2
dx

Entonces: f ’ ( x ) = 0 2x - 2 = 0

x = 1 xi = 1
Este es el valor x = x1, que dá el mínimo valor de la función: A = f ( x )

'mínimo
= f ’ ( x ,) = f = 12 - 2 ( 1 ) + 5
Amínimo - 4

d) El valor máximo de la función, se obtiene, según el gráfico, cuando x = 6 :

A má»mo = 62 - 2 ( 6 ) + 5 = 29.
* O bservaciones:

1°) Según las condiciones del problema, el valor mínimo de x, es cero. Este conduce al valor 5
del área, el cual no es el mínimo de la función.

2o) Como la gráfica corresponde a una porción de parábola “abierta hacia arriba”, el valor mínimo
de la función se encuentra en el vértice, hallándose la abscisa de este punto al igualar a cero
la derivada, ya que aquí la recta tangente es paralela al eje x y tiene pendiente cero.

Ejem plo 2.- El área de un triángulo se puede expresar por la función :


y = f ( x ) = 9 - ( x - 3 )2
H a lla r:

a) La gráfica de f ( x ).
b) El valor mínimo de f ( x ) y los valores de x que conducen a dicho mínimo
c) El valor máximo de f ( x ) y el valor de x que conduce a dicho máximo.

Solución
a) G rá fica :
Es una porción de parábola “abierta
hacia abajo”. ( Cóncava hacia a b a jo ).

b) Del gráfico, para x = 0 ó x = 6, se


obtiene el valor mínimo de f ( x );
hallados al hacer f ( x ) = 0.
978
Es sencillo observar, con el gráfico que el máximo de f ( x ) es 9 y ocurre cuando x = 3.
Sin embargo , este valor puede hallarse al hacer f ’ ( x ) = 0 , pendiente de la recta tangen­
te a la curva, en el punto T.

Así: f ’( x ) = A . [ 9 - ( x - 3 )2 ]
dx

f ’ ( x ) =_d_ ( 9 ) - _ d _ ( x - 3 ) 2
dx dx

f ’ ( x ) = -2( x - 3 )

Luego, haciendo: f J( x ) = 0 ; 2( x - 3 ) = 0
x = 3, tal como se comentó

Ejem plo 3.- La expresión : V = f ( x ) = , indica el volumen de un sólido.


x- 2

Hallar el valor mínimo del volumen, diferente de cero.

Solución
Como f ( x ) representa un volumen :
f(x) >0 V = f (x) i

Luego: > 0 x >2 (I). pend. = 0


x - 2
V : f ’ (x,)=o
Dando valores a x, se pueden hallar min.
los respectivos valores a f ( x ) y
graficar, aproximadamente ( bosque­
jo de la c u rv a ):

Del gráfico, Vmínimo ocurre para x = x , , el cual se obtiene para f ’ ( x ) = 0

df( x ) _ d
Así:
f ’(x >= dx "dx x- 2

2 x ( x - 2 ) - x2 (1)
f'(x) =
(x-2)2

x - 4x
Es d e c ir: f ’ (x) =
(x-2)2

Haciendo f ’ ( x ) = 0 , para hallar el valor x = x1 que dá el mínimo V

Y - 4x
= 0 -> x2 - 4x = 0
(x-2 )2

Al resolver, se obtienen : x = 0 ;
x = 4
979
Pero, por ( I ) : x > 2

Luego : x = 4 x, = 4

Reemplazando en la expresión de V:
42
Vmfnimo
mínimo = f < X1) = f ( 4 > =

Ejem plo 4.- Los catetos de un triángulo rectángulo tienen longitudes ( 7 + x ) y ( 5 - x )(


respectivamente.
Hallar el valor máximo del área, así como el valor de “x” que lo determina.

Solución
Dadas las longitudes de los catetos, el
área A, del triángulo rectángulo se
eval úa: (7+x)

A = ¿ (7 + x ) ( 5 - x )
(5-x)
A = 17,5 • x - — .......... ( I ).
2

Que es la expresión de la ecuación de una


* A = f (x) parábola, “abierta hacia abajo”.
m=0
x 1 da e! máximo valor del área; x1 se halla
haciendo f ’ ( x ) = 0 , ya que la pendiente
de la recta tangente a la curva en el punto
de abscisas x, es nula, por ser una recta
paralela al eje x.

2
Así, tenemos : A = f ( x ) = 17,5- x - *
2

x
Luego: f ’ ( x ) = 0 -1 - 2i—\

f ’ ( x ) = -1 - x

Entonces, al hacer f ’ ( x ) = 0 : - 1 - x = 0

x = -1 -> x 1 = -1

Y, el valor máximo del área, se obtiene reemplazando en ( I ), el valor x = -1 :

A máximo

Rpta: Amáximo 18

980
* Nota.- Obsérvese que x puede tomar valores positivos o negativos: x e < -7 ; 5 >

O bservaciones.- Para hallar el máximo o el mínimo en una aplicación geométrica ( física o


algebraica):

1o) Dibujar el problema planteado.

2o) Deducir la ecuación para la cantidad que se ha de maximizar ( o m in im iza r), como una
función de una sola variable, usando relaciones geométricas, trigonométricas, etc.

3o) Plantear las condiciones de problema ; por ejemplo , si ( x - 2 ) representa la longitud del
lado de un polígono : x - 2 > 0

4o) Hacer un bosquejo de la función.

Por ejemplo:

5o) Los valores de x tales que y = f ( x ) sea un máximo o un mínimo, se obtienen haciendo
f ’ (x) = 0

6o)Si x tal que f ’ ( x ) = 0, origina un máximo, un mínimo, o ninguno:

a) Si f ” ( x0) > 0: f ( x0 ) es un mínimo.

b) Si f " ( Xq ) < 0: f ( x0) es un máximo.


c) Si f ’ ( x ) < 0 para x < x0
f ( x 0 ) es un minimo
y f ' ( x ) > para x > Xq

( intervalo a ; b del gráfico a n te rio r)

d) Si f ’ ( x ) > 0 para x < x0


f( x0 ) es un máximo
f ' ( x ) < 0 para x > Xq

( intervalo < b ; c > en el gráfico a n te rio r)

7o) La función cuadrática : f ( x ) = ax2 + bx + c, ( a * 0 ) , tiene un máximo o un mínimo en


-b
x =
2a

Ya que : f ’ ( x ) = 2ax + b

df'(x)
f " ( x ) = 2a I f " (*)
dx

Luego , x = — será un mínimo si a > 0 y un máximo si a < 0.


2a
QQ1
\

PROBLEMAS RESUELTOS

1) Hallar las dimensiones del rectángulo de perímetro 36 cm. cuya área es máxima

Solución
Según dato :
2x + 2y = 36
x + y = 18
y = 18 - x ......... ( I ) .

El área: S = xy

C o n ( I ) : S = x ( 18 - x ) S = 18x - x2
dS
El valor de V que dá el máximo de “S” se encuentra al hacer = 0
dx

Es decir: _±! _( 18x- x2) = 0


dx

18-2x = 0 x = 9

Reemplazando en ( I ) : y = 9

Luego: “ entre todos los rectángulos que tienen igual perímetro, el cuadrado es de máxima
área
dS
Nota.- Como S’ = = 18 - 2x, se
dx
verifica que lo hallado es un máximo
ya que :

S” = d ( 18- 2x)
dx

S” = -2 < 0 , cumple con la


observación 6 : b

2) Dos lados de un triángulo tienen longitudes 5 y 8 unidades, respectivamente. Hallar el va­


lor máximo del área.

Solución
En este caso el ángulo a es variable.

El área se evalúa:

A = x 5 x 8 x sen a = 20 sen a

Hallamos el valor de a que dá el máximo


dA
valor de A, haciendo d a = 0
QQO
(20 sena) = 0
dx
Es d e c ir: 20 eos a = 0 a = 90
Entonces, el valor máximo del área será : A = 20 sen 90 ^máximo “ ^

Nota.- Hemos deducido que, dadas las longitudes de dos lados de un triángulo, aquel que
tiene área máxima es un triángulo rectángulo que tiene por catetos las longitudes conocidas.

Un jardinero posee una cuerda de 100 metros y quiere cercar con ella, en forma de sector
circular, un jardín, de modo que el área sea máxima. Hallar dicha área.
Solución
Si V es el radio del sector, entonces
la longitud del arco AB, es (100 - 2 x ).
El á r e a :
A
1
S = — ( longitud de A B ) .(radio)

S= ¿ ( 100 - 2x ) ( x )

S = 50x - x2 (I).
dS
Para hallar el máximo, hacemos : = 0
dx
Luego : 50 - 2x = 0
x = 25 ; este es el valor de wx" que dá el máximo de “S”. Reemplazando en ( I ) :
Smáximo = 50 x 25 - 252
Rpta ^ m á x im o = ® ^5 m 2

Hallar el valor máximo del área del rectángulo inscrito en un semicírculo de radio V , si uno
de sus lados está sobre el diámetro.
Solución
Considerando el gráfico adjunto :
y = r sen a
x = 2r eos a
El área del rectángulo :
A = x y = 2 r2 sen a eos a
A = r2 sen 2 a . Se observa que A es
función de a, donde 0o < a < 90°.

Para hallar el valor de a que dá el máximo valor de A : dA = 0


da

J L ( i2 sen 2 a ) = 0
da

9P
r2 . 2 cos 2a = 0 -» cos 2 a = 0 2 a = 90°
a = 45°
Entonces: Amáxim0 = r2 sen 2 x 45° = r2 sen 90

Rpta: A m áxim o = r2

5) En un rombo ABCD, cuyo lado mide 3 <J~3 m , se traza por el vértice A, una recta paralela
a la diagonal BD ; luego se hace girar el rombo en torno a esta recta, una vuelta completa,
con lo que se generará un sólido cuyo volumen máximo debe calcularse, así como la longitud
de BD, para este volumen.
Solución
Sea, según el gráfico :

AO = x ; BO = y ; con el teorema de Pappus-Guldim,


el volumen del sólido generado, es:

V = 2 n x . ( área ABCD )
w ~ f 2 x . 2y
V = 2nx. ' V = 4 7i x2 y ( 1) .

En el A AOB : x2 = ( 3 / 3 )2 - y 2

Reemplazando en (1 ) :
T
V = 4 k ( 27 - y2) y (2).
dv
Como V = f ( y ), hallamos e igualamos a cero para obtener el valor de “y" due dá el
dy
máximo:
dV
= J L [ 108 n y - 4 tc y3 ] = 108 re- 12 7t y:
dy dy dy

dV
= 0 1 0 8 n - 12 ti y2 = 0 y = 3
dy

Luego, BD = 6m y el volumen máximo :


V = 216 7cm3

6) Un árbol de 9 metros de altura se encuen­


tra en la cima de una colina de 17 metros
de alto. Si el ojo de un observador se
encuentra a 1m. del suelo, ¿a qué distan­
cia debe encontrarse de un punto directa­
mente bajo el árbol, para que sea máximo
el ángulo formado por las visuales a la
base y a la copa del árbol?

Solución
Sea el gráfico adjunto :

984
Incógnita: x, cuando a es máximo
Se observa : a = ó- p
t g a = tg ( <t>- P )
_ tg<> - tgP
tg a - 1 + tg<)). tgP

AD 25
Del gráfico: tg ó =
DE x
BD 16
tg P =
DE x

25 _ 16
Luego: tg a = x x
25 16
1+
x ' x
9x
tg a = ~ 77T
* x¿ + 400

Como tg a = f ( x ), ( función de “x” ), a será máximo cuando tg a lo sea.


9x
Luego, derivamos f ( x ) = x2 + 400 e '9 ua*amos a cer0 Para hallar el valor de “x” que
dá el máximo:
M x l = 0
dx

9(x2 + 4 0 0 ) - 9 x ( 2 x ) =0
<x2 + 4 0 0 )
De donde: 9 ( x2 + 400 ) - 18 x2 = 0

Rpta: x = 20 metros

7) Determinar las dimensiones del cono circular recto circunscrito a una esfera de radio 2m,
de tal modo que el volumen de dicho cono sea mínimo.

Solución B
Sean V e “y”. e l radio de la base y
la altura del cono, respectivamente.
El volumen V , del cono , es :
7C
V = — x2 y (1 )
3

En triángulo AMB:

AB = V x2 + y 2

985
Se trata de despejar una de las variables ( x ó y ) , en función de la otra , para reempla
zar en (1 ).

AOTB-AAM B: — = — - = y_2
AM AB x

4y
De donde: x2 = (2).

Reemplazando esto en (1 ) :

4ny 4n
V = V =
3( y “ 4 ) 3 (y - 4)
Derivando respecto a Y ’ :

dV 4n d
dy 3 dy (y-4)

d V ^ 4 ti V -8y
dy ~ 3 (y-4 )2

Igualando a cero para hallar el valor de “y” que dá el mínimo “V1

4 71 f -Q y
= Q
(y-4)2
De donde : y2 -¿ y = 0
0, dá un mínimo.
y=
8, dá el mínimo buscado

y = 8

Para hallar “x”, de ( 2 ), expresión a n te rio r:


4x8
= 8
y -4 8 -4

\ x = 2 /2

Luego, la altura del cono es 8m. y el radio de la base 2^/~2 m.

8) Con un alambre de longitud 10Om. se forma un cuadrado y una circunferencia. ¿Cómo debe
cortarse el alambre para que la suma de las áreas encerradas sea:

a) Mínima.

b) Máxima.

22
N ota.-Tom ar 7t = —

986
Solución
Se tiene:

alambre

Debe cumplirse: 2 n x + 4y = 1 0 0
22 44 x
con k = + 4y = 100
7 * 7

De donde: 11 x + 7y = 175
11 x
Luego, despejando “y” : y = 25 - (I)

La suma de áreas encerradas


A = tz x2 + y 2
C o n ( I ):

A 2 2 2
11x
A = — x¿ + 25- —
\ 7 ,

A =^ x 2- ^ x + 6 2 5 ...(||).

Cuya gráfica se muestra:

Se puede responder a las preguntas


planteadas, observando el gráfico de
la función A = f ( x ).

a) x , , dá el mínimo valor A, de la función A = f ( x ), el cual se obtiene haciendo t ' ( x ) = 0


dA d ' 2 Z 5 x* - 5 5 ° x + 625
Veamos :
dx dx , 49 7 /
dA 550 550
dx ~ 49 X 7
550 550
Igualando a cero : "49” x — ~ ~0

de donde: x = 7 ( este es el valor x1)

550 550
Y, se verifica analíticamente que es el mínimo ya que f ’ ( x ) = x ---------
49 7
550
Siendo la segunda derivada :
f"(x) = 7 9 "> 0

987
Así, el valor mínimo de A , se obtiene al reemplazar x = 7, en ( I I ):
= 275 ( 7 ) 2 . 550 (7 ) + 6 2 5
mínimo
49
mínimo = 350 m2
- El alambre en este caso , debe cortarse para obtener una circunferencia de longitud
22
2n x = 2 ( — ) (7) = 44 metros, siendo el resto de 100 - 44 = 56 m, para formar el cua-

56
drado de lado y = = 14 metros.

b) De la curva A = f ( x ), se observa que x3 dá el máximo valor de A, el cual ocurre cuando x


es máximo. Es decir, cuando “y ” sea mínimo. Hecho que ocurre al dedicar todo el alambre
a formar una circunferencia.
Entonces : y = 0. De modo que, al reemplazar en ( I ), se obtiene :
11 x 175
0 = 25- —> x = ----- valor de x3.
7 11
Al sustituir en ( I I ) , hallamos elmáximo valor de A = f ( x ):
2
275
11175J] 550 175 'j
+ 625
'máximo
49 7 { 11 J
8750 2 7nc AC 2
'máximo m = 795,45 m 2
11
Nota.- En la curva A = f ( x ), el punto A 2 se obtiene para x = 0, dando el valor A = 625 m2.,
en la expresión ( I I ). Esto ocurre cuando todo el almabre se dedica a formar un cuadrado de

lado = 25 m.
4
En el intervalo [ 0 ; X3 ], el mínimo es A1 y el máximo A3.

9) La figura muestra una porción de car­


tulina, en forma de sector circular de
radio MR", con la cual se quiere obte­
ner la superficie lateral de un cono de
revolución, de volumen máximo.

Hallar la longitud del radio de la base


del cono.
Solución
Llamando Mx”, el radio de la base del cono;

la generatriz R y la altura : ^ R2 - x
El volumen V, se evalúa:

V = - x 2 J R2 - x2
3 y
K
V = - x2( R 2 - x 2)1'2
988
t
dV
Hallamos, ahora V ’ =
dx

• ♦ V ’ = | - ( 2x )( R2 - x2)1/2 + - x2 . -2x )( R2 - X2 )1/2'1


3 3 2
2 K
V ’ = - 7 t x ( R 2 - x 2)1/2 - - x3 ( R2 - x2 )-1/2
O o

Igualando a cero, para hallar el valor de x que dá el máximo V :


2 7t
V' = 0 -* — 71 x ( R2 - x2 )1/2 - — x3 ( R2 - x2 ) 1/2 = o
O o

_ R^~2
De donde : x =

10) Hallar la longitud máxima de una varilla de acero que se puede trasladar de un corredor a
otro, de anchos 27 y 64 pies, respectivamente, suponiendo que la varilla debe permanecer
siempre paralela al piso.

Solución
La situación es como indica la figura: T
64
donde la varilla AC podrá deslizarse a
través de A y C.

AC = AB + BC

AC = 27 cosec a + 64 sec a (1 ).

Se observa que AC = f ( a )

Luego : —~ a ) = -27 cosec a . ctg a + 64 sec a . tg a


da

igualando a cero, para hallar el valor de a que cumple la condición planteada:

-27 cosec a . ctg a + 64 sec a . tg a = 0

De donde : tg a = -

Finalmente, reemplazando en (1 ):

Rpta AC = 27 x - + 64 x - AC = 125 pies

11) En un sector circular de área conocida “A”, hallar la medida del ángulo central, para que el
perímetro sea mínimo.

989
\

Solución
Sea x la longitud del radio y a ( en
radianes), la medida del ángulo cen­
tral.
La longitud del arco será : a x.

2A
El dato: = A x =
a

Para el perímetro :
P = 2x + a x = ( 2 + a ) x
2A
P = ( 2 + cc)
a
1 2
2 + 4 2A . a 2
ó mejor : P = 2 ^ / 2 A . a 2 (I)
Como P es una función de a, para hallar el valor mínimo de P, hacemos
dP
= 0
da
Así:

p' = — = 2 S 2 K .

da

2
dP
P' = ^ - = - / 2 A a-2 + V2A a 2 (II)
da

Haciendo P’ = 0, para hallar el valor de a que dá el mínimo valor de P


2
r— -f J2A 2
~4 2 A a 2 + a = 0

De donde : a = 2 rad.

Nota.- Verificamos que a dá un mínimo de P, comprobando que P" > 0, para a = 2. En

efecto, derivando respecto a a, la ecuación ( I I ) para hallar P” : P” = P’ ¡


da

2
2
-/2 A a 2 + .a
da

V~2A
P M= - 4 2 A • - - a- i - +
J V
3
V 2A 2
P " = —J~2A . a 2 .a
2
990
o -2. IpA 3
Para a = 2 : P " = - / 2 A .2 2 - ^ ------ . 2 ' 2
2 4

1 1
Esto e s : P"=3
2 ' ^2®
Efectivamente :

p„ 3/ Á /A / Á rQ
8 8 4
a = 2, dá el mínimo valor de P.

El gráfico

P vs. a, es

El perímetro de un triángulo isósceles es “ L Hallar el valor máximo del área

Solución

Sea el triángulo ABC, con AB = BC


B
Sean: AB = y ; AC = x (x e y : variables)

Dato: 2y + x = L (I).

Incógnita : Valor máximo del área ABC.

Llamemos “S” al área del triángulo ABC

S = - . AC . BH
2

Siendo, AH = HC = — ; en el triángulo AHB : BH - V AB" - AH" = .( y2 - ii-

L uego:
s =f X-V y2~ 7

Debemos expresar “S" en función de una sola variable. Para ello, de ( I ), despejamos “x”
x = L - 2yt En la expresión a n te rio r:

S = l ( L - 2 y ) J y 2 - ( L ~ 2y)

Simplificando:

s =| | - y Ly- (II)
/
Para hallar el valor de “ y ” que dá el máximo “ S ”, igualamos a cero , la derivada d e HS ”
respecto a “ y ” :

991
dS
= 0
dy

d /
Ly- = 0
dy V2 ^

Efectuando el desarrollo de la derivada del producto de las funciones^ — - y Ly-

con la regla ( V ) vista anteriormente; así como la regla ( V I I );


/
d ( L
Ly- - - y Ly- = 0
4 dy k 2 +l 2 _ y dy
V )
1
1 L2
• • (L) = 0
) 2 ,\ L y - T /
1
/
Ly- = o
v 4 2 V j

Es decir :
ILz - JK ¡ ^ a

Ly-

L
Luego: Ly-
2 V
L
De donde : y = —. De modo que, al reemplazar en ( I ) : x =
3

Es decir: AB = BC = AC = — , resultando ABC , un triángulo equilátero. El valor del


3
área e s :

2 {3 L2 / 3
\ 4 36

i\
13) En la figura, se conocen las longitu­ \ V
XV
des “a", "b” y “L”. \\
3
a A
Hallar el valor de x , de modo que la \\ b
sum a: T \ / r
S = AP + PC, sea mínima. B x P D
\--------- L ---------1
992
Solución
Haciendo uso del teorema de Pitágoras en los triángulos ABP y PDC :

AP = ^ a2 + x2 y PC = ^ ( L - x f + b2

Luego : S = •/ a2 + x2 + ^ ( L - x )2 + b2

dS
Para hallar Sm(nimo, hacemos : = 0
dx

Así : / a 2 + x^ + ^ ( L - x )2 + b2
dx

a*2 + x¿
2 + d
dx y dx V(L- x)2 + b2 = 0
1
ó — ( a2 + x-a )2
k + 2 + b2 = 0
dx dx

Efectuando : a2 + x2)1/2'1 . ( 2x ) + — ( ( L - x )2 + b2 )1/2'1 . 2( L - x ) . (-1 ) 0

(L -x )
= 0
Simplificando : a¿2 + x¿
2
^ ( L - x )2 + b2

De donde: ( b2 - a2) x2 + 2a2 Lx - a2 L2 = 0

- 2 a 2 L ± J ( 2 a 2 L )2 - 4 ( - a 2 L2 )( b2 - a 2 )
Resolviendo para x : x=
2(b2 - a 2 )

- 2 a 2 L ± 2ab L
X =
2 ( b 2 - a2 )

aL
; si b * a
b +a
Dando : x= ó
aL
a - b

3 I aL
Pero, co m o > 1, entonces > L, por lo cual no se acepta como solución
a - b a - b a - b’
ya que x debe ser menor que L.
A s í , la respuesta será :

993
14) De una hoja de cartón cuadrada, de lado L, hay que hacer una caja reectangutar abierta, de
mayor capacidad posible, recortando para eHo cuadrados de lado x, en los ángulos de la hoja
y doblando después los salientes de la figura, en forma de cruz. Hallar el valor de x.
L L
A> 2 B> 3 C> 4 D> ? E> 6
Solución
- Se tienen los esquemas:

- El volumen V de la caja:
V = ( L - 2x )2 . x
V = L2x - 4Lx2 + 4x3 /i A
y
y
A
A
A
A
A
L ~2x
Para el máximo:
dV
= 0
I I— L -2x H
dx

( L2x - 4Lx2 + 4X3 ) = 0 L2 - 8Lx + 12x2 = 0


dx

De donde :

- La solución, será

Rpta: ( E ).

15) En una semiesfera de radio R, se inscribe un


cilindro circular recto, de volumen máximo.
Dicho volumen, e s :

A) f * R 3 B ) f * R 3V2

C ) | k R3 ^ 6 D ) f * R 3^3

E) N.A.
Solución
Sean x e y, radio de la base y altura del
cilindro, respectivamente.
El volumen V, del cilindro:
V = 7cx2y ( I ).
En el triángulo rectángulo, obtenido de la
sección indicada: x2 = R2 - y2 ( I I ).
Reemplazando ( I I ) en ( I ):
V = n ( R2 - y2 ) y
V = ti R2 y - n y3
*94
Es decir, hemos expresado V como una función de y.
Para hattar ei valor de y, que dá elmáximo V, igualando a cero la derivada
dV
dy

JL( k R2y - n y * ) = 0
dy

n R2 - 3 * y2 = 0

De donde :

Ahora, en ( I I ):

Y, finalmente, en ( I )

máximo
= - tcR3 J 3
9 V
Rpta: ( D )

La-figura muestra un octavo de esfera, de radio


B R, en el cual se inscribe un cilindro circular
recto, de volumen máximo. Dicho volumen, es:

A )f *R B ) - k R3 / 6

C ) | » t R 3( 3 - ^ 2 ) D) | tcR3( 3 V 3 - V 6 )

E) | ^ R 3 ( 3 V 3 - 2 V 6 )

Solución

Sen x e y, radio de la base y altura del


B
cilindro.

El volumen V : V = n x2y ( I ). 'J ~ ü '

R / y
E, es el punto de contacto de la super­ y
y
y

y
ficie esférica con la circunferencia de y
/

y
la base superior del cilindro. y
y

/
71 □ — * _ c

OM biseca el Z AOC .*. OM = x ^ 2

En el A AHE, de la sección BOD :


995
y2 = R2 - ( x / 2 +x f

# »

y2 = R2 - x2 ( 3 + 2 / 2 )

De donde:
R2 - y2
x2 = (ID
3 + 2 /2
- Reemplazando esto, en ( I ):
/
R2 - y2
V= 71
k3 + 2 /2 /

V = 7t ( 3 - 2 / 2 )R2 y - 7t ( 3 - 2 / 2 ) y3

Entonces, para el Vmáiomo:


dV
= 0 -> J t( 3 - 2 /2 )r2 - 371( 3 - 2 / 2 )y 2 = 0
dy

- Efectuando : y = 5 /3

- En ( I I ) : x2 = ( 3 - 2 / 2 )( - R 2 )
3
- Finalmente, en ( I ):

Rpta: ( E ).

99B
r

CAPITULO 3 5
mammmm
B H B M m

í
: XNTrt**

1
TEMAS SELECTOS:
D E M O S T R A C IO N DE T E O R E M A S Y P R O P IE D A D E S EN
PO LIED R O S.

1. SUPERFICIE ESFERICA TANGENTE A LAS ARISTAS DE UN CUBO.

D
Consideremos el cubo ABC...H, cu­
yas aristas tienen longitud a. Determi­
nemos la longitud x del radio de la
superficie esférica tangente a todas
las aristas. El punto de contacto con
ellas, es su punto medio. El centró de E^a
la esfera es el mismo centro del cu­
bo.

D - Luego , si M es el punto de tangencia en


EF: OM = x.
b / I

i 0
- P, centro de la cara AHEF.
i
a En el A OPM : OP = PM = -
Hj í ' vX
/ ' " fe ::
M
/V F
1------ a ------ i

SUPERFICIE ESFERICA TANGENTE A LAS ARISTAS DE UN TETRAEDRO REGULAR.


Consideremos un tetraedro regular ABCD, cuyas aristas tienen longitud a. Queremos hallar
la longitud x, del radio de la superficie esférica tangente a todas la aristas del tetraedro. Basta
notar que el centro de esta esfera es el mismo que el de la esfera circunscrita, donde :

OA = OB = OC = OD = R.
QO'T
C
Además, en la sección AOD , OM 1 AD , OM = x

Entonces : MD = — o

Y, en el A OMD:

x2 = R2 -

Como se sabe: R = a J l . ( V e r el n° 10; 2o)


4
\
a
x2 = 1 ^
2 /

X= 4 /2

3. TEOREMA DE EULER.- En todo poliedro convexo, el número de caras aumentado en el de


vértices, es igual al de aristas más dos.
Siendo C el número de caras, V el de
vértices y A el de aristas, hay que probar
que:
' C + V = A + 2 I .......... ( 1 ).

Sea una superficie poliédrica abierta termi­


nada en una línea poligonal plana o no
plana ABCDEFGHIJ (fig. 1).
Los elementos de ella cumplirán esta rela­
ción: j------------------------- .
fig. 1 C + V = A + 1 I ............( 2 ) .

En efecto, se cumple en el caso de una sola cara, pues el número de lados es igual al de
vértices. Bastará probar que si se cumple la fórmula anterior para una superficie de C caras,
se cumple para C + 1 caras. Añadamos a la superficie de C caras una cara más CDEFK
( fig. 2 ) con m lados y m vértices. Suponiendo que esta nueva cara deje todavía abierta la
superficie poliédrica, su contorno no podrá coincidir con la línea que antes limitaba la
abertura, sólo coincidirán p de los m lados. Al tener p lados comunes con la superficie,

998
tendrá p+ 1 vértices comunes, o sea
las caras son ahora C + 1 Jos vér­
tices V + m - ( p + 1 ) y las aristas
A + m - p.

Y componiendo la relacción que pro­


pusimos ( 2 ) :

C + 1 + V + m - ( p + 1 ) = A + m-p+1

C + V = A +1

Queda, pues, probada la exactitud de la fórmula ( 2 ) en virtud del principio de inducción.


Pero ocurre que al añadir la última cara que cierra el poliedro, el número de vértices y el de
aristas no aumentan, pues unos y otras son comunes a la superficie y a la cara que se añade.
En cambio las caras aumentan en una unidad.
Así, en la fórmula ( 2) , si el primer miembro ha aumentado en una unidad, para que subsista
la igualdad habrá que añadir uno al segundo miembro, quedando:

C +V = A + 2

DEMOSTRACION DE QUE, EN TODO POLIEDRO, LA SUMA DE LA MEDIDA DE LOS


ANGULOS EN TODAS LAS CARAS, ES:_____________
lis caras = 3 6 0 ° ( V - 2 ) ~ |

Supongamos un poliedro que tiene


m, caras de n, lados cada una;
m2 caras de n2 lados cada una;
m3 caras de n3 lados cada una;
etc. Entonces:

Ito d a s las caras = mi x 180° ( ni * 2 ) + m2 X 180° ( O, - 2 ) + ...

= 180° [ rr^n , + m 2n2 + ... -2m1 - 2m 2 -... ]

^-todas las caras ~ t ^1*^1 * ^2*^2 * " " ^ ^ 1 * ^ 2 ■* ■• • ■) ] ............ ( ^ )•

Por otra lado: m, + m2 + ... = C, ( número total de caras ).

y : m ,^ + m2n2 + ... = 2A, ( siendo A, el número total de aristas ).

Reemplazando esto en ( 1 ):

Xtodas las caras = 180°t2A * 2C]= 360° ( A - C ); pero , por el Teorema de Euler:
A+2 = C+V -» A-C =V-2

Luego. X to d a s las caras = ( V " 2 ) .............. l. q . q . d .

999
5. DEMOSTRACION DE QUE SOLO EXISTEN CINCO POLIEDROS REGULARES.
Vamos a demostrar que sólo existen 5 poliedros regulares, es decir 5 poliedros convexos
cuyas caras, en cada caso, son todas regiones poligonales regulares congruentes. Para ello,
consideremos:

C = número de caras.
C vértice

A = número de aristas.
V = número de vértices

También:

n = número de lados en cada cara,


m = número de aristas que concurren en cada vértice

- Se tienen : mV = 2 A ............ ( I ).
nC = 2 A ( I I ).

Igualando los primeros miembros de ( I ) y ( I I ):


nC
mV = nC -> V = (III)
m

- Por el Teorema de Euler:


C +V = A + 2

Escribamos esta relación sólo en términos del número de caras ( C ). Para ello, de ( I I )

a = ü£

nC nC
Y , con ( I I I ) : C+ ------- = — + 2
m

- Despejando C : (a )

- Ahora estamos en condiciones de averiguar cuántas caras tendrán los poliedros regulares,
dando valores a m y n . Es evidente que, el menor valor de m es 3. También el menor valor
de n , es 3.

- Probando, para m primero y luego n, cuidando que C resulte entero:

12
( 1 o ) Si m = 3, reemplazando en ( a ), queda : C = --------
6 - n

- Luego, en esta última expresión de C,

* Si n = 3 -> C = 4 ( Tetraedro regular)


* Si n = 4 C = 6 ( Hexaedro regular)

* Si n = 5 C = 12 ( Dodecaedro regular)
9

1000
8
( 2o ) Para m = 4; en la expresión (a), luego de simplificar, queda : C =
4 - n

* Aquí, el único valor posible de n, es 3 ; lo cual conduce a C = 8 ( Octaedro regular)

20
( 3o ) Si m = 5, en ( a ) : C =
10 - 3n

* Deducimos que, en este caso, el único valor posible de n ; es 3. Resultando C = 20 ( Ico


saedro regular).

(4o) Si m = 6, en (a), queda : C =


3 - n
expresión que no lleva a determinar algún otro poliedro.

(5o) Análogamente, para otros valores de m, no se logra solución.

- Así, queda demostrado que sólo existen 5 poliedros regulares

6. VOLUMEN DEL ICOSAEDRO REGULAR.- Consideremos un icosaedro regular de arista a


- Tenemos:

^icosaedro “ 20 ^OFAG ’ es decir , 20 y \ s

veces el volumen de la pirámide


OFAG :
1
ico sae d ro = 20 x - x área FAG x OP

( 1) -

La apotema OP del icosaedro, se


calcula en ef A OPF:

OP -V OF -P F

Siendo, en el A FAG, equilátero : PF f 3 = FG P F /3 = a

• • PF =

De otro lado : OF = — = AF2 + AC2 , en el A FAC

AC es diagonal del pentágono regular ABCHG : AC = >/~5 + 1)

Entonces 1 /2 a
0 F = n l a‘ + T ^ + 1) = J a/ 10 + 2 >/5

1001
a2 a 7 + 3 /5
- Reemplazando en OP OP = J 10 + 2^~5 j -
3 2
- Finalmente, en ( 1 ):

V 313 7 + 3 /5 _ 5a3 7 + 3 /5
V-
ic o s a e d ro -” 2«0 xA a
a 2 V x 3 '•
A , _______ = _ “ * Vicosaedro

o también: V icosaedro (3 + V 5 )
12

7. VOLUMEN DEL DODECAEDRO.- Consideremos un dodecaedro regular, cuya arista tiene


longitud a.
El volumen del dodecaedro será 12
veces el volumen de la pirámide
OABCDE:

dodecaedro
= 12( VOABCDE ) (1). V

OP es la perpendicular del centro del


dodecaedro, a la cara ABCDE.

OP es apotema del poliedro regular.


P, es centro de la cara ABCDE.

Entonces :
N
1
OABCDE
= - ( área ABDCE ) x OP ... ( 2 ).

- PA es el circunradio del pentágono regular ABCDE. Se sabe :


PA í------------ ,— PA
AB = — ^ 1 0 - / 2 0 a= ^10 - 720

2a
De donde : PA = (« )
^ 1 0 -/2 0

- En el A OPA : OP = OA - PA (3).

Al 1 / 2 n ^ I =2
De otro lado : OA = — = — y AH + Hl = —y a2 + Hl f en el A4HA
2 2
— TH / rz-
Hl es diagonal del pentágono regular TIMBH : Hl =

- TH es diagonal del pentágono regular de lado a: TH = >/"5 + 1)

- Entonces , reemplazando en lo anteior Hl = f e + 1)

1002
H, = | ( 3 + ^ 5 )

y , en OA : OA = j J a 2 + 3+ )2

OA2 = - y ( 9 + 3 / 5 ) (P)

Reemplazando ( a ) y ( p ) , en ( 3 ):

OP2 = —2 ( 9 + 3 / 5 ) - 4 a2
8 1 0 - J 20

a i 25 + 1 lJ 5
De donde OP = — J ----------- — (4).
2V 10

El área del pentágono ABCDE :

1 5 PA / 1— \
área ABCDE = 5 x — x DC x PZ = — x a x y 5 + 1]

área ABCDE = -a . n,f


8 ^ 1 0 -/2 0

área ABCDE = — ^ 2 5 + 1 0 / ? ............( 5 ).

Finalmente, reemplazando ( 4 ) y ( 5 ), en ( 2 ) y luego en (1 ):

V - ip y
dodecaedro ~
3 4 ’ v 2) 10

_ 5a3 47 + 2 1 /5
Vdodecaedro ó , tambiém :
10

RELACION ENTRE LAS ALTURAS Y EL RADIO DE LA ESFERA INSCRITA EN UN


TETRAEDRO.
Sean ha, hb, hc y hd, las longitudes de B
las alturas del tetraedro ABCD, traza­
das desde los vértices A, B, C y D,
respectivamente.

Sean, además, r el radio de la esfera


inscrita y Sa, Sb, Sc y Sd. áreas de las
caras opuestas a los mismos vértices.

El volumen V, del tetraedro, se puede


escribir:
1003
- Además, al trazar OA , OB , OC y OD:
V +V +V + VOABC = v ABCD
v O B C D t v O A C D ^ V OABO ^

1 1 1 1
— x S a x r + — x S bx r + — x S c x r + — x S tíx r = V

l r [ Sa + Sb+ Sc + Sd] = V

- Reemplazando aquí, los equivalentes de Sa,


a» Sh,
b’ S.c y Sri, según las expresiones de ( * )

1 3V 3V 3V 3V V
—r + + + -----
3 h. fv h_

- De donde, al simplificar V y 3 , pasando luego r al primer miembro, queda:

(« )

9. RELACION ENTRE LAS ALTURAS Y LOS RADIOS DE LAS ESFERAS EXINSCRITAS EN


UN TETRAEDRO.-

Consideremos un tetraedro ABCD:


E, centro de la esfera exinscrita relativa
a la cara BCD, y ra su radio. Esta
superficie esférica es también tan­
gente a los planos de las otras tres
caras.

Llamemos rb, rc y rd los radios de las


otras esferas exinscritas. ( La denomi­
nación es tal que las esferas son
opuestas a los vértices A, B, C y D,
respectivamente).

Sean también ha, hb, hc y hd, alturas respectivas del tetraedro.


El volumen V del sólido:
• • V
v
= V EABC ^+ V EABD r+ V EACD - V EBCD

inru
\

1
V = “o [^ Sria x r. a+ S.c x ar_ + Shb x raa - S.a x r.a J]

Es decir:
1
V = -o x r aa[1 S rid + Src + Sh-
b
SJ
a*

3V
Pero, con las relaciones de la demostración anterior : SQ
o
=

- Entonces:

3V 3V 3V 3V
V = - x r + + --------------

De donde:

(P )

En forma análoga:

_
1 =
1 L +J_ + _l
rb ha hb hc hd

1 1 1 1 1
— + —
rc ha hK
b
h„c hH
d

1 1 1 1 1
— + + —
rd ^a hc

10. CASO DEL TETRAEDRO REGULAR.-

1°) Radio de la esfera inscrita.- En la fórmula ( a ), al reemplazar:


longitud de la altura del tetraedro regular:
1 1 1 1 1
h h h h r

( Y)

- Además, recordando que, si a es longitud de la arista : h = —^ 6 . Entonces, la longitud


3
del radio de la esfera inscrita, en términos de la longitud a de la arista, es :

( e ).

2o) Radio de la esfera circunscrita.- En el gráfico adjunto O, es el centro común de las esferas
inscrita y circunscrita al tetraedro regular ABCD. O, estará sobre la altura BH del sólido.
Luego: BO + OH = BH
1005
R + r = h. Pero, según ( y ) : r = ™

3
Entonces : R = - h R =3r (5 )
4

3°) Radio de la esfera exinscrita.- En la expresión ( p )

_ L = _ 1 + 1 + 1 + 1 , siendo ra = rb = ... = r’
a

De donde : ( ( 0)

ó, en función de la longitud de la arista a : <»

11. PRISMOIDE O PRISMATOIDE.- fig. 2


H

Es el sólido que tiene por bases dos


polígonos situados en planos parale­
los y cuyas caras laterales son trape­
cios o triángulos. Algunos ejemplos,
son:

Prismatoide de bases
ABCD y EFG

1006
Volumen del Prismoide.- Si S, y S2, son áreas de las bases; d, distancia entre los planos
de dichas bases, y S el área de la sección equidistante de ellos, elvolumen V del prismatoide
viene dado por la fórmula:

V = | ( S1 + S2 + 4S ) I

Para demostrar esta expresión, veamos un teorema previo.

Teorema.- El volumen de una pirámi­


de trapecial cualquiera, es igual a
cuatro veces el volumen de una pirá­
mide que tiene por vértice cualquiera
de los vértices del trapecio y por base
la región triangular formada al unir el
vértice de la pirámide trapecial y los
puntos medios de los lados no parale­
los del trapecio.

La figura 4, muestra una pirámide de vértice


O y base ABCD, donde BC 11 A D .

En la figura 5, MN es mediana del trapecio.


Debemos demostrar, que:

OABCD - 4 ( V BMN0)

En efecto, sabemos, por propiedad para


todo cuadrilátero, que :

área MBN = ^ ( área ABCD ) ........... ( Z ).

Si h, es la distancia de O al plano ABCD

v oabcd = l ■( área ABCD ) • h

1
V BMNO . ( área MBN ) . h {Al tomar por vértice O y base MBN.

Dividiendo miembro a miembro estas dos últimas expresiones:


VOABCD área ABCD
Y , con( Z ): V0ABCD - 4 ( VBMN0 ) l.q.q.d
V,BMNO área MBN

1007
c) D em ostración de la fó rm u la del volum en del p rism o id e .

- Consideremos el prismatoide de
bases FEG ( triangular) y ABCD
( cuadrangular).
área FEG : S , área S

área ABCD: S,

distancia entre las bases: d.

MNPR, es la sección equidistante


de las bases y tiene área S. área S 1
Sea O, un punto de la región MNPR. Los planos determinados por O y las aristas,
descomponen al prismatoide en pirámides cuyo vértice común es O y bases las del
prismatoide y las caras laterales del mismo.

El volumen V , del prismoide se evalúa sumando ios volúmenes de las pirámides:

V = V q a b c d + V 0 E F G + V q e f a d + ^ o c d e + ^ o e c b g + V OABGF ................ ( 1 )•

Si x d
_ °1
Siendo: V OABCD (2 )

_ S 2 xd
V OEFG ¿(s2)' 1 ' (3)
V 2 y

Aplicando el teorema anterior:


1 4d
^ o efa d “ ^ V EMN0 “ ^ x 3 x ^ re a O M N x 2 - § x área O M N (4)

1 d 4d
^ocde — ^ ^eonp —^ x 3 x área NOP x ^ x área N O P (5)

f d 4d
^oecbg “ ^ ^eopr “ ^ x 3 x área OPR x ^ x área O P R (6 )

1 d 4d
^oabgf ” ^VFMOR “ ^ x 3 x área MOR x ^ x área M O R (7)

- Reemplazando en (1 ) , las expresiones del ( 2 ) al ( 7 ):


v = Si x d + S2 x d + 4_d {área QMN + área NOp + área o p r + áfea MQR j
6 6 6
Si x d S2 x d 4d
V = ■ -- - + — + — [ área MNPR ]

Siendo : área MNPR = S


St x d S2 x d 4d
Entonces : + + — x S

De donde:
V = - ( S 1 + S2 + 4 S ) .. I.q.q.d

1008
\

CAPITULO 3 6

I. TRAZADO DE PARALELAS Y/O PERPENDICULARES CON ESCUADRAS

n. CONSTRUCCIONES GEOMETRICAS, CON REGLA Y COMPAS.

III. LOS TRES PROBLEMAS FAMOSOS DE CONSTRUCCION.

I. TRAZADO DE PARALELAS Y/O PERPENDICULARES CON ESCUADRA.

Se coloca el cartabón de forma que su hipotenusa, FH, coincida exactamente con la recta
dada. La hipotenusa, CE, de la escuadra se coloca firmemente sobre el cateto FG del
cartabón, bien en la forma dibujada en la figura o bien con el ángulo de 60° ( E ) en el lugar
del de 30° ( C ). A lo largo del lado CE se va deslizando el cartabón y trazando las paralelas
que se deseen. Pueden trazarse igualmente y con el simple giro del cartabón, paralelas
perpendiculares a la dada AB, o bien con 45° de inclinación. La escuadra fija debe mante­
nerse bien sujeta con la mano izquierda. La mano derecha desplazará la escuadra móvil.
II. CONSTRUCCIONES GEOMETRICAS CON REGLA Y COMPAS.
Haciendo uso de una regla sin marcas de unidades y un compás, vamos a solucionar
algunos problemas gráficos de construcción.

II. 1 Problemas Básicos:


1) Ubicación del punto medio de un segmento.-

Dado el segmento AB, ubicar su pun­


to medio.

Solución
Haciendo centros en A y B, con radios A- •B
r, aproximadamente mayor que la mi­
tad de AB, se trazan dos arcos, los
i
cuales se cortan en los puntos P y Q. J
'] 'P
f i
Si se trazan AP , PB , AQ y Q B , el tt ii
cuadrilátero APBQ será un rombo.
Entonces, PQ, es mediatriz de AB. vq

- El punto medio de AB es la intersec­


ción de PQ y AB.

2) Trazo de la bisectriz de un ángulo.-

- Dado el ángulo AOB, trazar su bisectriz.

Solución
- Haciendo centro en O, vértice del Z
AOB, se traza un arco con cualquier

- Luego, con centros en P y Q, y el


mismo radio R (que puede ser igual a
r), se trazan otros dos arcos, los cua­
les se cortan en el punto M, interior al
ángulo.
■ — %
- Entonces, OM será la bisectriz del
ángulo AOB, ya que los triángulos
OPM y OQM son congruentes.
1010
Graficar un triángulo, conociendo las longitudes de los tres lados.

Sean a, b y c, las longitudes de los lados del


triángulo que se quiere formar.

Solución
Sobre una recta, tomamos un punto
\ s
M / ✓
P. Luego, con el compás, medimos \ ✓

una longitud PQ = a.

Con centro en P y radio b, trazamos un


arco. Con centro Q y radio c, trazamos
otro arco que corta al anterior en el
punto M.

PMQ es el triángulo que dá la solu­


ción.

Copiar un ángulo dado.-

- Dado un ángulo ABC, copiarlo de modo


que uno de sus lados sea el rayo dado
OP.
o

Solución
Con centro en B y cualquier radio r ,
se traza un arco que corta BA en M y BC
en Q.

Haciendo centro en O y radio r, se traza un


arco, que corta OP en F.

Luego , con centro F y radio QM , se


traza otro arco que corta al anterior en el
punto T.

El ángulo TOP, es congruente al ABC, ya


que los triángulos TOF y MBQ, son con­
gruentes. ( LLL).

Por un punto exterior dado, trazar una recta paralela a otra dada

Dados el punto P y la recta m, trazar J*


por P, una recta m’ paralela a m.
♦m

1011
Solución

- Tomemos dos puntos A y B, de m.

Con centro en P y radio AB, se traza m


un arco. r \

Haciendo centro A y radio PB , se


traza otro arco que corta al anterior en
el punto C. B m

Como PC = AB y AC = PB, entonces


ACPB es un paralelogramo.

- m’ pasa por P y C.

Otro procedimiento:

- Se toma un punto O, de m y haciendo


centro en él, con radio O P , se traza una
Q m
/
> S N semicircunferencia que corta m en E y F.
e n

- Con centro en F y radio EP, se traza otro


arco que corta al anterior en Q.

i —
o F m- Como EP = F Q , entonces m.

6) Trazar una perpendicular a una recta, por un punto dado

a) Si el punto está fuera de la recta dada. P

- Dados: L y P. ♦ L

Solución
Haciendo centro en P, se traza un arco
i
con radio suficiente, de modo que corte
a L en los puntos A y B.

Jn
i La mediatriz de AB dará solución al
i B
A i problema, ya que P equidista de A y B.
i
ii
s i*
,
V 4
Para trazar dicha mediatriz, basta trazar
\ |

> :q dos arcos con centros A y B, por debajo


de L, con el mismo radio y ubicar el punto

Q. Entonces : PQ _L L.

b) Si el punto está en la recta dada


♦ L
- P, sobre L.

1012
Solución
*
Se traza una circunferencia con cen­
/t y
i \\
tro P y cualquier radio, cortando a L en / ii \
i
los puntos A y B.

Luego, se traza la mediatriz de AB.


Ib
PQ, dá solución al problema.

Otros procedimientos:

c) Por el extremo B, trazar una perpendi


B cuiar al segmento dado AB.

1 a S o lu c ió n

Con centro en B, se traza un arco cual­


quiera y con el mismo radio, otro arco que
corta al anterior en el punto D.
\

Se prolonga CD hasta E, de modo que *


DE = CD. Dy
'jtr
' * \\
Entonces : EB 1 AB, ya que :
i /
¿ ECB = 60°, por ser A CDB equilátero
\L \
y CE = 2CB. A B

2da Solución
Se traza una semicircunferencia AC, de
centro B y radio B A .
P/
s. i - -
Q
V >

/ '.
X
Con el mismo radio y centros A y C, se
\ Nx
\ > trazan arcos que cortan al anterior en los
/ /
/ / puntos P y Q.
/ /

/ * V> \ \V

> \
f

I /
,
Las prolongaciones de AP y CQ, se en­
\ »
A¿ \ B ::5C cuentran en F: FB _L AB. ( ¿ Por qué ? )
T \

Dividir un segmento en un número dado de segmentos congruentes.

Dividir el segmento AB, en 3 segmen


tos congruentes. B

Solución

Por A, se traza un rayo cualquiera, que no esté en AB.

Sobre este rayo, se toman sucesivamente, con el compás, 3 segmentos congruentes :


APi , Pt P2 y P2 P3 . ( La longitud común a estos segmentos, puede ser cualquiera ).
1013
- Luego, se traza P3 B .

- Por los puntos P2 y P v se trazan


paralelas a P3 B, obteniéndose los

puntos M y N , sobre AB.

- Como AP1 = P1P2 = P2P3 , entonces:


AN = NM = M B , quedando resuelto
el problema.

* Nota.- El procedimiento es el mismo


para cualquier otro número de partes
congruentes.

II.2 Aplicaciones.

1) Construir un rectángulo, dadas sus dos longitudes.

- Dadas a y b, longitudes de los lados de


un rectángulo, construir dicha figura.
a

Solución
Sobre una recta n, se toma un punto
A, el cual será un vértice del rectángu­
lo.

Desde A, sobre n, se toma una longi­


tud AB = b.

Por B, con alguno de los procedimien­


tos estudiados en el problema básico
número 6, se traza una perpendicular
a AB y sobre ella se toma BC = a.

Con centro en A y radio a se traza un arco que corta en D al arco trazado con centro C,
y radio b.

ABCD, es el rectángulo pedido.

* Nota.- Si se pidiera una longitud ^ a2 + b2 , la solución es AC.

2) Construir un cuadrado, conociendo la longitud del lado

- Dada la longitud L del lado de un


cuadrado, construirlo. L

1014
\

Solución

E! procedimiento es igual al anterior, ha


ciendo:
a = b = L.

A L B
3) Construir un cuadrado, conociendo la longitud de su diagonal.
- Dada la longitud d de la diagonal de un
cuadrado, construir dicha figura.

Solución
Sobre una recta n, se toma un punto A
im
y a partir de él una longitud AC = d.

Se traza la mediatriz de AC , es de
ci r :
m 1 n , en M ( punto medio de AC ).
n
- Sobre la recta m, desde el punto M, se
ubican los puntos B y D, de modo que
MB = M Ay MD = MA.

- ABCD, dá solución al problema.

4) Conociendo la longitud de un segmento, construir otro de longitud k veces el dado.

4.1) a Dado el segmento de longitud a, construir


otro de longitud doble, t r i p l e , e t c .

Solución

Sobre un rayo cualquiera Á ?, se to­


man los puntos Av A2, A3, .... de modo
A A 1 A2 Ag A^
que AA1 = a; A ^ 2 = a; A2A3 = a;
sucesivamente.

Entonces:
AA2 = 2a; AA3 = 3a; AA4 = 4 a ;...

4.2) Dado el segmento de longitud a, construir otro cuya longitud sea una fracción de a

- Por ejemplo,obtener un segmento de


a
3
longitud —a .
Solución
Sobre un rayo cualquiera se toma la
longitud AB = a.

Por A, se traza otro rayo, en el cual se


toman longitudes iguales entre sí :
AP, = P,P2 = P2P3 = P3P4 = P4P5.
(Una cantidad igual al numeradordela
fracción dada.)

Se traza P5 B y luego las paralelas

r-jc , ...

- Entonces: AE = EF = FG = GH = HB y Es la solución

4.3) Dado un segmento de longitud a, construir


otros de longitud :
aj~2 , a ^ 3 , a / 5 ,...
Solución

1o) Longitud a ^ 2 :

Se toma una longitud AB = a , y luego


se traza BC 1 AB, con BC = a.

Como el A ABC es isósceles, recto


en B :

2o) Longitud a ^ 3 :

Teniendo las longitudesay a ^[2 ,se puede


graficar un triángulo rectángulo cuyos
catetos tienen estas medidas :
PQ = aj2. y QF = a.

- Entonces:

PF - PQ + QF -iw ij + a

3o) Longitud a J E :

- Ahora, con las longitudes conocidas: aJE y aj~2

EM = afó y MH = aj2
1016
EH = ^ E M 2 + M H +( , ^ >

Dada una circunferencia, cuyo centro ( O )


es conocido. Trazar las tangentes a efla,
desde un punto ( P ), exterior.

Se traza PO •

Se ubica M, punto medio de PO-


P
Con centro M, se traza la circunferen­
cia de radio MP, que intersecta a la
circunferencia dada, en los puntos A
y B.
PA y PB, son las tangentes pedidas ya que los ángulos inscritos OAP y OBP, en la
circunferencia de diámetro O P , son rectos.

Dados un segmento AB y un ángulo P, de


medida ot° construir el arco capaz de dicho
ángulo, cuyos extremos sean A y B.

Solución
Se copia el ángulo P, teniendo por
vértice A y un lado AB, obteniéndose
BÁD.
Se traza por A, una perpendicular a
ÁD.

Se traza la mediatriz de AB.

Ambas perpendiculares se cortan en


el punto O.

i 1017
Con centro O y radio OA, se traza una circunferencia, en la cual el arco ACB dá solución al
A AB
A

problema, ya que : ACB = BAD = — -

7) a
Construir un triángulo, conociendo las lon­
b
gitudes de dos lados (a y b) y la altura hc,
h hacia el tercero.

Solución
- Sea el A ABC. Veamos los casos

1o) Si A y B, son agudos.

- Sobre una recta cualquiera n, toma­


mos un punto arbitrario H.

- Por H, elevamos una perpendicular a


ñ, sobre la cual tomamos la longitud
HC = hc.

- Con centro C y radio a, trazamos un


arco que corta a ñ en el punto B. n
- Con centro C y radio b, trazamos otro
arco que corta a ñ en otro punto A,
estando H entre A y B.

- El A ACB, es la solución al problema.

2o) Si uno de los ángulos es obtuso. Por ejemplo Z B

- Sobre una recta cualquiera ñ, se toma un


punto H.

- Por H, se eleva una perpendicular a ñ,


sobre la cual se toma HC = h .

- Con centro en C y radio a, un arco que


corta a ñ en B.

- Con centro C y radio b, otro arco, determi­


nándose A.

- A ABC, obtuso en B, dá solución al proble-

8) Dados dos segmentos AB y CD, tra­


zar la bisectriz del ángulo que deter­
minan. D
1018
Solución
Por B y D, se trazan perpendiculares
B
a AB y CD, respectivamente, sobre
las cuales se toman longitudes
BP y DQ, en la región interior al án­
gulo y de modo que BP = DQ.

- Por P, paralela a AB. Por Q, paralela


a C D . E , es el punto de corte de las
rectas trazadas.
D
- ÉF, bisectriz del ángulo PEQ, dá so­
lución al problema.

9) Dado un segmento, construir su porción áurea.

- Sea a, longitud del segmento dado.

Se quiere construir un segmento de longitud


w a
A B

9.1) Primera Solución

- Se debe encontrar un segmento de


longitud:

- ^ 5 - -
2 2

- Se ubica M,punto medio del segmen­


to dado AB.

- Por B, se eleva una perpendicular a


AB y se toma la longitud BE = a.

- Se traza E M . Luego : EM =

Con centro M y radio — , se traza un arco que corta a EM en el punto F.

Como MF = — , entonces : EF = EM - MF - / 5 - -
2 2

- EF: solución al problema.

1019
9.2) O tra Form a:
/-
- Con centro en B y radio a, se traza un //

cuarto de circunferencia AE. X 4


' / \N
' > //
/
** '\ \ /
f!
- Se ubica el punto medio M, de AB. t
'
\
\ *
*

»
i
/
//
/
x \
\V
Con centro M y radio M E , se traza otro ti t
/
arco que corta a la prolongación de iij
k
AB en el punto P. BP, es solución al M B
problema. a

10) Dividir una circunferencia en 4, 8, 16, ... partes iguales, y construir el cuadrado, octógono
regular, polígono regular de 16 lados, .

Solución

- Para construir el cuadrado, se trazan dos


B diámetros perpendiculares entre sí, por
ejemplo AC y BD.

- ABCD es el cuadrado pedido : AB = tA

- Ai trazar las mediatrices de AB , BC , CD y


AD , se determinan los puntos M, N, P y Q.

- Se construye el octógono regular, al unir

D AM , MB , BN ,... A s í : AM = PQ

- Trazando las mediatrices de los lados del octógono, se encuentran los otros vértices del
polígono regular de 16 lados. Se observa : AT = ¿16

11) Dividir una circunferencia, en 3 ,6 ,1 2 ,2 4 ,... partes iguales, y construir el triángulo equilátero,
hexágono regular, dodecágono,...

Solución
- Para dividir la circunferencia en 6 par­
tes iguales, se marca un punto cual­
quiera A y con radio R (el de la circun­
ferencia dada), se obtiene los puntos
B y F. Haciendo centros en B y F, con
radio R, se ubican C y E, respectiva­
mente. D se obtiene con centro C y
radio R.

- ABCDEF es el hexágono regular:

ÁB = = R

1020
- El triángulo equilátero inscrito se logra al unir los vértices del hexágono de dos en dos:
BF = e3 = R /3

- La mediatriz de AB, determina el punto M sobre el arco AB. AM es el lado del dodecágono
regular inscrito : AM = ^12

12) En una circunferencia de radio R, obtener los lados de los polígonos regulares inscritos:
pentágono y decágono.

Solución
p
- La longitud del lado del decágono :

regularinscrito.es: ¿10 = f E - 1)

- Esto se puede lograr con el método


visto en el 9.2 de esta sección:

- Se traza un diámetro AB y el radio.


OC, perpendicular a él.

- Se ubica M, punto medio de AO y con centro en dicho punto y radio MC, se traza un arco
que corta a OB en D.

- Se verifica :

- Además , como OC = R -» OC = £6

- Y , finalmente : CD =

13) Construcción de Polígonos Regulares. ( Método Generalizado )

A
13.1) Dada una circunferencia de Centro O
y diámetro d, inscribir en ella un po­
lígono regular de n lados ( n = 7, en
la figura). c

- Se traza un diámetro cualquiera AB y


se divide en n partes iguales.

- Con centro en los extremos del diá­


metro y radio AB se trazan dos arcos
de circunferencia que se cortarán en
un punto C. AEFGHIJ: Eptágono regular.
1021
- Se une el punto C y el segundo punto de división del diámetro ( para cualquier caso, siem­
pre es el segundo punto de división ) con una línea recta que se prolonga hasta que corte
a la circunferencia: el segmento AE obtenido será el aldo del polígono regular buscado, que
se lleva sobre la circunferencia en forma sucesiva hasta volver al punto inicial: AE, EF, FG,
etc.

13.2) Dado un segmento que se sabe es el lado de un polígono regular de n lados, construirlo
( AB es el lado de un pentágono regular, en la figura ).

- Se traza una semicircunferencia con centro en A y radio AB para obtener el punto C.

- Con el compás de puntas secas se divide la semicircunferencia en tantas partes iguales


como lados debe tener el polígono buscado y se numera como se muestra en la figura.

- El radio A - 2 ( siempre A - 2 ) será el lado adyacente al lado dado AB.

- Se trazan y prolongan los radios A - 3 y A - 4 donde estarán los vértices que faltan.
^ ^

- Con centro en 2 y radio AB se obtiene D sobre A - 3 y E se obtiene sobre A - 4 ha­


ciendo centro en D o B y con radio AB.

14) Rectificación de circunferencias.


Dada una circunferencia de centro O y diámetro d, rectificarla gráficamente ( Es decir, obtener
un segmento de longitud aproximada a la de dicha circunferencia ).

- Se traza un diámetro AB y perpendi­


cular a él una tangente B X .

- Sobre BX se toma una longitud BC


igual a tres veces el diámetro:
BC = 3( AB).

- Se traza un radio OD perpendicular a


AB.

- Con centro en D y radio r(OD) se traza


un arco de circunferencia que cortará
al arco AD en un punto E.
Desde E se traza una paralela a OD para obtener el punto F sobre AB

Se une C con F, por una recta y este segmento será la circunferencia rectificada con un error
de 1:21 800, aproximadamente:__________
CF = 27rr I

ni LOS TRES PROBLEMAS FAMOSOS DE CONSTRUCCION.


Los antiguos griegos descubrieron todas las construcciones que hemos estudiado hasta
ahora y muchas otras más complicadas. Hubo, sin embargo, varios problemas que los
mejores matemáticos griegos trataron de resolver, durante muchos años, sin éxito alguno.

Para lograr una idea de lo difícil que


puede ser un problema de construcción,
consideremos el problema de dividir con
regla y compás una circunferencia en 17
arcos contiguos, de manera que cada
arco sólo tenga un extremo común con el
arco siguiente.
Cuando se dibujan las cuerdas corres­
pondientes, se obtiene una figura llama­
da polígono regular de 17 lados. Este
problema era muy conocido, pero perma­
neció sin resolver durante más de dos mil
años. Finalmente, en el siglo pasado, el
matemático alemán C.F. Gauss descu­
brió la construcción requerida.
Sin embargo, algunos problemas de los antiguos griegos resultaron mucho más que difíciles;
en realidad, sus resoluciones eran imposibles.

(1) EL PROBLEMA DE LA TRISECCION DEL ANGULO

Se dá un ángulo Z BAC cualquiera ;


queremos construir dos rayos AD y AE
( con D y E en el interior del Z BAC ) de
manera que :

Z BAD = Z DAE = Z EAC

Para esta construcción, sólo debemos


emplear una regla y un compás. Lo prime­
ro que la mayoría de las personas trata de
hacer es tomar AB = AC, trazar BC y ,
luego trisecar B C , como se indica en la
figura de la derecha. Esto no funciona; se
puede demostrar que los ángulos Z BAD
y ZEAC son congruentes, pero ninguno
de estos ángulos es congruente con el
1023
Z DAE. En realidad, nadie ha encontrado un método que efectúe la construcción.

( 2 ) LA DUPLICACION DEL CUBO.


Un cubo de arista a tiene un volumen
igual a a3. Dado un segmento de lon­
gitud a, queremos construir un seg­
mento de longitud b, tal que el cubo de
afista b tenga un volumen doble que el
cubo de arista a. Algebraicamente,
desde luego, esto significa que :

b3 = 2 a3 ó b= a 2 V = a3

Tampoco nadie ha podido resolver este famoso problema. Hay una leyenda curiosa acerca
de éste. Se cuenta que los habitantes de una cierta ciudad griega se morían en gran número
a causa de una plaga, y decidieron consultar al oráculo de Detfos para averiguar el dios que
estaba enojado y por qué. La respuesta dada por ef oráculo fue que Apolo estaba enojado.
El altar dedicado a Apolo en la ciudad consistía en un cubo sólido de oro y Apolo quería que
su altar fuese exactamente eJ doble.

Cuando la gente regresó de Delfos, construyeron un nuevo altar, con una arista doble que
la del antiguo. Entonces, la plaga empeoró en lugar de mejorar, y la gente se dió cuenta de
que Apolo debió haber estado pensando en el volumen de su attar. (Desde luego, al hacer
la arista el doble, el volumen se multiplicó por ocho en lugar de por dos.) Esto planteó el
problema. De modo que la primera oportunidad de aplicar la matemática a la salud pública
fue un fracaso total.

( 3 ) LA CUADRATURA DEL CIRCULO. Dado un círculo, de radio a, queremos construir


un cuadrado cuya área sea igual a la del círculo.

b
L

1
b
A = b2

Algebraicamente, esto significa que b = a J ñ

Durante más de dos mil años, los mejores matemáticos trataron de resolver estos problemas
mediante construcciones con regla y compás. Finalmente, se descubrió en tiempos
recientes que los tres problemas son imposibles de resolver con sólo regla ( no graduada,
ni con marcas ) y compás.

1024
CAPITULO 3 7

T R A N S F O R M A C IO N D E C O O R D E N A D A S

Q T R A S L A C IÓ N Y R O T A C IÓ N DE E JE S C A R T E S IA N O S

1. Introducción - En el presente capítulo se considerará los aspectos matemáticos básicos


para la traslación y rotación de los ejes coordenados así como la transformación de coorde­
nadas cartesinas a coordenadas polares. No olvide que todo lo concerniente a las coorde­
nadas cartesianas ya lo abordamos en el capítulo 32.

2. Fórmulas de Transformación de Coordenadas - Los ejes coordenados originales x,y,


para los cuales el plano IR2 permanece inmóvil, constituyen el punto de partida para la
transformación de coordenadas en lo que respecta a traslación y rotación.

Cuando nos referimos al plano IR2 inmóvil asumimos que los puntos, rectas y gráficas en
general no se moverán mediante una traslación o rotación de los ejes coordenados, sino
que lo que cambiarán serán sus representaciones eon respecto a los nuevos ejes.

Observemos el siguiente gráfico:

1024-A
C o n s id erem o s, c o m o eje m p lo , en el p lano IR2 dos s is te m a s d e ejes c o o rd en ad o s: a) x,y ;
b) x \ y ’ a d e m á s de un punto fijo P; el m ism o que referido a los e je s x,y tien e las c o o rd e n a d a s
P = (4,5 )

Si asu m im o s q u e los ejes originales x.y han sido rotados m e d ia n te el vector unitario de
rotación ü = (u r u2) y tra sla d a m o s al nuevo origen Po o b te n e m o s los nuevos ejes co o rd en ad o s
x:y \ Por lo tanto el m ism o punto P te n d rá las sig u ien tes c o o rd e n a d a s .

(x\y) = (3,2)’

Lo cual quiere decir + 3 u n id a d e s en el eje x’ y + 2 u n id a d e s en el eje y \ A d e m á s , te n e m o s

P = (4,5) = Po + 3u + 2ü J* 0
Ya
Por tanto, te n d re m o s el siguiente gráfico.

A hora deducim os:

El vecto r unitario

u ( u v u 2) = (eos G, sen 0)

es originado por la rotación


dei eje x en un ángulo 0.

Donde:

P0 rep resenta el vector de traslación d e ejes y el vecto r ü re p re s e n ta la rotación d e los ejes.


Teniendo en c u e n ta las c o n s id e ra c io n e s anteriores, la fó rm u la d e tran sfo rm ació n s e ría la
siguiente:

donde, luí = 1

3. Casos Particulares

Si la transform ación consiste en una rotación pura, entonces:

- Po = 0

- (x,y) = x’u + y’u 1 Rotación

Si la transform ación consiste en una traslación p ura sin rotación, entonces:

- 0 = 0

(*) Vector li1 ; Vector perpendicular a ü (Concepto de ürtogonalidad). Su representación gráfica es siempre tal que
parece que el vector Ti hubiese sido rotado 90° en sentido antihorario.
Se cumple que si ú = (a.,a,) ; rix = í-a ?.a,)

1024-B
u = (cos 0 , sen 0 ), lo que indica que no hubo rotación en el eje x, por lo tanto

p0 = ( v y0)
(x. y) = (X0. y0) + x' + y-

Traslación

Las fórm ulas d e tran sfo rm ació n inversa que e x p re s a n las c o o rd e n a d a s ( x \ y ‘) en té rm i­


nos de las c o o rd e n a d a s originales (x,y) se p u e d e n d e s p e ja r a partir d e la ecuación:

P = (x, y) = P 0 + x ’u + y’ u-1-

p ara ello m ultiplicam os p or u y luego por u 1 d e e s ta m a n e ra o b tenem o s:

x’ = ( ( x , y ) - P 0 ) u

y’ = ( ( x , y ) - P 0 ) u-1-

Problemas Resueltos

1. H a lla r las n u e v a s c o o rd e n a d a s del punto ( - 2 , 4 ) cu a n d o los ejes han sido rotados en


4 5 ° y luego tra s la d a d o s al n u e vo origen (5 ,7 ).

Solución:

0 = 45°

ü = (c o s 4 5 °,s e n 4 5 °) = ( - f e 12, 42.12)

P0 = (5 ,7 ) ; P = (x,y) = ( - 2 , 4 )

D e a c u erd o a las fó rm u las de transform ación:

x’ = ((—2 , 4 ) - P 0) . ü = ((—2 ,4 ) —((5 ,7 )) . Ü = ( - 7 , - 3 ) . ü = - ( 7 4 2 + 3 4 2 )I2 = - 5 4 1

y ’ = ((—2 ,4 )—P 0) . ü-1- = ((—2 ,4 )—(5 ,7 )) . ^ = ( - 7 , - 3 ) . ¡ > = (7 4 2 - 3 4 2 )I2 = 2 4 2

Las n u e v a s c o o rd e n a d a s del punto ( - 2 , 4 ) s e ría n ( - 5 4 2 , 2 4 2 )

2 - P o r traslación d e los e je s c o o rd e n a d o s al nuevo origen (5 ,3 ) y por rotación en 5 3 °, las


n u e v a s c o o rd e n a d a s d e un punto P resultan (6 ,8 ). E n co n trar las c o o rd e n a d a s origina­
les de P.

Solución:

P 0 = (5 ,3 ) 0 = 53°

3 4
u = (cos 53°, sen 5 3 ° ) = ~

P' = (x1, / ) = (6,8)

1024-C
M e d ia n te la utilización de las fórm u las d e transform ación

P = (x,y) = P0 + x ’u + y’u-1
4 3
P = (5, 3) + 6 + 8 —— --
l 5 ■5
/
P =

3 - H allar la e c u a c ió n e n las c o o rd e n a d a s tra n s fo rm a d a s de una recta c u y a ecu ación en

las c o o rd e n a d a s origínales es L: y =* x + 5 J 2 d e s p u é s d e una rotación en 4 5 ° d e los


e je s xy.

S o lu c ió n :

S e g ú n los dato s p ro p o rc io n a ­
dos, la recta L fo rm a un á n ­
gulo de 4 5 Q con el e je X, por
lo tanto en el sistem a x 'y \ L
s e rá horizontal. O b s e rv e m o s
el siguiente gráfico:

L a recta L te n d rá la ecu ac ió n : y* (co n sta n te en x ’y ’). P o r lo tanto e s te caso es d e rota


ción pura con un ángulo d e 4 5 ° ( 0 = 4 5 ° ) y

u = (eos 0 , sen 0 ) = ( J 2. 12, J z f 2 )

R esolviendo:

(x, y) = x’ U + y ’uJ' = x ( ^ 2 / 2 , 4212) + y’( - 4 2 12 , 4212)

= ( ( * ’4 2 - y" V2 / 2 ) ), ( (x’ 4 2 + y’ 4 1 / 2 ) )

D e d on de deducim os:

x = 4 2 (x’ - y’) / 2 = (x ’ - / ) / 4 2
y = 4 2 (x' + y’) / 2 = (x ’ + y ' ) / 4 2

Al re e m p la z a r en y = x + 5 4 2 , te n e m o s

X’ + V' X' _ y ' + 5 4 2


42 ~ 42

1024-D
t

X '+ y’ - x ’ - y’ + 10 V 2 => 2y’ = 10

La ecu ación d e la recta en el sistem a x’y ’ es:

y = 5

4 .- H allar la ecu ación tra n s fo rm a d a de la curva xy = 8, si los ejes son rotados en 4 5 °.

S o lu c ió n :

S e g ú n los datos proporcionados:

0 = 45° ü = (1,1)/^2, P0 = 0

(x, y) = x' ü + y ’ü J- = ( x’ - y ’, x ’+ y ’) / j 2

x = (x’ - y ’) / V 2 ,y = (x' + y’) / V 2

Al re e m p la z a r en la ecuación: xy = 8, o b tenem o s:

x' - y 1 x' + y 1

T 7 T ■~ 7 T = 8

x2 - y2 = 16

N o ta : O b s é rv e s e q u e la ecu ación inicial d e !a curva te n ía del térm in o mixto «xy»,


d e s p u é s d e la rotación de los e je s se o b tien e una e cu ació n q u e no co n tien e dicho
térm ino.

5 - T ra n s fo rm a r la ecu ación d e la circunferencia:

x2 + y2 + 6x - 8y - 12 = 0

tra sla d a n d o el origen de c o o rd e n a d a s a su centro y co n se rv an d o la m is m a dirección


d e los ejes.

S o lu c ió n :

P ro c e d e m o s a c o m p le ta r cu adrad os: (x + 3 )2 + (y - 4 )2 = 3 7 , d o n d e P 0 = C = ( - 3 , 4)

adem ás: 0 = 0 ° (no h ay rotación)

x = x0 + x ’ = - 3 + x’

y = y° + y ’ = 4 + yT

Por lo tanto: x + 3 = x’
y -4 = y’

R e e m p la z a n d o en la e cu ació n inicial: (x + 3 )2 + ( y - 4 ) 2 = 37

La ecuación tra n s fo rm a d a será: x ’2 + y ’2 = 3 7

1024-E
6 .- Los e je s xy son rotados m e d ia n te un vecto r unitario u = (4 ,3 )/5 y la ecu ación de una
circunferencia es tra n s fo rm a d a en:

C= x’2 + y ’2 - 10x' + 8 y ? + 2 5 = 0

Encontrar las c o o rd e n a d a s del centro de la circunferencia: *

a) En el sistem a x’y’

b) En el s is te m a xy (utilizando las fórm ulas de transform ación de c o o rd e n a d a s )

Solución:

a) C o m p le ta n d o cuadrad os: (x’ - 5 ) 2 + (y ’ + 4 ) 2 = 16

El centro en el s is te m a x ’y ’ será: C ’ = (5, - 4 ) ’ = ( h \ k’)

b) C o m o sólo se ha e fe c tu a d o una rotación m e d ia n te u = (4 ,3 )/5 .

E n ton ces: (x, y) = x’ u + y'u 1

y el centro C = (h, k) será:

C = (h, k) = h’ u + k’ u-L

= 5 (4 /5 , 3 /5 ) - 4 (—3 /5 , 4 /5 )

C = (3 2 /5 , —1/5)

( Ñ ) SISTEMA DE COORDENADAS POLARES


En los capítulos a n terio res s e h a estudiado el s is te m a d e c o o rd e n a d a s re c ta n g u la re s p ara
localizar un punto en el plano. A h o ra a b o rd a re m o s otro s is te m a d e n o m in a d o S is t e m a d e
Coordenadas Polares. En este sistem a, las c o o rd e n a d a s consisten en u n a distancia y la
m e d id a de un ángulo respecto a un punto fijo y a un rayo fijo.

1. Posición de un punto en Coordenadas Polares


En un s is te m a d e c o o rd e n a d a s polares un punto P del p lano se re p re s e n ta por un p ar
d e n ú m ero s (r, 0), d o n d e r e s la distancia del origen, lla m ad o Polo, al punto dado.

O b s e rv e m o s el siguiente gráfico:

0 es el ángulo d e inclinación del radio


v e c to r O P con resp ecto al s e m ie je x+
(positivo). El ángulo polar se e x p re s a en
rad ian es.

1024-F
2. Notación

Las c o o rd e n a d a s p o lares de un punto P se indican d entro d e un p arén tesis escribiendo


prim ero el radio vecto r y luego el ángulo polar.

Ejemplo: P(2, k/6) se d eterm in a cons­


truyendo el ángulo de k / 6 rad ian es
que tiene su vértice en el polo y su
lado inicial en el eje polar, luego s o ­
bre el lado term inal del ángulo ubi­
cam o s el punto P a 2 u n id a d e s del
polo.

0 x (E je Polar)
3. Consideraciones importantes

a) El ángulo polar p u e d e s e r positivo o negativo, d e p e n d ie n d o del sentido d e rotación.

b) En el s is te m a de c o o rd e n a d a s p o lares c u a lq u ie r punto tien e un n ú m ero ilimitado de


c o o rd e n a d a s , esto se d e b e a que todo áng u lo tien e infinitos áng u lo s coterm inales.

Ejem plo: Si p ro c e d e m o s a ubicar los pun-


\ / \

' Tt' 5n 4
tos: 7 ; nos d a r e ­
\ ’ *3 / V / ? ; 3*
mos c u e n ta que tien en la m is m a posición
com o c o n s e c u e n c ia d e que dichos puntos
tienen igual radio vector y ángulos
co term in ales.

c) En el caso que el radio v e c to r s e a negativo, en lugar d e q u e el punto se e n c u e n tra en


el lado term inal del ángulo, está sob re la prolongación del lado term inal (a c u é rd e s e
que r es negativo)

En el gráfico a d y a c e n te , el punto P e s tá
en el lado term inal del ángulo (esto s e
d e b e a q u e r es positivo), en c a m b io Q
e s tá en la prolongación del lado te rm i­
nal (se d e b e a que r es negativo).

4. Relación entre Coordenadas Polares y Rectangulares

Es posible relacionar las c o o rd e n a d a s re c tan g u lares d e un punto con sus c o o rd e n a ­


das polares, p ara d e te rm in a r dicha relación te n e m o s q u e ubicar el polo con el origen

1024-G
del sistem a d e c o o rd e n a d a s rectan g u lares y luego el eje polar lo hacemos coincidir».
con la p arte positiva del eje d e abscisas.

O b s e rv e m o s el siguiente gráfico:

Y
P es un punto cualq u iera, sie n d o sus
D
l
c o o rd e n a d a s re c tan g u lares (x,y) y sus
c o o rd e n a d a s p o lares (r, 0).

\
y

\ 0
X

P o d e m o s deducir:

* x = r cos 9 * y = r sen 0

* x 2 + y 2 = r2 * tg 0 = -
X

Ejemplo aplicativo

Hallar las coordenadas rectangulares de un punto cuyas coordenadas polares son


J
Solución:

S e deduce: r = 5 y 0 = (6 0 °)
ü

K 5
x = r cos 0 = 5co s X = —
2
n 5^3
y = r sen 0 = 5 s e n — y =
O
./

5 5j3 '
Las c o o rd e n a d a s re c tan g u lares del punto son
2 ’ 2
V /

* En caso d e c o n o c e r las c o o rd e n a d a s re ctan g u lares de un punto, e n to n c e s aplican-


y
do las relaciones: r2 = x2 + y2 y tg 0 = — , p o d e m o s d e te rm in a r sus c o o rd e n a d a s
X

polares.

* D e no h a b e r alg u n a indicación sobre r o 0 , se a s u m e que r es positivo y q u e 0 está


d a d o por -7 1 < 0 < te

1024-H
Ejemplo aplicativo
H a lla r las c o o rd e n a d a s polares de un punto P cu yas c o o rd e n a d a s re c ta n g u a re s son
(-4 .4 )

S o lu c ió n :

P ro c e d e m o s a graficar para
ubicar con m a y o r facilidad el
ángulo 0

* X

N o h ay ninguna indicación sob re r o 0, por ello v a m o s a c o n sid erar a r positivo y a


0 e II C u a d ra n te .

Por lo tanto:

r = , / ( - 4 f + {4f => r = V32 = 4 ^ 2

y -4 3n
tg 9 = — = — tg 0 = - 1 0=
a x 4

3n
Las c o o rd e n a d a s p o lares d e P serán 4 J2 ;
\

Diferentes ubicaciones de un punto en el Sistema de Coordenadas Polares.


A continuación se p resen tan gráficos en las q u e se ubican algunos puntos con sus
respectivas c o o rd e n a d a s polares.

►A

n
• 'ó -3 ;

-2 '
/ \
\ ' 3 tt ^ í 2 tt i
• Q • p
\C 2 , 4 j/ V- s ' - 3
/

1024-1
6. Problemas resueltos

t . - Determinar las coordenadas polares del punto Q cuyas coordenadas cartesianas son
(x.y) = (6,-6)
Solución:

S e deduce: x = 6 : y = - 6

r2 = x2 + y2

r2 = 6 2 + ( - 6 ) 2 = 72 r = ±6^2

-6
tg 0 = — = - 1 ; por lo tanto: 0 = 3 7t/4 ó 7
6
y c o m o Q está ubicado en el cuarto
c u a d ran te, entonces:

r = 6^2 * si 0 = 7 n/4

r = -6 si 0 = 3 tt/4

O b s e rv e m o s el siguiente gráfico:

2 - T ra n s fo rm a r la e cu ació n r2cos 2 0 = 8, a c o o rd e n a d a s cartesian as.

Solución:

S a b e m o s que cos 2 0 = eos2 0 - s e n 2 0 (funciones del áng u lo doble)

r2 cos 20 = 8 r2 (eos2 0 - s e n 2 0) = 8

(r cos 0) 2 - ( r s e n 0 )2 = 8

= 4

Nota: La ecu ación x2 - y2 = 4 c o rresp o n d e a una hipérbola e q u ilá tera con centro
en el origen, eje focal en el eje x, y con s e m ie je s a = b = 4

3 .- T ra n s fo rm a r la ecu ación c a rte s ia n a x2 + y 2 - 4 x = 0 a c o o rd e n a d a s polares.

Solución:
x2 + y2 - 4 x = 0 => r2 - 2 r cos 0 = 0
r (r - 2 cos0) = 0

D o n d e ten em o s: r= 0 o r = 2 cos 0
r = 2co s 0

r = 0 rep resen ta al origen (0 ,0 ) el cual ya está co n sid erad o dentro d e la ecu ación
r = 2cos 0 p a ra 0 = n/2

4 - H allar el á re a del triángulo cuyos vértices son el polo y los puntos (6, ni 4 ) y (1 0 ,7 n/12)

1024-J
Solución: P

Procedemos a realizar

el gráfico respectivo
Q

Área del triángulo POQ = (OQ)(OP) sen (ZPOQ) donde, ZPOG = —

1 k
Área A POQ = — (6)(10)sen

A APOQ = 15^3 u2

5 .- Hallar la ecuación rectangular del lugar geométrico cuya ecuación polar es r = 5sen 0

Solución:
Elevemos al cuadrado la ecuación polar dada
r2 = (5sen 0 )2
y
Sabiendo que: r2 = x2 + y2 sen 0 = —

Procedemos a plantear las siguientes ecuaciones

donde: r2 (x2 + y2) = 25y2

(x2 + y2)2 = 25y2

Problemas Propuestos

1 - Hallar Inecuación en las coordenadas transformadas de una recta cuya ecuaciones


y = x + 3 en el sistema xy, las que serán rotadas en 45°.

2- Hallar la ecuación en la que x2 - 2x + 4y2 - 16y + 13 = 0 es transformada si el origen


se traslada al punto (1, 2).

3- Hallar la ecuación en la que 4x2 + y2 - 8x + 4y + 4 = 0 es transformada si los ejes


coordenados son trasladados de manera de eliminar los términos lineales.

4- Hallar la ecuación en la que cada ecuación de (a) a (d) es transformada si el origen es


trasladado al nuevo origen.

1024-K
a) x2 + y2 - 2x - 4y - 4 = O P0= (1 : 2)
b) 4x2 + y2 - 24x + 4y + 36 = O P0 = (3 .-2 )

5- Hallar la ecuación en la que cada ecuación de (a) a (h) es transformada si los ejes son
trasladados de manera de eliminar los términos lineales cuando sea posible:
a) 4x2 + 9y2 - 8x - 36y + 4 = 0
b) 4x2 - y2 + 24x + 4y + 35 = 0

6 .- Eliminar los términos lineales de (x - 2)2 = 8(y - 1) por una traslación, si fuese posible.

7 .- ¿Hasta qué punto debe trasladarse el origen con el fin de eliminar el término lineal en
y, y los términos constantes en y2 + 2x - 2y - 7 = 0?

8 .- ¿En qué ecuación se transforma (x - h)2 + (y - k)2 = r2 si el origen se traslada a (k,h)?

9- Escribir cada una de las siguientes ecuaciones cartesianas rectangulares en términos


de las coordenadas polares r, 0.
a) x2/4 + y2 / 9 = 1 b)x~2y + 3 = 0
c) x2 + y2 - 6x = 0 d) y = - 4

10.-Expresar cada una de las siguientes ecuaciones, dadas en coordenadas polares, en


términos de las coordenadas rectangulares x,y.
3
a) r = sen 0 b) r = 3 + 2cos 0 c) r2 = 1 - eos 0
r 1- COS0

RESPUESTAS

1 - y’ = 3/^2 , horizontal en x’y

2 .- x’2 + 4y’2 = 4

3 .- P0 = (1 -2 ) ; 4x’2 + y’2 = 4

4 .- a) x’2 + y’2 = 9 b) 4x’2 + y’2 = 4

5 .- a) 4x’2 + 9y’2 = 36 b) 4x’2 - y’2 = 4

6 - No es posible

7 .- P0 = (4,-1)

8 .- x’2 + y’2 - 2 (h - k) (x* + y’) = r2

9 .- a) r2 = 36 / (9 eos2 0 + 4 sen2 0) b) r = 3 / (2 sen 0 - eos 0 )


c) r = 6 eos 0 d) r = - 4 / sen 0

10.-a) x2 + (y - V2)2 = V*

1024-L
MISCELANIA DE PROBLEMAS
PROPUESTOS

S E G M E N TO S , A N G U L O S , IN TE R S E C C IO N DE FIG U R A S .

1) ¿Cuántas de las cantidades siguientes no pueden representar la distancia entre dos


puntos:

^5; -2 ; -1 ; / 3 - J~2 ; / 5 - 3
0 : ! ;

A) 4 B) 2 C) 1 D) 3 E) 5

2) P , Q , R , S y T son puntos consecutivos de una recta. Q, biseca PT ; PR = 3 RS, QS = 12


y PT = 40. Hallar QR.

A) 2 B) 3 C) 5 D) 6 E) 4

3) Dados los puntos colineales y consecutivos A. B, C, D, donde AB = 18, AC = 3 BD y


2 BC = 3 CD. Hallar BC.

A) 2,4 B) 4,5 C) 3,5 D) 4,8 E) N.A.

4) En una recta se toman los puntos consecutivos M, P, Q, R yS, donde:


MP = PQ y QR = 2 RS. Entonces, es cierto que:

2 3
A) MR = - (MP + MS) B) MR = - (QR + PS) C) MR = 4(MS - PQ)
O ¿L
2
D) MR = 3MS - (1/2)PR E) MR = - ( M S - M P )
O

5) A, B y C, son tres puntos colineales y consecutivos, tales que:


AB 2
— = - y 2 AB + 3 BC = AC + 96. Hallar AB.
□O o

A) 12 B) 36 C) 28 D) 24 E)N.A.

AE
6) Se consideran los puntos colineales y consecutivos A, B, C, D y E, siendo BD

AD + BE = 20. Hallar la longitud de BD.

A) 2 B) 3 C) 4 D) 5 E) 6
1025
7) Sobre una recta, se marcan los puntos consecutivos A, B, C, D. E y F, tales que:
AB = BC = CD y CF = 2 BE = 4 AD. Hallar CE, sabiendo que EF = 28.

A) 14 B) 10 C) 21 D) 20 E)N.A.

8) Sean los puntos colineales y consecutivos A, B, C, M y D, donde M es el punto medio de


C D ; CD = 2 BC y AM = 30. Hallar BC, siendo además: BD . AC = 2 BC . AM + BC . CD.

A) 5 B) 6 C) 7 D) 8 E) 9

9) A, B, C, D, E y F, son puntos colineales y consecutivos. AC + BD + CE + DF = 40 y


5 BE = 3 AF. Hallar AF.

A) 20 B) 24 C) 26 D) 28 E) 25

10) Sean los puntos colineales y consecutivos A, B, C, D, E, F.


2
Hallar AF, si: DE = AB; AD = - A F y AC + BD + CE + DF = 35.
5

A) 25 B) 20 C) 30 D) 21 E) N.A.

11) A, B y C, son puntos colineales y consecutivos. M y N, bisecan AB y B C , respectivamente.


Hallar AC, si: 3 MN = 2 MC y AB - BN = 2.

A) 6 B) 12 C) 8 D) 10 E) 16

12) Se marcan los puntos colineales y consecutivos A, B, C, D; siendo: AB = 2 BC = 3 CD.


En AB y CD se ubican los puntos R y S, tal que: RB = DS y A R - C S = 25. Hallar: RS.

A) 25 B) 15 C) 18 D) 30 E) 20

13) Se marcanlos puntoscolineales de una recta: A,B, C, D, E, tales que:


1 3
AC = BD, BC = - D E y - A B + DE = 36. Hallar AE.
O

A) 24 B) 72 C) 48 D) 54 E) 27

14) Sean los puntos colineales y consecutivos A, B, M, C, siendo M punto medio de BC.

Hallar: AB + AC
AM + BM

1 ^ 1
A) 1 B) 2 C) - D) 4 E) -

15) Sobre una recta se toman los puntos consecutivos A, B, C, D, E, tal que F sea punto medio
de AB y H punto medio de DE, si;

1026
AB = BC, CD = DE y AB + DE = 40. Hallar FH.

A) 40 B) 50 ’ C) 30 D) 60 E) 75

16) Los puntos A, B, C y D, son colineales y consecutivos, tales que: AB . CD = BC . AD.


Entonces, se cumple:

1 1 1 1 1 2 1 2
A) + -----= — + — B) ------ + ------ = — +
* AC CD AB BD } AC CD AB BD
2 1 2 1 2 1 1 2
C) + ----= ------ + ----- D I ------- + ------ = ----- + ------------ RNA
' AC CD AB BD ; AC CD AB BD ;

17) En una línea recta, se consideran los puntos consecutivos A, B, C y D.


AB . BC = AD (AC - CD), AC > CD, AB = a y BC = b. Indique la relación correcta.

A) (CD)2 = a2 + b2 - ab B) (CD)2 = a2 + ab - b2 C) (CD)2 = a2 + b2


D) (CD)2 = a2 - b2 E) (CD)2 = a2 + b2 + ab

18) En la figura:
AB = 12 , BC = 10,
AD = x ; DC = x -1 B
y AC toma su mínimo valor entero.

Hallar la razón entre los valores


enteros, mínimo y máximo de “x”.
2 3
B) C)
11 10
2_
D) E)
13 13

19) En la figura:

AB = BC = CD = AO = OD = 2.

Hallar el valor entero de: AE + EF + FD

A) No existe. 8 )6 C) 5

D) 4 E) 3

20) A, B y C, son tres puntos distintos de un plano. Si: AB = 18. y AC = 15; entonces la razón
entre los valores enteros mínimo y máximo de BC, es:

A)
_1_
B) C)
2_t E) N.A.
16 33 D)
11 35,

1027
21) Para el gráfico, hallar la razón entre los
valores enteros, mínimo y máximo, de:
OA + OB + OC + OD.
(O —* Punto interior cualquiera ).

1 1
A) B) C)
3 4 12
6 8
D) E)
17 19

22) En el interior de un A ABC, se toma el punto “O".


Si: AB = 13, BC = 14 y AC = 15. Hallar la relación entre los valores enteros mínimo y má
ximo, de: OA + OB + OC.
22 21 21 22
C) D) E)N.A
a >7T B> 43 44 44

23) En la figura:
AB = 12, BC = 10, CD = 11 y AD = 15.
Sean m y M, los valores enteros, mínimo
B
y máximo, respectivamente, de AC + BD.
m
Hallar: —
M

1 23 25
A) á C)
B> 49 47
26
E) N.A.
D> ^

24) Sobre una recta se marcan “n” puntos consecutivos. Al borrar uno de dichos puntos, se
cuentan 12 segmentos menos, determinados por los restantes. Hallar “n \

A) 12 B) 13 C) 14 D) 24 E) 23

25) A, B, C y D, son puntos colineales y consecutivos. AD = 24 , AB = x - y ,


BC = x + y, CD = 2 y - x . Hallar el valor entero de y.

A) 5 B) 6 C )7 D) 8 E)N.A.

26) Se tienen los ángulos consecutivos AOB y BOC, donde: m A Ó B - m BÓC = 42


Si ÓM, es bisectriz del AÓC. Hallar: mM Ó B .

A) 42 B) 84 C) 21 D) 10,5 E)N.A.

27) AÓB, BÓC, CÓD, D Ó E y EOF, son consecutivos. ( A Ó F , llano). Si


AÓ B = EOF, BÓC = DÓE y m A Ó C = 8 0 ° .
1028
Hallar: m C Ó D .
A) 40° B)30° C) 10° D) 15° E)20°

28) Dos ángulos adyacentes se diferencian en 12°.


Hallar el ángulo que forma el lado común con la bisectriz del menor ángulo.

A) 42° B)48° C) 21 ° D) 24° e)N.A.

29) Sean los ángulos consecutivos AÓB, BÓC y CÓD. Si: A Ó C + BÓD = 140°.
a a

Hallar la medida del ángulo formado por las bisectrices de los ángulos AOB y C O D

A) 20° B) 70° C) 90° D) 50° E) 30°

30) Sean los ángulos consecutivos AÓB y BÓC: A Ó B - B Ó C = 44°.


OM, biseca AÓB; o ñ , biseca BÓC; ÓR, biseca MÓN. Hallar RÓB.
A) 22° B) 44° C) 11° D)12° E)N.A.

31) Se tienen los ángulos consecutivos AOB y BOC, se traza Op y Ó Í bisectrices de los
ángulos AOB y AOC, calcular la medida del ángulo FOE, si BÓC = 60°.

A) 20° B) 25° C) 30° D)35° E) 40°

32) Calcular el valor de un ángulo sabiendo que los — del suplemento de su complemento es
4
igual a un ángulo recto.

A) 60° B) 45° C) 30° D) 15° E) 75°

33) Se tiene los ángulos consecutivos SOR, ROE y EOA, los rayos OI y Ó íí son bisectrices
de SOR y EOA respectivamente; además m ION = 102° y m SOE - m ROA = 8o.
Hallar m SOE.

A) 102° B) 105° C) 110° D) 106° E) 112°

34) Se tienen los ángulos consecutivos AÓB y BÓC, ( A Ó B < BÓC ), tales que:
AOB + 2BOC = 148°. Se traza Ó ? , bisectriz del AÓB y OP bisectriz del EÓC.
Hallar la medida del ángulo EÓP.
A) 17° B) 27° C) 3 7 ° D) 47° E) 57°

35) AÓB y BÓC, son adyacentes. AÓB, mide 144°. ÓM, biseca AÓB; ON, biseca BÓC;
Ó^.biseca A O N y 0 (5 , biseca MÓC.

1029
Hallar: m P Ó Q .
A) 30 B) 45 C) 20 D) 25 E)N.A.

36) En la figura, BD es bisectriz del ángulo


CBE y la suma de los ángulos ABC y
ABE vale 52°. Calcular el valor del án­
gulo ABD.

A) 52c B) 39 C) 26

D) 34 E) 28

37) Hallar “x”, si: rñ / / ñ .

m A) 16c
B) 15'
C) 14
D) 17
E) 18'

38) De la figura, obtener el valor de “x", si: p = 30° y y = 40° ( L, / / L2)

A) 15°
B) 20°
C) 24°
D) 32°
E) N.A.

39) Si: l J I l 2í entonces calcular x + y :

^ M _i
1 A ) 180°
B) 170°
C) 190°
D) 200°
^ )N .A .

►I_2

1030
40) En la figura: m //ñ .
Hallar el valor de x

A) 100c B) 120'
C) 130 D) 110
E) 140'

41) Hallar “y”, si r I I s :

A) 65°
B) 70'
C) 75
D) 80
E) 60'

42) Hallar V , si m //n y r / / 1

A) 29c

B) 31'

C) 30

D) 32

E) 28‘

43) AÓB y BÓC, son ángulos consecutivos ÓP, biseca AÓB; OQ, biseca BÓC.
Si: m ^ P O C ó m P Ó C = 57° y m AÓQ ó m ^ A O Q = 36°. Hallar: m ^ A O C

A) 59 B) 60 C) 61 D) 62 E) 64

1
44) 7 rectas secantes, 8 circunferencias y 9 triángulos, se cortan como máximo, en;
A) 963 puntos B) 693 puntos C) 396 puntos D) 973 puntos E)N.A.

45) 5 ángulos y 8 circunferencias, se cortan como máximo, en;

A) 136 puntos B) 160 puntos C) 216 puntos D) 296 puntos E) 246 puntos

46) 7 rectas paralelas, 6 secantes y 12 pentágonos, se cortan como máximo en:

A) 1029 ptos B) 929 ptos C) 1129 ptos D) 1309 ptos E)N.A.

47) Se tienen “n” triángulos secantes. Si se quitan 3 triángulos, el número máximo de puntos
de corte, disminuye en 54. Hallar “n”.

A) 10 B) 6 C) 7 D) 5 £) 8

48) Hallar el máximo número de puntos de corte entre *n" elipses y H2nn rectas, todas
secantes.
A) 4n(3n-1) B) 3n(4n-1) C) 2n(n + 2) D) 3n(4n + 1) E)4n(n-1)

49) Hallar el máximo número de puntos de intersección entre 5 octógonos y 6 decágonos,


convexos.
A) 460 B) 480 C) 940 D) 840 E) N.A.

50) “n" rectas secantes, “n" circunferencias y “n" triángulos, se cortan como máximo, en:

A) n(29n - 9) B) ^ (9 n - 29) C) 2n(n - 1) D) |( 2 9 n - 9) E) | ( 1 9 n - 9)

51) Hallar el máximo número de puntos de corte de 5 rectas que tienen un punto común y 5
circunferencias que tienen un punto común.

A) 60 B) 61 C) 62 D) 63 E) 64

52) Hallar número máximo de puntos de corte, entre 10 rectas paralelas y 50 circunferencias
dispuesta así;

A ) 1980
B) 1890
C) 1098
D ) 1690
E) N.A.

53) Hallar el número de puntos de corte entre 10 circunferencias concéntricas y 20 rectas que
pasan por el centro común.
A) 400 B) 401 C) 200 D) 201 E) 101
1032
54) ¿Cuántos de los siguientes conjuntos de puntos no son convexos:
I) Una circunferencia II) Un círculo
III) Un cuadrado IV) Una superficie cónica
V) Una superficieesférica VI) Una esfera
VII) Un triángulo VIII) Una región triangular.

A) 1 B) 2 C) 3 D) 4 E) 5

55) Indicar verdadero (V) ó falso (F):


(I) Si un triángulo equilátero tiene igual perímetro que un cuadrado;entonces la longitud del
lado del cuadrado es mayor que la del lado del triángulo.
( II) Una región cuadrada equivalente a una región rectangular delados 9cm. y 4cm. ,tiene
perímetro 26cm.

( I I I ) Una esfera y un cono, pueden ser equivalentes.

( I V ) Toda poligonal envolvente es siempre mayor que la envuelta.

A )W W B)FFFF C )F W F D)FFVF E) F F W

P R O P IE D A D E S B A S IC A S EN T R IA N G U L O S

56) En un A ABC, se traza la ceviana BR.Si: AB = BR = RC y m Z ABR = m Z C.


Hallar m Z C.

A) 30° B) 32° C)36° D) 40° E) 60°

57) En un A ABC, m Z B = 72°. Sobre las bisectrices de los ángulos exteriores A y C, se


toman los puntos P y Q. Luego se traza PQ. Hallar la medida del menor ángulo que for­
man las bisectrices de los ángulos APQ y PQC.

A) 63° B) 72° C)54° D) 36° E) 27°

58) En un A ABC, recto en B, se traza la ceviana interior AE, que prolongada corta en D a l a
perpendicular a AC trazada por C. La bisecriz del ángulo EAC, corta en F a EC.Si:
EC = CD, BE = 3 y AB = 4. Hallar EF.

A) 2,5 B) 2 C) 3 D) 1 E) 0,5

59) En un A ABC, recto en B, AB = 12 y BC = 16. Se traza la altura BH. E n e l AHBC. se


traza la bisectriz interior BF. Hallar FC.

A) 10 B) 9 C) 8 D) 7 E) 8,5

1033
60) En un A ABC, m Z A = 24° y m Z C = 22°. Se trazan las alturas AQ y C R .
Hallarla medida del menor ángulo que forman las bisectrices de los ángulos QAB y BCR.
A) 84° B) 40° C) 46° D) 45° E) 90°

61) En un A ABC, m Z C - m Z A = 38°. Se traza la bisectriz exterior BE. Hallar: m Z BEC.

A) 38° B) 17° C)19° D) 26° E) 52°

62) A, es un punto exterior al A BCD, relativo a CD; de modo que A C D = BCD. Si:
m Z BDC = 80° y m Z BAC - m Z ABC = 24°. Hallar: m Z ABD, sabiendo que
AB y CD se cortan en H.

A) 24° B) 23° C)26° D) 22° E) 28°

63) Enun AABC, m Z B = 72°. Se trazan las bisectrices interiores AQ y C R . Hallar la medida
del menor ángulo que forman las bisectrices de ARC y AQC.

A) 18° B) 28° C)26° D) 27° E) 25°

64) El perímetro de un triángulo rectángulo,es 18 cm. Si “x” es el valor de la longitud de la


hipotenusa, entonces:

A) x < 6 B) x < 5 C)x > 6 D) x = 6 E)N.A.

65) ABC, es un triángulo equilátero y R un punto exterior, relativo a AC, tal que RC = AB y
m Z ACR = 22°. El menor ángulo que forman AC y BR.es:

A) 710 B) 49° C)44° D) 69° E) 72°

66) En la figura: AB = BC y AD = AB + CD. Entonces:

A)x = 2y
B) x + y = 120°
C) x + y = 180°
D) 2x = 3y
E) No se sabe.
a D
67) Enun AABC, recto en B, se traza la ceviana interior BF.de modo que BF = FC. Entonces,
BF, es:
A) Altura B) Bisectriz interior C) Mediana D) No se sabe.

68) Enun AABC, m Z A = 2 ( m Z C ) y AB = 5. Hallar el máximo valor entero de AC.

A) 12 B) 11 C) 13 D) 14 E) 15

1034
69) En un A ABC, obtuso en B: AC = 10 y BC = 2. ¿Cuántos valores enteros toma AB?
A) 1 B) 0 C) 2 D) 3 E) 4

70) En la figura:
AB = BC = CD = DE = DF
y m ^ F = 46°.
Hallar el valor de x.

A) 23c B) 21 C) 24
D) 22 E) 26

71) En la figuran
B
AD = BD y AB = AC.
Hallar el máximo valor entero de a.

A) 46c B) 59 C) 41
D) 32 E) 44

72) En un A ABC, las bisectrices de los ángulos, interior de A y exterior de C, se cortan en el


punto E. La bisectriz del ángulo AEC, corta a AC en F. Si m ^ CFE = 50° y EFÜAB;
hallar la m ^ ACB.
A) 25 B) 30 C) 35 D) 20 E) 50

B
73) En la figura: A ABD, equilátero.
BC = AD. Entonces:

A) a + b = 60£ B) 3a = 2b
C )a + b = 45 D) 2a = b C
E)N.A.

74) En un A ABC, m ^ A = 2(m X- C ). Se traza la bisectriz interior BF.Si AB + AF = 10cm.


Hallar BC.

A) 5 cm. B) 15 cm. C) 10 cm. D) 8 cm. E) 7,5 cm.

75) En el interior de un A ABC, se toma el punto F. S i : AB = FA = FC; m ^ ABC = 8a,


m ^ FCB = 2a y m xf FCA = a. Hallar a.

A) 6 B) 8 C) 9 D) 12 E) 10

1035
C O N G R U E N C IA DE T R IA N G U L O S

76) En un A ABC, m ^ B = 50°; exteriormente se dibujan los triángulos equiláteros ARB


y BFC. Hallar la medida del Y BRC, si m Y BFA = 34°.

A) 34° B) 25° C) 69° D) 59° E) 36°

77) Se tiene un triángulo equilátero ABC. R, un punto de AC. Exteriormente, se dibuja el triángulo
equilátero RFC. Si: m Y ABR = 23°. Hallar: m Y FAR.

A) 23° B) 37° C) 30° D) 45° E) 15°

78) AD y BE, son dos segmentos no perpendiculares, que se cortan en el punto H, de modo
que BÉD = BDE. Sobre HD se toma el punto C, tal que AB = BC. Si, además: AE = CD
y m Y EAD = 42°. Hallarla m Y BAD.

A) 42° B) 84° C) 69° D) 88° E) 79°

79) En el interior de un Á ABC, se toma el punto R, de modo que m Y RCB = 3(m Y RAB).
F, es un punto exterior al A ABC, relativo a B C , de modo que sea equilátero el A RFC. Si:
m Y RFB = 68°. Hallar m Y RAB.
A) 34° B) 22° C) 44° D) 17° E) 19°

80) En un AABC, F es un punto de AB y N un punto de AC. de modo que AN = BC = AB


y AF = NC. Si: m Y NBC = 30°. Hallar m Y FN&-
A) 30° B) 20° C) 40° D) 45° E) 10°

81) En un AABC, m Y B = 128°. Las mediatrices de AB yBC, cortan a AC en los puntos


R y S, respectivamente. Hallar m Y RBS.

A) 76° B) 64° C) 52° D) 74° E) 68°

82) En un A ABC, las mediatrices de AB y BC, se cortan interiormente en el punto H. Si:


m Y ABC = 80°. Hallar m Y HAC.

A) 40° B) 20° C) 30° D) 10° E) 5o

83) En un A ABC, obtuso en B, las mediatrices de AB y B C , se cortan en el punto R.


Si m Y ABR = 70°. Hallar m Y ACB.

A) 40° B) 10° C) 15° D) 12° E) 20°

84) En un A ABC, recto en B, se traza AH _L BM, siendoBM mediana. Si:


m Y H A M - m Y RAB = 12°. Hallar m ^ C.

A) 24° B) 28° C) 26° D) 22° E) 36°


1036
85) En un A ABC, recto en B, la altura BH y la bisectriz interior AD se cortan en el punto F.
Si AF = FD. Hallar m X C.
A) 45° B)26° C) 15o D) 30° E) 60°

86) Sobre la bisectriz de un Z A, se toma el punto F y se traza FB perpendicular a un lado


y BD perpendicular al otro lado. Si AB = 17 y BD = 15.
Hallar la distancia de F a BD.
A) 8 B) 2 C) 9 D) 10 E) 6

87) En un A ABC, AB = 16, BC = 15 y AC = 18. Por B, se trazan BÉ y BF, perpendiculares


a las bisectrices de los ángulos, interior de A y exterior de C. Hallar EF.

A) 8,5 B) 9 C) 9,5 D) 10 E) 11

88) En la figura: C
OB = OC, m Z A = 45° y
m Z D = 30°; AB = 6.
Hallar CD.

A) 8 B) 6 C) 4

D) 3 E) 6 ^ 2 B

89) En un A ABC, se traza la ceviana interior CD. Si : m X ACD = 2x¡ m ^ DCB = 4x y


m X ABC = 8x. Hallar el valor de x.

A) 10° B)8° C)6° D) 9° E) 12°

90) En un A ABC, recto en B, sobre AC se toma el punto M, tal que AB =MC. Las mediatrices
de BC y AM se cortan en “R”. Hallar m ^ RCM, si m ^ ACB = 10°.

A) 10° B)20° C) 15° D) 30° E) 40°

91) En un A ABC, AB = BC, recto enB, se traza la ceviana interiorBQ y luego AR _L BQ


( R en BQ ). Si m X RAB = m X RCQ, y BR = 6. Hallar RQ.

A) 6 B)3 C) 4 D) 2 E) 1

92) En un A ABC, “O” es un punto interior de modo que AO 1 OB ;


m \ OAC = 2(m ^ OCA) = 30°, BO = 2, OC = 2 / 2 y AC = 4. Hallar m ^ OCB.

A) 18,5° B) 26,5° C) 7,5° D) 15° E) 22,5°

1037
93) En un AABC, m Z B = 80° y m Z C = 70°, se traza la bisectriz interior B D .S i AD = 8
Hallar la distancia del punto medio de BD, a AB.

A) 4 B) 8 C )1 D) 2 E) yj~3

BC
94) En la figura, hallar:
CD

A )/6

Je
B)

C)

3
D> 2 D

E) N.A.

95) En un AABC, m Z A = 30°, se traza la ceviana interior BD. Si AD = BC y BD = DC


Hallar m Z C.

A) 30 B) 15 C) 45 D) 10 E) 20

P O LIG O N O S , C U A D R IL A T E R O S , C IR C U N F E R E N C IA S .

96) Hallar el número total de diagonales de aquel polígono regular en el cual la medida de un
ángulo interior es 156°.

A) 15 B) 90 C) 72 D) 54 E) 135

97) Las medidas de un ángulo central y un ángulo interior, de un polígono regular, son entre
sí, como 1 a 19. Hallar el número de diagonales que se pueden trazar de un sólo vértice.

A) 6 B) 37 C) 40 D) 17 E) 43

98) Hallar el número de lados de un polígono regular, sabiendo que la longitud de cada lado
es 3 cm., y el número de diagonales es 2 veces el perímetro en cm.

A) 6 B) 9 C) 12 D) 15 E) 18

99) En un polígono de 32 lados, convexo, las medidas de 30 ángulos consecutivos suman


5212°.

1038
Hallar la medida del menor ángulo que forman las bisectrices de los ángulos restantes.

A) 94° B) 47° C) 43° D) 86° E) N.A.

100) En un pentágono convexo ABCDE, Z A = Z E = 90° y Z B = Z C = Z D. Si:


BC + CD = 12cm. Hallar AE.

A) 6cm. B) 12cm. 0 ) 3 ^ 3 cm. 0 ) 6 }[ 3 cm. E)N.A.

101) En un romboide ABCD, ( AB < BC ), las mediatrices de BC y CD, se intersectan en un


punto R de la prolongación de BA. Hallar: m Z ADR, si m Z ABC = 52°.

A) 26 B) 38 C) 19 D) 24 E) 30

102) En un rombo ABCD, AB = 18^2 cm. La bisectriz del ángulo B intersecta a AD en el


punto F. Hallar la distancia entre los puntos medios de BD y FC-

A) Faltan datos. B) 18 cm. C) 9 cm. D) 1 8 / 2 cm. E) 9 / 2 cm.

103) En la figura: PM = MC
BC = 12cm.
AD = 28cm.
CD = 24cm.

Hallar MQ.

A) 8cm. B) 9cm

104) En un paralelogramo ABCD, AB = 12cm. y BC = 22cm. Las bisectrices de los ángulos


A y B, se cortan en el punto P. Las de C y D, en Q. Hallar PQ.

A) 12cm. B) 11cm. C) 8cm. D) 9cm. E) 10cm.

105) En la figura, O es punto medio de la mediana BM.A, B, C distan de r, 4; 22 y 12cm.


respectivamente.
Hallar OT.
B
A) 7cm.
B) 6cm.
C) 8cm.
D) 9cm.
E)N.A.

1039
106) En la figura:
ABCD, romboide. B C
CM = MD, BR = 12cm.
y RM = 2cm.

Hallar: AR

A) 14cm. B) l6cm. C) 15cm.


D) 17cm. E)N.A.

107) En un triángulo ABC, M es punto medio de AB: Se traza MH perpendicular a AC (H en


AC). Si Q es punto medio de M H , y R es un punto de BC, tal que Q R / / A C .
Hallar QR, sabiendo que AH = 3cm. y HC = 11cm.

A) 10cm. B) 9cm. C) 8cm. D) 9,5cm. E)N.A.

108) En un trapezoide ABCD, hallar la medida del menor ángulo que forman las diagonales,
si: m Z ABD = 50°, m Z DBC = 65°, m Z ADB = 70° y m Z BDC = 55°.

A) 90° B) 80° C) 85° D) 75° E) N.A.


109) En un rombo ABCD, M es punto medio de BC. AM y BD se intersectan en el punto R.
Hallar BD, si RM = 2 y m Z BRM = 45°.

A) 6 / 2 B)10/2 C) 1 2 / 2 D) 1 8 / 2 E) 1 4 / 2

110) ABCD, romboide: A R / / B D ; RM = MC, BM = 1 2 y M D = 5.


Hallar: AR. B

A) 8,5
B) 7
C) 7,5
D) 9
E) 10

111) En la figura, ABCD es un cuadrado.


AN = MC = NC = B C / 2 ,m ¿M CB = 17°.
Hallar: m, Z MN A .
A) 14°
B) 28°
C) 17°
D) 34°
E) 20°

N
1040
112) En la figura, I es incentro del triángulo ABC.
m Z ADB = 111° B

m Z AHB = 93°.
Hallar: m Z C.

A) 66e B) 76° C) 86
D) 74 E)N.A.

13) En un trapecio ABCD, A B / / D C , BC = 15cm.


CD = 18cm, m Z BAD = 45° y m Z BCD = 127°.
Hallar la longitud de A B .

A) 13cm. B) 39cm. C) 26cm. D) 42cm E) 52cm

114) En el trapezoide ABCD


C

AB 1 A D , AB = AD, m Z C = 45° y BC = 10cm

Hallar la distancia de A a BC.

A)10cm. B) 5cm. C) 20cm.

D) 5 / 2 cm. E) 8cm.
E
A D
115) Hallar la longitud del diámetro de una semicircunferencia, sabiendo que las distancias de
sus extremos a una recta tangente son 5 y 9cm. respectivamente.

A) 7cm. B) 12cm. C) 15cm. D) 14cm. E) 13cm.

116) En el interior de una circunferencia de radio 4cm., se tienen dos circunferencias menores
tangentes entre sí, exteriormente y tangentes a la mayor.
Hallar el perímetro del triángulo formado al unir los centros de las tres circunferencias.

A) 4cm. B) 6cm. C) 8cm. D) 12cm. E) N.A.

117) En la figura: m MF = 90
O, centro.
MH = 12 y EF = 5.
Hallar MF.

A) 1 3 / 2
B) 13
C) 18
D) 17 E B

E) 8,5
1041
118) En la figura, A y B son puntos de tangencia. O, centro. mCB = 31°. Hallar: mAB

A) 59°
B) 93°
C) 62°
D) 149°
E) 121°

119) En la figura, D es punto de tangencia, m Z AED = 110° y mAC = 78°.


Hallar: m Z BDR.
C
A) 39c
B) 31'
C) 55
D) 35
E) 29*

120) En la figura: AB y AC son diámetros. CT, tangente. mAE' = 72°. Hallar: mFC

A) 72c
B) 36'
C) 45
D) 54
E) 60‘

121) En la figura: BC = 12, AD = 7 y CD = 10. Hallar el radio de la circunferencia inscrita


en el triángulo AEO, si AO = 5.
A) 2
B) 1,5
C) 2,5
D) 3
E)N.A.

122) En la figura: A, B y C son puntos de


tangencia. mAC = 122° y mBC = 123
Hallar: mAB.

A) 124 B) 121' C) 115


D) 65° E)N.A.
1042
123) Se tienen dos circunferencias de radios 1 y 3cm. tangentes exteriores.
Hallar la medida del ángulo que forman una de las tangentes exteriores comunes y la recta
que une los centros.

A) 30 B) 45 C) 15 D) 60 E) 37

124) EnuntriánguloABC, m Z A = 62° y m Z C = 5 2 °.S iL y O, son el ortocentro y circuncentro,


respectivamente, del triángulo ABC. Hallar: m Z OBL.

A) 10 B) 12 C) 5 D) 20 E)N.A.

125) Hallar el valor de x, si A, B y C son puntos de tangencia.

A) 60°
B) 45°
C) 36°
D) 72°
E) 75°

126) Las circunferencias de la fig. son ortogonales entre sí. PÁ y PB, tangentes.
mÁBC = 104°
mBFC = 112°.
Hallar el valor de x

A) 36°
P B) 72°
C) 45°
D) 54°
E)N.A.

127) En la figura, A, B, C, D, T, son puntos de tangencia


Si: mAC + mBD = 224°.
Hallar: mCT + mTD.

A) 224°
B) 180°
C) 192°
D) 164°
E) N A

1043
128) En la figura: AB = 17 y AC = 7.

Hallar MN.

A) 10
B) 5
C) 8
D )7
E) N.A.
A C

129) En la figura: AB = BC; CD i DB, AM = MC y m z ABC = 32*.


Hallar: x.

A) 58°
B) 64°
C) 74*
D) 60*
E) N.A.

A , B
130) En la figura: O es centro mAF = 30°. A
Hallar EB.

A) 8
B) 4
C) 2
D) 2 / 2
E) 4 / 2

131) En un circunferencia de radio r = 4cm., se inscribe un triángulo ABC, de modo que


m Z BAC = 23° y m Z BCA = 22°. Hallar la longitud del lado AC.

A)2cm . B) 4cm. C) 2<J~2 cm. D )4 ^ c m . E) N.A.

132) Hallar r, si AB = 5 y BC = 12.

A) 17
B) 12
C) 11
D) 13
E)N.A.

1044
133) En la fig. E, T, puntos de tangencia
B
mTL = 40°, m Z A = 70°.

Hallar: mEN.

A) 20c B) 30 C) 35
D) 25 E) 40

134) Hallar “x”, si O y P, son centros.

O, punto de tangencia.

A) 90°
B) 75°
C) 72,5°
D) 82,5°
E)N.A.

135) En la fig.: FA = AG. Hallar el valor de x.

A) 20°
B) 18°
C) 12°
D) 15°
E) 10°

136) ¿Cuáles son correctas:

I.- Un trapecio escaleno puede ser inscriptible.


II.- Un trapecio isósceles es inscriptible.
III.- Todos los cuadriláteros simétricos son inscriptibles.
A) Sólo I B) Sólo II C) Sólo III D) 1 y II E) I y III.

137) Se tiene dos circunferencias que tienen únicamente dos tangentes comunes, entonces:

A) son congruentes. B) son tangentes interiores.


C) son tangentes exteriores. D) son secantes.
E) son exteriores.

138) En la figura: O ÁP = 3 0 °: P Á B = 40°.


Calcularla mCD si CDAOPB inscriptible. (O, centro).
1045
A) 10°
B) 20°
C) 30°
D) 40°
E) 50°

139) En un triángulo ABC, recto en B, se traza la altura B H , luego la bisectriz interior BF del
triángulo ABH, si: AB = 6m y AF = 1m. Calcular el inradio del triángulo ABC.

A) 3 m. B) 2,5 m. C) 2 m. D )1 ,5 m . E) 1 m.

140) Untriángulo ABC está inscrito en una circunferencia. La bisectriz exteriordel ángulo B corta
en F al arco BC, mientras que la bisectriz del ángulo BAC corta en G al arco FC.
¿Cuánto mide el arco FG, s i : C = 50o?

A) 75° B) 25° C) 30° D) 45° E) 50°

141) Hallar: a + p + 0

A ) 100°
B) 200°
C ) 140°
D) 240°
E) 250°

142) En un A ABC, AB = 5, BC = 6, AC = 7. E e AB , F e BC y EF es tangente a la circunfe­


rencia inscrita. Hallar el perímetro del A EBF.

A) 6 B) 4 C) 7 D) 5 E) 8

PU N TO S N O T A B L E S D EL T R IA N G U LO
143) Del gráfico, hallar el valor de V .

A) 26°
B) 28°
C) 27°
D) 30°
E)N.A.

1046
144) En un triángulo ABC, de baricentro G, sobre la prolongación de la mediana BM se toma
el punto E, de modo que ME = MG. Hallar la distancia de G a AE, si la suma de, B y M
a AE, es 24 cm.

A) 8cm B) 9,6cm C) 9cm D) 9,5cm E) 12cm

145) En un triángulo ABC, AB = BC, la altura BH corta a la mediana AM en "R"


Si RM = 2 y BRM = 45°. Hallar RB.

A) 2 ij~2 B) 4 C )3 /2 E )2 /6

146) En la figura:
AH = HC

O Á C = 30°

O ÁB = 40°

OCA = 50°.

Hallar la medida del OBC


A) 20 B) 15 C) 10 D) 5 E)N.A.

147) En un A ABC, obtuso en “B”, de circuncentro “O". BÁ C = 15° y O Á C = 25


Hallar la medida del ACB.

A) 40 B) 50 C) 60 D) 30 E) 35

148) Hallar el valor de V , en la figura adjunta

A) 15°
B) 30°
C) 20°
D) 18°
E) 12°

149) A ABC, T - » incentro. Hallar "x”


B
A) 44°
B) 35'
C) 36
D) 45
E) 60’

1047
150) A ABC, "O” circuncentro.
Hallar V .
B
A) 40e
B) 50'
C) 30
D) 60
C
E) 45’

151) A A B C : O -> circuncentro. Hallar V .

B A) 60e
B) 70'
C) 80
D) 90
E) 75'
C

152) AABC : E excentro. Hallar " 9 ”.

B
A) 70°
B) 80°
C) 90° C
D) 75°
E) 65°

153) AABC : E -> excentro. Hallar “x”

A) 100c
B) 105’
C) 110
D) 115
E) 120*

154) Dado el triángulo ABC, recto en B, se dibujan exteriormente-los triángulos equiláteros ABD
y BCE.
— ■ — <—^
Las prolongaciones de DA y EC se cortan en UP”. PB corta a DE en "J".
Hallar la medida del ángulo BJD.

A) 90 B) 60 C) 45 D) 75 E) 30
1048
155) ABCD, es un cuadrilátero convexo. Hallar la medida delmenor ángulo formado por
ÁC y BD, si: ABD = 5 0 °, DBC =6 5 °, ADB = 70° y BDC = 55°.

A) 70° B) 80° C) 75° D) 90° E)85°

156) En un triángulo ABC, AB = BC y B = 120°; se ubican “O", T , “E”, circuncentro, incentro,


y excentro relativo a BC, respectivamente. Hallar la medida del ángulo IEO:

A) 25° B) 30° C) 45° D) 20° E)35°


B
157) Hallar el valor de “a ”, si:
OC = BC ; O ÁB = O ÁC = 3a.

A) T B) 6o C) 7,5°
D) 9o E) 8o

158) Hallar el valor de “a" en la figura:

A) 12°
B) 15°
C) 18°
D) 20°
E) 16°

159) La hipotenusa de un triángulo mide 8cm. Hallar la distancia entre los excentros relativos
a los catetos.

A)8cm . B) 16<J~2 cm. C) 1 6 ^ cm. D ) 8 ^ 2 cm. E)N.A.

160) En un A ABC, acutángulo, “L” es ortocentro. La distancia entre los circuncentros de los
triángulos ABC y AOC, es 15cm. Hallar LB.

A) 10cm. B) 15cm. C) 30cm. D) 25cm. E)N.A.

161) E nun AABC, m Z A = 45° y la recta de Euler es paralela al lado BC.


Hallar la longitud del circunradio si la altura AH = 12cm.

A )4 ^ c m B )4cm . C) 6cm. D) 8cm. E)N.A.

162) En un A ABC, acutángulo, la distancia del circuncentro al lado AC, es 6cm.


Hallar la distancia del circuncentro de su triángulo complementario al segmento que une
los puntos medios de AB y BC.

A) 3cm. B) 6cm. C) 1,5cm. D) 2cm. E) N.A.


1049
LIN EAS P R O P O R C IO N A LE S Y S E M E JA N Z A DE T R IA N G U LO S

163) En un triángulo ABC, AB = 8, BC = 6 y AC = 7; se traza la bisectriz interior BD.


Hallar AD - DC.

A) 2 B)0,5 C) 1 D) 1,5 E) 0,75

164) En un trapezoide ABCD, las bisectrices de los ángulos B y D, se cortan en un punto de


la diagonal AC. Si AB = 6, BC = 8 y CD = 12. Hallar AD.

A) 9 B)10 C) 15 D) 7 E) 11

165) En un AABC, AB = 12, se traza la mediana BM. Si: m Z MBC = m Z A + m Z C .


Hallar BM.

A) 4 B) 5 C) 8 D) 12 E) 6

166) En un A ABC, se trazan las bisectrices, interior BD y exterior B F .S i AD = 10 y DC = 6.


Hallar CF.

A) 16 B) 32 C) 45 D) 48 E) 36

167) En la figura, B y T son puntos de tangencia. Los radios de las semicircunferencias, miden
3 y 4. Hallar DT, si AT = 6 .

A) 3
B) 2,8
C) 3,6
D) 4
E) N.A. A B

168) En la figura: M N //B C . AN = NQ, AM = 28, MB = 15 y MP = 18. Hallar PQ.


B A) 9,6
B) 12,8
C) 7,2
D) 6,4
E) N.A.

169) En la figura, T es punto de tangencia


AB = 3, AT = 5 y TC = 6.
Hallar CD.

A) 2 B) 3,6 T
C) 2,8 D) 2,4
E)N.A.
1050
170) En la figura:
B
AD = DC ,
BC = 2 A B ,
BE = 8 y EF = 3.

Hallar FD.

A) 5 B) 6 C) 6,6
D) 7,2 E) 5,4

171) En un A A B C .A B = BC ; = ÜL Si la longitud del inradio, es 9cm, hallar la distancia del


AC 3
Incentro al Baricentro.

A)2cm . B) 3cm. C) 4cm. D) 5cm. . E) 1cm.

172) En un A ABC, la circunferencia inscrita es tangente a AB en E, BC en F y AC en G.


Las prolongaciones de AC y EF se cortan en H. Si AG = 13 y GC = 8. Hallar CH.

A) 16,8 B) 33,6 C) 24,6 D) 36 E) 28,8

173) El segmento AB mide 23cm, el segmento AM 15cm. ¿Cuánto mide el segmento AN,
siendo N el punto conjugado armónico de M con relación a AB?

A) 33,00 cm. B) 49,28 cm.


C) 45,26 cm. D) 33,95 cm. a T¡ n m

E) Ninguno de los valores indicados.

174) Hallar el valor de x, en:

A) 2,5
B) 2
C) 3
D) 2,75
E)N.A.

175) En un AABC, AB = BC = 10 y AC = 8; la circunferencia inscrita es tangente a AB en


E y BC en F. Hallar EF.

A) 4 B) 4,2 C) 4,5 D) 4,8 E) 5


1051
176) En la figura, G es Baricentro del
A ABC. BE = 10.EC = 2 y AC = 15.
Hallar CH.
B
A) 4
B) 5
C) 6
D) 7,5
E )7

177) En la figura: EM = MF

AB = 12, BF = 8 y BM = 3
Hallar MC.

A) 18
B) 15
C) 16
D) 19
E)N.A.
C

178) ABCD, es un trapecio recto en A y B, BC = 4 y AD = 9. Si M, es punto medio de


AB y CM 1 MD, hallar AB.

A) 6 B) 9 C) 8 D) 12 E) 16

179) En un triángulo ABC.AB = 12 y AC = 8. Se traza la bisectriz interior AF y luego F R //A C


(R en AB). Hallar RF.

A) 4 B) 6 C) 4,8 D) 5 E) 5,6

180) En un A ABC , AB = BC, la mediatriz de BC, corta a AC en el punto R. Luego, se traza


R F //B C . (F en AB). Si FB = 8 y RC = 10. Hallar AF.

A) 12,5 B) 9,5 C) 11,5 D) 10,5 E) N.A.

181) En un A ABC, AB = BC, N es punto medio de AC y R un punto de AB, tal que AR = 2 RB.
Hallar AC, si m Z ANR = 30° y RN = 2

A) 4 B) 6 C )6 ^ 3 D) 8 E )8 /3

182) En un A ABC, B C - AB = 72. G, es el Baricentro e I, Incentro . Si Gl / / AC. Hallar Gl


A) 12 B) 9 C) 8 D) 18 E)N.A.
1052
183) En un A A B C .m / A = 2(m Z C), se traza la bisectriz interior AF. Si BF = 4 y FC = 5:
hallar la distancia de F al excentro del A ABC, relativo a BC-

A) 4,5 B) 8 C) 6 D) 5 E) 10

184) En un triángulo ABC, P y Q son puntos de AB y BC, respectivamente,de modo que


PQ //A C . Hallar la longitud PQ, si el triángulo PBQ y el trapecio APQC, tienen igual
perímetro; siendo; AB = 12, BC = 8 y AC = 10.

A) 5 B) 6,75 C) 8 D) 3,75 E) N.A.

A
185) En un triángulo ABC.AB = 4, BC = 6 y B = 120°. ¿Cuánto mide la bisectriz interior BD?

A) 5 B) 3 C) 2,8 D) 2,4 E) 2,1

186) En la figura: O, es centro.


AP = 10, AB = 12 y PB = 4.
Hallar PC.

A) 4
B) 5
C) 6
D) 4,5
E) N.A.

187) La tangente trazada por el vértice B, a la circunferencia circunscrita al triángulo ABC, corta
a la prolongación de CA en el punto P. Si: AB = 6, BC = 8 y AC = 7, hallar PB.

A) 10 B) 10,5 C) 11 D) 11,5 E) 12

188) En un triángulo ABC, la bisectriz del ángulo externo B, corta en "E" el arco AB d é la
circunferencia. EB y AC prolongados se cortan en “D”. Si:AE = 13 y EB = 5, hallar BD.

A) 18 B) 27,5 C) 28,8 D) 16 E) 20

189) Los lados de un triángulo ABC, miden AB = 8, BC = 10 y AC = 12.


Hallar la longitud del segmento EF, paralelo a AC, que pasa por el incentro.

A) 9,2 B) 6 C) 7,2 D) 8,2 E)N.A.

190) En la figura: B C //A D y C es punto de tangencia.

Si: AB = 6 y BC = 9, hallar ED.


1053
A) 3
B) 6
C) 5
D) 4
E) N A

R E LA C IO N E S M E TR IC A S EN T R IA N G U LO S R E C T A N G U LO S

191) En una circunferencia, cuyo radio se quiere conocer, una cuerda de 2dm. tiene una flecha
de 2cm. Dicho radio, mide:

A) 18cm. B) 26cm. C) 20cm. D) 24cm. E) 28cm.

192) ABCD, es un trapecio inscrito en una circunferencia de diámetro AD = 34cm. Hallar BC,
si la altura del trapecio es 15cm.

A) 17cm. B) 16cm. C) 14cm. D) 28cm. E) 18cm.

193) En un trapecio isósceles ABCD, B C //A D , A C J _ C D .B C = 7 y AC = 20. Hallar AD.

A) 25 B) 26 C) 30 D) 21 E) 24

194) El segmento tangente común a dos circunferencias ortogonales, de radios 12 y 18cm.


mide:

A) 1 2 ^ 6 cm. B) cm. C) cm. D) 14cm. E)N.A.

195) Hallar la longitud de la cuerda común a dos circunferencias ortogonales, de radios 6 y 8cm.

A)9,6cm . B) 4,8cm, C) 10cm. D) 7cm. E)N.A.

196) En un triángulo ABC, recto en B, se trazan la altura BH y la ceviana AE.


Si AE = EC = HC = 1. Hallar BC.

A) 2 B )/2 C ) 3/ 2 D )/^ E) 3

197) ABCD, es un rectángulo. Se traza AH J_ BD ( H en BD).


Hallar BD, si H dista 2 y 6^/~3 unidades.de BC y CD, respectivamente.

A) 8 B) 12 C) 16 D) 18 E) 20

1054
198) PA y PB, son tangentes a una circunferencia de radio V , de modo que PA i. PB.
Hallar r, si un punto del menor arco AB, dista 8 y 9cm., respectivamente, de PA y P B .

A)29cm . B) 17cm. C) 34cm. D) 25cm. E) 30cm.

199) Hallar la longitud de la altura de un trapecio,cuyas diagonales miden 6 y 8cm., siendo


perpendiculares entre sí.

A) 9,6cm. B) 2,4cm. C) 6,4cm. D) 4,8cm. E) 5cm.

200) En un A ABC, recto en B, se traza la bisectriz interior BF. Hallar A B .si AF = 7 y FC = 24.

A) 8,68 B) 9,28 C) 10,28 D) 9,88 E) N.A.

201) Se tiene un A A B C .rectoen B: AB = 6 y B.C = 8. Exteriormente, con AB y BC de diáme­


tros, se dibujan dos semicircunferencias, respectivamente. La longitud del segmento
común a estos arcos, mide:

A) 4 / 3 B )8 /3 C )/2 6 D) 3 / 3 E) 2 / 7

202) En un A ABC, recto en B, la mediatriz de AC corta a BC en el punto F. Si BF = 3 y


FC = 5. Hallar AB.

A) 6 B) 4 C) 8 D) 3 E) N.A.

203) En un A ABC, la altura CH corta a la altura BQ en el punto R. Si HR = RC y AB = 6.


Hallar AC, sabiendo además que AC = BC.

A) 6 B) 9 C )3 /2 D )3 /6 E) 3 ^ 3

i
204) En un A ABC, recto en B, las medianas BM y AF se cortan en el punto H. Si BM 1 AF,

hallar: —
BC

1 ^ J2 13
A )- B) 2 C )/2 D )^ - E )^ -

205) La suma de los cuadrados de las medianas de un triángulo rectángulo, es 96cm2. La


longitud de la hipotenusa, es:

A)4cm. B) 6cm. C) 9cm. D) 8cm. E) 10cm.

206) Sobre una circunferencia de diámetro AB, se toma el punto C, el cual va a ser centro de
otra circunferencia tangente a AB en el punto H. Si: AH = 18 y HB = 8; hallarla longitud
1055
del segmento tangente común a ambas circunferencias.

A) 2 / 4 2 B )/4 2 C) 2 / 4 1 D) 3 / 4 2 E) 2 / 4 0

R E LA C IO N E S M E T R IC A S EN T R IA N G U LO S
O B L IC U A N G U L O S

207) Hallar el perímetro de un trapecio isósceles ABC acutángulo, sabiendo que la proyección
de AB sobre AC, mide 16cm y la proyección de AC sobre AB, 20cm. Además AC = BC.

A) 100cm. B) 120cm. C) 130cm. D) 150cm. E) 140cm.

208) Interiormente a un cuadrado ABCD, tomando como diámetro AB, se dibuja una
semicircunferencia y sobre ella se toma el punto E. Si AE = 12 y BE = 7. Hallar ED.

A) 19 B) 15 C) 13 D) 14 E) 16

209) La circunferencia inscrita en un cuadrado ABCD, es tangente a AB en M y a BC en N

Sobre el arco MN, se toma un punto F. Hallar AB, si AF + FC = K .

A) K / 6 B) K 4 r O K ^ r D ) K ^ r E )N .A .

210) OA y OB, son radios de un cuarto de circunferencia AB,donde UA = 3cm. Haciendo centro
en B, y con radio 4cm. se traza un arco que corta a OA en E y a AB en F. Hallar el radio
de la circunfernecia inscrita en el triángulo mixtilíneo AEF.

A)0,2cm . B)0,1cm . C) 0,25cm. D) 0,5cm. E) 0,3cm.

211) En un AABC, la bisectriz interior AF corta a la ceviana BE en el punto P. Hallar AP, si


AB = 6 , BF = 3 , AE = 4 y ABP = C.

A) 2 /3 B) 2 / 6 C )3 /2 D) 4 E) 5

212) Hallar la longitud de la altura de un trapecio cuyas bases miden 4 y 10cm., siendo las
longitudes de las diagonales 13 y 15cm.

A) 11cm. B) 10cm. C) 13cm. D) 9cm. E) 12cm.

213) En un A ABC, el ángulo A es agudo y AB = 3 , AC = 5. El máximo valor entero de BC,


es:
A) 7 B) 5 C) 3 D) 6 E)N .A.

1056
214) En un A ABC obtuso en B, AB = 8 y BC = 9; hallar el mínimo valor entero de AC.
A) 2 B) 12 C) 13 D) 14 E) 15

215) Las longitudes de los lados de un triángulo obtusángulo, son tres números enteros y
consecutivos. El perímetro del triángulo, es:

A) 6 B) 12 C) 15 D) 21 E) 9

216) En un A ABC, se cumple: a2 = b2 + c2 + ^[3 be. El ángulo A, mide:

A) 120° B) 150° C) 135° D) 165° E) 60

217) ABCD, es un trapecio circunscrito a una circunferencia. ( BC l AD ). Si:

m BA D + m ADC = 9 0 °, BC = 4 y AD = 9; hallar AC + BD .

A) 97 B) 25 C) 125 D) 96 E) 89

218) En un trapecio ABCD, BC I AD; AB = 5 , BC = 6 , CD = 7 y AD = 11. La proyección de


AB sobre A D .e s:

A) 1 B) 0,1 C) 0,2 D) 0,125 E) 0,375

219) En un trapecio ABCD , BC AD; hallar la longitud de la diagonal AC , si:


AB = 15 , BC = 4 , CD = 13 y AD = 18.

A) 17 B) 16 C) 18 D) 15 E) 20

220) La suma de los cuadrados de las longitudes de los lados de un triángulo, es 24cm2. Hallar
la suma de los cuadrados de las longitudes de las medianas.

A) 18cm2. B) 24cm2. C) 30cm2. D) 16cm2. E) 15cm2.

221) Se tienen tres circunferencias tangentes exteriores entre sí, dos a dos, de radios 6 ,7 y 8cm.,
respectivamente. Hallar la distancia del centro de la que tiene longitud intermedia de radio,
a la recta que une los centros de las otras dos.

A) 12cm. B) 9cm. C) 11cm. D) 10cm. E)N.A.

222) En un A ABC, se traza la altura AH, la mediana AM y la bisectriz interior AD.


Si A B - A C = 12 y MD = 4 ; hallar MH.

A) 8 B) 9 C) 7 D) 6 E) 10

223) ¿De qué naturaleza es el triángulo cuyos lados miden 6, 9 y 11cm?

A) Rectángulo B) Acutángulo C) Obtusángulo D) No se sabe.


1057
224) Un triángulo cuyos lados miden 10, 14 y 17m. es:

A) Acutángulo B) Obtusángulo C) Rectángulo D) No se sabe.

225) En un A ABC, AB = 6 , BC = 10 y AC = 14. Hallar la longitud de la proyección de AB


sobre la recta que contiene a B C .

A) 1,5 B) 2 C) 2,5 D) 3 E) 3,5

226) Los lados de un triángulo, miden 39, 40 y 25cm. respectivamente. La menor altura, mide:

A) 37,44cm. B) 24cm. C) 23,4cm. D) 22cm. E) 20,2cm.

227) En un A ABC, m Z A = 6 0 °, AB = 16 y AC = 10. Hallar BC.

A) 7 B) 12 C) 13 D) 15 E) 14

228) En un A ABC, hallar la medida del ángulo C, si: AB = 14 , BC = 10 y AC = 6.

A) 60° B) 30° C) 150° D) 120° E) 135°

229) Hallar la longitud de la cuerda común a dos circunferencias secantes, de radios 15cm. y
41cm.; sabiendo que la distancia entre sus centros es 52cm.

A)9cm . B) 18cm. C) 16cm. D) 24cm. E) 14cm.

230) En un A ABC, BC = 12 y AB2 - AC 2 = 264. Hallar la proyección de AC sobre BC.

A) 4 B) 6 C) 5 D) 4,5 E) 7

231) En el problema 230, hallar la proyección de AB sobre BC.

A) 7 B) 12 C) 13 D) 15 E) 17

232) En un A ABC, AB = 17 , BC = 24 y AC = 13. Se traza la ceviana interior AM, tal que


BM = 2MC. Hallar AM.

A) 9 B) 10 C) 11 D) 12 E) 8

233) Los lados de un triángulo, miden 8; 26 y 30cm. La mayor altura, mide:

A)9,6cm . B) 12,8cm. C) 24cm. D) 18cm. E) 26cm.

234) Hallar la longitud de la menor mediana de un triángulo cuyos lados miden 16; 18 y 22m.

A) 17m. B) 12m. C) 14m. D) 13m. E) 15m.


1058
235) En un trapecio: ABCD, BC AD ;AB = 21 ,BC = 1 2 ,CD = 13 y AD = 26. Hallarla distancia
entre los puntos medios de BC y AD.

A) 16 B) 14 C) 15 D) 7 E) 13

236) Enun A A B C .A B = 12 , BC = 18 y AC = 15; la bisectriz interior AF, mide:

A) 5 B) 11 C) 9 D) 10 E) 8

237) La suma de los cuadrados de las distancias de los vértices de un triángulo, al Baricentro,
es 27m2. Hallar la suma de los cuadrados de los tres lados.

A) 54m2 B) 36m2 C) 63m2 D) 56m2 E) 81 m2

238) Se tiene semicircunferencia de diámetro AC. Sobre AC se toma el punto B, de modo que
AB = 2R y BC = 2r. Tomando como diámetro AB y BC, se dibujan interiormente otras
dos sem icircunferencias. D em ostrar que la circu n fe re n cia tangente a las tres
R r( R + r )
semicircunferencias anteriores, tiene radio : x = - 2 2
R + r + Rr

239) Enun trapecio ABCD, AB = 5; BC | AD ; BC = 6 , CD = 7 y AD = 10. Hallar: AC + BD

A) 210 B) 196 C) 194 D) 190 E) 184

R E LA C IO N E S M E TR IC A S EN L A C IR C U N F E R E N C IA
Y P O TE N C IA

240) ABCD, es un paralelogramo con el ángulo B obtuso. La circunferencia circunscrita al


A ACD, corta en F a la prolongación de D B .S i FB = BD y AC = 6; hallar BD.

A) 6 B) 12 C )3 /3 D )2 /3 E )4 /3

241) C, y C2 , son dos circunferencias tangentes interiormente en el punto T. (C, < C2). La
prolongación de la cuerda TF d e C 1 tcortaa C2 en el punto H. Por H, se traza una cuerda
HP de C2 , que es tangente a C, en J. FJ prolongada en ambos sentidos corta al arco
HT en A y al arco HP en B. Si AF = FJ = JB y HF = 2FT; hallar JP, sabiendo que
HJ = 12cm.

A)6cm. B) 3cm. C) 9cm. D) 8cm. E) 4cm.


1059
242) C } y C2, son dos circunferencias tangentes exteriormente en el punto T. (C, > C2). La
prolongación de la cuerda A B ,d e C , , es tangente en D a C2 y la tangente interior común
a ambas circunferencias, corta a BD en el punto E. Hallar ED, si AB = 8 y BD = 4.

A) 2 B) 3 C) 4 D) 5 E) 6

243) En la figura, T es punto de tangencia. Hallar CT, si AB = 8 y BC = 1.

A) 4,5
B) 2
C) 3
D) 1
E) N.A.

244) M, es punto medio del lado AC de un A ABC. Por B y M, pasa una circunferencia que corta
a AB en E y a BC en F. Si AE = 3 , EB = 33 y FC = 4 , hallar BF.

A) 27 B) 26 C) 25 D) 24 E) 23

245) Sobre la cuerda A B , de una circunferencia de centro O, se toma el punto F, de modo que
AF = 18 y FB = 8. Sea C, un punto del arco AB, tal que FC 1 OF. Hallar FC.

A) 6 B) 14 C) 12 D) 16 E)N.A.

246) AO y AB, son diámetros de dos semicircunferencias tangentes en A y AO = OB. La cuer­


da BF de la mayor es tangente a la otra en el punto H. Hallar AH, si HF = 4.

A )4 ^ 2 B )4 ^ C )4 /? D) 4 / ? E) 8

247) Desde un punto D, exterior a una circunferencia, se trazan las tangentes DT y DQ. Luego
se traza la secante DCA, que corta a la cuerda TQ en el punto B. Si AB = 1 y BD = 8;
hallar BC.

A) 1 B) 0,5 C)0,1 D) 0,25 E) 0,75

248) Desde un punto C, exterior a una circunferencia, se trazan las secantes CPA y CQB.
Por P, Q y C, pasa una circunferencia “O”. Luego se trazan AF y BE, tangentes a “O".
Si AF = 15 , AC = 24 y BC = 20, hallar BE.

Á) 30 B) 8 C) 12 D) 7 E) 10

1060
249) C, y C2, son dos circunferencias secantes en los puntos A y B. La prolongación de la cuerda
HB, de C v corta a C2 además en F. Por H, se traza HT .tangente a C2 y por F, FQ tangente
a C j. Hallar FQ, si HT = 12 y H B = 9.

A) 14 B )2 /7 0 )2 /1 4 D) 4 / 3 E) 4 / 7

250) A, B y Q, son tres puntos colineales y consecutivos. Por AB y BQ, pasan dos circunfe­
rencias Cj y C2, respectivamente. Luego se trazan: AT, tangente a C2 y QF tangente
a C y Hallar AQ, si QF = 8 y AT = 15.

A) 16 B) 19 C) 18 D) 20 E) 17

251) En una circunferencia de centro O y radio 5cm., se tiene un punto A. Hallar la potencia de
A, respecto al punto O.

A) -25cm 2. B) 25cm2 C) 5cm. D) -5cm E) CERO

252) En un AABC, AB = 6 , BC = 7 y AC = 9; hallar la potencia del vértice B, respecto al centro


de la circunferencia inscrita en ABC.

A) 2 B) 4 C) 8 D) 6 E) N.A.

253) En un triángulo isósceles, la base mide 14cm. y uno de los lados congruentes, 25cm. Hallar
la potencia del Baricentro de dicho triángulo, respecto al Incentro.

A) CERO B) 20cm2 C) -20cm2 D) 16cm2

254) En un triángulo, cuyos lados miden 8 ,1 5 y 17cm., hallar la potencia del Incentro, respecto
al circuncentro.

A) -50cm 2. B) -49cm 2. C) -61cm 2 D) -41cm 2. E) -51cm 2

255) Para el problema anterior ( 254 ), la potencia del circuncentro, respecto al Incentro, es:

A) 51 cm2 B) 12,25cm2 C) 12,75cm2 D) CERO E) -51 cm2

1061
P O LIG O N O S R E G U LA R E S Y LO N G ITU D
DE L A C IR C U N F E R E N C IA

256) El lado del dodecágono regular circunscrito a una circunferencia de radio r, tiene longitud

A )r^ 2 -/3 B) 2 ^ 2 - y j l C )r(2 -/3 ) D) 2 r / I E) 2r ( 2 - J~3 )

257) En una circunferencia de radio R = 1cm, se tienen los puntos consecutivos A, B, C, D, de


modo que AD = cm y las prolongaciones de AB y DC se corten en un punto J, tal

que m BJC = 15°. Hallar la longitud de BC.

A ) ^ 3 cm B) 1cm C ) / 2 cm D) ^ 2 - / 2 cm E) ^ 2 - / 3 cm

258) El cuadrilátero ABCD, está inscrito en una circunferencia de radio R.

AB = — ( - 1 ) y CD = R ^ 2 - ^J~3 . Hallar la medida del menor ángulo que forman

las diagonales.

A) 66° B) 6 C) 3 D) 33 E) N.A.

259) En un pentágono regular de lado “L", la longitud de una diagonal, es:

+ 1) C ) ^ ( /5 + l) □) L ( - 1) E)N.A.

260) En un A ABC, m Z A = 80°, m Z C = 55° y AC = 2cm , se trazan las alturas AQ y CH


Hallar QH.

A) tcm . B) V 2 cm C )/3 cm D) ^ 2 - / 2 cm E)N.A.

261) En un trapecio isósceles ABCD, BC 11 AD, BD = AD, m Z CAD = 22,5° y AD = L. Hallar


BC.

A )L ^ 2 + / 2 B )L L /2 + / 2 + 1 C) Ü V 2 + / 2 - 1 D )L ^ 2 - J~2 E) N.A.

262) En una circunferencia de radio R, hallar í 15, longitud del lado del polígono regular de 15
lados , inscrito.

R
^10 + / 2 0 - / T 5 + / 3 yj 10 + / 2 0 + y j l 5 - y[3
B> 4

C) ^ ( / 5 + / 10+ / 2 ) D )^(/5+l) E) N. A.

1002
263) En una circunferencia de radio / 6 cm, se tiene inscrito un cuadrilátero ABCD.

Si AB = 3 / 2 cm y BC = 2 / 3 cm.; hallar m CD, sabiendo que m AD = 42'


*

A) 84° B) 108° C) 102° D) 48° E) 138°

264) La longitud del lado del icoságono regular inscrito, en una circunferencia de radio R, es:

A )^ (^ 5 -1 ) B) f ( V 3 + ^ 5 - V 3 " ^ 5 ) C) f ( / S T / S - )

R
D )-(/Í0 + yf b) E)N.A.

265) En una circunferencia, están inscritos un triángulo equilátero y un cuadrado, cuyos

apotemas difieren ( - / 3 ) cm. Hallar el apotema del hexágono regular inscrito en la


misma circunferencia.

A)1cm . B) 2cm. C ) / 3 cm. D) 3cm. E) 2 / 3 cm.

266) ABCDE, es una pentágono regular. AC = 2cm ; BD corta a AC en elpunto H. Hallar BH

A) ( 3 - / * 5 ) cm. B)( / 5 - 1) cm. C) ( 2 - / ¥ ) cm. D) ( 5 - / " 5 ) cm. E) N.A.

267) La longitud del arco de un sector circular de 60°, es a. Hallar la longitud de la circunferencia
inscrita en dicho sector.
3a
A )a B) 2a C) 3a D) — E) N.A.

268) ABC, es un triángulo equilátero. Una semicircunferencia de diámetro AC, corta a AB en


M y BC en N. Con centro B, se traza un arco de circunferencia tangente a AC y que corta
AB en E y BC en F.
Hallar la longitud del arco EF, si el arco MN mide 6 cm.

A)6cm . B) 9cm. C )6 /3 c m . D )2 /3 c m . E)N.A.

269) ABCDEF, es unhexágono regular. Haciendo centro en A, setraza el arco CE, de longitud
12cm. Luego, haciendo centro en A y radio AD, se traza un arco que corta a la prolonga­
ción de AB en P y la prolongación de AE en Q. Hallar la longitud dei arco PQ.

A)24cm. B) 12 /3 c m . C) 1 2 /6 c m . D) 32cm. E) 36cm.

1063
270) ABC; es un triángulo rectángulo isósceles, recto en B. Con centro A y radio AC, se traza
un arco de circunferencia que corta en F a la prolongación de A B .
Luego, se dibuja exteriormente al AABC, una semicircunferencia de diámetro AF. Por A,
B y C, pasa un arco de circunferencia. Hallar el perímetro de la curva ABCFA.si AC = 8cm.

A)8ncm . B)97rcm. C) lOrccm D) 127i»cm. E)147tcm.

AREAS

271) Por el Baricentro de un triángulo, pasa una recta paralela a un lado, determinando un
trapecio que encierra una región de área 20cm2. El área de la región triangular original, es:

A)40cm2 B) 60cm2 C) 36cm2 D) 48cm2 E) 32cm2

272) En un paralelogramo ABCD, E es un punto de BC . AE , corta a BD en H. Si


área ABH = 24m2 y área ECD = 14m2. H a lla r: área ABCD.

A) 56m2 B) 78m2 C) 84m2 D )1 1 2 m 2 E) 96m2

273) ABC, es un triángulo recto en B. H, un punto de AC. Se trazan HE 1 AB y HF 1 BC.


Si: área AEH = 16m2 y área HFC = 25m2.Hallar área EBFH.

A) 20m2 B) 36m2 C) 30m2 D) 41 m2 E)40m2

274) En un A ABC, E es un punto de AB y F un punto de B C , talque EF 11 AB . AF y EC

se cortan en el punto O. Si: = — = J_


AE FC 2

Hallar: área AEBF


área A AOC

2 3 4 2 1
A )? B> 8 C ) 15 D> ? E )4

275) En un A ABC, AB > BC, se trazan las bisectrices BD y BE, interior y exterior, respec
tivamente. Si: área ABD = 17m2 y área BDC = 7m2 , hallar el área BCE.

A) 24m2 B) 16,8m2 C) 18,6m2 D) 19m2 E) 20m2

276) Hallar el área del octógono regular inscrito en la misma circunferencia circunscrita a un
dodecágono regular que encierra una región de área 9m2.

A) 8m2 B) 6m2 C) 6 ^ 2 m2 D) 6 / 3 m2 E)N.A.


1064
277) Hallar el área de un trapecio rectángulo, cuyas bases miden 4 y 13 m., sabiendo que una
diagonal es perpendicular a un lado.

A) 42m2 B) 51 m2 C)64m2 D) 36m2 E) Ninguna

278) Calcular el área de un decágono regular, cuyo apotema es el quíntuple del apotema de otro
decágono regular cuya superficie es de “L" m2.

A ) i m2 B) Lm2 C)-^ m 2 D) ¿ m 2 E) Z m2

279) Hallar el área de un rombo ABCD si “M"biseca a BC; AM corta BD en "R",

RM = 2u y BRM = 45°.

A) 12u2 B) 24u2 C)36u2 D) 48u2 E) 60u2

280) Si r, , r2 , r3 son los exradios de un triángulo donde r1 . r2 = 8m2 ,r2 . r3 = 6m 2 y


r1 . r3 = 4m2. Hallar el perímetro del triángulo.

A) 8m. B) 4m. C)10m. D) 6m. E) 6 / 2 m.

281) Hallar el área del trapecio rectángulo ABCD, si AD J_ DC; la base menor es DC = 4; el
lado no paralelo CB = 13 y la diagonal DB = 15.

A) 68 B) 78 C) 88 D) 98 E) N .A.

282) AB, es un cuarto de circunferencia de centro O. Con centro en B y radio BA se traza un


arco AF, estando F en la prolongación de BO.
Si: área AOF = 2m2, hallar el área del segmento circular AB.

A) 1m2 B) 2m2 C) 3m2 D) 4m2 E) N.A.

283) ABCDEF, es un hexágono regular de lado 6cm. Con centro en A, se traza un arco CE. Luego
con centro en D se traza otro arco de radio 6cm.
y
Hallar el área de la región que encierran dichos arcos.

A )(3 0 tc -3 6 /3 )cm2 B) ( 36n - 3 0 / 3 )cm2 C ) ( 2 0 tc - 2 4 /3 )cm2

D )( 12ti - 9 / 3 ) cm2 E) N.A.

284) ÁC, es diámetro de una semicircunferencia circunscrita al triángulo isósceles ABC. Con
centro en A y radio A C .setrazaunarco CF, estando F en la prolongación de AB. Luego.ex-
teriormente al A ABC, se dibuja otra semicircunferencia de diámetro A F . Hallar el área de
la región que encierran las curvas ABC.CF y FA, si la reglóntriangular ABC, tiene área 8m2

A) 8m2 B) 16m2 C) 87tm2 D) 4nm2 E) 167tm2

1065
285) Sobre el diámetro AC de un semicírculo se toma el punto B y se dibujan interiormente
dos semicircunferencias ÁB y BC. ( AB > BC ). Hallar el área de la región que encierran
AB , BC y AC, si el segmento tangente común a AB y B C ,m id e 8 cm.

A) 64rccm2 B) 247tcm2 C) 32rccm2 D) 167icm 2 E) 87tcm 2

* Nota.- Al final del capitulo “Areas de Regiones Planas”, hay más problemas propuestos.

RECTAS, P LA N O S , D IEDR O S, TR IE D R O S Y P O LIE D R O S

286) Indicar, verdadero ( V ) ó falso ( F ):

Dos rectas paralelas a un mismo plano son siempre paralelas entre sí.
Dos rectas que no se cortan son siempre paralelas.
Por un punto exterior a una recta, sólo puede pasar otra recta perpendicular a la primera.
Dos planos perpendiculares a una misma recta, son paralelos entre sí.

A)FFFF B) F W F C )W W D)FFFV E)N.A.

287) En el espacio se tiene 5 puntos no colíneales y 10 rectas paralelas. Calcular el número


máximo de planos que se pueden determinar.

A) 55 B) 95 C) 85 D) 105 E) 100

288) Indicar la proposición que no es incorrecta:


I) Dos rectas alabeadas determinan un plano.
II) La intersección de tres planos es siempre una recta.
III) Dos planos perpendiculares a un tercero, son paralelos entre sí.
IV) Dos rectas paralelas a un mismo plano, son siempre paralelas entre sí.

A) Sólo I B) Todas C) I , II y III D) Sólo IV E) Ninguna

289) Desde un punto P, exteriora un plano, se trazan las oblicuas PA y PB (A y B sobre dicho
plano), formando ángulos de 30° y 45°, respectivamente con el plano. SiPA = 6 ; hallar
PB.

A) 3 B )3 /2 C )3 /3 D) 4 E) 2 ^ 6

290) ABCD, es un tetraedro regular de arista “a", BH es una altura del sólido y M punto me­
dio de BC. Hallar la mínima distancia entre BH y DM.

A )i^ B)J L ^ c , | ^ d)| ^ E )f^

1066
291) Por el circuncentro “O” de un triángulo ABC, se levanta la perpendicular OL al plano
del triángulo. Si AC = 6 ; OC = 5 y AL = / 4 1 - Hallar el diedro AC, que determinan
los planos ABC y ALC.

A) 15° B) 30° C) 45° D) 60° E) 75°

292) En un AABC, B = 90°,AB = 6 y BC = 8 .M ,espuntom ediode AC. Se eleva BF, perpendicular


al plano ABC, tal que BF = / T í . Hallar la medida del ángulo con que se cruzan las rectas
AB y FM.

A) 45° B) 53° C) 37° D) 60° E) 30°

293) Se tiene dos segmentos AB = 12 y CD = 16, los cuales son ortogonales. Hallarla medida
del segmento que une los puntos medios de AC y BD.

A) 14 B) 11 C) 13 0)5/5 E) 10

294) La recta AB de intersección de dos planos perpendiculares PyQ es paralela a una recta
L, del plano P y a una recta L2 del plano Q.
Luego se ubican los puntos M y N de tal manera que M pertenece a L i , N pertenece
<->
a L2 .
Calcular la menor distancia entre las rectas MN y AB, si las distancias de M y N a la
recta AB miden 12 y 16cm., respectivamente.

A) 8,6 B) 4,6 C) 5,6 D) 9,6 E) 7,6

295) De las siguientes proposiciones, cuáles son verdaderas:


I.- Dadas dos rectas alabeadas, por una de ellas es siempre posible trazar un plano paralelo
a la otra.
II.- Por un punto cualquiera del espacio siempre se puede trazar un plano perpendicular a dos
planos dados.
III.- Por un punto cualquiera del espacio, es siempre posible trazar un plano perpendicular a
otro plano dado y paralelo, a la vez, a una recta dada.
IV - Por una recta exterior a un plano siempre se puede trazar un plano perpendicular al primero.

A) I y II B) I, II y IV C) I y III D) Ninguna E) Todas.

296) Si una recta a es perpendicular a las rectas b, c y d, entonces, es cierto que:


A) Las rectas b, c y d, necesariamente son paralelas.
B) Las rectas b, c y d, tienen que ser coplanares.
C) Las rectas b ,c y d, necesariamente se cortan.
D) Las rectas b, c y d, no necesariamente son paralelas.
E) Ninguna anterior.

1067
297) En un poliedro, el número de caras, más el número de vértices y más el número de aristas,
es 28. Si la suma de las caras de todos sus ángulos sólidos es 1 800°.
Hallar el número de caras.

A) 13 B) 7 C) 9 D) 10 E) 8

298) Se tiene un trapecio isósceles ABCD de bases BC y AD ( AB = BC = C D ) inscrito en


una semicircunferencia con diámetro AD; por “C" se levanta una perpendicular C F .a l
plano de la circunferencia. Calcular el ángulo que forma FD con el plano, si AF forma
con dicho plano 45°.

A) 45° B) 60° C) 75° D) 53° E) 30°

299) Calcular el número de vértices de un poliedro convexo que está formado por 6 pentágonos,
2 0 triángulos y 6 hexágonos.

A) 28 B) 42 C) 32 D) 33 E) 54

300) Si en un triedro, dos diedros miden 80° y 70° contestar verdadero V ó falso F.
El tercer diedro puede medir 150°.
El tercer diedro puede medir 35a.
El tercer diedro puede medir 160°.
El tercer diedro puede medir 175°.

A )W W B) W F F C) FVFF D) FFFF E )W V F

301) En una pirámide OABCD la base ABCD está inscrita en una circunferencia. Si los diedros
OA, OB y OC miden 100°, 85° y 95°. Hallar el diedro OD si la altura cae sobre el centro
de la circunferencia.

A) 110 ° B) 90° C) 165° D) 115° E) Ninguna.

302) A A \B B \C C ' y D D \ son aristas de un cubo. P, punto medio de AB; Q biseca AD y R


punto medio de DD'. Si AB = a, hallar la mínima distancia entre PQ y RC.

A aJ6 a Jl ^a/¡3 a JJ
) "4 B )-V C )-V D) — E) N.A.

303) Hallar el área de una región triangular, sabiendo que sus proyecciones sobre las caras de
un triedro trirectangulo, tienen áreas 3; 4 y 12m2, respectivamente.

\) 19m2 B) 12m 2

304) Una región triangular ABC, de área 36m2, se proyecta sobre un plano H, obteniéndose un
triángulo rectángulo Isósceles de hipotenusa 6 / 2 m. La medida del diedro que forman tos
1068
planos ABC y H, es:
A) 30° B) 45° C) 75° D) 15° E) 60°

PRISM A, P IR AM ID E, C ILIN D R O , C O N O Y E S F E R A

305) Hallar la distancia entre los baricentros de las bases de un tronco de prisma triangular de
aristas 9, 12 y 18.

A) 15 B) 16 C) 12 D) 14 E) 13

306) Calcular el volumen de un prisma en el cual el área de una cara lateral es 20m2 mientras
que la distancia desde la arista opuesta hasta dicha cara es 6m.

A) 120m3 B) 90m3 C) 60m3 D) 75m3 E) 80m3

307) En un tronco de prisma recto que tiene por sección recta un triángulo se inscribe una
pirámide que tiene por base la misma del tronco de prisma y por vértice el punto de
intersección de las medianas de la base superior del tronco. Calcular la relación de
volúmenes que existe entre los dos sólidos.
1 1 1 2 4
A) 3 O - D )- E)-

308) Calcular el volumen de una pirámide regular cuya base es un hexágono de 12m. de
perímetro, el apotema de la pirámide mide 4m.

A) 4 / 3 9 m3 B) 5 / 3 9 m3 C) 2 / 3 9 m3 D) 6 / 3 9 m3 E) 3 / 3 9 m

309) Hallar el volumen de un tronco de prisma recto cuya base es un triángulo isósceles ABC
de lados:
AB = AC = / T Í 6 m y BC = 8m
las aristas laterales están en progresión aritmética, siendo la intermedia 3.

A) 100m3 B) 120m3 C) 110m3 D) 140m3 E) 150m3

310) La semisuma de las aristas laterales de un tronco de prisma recto es 12cm.


Si la base es un triángulo equilátero de perímetro: 18 3 cm.
El volumen del tronco, es:

A) 256 cm3 B) 188 cm3 C )2 1 0 c m 3 D) 180 cm3 E )2 1 6 c m 3

311) Calcular el volumen de un prisma recto de 384m2 de área lateral, si su base es un triángulo
cuyos lados se hallan en progresión aritmética de producto 480m3 ; la altura del prisma es
1069
el duplo del lado medio del triángulo base.

A) 266m3 B) 284m3 C) 366m3 D) 384m3 E) 464m3

312) Un prisma tiene por base un rectángulo de lados 5 y 4m ; sus aristas laterales miden 6m.
y están inclinadas a 30° sobre el plano de la base y se proyectan siguiendo la dirección de
los menores lados del rectángulo de la base, calcular el área total del prisma.

A) 124m2 B) 114m2 C) 130m2 D) 140m2 E) 120m2

313) La base de una pirámide regular, es un hexágono de perímetro 36cm. Si cada arista lateral
mide 10cm. Hallar el volumen.

A) 144 cm3 B) 1 4 4 /2 cm3 C) 1 4 4 / 3 cm3 D) 1 4 4 /6 cm3 E)N.A.

314) El volumen de un tetraedro regular, de arista "a", es:

A) a3 B )a 3 / 2 c ) a3 ^ D) a / ^ E) a3^

315) Una pirámide tiene 30 aristas. ¿Cuántas caras tiene?

A) 10 B) 12 C) 15 D) 16 E) 11

316) En número de caras de una pirámide es 10. Calcular la suma de las medidas de los ángulos
internos de todas sus caras.

A) 2460° B) 2680° C) 2880° D) 3140° E) 3420°

317) Hallar el volumen de un tetraedro regular, en función de radio V , de una esfera exinscrita.

Í2 Í3 Í2
A) r3^ — B) r3- — D) r3 —— E) N.A.
12 12 ' 8 8

318) Se tiene una pirámide O -A B C , tal que ABC y OCB son triángulos equiláteros. Calcular
su volumen, si: AO = 6 y AB = 4 / 3 .

A) 24 B) 28 C) 32 D) 36 E) 48

319) Hallar el volumen de un poliedro de área total A, circunscrito a una esfera de radio R.
AR AR
A)AR B )— C )— D) 2AR E) 3AR

320) En una pirámide regular de base cuadrangular el plano que pasa por un lado de la base
y la línea media de la cara lateral opuesta, forma con la base un ángulo de 60°.
Si la arista básica es a. Hallar el volumen de la pirámide.

A) a3 B) a3 C) a D) a3 E) a

1070
321) Calcular el volumen de un tronco de pirámide regular, de base cuadrangular, donde un
plano diagonal forma- con las bases ángulos de 60°. Las aristas básicas miden 2 y 4
centímetros.

A) 28cm3 B) 2 8 / 3 cm3 C) 20cm3 D ) 2 l/3 c m 3 E) 24cm3

322) La altura de una pirámide es H.


A una distancia V del vértice se traza un plano paralelo a la base. Tomando como altura
V y como base la sección resultante en la superficie lateral de la pirámide, se dibuja un
prisma recto.
Si el volumen del prisma asi construido es igual al volumen del tronco de pirámide obtenido.
Hallar Y .

H H H H
A)-4 B) / 2 C) 3/ 2 D) 3y ¡ l E) N A.

323) La calamina de un techo nos dá la idea de:

A) Una superficie plana.


B) Una superficie cónica.
C) Una superficie cilindrica.
D) Una superficie esférica.
E) Una superficie prismática.

324) En la figura, calcular el volumen, si AB = 30, CD = 10, y ABC = 37°, (P y Q son centros).

23047c
A ) —

B) 398 7t

4 5 3 6 tt
c> —

D) 412 ti

256471
E> —

325) A qué distancia del vértice “V" de un cono de revolución habrá que hacer pasar un plano
P paralelo a la base y secante al cono, para que el tronco de cono que se forme tenga una
altura de 25cm., el radio de la base inferior mida 8cm. y el de la base superior mida 6cm.
A) 50cm. B) 75cm. C) 80cm. D) 100cm. E) 125cm.

326) Los diámetros de las bases de un tronco de cono de revolución tienen por longitud 4m y
22m. Hallar el radio de la base de un cilindro recto de revolución equivalente al tronco si
se sabe que tienen la misma altura.
A) 6m. B) 4m. C) 5m. D) 7m. E) 8m.
1071
327) Hallar el volumen de un cilindro cuyo desarrollo del área lateral es un rectángulo de diagonal
15m. La altura del cilindro mide 9m.

324 118 224


A) 324 n B )1 1 8 jc C ) ------- D ) ------- E ) -------
71 n k

328) Al desarrollar la superficie lateral de un cono recto se obtiene un semicírculo de 4m. de


diámetro. Hallar el volumen del cono.

J~3 k 3J~3 k .— ,— k J~3


A) B) — St— C) 2 k /3 D) / 3 tc E) - J -

329) Los radios de las bases de un tronco de cono de revolución son R y r ( R > r ). Calcular
la altura del tronco si el área lateral es igual a la suma de las áreas de las bases.

2Rr 2Rr Rr Rr 2R r
A) ( R + r ) B) ( R + 2 r ) C) ( R + r ) ° ) 2 ( R + r) E) ( R - r )

330) El área de la esfera inscrita en un cubo es 18cm2. Calcular el área de la esfera circunscrita
al cubo.

A) 9cm2 B) 36cm2 C) 27cm2 D) 54cm2 E) 72cm2

331) ¿Cuánto debe medir la altura de un casquete esférico para que en una esfera de 4m. de
radio, su área sea la quinta parte del área de la esfera?

A)2,4m. B) 1,2m. C) 0,8m. D) 1,3m. E) 1,6m.

332) Hallar la relación de volúmenes de las esferas inscrita y circunscrita a un tetraedro regular.

A )¿ B) ~ C )± D E ) N . A .

333) Se tiene una zona esférica equivalente a un huso esférico (en la misma esfera de radio R).
Calcular el ángulo del huso esférico si la altura de la zona es R/3.

A) 30° B) 60° C) 45° D) 90° E) 36°

334) ¿ A qué distancia del centro de una esfera de radio R, debe de trazarse un plano secante
para que las áreas de los casquetes determinados estén en la relación de 1 a 3. ?

R R 4R 2R R
A) ~2 B )3 C> T D> T E )5

335) Calcular el volumen generado al girar el triángulo equilátero sombreado, cuyo lado mide
4m.( 360° alrededor del eje mostrado, si p = 15°.
1072
A )4 j i/ 2
EJE
B )2 rc /2

C) 8 Jt / 2

D) 1 6 tc/ 2

E) 32 7C/ 2

M A X IM O S Y M INIM O S

336) Tres lados de un trapecio, miden 4cm. cada uno. Hallar la longitud del cuarto lado, para
que el área sea máxima.

A) 2cm. B) 3cm. C) 4cm. D) 6cm. E) 8cm.

337} En una esfera de radio R, hallar la altura del cilindro circular recto inscrito, que tiene área
total máxima.

A) R->/~2. B )R J 6 C )R /3 E)N.A.

338) En un cono circular recto, de radio 12m. y altura 27m., se inscribe un prisma regular
cuadrangular, de volumen máximo. El volumen del prisma, es:

A) 684 m3 B) 6 8 4 / 2 m 3 C) 648 m3 D) 6 4 8 / 2 m3 E) 3 2 4 / 2 m3

339) En un a A B C .A C = b y la altura BH = h. Hallar la altura del rectángulo de área máxima


que se puede inscribir en el triángulo, y que tiene su base en AC.

3h h
E) N.A.

340) Hallar la longitud de la altura de un cilindro circular recto, inscrito en un cono de revolución
de radio r y altura h. El volumen del cilindro es máximo.

3h
E) N.A.

341) En una semiesfera de radio R, se inscribe un cilindro circular recto de volumen máximo.
El radio de la base del cilindro, es:

C ) f/i D )fV 3

1073
342) Se tiene un cilindro circular recto de radio R y altura h. Hallar la altura del cono circular
recto, de volumen mínimo, circunscrito al cilindro.

A) 2h B) 3h C) 4h D) 6h E) 9h

IN TR O D U C C IO N A LA G E O M E T R IA A N A L IT IC A
Y V E C TO R E S

343) Los puntos: A(2 ; 4 ), B(-1 ; 2) y C(1 ; 2)

A) Son colineales.
B) Forman un triángulo equilátero.
C) Forman un triángulo rectángulo.
D) Forman un triángulo acutángulo.
E) NA

344) ABCD, es un paralelogramo. A(2 ; 5 ), C(8 ; 0) y D(0 ; 0). Las coordenadas de B, son:

A) (5 ; 10) B) (1 0 ; 5) C) (8 ; 5) D) (8 ; 10) E) NA.

345) La mayor base de un trapecio isósceles une los puntos (-2 ; 8) y (-2; 4). Uno de los extremos
de la otra base tiene coordenadas (3 ; -2). La menor base tiene longitud:

A) 8 B) 10 C) 6 D) 9 E) 12

46) El punto A(2 ; 3), es la reflexión del punto B, a través del punto C(5 ; 4). Las coordenadas
del punto C, son:

A ) (8; 5) B ) (-1 ; 2) C) D ) (7 , 7) E)NA

347) El punto (3 ; n), está en la mediatriz de AB, siendo A(7 ; 9) y B(-5 ; -7). Hallar n.

1
A) -2 C) -1 D) 2 E)N.A.
B> " 2

348) Hallar el área del triángulo que tiene por vértices: (-2 ; 3 ), (4 ; -1) , (5 ; 5).

A) 10u2 B) 20u2 C) 30u2 D) 40u2 E) N.A

349) Hallar las coordenadas del tercer vértice de un triángulo ABC, de área 3u2 , sabiendo que
A(3 ; 1), B(1 ; -3) y cuyo Baricentro está en el eje OX.

A ) (5; 2) B) (2 ; 2) C) A ó B D) N.A.

L074
350) Hallar el área del rombo de lado 5^/TÓ y dos de cuyos vértices opuestos son los puntos
A(4 ; 9) y Q(-2 ; 1)

A) 100u2 B) 120u2 C) 130u2 D) 150u2

351) ABCD, es una región paralelográmica, cuya área se quiere hallar sabiendo que
A (-2 ; 3 ), B(4 ; -5 ) y C (-3 ;1 )

A) 10u2 B) 12u2 C) 20u2 D) 25u2

352) Hallar la distancia del punto Q = ( 7 ; 9 ) a la recta L: 3x + 4y - 7 = 0

A) 5 B) 6 C) 7 D) 9 E) 10

353) La ecuación de la recta mediatriz del segmento AB, donde A = (3 ; 5) y B = (0 ; 7), es

A) 6x + 4y + 15 = 0 B) 6x - 4y + 15 = 0 C) 6x - 4y -1 5 = 0
D ) 4 x - 6 y + 15 = 0 D) N.A.

354) Hallar el punto de intersección de las rectas:


L, : pasa por (-1 ; 2), paralela a (1 ¡2) L2 : pasa por ( 1 ; 1 ) y ( - 1 ; 5 )

1 7
A) (-1 ; 14) B) C) (-4 ; 7) D ) (-1 ; 7) E)N.A
4 ’ 2

355) En la figura:
P = (-2; 1 ) , Q = (-1 ¡2) y C = (9 ; K)
El valor de K, es:

A) 8 B) 9 C) 10
D) 11 E) 12

356) L es una recta no paralela a los ejes coordenadas, que pasa por los puntos (2 ; 2 ); (0 ; n);
(m ; 0) donde m x n * 0.
1 1
Hallar: — +-
m n
1 1
A) 1 D) E) N.A
B> 2 C> 3

f 1 9 5 ^ ( 5 • 3 ^ ' 1 If 3 1 , sean
357) Hallar el valor de K, para que los puntos t —; K + —
{ ~ 2 ’ ~ Í 4 } ^ 14 ’ 14 , y [ 7 14 )
colineales.
1 7
A) 7 B) 7 C) D) -7 E)N. A.
7

J075
358) La recta de ecuación y = x + 5 contiene a la diagonal AC del cuadrado ABCD.
Si: A(1 ; a) y Q(0 , q), siendo Q el centro del cuadrado. Hallar el área ABCD.

A) 2u2 B) 3u2 C) 4u2 D) 8u2 E) N.A.

359) Determinar n, de modo que la recta: 1 2 n x - 9 y + 129 = 0, intersecta al segmento de extre-


2
mos A(2 ; 3) y B(11 ; 6) en la razón —.

A) 1 B) 2 C )-2 D) -1 E) 3

360) La recta de ecuación: ax + by + 6 = O, pasa por los puntos (1 , 4) y (3 ; -2). Hallar —.


b

A) 3 B )^ C )-3 D) E)N.A.

361) La ecuación de la circunferencia circunscrita al A ABC, donde A = (1 ; 1 ), B = (1 ; -1) y


C = (2 ; 0), es:

A) x2 + (y - 1)2 = 1 B) (x - 1)2 + (y - 1)2 = 4


C) (x -1 )2 + (y + 1)2 = 1 D) (x -1 )2 + y2 = 1

362) El punto A(3 ¡ -1), es centro de una circunferencia que intercepta en la recta 2 x - 5y + 18 =
O, una cuerda de longitud 6. La ecuación de dicha circunferencia, es:

A) (x - 3)2 + (y + 1)2 = 36 B) (x - 3)2 + (y + 1)2 = 37


C) (x - 3)2 + (y + 1)2 = 38 D) (x - 3)2 + (y - 1)2 = 38

363) El centro de la circunferencia del anterior problema, tiene coordenadas:

A) (1 ; 2) B) (1 ;-2) C) (-1; 2) D) (2 ; -1)

364) Hallar el radio de la circunferencia que tiene su centro en la recta: 2x + y = Oy es tangente


a las rectas: 4x - 3y + 10 = O ; 4x - 3y - 30 = O

A) 8 B) 16 C) 4 D) 2

365) Hallar la ecuación de la circunferencia de diámetro AB, donde:A =(3 ; 2) y B = (-1 ; 6).

A) (x - 1)2 + (y - 4)2 = 64 B) (x - 1)2 + (y - 4)2 = 8


C) (x - 4)2 + (y - 1)2 = 64 D) (x - 4)2 + (y - 1)2 = 8

366) Hallar la ecuación de la circunferencia que pasa por el origen de coordenadas y tiene centro
en (6 ; -8)

A) (x - 6)2 + (y + 8)2 = 100 B) (x + 6)2 + (y - 8)2 = 100


C) (x - 6)2 + (y + 8)2 = 10 D) (x + 6)2 + (y - 8)2 = 125 E)N.A.

1076
367) Hallar la ecuación de la circunferencia centrada en el origen de coordenadas y tangente a
la recta: 3x - 4y + 20 = 0

A) x2 + y2 = 20 B) x2 + y2 = 36
C) x2 + y2 = 16 D) x2 + y2 = 49

368) Hallar la ecuación de la circunferencia de centro ( 1 ; -1 ) y tangente a la recta


5 x- 1 2 y + 9 = 0.

A) (x - 1)2 + (y + 1)2 = 4 B) ( x - 1 ) 2 + (y + 1)2 = 16


C) (x - 1)2 + (y + 1)2 = 9 D) (x - 1)2 + (y + 1)2 = 169

369) Hallar el radio de la circunferencia de ecuación: x2 + y2 - 2x + 4y- 20 = 0

A) 2 B) 3 C) 4 D) 5

370) Hallar la ecuación de la recta que contiene al diámetro de la circunferencia:


x2 + y2 + 4x - 6y -1 7 = 0 y es perpendicular a la recta: 5x + 2y -1 3 = 0

A) 5x - 2y + 19 = 0 B) 2x + 5 y - 19 = 0
C) 2x - 5y + 19 = 0 D )2 x + 5 y+ 19 = 0

371) Dada la circunferencia de ecuación: x2 + y2 - 10x + 16 = 0 y la recta: y = mx


Hallar el valor de m, si la recta es secante a la circunferencia.
3 2 1
A) | m | < — B) m < — C) m < — D) m < —
4 5 2

372) En el anterior problema, hallar el valor de m, si la recta es tangente a la circunferencia.

A) m = ± j B) m = ± | C) m = ± - | D) m =

373) Para el problema anterior, hallar el valor de m, si la recta es exterior a la circunferencia.


1
A) m > - B)m>¿ C)m>- D)m< j

374) Hallar la ecuación de la tangente a la circunferencia ( x + 2 )2 + ( y - 3 )2 = 25, en el punto


A ( - 5 ; b).

A) 3x - 4y + 43 = 0 B) 4x - 3y + 43 = 0 C) 4x + 3y - 43 = 0
D) 3x + 4y - 43 = 0 E)N.A.

'5 . 5
375) Desde el punto A se han trazado tangentes a la circunferencia x2 + y2 = 5 ¿Cuál
, 3 ’ "3 )
de las siguientes, no es ecuación de una de estas tangentes:

A) x * 2y - 5 = 0 B)2x - y - 5 = 0 C) x + 2y - 5 = 0 D)N.A.
1077
376) Hallar la ecuación de la tangente a la circunferencia x2 + y2 = 5 , en el punto de ab-
cisa -1 .

A) x + 2y - 5 = 0 B) x + 2y + 5 = 0 C)x-2y-5 = 0
D) x - 2y + 5 = 0 E)N.A.

377) En un triángulo ABC, M es punto medio de AB y N de B C .S i:A = (2 ; 4 ), C = (6 ; 12)


y M = (3 ; 7); hallar las coordenadas de N.

A) (11 ; 5) B) (7 ; 5) C) (10 ; 5) D) (11 ; 4) E) (5 ; 11)

378) Q, es un punto del segmento AB, tal que 2 d(Q ; A) = 3 d(Q ; B).
Si: A = (5 ; 10) y B = (15 ;20). Hallar Q.

A) (11 ; 13) B) 16; 11) C) (11 ; 16) D) (13 ; 11) E)N.A.

379) En la figura: A= (1 ; 1 ), B = (2 ; 9 ). C = (8 ; 3).


Hallar: d(A , E).

A )/?

B )3 /5

C)2fE
D )4 ^
E) N.A. A C

380) Hallar el ángulo entre los vectores: á = (-1 ; 7) y b = (3 ; 4)

A) 60° B) 90° C) 45° D) 75° E) 30°

381) Si los vectores: v = ( 2 ; - 3 ) y w = (1 ; n ), son perpendiculares; hallar el valor de n.

A )j B) 3 C )| D )| E) 2

382) Los vectores: p = (a -1 ; a + 2) y q = (5 ; 7), son paralelos.


El valor de a, es:

A) 17 B) 2 C )y D) ^ E)N.A.

383) Sean h y k, dos números reales, tales que: á = hb + k c donde: á = (3 ; - 2 ) , b = (-2 ;1)
ye = ( 7 ; - 4 ) . H allar: h + k

A) 1 B) 2 C) 3 D) 4 E)N.A.
1078
384) En un A ABC, hallar la medida del ángulo B, si: A - (2 ; 3 ), B - (1 ; 2) y C - (-1 ; 2)

A) 45° B) 60° C) 120° D) 135° E)N.A.

385) Si cada cuadrado pequeño, tiene área


1u2: hallar el m ódulo del vector:
r=á+ b+ c+ d

A) 3 / 2 B )3 /3

C )3 /5 D )3 /6

386) la figura muestra un cubo cuya arista tiene longitud a. Hallar el módulo del vector:
c D r = AB + CM

A) a

B) 2a

H C) 2 a

/ D) 3a
/
/
A

387) Los vectores a y b forman un ángulo de medida 30°. Si : | a | = y b = 1

Hallar la medida del ángulo formado por los vectores p = á + b y q = a - b


/ \ / \

A) are eos 2 B) 30° C) are eos 1 D) 15

388) A partir del gráfico adjunto.


Demostrar que:

1 —

r = -
b a +
+ b

-i
389) Dado: a = (3 ; -4), hallar a

A) (3 ; -4) B) (-3 ; 4) C) (-3 ; -4) D) (-4 ; 3) E) 14 ; 3)

390) Si P, es un punto de la recta L, que pasa por el punto P0 y es paralela al vector a, entonces:

A) P = P0 + t á , t e [0 ; 1] B) P = P0 + 1 a , t e IR2 C)P = Pn+ t i ,te IR


D) P = P0 + 1 á , t > 0 E) Ninguna
1079
391) Respecto a la ecuación de la recta en el problema anterior, contestar verdadero o falso

I) á, es denominado “vector direccional” de t?.


II) Al coeficiente t, se le llama parámetro.

A) W B) FF C) VF D)FV E)N.A.

392) Dada la recta: L = {P = (1 ; 3) + 1 (2 ; -3) / 1 e IR }. Indicar verdadero (V) o falso (F):

I) El punto (3 ; 0) no pertenece a L.
II) El punto (3 ; 1) pertenece a L.

A) W B) FF C) FV D)VF E) N.A.

393) Sean las ecuaciones paramétricas de una recta:

x = 2 + 3t
y = 1+ t
Entonces, la ecuación simétrica, es:

A )l z l = l z l p v x “ 2 _ y -1
A) 3 2 B) 2 “ 1 C) 1
x- 2 y + 1
D) - ^ ~ = E )N A

394) Si:
L, = { P 0 + t(a + 4 ; a - 4 ) / t e IR } L2 = { P0 + r ( i - 2a ; 3a) / r e IR}
son rectas iguales ; hallar el valor de a.

A)1 B) 0 C )-1 D) 2 E) N.A.

395) Las rectas:


L] : (x ; y) = (1 ; 3) + 1 (t ; -2) L2 : ( x ; y ) = (3 ; -1) + r(-1 ; 2)
r, t e IR

A) Son paralelas B) Son perpendiculares C) Son oblicuas


D) Son iguales E)N.A.

396) Las rectas:


L, : ( x ; y) = (2 ;1 ) + t(-1 ;2 ) L2 : (x ; y) = (1 ;2 ) + r(2 ;-1 )
t , r e IR.

A) Son paralelas B) Son perpendiculares C) Son oblicuas


D) Son iguales. E)N.A.

1080
397) Lv es una recta que pasa por el punto A(7 ; 9).
L2,es otra recta que pasa por los puntos B(1 ; 1) y C(3 ; 4)

Entonces, una ecuación vectorial de L, es:


A) P = (x ; y) = (7 ; 9) + t(1 ; 1), t e IR B) P = (x ¡ y) = (7 ¡ 9) + t(3 ; 4), t € IR
C) P = (x ; y) = (7 ; 9) + t(4 ; 5), te IR D) P = (x ; y) = (7 ¡ 9) + t(2 ; 3), t e IR
E) N.A.

398) La recta que pasa por el punto (5 ; 6) y es perpendicular al vector (-2 ; 3), tiene ecuación:

A) P = (5 - 2 t ; 6 + 3t), t e IR B) P = (5 + 3 t ; 6 - 2t), t e IR
C) P = (5 + 3 t ; 6 + 2t), t e IR D) P = (-2 + 5 t : 3 + 6t); t e IR
E) P = (3 + 5 t;- 2 + 6 t) ;te IR

399) Sea la recta L, cuyas ecuaciones paramétricas son:


í x = 2 +t
l y = 3 +t
Entonces:

I) (-2 ; 3) es un punto de paso, de L.


II) (1 ; 1) es un punto de paso, de L.
III) L, forma un ángulo de 30° con el eje de abscisas.

A)FFV B) V W C)FFF D)FVF E)N.A.

400) Sean las ecuaciones paramétricas de una recta:


í x = 2 + 3t
' y = 1 +t
Entonces:
I) (3 ; 1), es un punto de paso.
II) La pendiente, es 2.
II!) (3 ; 1), es un vector direccional.

A )V W B) FFF C)FFV D)VFV E)N.A.

401) Respecto a la recta L, cuyas ecuaciones paramétricas son:


J x = 1-t,
i y = 2

(I) Tiene como punto de paso (1 ; 2)


(II) El vector direccional, es (-1 ; 0)
(III) Es una recta vertical

A )W V B) W F C)FFF D)FFV E)N.A.

1081
402) La ecuación del rayo con origen en el punto P0(3 ; 5) y paralelo al vector V = (1 ; 2), es

A) P = (3 + 5 t;2 + t ) , t e IR B) P = (3 + 1; 5 + 2t), t e IR
C )P = (3 + t ; 5 + 2t), t > 1 D) P = (3 + 1 ; 5 + 2t), t > 0 E)N.A

403) La recta: x = 1 + 2t
y = 3 - 4t

tiene forma simétrica:


x - 1_ y + 3 x - 1_ y - 3 x - 1 y - 3
A) B) C) D) E) N.A
2 " -4 2 ” -4
B f 5; 9 )
404) Si P = (x ; y), es un punto cualquiera
del segmento AB, entonces:

A) P = (2 + 5 t ; 4 + 9 t) ,te [0 ; 1]
B )P = (2 + 3 t ; 4 + 5 t) ,te IR C) P = ( 2 + 3 1 , 4 + 51), t e [ 0 ; 1 ]

D )P = (2 + 5 t ; 4 + 3 t) .te [0 ; 1] E) P = (2 + 3 t ; 5 + 4t), t e [ 0 ; 1]

40o) En el gráfico adjunto, L es una recta paralela al segmento RS


Entonces, la ecuación vectorial de L, es:
A) P = (1 ; 0) + 1(2; 3), t e IR
B) P = (0; 1) + t(3 ; 2), t e IR
C) P = (0;! 1) + t(2; 3), t e IR
D) P = (0; ; -1) + 1(2 ; 3), t € IR
E )P = (0 ; -1) + t(3 , 2), t e IR

CONSTRUCCIONES CON REGLAYCOMPAS


B

406) Construir el cuadrado inscrito en


el A ABC, uno de cuyos lados es
paralelo a AC.

407) Copiar el triángulo ABC del problema anterior y ubicar un punto H, que equidiste de los
vértices A y C, estando además a igual
distancia de los lados AB y BC.

408) Dibujar las circunferencias, ins­


crita y circunscrita, al A A B C .d e
la siguiente figura:

1082
409) Graficar un triángulo, conociendo
las longitudes de sus tres me­ mi
dianas: m ~>
m3

Graficar una recta tangente a la


410)
circunferencia dada y que forme un
ángulo de medida a°, con la recta
m:

411) C o n s tru ir un triá n g u lo ABC


conociendo la longitud del lado AC
la del circunradio “R” y la medida a°
del ángulo BAC.

412) Construir un rectángulo, conociendo las longitudes respectivas de la base (b) y la diagonal
(d). b

413) C o n s tru ir un triá n g u lo PQR,


conociendo la longitud “r" del inradio
y las medidas a ° , p° de los ángulos
QPR y PRQ, respectivamente.

414) Construir un trapezoide ABCD, sabiendo que M ,N y Q, son puntos medios de los lados
Á B .B C y CD, respectivamente. N

M
Q

415) Graficar un A ABC, conociendo la longitud a, del lado BC y las longitudes de las alturas

h» y h ‘ : a hb h 0
é--------------------------------------------- •----------------1 ♦---------------------------------- •

IS O M ETR IAS
416) La reflexión del trapecio ABCD a través del punto E, es: B C

B__________C D
A) / \ B)

D A B
1083
B D) A E)N.A.

C B
417) ¿Cuál representa un correcto gráfico de la reflexión de la figura ABC, a través del punto 0?
c c
A)

B O
A'
B)
O

C) B'
A
o
T
B A’
D)

A C B A’ C B’

E)N.A.

418) P' y Q' son las reflexiones de P y Q, respectivamente, a través del punto 0. L 1 1 L2 ,

d(P, Q) = 20u y QPO = 30°. El área de la superficie PQP’Q’, es:

A )2 0 0 u 2 L

B) 400u2

C) 1 0 0 /3 u2
O
D) 2 0 0 / 3 u2
E)N.A.

419) La reflexión de la figura mostrada, a través del punto 0, es:

5 -o

1084
A) B) C) D)
m E) N.A.
420) Indicar verdadero (V) o falso (F):
I) Una rotación de 180° equivale a una reflexión a través del mismo punto.
II) Si P y Q son puntos distintos y F una isometría, entonces:
d(P.Q) = d(F(P), F(Q))
III) La reflexión de un paralelogramo a través de su centro es equivalente a reflejarlo a través
de una diagonal.

A )V W B) W F C) FFF D)FFV E)N.A.

421) Se rota un triángulo equilátero, de lado 6cm, 90° respecto a un vértice. La distancia entre
los baricentros del triángulo y su imagen, es:

A)6cm . B) ^ 6 cm. C)2^~6cm D )3 ^ 2 c m . E)N.A.

422) Un hexágono regular ABCDEF, de lado 4cm., se refleja a través del eje que contiene a la
diagonal AC. La región común a ABCDEF y su imagen, tiene área:

A )4 ^ 3 ^c m 2 B) 8^~3cm 2 C) 4cm2 D) 6cm2 E)N.A.

423) ABCD, es un rectángulo, donde AB = 6 y BC = 10. Se refleja ABCD a través de la diagonal


AC. La región común a ABCD y su imagen, tiene área :

A) 45,8 B) 40,8 C) 32,8 D) 64 E) N.A.

424) Se refleja un triángulo equilátero, de lado 6u, a través de su baricentro. El área de la región
común al triángulo y su imagen, es:

A) 6u2 B )6 /2 u 2 C )6 /3 u 2 D ) 2 / 6 u2 E)N.A.

425) La reflexión del triángulo equilátero ABC a través del centro O, equivale a una rotación de
P° respecto al mismo punto: entonces:

A )P = 120 B

B) p = 90

C )p = 150

D )p = 180

E) No existe.

426) La figura es un rectángulo donde: d(A, C) = 5 ^ 2 u. Si RL es la reflexión a través de la


recta L.
1085
Hallar la suma de la áreas de las regiones BCB’ y CDD\ si B’ = RL(B) y D’ = RL(D)

A )1 0 0 u 2 B
L
B) 50u2

C) 25u2

D) 10u2

E) N.A.

427) Si G es una rotación aplicada a un punto, entonces, señale verdadero (V) o falso (F)

I) G_io° - ^ io ° II) G-2o° - G 34O' III) ^800* - ^80 IV) ^-5000° “ ^40"

A )W W B)FFFF C) F V W D) F V W E)N.A.

428) Marcar verdadero (V) o falso (F). Si H es una isometría; P y Q puntos distintos, entonces

I) H (P )* H(Q)
II) P Q //H ( P )H( Q ), siempre.

III) Si: PQ ± H( P )H ( Q ), entonces H puede ser alguna rotación.

A)VFV B )V W C) FFF D) F W E) N.A.

429) P y Q son dos puntos distintos de una circunferencia cuyo centro es otro punto M. Sean
P’ y Q’ imágenes de P y Q, a través de una rotación de 42° respecto a M. Entonces, PQP'Q',
es:

A) paralelogramo B) rombo C) rectángulo


D) trapecio isósceles E) trapecio rectángulo.

430) El triángulo ABC, se refleja a través de la hipotenusa AC. Halla la distancia de B, a su imagen
B \ si d(A, C) = 6 y Z BAC = 30°.

A) 6 B )2 /3 C )6 /3 D )3 /3 E) N.A.

431) En la figura: d(A, O) = 2


A’ es la reflexión de A a través de O
A" es la rotación de A ’, un ángulo de 90°
respecto al punto O. Hallar d(A, A ”). ./
A 30"

A) 2 B )/2
OP
D) 4
E)N.A.
108*)
432) En la figura, la distancia de A, a la recta m es 2. A ’ es la reflexión A, a través de m. A” es
la reflexión de A’ a través de O. Hallar d(A, A").

A) 4
B) 6
C )2 /3
D )4 /3 30a
o in
E) N.A.

433) Los segmentos PQ y QM son catetos de un triángulo rectángulo. R es el punto medio


de la hipotenusa. La reflexión de Q respecto al punto R. es Q’. Hallar d(Q, Q’), si:
d(P, Q) = 8 y d(Q, M) = 15

A) 23 B) 11,5 C) 16 D) 17 E) 14

434) Sea AOB un triángulo rectángulo recto en O. La aplicación Fr es una dilatación respecto
al punto O e indica que a cualquier punto P se le asigna el punto Fr(P), ubicado sobre
el rayo de vértice O y que pasa por P, a una distancia de O, igual a “r" veces d(0, P).
H allar: d(F3(A), F4(B)), si:
d(O .A) = d(0, B) = 2
A) 5 B) 10 C) 8 D) 14 E) 12

435) En la figura O, P y Q son puntos de una recta. d(0, P) = 7 y d(P, Q) = 17. La aplicación
G es una rotación respecto al punto O ; sean : P’ = G Itr Q’ = G 10QO

1087
C L A V E S

1. D 26. C 51. C 76. E 101. D 126. D

2. E 27. E 52. C 77. B 102. E 127. E

3. B 28. A 53. B 78. C 103. D 128. A

4. A 29. B 54. E 79. D 104. E 129. C

5. D 30. C 55. D 80. C 105. A 130. E

6. D 31. C 56. C 81. A 106. B . 131. C

7. D 32. C 57. A 82. D 107. B 132. C

8. B 33. D 58. D 83. E 108. B 133. B

9. E 34. C 59. C 84. C 109. C 134. D

10. A 35. B 60. E 85. D 110. B 135. E

11. C 36. C 61. C 86. C 111. A 136. B

12. E 37. B 62. D 87. A 112. B 137. D

13. C 38. E 63. D 88. E 113. B 138. B

14. B 39. E 64. C 89. D 114. B 139. B

15. D 40. B 65. A 90. E 115. D 140. E

16. D 41. A 66. C 91. C 116. C 141. B

17. E 42. B 67. C 92. A 117. B 142. B

18. B 43. D 68. D 93. D 118. E 143. B

19. C 44. A 69. A 94. B 119. B 144. B

20. C 45. D 70. D 95. E 120. B 145. D

21. D 46. A 71. E 96. B 121. A 146. C

22. A 47. D 72. B 97. B 122. D 147. B

23. C 48. B 73. D 98. D 123. A 148. B

24. B 49. C 74. C 99. D 124. A 149. A

25. C 50. D 75. E 100.D 125. D 150. A

1088
\

151. C 176 . B 201. C 226 C 251. E 276. C

152. A 177 . B 202. B 227. E 252. B 277. B

153. B 178. D 203. E 228. D 253. C 278. E

154. A 179. C 204. D 229. B 254. E 279. D

155. B 180 . A 205. D 230. C 255. B 280. E

156. B 1 81 . C 206. A 231. E 256. E 281. B

157. C 1 82 . A 207. E 232. A 257. C 282. B

158. C 183. E 208. C 233. C 258. D 283. A

159. D 184. E 209. D 234. D 259. B 284. D

160. B 185. D 210. B 235. A 260. B 285. D

161. A 1 86 . B 211. C 236. D 261. C 286. D

162. A 187. E 212. E 237. E 262. A 287. D

163. C 188. C 213. B 238. - 263. B 288. E

164 . A 1 89 . C 214. C 239. C 264. C 289. B

165. E 190. D 215. E 240. D 265. D 290. B

166. D 191. B 216. B 241. E 266. A 291. C

167. C 1 92 . B 217. A 242. B 267. B 292. D

168. E 1 93 . A 218. B 243. C 268. C 293. E

169. B 194. B 219. D 244. E 269. E 294. D

170. C 195. A 220. A 245. C 270. B 295. E

171. C 196. C 221. A 246. B 271. C 296. D

172. B 197. C 222. B 247. C 272. D 297. E

173. B 1 98 . A 223. C 248. D 273. E 298. B

174. C 199. D 224. A 249. E 274. D 299. D

175. D 200. A 225. D 250. E 275. B 300. E

1089
301. A 326. D 351. C 376. D 401. B 426. C

302. A 327. C 352. E 377. E 402. D 427. D

303. C 328. E 353. B 378. C 403. B 428. A

304. E 329. A 354. B 379. B 404. C 429. D

305. E 330. D 355. E 380. C 405. E 430. D

306. C 331 E 356. B 381. D 406. - 431. C

307. A 332. D 357. C 382. C 407. - 432. C

308. E 333. B 358. C 383. C 408. - 433. D

309. B 334. A 359. C 384. D 409. - 434. B

310. E 335. D 360. A 385. C 410. - 435. A

311. D 336. E 361. D 386. C 411. -

312. A 337. C 362. C 387. A 412. -

313. C 338. D 363. B 388. - 413. -

314. E 339. D 364. C 389. E 414. -

315. D 340. B 365. B 390. C 415. -

316. C 341. E 366. A 391. A 416. D

317. C 342. B 367. C 392. B 417. C

318. D 343. E 368. A 393. C 418. D

319. C 344. B 369. D 394. C 419. A

320. B 345. A 370. C 395. D 420. B

321. B 346. A 371. B 396. C 421. C

322. D 347. B 372. C 397. D 422. B

323. C 348. B 373. C 398. C 423. B

324. A 349. D 374. A 399. C 424. C

325. B 350. D 375. C 400. C 425. D

1090
\

B IB L IO G R A F IA

1) Geometría. Serie Matemática Moderna


Edwin E. Moise - Floyd L. Downs.
Editorial Norma

2) Geom etría Elemental Edwin M. Hemmerling. Editorial Limusa

3) Geom etría Moderna. R. Jurgensen - A. Donnelly - M. Dolcíani


Publicaciones Cultural S.A.

4) Plañe Geom etry Problem s Marcus Horblit - Kaj L. Nielsen.


Barne and Noble, Inc., New York

5) Cous de Géom etrie. Par une reunión de professeurs.


Librairie Genérale - París.

6) M atem áticas. Curso Preuniversitario.


Constantino Marcos - Jacinto Martínez
Ediciones S.M. Madrid

7) Exercices de Géom étrie. F.G .M .


Librairie Générale - París

8) E xercicios de Geom etría Plana. Edgar de Allencar Filho.

9) Problem as de Geom etría A nalítica. D. Kletenik.


Editorial Latinoamericana

10) Geom etría Superior. Bruño.

11) Geometría Analítica. H. B. Phillips. Unión Tipográfica


Editorial -Hispano-Americana

1091
FE DE E R R A T A S

Página Referencia Dice * Debe Decir

27 Problema 6 A) 7414 A) 7876

29 f) AI principio 8 x 11 x 21 = 1848 ptos 10 . 11 . 21 = 2 310 ptos


de página 2 x 4 = 8 ptos 2 . 5 = 10 ptos
MNPC = 120 + ... + 1848 MNPC = 120 + ... + 2 310
MNPC = 7414 ptos MNPC = 7 876

89 En oblicuas x1 y x1 y

200 Prob. 54
Solución

272 Casi al final Propiedad: Por tres puntos Propiedad: Por tres puntos
de página colineales no colineales

AD AB
348 a) Bisectriz interior Debajo, colocar :
DC BC

EA AB
b) Bisectriz exterior -> Debajo, c o lo c a r:
EC BC

349 Teorema del Excentro (El gráfico debe ir a la altura de la fórmula)

469 Problema 33
OI = R( R - 2 r ) OI 2 = R ( R - 2 r )
(Fórmula)

556 Problema 42 En el gráfico, debe ser S1donde está S2 y S2 donde


está S1

595 En a.2) "Líneas Proporcionales" "Semejanza de Triángulos"


(Capítulo 10) (Capítulo 12)

885 Enunciado del EF


Problema 10 ■
995 Ultima línea AAHE AOHE

1 025 TITULO MISCELANIA MISCELANEA

1 045 Gráfico
Prob. 34
/c
1 050 Prob. 167 Hallar DT, si AT = 6 borrar si AT = 6

1 078 Prob. 379


(Figura)
I m p r e s o e n lo s ta lle r e s g r á f ic o s d e E d ito r ia l S a n M a r c o s
A v. L a s L o m a s 1 6 0 0 U rb . M a n g o m a rc a - S J L . R U C 10090984344
P e d id o s :
N a t a lio S á n c h e z 2 2 0 o f. 3 0 4 ( c d r a . 5 A r e n a le s ) L im a T f. 3 3 0 - 8 5 5 3
A v . G a r c i la s o d e la V e g a 9 1 1 - O f. 4 0 4 - U r n a , T f. 4 2 4 * 6 5 6 3

Você também pode gostar